סיקור מקיף

אסטרונומים מדדו את ריכוז החומר האפל לפני כ-6 מיליארד שנים

אסטרונומים מיפו את החומר האפל בקנה המידה הגדול ביותר שנצפה אי פעם. ממצאים חדשים חושפים כי היקום מורכב מרשת קוסמית מורכבת של חומר אפל וגלקסיות המשתרעת למרחק של למעלה ממיליארד שנות אור

תצפית המראה כי החומר האפל ביקום מפולג בצורת רשת של איזורים דחוסים (בלבן) וריקים (בשחור). האיזורים הלבנים הגדולים ביותר הם בגודל של הירח המלא בשמים. צילום: ואן-וירבק, הימאנס ושותפות CFHTLenS
תצפית המראה כי החומר האפל ביקום מפולג בצורת רשת של איזורים דחוסים (בלבן) וריקים (בשחור). האיזורים הלבנים הגדולים ביותר הם בגודל של הירח המלא בשמים. צילום: ואן-וירבק, הימאנס ושותפות CFHTLenS

אסטרונומים מיפו את החומר האפל בקנה המידה הגדול ביותר שנצפה אי פעם. ממצאים חדשים חושפים כי היקום מורכב מרשת קוסמית מורכבת של חומר אפל וגלקסיות המשתרעת למרחק של למעלה ממיליארד שנות אור.

ד”ר קתרין היימס ופרופ’ לודביק ואן-וירבק מאוניברסיטת בריטיש קולומביה שבונקובר, קנדה הציג את התוצאות ב-9 בינואר בכנס האיגוד האסטרונומי האמריקני שהתקיים באוסטין, טקסס.

צוות בינלאומי של חוקרים בראשות ואן –וירבק והיימנס הגיעו לנתונים אלה מניתוח תצלומים של 10 מיליון גלקסיות בארבעה איזורים נפרדים של השמים. הם חקרו את תפוצת האור הנפלט מגלקסיות אלו, המכופפות כאילו הם חולפים דרך גושים מאסיביים של חומר אפל במהלך מסעו של האור לכדור הארץ.

הפרויקט הידוע בשם סקר העידוש באמצעות הטלסקופ של קנדה, צרפת והוואי (CFHTLenS). תמונות אלו שהצטברו במשך חמש שנים תוך שימוש בשדה הזוית הרחבה MegaCam המצלמת שדה ראיה של מעלה על מעלה ברזולוציה של 340 מגה פיקסל בטלסקופ CFHT בהוואי.

הגלקסיות הכלולות בסקר נמצאות בממוצע במרחק של 6 מיליארד שנות אור. האור שצולם ואשר שימש למחקר נפלט כאשר היקום היה בן 6 מיליארד שנה, או בערך חצי מגילו הנוכחי.

מסקנות המחקר נחשדו כבר בעבר בהתבסס על סימולציות מחשב, אך היה קשה לאמת זאת בשל הטבע הבלתי נראה של החומר האפל. זוהי ההצצה הגדולה לחומר האפל בקנה מידה גדול שהוכיח את קיומה של הרשת הקוסמית בכל הכיוונים.

ד”ר תומס קיטצ’ינג מרכז קבוצת העבודה הקוסמולוגית אומר: “מפת החומר האפל איפשר לנו למפות אחורנית למעלה מ-75% מגיל היקום כיום, לזמן שבו הוא היה שונה לחלוטין. באמצעות התפתחות היקום במהלך תקופות קוסמיות, הצוות באדינבורו יחקור כיצד האנרגיה ההפעלה הגיעה למצב שבו היא דומיננטית ביקום של היום.

במהלך החודשים הבאים נשתמש במידע כדי למפות את התפתחות התפשטות היקום וללמוד עוד אודות החומר האפל, הגורם להתפשטות היקום. אנו ננסה את תיאוריות הכבידה כדי לקבוע אם תיאורית היחסות הכללית של איינשטיין נכונה או לא. אנו גם נשתמש בנתונים כדי לקבוע את תכונות הניוטרינוז, חלקיקי הרפאים המקיימים קשר חלש מאוד עם החומר.

המחקר נתמך בידי מועצת המחקר האירופית, מועצת המחקר למדעי הטבע והנדסה בקנדה, המכון הקנדי למחקר מתקדם ומרכז הנתונים האסטרונומיים של קנדה.

להודעה של החוקרים

 

 

1,188 תגובות

  1. ר.ח,
    1) נכון מאד וגם יפה 🙂
    2) יפה, אך לא. כדי לקבל את כל תריסר הנקודות תצטרך להתאמץ מעט. שמו של המלך מופיע בספר מלכים יותר מפעם אחת. אגב הזכרת שאול, זהו שמה של ממלכת המתים ונראה לי שהענקת השם הזה במקרא לא נעשתה מתוך אהבה.
    אכן, בזבוז זמן נוראי אך לא רק שלך אלא, ובעיקר, שלו.

  2. גרציאס סניור יובלי, גרציאס.

    ר.ח.
    הכתבה הזו כבר די חבוטה וגמורה, אלא אם כן אתה מתכוון להמתין רבע שעה לטעינה של תגובות. אם ברצונך להמשיך, מצאתי כתבה רגועה ושקטה בקצה היקום, שלא תפריע לאף אחד ושם נוכל להתחרע כאוות נפשינו. אם מישהו אחר מעוניין לעבור גם כן, אני מציע שהכלל יהיה שמי שעובר יתחייב לדבוק בעניינות בלבד.

    https://www.hayadan.org.il/vlt-hubble-smash-record-for-eyeing-most-distant-galaxy-2310104/#comment-333491

  3. יובל,
    1) “וַיֹּאמֶר אֵלָיו מַה שְּׁמֶךָ וַיֹּאמֶר יַעֲקֹב.” (בראשית ל”ב כ”ח)

    2) אתה בטוח במלכים? כי בבראשית ל”ו ל”ז נאמר “”וַיָּמָת שַׂמְלָה וַיִּמְלֹךְ תַּחְתָּיו שָׁאוּל מֵרְחֹבוֹת הַנָּהָר.” האם לזה התכוונת?

    ישראל,
    1) תסלח לי אבל אתה לא באמת קורא את מה שאני כותב. אני מרגיש שאני חוזר וחוזר וכל פעם אתה פוטר אותי באיזה אמירה סתמית”לא קרינה אלקטרומגנטית”. סליחה, אבל דוגמת הסופר נובה לא מתייחסת כלל וכלל לאור או קרינה אלקטרומגנטית אלא לשעוני טמפרטורה עצמם.

    2) חוץ מזה אתה מתעלם בנחרצות מהאור ה”נעלם” במכונית לפי המודל שלך.

    3) לא הבנת את השאלה שלי “מה מתקרר”. שעוני הטמפרטורה שלך מודדים קרינת רקע של היקום שמוצאה מהמפץ והולכת ומתקררת. אם אתה שולל את קיומו של המפץ שללת את הבסיס של שעוני הטמפרטורה. כי אחרת מה הם מודדים?

    4) אפקט דופלר הוא לא רק במערכות מואצות אלא כאלה שמתרחקות זו מזו, גם במהירות קבועה. הראיה ששנים דיברו על יקום מתפשט לפי דופלר ורק לאחרונה על יקום מאיץ. נפל לך טיעון “דופלר רק במערכות מאיצות”.

    5) לגבי ג-ג , מה אתה רוצה? אתה אומר יראו אותו דבר. אני וויקפדיה וכל הפיזקאים שחקרו את התארכות המנים אומרים “לא נכון, כל אחד יראה את השני מאט”. כנראה שעד שלא נעשה את הניסוי לא נדע. אז למה אתה כופה פה איזו שהיא הסכמה?

    בבקשה תתיחס לנקודות ואל תפטור אותן כלאחר יד כי אחרת אני מרגיש שזהו בזבוז זמן נוראי מבחינתי.

  4. ר.ח, חן חן. וכעת תורי
    א) איזה פסוק מסתיים במילים “ויאמר יעקב”?
    ב) שמו של איזה מלך שקדם לדוד מוזכר בספר מלכים?
    8)

  5. יובל
    דרום לוס אנג’לס ללוס אנג’לס כמו שדרום צרפת (מרוקו) לצרפת.

    שאלה לצבי
    האם אתה מוצא פגם בטיעון שלי שאם נשתמש בנוסחת פרידמן לקשר בין הזמן שחלף מעת המפץ הגדול לטמפרטורה, הרי שנוכל לבנות שעון שיראה את הזמן שחלף מאז המפץ רק עיי מדידת טמפרטורת הקרינה הקוסמית?

    הנוסחה + מחשבון הקושר את הטמפרטורה לזמן נמצאים ב
    http://hyperphysics.phy-astr.gsu.edu/hbase/astro/expand.html#c3

  6. רובי

    אתה חוזר לאותה טעות מפניה ניסיתי להזהיר ומתייחס אל אנלוגיית הבלון כאל הדבר האמיתי ומכאן מגיעה לשאלה מה קורה במרכז הבלון.
    אנלוגיית הבלון היא אנלוגיה כאשר למרכז הבלון אין תפקיד במציאות! זה לא קיים!

    אסביר ביתר תיאור איך הגיעו לרעיון המפץ הגדול ואז אולי תבין עד כמה לא תוכל לשאול על “מרכז הבלון”.

    לאחר שפרסם איינשטיין את משוואת השדה של תורת היחסות הכללית, לקח רוסי בשם פרידמן את המשוואה וניסה להבין ממנה מה קורה ליקום כולו. הוא הניח רק שתי הנחות על היקום שעיקרן היותו אחיד בכל מקום ולכל הכיוונים – כלומר, הוא הניח שאין איזו נקודה מיוחסת, מרכז חור שחור או כל דבר אחר.

    מההנחה הזו הוא קיבל שלוש מטריקות אפשריות – כלומר שלושה פתרונות אפשריים של משוואת השדה של איינשטיין.
    אם כן יש רק שלושה מבנים עקרוניים שיכולים להיות ליקום* תחת ההנחה כי היקום אחיד. כל שלושת המבנים הללו דורשים מפץ גדול. כיוון שההנחה היא כי היקום אחיד על סקאלות גדולות מאד היא הנחה סבירה ומקובלת מאד, וכיוון שנתגלו ראיות לכך שהיה מפץ גדול – נראה כי התאוריה של פרידמן קולעת למדי למציאות – כלומר היקום מתנהג בגדול לפי אחד משלושת המודלים שחזה פרידמן.
    המשמעות היא שאין נקודה מיוחסת (מרכז היקום) – כי בדיוק מההנחה הזו נולד המפץ הגדול ולכן לטעון עכשיו שכיוון שהיה מפץ גדול הרי שליקום יש מרכז, זו פשוט חוסר הבנה עקרונית.

    כדי להסביר לאנשים איך יכול משהו להתרחב ללא נקודה מיוחסת, נותנים את הדוגמא של נקודות על פני בלון – אבל הדוגמא הזו היא ממש לא הדבר האמיתי ואתה לא יכול ללמוד ממנה על הדבר שהיא נועדה לייצג דווקא בדברים בהם היא לא ממחישה את המציאות.

    —————————————————————————————
    * נכון להיום (מאז גילוי התפשטותו המואצת של היקום ב-1998), נראה כי היקום אינו מתנהג עפ”י יקום פרידמן אלא ע”פ מודל אחר הקרוי למבדא CDM. הטעות אינה בפתרון של פרידמן, אלא במשוואת השדה של איינשטיין ולכן זה כל כך מעניין. בכל מקרה גם מודל היקום הנוכחי כולל מפץ גדול כמו מודלי פרידמן השונים, וכן הוא מודל אחיד בקנה מידה גדול דיו.
    מעניין לציין כי משוואת השדה המקורית של איינשטיין (זו עם מה שנקרא הקבוע הקוסמולגי) שתוקנה לאחר מכן להיות המשוואה כפי שפרידמן פתר, היא דווקא בסופו של דבר זו שנכונה.

  7. ר.ח.
    איני חושב שהתעלמתי אי פעם ממשהו שאמרת. הסיבה לכך שאינני מפרט תשובה לכל הסתייגות שלך היא פשוטה: כל מה שאתה מעלה, החל מגלקסיות מתרחקות, עבור דרך מקור רדיו דוגמת גריניץ וכלה בסופרנובה עוסק במדידה באמצעים אלקטרומגנטים, שהם בעצם אור בתדרים שונים. לכן הם אינם קבילים לדיונינו.

    לשאלתך מס 2:

    א. שום דבר לא מתקרר. או אם מתקרר, ללא תלות נוסחתית בפונקציה רציפה כמו נוסחת פרידמן.

    ב. התארכות הזמנים הוכחה נסיונית רק למערכות מואצות. עבור על ניסוי המואונים או הדופלר. תגלה שהם במערכות מואצות.

    ג. אם לא היה מפץ התארכות הזמנים הגיונית, אך אינה מתחייבת. זהו רק פירוש אחד. ראה הרצאתו של סוסקינד.

    ד. הסתירה קיימת. בחיאת ר.ח., למה שלא תסיים עד הסוף את דוגמת ג’-ג’ מתוך ההנחה שאין הבדל בין המערכות האינרציאליות? אם תוכיח לי שאין סתירה אחרי שעשית זאת, משקלך בזהב.

    למעשה, אינני מוצא טעם רב בהמשך הדיון ללא שנגיע להסכמה על נקודה זו. אם אתה מעוניין להמשיך מהנקודה בה הפסקנו, תפדל. אך אם אתה מתכוון להמשיך לטעון שיש הבדל בין המערכות הלא מואצות, ששעוניה של ג’יל יראו יחס שונה משעוניו של ג’ק, שמצלמות באותה נקודה ובאותו הרגע מצלמות תמונות הפוכות, הרי שנשמט הבסיס לדיון לוגי ביננו, אלא אם כן תסביר למה אתה מתכוון כשאתה טוען זאת. אני אמשיך להאמין שקיימת סתירה.

    נקודה חיובית – אם תעבור על הרצאותיו של סוסקינד, לא רק בעניין היחסות, אל גם בקוואנטים, שזירה, קוסמולוגיה ועוד – אולי תסכים איתי שלא צריך ללכת יותר לאוניברסיטה. שמות בכיס הקטן את כל מה שלמדתי אי פעם בUCLA.

  8. ישראל,

    התעלמת מחלק ניכר מהטענות/שאלות שלי.

    1) מכונית – אפשר לגרום למקור אור לשחרר מספר מוגבל של פוטונים בצורה כמותית. זה הרי נעשה בניסוי החריצים המפורסם. אם נבצע זאת ממכונית נוסעת הרי שהפוטונים הללו, או לפחות חלק ניכר מהם, צריכים להעלם לפי המודל שלך. האם זה מה שקורה???

    2) התנגשות בין מפץ גדול להתארכות זמנים. אני לא מבין מה טוען. אם יש התנגשות בין שני אלה אזי יש מספר אפשרויות:
    א. יש התארכות זמנים אבל לא היה מפץ. אז אם כך מה מתקרר? איך זה שכל היקום ביחד מתקרר. ומה שווים שעוני הטמפ’ שלך אם לא היה מפץ?
    ב. היה מפץ אבל אין התארכות בזמנים. התארכות הזמנים הוכחה נסיונית והיא לא נתונה בויכוח.
    ג. אין התארכות ולא היה מפץ – ראה א + ב
    ד. שניהם נכונים ואין סתירה- בחיית ישראל, תשקול את האפשרות הזו שוב.

    אתה כמובן יכול להגיד שאין התנגשות בין התארכות זמנים של גופים בתאוצה אולם בגופים במנוחה אין התארכות זמנים, אולם גם זה הוכח ונבדק ע”י אפקט דופלר שאתה מסרב לדבר עליו.

    לסיום התעלמת מהאנלוגיה שלי בין המפץ לסופר נובה. האם יש לך תשובה לזה? האם לא תהיה סתירה בין התארכות הזמנים לכל סופר נובה שאי פעם קרתה?

  9. ר.ח.
    לעניין הסתייגותיך.
    אינני שולל את המפץ הגדול – פשוט איני רואה איך הוא מסתדר עם התארכות הזמנים אתה כן? אז גמור את דוגמת ג’-ג’. שאלה: נאמר שאתה מקבל את זה שגם ג’ק וגם ג’יל רואים יחס של 1:1. האם גם אז אינך רואה בעיה עם התארכות הזמנים?

    ובענין המטוטלת הבליסטית בודאי שלמכונית יהיה אור. פשוט גלאינו מגלים את הפוטון במהירות אחת בלבד, כמו שעינינו רגישות לאור בקשת תדירויות מסויים ועוורות לאחרות.

    בעניין צרן. יש הסבר מדוע איננו יכולים להאיץ פרוטונים מעל מהירות מסויימת, אך הוא אינו מיינסטרים, ולכן לא נגעתי בו. עדיף שנגמור את הדברים האחרים קודם.

    אינני יודע אם אתה מעוניין לפתוח שנית את הנושא, אך במידה וכן, אינני מוצא טעם להמשיך אותו לפני שהגענו להסכמה על הנושא העיקרי, התארכות הזמנים, ומכאן ג’-ג’.

    רק בקשה אחת: ציינתי אותך לטובה על קליטת הפרטים המהירה. אבל אנא, אם אנו ממשיכים, שלא נגלה אחרי חודש ששנינו מדברים על דברים שונים, כמו שקרה עם שעוניה של ג’יל, שאתה חשבת שהסכמתי על כך שהם מראים יחס שונה מ1:1 ואילו אני כתבתי בפירוש שלא כך הדבר.

    הייתי גם מציע לך להקשיב להרצאות המצויינות של סוסקינד בנושא, כמו בכל נושא אחר.

    http://www.youtube.com/watch?v=BAurgxtOdxY

    שים לב גם שבדקה ה19 אחד המאזינים מעלה שאלה דומה לשלי, ושסוסקינד די מחפף אותו.

  10. רובי
    אני רואה שלא הבנת מה כתבתי.

    כתבת:
    “תתעלם מהנקודה הסינגולרית ותתייחס למעטפת הנקודה, כלומר מספר שניות של התפשטות החוצה משם ואז יהיה לנו אפשרות אולי לנסות להבין מה קורה שם במרכז.” – טענה לא ברורה.

    וגם:
    “אני תוהה למה כל המומחים מתייחסים למודל הבלון המתנפח של היקום שנובע מתצפיות האסטרונומיות כמובן ומשוואות איינשטיין ליחסות הפרטית והכללית אבל אף אחד לא מנסה לטפל במה שקורה במרכז הבלון.” – גם כאן, לא ברור לי מה אתה מנסה לטעון. למה התכוונת ב: “אף אחד לא מנסה לטפל במה שקורה במרכז הבלון.”?

  11. ר.ח.
    אני מכיר את המושג של נקודה סינגולרית והיתה לי הזדמנות לתהות על קנקנה בלימודי ההנדסה בעבר.
    זה לא הנושא, אני תוהה למה כל המומחים מתייחסים למודל הבלון המתנפח של היקום שנובע מתצפיות האסטרונומיות כמובן ומשוואות איינשטיין ליחסות הפרטית והכללית אבל אף אחד לא מנסה לטפל במה שקורה במרכז הבלון.
    על פי הידע הפיסיקלי שלנו ותצפיות על סופר נובות אנו יודעים שהתפשטות החוצה של אנרגיה, אור וחומר מחייבת גם התכווצות פנימה הזהה לה בעוצמתה ומנוגדת לה בכוונה.
    תתעלם מהנקודה הסינגולרית ותתייחס למעטפת הנקודה, כלומר מספר שניות של התפשטות החוצה משם ואז יהיה לנו אפשרות אולי לנסות להבין מה קורה שם במרכז.

  12. רובי
    https://www.hayadan.org.il/astronomers-reach-new-frontiers-of-dark-matter-130112/#comment-333377

    “מתעלמים ממרכז הנקודה הסינגולארית”- לנקודה סינגולרית אין מרכז. הנקודה הסינגולרית מייצגת את המרכז עצמו. הבעיה היא עם המרכז של הנקודה. ניתן לטעון שכל נקודה תהיה מרכז המעגל, ולמרכז המעגל יהיה גם מרכז. הרי כשאתה נועץ את המחוג בשביל לצייר מעגל אז גם למקום שבו ננעץ המחוג (שמהווה את מרכז המעגל- כלומר מהנקודה הזאת והלאה נמדד הקוטר של המעגל) יש מרכז. אם אתה מצייר עיגול בעזרת מחוגה על הדף אז המרכז של מרכז המעגל יהיה חור בדף עצמו, למשל.
    במתמטיקה (לפי ויקיפדיה) “נקודה סינגולרית היא נקודה שבה פונקציה (בדרך כלל פונקציה מרוכבת) או משוואה דיפרנציאלית איננה מוגדרת היטב.”.
    ביקום פיזיקלי כמו שלנו, לנקודה הסינגולרית יש משמעות נוספת. היא באה לתאר את המערכת שממנה הכל החל. לפי תאורית המפץ הגדול (או יותר נכון מהמשוואות שעולות ממנה) הנקודה הסינגולרית צריכה להיות קטנה מאורך פלאנק. כלומר כל החומר והפיזיקה וכל מה שיש נוצר מתוך ‘דבר’ שאורכו קטן מאורך פלאנק. במילים אחרות אפשר להגיד שהחלקיק הראשון נוצר מתוך דבר כלשהו, שהוא גם יותר קצר ממנו גם שוקל פחות ממנו וגם היה צריך להיות מקום שבו האנטרופיות גבוהה ביותר ברגע הקריטי של התחלת המפץ הגדול (כאוס מוחלט של המערכת בגלל אופי התנהגות החלקיקים שמרכיבים את המערכת שנמצאים בטמפ’ הכי גבוהה).

    הנקודה הס’ זה תהליך שמתרחש במרחב שיוצר את המרכז של (המעגל) ה’ספירה’ שאותה מכנים ‘יקום’.
    אחרי החלת המרכז של המעגל, ניתן יהיה לחשב את קוטרו. (או במקרה הנדון הקוטר {והנפח} של היקום).
    במתמטיקה אין משמעות למרכז הזה מכיוון שהמרכז הזה (נקודה סינגולרית) בהגדרתו אינו ניתן למדידה (ובעצם, נקבע שהנקודה הסינגולרית היא מקרה בעייתי בכל מה שקשור להגדרה וככזאת אינה ניתנת להגדרה מתמטית {התוצאות, במשוואות, אינן מסתדרות בעולם פיזיקלי. האובייקט שמהווה את מרכז המעגל שהוא היקום – אינו מוגדר מתמטית}).
    לפי ויקיפדיה האנגלית:
    “For example, the function: f(x)=1/x
    on the real line has a singularity at x = 0, where it seems to “explode” to ±∞ and is not defined. The function g(x) = |x| (see absolute value) also has a singularity at x = 0, since it is not differentiable there. Similarly, the graph defined by y2 = x also has a singularity at (0,0), this time because it has a “corner” (vertical tangent) at that point.”
    שים לב שהנקודה הס’ היא ‘קבוצה סגורה’.
    כלומר מתוך קבוצה סגורה נוצרת (כמו בפעולת העוקב במערכת פאנו) קבוצה פתוחה (שהיא ה’יקום’) .
    הרי לפי המפץ, מתוך הקבוצה הסגורה והבלתי מוגדרת נוצרה התפיחה (שגדלה באופן מעריכי) של המרחב עצמו, כלומר הקורדינאטות של המרחב הפיזיקלי קיבלו ערכים גדולים מ-0.
    אם עד כאן הבנת, אני יכול להמשיך. חשוב להבין דבר נוסף, הנקודה הסינגולרית אינה נעלמת בעקבות המפץ הגדול. היא עדיין שרירה וקיימת (הודות לחוקים התרמודינאמים).

  13. יובל, תשובה יפה דוז פואה!

    ישראל,
    מכיוון שפתחת את נושא המפץ הגדול הגיע סוקרטס בריצה כולו נסער:

    1) אם אתה שולל את המפץ הרי שאתה משמיט את הקרקע מתחת לרעיון שעוני הטמפ’ שלך. כי אם לא היה מפץ אזי התקררות של מה הם מודדים? ההנחה בבסיס השעונים היא שההתקררות אחידה בכל מקום ואתר ביקום כי היה מפץ. אם לא היה אזי מה שנמדד אלו הן פלקטואציות מקומיות של טמפ’ וכל רעיון השעונים לא שווה כלום.

    2) סוקרטס מציע את הניסוי הבא: ג’ק וג’יל יזהו סופרנובה ומייד ייתייצבו שם. מה שיהיה שם זה ענן גז שהולך ומתקרר, הפלא ופלא לפי נוסחת פרידמן. ג’ק וג’יל יישלפו את שעוניהם ויתחלילו למדוד, לצלם להשוות עם שעוני הצזיום וכל אותם דברים אותם דיסקסנו בעבר.
    אולם אז יופיע לפתע מר ישראל שפירא שיגיד להם ” הי חבר’ה, דמיינתם את הסופר נובה שהרי קיומה סותר את היחסות ואת התארכות הזמנים. לא ייתכן שהיא קרתה!”

    וסוקרטס שואל, אתה רואה את הסתירה?

  14. ר.ח,
    בספר שמואל ב פרק י”א פסוק ג’ מסופר כי בת שבע היא בת אליעם. בפרק כ”ג פסוק ל”ד מסופר כי אליעם הוא בן אחיתפל. אם זה אותו אליעם אזי סבה של בת שבע מצד אביה הוא אחיתפל. ספר דברי הימים, לעומת זאת, מכנה אותה בת שוע בת עמיאל.
    ואני מציע לא להמשיך להזין את מדורת הויכוח הבלתי נלאית הזאת. אנחנו רק גורמים לו נזק ומבזבזים את זמנו. הוא צריך להתעמק ברעיונותיו עם חוקרים מקצועיים באקדמיה ולא עם הדיוטות כאן.

  15. ישראל,
    ועוד משהו. לפי מודל המטוטלת הבליסטית שלך למכונית נוסעת לא היה צריך להיות אור כלל וכלל. אתה טוען שאנחנו יכולים לחוש אור רק במהירות – C ושמהירות האור היא יחסית. אם כך, האור שיוצא ממכונית הוא במהירות C+100 קמ”ש היה צריך להעלם. בנוסף גם אף פעם לא נתת הסברים מניחים על הדעת למה אי אפשר לבנות גלאי ש”יראה” אור יותר מהיר. למשל מסך בעובי עצום. מטוטלת כבדה באנלוגיה שלך. לא הסברת למה לא רואים אור איטי מ-C דבר שמתבקש מהמודל שלך. פטרת את זה כדרכך כדבר מעניין אך לא רלוונטי.

  16. ישראל,
    אני מצטער שאחרי כל המלל שנשפך וכל הסיכומים אתה רואה את מהות הויכוח ביננו בנקודה שציינת בלבד.
    מהות הויכוח לדעתי היא שלא הצלחת להראות שום סתירה חדשה, כזו שלא נידונה בעבר וניתנו לה פתרונות. למעשה מה שהבנתי בסוף היה שאתה פשוט מדבר על פרדוקס התאומים שנחקר הוכח ונדון אינספור פעמים. ניסית להתחמק ממנו בנפנופי ידיים שהפרדוקס נוגע רק למערכות שאינן מואצות, מה שלא ממש נכון. התעלמת מסיבה עלומה מהוכחות מבוססות אפקט דופלר למרות שהוא לא נוגע רק למערכות מואצות ולקינוח החלטת להתעלם גם מכל החלקיקי בעלי מסה. למה? כי הם לא מסתדרים לך.

    אבשלום, לדעתי מדובר על בחור מתוסכל שאחותו נאנסה באכזריות והוא ראה בעניים כלות שאביו אינו מבצע שום פעולת עונשין כנגד הפושע. לאחר מכן כשהוא מתנקם, דוקא הוא זה שצריך לנוס על חייו. ואז כשהוא חוזר הוא זוכה להתעלמות מופגנת שהגבירה כנראה את השאט שהוא חש כלפי אביו עד לנקודה שבה התפרץ במרד.
    חידה: מי היה סבה של בת-שבע?

  17. יובל

    “אני מתפעל כל פעם מחדש מן הכישרון שלך למשוך אנשים לויכוחים שמשאירים בפיהם טעם של לא-עוד”

    אני גם כן די מתפלא על הכישרון שלך לקרוא רבע משפט ומיד להסיק מסקנות, בדרך כלל שגויות.

    לדוגמה: נראה שאתה מסיק שאני מעוניין להמשיך בויכוח, ונראה שאתה מסיק שאני מושך לאיזה שהוא כיוון. מכיוון שאהי היחידי שיודע מה הן כוונותי, אני מסיק שאתה טועה. כרגיל.

    ” שעוני טמפרטורה הינם יצירי תיאוריה ודמיון. כדאי ורצוי לבנות אותם, אך כל עוד אין כזה בנמצא, חבל על שפך המילים.”

    אילו טרחת לקרוא את הדיון ביני לבין ר.ח., היית מוצא שהוא בנוי בצורת נדבכים, שעל כל אחד מהם צריכים שני הצדדים להסכים. על אפשרות בנייתם של שעוני טמפ’ והתקנתם איפה שרוצים הסכמנו בשלב מוקדם מאוד. יתכן שאתה חולק על כך, כמו על כל נושא אחר, אך זה לא משנה לעצם הדיון, שבו לא טרחת להשתתף, לא העלית את ההסתיגות הנ”ל, ולכן לא קיבלת תשובה מדוע הדבר אפשרי. (מדוע לא אגב? אם הזמן קשור לטמפרטורה עיי נוסחת פרידמן ואת הטמפרטורה ניתן למדוד, מדוע אי אפשר לצרף את השניים לשעון טמפ’?).

    “אתה מעיד על זה שאינך מבין “איך אפשר לדבר על התחלה לזמן, מה היה קודם”, אך בין אי-הבנה לבין שלילה המרחק גדול.”

    כרגיל, לא קראת או לא הבנת. מי שלל? כל מה שאמרתי במשך כל הדיון הוא שהתארכות הזמנים ביחסות אינה מתיישבת עם הזמן המוחלט של המפץ הגדול. זהו. מכיוון שר.ח. הסיק שאולי אני שולל את היחסות, ציינתי שאני אישית מעדיף אותה על המפץ, אך לא שללתי אף אחת, רק הצבעתי על מה שכיניתי “סתירה כביכול”.

    לעצם העניין: יש הגורסים, בצדק כנראה, שבכביש, בפוליטיקה ובאהבה, הצדק אינו חשוב. אני מאמין שבמתמטיקה ובפיזיקה חשוב להיות צודק, ושההפך מצודק זה טועה.

    הויכוח ביני לבין ר.ח. נתקע על השאלה הבאה: ר.ח. טוען שיש מערכת אינרציאלית שנוכל להגיד שהיא “נחה” באמת – בדוגמה שלנו ג’ק, שיחס השעונים שלו הוא 1:1, ושיש מערכת אינרציאלית שהיא “נעה” באמת – בדוגמה שלנו ג’יל, שיחס השעונים שלה מיליארד לאחד.

    אילו היה ר.ח. מקבל את הנחתי נראה לי שהסתירה בין הנחת התארכות הזמנים והזמן המוחלט של שעוני הטמפ’ היה ברור לחלוטין (אם היחס הוא 1:1 בשתי המערכות, הרי שהשעוני הצז’ והטמפ’ במערכת נתונה מראים תמיד את אותו הזמן, ומכיוון ששעוני טמפ’ מראים את אותו הזמן בזמן המפגש, הרי שכל 4 השעונים מראים את אותו הזמן, בניגוד להתארכות הזמנים שבה שעוני הצז’ מראים זמנים שונים).

    ומכיוון שאני מרגיש על אדמה מוצקה לחלוטין כאשר אני קובע שלא ניתן להבחין בין מערכות אינרציאליות (מלבד אולי בהתיחסות למערכת הCMBR, שלא התייחסנו אליה בדיון, וגם לא איינשטיין ב1905 כאשר לא ידע אפילו על קיומה) אין לי אלא להסיק שר.ח. לא הצביע על שום טעות קבילה בדיון הארוך ההוא.

    בעניין ההסתייגות שלו לסתירה כביכול עם מאיצי החלקיקים בצרן – התוכל להסביר את נושא עליית המסה של גופים במהירות לפי המודל שלך? לפי מודל האתר האקטיבי התשובה היא די ברורה, למרות שלא הייתי מעוניין לפתוח חזית חדשה בלי שהצלחנו לפתור את הבעיה הפשוטה יותר של התארכות הזמנים.

    ובעניין דוד ואבשלום – לדעתי שניהם גילו מאוחר מדי את חוסר התכלית של פירוק המשפחה על רקע של שררה וכבוד. אבשלום אולי חשב על כך כשנתלה בין ענפי האלה, ודוד כשגלה מרצון מהארמון על נשותיו הבוגדניות.

    אבשלום, אבשלום בני אבשלום, מי יתן מותי תחתיך היום. הו בני בני אבשלום.

  18. יובל, נראה לי שהנסתר רב על הגלוי.
    אנסה לחפש תשובות ואם יהיה מה לחדש, אעדכן את הפורום.
    תודה על הנסיון להסביר

  19. רובי,
    אין לי הרבה מה להוסיף על דבריהם של ר.ח וצבי, אך אנסה לפשט את הדברים. הדוגלים בתאוריית המפץ הגדול מציגים תמונה לפיה כל היקום החל בנקודה מיקרוסקופית. הרושם המתקבל הוא כאילו היקום היה מרחב גדול וריק שבמקום מסוים בו הופיעה נקודה סינגולרית והתחילה להתפשט. אך התיאור הזה אינו בהכרח נכון היות שהנקודה הזאת איננה נקודה אחת בתוך מרחב אינסופי אלא היא בעצמה כל היקום כולו. היות שאיננו מכירים יקומים אחרים להשוות עם היקום שבו אנו קיימים הרי, אם תרצה, כל היקום הגדול הזה הוא עדיין אותה נקודה מיקרוסקופית. מותר להגיד, ויתכן שזה אף נכון ומדויק, שהיקום אינו מתפשט אלא מתחלק. היקום הנקודתי התחלק לשניים (בדומה להתחלקות תא חי בביולוגיה), כל חלק התחלק גם הוא לשניים וחוזר חלילה עד אינסוף. באופן הזה היקום נותר בגודל של נקודה אך מנקודת המבט של המרכיבים הפנימיים שלו הוא גדל ומתפשט ללא הפסק. מן התיאור הזה (ויתכנו אולי גם תיאורים אחרים) מתקבל כי אין משמעות לנקודה סינגולרית.

  20. רובי,

    “זה בדיוק מה שמפריע לי, שאין ליקום מרכז. אם הכל התחיל מנקודה אחת מרוכזת אין סופית, לא הגיוני שהכל יצא החוצה (לקרום של בלון אחד) מבלי שחלק כלשהו יתכנס פנימה למרכז של חור שחור.”

    למה זה מפריע לך? האם מחור שחור יש תנועה דו כיוונית? למה לא יכול להיות שזה נקודה סינגולרית שהולכת ותופחת ואין התכנסות פנימה? מעין הופכי של חור שחור?

  21. צבי, רעיון הבלון היקומי שמתפשט ידוע ומאד פופולרי, אם ניקח את המצב הנוכחי ונעשה Rewind אחורה בזמן, נקבל נקודה אחת במרכז ומכאן הגיע ההנחה שהיה מפץ גדול.

    ההנחה שהיקום אחיד אינה תואמת לתאורית היקום / בלון מתנפח הווה אומר מעטפת מתנפחת ללא חומר בפנים הבלון.

    לעניות דעתי, מערכת הצירים הבלונית שגדלה / מואצת עם הזמן לא נותנת הסבר מספק לשאלה למה במרכז היקום “(ראשית הצירים) אין חור שחור.

  22. רובי ויובל,

    ליקום אין “מרכז” שיש לברר מה קורה בו.

    רעיון המפץ הגדול נולד מעבודתו של פרידמן שחיפש פתרונות למשוואות איינשטיין שבבסיסם ההנחה כי היקום הומוגני ואיזוטרופי – כלומר, ההנחה שהובילה מלכתחילה לרעיון המפץ הגדול היא כי אין ביקום נקודה ייחודית כלשהי וכי כל היקום על סקאלות גדולות מספיק, אחיד למדי.

    השאלה מהי הנקודה בה הכל התחיל מניחה מעין עצם אשר הולך ומתפשט במרחב אוקלידי ואתם שואלים מה קורה בקורדינטה בה הכל התחיל – אך ביחסות כללית זה פשוט לא כך – מה שנוצר במפץ הגדול הוא אם תרצו מערכת הקורדינטות.

    האנלוגיה המקובלת לזה היא כלהלן:
    דמיינו בלון שהולך ומתנפח, כאשר הגלקסיות הן נקודות המצויירות על פניו.
    הגלקסיות הולכות ומתרחקות זו מזו במהירות אשר גדלה עם מרחקן זו מזו ואף על פי כן לא תוכלו לומר איזו נקודה נמצאת במרכז – כל הנקודות מצויירות על הבלון ומבחינת כל אחת מהן היא הנייחת וכל האחרות זזות.
    המציאות כמובן קצת יותר מסובכת מהאנלוגיה,שכן באנלוגיה העקמומיות היא חיובית (סכום זוויות במשולש על פני הבלון גדול מ-180 מעלות) בעוד המציאות היא עד כמה שרואים שטוחה. כמו כן אין שום דבר מציאותי המקביל לפנים הבלון ואף על פי כן אני מקווה שהאנלוגיה תורמת משהו להבנה.

  23. ר.ח.
    זה בדיוק מה שמפריע לי, שאין ליקום מרכז. אם הכל התחיל מנקודה אחת מרוכזת אין סופית, לא הגיוני שהכל יצא החוצה (לקרום של בלון אחד) מבלי שחלק כלשהו יתכנס פנימה למרכז של חור שחור.

  24. רובי,

    תאוריית המפץ הגדול הנוכחית גורסת שאין מרכז ליקום ואי אפשר לדעת היכן הייתה הנקודה הסינגולרית. דמיין שאתה נקודה מצויירת על בלון ללא פייה שהולך ומתנפח. מה שתראה זה שכל הנקודות האחרות שמצויירות על הבלון הולכות ומתרחקות זו מזו וממך. אין משמעות לשאלה מהיכן החל הניפוח או היכן היה הבלון כשהיה בעצמו נקודה.

    עכשיו תדמיין את זה ב 3 מימדים וזה מה שהתאוריה טוענת.

  25. יובל, אין לי הרבה מה להרחיב מלבד זה שתאורית המפץ הגדול לא תהיה שלימה ללא התייחסות למה שקורה במרכז היקום. לא נראה לי הגיוני שהחומר מתפשט החוצה בלבד ללא התכנסות פנימה למרכז. עדיין לא ראיתי חומר בנושא זה. (לא מאמין שאף אחד לא חשב על זה לפני).
    אולי זה גם כן נושא לתיזה שלי בלימודי המשך באוניברסיטה…

  26. יובל, ההתכנסות פנימה יכולה להיות מלבד גרביטצייה גם בגלל כוחות נגדיים (החוק השני של ניוטון) שגרמו להתפשטות. אני מבין שלפני המפץ לא היה זמן ולא היו חוקים פיזיקליים אבל שניות אח”כ כבר אפשר לדבר על כוחות, מהירויות / תאוצות וזמן.
    תמיד מדברים על הגלאקציות שמתפשטות / מאיצות במרחקים של מליארדי שנות אור אבל מתעלמים ממרכז הנקודה הסינגולארית שמממנה הכל התחיל. מה קורה שם? חור שחור? מעיין של יקומים נוספים? יש לך איזה לינק לנושא זה?

  27. רובי,
    אני מניח שכשאתה אומר “התכנס במקביל גם פנימה כמו בסופר נובות” אתה מתכוון להגיד שתוצרי הפיצוץ חוזרים ומתכנסים כתוצאה מכוח המשיכה הפועל בין מאסות.
    האסוציאציה המתקבלת מן השם “מפץ גדול” עלולה להטעות. התיאוריה לא מדברת על פיצוץ כמו זה של חומר נפץ אלא על האופן בו נוצר כל החומר ביקום ונוצרו חוקי הפיסיקה. למשל, החומר, ברגעים הראשונים שלאחר המפץ הגדול, לא היה החומר שהפיסיקה מכירה היום. גם הגראביטציה לא הייתה קיימת בעת ההיא אלא נוצרה זמן מסוים לאחר הרגע הזה.
    לאור תצפיות בספקטרום המגיע מגלקסיות רחוקות ותליית ההסבר באפקט דופלר, מניחים רוב הפיסיקאים כי היקום לא זו בלבד שהוא מתפשט אלא הוא אף מאיץ. הפיסיקה אינה מכירה תאוצה ללא השקעת אנרגיה, אך היות ששום מקור אנרגיה לא נראה, זכתה התופעה הזאת בשם “אנרגיה אפלה”. לכן, בטרם נתפנה לחפש השוואה בין המפץ הגדול לבין סופרנובות, עומדות בפנינו חידות קודמות.
    אם הבנתי נכון, אז האנלוגיה לקליפות הבצל היא במקומה. היות שהיקום אינו מתפשט רק על פני שטח מסוים אלא על פני נפח, הרי זה לא רק בלון אחד כי אם הרבה בלונים (או “קליפות בצל”) אחד בתוך השני

  28. יובל, נראה לי שנסחפנו למחוזות רחוקים מידי…
    יובל, אבי, לא קיבלתי תגובה לתאורית קליפות הבצל שלי. כאחד שמתעניין בנושא ורואה את כל תוכניות המדע העוסקות באסטרונומיה (היקום, חקר כדור הארץ וכדומה), לא הבנתי למה תאורית המפץ הגדול מניחה שאותה נקודה קטנה שממנה התפשט היקום התפזר החוצה בלבד ולא התכנס במקביל גם פנימה כמו בסופר נובות?
    יש למישהו תשובה?

  29. רובי, ברשותך אנסה להתרחב מדבריך אל המיתולוגיה השומרית (בהתנצלות לאבי – לעניין שלי במודלים המיתולוגיים השונים של בריאת העולם אין קשר עם האמונה האישית שלי):
    “קודם היתה ראשית ליקום” וחָשָׂך (תרגום מן המילה אפס, האל אפסו. ספר בראשית מציג משחק מילים: המילה חשך משמשת בשני מובנים: “ריק” ו”אפלה”) על פני תהם (אינסוף), זו תהמת מן המיתולוגיה. המודל המטאפיסי שממנו שאלה המיתולוגיה השומרית מדבר על אפסו ותהמת, אפס כפול אינסוף (חשך על פני תהם) שהיווה את היקום הבראשיתי. אני מעריך שהחומר האפל שמדברים עליו היום הוא תולדה של פעולה מטא-מתמטית מן הסוג הזה (אפס מחולק באפס או אפס כפול אינסוף) והוא ממלא את כל חלל היקום. המודל השומרי קולע די במדויק להערכה שלי.
    הארץ היתה מורכבת משני הסלעים תהו ובהו שהמפלצת תהמת בלעה בטרם הצליחה קואליציית האלים לנטרל אותה. ראש קואליציית האלים (מרדוך) שחרר אותם מבטנה של תהמת ויצר מהם את הארץ. הסיפור הזה לקוח ממודל מטאפיסי אחר, שהיה מאד פופולארי, אך אינני מתעניין בו כרגע.
    מה שיש בבראשית ואין במיתולוגיה השומרית הוא יצירת האור. נמצאו טקסטים של המיתולוגיה השומרית המתוארכים לאלף הששי לפני הספירה. התורה נכתבה כמה אלפי שנים אחריהם. אם נצא מנקודת הנחה די מבוססת שהתורה עברה שכתוב ועריכה מאסיביים בימי המלכים חזקיהו ויאשיהו, בעת שבאיים האיונים פרחה הפילוסופיה הפרה-סוקראטית, נוכל למצוא הקבלה בין יצירת האור לבין רעיונותיו של הירקליטוס: אנרגיה (האור) ומאסה (החומר האפל) הם שני פנים של אותו דבר כפי שטוענת גם הפיסיקה המודרנית.

  30. אבי, מי אמר שהשתמשו בטלפונים אלחוטיים?
    אני לא הנציג של התורה אבל כמו שאתה יודע, כל מה שאנחנו יודעים על היקום נעשה בעזרת תצפיות ושימוש בחושים הידועים שלנו ובעזרת מכשירים שקולטים מידע נוסף מעבר לחושים שלנו כמו שדות אלקטרומגנטיים ואורכי גל שונים של קרינה וגלי קול.
    להערכתי ישנו עוד מידע רב שהטכנולוגיה שיש כעת לא מזהה ובעתיד כנראה נחשוף חלק קטן ממידע זה.
    אז אנחנו צריכים להיות קצת צנועים ולא לחשוב שאנחנו יודעים הכל…

  31. רובי, באותה המידה איך אנחנו יודעים שלפני אלפיים שנה השתמשו בטלפונים אלחוטיים? לא מצאו שום כבל באדמה מאותה תקופה. זו בערך רמת התשובה שלך

  32. אולי חושך על פני תהום?
    אולי צבא הכוכבים ומסילותיהם?
    כידוע לך התורה מלאה רמזים…זה כבר קשור לתורת הנסתר.

  33. איך היה אומר שמוליק רוזן למשמע חידתך:
    בן סורר ומורה, קפד ראשו, שם ניתן למצוא את לשונו של חיים.

  34. ר.ח, בדיוק 🙂
    אבשלום מבקש רשות ללכת חברונה באיזשהו תרוץ נלוז. אביו, החובש ברגע זה כובע של נביא, יודע את העתיד לקרות ואומר לו כאילו “פסדר” כזה, “לך אבשלום”. בניסוח הזה אין ברכה אלא מתן היתר. אך בהשמיטו את האות הראשונה של שמו, “לך [א]בשלום” מה שלכאורה מתפרש כברכה, הוא מנבא לו כי יאבד את ראשו…

  35. יובל,

    האם התכוונת לברכתו של דוד לאבשלום שהולך כביכול לזבוח בחברון ולמעשה הולך למרוד: “לך בשלום”?

    אם כן מדוע זה מציין את סופו המר?

  36. ישראל,
    כמי שמאמין בקיומו של רצון חופשי, עלי להודות שאינך גורר אף אחד בעל כורחו. לכן אני מתפעל כל פעם מחדש מן הכישרון שלך למשוך אנשים לויכוחים שמשאירים בפיהם טעם של לא-עוד. נראה לי כי מה שמושך הוא השאלות שאתה מעלה, שהן בעלות חשיבות עליונה. מה שדוחה (אותי, לפחות) הוא המסקנות שאתה מגיע אליהן בדרך נפתלת שמתישה מאד את הנכנס איתך לדיון (גם כאן אני מדבר רק בשם עצמי).
    ולתהיותיך על הזמן בפוסט האחרון שלך:
    1) שעוני טמפרטורה הינם יצירי תיאוריה ודמיון. כדאי ורצוי לבנות אותם, אך כל עוד אין כזה בנמצא, חבל על שפך המילים.
    2) אתה מעיד על זה שאינך מבין “איך אפשר לדבר על התחלה לזמן, מה היה קודם”, אך בין אי-הבנה לבין שלילה המרחק גדול. אתה חי בתוך זמן ומודע לו, ומתייחס אליו כאל ישות נצחית הקיימת מעבר לכל תופעה פיסיקלית ומשום כך אינך יכול להבין “זמן או כל דבר אחר שקדם לזמן”. אך לא מן הנמנע הוא כי התייחסותך שגויה, והזמן אינו ישות נצחית כי אם תופעה פיסיקלית. הרי לך משל: כל היקום כולו הוא שעון פועם; כל חלקיק, בין אם חומר ובין אם חלל ריק, הוא שעון אחד במכלול הגדול הזה שנקרא היקום; כאשר חלקיק משתכפל מחלקיק אחר (או תא חי מתא חי אחר) הוא יורש מן האב את תכונת השעוניות; כעת הקבל בין השעון היקומי הזה לבין שעון מסוג המכשירים שאנחנו משתמשים בהם למדידת זמן – למשל שעון הפועל על אנרגיה של קפיץ או חשמל; השעון הזה מתחיל לפעום רק אחרי שמישהו דרך אותו או התקין בו סוללה. למאורע המכונן הזה אפשר לקרוא בהרבה שמות, כמו “בראשית” או “מפצגדול” 🙂 או מה שתרצה. שים נא לבך לכך ששעוני הטמפרטורה, אהוביך, אינם מודדים זמן מוחלט אלא רק החל מאותו מאורע מכונן.
    סע בשלום ובדרך נסה בבקשה לפתור את חידתי. לא מרים ולא בנה, אך יש אומרים כי קרוב משפחה.

  37. ר.ח רפאי.ם, 😀
    החידה הזו באה כדי למלא את הזמן ולהפנות את המרץ מן הויכוח המתיש ההוא.
    אתה מוזמן להשתתף גם. הפתרון יפה ומפתיע. רמזים על פי דרישה…

  38. ממש: אתוס, פרטוס ואראמיס.
    צריך לשנות את הכותרת ל: “שלושת המוסקטרים – הבלוג”
    המוסקטר סוקרטס ישאל שאלות בסגנון אריסטוטל (מי מכיר את הבית יותר טוב- מי שגר בו, או מי שבנה אותו?) ויזכה את קוראיו בתשובות מהתנך.

  39. ר.ח. יקר

    אני אינני גורר אף אחד בעל כורחו לשום דבר. אם התמצינו, אז כדאי שנסיים, אפילו אם לא הצלחנו לשכנע זה את זה.

    1. פרדוקס התאומים – ניתן לפי איינשטיין לסנכרן את השעונים, אך רק עיי תקשורת בינהם, ובאמצעים שמהירותם האור ומטה. מכאן נובעת התארכות הזמנים.

    על ידי שימוש בשעוני טמפ’ הסינכרון הוא מיידי, ולכן מעקר את התארכות הזמנים.

    2. טענתי גם טענתי. אינך זוכר כמה פעמים אמרתי שאינני מבין איך אפשר לדבר על התחלה לזמן? מה היה קודם? רוצה שאמצא את הפוסטים הספציפים?

    3. מחשבונים כאלו יש המון. ברור שזה מה שטוענת היחסות. אי אפשר להוכיח לי את הולדתו של ישו, אם איני מקבל את קיומה של מריה.

    4. הנוסחאות והחישובים אינם יכולים להגיע לפני שמסכימים על העיקרון. מכאן ג’-ג’.

    5. גם אני מסכים שדיונים כאלו עדיפים על קריאה יבשה של חומר. הם מכריחים אותך להיות מעורב לגמרי. לכן תודה גם לך על ההשקעה הרבה.

    6. עשיו. סתם התנכלו לו הדוסים לדעתי. איש נחמד ותמים, שסודר עיי אמו ואחיו התחמנים. בל נלין על עמלק אחרי מה שעשינו לסבא.

    אני נוסע לתערוכה בווגאס כך שלא אוכל להגיב בקרוב. חשבו בינתיים על הנושא הבא:

    בתחילת המאה ה20 היו התעשיה והתחבורה מפותחות למדי. רכבות חצו יבשות, אניות בסדר גודל של הטיטניק חצו אוקינוסים, אינטרסים כלכליים אדירים הושקעו בכל נושא התעבורה. עשרות אלפי מהנדסים עבדו על בניה ושיפור של אמצעים קיימים.

    ולכן מוזר הדבר שדווקא ההמצאה החשובה מכולן – המטוס – הושארה לשני מתקני אופניים חסרי השכלה מאוהיו, וילבור ואורביל רייט.

    אז מה זה אומר?

    זה אומר דרשני.

  40. ישראל,

    עכשיו אתה גורר אותי בעל כורחי לדיון.

    1) “לכן ברור שאם ניתן לסנכרן את השעונים ללא קשר ישיר, כמו שהצעתי, התארכות הזמנים מעוקרת.” למה? האם בפרדוקס התאומים אי אפשר לסנכרן את השעונים למרות שהזמנים שונים אצל כל תאום? האם שעוני הטמפ’ שלך או כל שינוי קוסמי אחר לא יוכל להוות ” עוגן” סינכרון במערכות מואצות? בודאי שכן.

    2) אתה טוען שאני מחמיץ את הנקודה העיקרית “שאינני נגד היחסות, אלא מצביע על הסתירה בין 2 התאוריות. אישית, אני מאמין ביחסות והזמן היחסי שלה הרבה יותר מאשר במפץ הגדול והזמן המוחלט שמשתמע ממנו.”

    זה המשפט שהחזיר אותי בכח לדיון. איך אתה יכול לטעון אחרי למעלה מ 1000 פוסטים שאנחנו מתווכחים על היחסות ולא העלית אפילו פעם אחת הסתייגות מהתוצאות המעידות על המפץ הגדול, שאתה מאמין יותר ליחסות וחושב שהמפץ הגדול שגוי. פתחת פה חזית חדשה בגודל של קו מאז’ינו .

    3) ראה את המחשבון הבא: http://www.wolframalpha.com/input/?i=time+dilation+calculator&a=*FS-_*RelativisticTimeDilationFormula.to-&f2=1+s&f=RelativisticTimeDilationFormula.to_1+s&f3=3×10%5E7+m%2Fs&f=RelativisticTimeDilationFormula.v_3x10%5E7+m%2Fs

    שמחשב התארכות זמנים. שים לב שאין בו שום תאוצה אלא רק מהירות (velocity).

    4) בכל מקרה הסיבה שפרשתי מהדיון היא לא שהשתכנעתי או שאני מנסה לשנות נושא. להרגשתי מיצינו את הדיון הפילוסופי והויכוח צריך לעבור לפסים של נוסחאות וחישובים. מאחר ולצערי אין אני בקיא מספיק ואין לי , שוב לצערי, את הזמן ללמוד את משוואות היחסות לעומק אני נוטש את הזירה לטובת מי שמתמצא בעניין.

    5) מה שכן, אני מודה לך על הדיון שבעקבותיו אכן הועמקו ידיעותי בתורת היחסות בצורה ניכרת ולפחות זו תוצאה חיובית אחת מכל הטרררם הזה.

    6) תמיד אשמח לשוב ודון בדמויות תנכ”יות

  41. רובי
    כשערכתי משפט לחוק השני, זה בגלל האמונה שלי שאין דבר כזה אנשים רעים. אם תיקח שני אנשים, אחד “טוב” לפי כל קנה מידה והשני “רע” ותאמר להם שהנה הולך להיוולד להם ילד, והם יכולים לבחור עכשיו אם כשהילד יגדל הוא יהיה לאדם טוב או רע – שניהם יבחרו בטוב.

    מסקנה – כולנו טובים, והרע הוא תוצאה של נסיבות, או החוק השני.

    המסקנה שלי לגבי מערכות אותן אני מכנה “פסיכומכניות” אינן ערכיות או מוסריות. הן עובדתיות. במלחמת העולם השניה מתו כ60 מיליון איש. מאז מתו במלחמות עוד יותר מ100 מיליון. אחרי נפילת הגוש המזרחי דובר רבות על “קץ ההסטוריה” ועל שלום עולמי. במקום זה אנחנו עדים לעליית הפאונדמנטליזם האיסלאמי. המערכות פשוט מתחלפות, וזה מה שניסיתי להראות בדוגמת איראן.

    הנה תחזית שלי, המבוססת על עקרונות הפסיכומכניקה, ושנוכל לראות אם יש בה ממש: בעשור הנוכחי אחד מאויבינו הראשיים, חמס, איראן, לבנון או סוריה או מישהו אחר – ימתן מאוד את יחסו אלינו, כפיצוי על עליית מפלס השנאה מצד צדדים אחרים. זו אגב תוצאה הכרחית של החוק הראשון של הפסיכומכניקה.

    ר.ח.

    לא גמרנו את דוגמת ג’ק-ג’יל. אילו התפקידים היו הפוכים, ואתה היית במקומי, אני מאמין שהיית מאשים אותי בנפנוף ידים, ובנסיון לשינוי הנושא. מאז ומעולם טענתי שאם מפצגדול נכון, יש התארכות זמנים במערכות מואצות, אך שהיא איננה אמיתית. אם נפריד שני גזרים צעירים בינקותם ונכניס אחד מהם למקרר, הוא יזדקן לאט יותר מאחיו ועימו כל מערכת היחוס שלו: הירקות האחרים במקרר. אולם דבר זה אינו מהווה “פרדוקס תאומי הגזר”.

    לפי היחסות, שני תאומים שנפגשים הם בעלי זמן שונה, ואין לאף אחד עדיפות על השני. לא נוכל לסנכרן את שעוניהם ללא קשר ישיר בינהם, ואין דבר כזה “זמן אמיתי” שניתן לקביעה עיי שעוני טמפ’ או שעוני התרחקות גלקסיות. כל נושא התארכות הזמנים נובע מכך שאם ניצור קשר בין נקודה א’ לב’ באמצעות קרני אור במטרה לסנכרן את השעונים, הרי שאם הנקודות בתנועה יחסית, הזמנים משתנים בגלל קביעות מהירות האור.

    לכן ברור שאם ניתן לסנכרן את השעונים ללא קשר ישיר, כמו שהצעתי, התארכות הזמנים מעוקרת.

    אם תרצה להמשיך בדיון בשלב כלשהו בעתיד, אעמוד על כך שהוא ימשיך מהנקודה בה הפסקנו: הניסוי המחשבתי ג’-ג’. כל הפניה ל”עובדות” כביכול ו”הממסד המדעי” כביכול, אינן רלוונטיות, במיוחד בגלל שנראה לי שאתה חוזר ומחמיץ את הנקודה העיקרית: שאינני נגד היחסות, אלא מצביע על הסתירה בין 2 התאוריות. אישית, אני מאמין ביחסות והזמן היחסי שלה הרבה יותר מאשר במפץ הגדול והזמן המוחלט שמשתמע ממנו.

    יובל – דברי כיבושין ממיסים לבבות. אין לי בעיה לפתוח דף חדש. אני די מתגעגע לימים ששלושתינו, ר.ח, אתה ואני, היינו מסתלבטים על דמויות תנ”כיות מיתולוגיות ( תמר, איוב, בלק..)

    אבל דיר בלק מלהתחיל עם ישראל!

  42. ר.ח וישראל, חטאתי לכם.
    אני מבין היטב את מאבקו של ישראל ומזדהה איתו. אני מניח שעבור ר.ח מדובר בתרגול טוב (בחינם) של כישורי הביקורת המעולים שלו. מדי פעם שלחתי בכם חצים של לגלוג, אך אני חייב להודות כי למדתי מכם לא מעט והפקתי תועלת רבה מאד גם בעניין המודל הפרטי שלי. צר לי על המילים הקשות שהוטחו ביני לבין ישראל ועל ידידות טובה שהוחמצה, אך אני מתנחם בכך שמצאתי אדם הדומה לי מאד בדמיון פורה, באמביציה ובמזג חם. סביר מאד שגם ביחסי אנוש יש לי הרבה מה ללמוד אך לצערי דווקא בהפקת לקחים בתחום הזה אינני מוצלח כלל ועיקר.
    ישראל! אני מצטרף לאיחוליו של ר.ח, ולחלוטין לא בציניות. אתה מן הכותבים היותר מוכשרים שהתמזל לי להיות איתם בקשר, ואני בטוח שתזכה לכותרות ראשיות חיוביות, גם אם לאו דווקא בקשר לסטוקהולם.

  43. ישראל,

    OK גם מבחינתי אפשר לסכם כאן את הדיון. סיכום שלי:

    * לאחר כל הפוסטים וכל הטענות לא הצלחת לשכנע אותי שיש סתירה בין המפץ הגדול להתארכות הזמנים.

    * עיקר הטענות שלך נוגעות להתארכות הזמנים ומהווה את אחד הנדבכים הכבדים עליהם עומדת היחסות. התארכות הזמנים הוכחה במגוון נסיונות . אולם, אתה מבטל אותם בהינף יד כ “נסיונות שנעשו במערכות מואצות”. אני מניח שאם מישהו יראה לך ניסוי במערכת שאינה מואצת תטען “אלה נסיונות שנעשו לייד מרכז גרויטציה – כדור הארץ, שמש, גלקסיה)” לדעתי אלה הן התחמקויות ונסיונות להחייות תאוריה שהראיות נגדה. אני מכיר דוגמאות רבות דומות בביולוגיה. בחיפוש נפט קוראים לזה “חפירות באלכסון”.

    * היו כאן הרבה דיונים עם בריאתנים כמו אותו שינגואה שעל כל טענה המראה את האבולציה הם פוטרים בטענה דומה לשלך “אף אחד לא ראה מאקרו/מיקרו השתנות חלבונים ועוד כל מיני טענות מופרכות. ובסוף תמיד מסכמים בהתעלמות מופגנת מכל העובדות “אף אחד לא הוכיח לנו”. זה לא מדע ולא כל חוקרים. טענותיך לעניות דעתי אינן כה מופרכות כמו שלהם אולם גם אתה נוהג לפטור כל דבר שלא מסתדר לך כ “זה לא מן העניין, זה מערכת מואצת”.

    * לא הצלחת לשכנע שסביר שהאור נע במהירות אחרת מ- C ופשוט מכשירנו אינם קולטים חלקיקים מהירים. נתת אנלוגיה מפוקפקת למטוטלת בליסטית. לא הסברת מדוע לא ניתן לייצר גלאים טובים יותר? (באנלוגיה שלך מטוטלת שעשויה מפלדה בלתי עבירה לכדורים). לא הראית למה לא רואים אור איטי מ C. למעשה לא הראית שום ראיות לכך פרט לטענה שזה אולי אפשרי. מעבר לכך כשהראיתי לך שזה
    לא מה שקורה במאיצי חלקיקים פטרת את זה שוב בחפירות באלכסון בטענה שלא מדברים על חלקיקים עם מסה. האם ייתכן ש C נכון לחלקיקים עם מסה ודוקא לא לאור שממנו הוא נגזר?

    בכל מקרה אין האמור לעיל לנסות לרפות את ידיך. אדרבה תמשיך ותבורך ואני מאחל לך שתצליח וניפגש השטוקהולם.

  44. ישראל, לגבי מערכות פסיכומכניות סגורות או פתוחות, הן מתייחסות לאדם כאל גוף פיזיקלי ללא נפש או נשמה או מח שמסוגל לקבוע חוקים וליישם אותם. העולם הולך לכוון גולבליזצייה עם שלטון מרכזי שמשליט סדר במערכת כדור הארץ למרות כל תנועות ההתנגדות. כדי לשלוט על 7 מיליארד אנשים ולספק להם דלק ומזון ומיטה חמה צריך ארגון מרכזי חזק. תרמודינמיקה חברתית היא שוות ערך לאנרכיזם חברתי שמנסה להביא לאי סדר בכדור. הכח החיצוני שמביא לאיפוס כדור הארץ הוא אסון גלובאלי כגון מגה מטאור או פולס קרני גאמה של סופר נובה או מגה רעידת אדמה / הר געש שיביאו לתקופת קרח חדשה.
    לגבי תאורית המפץ הגדול היא התאוריה השלטת על ידי אסטרונומים על סמך ראיות טלסקופיות המצביעות על הפשטות היקום. ישנן תאוריות חדשות שהיקום בצורת ממברנה וישנם יקומים נוספים בצורות ממברנות ומידי פעם יש מגע בין ממברנה אחת לשני וזה גורם למפץ גדול וחוזר חלילה.
    אני טוען שבמפץ גדול חומר התפשט החוצה בצורה כדורית כמו בתאוריה השלטת אך גם התכווץ פנימה כמו בסופר נובה וגרם ליצירת חור שחור ענקי ומידי פעם מבעבע יקום נוסף כמו בקליפת בצל, שמישהו יפריך את התאוריה שלי….

  45. ר.ח.
    זה לא מה שאני אומר.
    בוא אומר לך מה בדיוק אני אומר, כדי שלא תהיינה אי הבנות:

    התארכות הזמנים ביחסות אינה מסתדרת עם תאוריית המפץ הגדול. זהו.

    להבנתי, כל הניסויים שהצעת, כולל מואונים ודופלר הם במערכות מואצות. התארכות הזמנים במערכות מואצות היא הגיונית, אם עקבת אחרי הדיון על מהות הזמן עם סטודנט.
    לא הראית לי שום ניסוי שמוכיח את תאוריית המפץ הגדול. אם אתה מכיר את ספרות הפיזיקה המחתרתית, יש הרבה התנגדות לתאוריה זו.

    בכל אופן, אני מניח שירדת מהנסיון להוכיח לי שיש סתירה בניסוי המחשבתי שהצעתי עם ג’ק וג’יל ושעוני הטמפ’.

    נוכל לסיים כאן, אלא אם כן תהיה מעוניין להמשיך. השלב הבא הוא שלב המצלמות.
    יהיה אשר יהיה, תודה על ההשקעה. אני יודע שזה די מתיש, אבל אני עדיין משוכנע בנכונות טענותי עד שיוכח לי אחרת.

  46. נו נו ישראל, באמת, אכן יש לך שיטה מדעית למהדרין.
    כל ממצא הנוגד את התאוריה שלך הופך מייד ללא רלוונטי.

    אומרים לך: התיזה שלך נוגדת את פרדוקס התאומים. אתה אומר “לא מדברים על מערכות מואצות”
    אומרים לך: התרחבות זמנים במערכות לא מואצות הוכחה ע”י אפקט דופלר. אתה אומר “לא מדברים על אפקט דופלר”.
    אומרים לך: המאיץ בסרן מראה לך שלא ניתן לעבור את מהירות האור ואין חלקיקים שנעלמים כי הגיעו למהירות גבוהה. אתה אומר “לא מדברים על חלקיקים עם מסה”

    בתור אחד שמתיימר לשנות את אחת מהתורות הפיזיקאיות הכי מבוססות שיש ולטעון שאתר כן קיים כנגד כל הקונצנזוס אתה צריך להראות משהו כללי ונרחב. אתה לא יכול לצמצם ולצמצם למקרה פרטי שבמקרה עוד לא נבדק ומנגד להגיד שכל דוגמא נגדית היא לא רלוונטית.

    הרי אם התאוריה שלך הייתה נכונה, היא הייתה נכונה גם במערכות מואצות וגם באפקט דופלר וגם בסרן וגם באפקט הפרפר באגן המזרחי של השקע הצפוני. בקיצור היא הייתה כללית ויפה.

    אני מקווה שלפחות את הבת הביוכימאית שלך חינכת למדע יותר טוב מזה.

  47. ישראל,

    בקיצור ואחרי כל הדיון הארכני הזה אפשר לפשט את הטענה שלך שאתה בעצם מאתגר את התארכות הזמנים. הטיעון שלך תקף להבנתי גם ללא שעוני הטמפרטורה. מה שאתה טוען, ותקן אותי אם אני טועה, שאם שני גופים נעים ללא תאוצה הם יראו את אותו זמן. טענה זו נמצאית בניגוד לטענת התארכות הזמנים של היחסות.

    אין לי לצערי מספיק ידע בפיזיקה, מה שאני יכול לעשות זה לחפש קצת. אז בחיפוש קצר מצאתי את האתר הבא:
    http://math.ucr.edu/home/baez/physics/Relativity/SR/experiments.html#Ives-Stilwell
    שמביא הוכחות ניסוייות לתורת היחסות. ולפי סעיף 4 יש הוכחות יפות להתארכות הזמנים ע”י אפקט דופלר (ושים לב שזה לא במערכות מאיצות).

  48. ר.ח.

    “הם ייראו דברים שונים.”

    בוא נבדוק את זה רגע, כי זו נקודה שהועלתה רבות עיי אחרים ובה טמונה הבעיה לדעתי.

    בוא נקצין כרגיל. יש לנו מכונית, שעל גגה שעון גדול ומצלמה חדת רזולוציה. מולה עוברת מכונית אחרת עם אותו התקן. כאשר הן חולפות זו על פני זו, מרעום קרבה גורם למצלמות לפעול, והן מצלמות את 2 השעונים ביחד.

    1. האם במהירות יחסית של 100 מ/שנ יש לך ספק ששתי המצלמות תראינה את אותו הצילום מבחינה זו שיהיה ניתן לדעת מה הזמן שמראה כל שעון?

    2. 1000 מ/שנ?

    3. 10,000, 100,000 וכך עד כמעט מהירות האור? באיזה שלב תראה כל מצלמה את ההפך מחברתה?

    מה גם שאין צורך להגיע כלל למהירויות גבוהות. תאורטית, עם המכוניות נוסעות זמן מספיק, הפרש הזמנים יכול להצטבר לכמה דקות גם במהירויות נמוכות. אתה מתאר לך מצב שבו 2 מכוניות חולפות זו על פני זו במהירות יחסית של 5 קמ”ש, המצלמות משקשקות, ואז כשמפתחים את הפילם מסתבר שהתמונות מראות תמונה הפוכה וכל צילום מראה את השעון האחר כאיטי?

    ובוא לא נלך רחוק. בדוגמת ג’ק ג’יל האורגינלית, זו שקראת בלינק, שניהם מסכימים ששעונה של ג’יל מפגר ב2 שניות. אין טענה מצד ג’יל שבעצם שעונו של ג’ק הוא זה שמפגר.

    הגד לי אם אתה עדיין חושב שכל מצלמה יכולה לצלם את השעון הנגדי כאיטי.

    ואל תתייחס לרפאים. התפקיד שלו זה לשבת במארב ולחכות לראות אולי יוכל להגיד איזה משהו נגדי. הראה לי תגובה אחת שלו בכל החודש האחרון שאינה קשורה בי. כל אחד והאובססיה שלו.

  49. ר.ח ורפאי.ם, בבקשה חדלו מן הירידות האלה שלא מוסיפות כבוד לאף אחד.
    ראו, למשל, מה קרה לי עם ישראל. כדאי תמיד לזכור כי לכל מקל יש שני קצוות.

  50. רונן ח.
    ” והיא יודעת גם על האתרים עם ריבוי ה- X שאתה מסתכל בהם?” – וואלה ? אם מדברים על טעויות פרוידיאניות.. (בטח רצית לכתוב משהו אחר נכון? D: )

  51. וואלה ? והיא יודעת גם על האתרים עם ריבוי ה- X שאתה מסתכל בהם?

  52. יובל
    אני מקווה שאתה צודק. אם כן, כולנו נרויח.

    רונן… אופסס סליחה, טעות פרוידיאנית. התכוונתי “ר.ח”:

    חד משמעית.

  53. מה קרה? קספר? חזרת מהאוב? או מיתחת לאפס המוחלט?
    אמא מרשה לשחק במחשב בשעות כאלה?

  54. ר.ח רפאים,
    נהפוך הוא. נראה לי שהוא יודע בדיוק על מה הוא מדבר. הבעיה היא שאף אחד אחר לא יודע. צריך לסלוח לכולם חוץ ממנו 😛

  55. ישראל

    אתה אומר : “אל תשכח שיש לנו מצלמות חדות רזולוציה בשתי החלליות. עם תמונות קשה להתווכח. מה שרואה ג’ק זה מה שרואה ג’יל.”

    לא שאני מבין איך זה יכול להיות אבל בפירוש לא! לאור מה שכתוב על התארכות הזמנים :

    When two observers are in relative uniform motion and uninfluenced by any gravitational mass, the point of view of each will be that the other’s (moving) clock is ticking at a slower rate than the local clock.

    הם ייראו דברים שונים. האמת שזה ניסוי לעשות! שתי חלליות עם שעונים על הדופן מצלמות זו את זו.

    דרך אגב אין קשר למיליארדית השנייה , לדעתי גם ממרחק זה יהיה אותו סיפור

  56. יובל.
    כצפוי, לא העלית ולו נקודה אחת תומכת בטענתך כאילו שהתחמקתי ממשהו או ניסיתי להטעות.
    אז אסכם את תגובתך האחרונה כניסיון פטתי נוסף להוריד את האנטרופיה שלך עיי כך שתגדיל את זו שלי.

    להבא, כשאתה מעלה מודלים או השמצות, נסה לגבות אותם במשהו נוסף חוץ ממידת הטמטום של קוראיך.

    בוא נסגור את זה כאן. באמת אין צורך שתענה. כל המוסיף – גורע.

  57. מכאל שפירא!
    האשמות, הכפשות, התנצלות… אתה בטוח שבחרת בקריירה הנכונה? את משפטו של החוק השני אתה סוקר במקצועיות שאין שניה לה. צר לי עליך אחי על כך שבמקום לעשות חייל בין כתלי בית המשפט אתה מבטל זמנך (וזמנם של אלה שלוקחים אותך ברצינות) לריק.
    אתה בחור טוב. מלח הארץ. מוכשר מאד. אינטליגנטי לעילא. אנשים כמוך צריכים מתישהו לעשות גם לביתם, ויפה שעה אחת קודם.

  58. ר.ח.
    כדאי אולי באמת שנעשה סדר.

    א. מסכים.
    ב. לא מסכים.
    ג. ומכאן הסתירה.

    נמשיך. “ג’יל האיצה בעבר” גם ג’ק. למערכת ג’ק – ג’יל ניתן להגיע באופן הבא: ג’ק וג’יל נמצאים ביחד גב אל גב בשדה הניסויים בטיזנבי. שניהם במנוחה. בשלב מסויים, שניהם מפעילים את המנועים ומתרחקים זה מזה. אחרי זמן של דקה לשניהם, שניהם מפסיקים את המנועים וממתינים שעה, עד שיווצר מרחק גדול בינהם. אז הם הופכים את כיוון הטיסה ומפעילים את המנועים שנית למשך 2 דקות, ואז מכבים.

    מה שקורה הוא, שכל אחד כעת נע יחסית לשני, ולפי היחסות, הזמן של כל אחד חולף לאט יותר יחסית לשני. אולם שים לב שהמערכת סימטרית לגמרי: אין דבר כזה ג’ק “נח” וג’יל “נעה” למרות שזה מה שנראה מנקודת הראות של כל אחד מהם.

    במצב זה קיבלנו את נתוני הבעיה. אתה הנחת שג’יל “נעה” אבל ג’יל מבחינתה נחה. וכך גם ג’ק. שניהם כרגע במנועים דוממים. שניהם נחים.

    נמשיך:

    אין מצב של א=ב=ג=ד בעולם.
    ( נניח : א ו ג טמפ’, ב’ ו ד’ צזיום)

    ג’ק רואה א=ב =ג ו ד איטי

    ג’יל רואה א = ג=ד וב’ איטי.

    אף אחד בעולם לא רואה א=ב=ג=ד.

    אל תשכח שיש לנו מצלמות חדות רזולוציה בשתי החלליות. עם תמונות קשה להתווכח. מה שרואה ג’ק זה מה שרואה ג’יל. אנחנו חייבים להסכים על נקודה זו לפני שנוכל להמשיך:

    האם אתה מסכים שברגע המפגש, במיליארדית השניה שג’ק וג’יל חולפים זה על פני זו, והמצלמות בשתי החלליות מצלמות את כל 4 השעונים, הן תראינה תמונות זהות? או שאתה מאמין שהתמונות תראינה זמנים שונים בשעונים. (לדוגמה: שמצלמתו של ג’ק תראה ששעוניה של ג’יל איטיים משלו, ושמצלמתה של ג’יל תראה ששעוניו של ג’ק איטיים משלה).

    נוכל להמשיך אם נגיע להסכמה בנקודה זו. (שאינה ברורה לגמרי, אגב. כזכור, זהו השלב שבו הובעה אי הסכמה עיי פיזיקאים מהשורה. זה בסדר, הצלחנו להתגבר עליה, אבל אני רוצה שנגיע להסכמה ביננו).

  59. יובל
    אתה האשמת האשמות והכפשת הכפשות. הראתי לך שבנקודות אותן העלית, או שלא טרחת לקרוא עד הסוף או שפשוט לא הבנת.
    מן הראוי שתעשה אחת מהשתיים:
    1. תראה לי מקומות אחרים בהם כדבריך אני מתחמק, או שתסביר מדוע לדעתך התחמקתי במקומות בהם העלית.
    2. תתנצל ותחדל.

    ואם אני מוציא לך את החשק לדבר איתי – מדוע אתה פשוט לא עושה זאת ומתעלם לגמרי מתגובותי?

    רובי.

    תגובה ממתינה, שתשפוך אני מקווה אור על מה שכתבתי.

  60. תודה רובי,
    התאוצה קדימה והתאוטה בדרך חזרה הן הפתרון המקובל לפרדוקס התאומים. אך נדמה לי שישראל מנסה לטעון משהו לגבי מצבים אחרים אשר עד כה אף אחד כאן לא הצליח להבין. כמו אינטלקטואל יהודי מן השורה הוא חותר בלא לאות להצגת תמונת העולם שלו, ובדרך מושך אל עצמו הרבה אש.
    מאז גלות בבל, ולמעשה עוד קודם לכן, היהודים תמיד היוו מיעוט בעולם. האוכלוסייה האנושית מתנהלת ממלחמה למלחמה, ובמלחמות מי שנפגע הכי הרבה אלה המיעוטים. השאלה איננה מהו הגורם להתנכלות העולם אלא מהו סוד הישרדותו של המיעוט המסוים הזה.

  61. כותרות העיתונים, שבישרו על פתיחת המשפט, דברו על “משפט כל הזמנים”.
    רוב הכותבים המטירו אש וגופרית על החוק השני תוך כדי שימוש בביטויים חריפים ביותר. “שחצן” “משחית” “אכזר” היו בין המתונים שבביטויים. עיתון אחד כינה אותו בפשטות “מפלצתי” והוסיף: “במדינה דמוקרטית העם הוא זה שקובע את החוקים. אינני רואה מדוע עלינו להמשיך לשלם מסים ולממן מערכת שהחוק היסודי ביותר שלה קורא להתפרקות מכל סדר ומשמעת”.
    ארגונים שמרניים דרשו להוציא מיד את החוק השני מכל ספרי הלימוד. הם הרבו לצטט מדבריו של פרופסור בולצמנסקי, ובמיוחד יצא קצפם על הקביעה ש”אין מערכת יכולה להקטין את האנטרופיה של עצמה אלא אם כן תגדיל את האנטרופיה של מערכות אחרות” שאותה הגדירו כ “דרוויניזם מוחלט!”. האם זו דמותו של העולם שבו רוצים אנו לגדל את ילדינו?” שאל דוברם בחמה שפוכה, והוסיף: “זה יותר גרוע מהאבולוציה”.
    פה ושם נשמעו קולות מהוססים שגרסו כי החוק הנו רע הכרחי, ושיש אמת בטענה שהוא בעצם מקור החיים. ללא החוק השני, טענו, כיצד הייתה מגיעה אלינו קרינת השמש? ואיך היו פועלות כל המכונות? ומנגנון החיים עצמו?
    קולות אילו הושתקו במהרה עיי מדענים נרגזים שהסבירו באריכות כי ניתן בקלות לארגן חוק חיובי יותר, שישמור על הסדר הקוסמי ללא הצורך בהתפרקות מתמדת.
    פרגמטיים יותר היו אלו שטענו כי בין אם יורשע או יזוכה, החוק הנו עובדה קיימת, ועלינו להתאים את חיינו אליו ולא לנסות לשנותו. כשם שגילוי החוק עיי קלאוזיוס במאה התשע עשרה גרם למהפכה במדע התרמודינמיקה ובפיזיקה בכלל, שומה עלינו לקבל את מציאותו של החוק בחיי היומיום ולפעול בהתאם.
    כלכלנים דברו על “החוק השני בכלכלה” – כלום אין זרימת ההון והנכסים בעולם מקבילה לזרימת החום במערכת תרמודינמית? האם אין חוקי שימור לעושר ועוני בעולם הדומים במהותם לחוק הראשון של התרמודינמיקה ומסבירים מדוע שגשוג באזורים גיאוגרפיים מסוימים כרוך תמיד בשפל באזורים אחרים? כלום אין מושגי ה”עושר” וה”עוני” עצמם יחסיים ורק ההפרש בין “עשיר” ל”עשיר יותר” יקבע את כוון זרימת ההון כמו שבמצב צבירה נתון רק הפרש הטמפרטורות הוא שקובע מהו “חם” ומהו “קר” ובאיזו כיוון תזרום האנרגיה?
    ואם חוקי הכלכלה דומים במהותם לחוקי התרמודינמיקה – האם אין זה מן הראוי להכיר בקיומה של “אנטרופיה כלכלית” – ההכרה בכך שכל שיפור במערכת כלכלית אחת יגרום בהכרח להרעה במערכת אחרת?
    סוציולוגים דיברו על “אנטרופיה חברתית”. ביולוגים על “אנטרופיה של מערכות חיסוניות” דיאטתיקנים הצביעו על העובדה הידועה שלמרות כל המאמץ המושקע בדיאטות ומכוני כושר, הולכת האוכלוסייה ומשמינה וגם על כך שרובן המכריע של הדיאטות מסתיים בחזרה למשקל המקורי עם קצת תוספת, בדיוק כפי שקורה לטמפרטורה במערכת תרמודינמית סגורה ופעילה.
    מעניינת במיוחד הייתה קבוצה שכינתה את עצמה “חבורת הפסיכומכניקה”. מייסדה, שהיה ידוע רק בכינויו “האיש של הפסיכומכניקה”, טען שמערכות אנושיות הכוללות מספר גדול של פרטים, מתנהגות בקירוב כמערכות תרמודינמיות. כשם שאין אפשרות מעשית לחשב את התנהגותה של מולקולה בודדת אלא רק את התנהגותן של מליונים רבים של מולקולות, כך אין הפסיכומכניקה עוסקת באנשים בודדים אלא בצבורים גדולים כגון מדינות או יבשות.
    הקבוצה, שלטענתה הסתייעה במכשור החדיש ביותר ונעזרה בשיטות הסטטיסטיות המתקדמות ביותר, מומנה עיי מקור פרטי עלום שם ומשכה אליה מוחות שנחשבו למבריקים במיוחד. סמלה היה הולוגרמה של כדור, עם האות הקטנה i במרכזו. השאלה המרכזית שניצבה בפניה הייתה: אם, כפי שנטען, מתנהגות מערכות אנושיות בקירוב כמערכות תרמודינמיות, האין זה מחייב קיומו של עקרון דומה לחוק האנטרופיה לחברות אנושיות? תשובה מסמרת שיער: הסתברות ששואפת לאחד ככל שהמערכת גדולה ואקראית יותר, או בפשטות: כן. ומכאן השאלה הגורלית: האם כדור הארץ, שהינו מערכת פסיכומכנית סגורה, יכול להקטין את האנטרופיה הפסיכומכנית הכוללת של עצמו? תשובה מרפת ידיים ומייאשת: לא בכוחות עצמו. ה”חוק השני של הפסיכומכניקה” שולל את האפשרות שהאנטרופיה במערכת פסיכומכנית סגורה תקטן, אלא אם כן תגדל האנטרופיה של מערכת אחרת.
    אפשר לאמור שלום לשלום ואחווה עולמיים. כל רווחה ושפע בקבוצה אחת יבואו תמיד על חשבונה של קבוצה אחרת. שעתו של מזל דלי לא תגיע בכוחות עצמה, ולא יעזור אם יזרח הירח בבית השביעי וצדק ומאדים יתייצבו כאיש אחד בשורה. ללא מקור חיצוני של אנרגיה פסיכומכנית, נידונה כל מערכת סגורה, בין אם מדינה, אמונה דתית או מעמד כלכלי, והנתונה בלחץ פסיכומכני, לנטייה מתמדת להתפשטות כדרך להקטנת האנטרופיה המקומית של עצמה – וזאת בהעדר מרחב פסיכומכני פנוי, על חשבון מערכת פסיכומכנית אחרת.

  62. יובל.
    הולך ומתחזק אצלי הרושם שמסיבה כלשהי, אולי הדיסלקציה, אתה כנראה קורא רק חצי משפט ומתעלם מן היתר.

    אז הנה מה שכתבתי בנוגע לשורש i. בבקשה לקרוא עד הסוף.

    “ובעניין ה1-. מה עם i, שורש של 1-? האם גם לו יש שורש?

    ובכלל, מה ההשלכות הפילוסופיות לגבי האמונה ב L1? האם נוכל אולי גם להאמין באל מינוס אחד? זה מאד הגיוני לדעתי, כי בעבר האמנו באלים רבים, צמצמנו לאל אחד, האתאיסטים מאמינים באפס, האם אין זה השלב הלוגי המתבקש, דוקי?”

    שאלה: האם אתה מתכוון ברצינות לענות לשאלות:
    “ובכלל, מה ההשלכות הפילוסופיות לגבי האמונה ב L1? האם נוכל אולי גם להאמין באל מינוס אחד?”

    כלום אינך רואה שכל הענין נאמר בבדיחותא? שלא התכוונתי בכלל שמישהו יתייחס לכך? שכל העניין היה “כאילו” המשך הגיוני של שאלות “מהותיות” בנוגע לתפקידו הרה הגורל של ה1- בחיינו?

    בוא נמשיך. “מהירות האור היא פשוט יחסית למקור”.

    התגובה הראשונה שלי בנושא הייתה:

    “אפשר בכלל לוותר גם על נושא הפייט, ולגשת ישר למדע. הסברתי לך את הבעייתיות שקיימת בפירוש שלך לניסוי מ-מ. (אני מאמין שהתכוונת לפירוש של לורנץ, התכווצות).

    שאלה: למה שלא תיקח את הפירוש הפשוט עוד יותר, שמהירות האור היא פשוט יחסית למקור האור? זה יסביר את תוצאות ניסוי מ-מ בקלות, לא?”

    והיא באה אחרי שעיינתי במודל שלך. מכיוון שלא ניתן להסיק לדעתי מן המודל את השאלה המרכזית שהיא: מדוע מהירות האור זהה לכל מודד? אלא רק את השאלה לעיל, שהיא משהו שונה, שאלתי אותך את אותה שאלה.

    אתה בקשת שאסביר, וזה מה שעשיתי. EMISSION THEORY מסביר יפה את תוצאות ניסוי מ-מ. (האם לזאת התכוונת? לא ברור לך למה התאוריה מסבירה את מ-מ? והאם זה לא כתוב במפורש בלינק אחרי שהצטרכת לגלגל את המושג? ומה הזעקה הגדולה על כך שלא צרפתי לך את הלינק המפורש. כלום אינך יודע שלינקים כאלו משהים את התגובות עד שהן מתיישנות?) אתה המשכת לחתור למה שנראה לך ( כפי שהבנתי ) ברור לגמרי שאליו התכוונתי: שאסביר מדוע מהירות האור זהה לכל מודד? אולם לא זה מה שאמרתי. עבור על השירשור.

    אותו הדבר כאשר אמרת שהזמן והחוק השני הם אותו הדבר. זה כמו להגיד שחור ועששת הם אותו הדבר. זה נכון למי שהוא רופא שיניים, אך לא נכון מושגית.

    ודעתך לגבי ה”הרחבות” שלי על נושא המזה”ת היא בדיוק זאת – דעתך. אין לה שום עדיפות בעיני על דעתי, או דעתו של מישהו אחר. כרגיל, אתה משמיע משפט כללי בלי להתייחס כלל לתוכן הדברים.

  63. ישראל,

    עכשיו אני ממש מבולבל. בוא נעשה rewind וניזכר בויכוח ביננו.
    להזכירך, כל הסיפור הארוך והמתיש התחיל מזה שאתה טענת שחלליות שמאיצות ומשתוללות יכולות לתאם התקפה על סמך שעוני הטמפרטורה (מה שהסכמתי לו) ואז טענת שזה מביא לסתירה עם המפץ הגדול (שאני טענתי שטין שום סתירה)
    א. מסכים ?
    מזה הגענו לג’ק וג’יל כדי לפשט את הסיפור. אם ג’יל מאיצה,
    ב. האם אתה מסכים שיהיה הבדל בשעוניה?
    ג. אם כן האם תהיה סתירה ליחסות או למפץ?

    אם ג’יל לא מאיצה אלא סתם נוסעת מהר במנוחה (ברור שפעם היא האיצה כדי להגיע לזה אבל נעזוב את הדקויות האלה) אזי אתה צודק שני השעונים יראו אותו דבר והזמן של ג’ק יעבור יותר לאט בעינה בעוד שהזמן שלה יעבור יותר לאט בעיני ג’ק ואז כביכול נוצרת סתירה במה שאתה מכנה א=ב =ג =ד אולם אין כאן סתירה אלא שזהו עיקרון התארכות הזמנים.

    אין מצב של א=ב=ג=ד בעולם.
    ( נניח : א ו ג טמפ’, ב’ ו ד’ צזיום)

    ג’ק רואה א=ב =ג ו ד איטי

    ג’יל רואה א = ג=ד וב’ איטי.

    אף אחד בעולם לא רואה א=ב=ג=ד.

    זו הרי מהותה של התארכות הזמנים. בדיוק כמו בויקיפדיה שממנה ציטטת :
    The point of view of the other observer will be that again the local clock (this time the other clock) is correct and it is the distant moving one that is slow. From a local perspective, time registered by clocks that are at rest with respect to the local frame of reference (and far from any gravitational mass) always appears to pass at the same rate

    When two observers are in relative uniform motion and uninfluenced by any gravitational mass, the point of view of each will be that the other’s (moving) clock is ticking at a slower rate than the local clock. The faster the relative velocity, the greater the magnitude of time dilation

    אז מה הסתירה?

  64. אני מסכים עם יובל, האנלוגיה של ישראל מול הערבים כמערכת תרמודינמית סגורה פשטנית מידי.
    הייתי משווה את מדינת ישראל יותר לאי קטן בים סוער ושוצף שמנסה לכסות אותו בגליו עם הצלחות זמניות.
    המיקום האסטרטגי של המדינה וזכות הראשונים שיש לעם היהודי באמונה באל אחד עם כל המשתמע בכך מרגיז הרבה אנשים, מוסלמים בהווה ונוצרים בעבר בעיקר.
    לגבי פרדוקס התאומים, ברור שאחד מהם נח או במהירות קבועה ואילו השני מאיץ ובחזרתו מאיט לפגישה עם אחיו הזקן יותר.

  65. ישראל,
    ביקשת, אז קבל (מזכיר לי את ראשי התיבות בז”ק. מעניין למה 😀 ):
    א) שאלת מהו השורש של i. הראיתי לך דרך לחישובו ולא קיבלתי שום תודה או, להבדיל, שום בעיטה, פשוט התעלמות טוטאלית. ביקשתי ממך להתייחס ואתה לא הגבת. ביקשתי ממך עוד פעמיים, ואתה פשוט התעלמת.
    ב) אמרת/שאלת/טענת: “למה שלא תיקח את הפירוש הפשוט עוד יותר, שמהירות האור היא פשוט יחסית למקור האור? זה יסביר את תוצאות ניסוי מ-מ בקלות, לא?”. ביקשתי ממך להוכיח והתחמקת ממושכות עד שאחרי הרבה זמן הבאת משהו מגומגם שאינו לעניין.
    ואלה רק דוגמאות.
    ומה שעשית כעת, כשבחרת להציג את הפוליטיקה של המזרח התיכון כ”תרמודינמיקה”, הוא חזרה על הגישה שאתה נוקט לכל אורך הדיונים שאתה מנהל, דהינו הכללה רחבה של תופעה תוך הזנחת חשיבותם של פרטים מהותיים. אמנם נכון ש”תמיד” (כאילו המדינה התקיימה כבר בימי המפץ הגדול) היו מלחמותיה של מדינת ישראל רק נגד מוסלמים, ולא חשוב היכן ישב היורה התורן, אך אלה הינן רק דוגמאות בודדות לתופעה רחבה הרבה יותר שהחלה זמן רב לפני קום המדינה וכללה צרים עוד מן התקופה בה העולם טרם הכיר את האסלאם של מוחמד ואפילו לא את הדת המשיחית.

  66. רובי,

    “תחשבו חיובי”? אם ככה אתה כשאתה חושב חיובי (סעיפים 1-4) אז מה קורה כשאתה חושב שלילית? אפוקליפסה? יום הדין ? גוג ומגוג?

  67. ר.ח.
    ” איך ג=ד איך?
    הרי אתה במו מיקלדתך כתבת מספר פעמים שיחס שעוניה של ג’ל הוא 1:1000,000 ?

    אז איך הם נהיו שווים פתאום ?

    לדעתי כל סיפור ג’ק – ג’ל שלך הוא פשוט גירסא של פרדוקס התאומים המפורסם.
    בפרדוקס זה כן אחד התאומים מזדקן והשני לא”

    הפעמים היחידות שבהן כתבתי שג=ד, שיחס שעוניה של ג’יל שונה מ1:1 היו כדי להראות שהנחה כזו מביאה אותנו לסתירה.

    פעם נוספת: אין שום הבדל בין המערכות של ג’ק וג’יל. אם אצל ג’ק היחס הוא 1:1 אז גם אצל ג’יל. כל הנחה אחרת מביאה לסתירה.

    יתכן שאתה מניח שהמערכת של ג’ק “נחה” באמת, ושל ג’יל “נעה באמת. אולם אין לכך כל תימוכין בנתוני הבעיה המקורית. כל הגיון שתפעיל כדי להראות שיחס שעוניה של ג’יל שונה מ1:1, תקף לגמרי לגבי המערכת של ג’ק.

    ולגבי פרדוקס התאומים – זו כידוע מערכת מואצת, ששונה תכלית השינוי ממערכת אינרציאלית. גם כל האישושים שמצאתי להתארכות הזמנים בנסויים ( ניסוי המטוסים, מואונים, צרן) עוסקים רק במערכות מואצות. אשמח לשמוע על ניסויים במערכות אינרציאליות שבהם בא לביטוי נושא התארכות הזמנים.

    יובל.

    “תכונת החמקנות שייחסתי לך.”

    מכיוון שאני מאמין שאני עונה על כל שאלה בנושאים אותם העלתי בפרטי פרטים מפורטים, אשמח אם תאיר את עיני על אותם המקרים בהם התחמקתי, הצתלפחתי, פיזרתי מסכי עשן ויתר ביטויים ססגוניים.

    אם תבקש, אעשה זאת בשמחה לגבי תגובותיך שלך.

    אם לעומת זאת, אחרי שעברת על השירשור, תיווכח שלא תמצא אפילו בדל ראיה להאשמות חסרות האחריות שבהן אתה מאשים אותי חדשות לבקרים, מן הראוי שתתנצל ותחדל.

    רובי.

    לגבי איראן, בוא נראה אם מה שלמדנו בכתבה זו יכול לעזור לנו לראות פנים נוספים של הנושא:

    המצב הפוליטי נכון ל למרץ 2012:

    גבולות חמים:

    לבנון
    עזה.

    גבולות שקטים:

    מצרים.
    ירדן.
    סוריה.

    אויבים בפועל: ( כאלו שיורים)

    פלסטינאים.
    לבנונים.

    אויבים בכוח (כאלו שמאיימים)

    איראן
    תורכיה.

    יושבים בשקט יחסי:
    מצרים. (הסכם שלום)
    ירדנים. (הסכם שלום).
    סורים. (גבול שקט שנים ארוכות).

    המצב לפני 40 שנה:

    גבולות חמים:
    מצרים.
    ירדן.
    סוריה.

    גבולות שקטים:
    לבנון.
    עזה.

    אויבים בפועל (כאלה שיורים)

    מצרים.
    ירדן.
    סוריה.

    יושבים בשקט יחסי:
    פלסטינאים.
    לבנונים.

    ידידות שהן כמעט בעלות ברית:
    איראן.
    תורכיה.

    אז מה נשתנה?

    פשוט, כל האויבים והפחות אויבים התחלפו בשמות ובתפקידים.

    מסקנה:

    תמיד יהיה איזה ערבי או מוסלמי שיורה עלינו. פעם קראו לו קאוקג’י, אחר כך הוא החליף את שמו לעבדללה, אח”כ לנאצר, סאדאת, ערפאת, נסראללה, יאסין, עכשיו הניה, ומחר אחמדניג’אד. אבל בשורה התחתונה, מתחת לכל ערימת הכפיות, זה אותו ערבי (או מוסלמי).

    וכל כך למה?
    כאן באה לעזרתינו כתבה זו. יש פיזיקה, ולפיזיקה יש חוקים, ובפיזיקה יש את ענף התרמודינמיקה, ולתרמודינמיקה יש את החוק השני, והחוק השני אומר שבמערכת סגורה האנטרופיה אינה יכולה לרדת לבד, ושאנחנו והמוסלמים בסביבה זו מערכת די סגורה.

    ולכן אם תעשה שלום עם מצרים – לבנון תירה עליך. ואם תכה את לבנון – עזה תירה עליך. ואם תכה את לבנון ועזה – אז איראן, שמעולם לא עוללת לה כל רע, ושלא גזלת את אדמתה, ושאין לך דבר נגדה, ושבשמחה תהיה חבר שלה, (כמו שתהיה גם עם שאר הערבים והמוסלמים) –
    היא פתאום זו שתתנפל עליך.

    פשוט מאוד: תרמודינמיקה.

  68. איראן תתקוף בישראל בכל מקרה של תקיפה אמריקאית משולבת או לא.
    העוצמה תהיה שונה וגם הטיעונים שלה לתקיפה בישראל לא יהיו תקפים למרות שזה לא ישנה לנו.
    האיראנים צריכים לחשוש ממצב שישראל “תשתגע” כתוצאה מנזק “לא סביר” שיגרם לה.

  69. תודה רובי, אף אני חש כמוך. אך אני גם חושש שתקיפה בידי ארה”ב או נאט”ו תיתן לאירן עילה מספקת לתקוף את ישראל אפילו אם זו לא תתערב.

  70. לגבי איראן:
    1. איראן יכולה לקנות פצצות גרעיניות מצפון קוריאה ולהכניסם לאיראן בעזרת צוללת, הובלה אוירית או יבשתית אחרת. הטיעון של הסכנה באחזקת פצצה לא תופס כאן.
    2. היכולת לפתח פצצה כבר קיימת, שום הפגזה לא תחסל סופית את היכולת רק אולי תעכב.
    3. המטרה של ישראל והמערב היא לחץ ונידוי איראן ולחולל תהליך של החלפת השלטון
    4. תרגעו, ישראל לא תפתח בהפגזה ראשונה ותתן לארה”ב ונאטו לבצע זאת
    בניגוד להפגזה בעירק שהיתה יחסית מבודדת, איראן בעלת ברית של חזבאללה וחמאס ואחרים כך שהתגובה לישראל תהיה מאוד כואבת.
    תחשבו חיובי

  71. זה מה שאני מנסה לעשות עם ישראל אבל זה לא הולך.
    אולי צריך לטוס למאדים בשביל זה

  72. ר.ח, אנא קבל את התנצלותי. לא התכוונתי לפשפש בנטיותיך הפוליטיות. פשוט, היות שהסכמנו כי מדינת ישראל לא תוכל לפעול ללא ידידתה הגדולה ומתקבל הרושם כי הנשיא הנוכחי לא ממהר לשום מקום…

  73. ר.ח, הסיפא של דבריך מקובל עלי. דהינו, אם ישראל מעוניינת במתקפה צבאית היא לא תוכל לפעול לבדה. כל השאר ספקולציות. האם נתת קולך למיט רומני?

  74. ישראל! שוב אתה ממציא הקשרים במקום שאין.
    לא על תקפותה של תורת היחסות ערערתי אלא על תקפות המסקנות שאתה ור.ח עשויים להגיע אליהן באין לכם דרך לבחון אותן הלכה למעשה.
    “מראה לך מאיפה אתה משתין” זו הרכבה של ביטוי ידוע עם תכונת החמקנות שייחסתי לך. כבר התרגלתי לזה שלא מבינים אותי, וזה כולל גם את נסיונות ההתבדחות שלי.
    על הערתך הנוגעת לנערות בברים של גלזגו אינני יכול להגיב, היות שאני לא מבקר במקומות ההם. ואת הערתו של נקודה אפשר להחיל בקלות גם על מה שאתה מנהל כאן.
    שוב אתה מצליח לגרור אותי לויכוחים מטופשים. איך אתה עושה את זה? מה הטריק שלך? ספר נא, ספר.

  75. יובל,

    אז איך תמנע מהם להשיג פצצה ללא תקיפה? אולי נבקש נורא נורא נורא יפה? אולי נשלח את בר רפאלי ? או את קטלין רייטר שתשיר להם?

    האם אנחנו יכולים לבנות או לסמוך על האופוזיציה הקיקיונית שתפיל את המשטר לפני יום הדין? האם יש איזה צעד דיפלומטי או סנקציות שישכנעו אותם?

    זוכר 1981 כור תמוז? אז גם עמד שמעון פרס ואמר אוי ואבוי יהיה אם נתקוף. תקפנו וזה היה מזל כי אחרת ב 1981 אולי היינו חוטפים סקאד אטומי במקום אלה שנפלו עם ראש בטון.

    מה שכן אני לא חושב שישראל מסוגלת להרוס לבדה את תוכנית האטום האיראנית והתקיפה חייבת להיות בינלאומית או לפחות אמריקאית.

  76. ר.ח,
    אני מניח שאם אירן תוכל להטיל פצצת אטום על תל אביב היא לא תדרוש נסיגה לגבולות 1948 אלא להחזרת כל היהודים לארצות מוצאם (ועדיף אף להשמידם, כהמלצת הקוראן) וחיסול מדינת ישראל והמשטר הציוני בכלל.
    יש לנקוט כל אמצעי כדי למנוע בעד אירן להשיג את הפצצה, אך מלחמה לא תועיל אלא רק תזיק. אם ישראל תתקוף את אירן, יקבל המשטר האירני לגיטימציה מלאה גם בעיני האופוזיציה האירנית להשיג את הפצצה. לדעתי, תקיפת אירן רק תעלה ביוקר ולא תועיל כלל.

  77. יובל
    ועשה זאת באמצעות ניסויים מחשבתיים – שעל תקפותם אתה מערער באומרך: “זה ויכוח סרק משום שהוא עוסק בניסויי חשיבה שאין בידינו אמצעים לאששם או להפריכם.”

    ואת דבריך: “את “נגד רצונך” אתה הסקת מתוך גישה בלתי אמפיריציסטית לחלוטין (וגם בלתי רציונליסטית, אגב).”

    ניתן להסיק מדברים שאמרת קודם כמו: “מראה לך בסבלנות אין קץ שחסרה לי מאיפה אתה משתין.”

    אז מספיק לשחק אותה ילד קטן. נקודה כבר העיר לך בעבר שהמודלים שלך מבוססים בעיקר על מידת הבורות של קוראיך. זה כולל גם מודלי תוקפנות. וכמו יתר המודלים, זה לא עובר את סף הנערות בברים של גלזגו. עליהן זה יכול לעשות רושם, לא עלינו.

  78. יובל,

    ומה תעשה בתור ראש ממשלה ביום שהאירנים יציגו פצצת אטום ויכריזו שאם עד סוף החודש הבא ישראל לא חוזרת לגבולות 1948 הם יטילו אותה על ת”א? מה אז? נשחק פוקר ונראה אם הם מבלפים או שהם רציניים?
    אפשר לקחת הימור כזה? ואם ניכנע? האם בזה ייסתיימו הדרישות ?

    מול סחטן יש רק דרך אחת.

  79. לא הבנתי את הקשר לפצצה האיראנית. אך היות ששאלת, הרי לך דעתי: פצצה גרעינית, אחרי הירושימה ונגסקי, היא מכשיר הרתעה המקנה למחזיק בו יוקרה וכבוד בקרב האומות. כשלאירן תהיה כזאת יהיה לה מרחב פעולה מדיני גדול יותר משיש לה היום.
    האנושות מתנהלת ממלחמה למלחמה. כמעט כל מדינה זקוקה למלחמה מדי פעם, ואירן אינה יוצאת דופן. למדינת ישראל, לעומת זאת, אין מה להרויח ממלחמה. יש לה רק הרבה מאד מה להפסיד.
    המלחמה עם אירן לא תהיה גרעינית, אך האיראנים יודעים הרבה תכסיסים ואסור לישראל לזלזל בהם.
    הם מחכים שישראל תתקוף כדי שיהיה להם תירוץ להכות את ישראל בלי שתואשם כתוקפת. הם מגרים את ישראל לתקוף. הם עושים זאת בהכרזות מאיימות, בפרובוקציות כדוגמת ההתקפות החובבניות על נציגויות ישראליות בעולם, בתמיכה במשטרים שונאי ישראל, באימון יחידות צבאיות במדינות האויבות את ישראל, ועוד. כעת הם אפילו מציגים את ישראל ככזאת שבוודאות זוממת לתקוף ראשונה ובכך מכינים את הרקע להצדקת מלחמה ביוזמתם. זכור כיצד פרצה מלחמת לבנון השניה: חזבאללה, בן טיפוחם של האיראנים, יצר פרובוקציה כדי לגרור את מדינת ישראל לפעולת עונשין קטנה שהתפתחה למלחמה שבמהלכה התברר כי הוא מאומן היטב ומצויד כהלכה בנשק מן השורה הראשונה. אני זוכר בונקר צהלי נטוש בלבנון שחיילי חזבאללה הסתתרו בו והמתינו בשקט הרבה שבועות וצה”ל בכלל לא חשד.
    אם לדעתי תשאל, אז אסור למדינת ישראל לתת לאיראנים שמץ של תירוץ ליזום מלחמה. היא צריכה לצאת בהצהרה פומבית החלטית גורפת כי היא מכבדת את העם האירני ומאמינה שהוא שוחר שלום וכי היא בשום פנים ואופן לא תתקוף אותו. במקביל, עליה לחשוף לעיני העולם את כל ההוכחות שיש בידי המודיעין הישראלי (ובטוח יש הרבה) על כוונותיו התוקפניות של המשטר הנוכחי באירן לא רק כנגד ישראל אלא גם נגד מדינות אחרות, ולקדם הרחבת הסנקציות הכלכליות הבינלאומיות.

  80. ישראל,

    1) איך ג=ד איך?
    הרי אתה במו מיקלדתך כתבת מספר פעמים שיחס שעוניה של ג’ל הוא 1:1000,000 ?

    אז איך הם נהיו שווים פתאום ?

    2) לדעתי כל סיפור ג’ק – ג’ל שלך הוא פשוט גירסא של פרדוקס התאומים המפורסם.
    בפרדוקס זה כן אחד התאומים מזדקן והשני לא.
    הפרדוקס לא מוגבל רק לתאוצה.
    פרדוקס זה נחקר ונלמד בצורה מסיבית ונשפכו עליו מליוני מילים וההסכמה היא שהוא לא פרדוקס ואין סתירה עם הרחבת הזמנים. יתירה מכך הוא הוכח ניסויית גם במטוסים וגם בחלקיקים במאיץ. כך שגם סיפור ג’ק ג’יל לא מהווה סתירה בין היחסות למפץ ואין כאן שום פרדוקס.

  81. לאיש.
    לך לאתר
    http://hyperphysics.phy-astr.gsu.edu/hbase/astro/expand.html#c3
    יש שם למטה נוסחה – נוסחת פרידמן לתלות הזמן בטמפרטורה.
    יש שם גם מחשבון, שתוכל באמצעותו להפוך זמן לטמפ’ ולהיפך.

    אילו לא היה קשר ישיר בין הזמנים שמופקים באמצעות הטמפ’ והזמן הרגיל היומיומי, לא היינו יכולים לדבר על יקום בן 13.7 מיליארד שנים, כי זה היה מתפרש כדבר שונה לכל אחד. לכן שנת טמפרטורה שווה לשנה רגילה שלנו.

  82. ר.ח.
    זה היה קיצור.
    המשפט המקורי היה: אם א=ב, ג=ד וא=ג ,אז א=ב=ג=ד.
    א – שעון טמפ’ ג’ק.
    ב – שעון צז’ ג’ק.

    ג – שעון טמפי ג’יל.
    ד – שעון צז’ ג’יל.

    א’ וב’ תמיד מראים אותו זמן (מערכת אינירציאלית).
    ג’ וד’ תמיד מראים אותו זמן (מערכת אינירציאלית).

    א’ וג’ מראים את אותו הזמן בעת המפגש ( שעוני טמפ’).
    ולכן: א=ב=ג=ד.

    ואין התארכות זמנים.

    מ.ש.ל.

    מה שכתבתי בלינק שהוספת היה:

    “הבעיה היא זו:

    1. גם ג’ק וגם ג’יל נמצאים מבחינתם במנוחה. המערכות שלהם אינן מואצות, ונקראות בשפת היחסות “אינרציאליות”. אילו היה ניתן להבדיל בינהן עיי פעולה של ספירת סיבובים לדוגמה, זה היה סותר את הפוסטולט הראשון של היחסות.”

    וזו הוכחה על דרך השלילה שמצב כזה לא יכול להתקיים.
    לכן היחס בין שעוניה של ג’יל חייב להיות זהה ליחס בין שעוניו של ג’ק.

    אם לא התמצית עדיין, אפשר לעבור לכתבה אחרת מלפני שנתיים, כך שהטעינה תהיה מהירה.

    יובל – איינשטיין הגה את כל היחסות בלי לצאת מהמשרד, ניסויי חשיבה עלק. האם זה אומר שאפשר להפסיק לחשוש מהפצצה האיראנית?

  83. אישור אישור אישור

    הדיון הזה הופך למעיק יותר ויותר. גם זמן הטעינה של הדף וגם החסימה הקבועה של התגובות.

    אבי בליזובסקי,

    אני מציע שתעשה רויזיה של מילות החסימה שלך. אין בהן שום הגיון. ראה תגובה אחרונה שלי. אתה גורם כך לעיקור של הדיונים.

  84. ישראל,

    איך א=ב וג=ד אז א=ב=ג=ד ?

    שכחת מה כתבת ב:
    https://www.hayadan.org.il/astronomers-reach-new-frontiers-of-dark-matter-130112/#comment-331692

    “. אם ג’ק סופר משווה את מס הסיבובים בין 2 השעונים (ואפשר בהחלט לסדר ששעוני הצזיום והטמפ’ יראה זמן בסיבובי מחוגים) הוא יראה יחס של 1:1. הוא איננו צריך למעשה לחכות כל הזמן עד למפגש. בכל פרק זמן נתון שהוא בודק, היחס יהיה זהה.

    2.אם ג’יל סופרת, היא תקבל יחס של נגיד, 1,000,000,000:1”

    אז איך א=ב וג=ד אז א=ב=ג=ד ?????

  85. ישראל!
    ניתן להסיק המון דברים, בהתאם להנחות מקדמיות שמוסיפים למרקחת. את “נגד רצונך” אתה הסקת מתוך גישה בלתי אמפיריציסטית לחלוטין (וגם בלתי רציונליסטית, אגב).
    זה ויכוח סרק משום שהוא עוסק בניסויי חשיבה שאין בידינו אמצעים לאששם או להפריכם. אני זוכר ויכוחים כאלה בדיוק, על תורת היחסות ופרדוקס התאומים, שניהלתי בכיתה ז עם שני חברים מהכיתה (זה היה בזמן החזרה הכללית למופע, סטייל צפון קוריאה, של ערב יום העצמאות באיצטדיון של האוניברסיטה העברית בגבעת רם בשנת 1966, כשרוב הזמן ישבנו באפס מעשה על איזשהו משטח מוגבה וחיכינו להוראות) וכבר אז הגעתי לאותן מסקנות שאני דבק בהן עד היום.
    שמתי לבי לכך שבני אדם נוטים להיגרר לויכוחים כאלה. באין הגדרה מדויקת יותר לגורמים לצורך האינטלקטואלי גוזל הזמן הזה, מכנים אותם “רצון חופשי”. השאלה אם היה או לא היה רצון חופשי, מקומה בבית משפט של מושבעים. כבר לפני מספר שבועות עלה בדעתי להציע לך להיות עורך דין פלילי כי נראה לי שיש לך כל הכשרונות הנדרשים על מנת להצליח יפה בתחום הזה, ובדעתי זו אני מתחזק מיום ליום.

  86. לעוסקים בדבר,
    מעניין לקרוא את השרשור.
    האם זה וודאי שטמפ’ אינה גודל יחסותי?

  87. ר.ח.
    מאז השפעת, אני הולך לישון מוקדם. תוכל לראות את תחילת הדיון ביננו ב
    https://www.hayadan.org.il/astronomers-reach-new-frontiers-of-dark-matter-130112/#comment-331534
    שים לב שמעולם לא קיבלתי את ההנחה ששעוניה של ג’יל יראו זמנים שונים. זוהי מערכת במנוחה, ששני השעונים בה מראים את אותו הזמן, ממש כמו שעוניו של ג’ק. אחרת ניתן היה להבחין בין מערכות אינרציאליות, בניגוד לפוסטולט א’.ונכון, זה אינו מסתדר עם היחסות, ומכאן הדיון.

    ד. הסתכלתי בלינק שלך. התאום הצעיר מסתובב. אי אפשר להסתובב בלי להאיץ ( או להאיט – היינו הך).

    לילה טוב.

  88. הראה לי מתי הסכמתי לב’.
    הרי כל הזמן אני מנסה להראות לך ששעוניה של ג’יל חייבים להראות את אותו הזמן כמו ג’ק. זוכר את: אם א=ב וג=ד אז א=ב=ג=ד?
    ג וד’ מתייחס לשעוניה של ג’יל.

  89. ישראל,

    לגבי הציטוט כבר עניתי לך למעלה. אבל נקרא לסוקרטס שוב

    א. האם שני השעונים של ג’ק מראים את אותו זמן? כמדומני שכבר הסכמנו על זה שכן, הלא כן? להזכירך 1:1

    ב. האם יש הבדל בין שעוניה של ג’יל ? גם על זה כבר הסכמנו שיש הבדל, הלא כן? להזכירך 9^1:10

    ג. אם ענית כן לשתי השאלות אז איך ייתכן שבתשובה ו’ אתה טוען “אני טוען שאצל שניהם עבר אותו הזמן בדיוק, גם בשעוני הצז’ וגם בשעוני הטמפ” ? אתה לא רואה שאתה סותר את עצמך?

    ד. איך בפרדוקס התאומים תאום אחד (שנקרא לו ג’ק) מזדקן ואילו השני שיצא למרחקים (שנקרא בסיפורנו ג’יל) נשארת צעירה אם הזמן זהה? ואל תגיד לי מערכות מאיצות, תקרא את הציטוטים שהבאתי לך למעלה מהערך פרדוקס התאומים שמראים שזה לא תנאי הכרחי.

    ה. זו לא שאלה אלא הצעה. תבדוק את תשובותיך ל א’ ו ב’ לא ייתכן שענית “לא” כי כבר הסכמנו עליהן והן היו למעשה רטוריות כמנהגו של סוקרטס הזקן, האם ייתכן שלא קראת בתשומת לב וענית כלאחר יד?

    ו. לגבי הויכוח שלך עם יובל זה נכון ולא נכון. זה נכון שאני חף מכל עוון וזה לא נכון שגררת אותי בעל כורחי.

  90. ר.ח.

    אם לא נשנה דבר בנתונים המקוריים, הרי תשובות סוקרטס.

    א. לא.
    ב.לא.
    ג. כן.
    ד.לא. ( נראה לי שהפכת את הסדר. אצל ג’ק הזמן חולף מהר. אצלו עברו מיליארד שנים).
    ה. לא. ( כנ”ל, הפוך).
    ו. איני טוען זאת. אני טוען שאצל שניהם עבר אותו הזמן בדיוק, גם בשעוני הצז’ וגם בשעוני הטמפ. זו בדיוק אי ההתאמה עם תאוריית המפץ – נושא דיונינו.
    ז. אם התלמיד שלו הוא אפלטון – אז שיתאבד.

    עכשיו, לשאלתי: “. האם אתה מקבל את זה שלפי היחסות שעונו של ג’ק נע לאט יחסית לשעונים במערכת של ג’יל בדיוק באותה המידה ששעונה של ג’יל נע לאט יחסית לשעונים במערכת של ג’ק?”

    ענית: “בודאי שלא!”

    איך זה מסתדר עם:

    When two observers are in relative uniform motion and uninfluenced by any gravitational mass, the point of view of each will be that the other’s (moving) clock is ticking at a slower rate than the local clock.

    האם ג’ק וג’יל לא מתאימים להגדרה פה בדיוק נמרץ?

    ובענין יובל:

    שים לב להערתו:

    “אולי עוד תצליח לסחוט ממני תשובה שלא אוכל לערוב לנכונותה אך די יהיה בה כדי לגרור אותי לויכוח סרק אינסופי כפי שעשית עם ידידנו החף מכל עוון.”

    שממנה ניתן להסיק שגררתי אותך נגד רצונך, כתינוק שנשבה, לאיזה ויכוח סרק שלא היית ואינך מעוניין בו.

    ומכיוון שבעניין הזה אני אמפיריצט ואינני עוסק בספקולציות, אשאל אותך ישירות: האם נכון הדבר? האם גררתי אותך לאיזה ויכוח נגד רצונך? ומה שחשוב יותר, האם אתה מעוניין שנסיים עכשיו?

  91. ישראל,

    סוף סוף ההודעות שלי יצאו ממכבש המשטר צחות וזכות. אז אנא גלל למעלה עד להודעה של רובי ותתייחס אליהן.

    יובל וישראל הנאהבים והנעימים,
    אז עברתם לשלב השירים? לנפלאתה לי אהבתך מאהבת נשים כבר היגעתם? טיול לברוקבק כבר אירגנתם?

  92. ואיך אמפיריצסט כמוך מסיק שזהו ויכוח סרק? ולמה אתה חושב שהוא נגרר? האם עברת על השירשור או שאתה קורא רק את תגובותיך?

  93. ישראל, קמעה קמעה אתה חושף את פרצופי. אכן, אכזר שבאכזרים אנוכי.
    הסתתרתי מאחורי השלט “אמפיריציסט” ובזה קיויתי שתבין מדוע אינני עונה. אך אתה, עקשן שבעולם, אולי עוד תצליח לסחוט ממני תשובה שלא אוכל לערוב לנכונותה אך די יהיה בה כדי לגרור אותי לויכוח סרק אינסופי כפי שעשית עם ידידנו החף מכל עוון.
    אגב אכזר, It takes one to know one

  94. יובל
    כי הנה ניקוי ראש הולך ונגמר,
    הוי כמה יפה יהיה המחר,
    הוי כמה נאה,
    ומה מאושר,

    ולמה לשאלתי
    לא ענית, אכזר.

  95. ישראל,

    למיטב ידיעתי הדלה פרדוקס התאומים קביל גם במערכות לא מואצות. אפילו הלכתי לבדוק את זה בשבילך. ראה כאן : http://en.wikipedia.org/wiki/Twin_paradox

    במיוחד תחת הסעיפים :
    Specific example ו Resolution of the paradox in special relativity

    בכל מקרה. קצת הלכנו לאיבוד. מה הטענה שלך בדיוק? ששעון הצזיום של ג’יל ו ג’ק יראה אותו דבר? הרי הסכמת שלא. כלומר ג’ק כן הזדקן יותר מג’יל אז מה אתה רוצה ממני?

  96. ♫ אז במצב הנוכחי, שלא הכל הכי הכי ♪
    ♫ רק מה צריך הבן אדם, אם לא לשמור על ראש נקי ♪
    אם לטרנטלה האינסופית לצלילי כינורו (הלא כל כך מכוון) של הכנר המהיר ביקום אתה קורא “ניקוי ראש”, הרי קורצת מפלדה קשיחה במיוחד. פלדת יהלום. כל הכבוד לסבלנות ולהתמדה 8) (עוד משהו חיובי שלמדתי ממך 🙂 )

  97. ר.ח. שתף אותנו במאמר, זרוק עצם לרעים מיתולוגיים מקוסמו.

    פרדוקס תאומים זה מערכות מואצות. אנחנו כרגע במערכות אינרציאליות. האם אתה מסכים לנאמר בסטטייטמנט, ללא התוספת שאינה קשורה כרגע לעניינינו, ושאולי נגיע אליה אח”כ? האם הוא ישים למערכות של ג’ק וג’יל?

  98. יובל,

    גם אני אמפיריציסט. למעשה זו העבודה שלי. הידען בשבילי זו תרפיה. בין לבין הגרפים והמאמר שאני כותב אני מציץ וקצת מנקה את הראש.

  99. הפוך כמובן, איך זה שרק התאום על כדור הארץ מזדקן וזה שטס נשאר צעיר אם הזמן של כל אחד מהם עובר יותר לאט יותר ביחס לשני?

  100. ישראל,

    למה שלא תסתכל קצת יותר למטה בדף שהעתקת ממנו על הסטטיטמנט :

    Time dilation would make it possible for passengers in a fast-moving vehicle to travel further into the future while aging very little, in that their great speed slows down the rate of passage of on-board time. That is, the ship’s clock (and according to relativity, any human travelling with it) shows less elapsed time than the clocks of observers on Earth. For sufficiently high speeds the effect is dramatic.

    פרדוקס התאומים מכיר? איך זה שרק התאום שטס מזדקן?

  101. ר.ח. מסכים איתך, השאלה היא מה טוען יובל.

    לפני שנמשיך, מה דעתך על הסטייטמנט הבא:

    When two observers are in relative uniform motion and uninfluenced by any gravitational mass, the point of view of each will be that the other’s (moving) clock is ticking at a slower rate than the local clock. The faster the relative velocity, the greater the magnitude of time dilation.

  102. כרגיל התגובה ממתינה לאישור. יש למישהו איזה רשימת מילים “אסורות” ?

    ישראל,
    מה השאלה? ברור שזה המצב. שעון הטמפ הוא פרוב או מד חום שיוצא מן החללית לחלל ומודד טמפ’. מה שיש בחלל זה מה שיראו משי החום והמצלמות שמצלמות אותן.

  103. הולך על זה. הנוסחה אגב נקראת נוסחת פרידמן. שאלה: אם נאמר יש שתי חלליות המצוידות בשעוני טמפ’ והן חולפות זו על פני זו במהירות גבוהה, האם מצלמות חדות רזולוציה שבחלליות המצלמות את שעוני הטמפ’ בשתי החלליות תראינה בצילומים את אותו הזמן בשני שעוני הטמפ’?

  104. ישראל,

    אתה מחמיץ לחלוטין את דוגמת המכונית ואת הדמיון לחלליות. אם אתה מתעקש אין בעיה ליצור את פונקציית המרחק כפונקציה חד חד ערכית (1-1) :
    שתי נקודות במרחק ק”מ זו מזו. ג’ק נוסע לאט מאד בקו ישר בין הנקודות ומשלים את המרחק בשעה.
    ג’יל נוסעת במהירות גבוהה בזיגזגים בין הנקודות ומשלימה את המרחק בשעה.
    לשניהם מד מרחק ו- GPS שמייצרים גרף מרחק לאורך זמן.

    סוקרטס שואל:
    1) האם הגרף שיראה מד המרחק של ג’ק יהיה לינארי?
    2) האם הגרף שיראה הGPS שלו יהיה לינארי ?
    3) האם הגרף של ג’יל ממד המרחק בגלגלים יהיה לינארי מרחק מול זמן ?
    4) האם הגרף שהGPS שלה ייצר יהיה לינארי ? ואני מדגיש ללא חזרות או ירידות, לינארי.

    אתה לא צריך לענות. התשובה היא כן לכל השאלות. אבל אל תדאג יהיו גם יהיו שאלות.

    אז מה יש לנו כאן? מערכת לוקאלית במכונית שמודדת מרחק (אנלוגית לחלוטין לשעון הצזיום) ומערכת ייחוס חיצונית, קרי ה- GPS (אנלוגית לחלוטין לשעון הטמפ’). הפלא ופלא במערכות החיצוניות (GPS ושעון טמפ’) המרחק בדוגמת המכונית והזמן בדוגמת החלליות זהים בין ג’ק לג’יל,
    א. סוקרטס שואל: מסכים?

    הפלא ופלא 2, במערכות הלוקאליות, קרי מד המרחק במכונית או שעון הצזיום בחללית יש הבדל עצום בין ג’ק לג’יל.
    ב. סוקרטס: מסכים ?

    האנלוגיה ברורה. יש להסתכל על הזמן כמימד ואז האנלוגיה למימד מהשלושה ה”רגילים” מתבהרת.

    לשאלתך: “. האם אתה מקבל את זה שלפי היחסות שעונו של ג’ק נע לאט יחסית לשעונים במערכת של ג’יל בדיוק באותה המידה ששעונה של ג’יל נע לאט יחסית לשעונים במערכת של ג’ק?”

    תשובה : בודאי שלא! בוא נסתכל על מספרים. בשעון הטמפרטורות ירדה אצל שניהם הטמפרטורה מ – 6000K ל 2.73 .
    ג. סוקרטס : מסכים?

    לעומת זאת בשעון הצזיום של ג’ק היו X התפרקויות שמתאימות לשעה אחת.
    ד. סוקרטס: מסכים ?
    ואילו אצל ג’יל היו X^10 התפרקויות שמתאימות למיליארד שנה.
    ה. סוקרטס : מסכים?

    אז לסיום סוקרטס, לפני שלוקח את כוס התרעלה זועק, אני לא יכול למות עד שאבין !
    ו. איך ייתכן ש י. שפירא טוען שהשעון של ג’ק יכול לנוע יותר לאט מזה של ג’יל?

    ז. האם תיתן לו להתאבד בשקט?

  105. המכשירים האלה מחשבים את הזמן שחלף מאז המפץ הגדול על פי נוסחת התקררות (המתבססת על הנחות כלשהן על קצב התפשטות היקום) והצבת נתוני טמפרטורת קרינת הרקע הקוסמית ברגע הנבדק. אם נראה לך שהשם “שעון טמפרטורה” מגלם בתוכו את הפונקציה הזאת אז לך על זה.

  106. לגבי הויכוח מי נוסע בכמעט מהירות האור ומי נח, יש לי פתרון בשבילכם, לאחר הנסיעה שיפגשו ויראו מי הזדקן ומי נשאר צעיר….

  107. יובל, נראה לך שנוכל לקרוא לשעונים אלו “שעוני טמפרטורה”?

  108. ר.ח. בדוגמת המכונית עם הGPS, אפשרי בהחלט שאותה המכונית באותה הנקודה תראה מרחקים שונים, מה שאינו אפשרי לגבי היחס בין הטמפ’ והזמן..

  109. ר.ח.
    אחלה דיל, שאלות סוקרטיות. נראה לי שכמעט והתכנסנו.

    ראשית, תשובות לשאלותיך:

    1. כן.
    2. א-ו לא.
    ז כן.
    3. לא
    4. לא.
    5.כן. שעוני הטמפ’ מודדים טמפרטורה יחודית שתלויה בזמן לפי נוסחה רציפה שתוצאתה אינה יכולה לחזור על עצמה לפרמטרים שונים, מה שנקרא פונקציה 1 – 1. לכל זמן נתון בפונקציה משוייכת אך ורק טמפ’ יחודית. במקרה המכוניות המצוידות בGPS, אפשרי בהחלט ששתי מכונית באותה הנקודה יראו מרחקים שונים. לדעתי, אתה יכול בשקט לוותר על המכוניות עם הGPS ולגשת ישר לאופני כושר: הן תראינה לך מרחק עצום, בלי שזזו מילימטר. בקיצור – אינני רואה רלוונטיות לספורינו.

    6. כן.

    נראה לי שהתכנסנו מספיק כדי לצמצם את הכל לשאלה אחת:

    1. האם אתה מקבל את זה שלפי היחסות שעונו של ג’ק נע לאט יחסית לשעונים במערכת של ג’יל בדיוק באותה המידה ששעונה של ג’יל נע לאט יחסית לשעונים במערכת של ג’ק?

    יובל.

    הבנתי שאתה אומר שניתן תאורטית לבנות שעון שיראה את הזמן האוניברסלי המוחלט רק עיי מדידת הטמפרטורה.

  110. ישראל,
    אם פרוטוקול התקשורת אינו מוגדר כהלכה אז בין מה שאחד אומר לבין מה שהאחר מבין סביר כי יתקיים פער. אמרתי: “בהנחה שטמפרטורת קרינת הרקע הקוסמית זהה בכל מקום ביקום, הרי לנו שעון זמן אוניברסלי מוחלט”. אמור מה הבנת מזה ואבדוק אם אוכל לשלול או לאשר.

  111. יובל
    אתה כותב “בהנחה שטמפרטורת קרינת הרקע הקוסמית זהה בכל מקום ביקום, הרי לנו שעון זמן אוניברסלי מוחלט (“גריניץ’ גלקטי”, כהגדרתך)” האם אתה מתכוון לזה?

  112. טוב התשובות שלי ממתינות לאישור. דרך אגב הפרצוף הקול יצא בטעות משמונה + סוגריים. מעכשיו מספרים רק עם נקודות.

    אז בנתיים עד שזה משתחרר לסוקרטס שלי יש גם כמה שאלות:

    1. האם אתה מסכים שיחס השעונים צזיום : טמפרטורה של ג’ק הוא 1?

    2) האם לדעתך יחס זה ישתנה אם (סמן את התשובות הנכונות):
    א. ג’יל תהיה במצב מנוחה?
    ב. ג’יל תנוע במהירות 100 קמ”ש ביחס לג’ק?
    ג. ג’יל תנוע ב 0.99 ממהירות האור ביחס לג’ק?
    ד. ג’יל תעשה שמיניות באוויר, פליק פלאק לאחור ובעיטת קונגפו משולבת?
    ה. ג’יל בכלל לא תהיה קיימת?
    ו. כל התשובות א-ה נכונות?
    ז. כל התשובות א-ה לא נכונות?

    3) האם לדעתך תוספת של ג’יין ארתור או יוהנס דה גרוט לסיפור תשנה את היחס 1:1 בין שעוניו של ג’ק? אפילו אם ארתור ייסע לאחור ?

    4) האם אתה רואה סתירה בין היחסות למפץ הגדול בסיפור המכוניות המצויידות במד מרחק ו-GPS?

    5) האם אתה רואה הבדל עקרוני בין סיפור החלליות עם שעון צזיום ושעון הטמפ’ ע”ש שפירא לסיפור המכוניות המצויידות במד מרחק ו GPS? אם כן נמק.

    6) האם אתה עדיין רואה סתירה בין היחסות למפץ הגדול שנובעת מהמצאת שעוני הטמפרטורה?

    ההגנה נחה.

  113. 1) כן
    2) כן
    3) כן
    4) כן
    5) לא הבנתי. האם ג’ין נעה באותה מהירות של ג’יל? (אני מניח שזה מה שאתה מתכוון ב”מסנוכרן”). מה הקשר למרחק מג’ק? ומה הקשר למערכת הייחוס של ג’יל?
    6) לא ממש. אם ג’יל היא זו שמפעילה מנועים ומוציאה עשן מאחוריה ומתקרבת לגלקסיית אנדרומדה בה נמצא ג’ק אז נכון שאתה יכול להתפלסף ולהגיד שבעצם ג’יל עומדת במקומה וכל היקום כולל ג’ק שנח בו הוא זה שנע. כשאתה נוסע לעבודה, האם אתה במנוחה והעבודה מתקרבת אליך??
    7) לא. אם כדבריך ג’ין וג’יל מסונכרנות אז למה שזה יקרה?
    8) NA באותה מידה הייתי יכול לשאול אותך האם אתה מקבל את מה שתורת הפיזיולוגיה והביוכימיה של התא טוענות.
    9) ראה תשובה 6. אפשר להסתכל על זה כך אבל זה לא מקדם אותו לשום מקום אלא רק מערים קשיים. כמו להגיד שכשאתה במטוס מקליפורניה לישראל אתה נח וישראל עפה אליך.
    10) לא. יחס השעונים של ג’ק אינו תלוי בג’יל ג’יין או סבתא של פופציק. הוא תלוי ביחס בין מהירותו של שעון הצזיום שהוא נושא לקרינה המתקררת בחוץ שחומה נמדד ע”י שעון הטמפ’ ע”ש שפירא.
    11) אכן כן אותו ג’ק
    12) לא
    13) כן, שאתה טועה ולא מקשיב למה שאני כותב לך כבר חודש. מערכת ייחוס, ולא משנה מה היא תעשה, לא תשפיע על היחס בין שעוני צזיום לוקאליים במערכת הנעה לבין שעוני הטמפ’ המודדים את הזמן לפי קרינת הרקע ביקום.

  114. זו לא שיטת סוקרטס. סוקרטס שואל שאלה אחת בכל פעם וממתין בסבלנות לתשובה. זה לא הציל אותו בסופו של דבר מעונש, אבל הוא הצליח לחיות ככה עד גיל 70

  115. בסדר. בוא ננסה עוד הפעם את שיטת סוקרטס.

    1. האם אתה מקבל את העובדה הנתונה בניסוי המקורי שבעת המפגש מראה שעונה של ג’יל זמן איטי יותר משעונו של ג’ק? כן/לא

    2. האם אתה מקבל את זה שהסיבה לכך היא שג’יל נעה יחסית למערכת של ג’ק?

    3. האם אתה מקבל את זה שמה שאנחנו מתכוונים ב”מערכת של ג’ק” הוא בסך הכל ג’ק ושעונו + השעון הנוסף שג’יל חלפה על פניו בדרכה לג’ק, וששני השעונים הללו, ג’ק + נוסף, נמצאים במהירות 0 יחסית זה לזה ומסונכרנים בינהם?

    4. האם אתה מקבל את זה שהצלחנו להראות שלפי ההגיון שלך צילום הוידאו של שעוני ג’ק בלבד יראה יחס קבוע של 1:1 ואיל של ג’יל 1,000,000,000:1?

    5. האם אתה מקבל את זה שאם יש שעון נוסף אחרי ג’יל, ששמו ג’ין, ושעון זה נמצא במרחק זהה למרחק של שעוני מערכת ג’ק, ומסונכרן עם שעונה של ג’יל, הרי קיבלנו את מה שנקרא “מערכת היחוס של ג’יל?

    6. האם אתה מקבל את זה שבמערכת היחוס של ג’יל, ג’ק הוא זה שנע, באותה מהירות שג’יל נעה יחסית לג’ק רק בכיוון ההפוך?

    7. האם אתה מקבל את זה שלפי היחסות כשג’ק יחלוף על פני ג’ין יראה שעונו זמן איטי יותר מאשר כאשר חלף על פניה של ג’יל?

    8. האם אתה מקבל את מה שטוענת היחסות?

    9. האם אתה מקבל את זה שמה שקיבלנו כעת היא את אותה התמונה רק במהופך, וכעת ג’ק הוא זה שנע יחסית למערכת של ג’יל?

    10. האם אתה מקבל את זה שלפי אותו הגיון קודם יחס השעונים של ג’ק אמור כעת להראות 1,000,000,000:1?

    11. האם אתה מקבל את זה שזהו אותו ג’ק בדיוק שעד כה הראו שעוניו יחס של 1:1?

    12. האם אתה רואה את הסתירה?

    13. האם יש לך הסבר?

  116. ישראל,

    1) נו באמת! ליכלכת, הרי ג’יל נוסעת 600 ק”מ בקשת שמסתיימת 1 ק”מ ממקום ההתחלה. רציף לחלוטין.

    2) “ברור לחלוטין שהמערכת שג’יל נעה יחסית אליה היא מערכת המנוחה של ג’ק. מכאן גם יחס הסיבובים המדויק.” באמת?? אתה שוב חוזר לאי אבחנה שלך בין מערכת ייחוס לאלמנט משפיע. ומה אם ג’ק לא יהיה קיים? האם השעונים של ג’יל לא יראו יחס של 1:600 ?

    3) אותו דבר. אם אתה לא מבין את אי התלות שבין ג’ק לג’יל אין לי דרך להסביר לך. ספציפית :

    “מה יקרה אם ג’ק ישים גז ויצטרף לג’יל לטיסה במבנה?” ==> מבחינת ג’יל ושעוניה? כלום, אין השפעה
    “מה יקרה אם ג’ק ישים גז ויצטרף לג’יל לטיסה במבנה?” ===> עוד יותר כלום, אין השפעה
    “האם זה מה שיגרום לשעון הטמפ’ של ג’יל להאיט?” ==> חס וחלילה, למה שהשעון שלה יאט כתלות בג’ק? האם יש לגעתך איזה קשר מיסטי בין ג’ק לשעון של ג’יל?

    “הרי עכשיו אין כל מערכת נוספת, רק היא והחושך. מה גורם עכשיו לפער בין השעונים?” ==> הפער בין השעונים לא נגרם בגלל ג’ק. הפער בין השעונים נגרם כי שעון הצזיום בחללית יאיץ את מהלכו כי החללית טסה מהר ביחס למולקולות המתקררות שבחוץ ולכן שעון הטמפ’ יראה שעברה שעה בחוץ ואילו שעון הצזיום יראה מיליארד שנה כי הוא היה בחללית היכן שהזמן הואץ ע”י המהירות.

    “המצביע על איזו מערכת וירטואלית מסויימת, שאיננה קיימת עוד” ===> ???

    לסיכום: אני לא יודע אם אתה לא צוחק ממני. אם כן תהייה בריא אכן חוש הומור נפלא ובוא ניפרד כאן כידידים לפני שיהיה מאוחר מידי 🙁
    אם לא אז באמת שאני לא מבין איך אתה לא רואה שאין שום קשר בין ג’ק לג’יל וכל תוצאה שתהיה בשעוניהם אינה תלויה במערכת האחרת.

  117. הקדמה סמנטית: במתמטיקה יש הוכחות. בפיסיקה יש אישושים והפרכות אך לא הוכחות.
    הפתרון שמצאתי מבוסס על המודל שדיברתי עליו יותר מדי והעליתי את חמתם של גדולים וחכמים אשר קצה נפשם בנפנופי הידים שלי – ובצדק. הבאתי הסבר שטחי(*) שלא הניב שום פידבק. אין טעם להביא הסבר מעמיק, משום שזה מצריך הבנה ולימוד לעומק של המודל מן היסודות. המודל שלי הוא עוד אחת מיני תאוריות שניסכו ועוד תנוסכנה על מזבח תופעות הפיסיקה, וכפי שהוא מגלם בתוכו פתרון אפשרי לבעיה שהעלית, לא מן הנמנע שיימצאו פתרונות נוספים – אפילו יפים יותר – במודלים אחרים.
    (* המודל שלי מציג את תנועת הפוטון ו/או האלקטרון כתהליך דו-כיווני, לפיו מה שנחשב למוקדם הינו גם בעל היבט מאוחר. זה מסביר את התופעה שהתגלתה בניסוי יאנג. גם הספין של האלקטרון כמו הקיטוב של הפוטון נקבעים בתהליך הדו-כיווני הזה, וזה מסביר את תוצאות ניסוי אספקט ודומיו)

  118. יובל.
    אתה טוען שיתכן וקיימת עוד אפשרות. כזכור, הבטחת: “ישראל! בבקשה, אל תמהר להגיד “אין מנוס מלהכיר במציאות: היקום הינו אי לוקלי”. נראה לי שעליתי על פתרון לוקלי של התעלומה.”

    נראה לי שזה הזמן לזרוק עצם.

    ר.ח.
    תשובה לשאלתך: ג’יל עברה 300 ק”מ צפונה + 300 ק”מ דרומה.
    זו אינה פונקציה רציפה (אפשר להסתובב באמצע) ולכן האינטגרציה היא בחלקים, שלא תיתן 0. נוסחת פרידמן היא כן פונקציה רציפה.

    אקל עליך: ברור לחלוטין שהמערכת שג’יל נעה יחסית אליה היא מערכת המנוחה של ג’ק. מכאן גם יחס הסיבובים המדויק.

    שאלה: אם כל הניסוי ג’-ג’, היה נערך בשדה הניסויים הקבוע שלנ בטיזנבי, שבו אין מערכות נוספות, רק חושך על פני תהום, מה יקרה אם ג’ק ישים גז ויצטרף לג’יל לטיסה במבנה? או פשוט יאיין עצמו לאין? האם זה מה שיגרום לשעון הטמפ’ של ג’יל להאיט? הרי עכשיו אין כל מערכת נוספת, רק היא והחושך. מה גורם עכשיו לפער בין השעונים? ועוד פער כה מדוייק, המצביע על איזו מערכת וירטואלית מסויימת, שאיננה קיימת עוד.

  119. ישראל,

    לגבי שאלתך “למה ג’יל מראה יחס של מיליארד ל1, בשעה ש ג’ק 1:1. רק זה.
    יחסית למה היא נעה? לא דברים כללים, רק את המערכת הספציפית שג’יל נעה יחסית אליה.”

    סיבובי השעון נקבעים ע”י התפרקויות הצזיום. אצל ג’יל קרו מיליארד התפרקויות ואצל ג’ק מליון. ההבדל נובע מכך שג’יל נסעה. בדיוק אותו מספר התפרקויות היה קורה אצל כל אחד מהם גם אם השני לא היה קיים.

    למה הדבר דומה? שג’ק ייסע במכונית איטית 1 ק”מ באותו הזמן ג’יל תיסע 300 ק”מ הלוך וחזור . לשניהם יש:
    1) מד מרחק מחובר לגלגלי המכונית
    2) GPS שמודד את מיקומם ביחס לנקודת המוצא.

    עכשיו הם באותו מקום משווים את יחס המדידות בין מד המרחק ל GPS. היחס אצל ג’ק 1:1 ואצל ג’יל 600:1.
    עכשיו בוא נחזור לשאלה שלך עם מודיפיקציה קלה: “למה ג’יל מראה יחס של 600 ל1, בשעה ש ג’ק 1:1. רק זה. יחסית למה היא נעה? לא דברים כללים, רק את המערכת הספציפית שג’יל נעה יחסית אליה.”

    עכשיו תענה לבד.

  120. כאמור, השליטה שלך בלוגיקה טובה. ההנחה השגויה היא בסעיף 4: “קיימות 2 אפשרויות”. אם היית אומר “קיימות רק 2 אפשרויות”, אזי אתה היית האשם העיקרי. אולם אתה רק מצטט אחרים ולכן נמצא זכאי מחמת הספק. שתי האפשרויות הן תולדה של חשיבה ולא ראיה שנמצאה בשטח. אין לשלול קיומן של אפשרויות נוספות. אנחנו מודעים לצרופי מקרים ולכן זה שהניסוי מאשש את המסקנה “באופן גורף” עשוי לשכנע חבר מושבעים בבית משפט ממוצע אך אינו אומר דבר וחצי דבר על אמיתותה של המסקנה.

  121. יובל

    בוא נעבור על הטיעון בשלבים. ראה אם תוכל להצביע על השלב בו יכולה להיות טעות:

    1. שני אלקטרונים יוצאים ממקור משותף בכיוונים הפוכים.

    2. לפי חוק שימור הספין, הספינים שלהם חייבים להיות הפוכים.

    3. הספין הוא לאורך 3 צירים. הספין הפוך לגבי 3 הצירים.

    4. קיימות 2 אפשרויות:

    א. הספין נקבע עם ההיפרדות (משתנים חבויים, איינשטיין).

    ב. הספין נמצא במצב של סופרפוזיציה, למעלה + למטה, ונקבע סופית ק עיי המדידה של אחד האלקטרונים. (קוואנטים, אי ודאות, הייזנברג, בוהר)

    ג. אם המצב הוא כמו ב’, כאשר נקבע מצב הספין בוחר אחיו מיד את הספין ההפוך. אי לוקליות.

    5. איך נחליט בין א’ וב’?

    6. משפט אי השוויון של בל: הוכחה מתמטית שמצב ב’ הוא הנכון.

    7. ניסויי אספקט ורבים אחרים ( על פוטונים, אך הפרנציפ זהה). מאשרים את הצפוי ממשפט בל באופן גורף.

    8. שאלה: מה יכול להיות שונה? מה הקשר למסלול? הרי התוצאה הוכחה מראש מתמטית, הניסויים רק אישרו מה שהיה ידוע שצפוי לקרות. האם דבר בניסוי יכול לסתור את משפט אי שוויון בל והנובע ממנו?

  122. ישראל,
    אינני אחראי על מה שהבנת או לא הבנת, אך אנסה שוב. פוטון יוצא מנקודה אחת, ופוטון מגיע לנקודה אחרת (לפי המודל שלי זה אינו אותו פוטון, אבל נעזוב את המודל שלי). הניסוי מתעד אמפירית את נקודת המוצא ואת נקודת היעד ואולי גם מספר סופי של נקודות בדרך, אך לא את כל המסלול. אין לנו אפשרות לדעת מה בדיוק קרה בדרך. כל מה שיש לאל ידנו לעשות הוא להפעיל חשיבה משלימה (למשל, משפט ערך הבינים) וכך מסיקים מסקנות שאינן בהכרח נכונות.
    היקש לוגי, לדוגמה מודוס פוננס: אם א אז ב; א; לכן ב. אבל אם א אינו נכון, אזי ב שגוי.

  123. יובל, אינני בטוח שאני מבין.

    ההוכחה של בל היא מתמטית. הניסויים של אספקט אמפירים. מה כאן יכול להיות שגוי? הלוגיקה? איזו מסקנה נוספת יכולה להיות אפשרית? תוכל להראות לי את הפרטים? לתת איזה כיוון, איזו דוגמה?

    וגם לא הבנתי את הקשר למהלכי פוטונים.

  124. ר.ח. זה לא מה ששאלתי.
    פשוט:
    למה ג’יל מראה יחס של מיליארד ל1, בשעה ש ג’ק 1:1. רק זה.
    יחסית למה היא נעה? לא דברים כללים, רק את המערכת הספציפית שג’יל נעה יחסית אליה.

  125. ר.ח, תודה על ההבהרה.
    אגב שטוחלנדיה, מצאתי קבצי PDF יפים, כולל סריקה של עותק מן המהדורה הראשונה. אשמח להעביר לכל דורש. לאחרונה יצאה גם גרסת סרט עלילתי באורך מלא.

  126. ישראל,

    אני לא מבין את ההתנגדות שלך לג’יל נעה וג’ק לא. נכון אפשר להתפלסף שכשג’יל נכנסת לחללית שלה מניעה את המנועים האטומיים יוצאית לדרכה ומגיעה לאחר שעה קלה לאנדרומדה בעצם היא נשארה במקום וג’ק עם כל כדור הארץ התרחקו ממנה ואנדרומדה פתאום הגיעה אליה. אכן כן אפשר גם לראות את זה ככה… כמו שמאך שלך מאמין.

    איך שלושה פיזיקאים יחסותיים מבריגים מנורה? אחד מחזיק את המנורה ושניים מסובבים את החדר (או העולם או היקום, תבחר).

    טיעון נוסף שהתעלמת ממנו הוא זה שמערכת ייחוס היא רק מערכת ייחוס ואינה סיבתית. אתה טענת זה שמשהו נע ביחס אליך לא באמת משפיע עליך. זה שעכשיו אי שם בחלל יש חללית שנעה קרוב למהירות האור ביחס אלינו לא משפיע כהוא זה על בחירתי בפסטה או אורז כתוספת לעוף.

  127. ר.ח.

    1. בוא נגיד ששעוני טמפ’, שעוני התרחקות גלקסיות וגריניץ אוניברסלי כולם נותנים אותו זמן לכל מערכת.

    האם זה סותר את התארכות הזמנים במערכת לא מואצת – אני מאמין שכן. מאמין גם שהוכחתי זאת בדוגמת ג’ק וג’יל. אם תוכל להוכיח את טעותי – שוט. אם תוסיף עוד מערכות מדידה, רק כדי להראות ששעוני טמפ’ אינם היחידים, אינך סותר את טענתי – אתה מחזק אותה.

    ואין לי שום פוסטולט. הכל הסברים והוכחות.

    2. איך לא שכנעתי אותך אם הראתי לך שג’ק וג’יל חייבים להראות את אותו הזמן בשעוניהם אם מקבלים את פוסטולט א’? אולי לא השתכנעת, אבל הארגומנט שלך, שג’יל נעה “באמת” וג’ק לא, אינו קביל עלי. אם זה הדבר היחיד שיש לך, אשאר באמונתי. אם יש דבר חדש – שוט.

    3. אינני מנתח את כל דקויות הסתירות בין כל התאוריות הקיימות. אני מדבר על נושא אחד קטן: התארכות הזמנים ומהירות האור. אם יש לך משהו חדש – שוט. אבל דבר אחד אני חייב לוודא: בדוגמת ג’-ג’: האם ההסתייגות היחידה שיש לך היא הנושא שג’יל נעה וג’ק לא, או שיש משהו נוסף?

    4. זה חוקי לספור. אך בווגאס קזינו נחשב למועדון פרטי שאינו חייב להכניס אותך אם אינו מעוניין. באטלנטיק אסור להם למנוע ממך לשחק, וכתוצאה חוקי המשחק שונו כך שאי אפשר להרויח אפילו אם סופרים. זו אחת הסיבות שכולם נוסעים לווגאס ולא לאטלנטיק. בשורה התחתונה, הם פגעו בעצמם.

  128. ישראל,
    על המסקנות שאנשים מסיקים מתוצאות ניסויים, אם אנקוט בלשונך: “הניסוי אחראי רק למה שהוא מראה”. טענות כגון “מהירות האור היא קבועה בכל מערכת ייחוס” או “אינפורמציית הספין עוברת מהר מהאור” משקפות מסקנות אך אינן מוכחות ישירות. כשאתה אומר שEPR טעו, אתה שופט על פי מסקנה מן הסוג הזה. הלוגיקה שלך אריסטוטלית למהדרין, אך היקש המסתמך על עובדות לא נכונות, מוביל בהכרח למסקנות שגויות. כאמור, המסקנה מניסוי בל היא רק מסקנה אפשרית. אין לנו (וגם אין דרך לקבל) רישום רציף של מהלכי הפוטונים לכל אורך הדרך. זו הטעות שאני מוצא בטיעון שלך. ר.ח חריף ובקי יותר ממני, אולי ימצא טעות יותר מהותית.
    אגב, ראה מה מצאתי באגף העתיקות:
    http://www.haaretz.co.il/misc/1.815624

  129. יובל,
    זה לא מה שהתכוונתי. מה שרציתי היה להראות לידידנו (אתם ידידים נכון? סטטוס לייק?) שהשעונים הטמפרטורים שלו הם לא דבר מיוחד אלא בסך הכל עוד מערכת ייחוס רגילה שמתנהגת לפי היחסות המהוללת.

    ישראל,
    1) התחמקת באלגנטיות של קיפוד. יופי גם אתה חשבת על זה, אז מה התשובה? האם עדייין אתה חושב ששעוני הטמפ שונים ממתקן הגריניץ הגלקטי? אם כן מדוע ואם לא מה המשמעות להנחות המוצא שלך לפוסטולטים שלך?

    2) “אתה מוצא טעות בטיעון שלי? שוט.” האם זה לא מה שאני עושה כבר כמה מאות הודעות? אתה מתעלם מנפנף ידיים או נותן פה ושם תשובות אבל לא שיכנעת אותי עדיין בשלב הראשון של המבנה שלך שיש סתירה בין תורת היחסות למפץ.

    2.5) אפרופו מפץ, אז עכשיו אתה טוען שאתה לא אוהב אותו, יש לך הסבר חליפי להתרחקות הגלקסיות + טמפרטורת הרקע? שוט!

    3) ” לפי ההגיון הזה, אז היחסות טועה. אינפורמציית הספין עוברת מהר מהאור. איינשטיין טעה בEPR. האם נבטל את היחסות? האם עכשיו יעברו החלקיקים במאיץ את מה”א? האם נפסיק לחשוש מפצצה איראנית? הרי אין ספק שאיינשטיין טעה!” ====> אם אתה חושב שתטיח בי “היחסות טועה” ואני אפול מהכיסא אתה טועה. הרי ברור ונהיר שמכניקת הקוונטים לא מסתדרת עם היחסות. ברור ונהיר ששתיהן עמדו בכל ניסויי ההפרכה שנעשו עד היום ולמרות זאת הן סותרות זו את זו. הנוסחאות של היחסות קורסות במרחקים קטנים. הגרויטציה ביחסות היא עיקום המרחק ונישאת ע”י גרויטונים ביחסות, EPR שלך ועוד כהנה וכהנה. אז מה חדש פה? כולנו יודעים שישי בעיה בתאוריות. זה למה התכנסנו פה, זה למה יושבים בכירי הפיזיקאים ומנסים לפתח תורות מיתרים.

    4) הטיעון היחידי שאני לא מוצא בו מתום הוא לגבי רפא*ים. אה, וגם לגבי הקזינו בזה שיכנעת אותי. האם מותר להם לזרוק כל אחד שיושב ומרוויח? יש לזה עילה חוקית? יש חוק נגד ספירה בלב?

  130. ר.ח.

    אכן הבראתי, וכולי מוכן ומזומן לקרבות.

    1. רעיון נחמד, שגם אני חשבתי עליו. קרא את המייל הבא, ושים לב לתאריך.

    From: [email protected]
    To: [email protected]
    Sent: 12/24/2011 2:30:44 P.M. Pacific Standard Time (Me
    Subj: (no subject)

    Hi Gil

    I Would like to Thank you for taking the time and discuss with me the issue of time dilation. After you left, I thought that what may demonstrate my idea of an absolute time is the following example: suppose we have a powerful transmitter on earth which transmits, using radio, to the universe earth’s time in 1 sec intervals.

    So the 0th’ point will be for example Dec 31 2011 at 12 midnight Greenwich’s time, and then every second it will send a short and powerful signal: 1, 2, 3, …..100^100.

    At each point in the near universe, a receiver which will get the pulse can calculate earth’s distance (at time of transmission) from the signal’s strength and Doppler shift, and thus arrive at what will be called: “UNIVERSAL ABSULUTE TIME”. in our example of attacking ships which try to synchronize their clocks, this universal time will give them a useful way to coordinate the attack. Note also that if there were more sources of signals which are arranged at the same way as the original one, and their clocks are synchronized initially with earth and they are moving in a constant velocity in space, no matter how far and in which direction , then every ship in the universe will receive from them the same “UNIVERSAL ABSULUTE TIME”, no matter which of the sources it will elect to use.

    א + ב. אינני יודע את התשובה. אני כל כך עסוק פה בדיונים, בתכנון ניסוי ובכל שאר עיסוקי היומיום, שלא יצא לי לעבור על הרעיון בפרטנות. הצגתי אותו בכמה פורומים פיזיקליים, אף אחד לא ענה.

    2. “אתה בהכרח טוען שהיחסות אינה נכונה”. זו הסקת מסקנות נחפזת. כמוה כמסקנתו של יובל שאם אני אומר שEMISSION THEORY מסבירה יפה את תוצאות ניסוי מ-מ, הרי פרוש הדבר שהיא מסבירה גם העובדה שמהירות האור זהה לכל מודד. בשני המקרים זה לא מה שאני אומר, ואני אחראי רק למה שאני אומר, לא למסקנות שמסיקים מדברי.

    אמרתי פה לפחות 10 פעמים: איני יודע. קשה לי לקבל אינטואיטיבית את הרעיון של התחלת הזמן (מה היה קודם?). אני אוהב את האלגנטיות המתמטית של היחסות. משהו לא מסתדר. זה הכל. אין לי יומרות לבנות תמ”ג.

    אתה מוצא טעות בטיעון שלי? שוט.

    3. “סרן כן מפריכה את התאוריה שלך. תורת היחסות מדברת על מהירות האור כגבול עליון לכל דבר, מסה או לא מסה.” לפי ההגיון הזה, אז היחסות טועה. אינפורמציית הספין עוברת מהר מהאור. איינשטיין טעה בEPR. האם נבטל את היחסות? האם עכשיו יעברו החלקיקים במאיץ את מה”א? האם נפסיק לחשוש מפצצה איראנית? הרי אין ספק שאיינשטיין טעה!

    אתה מוצא טעות בטיעון שלי? שוט

    4. בשביל דור שלם מדבר סיני היה בעיקר זירת ההתכתשות עם מצרים כל כמה שנים. האריק היחיד שהיה שם הוא שרון, שהיה מופיע מעוטר בתחבושת בדרכו לצד השני של התעלה.

    5. עד לפני כ50 שנה, בלקג’ק היה משחק די שולי. אחרי שת’ורפ כתב את הספר beat the dealer, הפופולריות של המשחק גאתה, כולם ניסו להביס את הבית, ואכן רבים הצליחו. הקזינוס שינו מיד את החוקים, כולל הוספת חפיסות, אז האנשים הפסיקו להגיע.

    כיום הושג סטטוס קוו: ניתן להביס את הבית, וידיעה זו מושכת המונים והפכה את בלקג’ק למשחק הפופולרי ביותר בקזינו. אולם רק מעטים מצליחים באמת לעמוד בעומס, ובעיקר במהירות שנדרשת כדי לספור במבט חטוף שולחן שלם ובו 6 שחקנים. עבורם, הקימו הקזינוס מערכת מעקב משוכללת שכוללת מערכת לזיהוי פנים, מחשבים המסוגלים לפי אופן ההימור לדעת אם שחקן הוא סופר או סתם מהמר, ואפילו סוכנות מיוחדת שנקראת גריפין, שתפקידה לזהות את הסופרים, שרבים מהם משתמשים בתחפושות.

    זו חוויה לא נורמלית כשאתה צעיר, במיוחד אם אתה אוהב טיולים. אתה יכול להסתובב בכל העולם, להתארח במלונות הטובים ביותר, לאכול במסעדות הכי שוות, לראות את כל ההופעות הכי נחשקות, לשמוע את כל הזמרים, הכל בחינם – ולצאת עם משכורת בסדר גודל של רופא פלסטי.

    בשורה התחתונה, ואני מתכוון לזה – זו מלכודת. קן יוסטון היה אחד הכוכבים של מערכת הפיננסים בסאן פרנסיסקו. נשוי, ילדים, קריירה. הוא עזב הכל בשביל אורח חיים של אלכוהוליסט, הקדיש את כשרונו המתמטי היחודי לחישובי הסתברויות של קלפים, וסיים לבד בדירה בפריז מוקף בקבוקים.

    7. רפאים, כשמו כן הוא, ישות שאין לה חיים, וכל תכליתו להציק למגיבים ללא הערה אחת עניינית (תיקון כן הערה אחת עניינית).

    אתה מוצא טעות בטיעון שלי? שוט.

    רפאים – למה שלא תתעלק על הבן דוד הצעיר והמוצלח ותשחרר את הזקנים להעלות זכרונות?

    מה חדש מתחת לאפס המוחלט?

  131. ר.ח,
    בהנחה שטמפרטורת קרינת הרקע הקוסמית זהה בכל מקום ביקום, הרי לנו שעון זמן אוניברסלי מוחלט (“גריניץ’ גלקטי”, כהגדרתך). לא היה לי ברור שלזה התכוונת.

  132. רע-פאם,
    עוד פעם לקחת את הכדורים הורודים במקום הסגולים? לא נורא תישן קצת תשחק בחוץ בארגז החול וזה יעבור.

    תיזהר רק לא ליפול מתחת לאפס המוחלט.

  133. שפירע

    זה הכל?
    הבן דוד שלי שהוא יותר צעיר ממני, מצליח יותר ממך (ואני לא אזכיר את האחרים).

    אגב, זה לא התרגיל שלך, ישראל.
    זה הגיל שלך.
    פוווף. נהיה פה ריח של זקנים… איך קוראים לזה? סיניליות?

  134. ישראל,
    אני שמח שהבראת ואני רואה שחזרת מלא עזוז ומרץ.

    1) סוקרטס שואל: נבנה מתקן שנכנה אותו הגריניץ’ הגלקטי שיכיל שעון צזיום שנמצא במנוחה ומשדר לכל הכיוונים את הזמן. האם החלליות שלך שמתכננות התקפה תוכלנה לדעתך לתאם התקפה אם הן יודעות את מיקומן המדויק ואת מיקומו של גריניץ?

    א. כן ==> אם כך מה ההבדל העקרוני משעון הטמפרטורה שלך?

    ב. לא ==> למה? הרי כל מה שכל חללית צריכה זה לקלוט את השדר, לנתח מה המרחק שלה מגריניץ’ להכפיל מרחק זה במהירות האור, להוסיף לתוצאת השדר והנה היא יודעת את זמן גריניץ’ המדוייק. דרך אגב נכון להיום יש כמה מתקנים כאלה שעובדים בהצלחה רבה. קוראים להם לוויני GPS.
    זה נכון גם לשני הצינורות שלך.

    2) אתה טוען שיש סתירה בין המפץ ליחסות. מאחר ואיני זוכר שטענת אי פעם טענות כנגד הוכחות למפץ (ותקן אותי אם אני טועה, בכל זאת >1000 הודעות) אתה בהכרח טוען שהיחסות אינה נכונה.

    3) סרן כן מפריכה את התאוריה שלך. תורת היחסות מדברת על מהירות האור כגבול עליון לכל דבר, מסה או לא מסה. טאכיונים הם חלקיקים דמיוניים שלא מצפו עדיין והם אינם יכולים לרדת מתחת למהירות האור. אתה בא ואומר, “כן לגבי מסה זה נכון האתר האקטיבי שלי לא משפיע והיחסות תופסת ובכלל אני לא מתעסק בזה. אולם לגבי פוטונים, אה, זה סיפור אחר”. האם זה לא נפנוף ידיים והכנסת סיבוכיות למערכת?

    4) בשבילי סיני זה מקום קסום של צלילות, פסטיבל נואבה, נודיסטים, חומרים, מדבר מדהים מדהים מדהים. טיזאנבי זה הכחול במוצב לייד ג’בל ברוך בלילה בין הנותבים האדומים והירוקים שנפסקו רק כשואן אריק הופיע בערוץ המזרח התיכון.

    5) תודה על השיעור. אבל שאלה. אם זה כל כך פשוט ונגיש איך זה שבלק ג’ק לא הוצא אל מחוץ לקזינואים? איך זה שלא כולם משתמשים בשיטות האלה?

  135. זהו, התמצתה שפעת הכוכבים.

    (יודעים אגב מה מקור המלה שפעת?).

    ר.ח. יקיר.

    1.

    עשית פעם הוכחות בגאומטריה?

    אינך יכול לגשת לשלב מתקדם לפני שהוכחת את השלב הקודם. ויהיה אשר יהיה, בתחתית שרשרת המזון תתקל באקסיומות, אותן הנחות מובנות מאליהן אשר לא ניתן להוכיחן, אך עליהן בנוי כל משפט.

    לא אוכל להוכיח לך את נכונות משפט פיתגורס אם לא תקבל את האקסיומה האומרת שבין כל שתי נקודות במישור עובר רק קו אחד בלבד, שהוא גם המרחק הקצר ביותר בינהן.

    המקביל לאקסיומה בפיזיקה הוא הפוסטולט.

    והפוסטולט הראשון של היחסות (ולא רק של היחסות) הוא השקילות של כל המערכות האינרציאליות. שכל מערכת שאינה מואצת נמצאת בעצם במנוחה, ואין להבדיל בינה לבין מערכת אחרת שאינה מואצת.

    הדיון שלנו על התארכות הזמנים היה בנוי כהוכחה גאומטרית, כשכל טיעון חדש נבנה על ההסכמה של שנינו על קודמו. בסיום כל שלב הייתי מעלה שאלה ושואל אותך אם אתה מסכים או לא.

    הגענו להסכמה ששני שעוני טמפ’ באותה נקודה יראו את אותו הזמן תמיד, לא משנה מה מהירותם היחסית.

    בדוגמת החלליות של ג’ק וג’יל, הגענו להסכמה שבמערכת של ג’ק יראה וידאו משותף של שעוני הצז’ והטמפ’ יחס סיבובים של 1:1., או אותו זמן בשניהם תמיד.

    נשארנו חלוקים לגבי החללית של ג’יל. אתה טוען שהיחס יהיה 1,000,000,000:1, מכיוון שהמערכת הזו בתנועה “אמיתית” ואילו אני טוען שאין דבר כזה תנועה אמיתית, עיין פוסטולט א’. ג’יל בתנועה לא מואצת, לכן גם במערכת של ג’יל היחס יהיה 1:1, ושני השעונים יראו את אותו הזמן תמיד.

    ברור שאי אפשר להמשיך בהוכחה אם לא מסכימים על נקודה כה עקרונית. למרות זאת, הראתי שאילו כן היית מקבל את הנחתי הקודמת, הרי שאין התארכות זמנים מכיוון שאם א=ב, ג=ד וא=ג, אז א=ב=ג=ד ולכן בעת המפגש בין ג’ק וג’יל יראו שעוניהם את אותו הזמן.

    ולכן הראתי לך בדוגמת הצינורות הנעים, שמכיוון שכל השעונים בשני הצנורות מראים את אותו הזמן תמיד, ולכן אין התארכות זמנים, אין מנוס מן הסיכום שאותו הפוטון נע במהירות גבוהה יותר יחסית לצינור שמתקדם בכיוון תנועתו.

    מ.ש.ל.

    אלא אם כן, כאמור אינך מקבל את פוסטולט א’, שורש המחלוקת ביננו. אבל אם אינך מקבל אותו, איך אתה יכול לקבל דבר כלשהו ביחסות בכלל? הרי היא בנויה עליו!

    הראה לי פגם נוסף בטיעון חוץ מפגם זה, ונוכל להתדיין. אחרת עלי להסיק שהסיבה היחידה שאתה עדיין דבק בהתארכות הזמנים, היא הטענה הקבועה שלך שיש מערכת שנעה “באמת” (ג’יל לדוגמה). אם כך המצב, אני ממשיך בלב שקט להאמין שההוכחה שלי היא ללא פגם, עד שיוכח אחרת.

    2. סרן לא מפריך לי דבר בתאוריה. ציינתי כמה פעמים שזהו נושא שונה. עבור על השירשור. הסכמתי אתך שיש התארכות זמנים במערכות מואצות. כל הדוגמאות שאתה מביא לסתירת טענותי, ניסוי המטוסים, סרן, הם ניסויים במערכות מואצות. הראה לי את הנקודה שבה התיימרתי לטפל במסות, חוץ מבמודל הגרביטציה והאינרציה שהוזכר כל הזמן רק כהצגה משנית, שבה נפתרת בעיית החיכוך אצל לסאז’ באמצעות המודל שהסברתי. כבר שנה אני מדבר פה על בעיה בנושא אחד: התארכות הזמנים. אין לי בעיה עם השאר. אם הובנתי לא נכון, אני מצטער ומתקן. אך ורק התארכות זמנים. נגמור עם זה – נראה אם יהיה כוח לשאר.

    וחוץ מזה ר.ח. יקר, שאתה חוזר וחוטא באותה הטעות שוב ושוב: הטענה שאני טוען שהיחסות מוטעית. זאת למרות שאני חוזר ומדגיש שאילו לא קיימת התקררות רציפה של היקום לפי נוסחת פרידמן, כפי שטוענת תאוריית המפץ הגדול, הרי שהיחסות מתחייבת לגמרי. אתה חוזר כל הזמן על נושא “המערכת החיצונית” שניתן למדוד זמן באמצעותה, כמו התרחקות הגלקסיות. מוכן לזרום אתך: אם אכן תוכל להסכים על זמן מוחלט באמצעות התרחקות גלקסיות, אוכל עדיין להוכיח לך שאין התארכות זמנים עיי שימוש בשעון גלקסיות, אך בתנאי אחד: עליך לאמץ ללבך את פוסטולט א’. איך אני?

    אז זכור – איני טוען שהיחסות אינה נכונה. אלא שהיא אינה מסתדרת עם תאוריית המפץ. למה שלא תטען שאני שולל את מפצגדול?

    3. הראה לי את הנקודה שבה טענתי שההסבר שלי על אתר אקטיבי מסביר אי לוקליות. הטענה שלי תמיד הייתה שהוא מאפשר ה י ת כ נ ו ת אי לוקליות. ( הוא לא?).

    איינשטיין הוא זה שטען שאי לוקליות אינה אפשרית בגלל היחסות (הוא לא טען? הוא לא טעה?).

    סיכום דבר: אם תמצא לי פגם בטיעון חוץ מדבקותי בפוסטולט א’ – אין לי בעיה להודות בטעות. אחרת אני ממשיך לדבוק בטענתי: יש סתירה בין התארכות הזמנים ביחסות לתאוריית נמפץ הגדול.

    4. מי זילזל? שמעת צליל זלזול? השם קצת, נו איך נגיד את זה, מעניין?

    5. זה הבעיה אתכם הצעירים. הריח החזק הזה, של הבשר הטרי מהבקו”ם. שאתם אומרים טיזנבי וג’בלחרה, ולא יודעים שאלו שמות אמיתיים של מקומות שפזמ”ניקים כמוני (ויובל ומאיר אני מניח) העבירו בהם את שירותם.

    6. נקווה שאבי לא יכעס יותר מדי. אם כן, קח את האחריות עליך.

    תמיד אני טוען שיש שני דברים שלכאורה נשמעים מסובכים, ולמעשה הם די פשוטים. הראשון הוא היחסות הפרטית. השני הוא ספירת קלפים בבלק ג’ק.

    בשיטה הקלסית של ספירה, שיטת הHI-LOW אותה שיטה שכל הקבוצות של MIT (שקיימות עד היום, אגב) משתמשות בה, משתמשים בשני דברים:

    1. אסטרטגיה בסיסית.
    2. ספירה וסיכום של קלפים גבוהים ונמוכים.

    האסטרטגיה הבסיסית היא שורה קשיחה של הוראות פעולה לכל מצב נתון. לדוגמה: יש לך 16 לדילר יש 10 – עליך לקחת קלף. לך 5 ולו 6 – עליך לעמוד. יש לך 11 – עליך להכפיל.

    אפשר להוריד אונליין, או לקנות בדולר בקזינו עצמו.

    2. קלפים מס 2-6 מקבלים ערך של +1.

    קלפים מס 7-9 הם חסרי ערך.

    קלפים מס 10 (כולל כל התמונות) ואס מקבלים ערך -1.

    מהלך המשחק בחבילה אחת כאשר אתה משחק לבד מו הדילר:

    1. שים את סכום ההימור המינימלי.
    2. שחק את היד הראשונה לפי כללי האסטרטגיה הבסיסית.
    3. סכם את כל ערכי הקלפים לפי המפתח שהבאתי. בסיום הסיבוב, תוותר עם מס מונה: שלילי, 0 או חיובי. אם ערך המספר המונה קטן מ2, השאר את סכום ההימור כפי שהיה. עם הוא 2, שלש אותו. 3, פי חמש. 4 ומעלה, עד כמה שתוכל, לפני שהסקיוריטי יגיעו וילוו אותך החוצה.

    אם יש שתי חבילות – חלק את המס המונה במס החבילות המוערך שנשארו ב”נעל” (הקופסה שבה מאוכסנות החבילות. כך תשאר עם מה שקרוי TRUE COUNT. עם 6 חבילות – חלק ב6. 600 – ב600 וכן הלאה. (זה בסדר, המקסימום הוא 8 חבילות).

    אחרי יומיים אימון תיווכח שזה נהפך לאוטומטי לחלוטין, כמו נהיגה או שפה זרה שבהתחלה קשים ואח”כ לא חושבים עליהם אפילו.

    הבעיה העיקרית אינה הספירה, אלא איך להישאר קר כקרח כשהסכומים גבוהים, ואיך למנוע מהקזינו למנוע ממך לשחק (בשפת הידען – לחסום את תגובותיך).

    זהו, אני מקווה שעכשיו אבי לא יחסום את תגובותי. בקזינואים אני חסום כבר שנים, לשמחתי.

    בשורה התחתונה – ביזבוז זמן. ע”ע KEN USTON, שבזבז שם את חייו ואת 170 נקודות האי קיו שלו, הרס את עצמו ומשפחתו.

  136. יובל,

    אני לא טענתי שאי אפשר לחשב זמן לפי ירידה של הטמפרטורה, להפך טענתי הייתה שאין בזה שום דבר מיוחד ולמעשה החלליות של ישראל יוכלו לקבוע מפגש למרות שכל אחת נסעה במהירות משתנה קרובה למהירות האור על סמך מגוון תופעות. הדוגמאות לתופעות שאולי ניתן למדוד זמן “אבסולוטי” כדרישתו יכולות להיות התרחקות גלקסיות, קצב דעיכת כוכבים, התאדות חור שחור, דעיכה של אטום רדיואקטיבי במקום מסויים. האחרון משול לשעון צזיום על כוכב שכל פעם החלליות של ישראל יצטרכו לעצור ולבדוק מה מצבו. כלומר גם אלה הם שעונים יחסותיים ואין שום דבר לא יחסותי כמו שהוא ניסה לטעון לשעוני הטמפ’.

  137. רובי,
    אם נסתכל על כל יחידת זמן כעל משהו מוגדר לעצמו, הרי היקום שלנו בנוי מאינסוף מימדים. אולם היות שאנו מקבלים את הזמן כמימד סופי (וגם יחיד), הרי שלושת המימדים שאנחנו מכירים מספיקים להתמצאות במרחב. את המימדים הנוספים של תורת המיתרים אפשר לראות כתתי-מרחב של שלושת הקיימים.

  138. ר.ח, תודה על ההפניה לשטוחלנדיה. יופי של ספר.
    אך לפני שאתה רץ לשלם לאמזון, הבא נא בחשבון שמדובר בטקסט שזכויות היוצרים שלו פגו לפני שנים רבות ולכן תוכל להוריד אותו בחינם מפרוייקט גוטנברג:
    http://www.gutenberg.org/ebooks/search.html/?format=html&default_prefix=titles&sort_order=downloads&query=flatland

    בקשר לשעון הטמפרטורות, רציתי להגיד לך שנדמה לי כי אתה לא לחלוטין צודק. כפי שמחשבים גיל של עצם על פי דעיכת איזוטופ (“זמן מחצית חיים”) יתכן שאפשר לחשב זמן מוחלט על פי קרבת הטמפרטורה לאפס המוחלט. זאת בהנחה שגם קצב ירידת החום הוא מעריכי.

    ההסבר שלי לתופעות “החזרה בזמן” מתבסס על המודל שלי. היות שאינני רוצה לפתח אותו כאן, אביא אותו רק בקיצור:
    הצגתי את הפוטון (כמו גם את האלקטרון) כ”פיסת חלל ריק”. חלקיק הנכנס אל אזור הפוטון מביא לכך שהפוטון חדל להתקיים, אך בו בזמן נוצר פוטון חדש במקום בו היה קודם החלקיק. באופן הזה, התנועה של הפוטון במרחב היא דו כיוונית. בניסוי “הבחירה המושהית” החלקיקים שנועדו לשמש את הפוטון בדרכו מוחלפים בחלקיקים אחרים בעקבות הסתרת המסך.
    שינוי הקיטוב של הפוטון (בניסוי בל), כמו גם שינוי הספין של האלקטרון, מוסבר גם הוא באופן דומה, ועל כך אינני מרחיב לעת עתה.

  139. תאורית המיתרים כבר בקונצנזוס שמאפשרת קיום של 10 ויותר מימדים, נראה לי שצריך לפורום הזה “דם” חדש שיפרה את הדיון.

  140. ישראל,

    סיכום בניים שלי:

    1) אתה חוזר ומטיח בי שדחיתי את יחסיות הזמנים למרות שלדעתי הראיתי לך באורים ותומים שלא כך הוא הדבר ושלשעוני הטמפרטורה שלך הם יחסיים כמו כל דבר אחר ובאתה מידה היית יכול להסתכל על כל תופעה איטית שקורית “שם בחוץ” לאורך זמן כמו התקררות כוכבים, התרחקות גלקסיות, התאדות חורים שחורים וכו’.

    2) התעלמת מהפרכה ברורה של המודל שלך שבו בסרן היו אמורים להעלם חלקיקים במהירויות גבוהות אולם זה לא קורה. להגיד שאתה לא מתעסק עם חלקיקי מסה זה נפנוף ידיים. האם ייתכן שהיחסות מתאימה רק לחלקיקי מסה?

    3) ההסבר שלך לאי לוקליות ע”י אותו אתר פעיל קלוש ביותר. לא ענית לי איך אלקטרון פה “מודיע ” במדוייק לאלקטרון בג’יהנם מאחורי טיז א נבי לשנות את הספין שלו בלי שהם מהווים למעשה יחידה אחת.

    3.5) הסבר שהאלקטרונים מחוברים במימד אחר אינו נופל בסבירותו מזה שהפוטונים נעים מעל מהירות האור רק מה לא נקלטים לצערנו בחיישנים שלנו.

    4) הגרוע מכל זילזלת במה שאינך מכיר. אם לא אכפת לך שטוחלנדיה (flatland) הוא לא ספר רוחות ורפאים אלא אלגוריה מתימטית משעשעת ומעוררת מחשבות עד כמה שזה נשמע מוזר על מימדים ומעמדות חברתיים. ספר מומלץ ביותר
    http://www.amazon.com/Flatland-Dimensions-Illustrated-E-Reader-ebook/dp/B004AM5AZO/ref=sr_1_1?s=books&ie=UTF8&qid=1332208762&sr=1-1

    5) רפידים ?? מה היית בבריגדה ? זה לא זה מגבעת חלפון?

    בכל מקרה נראה לי שהתמצינו ואנחנו קצת מתחילים ללעוס אוכל לעוס. אז אם אין לך איזה הברקה מדהימה בוא נניח את הדיון כאן עד שתפרסם את תוצאות הניסוי שלך.

    בנתיים אפשר לדון איך אתה יכול לספור יותר מחבילה או שתיים ללא מחשב בנעל שמציאתו תביא לשבירת עצמותיך קודם ע”י מאבטחי הקזינו ואחרי זה ע”י השוטרים המשולמים על ידם ולהגשת תביעה מכאן ועד הבלאג’יו?

  141. יובל תודה על הקומפלימנטים, אך הם לא מגיעים לי.
    סוזאן ותומר, כמו ישמעל ויצחק, זו יחידה לאמה וזה יחיד לאמו.

    אז, כן, שתל והשקה ללא ספק, אבל הגידול וההשקעה והקרדיט הם של לורי.

    פתרון לוקלי? את זה אני חייב לשמוע.

    ר.ח.

    1. “האפשרות הראשונה של איינשטיין הרבה יותר סבירה”

    נראה לי שאם קבלת כפי שאמרת את הרעיון שאדם בחללית תועה יוכל לדעת שהיום צמים כי זה יום כיפור רק עיי מדידת הטמפרטורה – הרי שדחית את יחסיות הזמנים. ההוכחה היא רק טכנית. אבל נראה לי שהתמצנו, אלא אם כן תהיה מעוניין להמשיך.

    2. “האם בכך שאתה טוען שיש חלקיקים מהירים מהאור פתרת את האי לוקליות?” אני טוען מה שטענתי תמיד: הפתרון שלי של האתר האקטיבי מאפשר התכנות של אי לוקליות.

    אפשר להסביר כל דבר באמצעות ממדים נוספים, קסמים, שדים ורוחות. שטוחלנדיה נשמע כמו יחידת הצפטיות שהייתה לנו ברפידים.

    שאלה: אחרי שעכלנו את נושא האי לוקליות, תוכל להסביר את תוצאות ניסוי הבחירה המושהית בלי להזדקק לנורא מכל: השפעה על העבר מן העתיד?

    3. אני גאה מאד. הייתי מעדיף להיות פחות גאה ושהיא תהיה קרוב לבית, אבל מה לעשות, אתה לא מצפה מסנוזי שהיא תלך לUSC לא? מה היא כמו החברות הפרסיות שלה שההורים החכמים שלהן לא מאפשרים להן ללמוד רחוק מהבית?

    כמו שאמרתי, המלכודת האמריקאית. וגם א ת ה כשיגיע הזמן, אם תוכל תאפשר לילדים את החינוך ה”מעולה” ביותר, אפילו במחיר איבוד הקשר היומיומי עמם, ותשב עם זוגתך בבית הריק. אבל מה שכן – תהיו גאים.

  142. ישראל ישראל ישראל בטח בה’,

    1) “המסקנה של איינשטיין: מהירות האור קבועה לכל מודד, מה שמשתנה הם הזמן והמרחק. שאלה: האם זו האפשרות היחידה שקיימת? האם העניין סגור סופית כמו אי הלוקליות? התשובה שלי היא שלא. קיימת עוד אפשרות תאורטית: מהירות האור היא רק יחסית לצופה.”

    נכון מסכים אפשרות תיאורטית תמיד יש. אולם האפשרות הראשונה של איינשטיין הרבה יותר סבירה לאור נסיונות אינספור שנעשו מאז 1905 על ידי כופרים כמוך ועד היום כולל ניסוי אופרה לא הצליחו להדגים שום טעות. אם תצליח בנסיונך שיחקת אותה ואפילו בתך תצדיע לך. ובכל מקרה כדבריך , זה יהיה כיף!

    2) אי לוקאליות – האם בכך שאתה טוען שיש חלקיקים מהירים מהאור פתרת את האי לוקליות? איך אומרים אצלכם? Give me a break ! איך תסביר שאלקטרון אחד מבין מיליארדים שולח אינפורמציה לאלקטרון אחר אחד קטן שנמצא שנות אור ממנו ומוסתר בדרך ע”י 7 גלקסיות ו 18 חורים שחורים ובתוך יער של מיליארדי אלקטרונים זהים לו? איך הוא יודע מי שזור אליו? ורק אליו בלבד?
    ההסבר הכי משכנע שאני שמעתי לאי לוקליות, גם אם אין לו שום תימוכין נסיוני, הוא ששני האלקטרונים מחוברים ומהווים למעשה גוף אחד כשהחיבור נעשה במימד אחר שבו הם למעשה צמודים. האם אתה מכיר את שטוחלנדיה של אדווין אבוט? שם הוא מבהיר איך דבר כזה ייתכן במעבר ממימד אחד ל 2 ומ 2 מימדים לשלושה.

    3) Cell and Molecular Biology זה כמו להגיד אסטרופיזיקה. זה תחום רחב ונרחב שתחת כנפיו אפשר לכלול כמעט כל מחלה מוכרת, כל אורגניזם וכמעט כל תחום בביולוגיה. כך שאתה צריך לברר יותר פרטים. תבדוק גם ב- pubmed את הפרסומים של המעבדה שהיא הולכת אליה ותראה. בכל מקרה U Penn היא אוניברסיטה מעולה ואתה צריך להיות גאה ולא רק לחשוב על עצמך ועל הממחטות שלך (אני גיבור עם ילדים בבית ספר יסודי, הדעות שלי ישתנו בעתיד אני מניח).

    יובל, נו כולנו במתח בנית את הציפיות עכשיו תפגיז.

  143. כל הכבוד גם לאב ששתל והשקה וגידל והשקיע 🙂
    ישראל! בבקשה, אל תמהר להגיד “אין מנוס מלהכיר במציאות: היקום הינו אי לוקלי”. נראה לי שעליתי על פתרון לוקלי של התעלומה.

  144. ר.ח. שמעתי.

    פילוסופיה.

    אפ”ר הקדישו 99% מהמאמר שלהם לנושאים טכניים ונוסחאות. בסוף במשפט אחד הציגו את האלטרנטיבה: אי לוקליות. זו נשמעה כה הזויה, כה “SPOOKY” במילותיו של איינשטיין, עד שעצם הצגתה הייתה אמורה לסתום את הגולל על הרעיון של משתנים שאינם חבויים. בוהר עצמו לא האמין באי לוקליות.

    אם אתה זוכר את המאמר של ניק הרברט, הוא מתחיל בכך שניק אומר “אני אוכיח שזה לא קיים” ולבסוף מוצא הוכחה אלגנטית ביותר לאי לוקליות.

    וגם אנחנו, יובל אתה ואני, לא האמנו בכך וחיפשנו מה לא בסדר: המקטבים? התווך? מערך הניסוי? הרי אף אחד לא מצפה באמת שנאמין שהיקום כולו מקושר עיי איזו “פונקציית גל” מיסתורית, שאינפורמציה זורמת דרכה, ועוד באפס זמן!

    ואף על פי כן אין מנוס מלהכיר במציאות: היקום הינו אי לוקלי.

    נחזור לניסוי הצינורות.

    שני צינורות נעים יחסית זה לזה ופוטון נע יחסית לשניהם. המדידות מראות שהוא נע באותה מהירות בדיוק, זו של האור, יחסית לשניהם. המסקנה של איינשטיין: מהירות האור קבועה לכל מודד, מה שמשתנה הם הזמן והמרחק.

    שאלה: האם זו האפשרות היחידה שקיימת? האם העניין סגור סופית כמו אי הלוקליות?

    התשובה שלי היא שלא. קיימת עוד אפשרות תאורטית: מהירות האור היא רק יחסית לצופה. כמו שלא תוכל לראות קרן אור אינפרא אדום אלא אם כן אתה נע במהירות מסויימת יחסית אליה, כך לא תוכל למדוד אלא את אותו רכיב הפוטון שנמצא במהירות האור יחסית אליך. כל השאר עלומים כלפיך, כמו אור אינפרא או אולטרא. הוא שם – אך שקוף מבחינתך.

    הזוי? אולי. השתנות זמן ומרחק נשמע הזוי לא פחות. יהיה אשר יהיה, שלא כמו במקרה האי הלוקליות, זו אפשרות שקיימת.

    יתרה מזאת: זה מסתדר יפה עם ההנחה שליקום טמפרטורה שהיא פונקציה רציפה של הזמן, ועם גילוי אופיו ההסתברותי והמרוח ביקום של הפוטון, דבר שאיינשטיין התנגד לו נחרצות כל חייו. בשבילו פוטון היה קוואנט של אנרגיה היוצא מנקודה א’ וטס במהירות אור לנקודה ב’, מה שקוראים אצלנו ברפת: פוטון – פוטון.

    האם יש משהו בדברי? קרוב לוודאי שלא. קרוב לוודאי שפשוט קיים הסבר, ושהוא וודאי די פשוט, שאני פשוט לא מודע לו.

    היתרון – ניתן לעשות ניסוי. אני אמשיך כמובן לחפש את ההסבר התאורטי, אבל אמשיך לעבוד גם על רעיונות לניסוי. יהיה אשר יהיה, זה יהיה כף.

    לשאלותיך:

    1. אני מאמין שלפי תמונת היקום ב1905, ההנחה שלו הייתה הגיונית ביותר. היום נראה לי שהוא היה מהסס.

    2. מדי טמפ’ אינם מודדים שינויים לידיעתי. הם מודדדים טמפ’, וזו יכולה להשאר יציבה גם זמן רב. המדחום ימשיך להראות אותה כל הזמן.

    3. הרעיון שלי דיבר על פוטונים, שמתוארים בו כגלים הנעים באתר האקטיבי. הוא אינו מתעסק בשלב זה במסות. שאלתך דומה לכך שאם אציג את תכונותיהם של גלי קול באויר, אצטרך גם להתעסק בתכונות האויר, או כל מדיום אחר לגלי הקול. לא שזה לא רלוונטי לחלוטין, אך יש להבדיל נחרצות בין גל לחומר, בין פוטון לפרוטון, למרות ששניהם נופלים לכאורה תחת הקטגוריה: חלקיקי יסוד. הם לא. מסה יכולה לנוע במהירוית שונות, כולל 0. לגל מהירות קבועה בד”כ, השונה תמיד מ-0 (אלא אם כן הוא עומד). פוטון לפי תאורי הוא גל בלבד, אשר כמו גל קול, נושא עמו תנע ואינו יכול להתקיים במצב מנוחה.

    4. התכוונתי שתוכל אולי להסביר לי מה זה התחום הזה, ומה צפוי בפניה. זה כמו שאם הילד שלך היה אומר לך: אבא אני הולך להיות מנהל מחלקת קרדיולוגיה בקפלן, היית מתמלא גאווה ומתחיל לחסוך, כדי לעזור לו עם המשכורת של ה5000 שקל, שלא כמו אחיו הפלסטיקאי המליונר.

    5. תודה לך וגם ליובל על המזל טוב, אין לי שום פרטים רק אתמול הודיעה לי. כמה שאני מכיר את סנוזי האסטרונאוטית – רק מחקר, שום פרקטיקה פרטית.

    מזל טוב.. על מה בדיוק? על זה שגם בעשר השנים הבאות נראה אותה יומיים בשנה כמו בחמש האחרונות? על זה שנפלנו כמו כולם במלכודת האמריקאית של המרדף אחר ביה”ס הטוב ביותר בלי לחשוב מה עולה בגורלה של המשפחה המתפזרת? איפה השבתות? איפה אבא? איפה אמא?

    ואיפה הממחטה?

    תומר נשאר אנאלפאבית.

  145. שמע ישראל,

    1) לא יודע למה איינשטיין הקדיש את הפרק הראשון לסנכרון שעונים ע”י האור ולא ע”י מד חום. אולי כי חשב שגם הטמפ הזו יחסית? מה אם יש עוד יקומים? אולי כי לא שמע על המפץ? אולי כי היה קר בשוויץ ב 1905?

    2) אם לא אכפת לך גם המדי חום יחסיים הרי אתה בודק בהם את שינו הטמפ מחום רב לנמוך. ביקום עם ללא שינוי עם טמפ’ קבועה לא תוכל למדוד כלום.

    3) האם המודל שלך מבדיל בין חלקיקים עם מסה לאלה ללא מסה? כלומר אם יש מסה אתה מקבל את מהירות האור כגבול העליון וכל היחסות נכונה אולם כשעוברים לפוטונים לפתע הכל משתנה והם יכולים לעבור את C אולם הם כבר אינם נקלטים במכשירנו? ההפרדה הזו נשמעת לי כנפנוף ידיים רציני מאד.

    4) הייטקיסט? סטרטפיסט?

    5) מזל טוב, סוף סוף מישהו עם רגליים באדמה שתרפא את אבא מהשפעת שלו. דרך אגב אצל מי תעשה את הדוקטורט ועל מה?

  146. ר.ח.
    אם אין בעיה, אז מדוע מקדיש איינשטיין את הפרק הראשון של היחסות לבעיית סינכרון שעונים באמצעות קרני אור? למה לא פשוט לשלוף מדחום וזהו?

    לפי הבנתי, לא קיים ביחסות זמן מוחלט וכל שעון איננו אלא מד קצב כפי שטוען יובל. סינכרון באמצעות התרחקות גלקסיות מצריך שימוש בקרני אור וכפוף לכל המגבלות של הסינכרון הדומה אותו מציע איינשטיין, אשר ממנו נובעת התארכות הזמנים, וממנו פטורים שעוני הטמפ’.

    ולא ציינתי שכל הדיון שלנו עד כה התייחס רק לזמנים ופוטונים? שגופים בעלי מסה, בהם מתעסקים בסרן, הם נושא חדש שעלול לארוך עוד 1000 תגובות?

    ושהסתירה המסוימת שהצבעת עליה ממשפט בל עליה אמר איינשטיין עצמו שאיננה יכולה להתקיים כי אי לוקליות תסתור את היחסות? וזה לא שאין הכרעה בינהן – יש הכרעה לכיוון אי לוקליות!

    ימאי, פיזיקאי…

    אגב, לא שאני, אבל מה ההגדרה של בעל דוקטורט במתמטיקה שמתפרנס ממכירת מכשירים רפואיים? רוכל?

    טוב בוא ננצל קשרים. היום קבלתי מייל מהילדה:

    Hi! I am going to the University of Pennsylvania to get my PhD in Cell and Molecular Biology

    אז מכיוון שאתה בתחום, ואני הרבה יותר בקשר אתך מאשר איתה לאחרונה, תוכל להסביר לי במה דברים אמורים?

  147. ישראל,

    לא ענית לאף אחת מטענותי. הסכמתי שאפשר לקבוע זמן לפי דברים שקורים במערכת חיצונית כגון טמפ’ היקום או התרחקות גלקסיות אולם אין אני מסכים שיש בזה כדי לסתור או להוות איזו שהיא בעיה.

    לא ענית כלל ועיקר מדוע זה שלא רואים חלקיקים נעלמים בסרן לא מהווה הפרכה בוטה לתאוריה שלך שגורסת שדברים עוברים את מהירות אולם פשוט אינם נקלטים במכשירנו.

    משפט בל הוא ממכניקת הקוונטים וכולנו יודעים שמכניקת הקוונטים סותרת את היחסות וזו סתירה אמיתית. זו הסיבה שכל העולם מחפש תאוריה מאוחדת.

    אני ימאי כמו שאתה פיזיקאי.

    דבר אחד ששנינו כנראה כן מסכימים עליו הוא ההתקרצצות של הטפיל הזה שמתקרא רפ*אים. אפס מוחלט.

  148. ר.ח.
    שכחתי להזכיר את הנקודה הראשונה שהעלית – את התקצרות הגופים המואצת.
    הסיבה היא שללא התארכות הזמנים, אין בכלל התקצרות גופים. העלתי זאת רק כדי להראות שאפילו אם היא קיימת – היא לא רלוונטית.

    רפאים, אני לא תופס, כמו שאתה אומר תמיד.

    אני לא תופס למה התעלקת על הדיון ביני לבין ר.ח., ענית על שאלה שכאילו שייכת לדיון, רק כדי לחזור ולגלות את אופייך הטפילי.

    מה אתה, איזה וירוס שמשנה את ההרכב הגנטי שלו כדי לרמות את המערכת החיסונית של התא? הרי ברגע שהיא מזהה נגיפים כמוך, היא משליכה אתכם מיד החוצה לחזור למעמד שהיה לכם לפני שהצלחתם להתחזות: עצם דומם חסר חיים.

    אז או שתענה, בפיזיקה, על השאלה שהתיימרת לענות עליה, או שתודה שאין לך מושג על מה מדברים פה בכלל, תעזוב ואולי תרוחם, או שחזרה הבייתה: האפס המוחלט.

    ומכיוון שכל המגיבים פה מירושליים והסביבה, אולי גם שם מקומך. שמעתי שיש שם המון דירות רפאים.

    ולא תפסת מה שאמרתי. אמרתי שאינני פיזיקאי, לא שלא למדתי. דווקא למדתי פיזיקה באחת האוניברסטאות הטובות בעולם. ואתה, אפס?

  149. ישראל
    לא מעט שמו לב שאינך פיזיקאי. רק אתה לא מבין את זה.

  150. ר.ח. יקיר.

    צר לי לאכזבך בפתיחת המילניום השני לתגובות – אינני פיזיקאי. (מישהו כאן כן, אגב? אני מתכוון חוץ מרפאים “ישראל אתה לא מבין”)

    לא שלא למדתי פיזיקה – דווקא כן – אבל לא מספיק כנראה. אחרת לא היינו פה.

    אני מעלה נושא זה פה כמו בפורומים פיזיקלים נוספים בתקווה להיתקל במישהו עם הרקע המתאים, או אפילו לא, שיוכל להצביע על טעות בטיעון.

    עדיין לא הצלחתי למצוא מישהו מוסמך שיהיה מסוגל ומוכן לעבור איתי על הנושא מההתחלה ועד הסוף, כולל בתשלום, כפי שעשית אתה.

    אולם נראה לי שאם המסקנה שהגענו לה שנינו שיכולה תאורטית להתקיים מערכת שתקלוט מהיקום את הזמן האבסולוטי כפי שהסלולרי שלי קולט את הזמן מAT&T , ושזמן זה יהיה מסונכרן אוטומטית עם כל מערכת דומה אחרת – נוגד את הרעיון הבסיסי של היחסות, כפי שהוא מובע בפרק הראשון שלה: “הגדרת הסימולטניות”.

    בקשר לשאלותיו של סוקרטס:

    בכל הדיון הארוך מעולם לא הזכרנו גופים בעלי מסה. רק פוטונים. יש הבדל בין גל, גל קול לדוגמה, אשר הוא חסר מסה אך בעל מהירות, כיוון, ולכאורה נושא עימו תנע, ובין המולקולות בהן הוא “משתמש” כאמצעי תעבורה (אויר, מים, חוט).

    הנוסחה E=MC^2 מתייחסת כמשתמע לגופים בעלי מסה. היא למעשה גלגול של הנוסחה E=PC מימי מקסוול. לפי נוסחת עליית המסה של היחסות תעלה מסתו של של גוף ככל שיקרב למה”א, ותגיע לאינסוף במהירות עצמה. המצב שונה עם טכיונים, שמעולם לא נעו במהירות נמוכה מהאור.

    לכן לשאלתך, אין לי שום בעיה עם מה שקורה בסרן. זהו תחום שונה.

    אם נראה לך שאי אפשר לעבור את מהירות האור – איך אתה מסביר את משפט בל וניסויי אספקט?

    אם נראה לך שאין בהם סתירה ליחסות, אמור, ואשמח לצטט מכר משותף של שנינו שבהחלט חושב שמעבר ספין במהירות העולה על זו של האור עומד בסתירה ליחסות. איינשטיין, במאמר הידוע כפרדוקס EPR.

    אגב, כימאי, אם אתה שם גז עם הספידבואט שלך, ישנו מחסום מהירות אחד שלא תצליח לעבור ולא משנה כמה חזק וגיבור מנוע הסירה: מהירות הקול במים.

    זו גם הסיבה שאין אפשרות כמעט למטוס מונע פרופלור לעבור בכוחות עצמו את מהירות הקול.

    מאיר – לא שכחתי אותך, פשוט הייתי עסוק כפי שעייניך רואות.. אני רק שאלה: שמעת על או יצא לך לקרוא את הספר “UNCOMMON KNOWLEDGE”?

  151. תודה מאיר,
    בשעתו הצעתי ניסוי מקביל לניסוי הבואינגים: ישימו שעון אחד במרתף של בנין גבוה ושעון שני על הגג. הטענה שלי היא שבגלל ההבדלים בכבידה השעון שבמרתף יפגר אחרי השעון שעל הגג. כשהפיגור כבר ייראה לעין יחליפו בין השעונים ואז השעון שקודם פיגר יתחיל כעת למהר. מטרת הניסוי הזה היא להראות שניסוי הבואינגים לא הוכיח שהסטיות בזמן הן בגלל מהירות. המרצה שלי ביטל את הצעתי ואמר שבגלל הבדלי הגובה השעון שעל הגג ינוע מהר יותר סביב מרכז כדור הארץ 🙂

  152. יובל,

    “לא מדויק להגיד ששעונים מודדים זמן. כל השעונים הקונבנציונליים, משעוני שמש, שעוני חול ומים, שעוני מטוטלת ועד שעונים אטומיים, אינם מודדים זמן אלא רק סופרים פעימות של מחזורים המתחוללים בתוכם ומציגים את תוצאת המנייה.”

    כל כך נכון.

    (למרות שאלו שעשו את ניסוי הבואינגים ואלה שמצטטים אותו כאסמכתא למשהו לא יודו בנכונות האמירה הזו גם אם לא יהיה להם משהו חכם להגיד נגדה)

  153. ר.ח רפאי.ם
    כדי להימנע מאיבוד תגובה ארוכה, אני כותב אותה מראש במעבד תמלילים, וזה שאני עושה כעת.
    דיברת על נקודה עצמאית סינגולרית שעל פיה אפשר לסנכרן שעונים. אמנם הדברים הבאים, על זמן ועל שעונים, אינם מיועדים רק לך, אבל היות שאתה היחיד שמגיב כאן לעניין ולא בציניות, אני מצרף אותם לתגובה המופנית אליך.
    לא מדויק להגיד ששעונים מודדים זמן. כל השעונים הקונבנציונליים, משעוני שמש, שעוני חול ומים, שעוני מטוטלת ועד שעונים אטומיים, אינם מודדים זמן אלא רק סופרים פעימות של מחזורים המתחוללים בתוכם ומציגים את תוצאת המנייה. על מנת להגיד כי מוני הפעימות האלה מודדים משהו אוניברסלי עלינו להניח כי משהו חיצוני פועל על כולם במידה שוה. במרוצת הדורות למדנו כי תנאים חיצוניים אכן משפיעים על פעולת השעונים. אם נשוה את תוצאת המנייה של שעון מים או שעון מטוטלת עם מחזור השמש היומי, למשל, נראה כי באזור הקטבים נקבל תוצאות שונות מאשר באזור קו המשוה וזאת בגלל הבדלי הטמפרטורות או השינוי בקבוע התאוצה. באופן דומה גם שעון אטומי מושפע מהבדלי טמפרטורות ועוצמת שדה הכבידה. מתצפיות אנו רואים כי אם מצליחים לשמור על תנאי טמפרטורה וכבידה זהים לשני שעונים זהים, הם אכן יראו את אותה תוצאת מנייה גם אחרי מספר רב של מחזורים.
    כשמדברים על סנכרון שעונים מתכוונים לוודא כי שני שעונים יראו קריאה זהה. בפעולות הדורשות תיאום בין גורמים שונים (פגישות עסקים, טיווח טילים וכדומה), לוקחים בחשבון כי עד לסיום הפעולות המתואמות השעונים השונים יראו מספר זהה או קרוב מספיק לזהה. השאלה הראשונה בהקשר לזמן, האמורה לעניין פיסיקאים (ליתר דיוק, פילוסופים של הטבע), היא מה גורם לכך ששני שעונים הממוקמים במקומות שונים ימשיכו ויראו קריאה זהה. אם נתלה זאת רק במבנה הפנימי של השעון, נצטרך להניח, למשל, כי השעון ניחן באיזשהו חוש טלפאתי המאפשר לו לדעת מה קורה בשעונים אחרים. הנחה כזו היא מאד בלתי סבירה, ואנו דוחים אותה על הסף. מאידך, היות שאנו כבר יודעים כי גורמים חיצוניים משפיעים על פעולת השעונים, יהיה יותר סביר להניח כי קיים גורם חיצוני המשפיע על כל השעונים במידה זהה. כדי להמחיש את ההנחה הזו אפשר להציג את היקום כולו כאילו הוא פועם. לא פעימות עצמאיות ייחודיות לכל מקום ומקום אלא פעימות אחידות בכל היקום כולו. כדי להמחיש זאת, נוכל להביא משל מעולם האנטומיה והרפואה: כדי שמערכת הדם תתפקד ביעילות, פעימות הלב והעורקים צריכות להיות מתואמות; התיאום הזה מושג באמצעות תאים מיוחדים היוצרים אותות חשמליים; כאשר התאים האלה חדלים לתפקד כיאות, אנו יכולים להחליף אותם בקוצב מלאכותי העושה את אותה עבודה. אם פעימות היקום מתואמות באמצעות קוצב כזה, אפשר לראות בו את הנקודה הסינגולרית שאליה התכוונת.

  154. ישראל,
    אכן קושיה יפה שאלת, אתה מתאמן לפסח? אז ככה:

    1) כפי שאתה יודע אני לא פיזיקאי וידיעותי מגרדות את קצה ידיעותיך אולם לעניות דעתי הטעות כאן היא במשפט “דוד ישראל חישב שבמהירות כזו נמוכה ההתקצרות כה זניחה, שיקשה לרשום אותה כאן”. כי גם הזניחה הזו תביא לכך שלא תהיה סתירה.
    בוא נסתכל מה יקרה אם צינור ב’ ינוע במהירויות קרובות למהירות האור. האם שם אתה מסכים שמה שיקרה זה שאורך הצינור יילך וייתכווץ והזמן ינוע לאט יותר מב-א’ ולכן אין סתירה?

    אם כן, באינדוקציה פשוטה זה גם מה שיקרה במהירות נמוכה וההבדל שאתה קורא “זניח” הוא זה שיעשה את ההבדל.

    2) סוקרטס שואל:
    האם מה שקורא במאיץ של סרן או בכל מאיץ אחר לא מהווה הפרכה לתאוריית החלקקיקים הנעלמים מעל מהירות האור?
    לפי איינשטיין – E=MC^2 כלומר לא משנה כמה אנרגיה תשקיע לא תצליח לעבור את מהירות האור ומה שיקרה זה רק שהמסה תעלה בריבוע. תשקיע אינסוף אנרגיה תקבל אינסוף מסה וה- C יישאר C.

    לפי שפירא – השקעת אנרגיה בחלקיקים תאיץ אותם למהירויות גבוהות מעל מהירות האור ואז מה שיקרה זה שהם ייעלמו כי החיישנים המיסכנים שלנו לא ייראו אותם לפי עיקרון המטוטלת הבליסטית. כך שאם נזרוק 100 פרוטונים ונאיץ ומאיץ ע”י המגנטים המפלצתיים שיש במאיץ לאחר מספר סיבובים לא רב מספר הפרוטונים יילך ויירד.

    עכשיו השאלה, מה לדעתך רואים בסרן:

    1) עלייה במסת הפרוטונים?
    2) העלמות פרוטונים ושמירה על המסה של אלה שנעדיין נקלטים?

  155. אין בכוונתי להיכנס שוב למודל שלי אלא רק להתיחס לשאלה שהתבקשתי אליה בעקבות שימוש שעשיתי ב”נוטוף”:
    https://www.hayadan.org.il/astronomers-reach-new-frontiers-of-dark-matter-130112/#comment-332405
    בשעתו, לפני תגובות רבות, הצגתי את הפוטון כפיסת חלל ריק. כאשר חלקיק נכנס לתוך המקום בו נמצא פוטון, הוא מותיר אחריו פוטון. באופן הזה תנועתו של הפוטון מתקיימת הדדית עם תנועתו של חלקיק בכיוון ההפוך.

  156. רפאים.
    אני צריך ללכת לישון עכשיו. כשיבואו האלו עם החלוקים הלבנים, לא להתפרע.
    ובאמת אין צורך שתגיב לתגובותי. ראינו בדיוק מה אתה מסוגל להפיק.
    למה שלא תנסה משהו פשוט יותר, אולי רקמה?
    לילה טוב.

  157. ישראל
    ברים? תל ברוך? לא לכולם יש אותן העדפות כמו שלך.

    מעבר לזאת, רציתי להגיב על תגובתך, למעלה, אבל אחרי שהבנתי מה רשמת שם הבנתי שגם אין צורך לחזור על דברי כי אתה פשוט לא תופס.

  158. רפאים.
    אחד מתנאי השחרור המוקדם שלך היה שתנצל את החופשה ללמוד מתמטיקה ופיזיקה. טוב, במקרה שלך – חשבון וטבע.

    לפי תגובתך האחרונה אנו נאלצים להסיק בצער שבזבזת את זמנך בברים ובתל ברוך, ולכן נאלצים להחזיר אותך למשכנך הקבוע – המחלקה הסגורה.

  159. ר.ח.

    בוא נראה אם אנו יכולים להסיק אילו מסקנות ממה שהגענו אליו עד עתה.

    עוברים לצינור.

    שהוא ארוך, ארוך כאורך הגלות. כהרגלינו, הכל מוגזם. בוא נגיד 100 שניות אור. קוטרו – 5 מטר. יסומן כא’.

    כל 100 מטרים מותקן ההתקן הידוע הכולל שעון צז’, שעון טמפ’ ומצלמה חדת רזולוציה. כל שעוני הצז’ מסונכרנים בינהם בדרך שהציע איינשטיין במאמר המקורי על היחסות. הם מכוילים להראות את אותו הזמן כמו שעוני הטמפ’ שכידוע לכל מסתנכרנים עצמאית.

    בתוכו צינור זהה אך בעל קוטר 4 מ’. נסמן אותו בב’

    כל המתקן בשדה הניסויים הקבוע שלנו בטיזנבי.

    א’ נמצא במנוחה מוחלטת לפי כל הסטנדרטים.

    ב’ יכול לנוע חופשית בתוך א’, לאורך הציר המשותף שלהם.

    מהלך הניסוי:

    1. אנחנו מושכים את ב’ מעט החוצה מא’, ואחרי שעה דוחפים אותו כך שמהירותו יחסית לא’ היא 3מ/שנ.

    מה שיוצא זה שב’ מגיח מימין ובזמן שמסומן בT0, אחורי שני הצנורות מתלכדים. ב’ ממשיך במסעו בתוך א’.

    2. בזמן T0 משוגר פוטון בודד לאורך הציר המשותף של הצינורות.

    כל הארוע מצולם בזכור, אך מכיוון שהכל יודעים עד כמה ביישנים ופגיעים הם פוטונים, שום דבר לא מגיע לפוטונינו ופונקציות גלו, למען לא תקרוס לנו באמצע הניסוי.

    עכשיו:

    פוטונינו עושה את דרכו לאורך הציר בשלווה ובנחת. הוא אינו מודע כלל לעובדה שהוא נע כעת יחסית ל2 מערכות הנעות במהירות 3 מ/שנ זו לזו.

    3. פוטונינו מגיע לקצהו השני של א’ אחרי 100 שניות בדיוק לפי שעוני א’.

    4. פוטונינו גם מגיע לקצהו השני של ב’ אחרי 100 שנ’ בדיוק לפי שעוני ב’.

    5. אבל כשפוט’ מגיע לקצה של ב’, ב’ כבר בולט כ300 מ’ מהצד השני של א’.

    אני לא אומר בדיוק 300 מ’ בגלל התקצרות המרחקים. דוד ישראל חישב שבמהירות כזו נמוכה ההתקצרות כה זניחה, שיקשה לרשום אותה כאן. בחצי C ההתקצרות היא כ13%. כאן – שבריר שברור.

    הפואנטה:

    6. אם מקבלים את ההנחה שכל שעוני הטמפ’ מראים תמיד את אותו הזמן בשני הצינורות, ושבגלל שהמערכות אינן מואצות שעוני הצז’ מראים את אותו הזמן כמו שעוני הטמפ’, הרי שכל צילום של 4 שעונים ביחד מא’+ב’ יראה תמיד את אותו הזמן בכל השעונים.

    7. מכיוון שפוט’ הגיע לקצה של ב’ בדיוק באותו הזמן שהוא הגיע לקצה של א’, ולקח לו קצת זמן לעבור את כ300 המטרים הנוספים, אין מנוס מהמסקנה שהוא נע מהר יותר יחסית לב’ מאשר לא’.

    8. עד כאן הגזמנו בכוונה באיטיות, כדי להקל על האינטואיציה. זה יעבוד באותה המידה בכל מהירות אחרת.

    9. זה יעבוד גם אם ב’ ינוע בכיוון ההפוך. כך נוכל לקבל פוטונים איטיים כרצונינו.

    10. זה די מסתדר עם התאור שיש לנו של פוטון כפונקצית גל המרוחה במרחב.

    11. זה פותח פתח להבנת אי הלוקליות.

    12. זה יכול להסביר את תוצאות ניסוי “הבחירה המושהית” של ווילר בלי להזדקק לנורא מכל – חזרה בזמן. (פרטים – לפי בקשה).

    מלבד התנגדותך הרגילה לעיקרון ההאינווריאנטיות, שא’ נע יחסית לב’ כמו שב’ נע יחסית לא’, כלום תוכל למצוא פגם בטיעון?

    קראתי כל מיני ספרים על איינשטיין וראיתי גם סרטים. אני גרופי שלו לא פחות מגלי.

  160. ישראל
    ההסבר הוא: על ידי מערכת ייחוס שונה מקרני אור או רדיו.
    כמו למשל נקודה סינגולרית דמיונית שעליה מסתכלים אותם חיזרים שהגיעו מהיקום שבו אתה כנראה חי.

  161. ישראל,

    דוקא לי אישית הרבה יותר קשה להבין מה זה יקום נצחי שהיה תמיד. אינטואיטיבית יקום עם התחלה נשמע הגיוני (לא שההגיון משחק פה איזה תפקיד). מה היה לפני זה? לא היה לפני זה. אתה זוכר מה היה לפני שנולדת? זה מה שהיה לפני המפץ הגדול. או דרך אחרת להסביר היא כמו שאמר ידידנו רפאי*ם באחת ההברקות שלו “טמפרטורה שמתחת לאפס המוחלט”, זה מה שהיה לפני המפץ הגדול 🙂

    לגבי הביוגרפיה של איינשטיין מעניין שבדיוק חבר שלי המליץ לי על זו של וולטר איזקסון (שבעצמו הוא סיפור מעניין) ואני אנסה להשיג אותה. האם זה מה שקראת?

  162. ר.ח.
    4K? בדקתי במחשבון. זה יוצא בערך לפני 6.5 מיליארד שנה. נראה לך שהיו כבר חלליות ממוכנות אז, או שעדיין השתמשו במשוטים?

    לגבי שקלול רכיב נוסף לנוסחת פרידמן – קטן על אפל.

    לגבי יחסות ומפצגדול

    וידוי אישי – מת על יחסות וגם מת על איינשטיין אישית. אינני יודע אם קראת את הביוגרפיה שלו, או את כל אמרותיו בנושאים חובקי יקום. הוא שם בכיס הקטן את ניוטון רע המזג או מאך האסטרונאוט. הוא גם יותר צנוע ופשוט מפיינמן המיתולוגי, למרות שהוא ממש ממש גאון.

    ואף פעם לא הצלחתי להבין אינטואיטיבית איך זה שלא היה כלום לפני המפץ ולמה היקום איננו אינסופי. אבל זה מה שאומרים המבינים, ושתי התורות אינן מסתדרות זו עם זו לדעתי. אין לי שום העדפה בין השתיים.

    נראה לי שהייתי יכול להוכיח לך זאת כבר עתה באמצעות מה שהסכמנו עליו, אילו היית מקבל את שקילותן של כל המערכות האינרציאליות. אני יודע שזה לא נראה מובן כלל, אך מבחינת ניוטון או איינשטיין, דג השוחה באוקינוס במהירות קבועה בהחלט יכול לראות עצמו כנח והאוקינוס ועמו כל כדה”א נע. אותו דבר לגבי מטוס או כדור רובה. כל מי שבתנועה לא מואצת, נמצא למעשה במנוחה. תנועה היא אך ורק יחסית למערכת אחרת.

    ניתן להתקף החום הנוכחי לחלוף, ואראה לך מדוע לדעתי אם תקבל את עיקרון היחסות הראשון, אז באמצעות מה שכבר הסכמנו עליו ניתן לסתור את התארכות הזמנים.

    רפאים.
    מה היה ב t=0?
    עד כמה שידוע לי אף אחד לא יודע.

    אבל השאלה שלי הייתה:

    1. האם תוכלנה החלליות לסנכרן את שעוניהן כך שהם יראו את אותו הזמן בלי להשתמש בכל אמצעי קשר כגון קרני אור או רדיו?

    ועל זה ענית כן. תוכל להסביר איך?

  163. על פי מה שקורה בסופר נובות, ההגיון אומר שאחרי המפץ הגדול חלק נכבד מהחומר שהיה במקור נדחס פנימה חזרה לגרעין הסינגולרי והעודף התפזר ביקום שלנו.
    לא ברור לי למה בכל הכתבות ותוכניות המדע שקראתי לא מדברים על אפשרות של חומר שנדחס לגרעין בתור חור שחור או שם סינגולרי אחרץ

  164. ישראל
    אתה לא מבין את הפואנטה.
    האם אתה יכול לענות מה היה מצב החומר ביקום בזמן t=0?
    ידוע שהייתה קרינה כלשהי שעברה טרנסופרמציה לחומר בזמן שאינו שווה לt=0.
    זאת אומרת, ברגע שהייתה התערבות כלשהי מצד ג’ כלפי הקרינה, באותו הרגע השתנתה הקרינה לחומר.
    לרגע הזה (תחילת המפץ) ניתנו הגדרות שכוללות את המושג (המופשט) זמן, בעקבות שינוי במצבו של החומר/קרינה/מרחב. וההגדרה קובעת שהזמן לא היה 0 ברגע התהוות היקום.
    בקיצור, (אתמול התחלתי לכתוב כאן תגובה וכשהיא היתה באורך מגילת אסתר בטעות נמחקה. אז עכשיו כתבתי בקצרה, ואם יש לך שאלות אשתדל לענות.) תמיד תהיה מערכת ייחוס כלשהי ביקום הזה.

  165. סטודנט, טכניון, על לא דבר
    ואם יש לך, או לכל מי שקורא את הדברים, דעה לחוות או הערה להעיר בנושא הנדון, אנא אל תחסכו במילים. מדובר בסתירה מוחלטת לכל מה שאנחנו מכירים על מושג הזמן. אחרי שכבר למדנו להסכין עם הרעיון המגולם בתורת היחסות, שלפיו הזמן נע בקצב משתנה אך בכל זאת שומר על כיוון קדימה, קמים בקרבנו כאלה המנסים לשכנע את הציבור התמים כי הוא אף נסוג לאחור. גם אם תרדו לפסים אישיים זה בסדר מצדי, היות שזה בדיוק מה שאני עושה כאן בעצמי לחלוץ הרעיון.

  166. יובל, רק רציתי לדעת בשם מי אתה מדבר. תודה על התשובה.

  167. סטודנט, טכניון
    נראה לי כי הפעם, שלא כהרגלי, הייתי מאד ברור. “אנחנו” זה כולנו, פיסיקאים והדיוטות כאחד. אפילו אני, המתיימר לפענח את כל סודות היקום עם המודל המסתורי שלי, לא באמת יודע.
    אם אתה סבור שאני טועה, כלומר שלדעתך יש מישהו המבין היטב את מהותו והתנהגותו של הפוטון ו/או אלקטרון ויכול להסביר על פיהן את התופעות הלא ברורות דלעיל, אנא הסגר אותו בידינו.

  168. סטודנט, טכניון
    שאלתך מזכירה לי את ישעיהו ליבוביץ’ עליו השלום. באחד הראיונות, כשהמראיין אמר “אנחנו…”, ליבוביץ’ אמר “מי זה אנחנו? אנחנו זה אני והתולעים שלי!” 🙂
    מותר לך להסתפק בזה, אם תחפוץ, אך הרשה נא לי לשאול אותך משהו. האם יש לך באמתחתך הסבר לכל התופעות הידועות הקשורות באלקטרונים ו/או פוטונים (למשל תוצאות הניסוי שהציג ה”ה ישראל שפירא למ.ר ס.ט) המתבסס רק על מה שידוע לך על המבנה שלהם ואופן התקדמותם במרחב?

  169. מ.ר ס.ט
    אנא הבא בחשבון שיתכן קיום תופעות שאיננו מכירים. למשל, כאשר פוטון נע ממקור אור אל מטרה, משהו אחר, לצורך העניין נקרא לו “נוטוף” (פוטון הפוך), נע בוזמנית מן המטרה אל מקור האור. באופן הזה, חסימת המסך אחרי שהפוטון עבר לא השפיעה על הפוטון שעבר אלא הסיטה את הנוטוף מן המסלול הקצר אל מסלול אלטרנטיבי.
    כאמור, אשמח להתכתב איתך ישירות. תמצא את כתובתי כאן:
    https://www.hayadan.org.il/astronomers-reach-new-frontiers-of-dark-matter-130112/#comment-332386

  170. ישראל,

    לגבי נוסחת פרידמן הטענה שלי פשוטה. הנוסחא כנראה נכונה למצב מנוחה עד תאוצה מוגבלת. במהירויות גבוהות תצטרך להוסיף לה רכיב נוסף. כמו שנוסחאות ניוטון נכונות עד למהירויות הגבוהות שאז מוסף להם הרכיב היחסותי.

    סוקרטס:

    אם אתה רואה סתירה בין תורת היחסות למפץ הגדול (שאודה ואתוודה שלא השתכנעתי שיש כזו) אז מה טענתך? שלא היה מפץ או שתורת היחסות אינה נכונה? (להבנתי אתה נגד היחסות אבל אני רוצה להיות בטוח שאנחנו On the same page).

    לגבי השאלות למעלה:
    1) כן, 100 חלליות יוכלו להסכים על הזמן. יוכלו לקבוע פתיחת אש ב 4 קלווין אפס אפס. מסכים.

    2) מה זה רלוונטי? 1905? יקום אינסופי? לא מבין.

    שפעת – יש חיסון לא רע שמופץ כל שנה. לקחת? האמת שקראתי שהשנה הוא לא היה מוצלח במיוחד. הבעייה עם שפעת שכל שנה הוא משתנה ויש זנים חדשים. עוד לא הצליחו ליצור חיסון אוניברסאלי לכל הזנים אלא רק פר זן כל שנה. למזלנו המחלה מתחילה להתפרץ במזרח אסיה ונודדת מערבה כך שאירופה + אמריקה יש זמן ליצור כל שנה את החיסון הרלוונטי. הבעיה שלעיתים יש הפתעות והחיסון שהכינו לא מתאים בדיוק לזן שהתפרץ או שהוא עבר שינוי בדרך.
    אבעבועות שחורות זה החיסון היחידי בהסטוריה שאכן הביא להכחדה מוחלטת של איזו שהיא מחלה. יש לכך סיבות רבות ביניהן שלאבעבועות שחורות אין מאכסן פרט לאדם כך שאם חיסנת חיסלת. חוץ מזה הוירוס לא משתנה ביעילות. קל לייצר חיסון (ג’נר השיג אותו מוירוס דומה של פרות). צרוף מקרים כזה לא חזר מאז.

    דרך אגב היום אדוארד ג’נר היה נכנס לכלא, אז הוא נחשב לגיבור.

  171. יובל,

    “איננו מבינים לאשורו את מבנה הפוטון ו/או האלקטרון וגם את אופן התקדמותו במרחב.”
    – כשאתה אומר “איננו”, על מי אתה מדבר?

  172. בוקר טוב.

    נראה שמתחיל קצת להתבאר האופק. אפילו הצלחתי לאכול תפוז. נקווה שזה סוף הסיפור הזה של הגריפה.

    רפאים שמתי לב שבתגובתך ענית על 2 בכן. תוכל להסביר?

    ר.ח.

    את הסתירה להתארכות הזמנים אני מאמין שאנחנו מלבנים פה כעת. היחסות היא הרבה יותר מהתארכות הזמנים גרידא. אם תצליח אתה, או מישהו, להראות לי היכן הטעות שלי, אודה ואעזוב בשמחה. מקווה שגם ארוחם.

    אני מאמין שציר הזמן הוא קצת יותר מציר. אם זכורה לי נכונה, הוקינס כבר אמר שהמפץ הגדול יצר את הזמן, שלא היה קיים לפניו. באמת אינך רואה את ההבדל היסודי בין הזמן ב1905, שנמשך ממינוס אינסוף לאינסוף, לתמונת הזמן שלנו היום?

    אבל אינני רוצה להתבדר לחזית נוספת. אנחנו יכולים בהחלט לראות אם נגיע למסקנות אם נמשיך בכיוון שהתחלנו בו.

    לגבי הניסוי המחשבתי שלך – אין לי מושג מה תהייה ההשפעה. אני מבין את הלוגיקה, קיטום ממדים. אך אפילו איינשטיין לא התייחס לזמן כמימד זהה ל3 האחרים.

    לגבי נוסחת פרידמן – אינני מבין את טענתך. האם אתה טוען שאי אפשר לדעת את כמות השניות שחלפו מאז המפץ הגדול רק עיי מדידת הטמפרטורה? שהמחשבון הקטן שבתחתית הלינק שגוי? זו אם כן הסתייגות חדשה לגמרי, אבל האמנתי שסגרנו את הנושא מזמן.

    בוא נחזור לעיקר. אם נסכים עליו, נוכל להתקדם:

    1. לפי איינשטיין, נוסעים שונים בחלל שנפגשים בינהם ולכל אחד שעון המראה זמן שונה, לא יוכלו בשום צורה שהיא לקבוע מהו הזמן האמיתי. אם חייזריו של חנן יחטפו אותך, יאיצו אותך על כל רחבי הגלקסיה ואח”כ יעזבו אותך בחללית תועה בלב החלל, לא תוכל בשום צורה לדעת מהו הזמן בכדה”א, אלא אם כן תיצור עימו קשר. הוא בהחלט כבר יכול להקיף ננס לבן.

    2. לפי מה שדיסקסנו פה, אפשר גם אפשר. מד קרינה, מחשב, ויאללה הביתה.

    אם תסכים עם 2, נוכל להתקדם.

    מ.ר.ס.ט.

    אם עדיין לא השתכנעת, לך ל

    http://www.youtube.com/watch?v=3A6ageOaS-E

    קיימים הסברים אלטרנטיביים, אך למיטב ידיעתי הם אינם מיינסטרים.

    יובל

    אחרי ה”שוטף מוחות” “סרסור מילים” ועוד ביטויים ססגוניים – מה זה בשבילי “זורע מסכי עשן צבעוניים וריחנים”?

    רוצה להיות ברוגז – אין בעיה. רק בבקשה לא להתחיל יותר עם ישראל.
    אני מבטיח חגיגית שלא אתחיל איתך.
    ואגב – לא זכור לי שנלחצתי אי פעם בכתבה זו או אחרות, ואינני חושב שהתעלמתי מנושא כלשהו, אלא אם כן ביקשו ממני במפורש להתעלם. אבל מי אני שאדע?

    העיקר שסיימנו.

  173. ישראל
    אין לי בעיה עם זה שתכה אותי חזק פי עשר או פי כמה שתמצא לנכון. יש לי רק בעיה “קטנה” אחת איתך: אתה מתעלם מדברים שאינם זורמים בכיוון הרצוי לך וזורע מסכי עשן צבעוניים וריחנים כשאתה לחוץ. אחוקים כמוך מיותרים לי.
    “בעגלא ובזמן קריב” זה גם מהקדיש ופרושו “במהרה בימינו”.
    ועל כל ישראל ואמרו אמן

  174. מ.ר ס.ט
    עלינו להיזהר ולהשתדל לא לקפוץ למסקנות חפוזות. איננו מבינים לאשורו את מבנה הפוטון ו/או האלקטרון וגם את אופן התקדמותו במרחב. הסבר אפשרי אחד לתוצאות הניסוי המדובר הוא כי הגורם הנמדד אינו מחליף בין המצבים גל וחלקיק אלא הוא שומר על שני המצבים בוזמנית בכל רגע נתון.
    אשמח להרחיב על המבנה וההתנהגות של החלקיקים האלה על פי מודל שבניתי:
    [email protected]

  175. מ.ר ס.ט

    לך ל

    ftp://ftp.biu.ac.il/pub/physics/optics/optics12_YoungMichelson.pdf

    סקרול למטה עד שתגיע ל”בחירה מושהית”
    שים לב לפסקה הבאה:

    “אולם ההחלטה אם להזיז את המסך או לא התקבלה רק אחרי מעבר
    האלקטרון בסדקים. נראה אפוא כאילו, שלאחר שהאלקטרון כבר עבר את הסדקים, הוא עדיין חופשי “להחליט”
    בדיעבד אם הוא עבר אותם כחלקיק או כגל, וזאת על פי החלטתנו איזה סוג של מדידה לבצע”

    שים לב שנערכו ניסויים רבים, בעיקר עם פוטונים, שאישרו תוצאות מוזרות אלו. המסקנה המקובלת היא שאנו יכולים להשפיע על העבר מן העתיד.

  176. ר.ח.

    נוכל להרחיב אחרי שאיישן את המשפחה החולה.
    ובאמת זו בושה וכלימה שבמאה ה21, אחרי שהצלחתם למצוא סידור נאות לאבעבועות שחורות עדיין לא גמרתם על השפעת.

    בינתיים – סוקרטס.

    1. האם אתה מקבל את זה שבאמצעות ההתקן אותו אני מכנה “שעון טמפ'” אם 100 חלליות תחלופנה בו זמנית זו על פני זו במהירויות שונות, ובכל חללית יש מצלמה חדת רזולוציה, הרי שהצילומים המשותפים של כל מאת השעונים בכל אחד מ100 הצילומים יראה את אותו הזמן בכל השעונים שבחלליות?

    2. האם תוכל לעשות את אותו התרגיל עם 3 חלליות לפי היחסות ב1905?

    הנה הזמנים של שעוני הצזיום של חלליות איינשטיין:

    א. 1905.
    ב. 17 טריליון ו83 שנים בעבר.
    ג. 564, 876, 456 שנים 165 ימים 14 שעות ו43 שניות בעבר.

    יובל.
    אשמח לחזור להיות אחוקים אם תוכל לדבוק בענייניות ולהמנע מפגיעות אישיות. אחרת, אקבל בהכנעה את גורלי המר, ולא נדבר.

    “בעגלא ובזמן קריב” – זה לא מהקדיש?

    מ.ר ס.ט

    יש בהחלט דבר כזה. אנסה למצוא לך קישור נאות עוד היום, או שאסביר בעצמי. כרגע אני קצת גמור.

    רובי.

    הבת שלי שמתחילה השנה את למודי הדוקטורט, טוענת שחוץ מהעניין החברתי, אוניברסיטה זה כל כך המאה ה20. כל הרצאה נורמלית שתשמע באוניברסיטה תוכל למצוא באינטרנט, ו20 טובות ממנה. כל מערכי השיעור, התמיכה, התרגילים, היועצים, הכל הכל שם. תגגל כל נושא שתרצה, מצא לך הרצאה כלבבך ביוטוב, ובניגוד לאוניברסיטה אמיתית, תמיד תוכל לחזור על כל נקודה כמה פעמים שתרצה, או אם אינך מבין מושג, לעצור את הרכבת ולחזור אליה אחרי שהתבאר לך הכל.

  177. על מימדי הזמן והמרחב
    המשך מ: https://www.hayadan.org.il/astronomers-reach-new-frontiers-of-dark-matter-130112/#comment-330693
    כפי שאנו מכירים מן האריתמטיקה, מהותם של המספרים ה”שליליים” היא ביטוי של פעולת החיסור כקשר חד מקומי (או קיצור ל”אפס פחות מספר”). באופן דומה, גם השלילה היא קשר חד מקומי (הביטוי “לא משהו” הינו קיצור ל”אמת וגם לא משהו”) וכזה הוא הדין גם עם שורש השלילה. וכפי שאנו מכירים מן הפעולות במספרים מרוכבים, גם החלה חוזרת של שורש השלילה היא בעלת מחזוריות מרובעת: שורש השלילה ← שלילה ← שלילת שורש השלילה ← חיוב (שלילת שלילה), ושוב: שורש השלילה ← שלילה ← שלילת שורש השלילה ← חיוב, וחוזר חלילה. החיוב הינו איבר ניטראלי, משום שהחלה עליו של כל קשר חד-מקומי מניבה את הערך של הקשר החד-מקומי המוחל. במחזור הזה מתקבלים ארבעה ערכים שונים, שאחד מהם הוא ניטראלי. אולם אותם ארבעה ערכים חוזרים על עצמם ללא הגבלה במחזורים הנפרדים זה מזה. את ההבדל בין מחזור אחד למחזור הבא אנו מכנים בשם “זמן”. הזמן הוא חד מימדי וארוך לאינסוף ולכן מימדי היקום הם אינסופיים, אך בגלל המחזוריות מתקבלים ארבעה מימדי מרחב שאחד מהם ניטראלי או “מנוון”. על כן המרחב (אם נתעלם מאינסופיות הזמן) הינו תלת-מימדי.

  178. התבלגנה התגובה הקודמת.
    כל החלק שאחרי שבת שלום צריך להיות בין “האם היחסות תשתנה כהוא זה” לבין “לגבי נוסחת פרידמן”

  179. ישראל,

    נו אז מה? את כל זה אנחנו כבר יודעים. איינשטיין הניח ב 1905 שהיקום אינסופי בזמן ואנחנו על “רשת” אינסופית של ציר X ציר Y וציר Z מרחביים וציר הזמן. בא המפץ הגדול והראה שלפחות לציר הזמן יש ראשית ציר. כלומר t0.
    מזה אתה מסיק סתירה לתורת היחסות?

    ניסוי מחשבתי: נניח שיום אחד ימצאו קיר שחוסם את היקום מבחינה מרחבית, מעין קליפה המקיפה את הכל (כמו בסיפורים הישנים והטובים בפנטסיה 2000 ז”ל) האם היחסות תשתנה בכהוא זה?

    לגבי נוסחת פרידמן, זו נוסחא תאורטית שחושבה על סמך המון הנחות שאחת מהן היא שההתקררות זהה בכל מקום ורק הזמן משפיע עליה אז מה שאתה אומר זה טעון מעגלי. אני בטוח שרק הזמן משפיע על ההתקררות כי נוסחת פרידמן שמניחה את זה אומרת שאין פקטור נוסף. נו באמת?

    בכל מקרה אני מאחל לך החלמה שלימה ושכלאבי לא יגמור כמו הכלבה לייקה, כלבם של בני בסקרוויל או לד הזקן שכמעט נטרף.

    יובל, תשובה לא נכונה. לא סליל. קו ישר.

    שבת שלום

    1) אם כן , כיצד?
    2) אם לא, מדוע המפץ נמצא בסתירה איתה?

    עכשיו בוא נחשוב לצד השני. נזיז את ראשית ציר הזמן לסוף. נניח שנגלה שהיקום לא באמת מתרחב אלא מתכווץ לקראת הקריסה הגדולה. האם זה ישנה ליחסות?

  180. אחלה דיל, שפעת כוכבים. בעיקר העניין הזה של אובדן התאבון. אם זה ימשך עוד שבועיים אחזור לגזרה שהייתה לי כשהייתי בן 20.
    ור.ח. שפעת כלבים אינה ישימה לגבי כלאבי. כלאבי איננו אנדרדוג רגיל, כזה שחולה, מזדקן, או מת. כלאבי הוא כלבלב מיתולוגי, בסדר גודל של הכלב לד, לייקה, או כלבם של בני בסקרוויל. לא הוא יתרגש מכמה וירוסים מסכנים.

    רפאים, ההסתייגות שלך במקום, אך מינורית. אם למדת פיזיקה, בטח זכור לך שתמיד עושים אידאליזציה של מערכות: מתעלמים מהחיכוך, מהקרינה, מהזברבירים. ההבדל איננו משמעותי, וניתן לשיקלול מאוחר יותר. נתתי את דוגמת האיזור השכוח בחלל בעיקר כדי להתגבר על התנגדותו של ר.ח. להנחה שכל מערכת שאינה בתאוצה נמצאת בעצם במנוחה. חוץ מזה שאפשר לבנות באופן תאורטי דגם מעבדתי של אותו איזור, שבו יתקיימו כל התנאים שתיארתי, רק הרבה יותר קטן, מה שלא ישנה לעצם העניין.

    יובל, אתה עוכר שמחות. אבל כיוון שהפנית, אז נכון: הפיסקה שהבאתי היא מתוך המאמר המקורי של איינשטיין שנודע ברבים כתורת היחסות הפרטית:

    http://www.fourmilab.ch/etexts/einstein/specrel/www/

    אולם שימו לב בעיקר לפיסקה הבאה מתוך המאמר:

    must bear carefully in mind that a mathematical description of this kind has no physical meaning unless we are quite clear as to what we understand by “time.” We have to take into account that all our judgments in which time plays a part are always judgments of simultaneous events.

    והגידו לי אתם: זה לא כל כך 1905? של תמונת עולם של יקום אינסופי ונצחי, שבו דברים תמיד היו ותמיד יהיו. שבו אתה יכול בשקט לחשוב על ארועים מלפני טריליון טריליון שנים, כפי שחשב פואנקרה כאשר הוכיח את הפיכותה של האנטרופית היקום בזמן, בשעה שסטודנט טכניון שלנו, הוכיח שהיא אינה הפיכה אפילו למערכת הקטנה בהרבה מסדר הגודל של מס’ אבוגדרו במשך חייו של היקום.

    כי לפי תפיסת העולם ב1905, באמת אינך יכול לדעת מהו הזמן בנקודה שאינך מחובר אליה ישירות. אין דבר כזה זמן מוחלט, ולכל מודד יש את הזמן הפרטי שלו. לכן, אם תמשיכו בקריאת המאמר, מציע איינשטיין לסנכרן את הזמן בין שתי נקודות באמצעות קרני אור, שמהירותן זהה לכל מודד, ומכאן הוא מסיק את התארכות הזמנים והתקצרות המרחקים.

    אולם לנו יש את היתרון שלא היה לאיינשטיין ב1905: לפי תאוריית המפץ, אין צורך לסנכרן כלל בין הזמנים של מערכות: הם מסונכרנים באופן טבעי, וכל שעלינו לעשות הוא למדוד אותם, כפי שהצעתי באמצעות שעוני טמפ’.

    לפי איינשטיין, כאשר מאה תאומים שנפרדו בעבר נפגשו שנית, אין דבר כזה שלאחד מהם “הזמן האמיתי”. כולם צודקים. שונה הדבר לפי תאוריית המפץ הגדול. אפשר בהחלט לדעת מהו הזמן האמיתי, ואפילו למדוד אותו. ההסתייגות של ר.ח. בעניין התנועה היחסית של מערכות, ניתנת להדיפה בקלות יחסית עיי הדגם החדש של שעוני הטמפ’, שבו משוקללת המהירות היחסית בפלט הזמן. זה ישים גם להסתייגותו של מייקל הנפקד לגבי “החזרה בזמן” של שעוני הטמפ’.

    ובעניין שאלתו של ר.ח. על פקטור ההתקררות – התשובה היא נוסחת פרידמן והמחשבון הקטן שמצורף בתחתית הלינק שנתתי לך. אילו היה פקטור נוסף, הנוסחה לא הייתה עובדת.

    נעשה הפסקה לעיכול רעיונות וביקורת. אם דיברתי שטויות רבות מדי, התירוץ שלי הוא המדחום, שמראה על 104 פרנהייט.

  181. ר.ח,
    הייתי לוקח קופסה בצורת גליל ומגלגל את הדף מחוצה לה באלכסון כך שכל שורה (חוץ מהראשונה) תהיה המשך של השורה שקדמה לה. אמנם הקו שאחבר בו את כל הנקודות יהיה, למעשה, סליל, אך מנקודת המבט של המישור המסוים הזה הוא יהיה ישר.
    על ג’ק המרטש אני לא יודע הרבה. ראיתי כמה סרטים שהשתמשו בדמות שלו (אחד מהם לקח אותו קדימה בזמן לרטש עלמות חן במאה העשרים: 1979 ,”Time After Time”). מעניין אזכורו של רוצח ראש הממשלה בהקשר של פנחס: בין מי שמעריץ את פנחס למי שמעריץ את הרוצח הזה יש הרבה קוים אידיאולוגיים משותפים.
    נ.ב. כשנרצח רבין מתה לי המדינה.

  182. יובל, יש לך המלצות איפה אני יכול לקחת קורסים אקדמיים בנושאים אלו? בהשכלתי אני מהנדס. אני מתעניין מאוד באסטרונומיה, חורים שחורים וכדומה.

  183. יובל,

    תעשה לי טובה, חשיבה מחוץ לקופסא. באותה מידה אתה יכול להגיד שג’ק המרטש או יגאל עמיר חשבו מחוץ לקופסא.

    חשיבה מחוץ לקופסא זה איך אתה מחבר 9 נקודות שמצויירות על דף בצורה של 3X3 , ע”י קו ישר אחד.

  184. ר.ח,
    פנחס חשב מחוץ לקופסא. לפעמים צריכים כאלה אנשים. בעוד משה וכל עדת בני ישראל עמדו פתח אהל מועד ובכו, קם פנחס ועשה מעשה. זה כמו, להבדיל, מעשה אלכסנדר מוקדון בביצת קולמבוס …. אופססס…. סליחה. התבלבלתי.

  185. רובי
    נכון. אנו מצפים לגלות “גן חיות” לא קטן של חלקיקים. אתה “חוטא” בניסויי חשיבה, כמו כולנו כאן. Welcome to the club. מה שאתה אומר, שההפרש בין המהירויות האמיתיות למהירויות היחסיות מיתרגם לעודפי אנרגיה (בהתנצלות על הניסוח הכושל שלי) יכול בהחלט לשמש תזה יפה למחקר מעניין, ולא אתפלא אם מישהו כבר חשב על זה לפניך. תורת היחסות הכניסה הרבה פרדוקסים לחיים שלנו, אבל בהצבה נכונה של המספרים בנוסחאות המתאימות אפשר ליישב את הכל. נראה לי שבמתמטיקה אני לא יותר חזק ממך. אנא הצב את הנתונים במקומות הנכונים וספר לי ולשאר הקוראים למה הגעת.

  186. יובל,

    לאור איך שאהרון חינך את נכדו פנחס שהפך לסמל של קנאות ורצחנות דתית הוא לא בדיוק המודל שלי.

    http://he.wikipedia.org/wiki/כהני_פינחס

    מעבר לזה שדמותו של אהרון עצמו לא משהו. הרי הוא זה שהוביל את חטא העגל ובסוף בגלל קשריו בהנהלה הוא יצא זכאי לגמרי בעוד ש- 3000 אחרים נטבחו. אל תביא לי גם את שמואל ששיסף את אגג בהתלהבות או את אליהו שטבח את כוהני הבעל או אלישע שחיסל ילדים שקצת צחקו לו על הקרחת.
    אם אתה מתעקש אז אולי שאול או שמשון הבליין התמים.

  187. ישראל,
    נכון לעכשיו, לא כיף לי לדבר איתך.
    רק אאחל לך החלמה מהירה ושתהיה בריא עד מאה ועשרים – בעגלא ובזמן קריב.

  188. יובל, תודה על התשובה אבל בכל זאת מצפים מההתנגשות למצא חלקיקים פיזיקליים כשבניהם בוזון היגס המפורסם.
    מה שאני רוצה להגיד שתוצר ההתנגשות הוא אנרגיה שמפרקת את הפרוטונים המתנגשים בעוצמה של MC**2 לכל פרוטון ואילו המהירות של כל פרוטון היתה חצי C העוצמה היתה MC**2/4 לכל פרוטון. זאת אומרת שאולי הפרוטון שנע במהירות C לא “רואה” את הפרוטון המתנגש במהירות 2c אבל מרגיש את עצמת ההתנגשות כאילו הוקטורים של המהירות מסתכמים.

  189. ר.ח,
    תודה על מאמציך 🙂
    יש סיפורים על אהרן הכהן שהיה אוהב שלום ורודף שלום ומשכין שלום בין הבריות. אם תתרום דגימת ד.נ.א, אהמר על כך שאתה מצאצאיו 😛

  190. מ.ר ס.ט
    אין לך מה להתאמץ. אין הסבר כזה. הדיונים שאנחנו עורכים כאן הם בעיקר ניסויי חשיבה. בדיונים כאלה מתייחסים בדרך כלל למספר מצומצם של עובדות או מסקנות ומתעלמים מכל השאר. בצורה כזאת אפשר להגיע למסקנות מעניינות ומפתיעות, אך הכל רק בתאוריה ולרוב מופרך.

  191. רובי
    השאלה שלך פותחת פתח לדיון מאד מעניין על מה בעצם מואץ במאיצים. למעשה, הפרוטון המואץ הוא גורם זניח. הוא משמש רק “כלי רכב” קטן שעליו נוסעת אנרגיה רבה. אם נקבל את נוסחת הקשר בין מאסה ואנרגיה (E=MC^2) נראה כי מה שנוסע שם הוא בעיקר אנרגיה. אחת המסקנות מתורת היחסות היא שגוף הנע במהירות מתכווץ. במהירות האור תביא התכווצותו של הגוף לכך שגודלו יהיה אפס. אם הפרוטון יואץ עד למהירות האור, הרי כל מה שינוע במערכת יהיה רק אור.
    כאשר שני פרוטונים מואצים כאלה מתנגשים, תוצר ההתנגשות ביניהם הוא זניח לעומת תוצר ההתנגשות בין שתי אלומות האנרגיה שהאיצה אותם. התשובה לשאלה שלך, מדוע מאיצים את שני הפרוטונים במקום להאיץ רק אחד ולהשאיר את האחר נייח, היא כי מטרת הניסוי היא ליצור התנגשות בין שתי אלומות אנרגיה.

  192. אני בהתקף חום שבטח יקח כמה שעות. הייתי רוצה שתקראו את הפסקה הבאה, ותראו אם זה מסתדר עם הרעיון של זמן מוחלט, שאפשר אפילו למדוד אותו באמצעות המתקן אותו אני מכנה “שעון טמפ'” או אפילו ההצעה שלך ר.ח. של התרחקות הגלקסיות, או כל גורם חיצוני אחר. העיקר שנוכל לסנכרן שעונים בלי כל קשר בינהם.

    § 1. Definition of Simultaneity

    Let us take a system of co-ordinates in which the equations of Newtonian mechanics hold good.2 In order to render our presentation more precise and to distinguish this system of co-ordinates verbally from others which will be introduced hereafter, we call it the “stationary system.”

    If a material point is at rest relatively to this system of co-ordinates, its position can be defined relatively thereto by the employment of rigid standards of measurement and the methods of Euclidean geometry, and can be expressed in Cartesian co-ordinates.

    If we wish to describe the motion of a material point, we give the values of its co-ordinates as functions of the time. Now we must bear carefully in mind that a mathematical description of this kind has no physical meaning unless we are quite clear as to what we understand by “time.” We have to take into account that all our judgments in which time plays a part are always judgments of simultaneous events. If, for instance, I say, “That train arrives here at 7 o’clock,” I mean something like this: “The pointing of the small hand of my watch to 7 and the arrival of the train are simultaneous events.”3

    It might appear possible to overcome all the difficulties attending the definition of “time” by substituting “the position of the small hand of my watch” for “time.” And in fact such a definition is satisfactory when we are concerned with defining a time exclusively for the place where the watch is located; but it is no longer satisfactory when we have to connect in time series of events occurring at different places, or—what comes to the same thing—to evaluate the times of events occurring at places remote from the watch.

    We might, of course, content ourselves with time values determined by an observer stationed together with the watch at the origin of the co-ordinates, and co-ordinating the corresponding positions of the hands with light signals, given out by every event to be timed, and reaching him through empty space. But this co-ordination has the disadvantage that it is not independent of the standpoint of the observer with the watch or clock, as we know from experience. We arrive at a much more practical determination along the following line of thought.

    If at the point A of space there is a clock, an observer at A can determine the time values of events in the immediate proximity of A by finding the positions of the hands which are simultaneous with these events. If there is at the point B of space another clock in all respects resembling the one at A, it is possible for an observer at B to determine the time values of events in the immediate neighbourhood of B. But it is not possible without further assumption to compare, in respect of time, an event at A with an event at B. We have so far defined only an “A time” and a “B time.” We have not defined a common “time” for A and B, for the latter cannot be defined at all unless we establish by definition that the “time” required by light to travel from A to B equals the “time” it requires to travel from B to A. Let a ray of light start at the “A time” $t_{\rm A}$from A towards B, let it at the “B time” $t_{\rm B}$ be reflected at B in the direction of A, and arrive again at A at the “A time” $t’_{\rm A}$.

    In accordance with definition the two clocks synchronize if
    \begin{displaymath}t_{\rm B}-t_{\rm A}=t’_{\rm A}-t_{\rm B}. \end{displaymath}

    We assume that this definition of synchronism is free from contradictions, and possible for any number of points; and that the following relations are universally valid:—

    If the clock at B synchronizes with the clock at A, the clock at A synchronizes with the clock at B.
    If the clock at A synchronizes with the clock at B and also with the clock at C, the clocks at B and C also synchronize with each other.

    Thus with the help of certain imaginary physical experiments we have settled what is to be understood by synchronous stationary clocks located at different places, and have evidently obtained a definition of “simultaneous,” or “synchronous,” and of “time.” The “time” of an event is that which is given simultaneously with the event by a stationary clock located at the place of the event, this clock being synchronous, and indeed synchronous for all time determinations, with a specified stationary clock.

    In agreement with experience we further assume the quantity
    \begin{displaymath}\frac{2{\rm AB}}{t’_A-t_A}=c, \end{displaymath}

    to be a universal constant—the velocity of light in empty space.

    It is essential to have time defined by means of stationary clocks in the stationary system, and the time now defined being appropriate to the stationary system we call it “the time of the stationary system.”

  193. אני יחסית חדש כאן ולא הבנתי איך ניתן לחזור לעבר ולשנות אותו. ישראל שפירה אמר עתמול בשעה 3:03 שאפשר לעשות את זה בעזרת מכניקת הקוונטים, אבל אני לא היצלחתי למצוא את ההסבר.

  194. ישראל,

    “חללית, בלב ליבו של חושך מצריים, מסוגלת לדעת מהו הזמן על פי מדידות עצמאיות שעשתה. הזמן הזה יוצג על לוח השעונים של החללית כמספר של שניות שחלפו מאז המפץ הגדול.” ==>

    כן. בתנאי שלא תתחיל להתפרע במהירויות מעל למותר. אם היא תתחיל לנוע היא תצטרך להשתמש במודיפיקציה של נוסחת פרידמן שלך שהוסף לה גם רכיב יחסותי המתייחס לשינוי בזמן לפי מהירות. כל עוד היא נחה זה רכיב זניח והנוסחא תקיפה. מסכים?

    “אם החללית תשדר את המספר הזה באמצעות רדיו, והוא יקלט עיי חללית אחרת במרחק שעת אור מן החללית המקורית, הרי שהוא יתאים בדיוק למספר המוצג על לוח השעונים של החללית השניה, אליו הגיעה בדרך זהה לדרך שנקטה החללית הראשונה, אחרי קיזוז השעה המבדילה בינהן.” ==>

    שוב, בתנאי שאף אחת מהן לא זזה. אם אחת מהן נעה היא חייבת להתחשב בכך בחישוב הזמן לפי קצב שינוי הטמפרטורה.

    1. מסכימים בתנאים וההגבלות שציינתי.

    2. לא. אבל אז אפשר היה למדוד לפי משהו אחר. קצב התרחקות גלקסיות, קצב דעיכת האור של כוכב זקן או כל דבר אחר שמשתנה לאורך זמן. נכון ששום מדידה לא תהיה לנצח אולם גם התקררות הקרינה שלך לא תימשך לנצח אלא תךף אסימפטוטית ל-0 המוחלט עד שאי אפשר יהיה למדוד אותה יותר.

    3. לא ענית לי מה הביטחון שלך שהפקטור היחידי המשפיע על ההתקררות הוא הזמן?

  195. ר.ח.
    לפי נוסחת פרידמן, הזמן קשור לטמפרטורה בנוסחה שהיא פונקציה רציפה. אפילו עם יש פרמטרים נוספים, במקרה שאנו דנים בו הם אינם באים לידי ביטוי.

    מכיוון שהגענו להסכמה לגבי 2, אני רוצה לחדד נקודה זו לפני שנמשיך:

    חללית, בלב ליבו של חושך מצריים, מסוגלת לדעת מהו הזמן על פי מדידות עצמאיות שעשתה. הזמן הזה יוצג על לוח השעונים של החללית כמספר של שניות שחלפו מאז המפץ הגדול.

    אם החללית תשדר את המספר הזה באמצעות רדיו, והוא יקלט עיי חללית אחרת במרחק שעת אור מן החללית המקורית, הרי שהוא יתאים בדיוק למספר המוצג על לוח השעונים של החללית השניה, אליו הגיעה בדרך זהה לדרך שנקטה החללית הראשונה, אחרי קיזוז השעה המבדילה בינהן.

    1. מסכימים?

    2. האם הטריק היה עובד ביקום איזוטרופי ונצחי שבו הטמפרטורה קבועה תמיד?

  196. ישראל,
    1) לא יודע אולי אפשר לחשוב על איזו הברקה פרט לטמפרטורות שלך. כל דבר מחוץ לחללית שמשתנה לאורך זמן יעזור להם. אטומים רדיו אקטיביים עם זמן מחצית גבוה שייאספו בחלל?, ריכוז אבק בינכוכבי?. אני מניח שגם בערבות הכי רחוקות יוכלו לראות במרחק קוואזרים ולבצע איזו שהן מדידות.

    2) כן.

    לפני שאתה מתחיל להתווכח איתי על התשובות בסעיף 1. “הרי אמרתי נאדה! זיפ! כלום אין אבק ואין זרביבים” אני רוצה שתדע שהתשובה המדוייקת לא מעניינת וככל הנראה מה שכתבתי שם לא נכון אולם בודאי אפשר לחשוב על משהו. וגם אם לא, כל כוונתי היא רק להראות לך שמדי החום שלך, גם אם זו הברקה נחמדה ואולי אף שימושית בעידן הטיסות הביגלקטיות המתרגש עלינו לטובה, הם לא מיוחדים. הם מודדים פרמטר מעניין מחוץ לחלליות שממנו אפשר לחשב זמן. ביג דיל. מזה להגיד שיש סתירה בתורת היחסות?
    חוץ מזה, אם להיות קטנוניים, האם אתה בטוח שהטמפרטורה דועכת אך ורק לפי זמן? אולי יש עוד פרמטרים? צפיפות כוכבים? כבידה בסביבה? אולי הקרינה לא באמת מפוזרת בצורה אחידה?

  197. “הכמעט” הזה הוא עקרוני מאוד, כפי שראית בחצי התשובה הראשון. אם השקעת כמות אנרגיה X כדי להגיע למהירות יחסית של 0.99C בין הפרוטונים, הרי שאם תשקיע עוד 1000X, תגיע למהירות יחסית של נגיד 0.999999C. אם תעשה נכון את חישובי האנרגיה לפי משוואות לורנץ, תראה שבהבדל הקטן לכאורה של 0.009999C קבורה אנרגיה עצומה של 1000X, פי 1000 מאשר כמות האנרגיה שנדרשה כדי להביא את הפרוטונים למהירות יחסית של 0.99C.
    וכדי להגיע לC עצמו, תזדקק ללא פחות מאינסוף אנרגיה.
    מבין אולי מדוע רעשו סיפי ארץ כאשר נראה היה שאיזה נייטרינו חצוף העז לעבור את מהירות האור? בלתי אפשרי, אליבא ד’יחסות.

  198. ישראל, אני חוזר לפיזיקה בסיסית: Ft=mV תקיפה ותנע. V אמור להיות הסכום הוקטורי של שני הגופים (פרוטונים).
    לפי מה שכתבת, שלא משנה המהירות מעל חצי C לכל פרוטון, הסכום הוקטורי יהיה תמיד כמעט C , ולכן תמיד התוצאה Ft תהיה זהה.
    אם כך מספיק להאיץ אותם לחצי C ואין טעם להשקיע אנרגיה נוספת?

  199. רובי

    יש הבדל גדול מאוד בין האנרגיה של פרוטון ב0.99C והאנרגיה שלו ב0.999999999C יכול להגיע בשקט לאלפי מונים.
    החלק השני של השאלה לא ברור לי.

  200. ישראל, תודה על התשובה אבל לא ברור לי למה משקיעים כל כך הרבה מאמצים להאיץ את הפרוטון לכמעט מהירות האור כדי שלבסוף המהירות היחסית תשאר כמעט מהירות האור.
    לגבי אנרגית ההתנגשות לא ענית אם היא תהייה זהה במהירות של חצי C ועד C של כל פרוטון.

  201. ר.ח.
    נראה לי שכל מחשב נורמלי יוכל לחשב את הזמן שחלף מהמפץ אילו החללית הייתה במנוחה. מחשבים של מקינטוש יעשו זאת אפילו מהר יותר. זה תופס גם אם כל מה שאנחנו מודדים בעצם הוא את ספקטרום קרינת הרקע. פשוט, אתה מודד את טמפרטורת הקרינה, מודד את מהירותך יחסית לקרינה, ומקזז. נוסחת פרידמן, והנה הזמן המדויק.

    אבל לא משנה. בוא נגיד שבאיזור מסויים של החלל, בערבות הנצחיות המשתרעות בתחום הדמדומים שבין הסופי לאינסוף, אין כוכבים, אין ערפיליות, אין זברבירים. רק חושך על פני תהום. כלום. נדה.

    בוא נגיד שלאותו איזור מקולל נקלעות שתי חלליות הנמצאות במרחק שעת אור זו מזו. מהירותן 0 יחסית זו לזו וגם 0 לפי התקנים המחמירים שלך כלפי כל מערכת חיצונית. (שו מערכת? אין שם כלום!).

    הצוות בכל חללית שולף שעון קוקיה חדש ונוצץ מהאריזה. השעונים לפי הוראות היצרן, מראים זמן 0 ומתחילים לפעול רק כשלוחצים על הצ’ופצ’יק למעלה.

    סוקרטס שואל:

    1. האם תוכלנה החלליות לסנכרן את שעוניהן כך שהם יראו את אותו הזמן בלי להשתמש בכל אמצעי קשר כגון קרני אור או רדיו?

    2. האם הן תוכלנה לעשות זאת עיי שימוש בשעוני טמפ’?

  202. רובי, יש את טרנספורמציות לורנץ, ולפיהן ניתן לחשב את המהירות היחסית של גופים המתקרבים למהירות האור.

    קח דוגמה קיצונית: חללית א’ טסה יחסית לארץ במהירות של 0.9C. חללית ב’ טסה במהירות של 0.9C יחסית לא’, באותו כיוון. ג’ באותה מהירות יחסית לב’, וכך עד חללית ת’. מה תהיה מהירותה של ת’ יחסית לארץ?
    התשובה היא שקצת פחות מC.

    ולכן מהירותם היחסית של הפרוטונים שלך תהיה בין 0.7C בערך, עד כמעט C, תלוי במהירותם.

    הסיבה למגבלת מהירות האור הינה לוגית בעיקרה. ניתן להראות שאם גוף עובר את מה”א, התוצאה תקדם לסיבה. אילולי משוואות היחסות, אכן כל גוף היה רואה את השני כנע במהירות שהיא סכום וקטורי פשוט, כמו אצל ניוטון, ולכן מעל מה”א.

  203. ישראל,

    0. http://www.alfavet.co.il/kennel_cough.html

    1. לא. דיברנו על זה, אני אקח את החום שלך כתירוץ לזה ששכחת. אם החללית נעה במהירות קרובה למהירות האור היא תדווח על זמן קצר. אם היא במהירות כמו שלנו היא תדווח על 13.7 מיליארד שנה. כלומר הדיווח יהיה תלוי מהירות. הרי איך השעון הזה עובד? הוא מודד את הטמפ הנוכחית, מניח את טמפ’ t0. ואז לפי קצב ההתקררות שנמדד קודם לכן במערכת (וזה בדיוק האלמנט תלוי המהירות!) השעון יחשב את הזמן.

    2. כן. הם יכולים להחליט שכש T=X כולם נפגשים. הראשון יגיע אחרי שעה מבחינתו, השני אחרי יומיים (מבחינתו) והשלישי אחרי שנתיים של נדודים (שוב מבחינתו) אבל כולם ייפגשו בזמן אחד כש T=X. על הש”ג בכוכב הסמוך יעברו 50 שנה אבל גם הוא כשיראה ש T=X הוא יצטרף.

    ושוב אני אומר, אין בטמפרטורה הזו שום דבר מיוחד. יכלו באותה מידה להחליט להיפגש כשהמרחק בין גלקסיה A ל B שמתרחקות זו מזו יהיה Y. גם אז כל אחד ייסע במהירות שלו יקרע את כל החלל אולם כששתי הגלקסיות תגענה למרחק הרצוי כולם ייפגשו.

    3. לא.

    4. תעשו מה שאתם רוצים. אני לא בורר בחינם ולא גננת בהתנדבות.

  204. ישראל, הבנת נכון, התנגשות של פרוטון עם פרוטון כשכל אחד מואץ בבכוון הנגדי במהירות האור (כמעט).
    אם לא היתה מגבלה של מהירות האור אזי כ”א היה רואה את המהירות היחסית שהיא כפול 2 אבל בגלל מחסום מהירות האור אתה טוען כך?
    מה כ”א היה רואה אם מהירות כל פרוטון היתה חצי ממהירות האור, גם כן אותו דבר?
    מה היה התנע של ההתנגשות? אם מהירות כל פרוטון היתה בין חצי למהירות האור?

  205. ר.ח.
    תפסתי כמה רגעים שקטים בין התקף חום למשנהו. שאלה, בתור אחד שמבין בוירוסים: איך זה שכל המשפחה חלתה חוץ מכלאבי? אולי זה הכיוון?

    אני מצטער שאינני מצליח לשכנע אותך בשקילותן של כל המערכות האינרציאליות, ושאין דבר כזה מערכת ש”נעה” באמת. אבל אפשר להמשיך גם בלי זה, בעזרתו של סוקרטס קשישא.

    1. האם אתה מסכים שחללית בחלל הריק תוכל לדעת מהו הזמן המדוייק שחלף מאז המפץ באמצעות המתקן אותו אני מכנה “שעון טמפ'”?

    2. האם באמצעות אותו מתקן תוכלנה כמה חלליות, אחרי שקרעו את כל הגלקסיה בתאוצות נוראיות, לכל הכיוונים, שהוציאו לגמרי את שעוני הצזיום שלהם מסינכרון, לתאם התקפת פתע משולבת על פלנטה, בשעה שללא שעוני הטמפ’ אין לא היו מסוגלות?

    3. האם אתה מסכים שלפי היחסות בלתי ניתן לתאם התקפה כזו?

    ובענין יובל – ראית כמה פעמים ביקשתי ממנו שירד ממני, הצעתי לו “תהדיה” החתמתי אותו על הסכם שלום, מה אני יכול לעשות אם הוא ממשיך? הייתי מאוד רוצה שהדברים יתנהלו על מי מנוחות, אך לא נראה לי שהוא מסוגל.

    רובי – התנגשות עם מה? אם אני מבין אותך נכון, ההתנגשות היא זה עם זה. התשובה להבנתי היא קצת פחות ממהירות האור.

  206. שאלה לפורום הנכבד,
    במאיץ החלקיקים LHC שב CERN מאיצים פרוטונים למהירות של 99.9999991 אחוז ממהירות האור, חלק עם כוון השעון וחלק נגד כוון השעון.
    השאלה היא איזה מהירות כל פרוטון רואה לפני ההתנגשות וכן מה התנע שנוצר בהתנגשות, של מהירות האור או כמעט כפול ממהירות האור.

  207. ישראל,
    איך יש לי זמן לעשות משהו אם אני מתעסק כל היום עם קרני אור בלתי נראות במהירויות גבוהות?
    דרך אגב גם לי הייתה שפעת עד לפני יומיים.

  208. חטפתי זן אלים של שפעת כוכבים. נמשיך אח”כ.
    ר.ח., לא היית אמור למצוא חיסון לשפעת??!?

  209. ישראל,
    2) למזלי ההשגות שלי לרעיונות שלך אכן נתמכות ע”י המיינסטרים וזו הסיבה שאני מרגיש על קרקע מוצקה אחרת לא הייתי מתווכח כל הרבה בנושא שאני לא שולט בו ב- 100%. לכותך ייאמר שלמדתי המון המון מהדיון הזה. ייתכן כמובן שאני ורוב הפיזיקאים בעולם טועים אבל תצטרך לעבוד קשה מאד ולהוכיח בניסויים עם אורים ותומים טעות שכזו.

    – אני אשמח לשמוע את הרעיונות שלך לגבי הזמנים והשפעות עבר עתיד והקשר שלהן לרעיון הקרניים השקופות שלך.

    3) מסכים עם כל מילה ולכן גם אין שום סתירה כמו שיסכימו איתי אותם אישים נכבדים שהזכרת. דרך אגב שים לב שאף אחד מהם לא טען שמערכת הייחוס משפיעה של המערכת הנלמדת כמו שאתה טענת.

    6) אבל אתה עונה במו כתיבתך על השאלה שלך. שים לב שכתבת “קרון רכבת אטום”. מה זה אטום? מנותק מכל מערכת ייחוס. אתה גם כנראה תסכים שאם הקרון לא אטום יש דרך על סמך מערכת ייחוס חיצונית בצורת עץ לקבוע האם אתה זז או לא. כנ”ל ג’ק/ג’יל שיכולים להסתכל על אנדרומדה/ביטלג’וס או טמפרטורה חיצונית לחללית שלהם.

    7) גם פה פיספסת את הנקודה שלי. אני לא מערכת משפט ולא חוש צדק טבעי ולא מעניין אותי מי התחיל ומי החזיר. אני רואה את עצמי כידיד של שניכם ונעים לי להתכתב עם שניכם אולם וחורה לי שאתם מתנהגים כמו שני ילדים. בצבא היינו לוקחים אותכם לאוהל סיירים בצאלים, שני לילות, והייתם נהיים אחוקים.

    8) גם פה יש ים (שחור משחור) וסקי (בלי שלג השנה..) אז אל תתנשא.

    9) אני מצטרף לקריאתו של מרסט, אתה לא צריך OK שלי. דרך אגב יובל לא בריון הוא רק רצה התייחסות עניינית לדבריו.

  210. ישראל תתעלם נא מיובל הבריון ואל תמתין לאוקיי של רח. תמשיך עם הרעיון שלך כי הוא מרתק

  211. ר.ח.
    ממתינה.

    יובל
    הכשרון שלך לחיות במציאות וירטואלית מעפיל לשיאים חדשים כל פעם. אתה הוא זה שמתחיל, ואתה הוא זה שמנבל. קח אחריות לשם שינוי. עבור על השירשור. ראה אם תצליח להצביע אי פעם על הנקודה שבה אני התחלתי איתך או ניבלתי את פי. אולי גם תמצא בהזדמנות את החישובים שיש לך על מולקולת מימן.

    ואין לי שליטה על כך שאתה קשה הבנה. ההפניות שנתתי לך מצויינות. אילולי שרפת את הגשרים שלך איתי, כמו שעשית פה עם כולם, הייתי עובר איתך לאט על כל הרעיון. אבל עכשיו, לך חפש את החברים שאין לך.

  212. ר.ח.

    2. הייתי שמח להשאר בתחומי המיינסטרים, אך מה לעשות שלדעתי התאוריות לא מסתדרות זו עם זו? לא גמרנו את הדיון שלנו בענין שעוני הטמפ’. אם נמשיך, אחד מאיתנו אולי ישכנע את השני. אם לא, אני מחוייב להמשיך עם התאוריה ולמצוא פירושים שנראים לי הגיוניים.

    אם תרצה, אוכל להסביר לך כיצד לפי מכניקת הקוואנטים ניתן מן ההווה להשפיע על העבר, וכיצד הרעיון שלי יכול להוציא אותנו מהתסבוכת.

    3. הרעיון שכל מערכת היא עצמאית ואין דבר כזה נע או נח איננו שלי. הוא של גלילאו, ניוטון ואיינשטיין. זהו למעשה הפוסטולט הראשון ביחסות. בדוגמת הGPS שלך, הסירות נעות יחסית למערכת הGPS. אילו הייתה מערכת GPS נוספת, הקוארדינטות היו משתנות בהתאם. אם ג’ק וג’יל יבחרו את אנדרומדה, זו תהייה מערכת יחוס שרירותית חדשה וכל אחד יצטרך לעשות את טרנספורמציות הקוארדינטות הדרושה כדי לעבור למערכת זו. התוצאות אגב, תהיינה זהות לחלוטין.

    שים לב אגב, שמכיוון שלא סיימנו את דיונינו, במערכת ג’ק – ג’יל אין צורך בגורם נוסף חיצוני.

    6. אינני מבין איך יכולים הדברים להיות לך ברורים, אם הגענו למסקנה ששעוני הטמפ’ והצזיום חייבים להראות אותה הטמפ’ במערכת אינרציאלית, אחרת זה יסתור את הפוסטולט הראשון של היחסות:

    עקרון היחסות:

    חוקי הפיזיקה אינם משתנים כאשר עוברים ממערכת ייחוס אינרציאלית אחת למערכת ייחוס אינרציאלית אחרת. כך לדוגמה, אדם הנמצא בקרון רכבת אטום לא יכול באמצעות שום ניסוי או מדידה פיזיקלית לקבוע האם הקרון נע במהירות קבועה או ניצב במנוחה.

    הרי לפי דבריך כל מה שצריך לעשות הוא להסתכל על לוח השעונים. לפי יחס הסיבובים בין שעון הטמפ’ והצזיום תדע בדיוק אם אתה נע, ובאיזו מהירות. (יחסית למה, אגב?).

    7. לפי ההגיון שלך, הבריון של הגן והילדים להם הוא מרביץ מתנהגים כמו ילדים קטנים, לא? עובדה, הם הולכים מכות. או אנס וקורבנו וכו.

    את מערכת המשפט ואת חוש הצדק הטבעי מעניינת מאוד השאלה מי התחיל ומי החזיר. הראשון תוקפן. לכלא. השני מתגונן. זכאי. אפילו אם השני החטיף לראשון מכות חזקות הרבה יותר.

    בעימותים ביני לבין יובל, יובל ת מ י ד מתחיל, זאת למרות שהוא התחייב מפורשות לא לעשות זאת. הדרך היחידה להתמודד עם בוליס זה להחטיף להם חזק בלקקן, לפני שילפתו עכוז יותר מדי. ע”ע הנודניק הנוסף שעזב אותנו כבר.

    ואל תחזיק לי יותר מדי מהחוף המזרחי שלך. הדילמה העיקרית בפני בני החוף המערבי היא שכשקמים בבוקר – טוב בוא נהיה מדוייקים, ב11 לפנות בוקר, ואחרי שהתמתחו עד 12 ואכלו בראנץ עד 1, היא לאן הולכים היום: לים (חצי שעה נסיעה) או לסקי (שעה).

  213. סטודנט, טכניון,
    אתה צודק לחלוטין. זה באמת לא עובד ככה. אני מציע שנניח לזה לפי שעה ובהזדמנות אפרסם את הכל בצורה מסודרת. כעת אני הולך לחפש מה זה “אורך קשר”. תודה. להתראות

  214. יובל,

    “ההסברים קיימים, אך בינתים אתה מתבקש לסמוך רק על המילה שלי.”

    – לא סומך. זה לא עובד ככה.

    “אולי לא הבנתי את שאלתך על אורך הקשר.”

    – אז תחפש מה זה אורך קשר (bond length).

    ומצטער, עדיין לא הבנתי אילו חישובים יש לך על מולקולת מימן.

  215. ישראל,
    2) זו תצפית שאיינשטין שאר העולם ואתה מסכימים עליה. הבעיה היא בפרשנות שלך של המכוניות השקופות ואלה שלא הגיעו. אבל ניחא אם תראה לי שלפרשנות הזו יש ייתרון בהסבירה תופעות כמו השפעת העבר על העתיד ( מה הכוונה בדיוק??) דיינו.

    3) זו נקודה שנראה לי שאתה לא מפנים. כל עוד יש שתי מערכות אתה צודק, כל אחת נחה והשנייה נעה ביחס אליה. אולם אם נוסיף מערכת ייחוס נוספת כמו העץ בחוץ או במאדים ניתן יהיה לדעת מי נוסע ומי נח.

    שאלה: איך אתה יכול לציין מקום של נקודה ביחס לנקודה אחרת ללא מערכת ייחוס? עם מערכת ייחוס אתה יכול לתת קוראורדינטות X Y או פולריות (מרחק + זוית מציר ה X). אבל אתה חייב מערכת ייחוס שרירותית.

    עוד דוגמא – שתי סירות בלב אוקיינוס. מי מהן נעה ומי נחה? לא ניתן לדעת. אולם אם שני מלחים בסירות מסתכלים ב- GPS, כלומר במערכת ייחוס לווינית ניתן לדעת בדיוק מי זזה ומי לא. זה למה המציאו מערכות ייחוס או צירים לגרפים.

    כנ”ל לגבי דוגמת החלליות שלך. ג’יל וג’ק יכולים להסכים שהקרינה הקוסמית או גלקסיית אנדרומדה או כל גורם אחר מחוץ לחליות שלהם שאינו מושפע מהן, יהווה את מערכת הייחוס. בעזרת מערכת הייחוס הלז הם ידעו בדיוק מי זז ומי לא.

    לגבי העשן. אתה צודק שהייתי צריך להדגיש שעשן יסמן מערכת מואצת. דאון שיוציא עשן יישאר בתוך ענן העשן גם אם יתקדם ואילו מטוס מאיץ ימשוך פס עשן מאחוריו. חשבתי שזה ברור.

    6) אני לא מסכים שיש סתירה או בעייתיות בנושא ועוד לא הצלחת לשכנע מדוע הפיזיקאים הסתבכו בו. לי ההדיוט הדברים נראים ברורים. אבל מה אני מבין? ביולוג, אתה יודע.

    7) מה שכן אני מבין זה שאתם מתנהגים כמו שני ילדים קטנים. הוא התחיל, הוא אמר לי ככה והוא ענה לי ככה. לא נאה ולא יאה ולא לכבודכם. יאללה תרימו כוס יין וירטואלית (ולא איזה מיץ מעמק הנאפה שלך, איכס, משהו אמיתי, ארופאי) ותחזרו לדון כמו שני בני תרבות אינטלקטואלים חובבי מדע.

  216. סטודנט, טכניון,
    תודה על מתן הפתח לטיהור שמי. ההסברים קיימים, אך בינתים אתה מתבקש לסמוך רק על המילה שלי. פשוט, הם מתבססים על הגדרות שזמנן טרם הגיע.
    אולי לא הבנתי את שאלתך על אורך הקשר. מן הסתם חשבתי שאתה מדבר על הרדיוס הקוולנטי של האטומים המעורבים בקשר. את זה קשה לחשב באטום של הכלור אך קל (יחסית) לחשב באטום המימן. את הגרעין של אטום המימן, הפרוטון, תיארתי כמושבה של חלקיקים המקיימים יחסי גומלין עם סביבתם. עדיין לא הבאתי חישוב של סדר גודל של מושבה כזו וטרם הראיתי איך מתקשר לשם האלקטרון ואיך נוצרים הכוחות המשמשים בקשרים בין האטומים. המלאכה על המודל מרובה והזמן העומד לרשותי קצר ואין לי עוזרים. מצטער.
    אין לי בעיה עם רפרנסים, אבל הוא לא הביא כאלה כי אם שלח אותי לחפש אותם בעצמי ולהגיע בסופו של דבר למאמר המפריך את טענתו. לאור זה, ראוי היה שיגיד שטעה, אך במקום זאת הוא מסיח את תשומת לב הקוראים מן הפשלה שלו ומפזר ניבולי פה (אמנם בשפה גבוהה, כראוי ל”מתורבת” שכמותו, אך ניבול פה הוא ניבול פה) ומוציא ממני כל חשק להמשיך לדבר איתו.

  217. יובל,

    “השאלה אם שיקרתי או לא נתונה לפרשנות.”

    – ממש לא. אם היו לך הסברים למה שכתבתי כשכתבת זאת, לא שיקרת. אם לא היו לך, שיקרת.

    מכל מה שכתבת בהמשך, לא הבנתי אילו חישובים יש לך על מולקולת מימן.

    “השיח כאן הוא ציבורי. אם אתה מפני אותי לקישורים וחיפושים, הרי אתה מטריח לא רק אותי אלא גם את כל מי שעוקב.”

    – מה לעשות, במדע רפרנסים הם חלק מדיון. ראית פעם מאמר שלא מפנה לקישורים?

  218. יובל.
    כולם כאן מפנים לקישורים. סטודנט הפנה אותי להרבה קישורים רק בכתבה זו. בניגוד לך, אני טורח לקרוא אותם.

    emission theory of light מסביר יפה את תוצאות מ-מ, וזה הדבר היחיד שאמרתי. ברור שיש איתו בעיות אחרות, כמו פירוש לורנץ, איינשטיין, וגם זה שלך.

    ובאמת אין לי בעייה עם האמינות שלי בעיניך.
    הצעתי לך תהדיה. אם תשמור עליה, נוכל להתדיין בניחותא. ברגע שתחזור למניירות האישיות כהרגלך, תחטוף פי 10.

  219. ר.ח.
    חזרנו מעבודה.

    2. ביחס למודד, הכוונה היא שלא משנה באיזו מהירות אתה נמצא, אתה תמדוד את האור כC. אם אתה זוכר את מודל האוטוסטרדה, מכוניות מהירות מדי הן שקופות עבורך, והאיטיות עוד לא הגיעו.

    3. בדוגמה שנתת העץ הוא מערכת סובייקטיבית לגמרי, ובאותה המידה היית יכול לבחור עץ על מאדים. נראה לי, אגב, שאתה חייב להפנים נקודה זו, שאין דבר כזה מערכת מוחלטת. לא תוכל כדבריך בעבר “להוציא עשן” מהחללית וכך לדעת אם החללית נעה. פרוטון שמואץ ל0.999C נמצא מבחינתו במנוחה, כל עוד מהירותו קבועה. גלקסיה המתרחקת מאיתנו במהירות הקרובה למהירות האור נמצאת במנוחה, ומבחינתה אנו אלו שנעים.

    4. כמו 3.

    5. לא קיבלתי את העקרון שכל תנועה היא יחסית לקרינה הקוסמית. גם איינשטיין לא. הוא אפילו לא ידע עליה ב1905.

    לא גמרנו את דיונינו הקודם, אך אני מאמין שראית את בעייתיות הנושא (מאמין לי עכשיו שגם פיזיקאים הסתבכו קשות?). אם תרצה, נוכל להמשיך, אך בהחלט אקבל זאת בהבנה אם תרצה לפרוש.

    רק דבר אחד – האור האיטי מסוגל להסביר את התופעה התמוהה אפילו יותר מאי לוקליות – ההשפעה על העבר מהעתיד.

    ובעניין יובל – יש את העניין הזה, של זכות ההגנה העצמית. אתה מוזמן לעבור על השירשור בכתבה זו ואחרות. תיווכח שאני אף פעם איני פוגע ראשון במגיבים. אך אין לי שום חשק לספוג ולסתום.

  220. רבותי הנכבדים יובל וישראל,

    אתה מוכנים להירגע ולזכור שזה דיון מדעי בשביל הכיף שבו? אם אתם לא נהנים אתם גורמים גם לי ולאחרים לא להנות ממנו ואם כך אין שום טעם בו ובואו נפסיק את הדיון.
    לצערי הרב אין כבר שלג ואני לא יכול לצאת לסקי, כי פעם שעברה זה גרם לאהבה גדולה בינכם. אז רבותי תתעשתו , תרגעו תשתו מיים ותתחילו לדבר לגופו של עניין ולא לגופו של סגנון , כבוד וכל היוצא בשטויות האלה. אף אחד לא ביקש שתתחתנו או אפילו תצאו לפאב לבירה ביחד. יאללה תרגעו.

  221. ישראל,
    השיח כאן הוא ציבורי. אם אתה מפני אותי לקישורים וחיפושים, הרי אתה מטריח לא רק אותי אלא גם את כל מי שעוקב. מן הראוי היה שתסביר דברים במילים שלך או שלפחות תביא מראה מקום לדברים המפורשים. על כל פנים, מחלתי על “כבודי”, גיגלתי ולא מצאתי, ואיני מאמין לך.
    מה שכן מצאתי, בקישור הנ”ל, http://en.wikipedia.org/wiki/Emission_theory, מציג את ההסבר שאתה מדבר עליו כמופרך.
    אם ברצונך לשמור על אמינות, גם אם רק בעיניו של נודניק טרחן כמוני, עליך להשתדל יותר.

  222. סטודנט, טכניון,
    השאלה אם שיקרתי או לא נתונה לפרשנות.
    המודל שלי מתיימר להציג את כל המשפטים היסודיים של מודלים פיסיקליים קיימים כנובעים מחלקיק יסודי אחד. ברגע שמשימה זו בוצעה לגבי מודל פיסיקלי קיים כלשהו, אפשר להמשיך ממנו. אפשר, כתרגיל, לבנות הכל ישירות מן המודל הבסיסי. זו עבודה שחורה מיותרת אך אפשרית. אם משתמע מדברי שביצעתי אותה עבור כל המודלים הפיסיקלים הקיימים, הרי שיקרתי ועל כך אני מתנצל.
    שאלת על אורך הקשר בין אטום המימן ואטום הכלור. אטום הכלור מכיל 17 פרוטונים ועוד מספר דומה של ניטרונים. כל אחד מן הנוקלאונים האלה מוסיף פרמטרים למשוואת חישוב הרדיוס הקוולנטי. תרומתו של כל נוקלאון תלויה בכמה גורמים, למשל במיקומו בתוך הגרעין ביחס לנוקלאונים אחרים. חישוב הרדיוס הקוולנטי של אטום הכלור מצריך, על כן, עבודה רבה. גרעין אטום המימן מכיל, בדרך כלל, רק נוקלאון אחד והרדיוס הקוולנטי שלו פשוט יותר לחישוב.

  223. ישראל,
    טוב אז בוא נמשיך.
    1) אנחנו מסכימים

    2) מה זה ביחס למודד? האם משפט שה לא שקול ל”מהירות האור אינה יחסית”? האם אתה יכול להדגים מערכת שאליה האור יתנהג בצורה יחסית?

    3) מסכימים, למעט הקביעה שרק הקרינה הקוסמית היא מערכת מנוחה. למרות שמה היא לא קסומה. לטענתי כל מערכת שלשית שתוגדר כמערכת מנוחה תגיד לך בדיוק מי נוסע ומי לא.
    דוגמא : אתה ברכבת בתחנת רכבת. מולך צמוד לחלון יש עוד רכבת. מתחילנ תנועה בין שתי הרכבות. איך אתה יודע מי נעה? הרכבת שלך או השניה?
    תשובה: אתה מסתכל לצד שני לעץ שנשקף מהחלון השני. ביחס אליו (ושים לב שהוא לא קרינה קוסמית) אתה בדיוק יכול לדעת מי נוסע ומי עומד.

    4) כמו 3

    5) למה לא מתיישבות? אם קיבלת את העיקרון שכל תנועה היא ביחס לקרינה הקוסמית למה כל תנועה בזמן לא יכולה להיות ביחס לנקודת ה – 0 המפץ הגדול?

    6) נמתין רגע ונבדוק את 1-5

  224. ר.ח.

    בדיוק ראיתי את התגובה שלך.

    הנה התשובות:

    1. יש התארכות זמנים במערכות מואצות. אינני יודע אם הדבר נכון במערכות שאינן מואצות.

    2. מהירות האור היא תמיד C יחסית למודד. הסיבות מדוע המהירויות הגבוהות או הנמוכות מC שקופות לנו אינן עלומות, והוסברו בדוגמת המטוטלת הבליסטית.

    3. אין אפשרות לקבוע בין שתי מערכות שאינן מואצות מי נעה ומי נחה באופן מוחלט, אלא אם כן אתה משווה את מהירותן למערכת המנוחה של הקרינה הקוסמית. חוץ ממנה, כל תנועה היא יחסית בלבד.

    4. אם הוספת מערכת נוספת ואתה מחליט שרירותית שמערכת זו במנוחה, אז כל תנועה תהיה יחסית למערכת זו. לכן התשובה היא לא (למרות שלא ממש הבנתי למה אתה מתכוון פה בכן או לא). אם שעון הטמפ’ כלול במערכת היחוס החדשה, אז התנועה תהייה יחסית אליו.

    5. כל הטיעון שלי הוא שתורת היחסות אינה מתיישבת עם תאוריית המפץ הגדול. אינני מתיימר להכריע בין השתיים.

    אבל אם אתה מקבל את תאוריית המפץ הגדול, הרי שקיבלת על עצמך זמן מוחלט במערכת אינרציאלית, ומכאן הסתירה ליחסות שאין בה זמן כזה. ע”ע דיונינו.

    6. אם נגמור קודם את הדיון הנוכחי, ובהנחה שתשתכנע בנכונות טענותי, תראה, כך אני מאמין, שניתן להסביר בצורה קלאסית מדוע מהירות האור זהה לכל מודד, ומדוע לא יכולה להיות התארכות זמנים במערכת שאינה מואצת. את היתרון של רעיון המהירוית המשתנות (שאיננו סתם פוסטולט, הוא מתחייב ממודל האתר האקטיבי), תוכל לראות כשניגש לטפל במודלים קוואנטים.

  225. התגובה האחרונה הייתה מכוונת כמובן ליובל הנודניק.

  226. אין לי שליטה על מה שאתה מבין מדברי, יש לי שליטה רק על מה שאני אומר.

    הצעתי לך לגגל emission theory of light, אילו היית עושה זאת, היית מוצא שם: Emission theory (also called emitter theory or ballistic theory of light) was a competing theory for the special theory of relativity, explaining the results of the Michelson-Morley experiment.

    העובדה שאתה מתעצל ללכת ללינקים אינה בעיה שלי.

    ובקשר לאטימות שלי: אני מקבל זאת כמחמאה ממי שמעיד על עצמו שהוא טרחן קשה תפישה.

    בכלל, קיווינו שעכשיו כשיש לך בלוג משלך, תהיה עסוק בלענות לאלפי המגיבים למודל שלך, בעיקר לאילו משוודיה, ותפסיק להתעלק על מי שממש לא מעוניין להתעסק איתך ומזגך המחורבן.

    יאללה, תירה עוד כמה קסאמים וגרדים כדי להראות לכולם מי פה הגבר, ותהדיה.

    רק אל תבוא בטענות על כך שאם אתה מציק לישראל יותר מדי, יהרסו לך את חצי עזה.

  227. ישראל,
    מרוב דובים כבר לא רואים את היער. בוא נלך אחורה לרגע ונקרא לעזרתנו לסוקרטס הקשיש והמורעל.

    בוא נסכם בבקשה בצורה חדה מהן טענותיך:

    אנא ענה על השאלות הבאות בכן ולא ונמק בקצרה במידת הצורך (לא לשכוח זמן ותאריך)
    1) אין התארכות זמנים?
    2) מהירות האור כן יחסית? (טוב את זה אני חושב שהבנתי, אתה טוען שכן אולם אנו במכשירנו רואים רק את האור הנע ב- C והיותר מהיר ויותר איטי שקוף לנו מסיבות עלומות?)
    3) אם יש לנו שתי מערכות,לא ניתן לקבוע מי נעה או מי נחה?
    4) אם ענית כן ל-3 אזי, אם נוסיף מערכת שלישית שאינה תלוייה בג’יל או ג’ק ונתייחס אליה כמערכת הייחוס, האם התשובה ל-3 עדיין תהיה “כן”? (מערכת שלישית יכולה להיות למשל ביטלג’וס, שעון טמפרטורות או אדמונד בחללית נוצצת)
    5) אם הסכמת עם 4 , האם עדיין אתה רואה סתירה בתורת היחסות? אם כן מהי?
    6) האם מודל המוזר של קרני אור בכל המהירויות הנעות על חלקיקי אתר עלומים שמזוהות ע”י חיישננו על כל סוגיהם – הפיזיקליים (אפקט פוטו אלקטרי), או הביולוגי – (עניים) רק במהירות היחידה C מהווה ייתרון על המודל הקיים וגם המוזר של אור עם מהירות לא יחסית?

    בהצלחה

  228. ישראל,
    אינני מתיימר להיות בעל סבלנות אין קץ כידידנו המשותף. הסבלנות שלי פוקעת מדי פעם, ולא רק בגללך. הטריגר שגורם לסבלנותי לפקוע פעם אחר פעם בשיחות אתך הוא האטימות שאתה מפגין, בין אם אמיתית ובין אם מדומה. אמרת “שאלה: למה שלא תיקח את הפירוש הפשוט עוד יותר, שמהירות האור היא פשוט יחסית למקור האור? זה יסביר את תוצאות ניסוי מ-מ בקלות, לא?” (https://www.hayadan.org.il/astronomers-reach-new-frontiers-of-dark-matter-130112/#comment-332017). מזה הבנתי שאתה טוען כי מהירות האור יחסית למקור האור מסבירה כיצד מהירות האור יחסית לצופה הינה קבועה, וציינתי כי עליך להוכיח את אמירתך זו. אולי הבנתי לא נכון, אך גם אז עליך לצאת בהצהרה בנוסח “דבריי הוצאו מהקשרם” או משהו כגון דא.
    ליל מנוחה/בוקר טוב (בחר את הנכון).

  229. אשאל אותך פעם אחרונה ודי: הוכחה של מה?
    ואם לא תענה הפעם בדיוק, אבקש ממך לעזוב אותי בשקט.
    מה גם ש 2 בבוקר בל.א. – לילה טוב.

  230. אמרתי מה? עוד הפעם אני אמור לנחש את כוונותיך? אינני זוכר שהתחמקתי אי פעם מאיזה נושא שהעלתי, ראה הדיון המפורט בן החודש שלי עם ר.ח.
    אבל אין לי שום מחויבות לדון בנושאים שאינני מעוניין בהם.

  231. סטודנט, טכניון, ברוך שובך
    התשובה לשאלתך היא “אולי”. והריני להסביר: המודל שלי מתחיל מן הבסיסי ביותר ועולה מעלה שלב אחר שלב. עד עכשיו נגעתי כאן בקושי בגרביטציה ואינרציה של פרוטון בודד. פרוטון הינו מערכת (“מושבה”) מאד מורכבת שלא נגעתי אלא באפס קצה. עדיין לא הראיתי כיצד פרוטון ואלקטרון “מוצאים” זה את זה, במה נבדל הניטרון מן הפרוטון, כיצד פועל הכוח החזק ומה הגורם לפחת המאסה בהתפרקויות גרעיניות. כמו כן, עלי להסביר את הכוח החלש ומשפחה לא קטנה של חלקיקים תת-אטומים, את הגורמים העומדים בבסיס מכניקת הקואנטים (כולל חוסר הלוקאליות בשזירה הקואנטית אשר לה טרם מצאתי הסבר) וכו’, ובסוף גם לתת “תחזיות לתופעות אשר טרם נתגלו” כראוי למודל המכבד את עצמו. מולקולה של HCl בנויה מכמה וכמה מערכות מורכבות המשולבות זו בזו. טרם הגיעה השעה לטפל בה ובאספקט המאד מסוים שעליו אתה שואל. ואחרי כל זה אני עדיין אומר רק “אולי”, כי נכון לעכשיו אין לי חישובים מן המוכן מעבר למולקולת המימן. יתכן שאבחר להתעסק גם בשאלה שלך או אולי אשאיר אותה למי שיעקוב אחר הדברים, כתרגיל. בכל מקרה, לא אעשה זאת כאן מעל דפי הידען אלא בבלוג שפתחתי לצורך העניין (בזכותך).

  232. ישראל! לא יפה ולא מכובד להתנהג כך. עליך לשאת באחריות למוצא פיך. אמרת משהו, אז תהיה גבר ותעמוד מאחרי הדברים שלך. כל עוד אינך עושה כן, איני רואה מנוס מן המסקנה שאתה חמקן סדרתי.

  233. חברה, לא עושים משהו לכבוד היומולדת של איינשטיין?

    בטח גלי תתן כתבה.

  234. יובל, האם נזכה לראותך מחשב את אורך הקשר של HCl בעזרת מודלך?

  235. בסדר יובל. אתה יכול אולי לטפס על מישהו אחר? פטרת אותי מלהגיב למודל שלך. תוכל לעשות אותו הדבר עם כל התגובות שלי? פשוט לשחרר?

  236. ישראל! אתה מנהל מלחמת התשה? שולח אותי ללינקים?
    אתה צריך לעמוד מאחורי מה שאמרת, באופן הפשוט הבא:
    משפט א, ממנו נובע משפט ב, משפט ג וכו’… ולכן “מהירות האור יחסית למקור האור מסבירה כיצד מהירות האור יחסית לעינו של הצופה היא תמיד קבועה”.
    אין כאן שום “מניירות אישיות”. זו רק ביקורת על האופן בו אתה מנהל שיח פסאודו מדעי.
    וזה שאינני מתערב בתכנו של הויכוח בינך לבין ר.ח זה רק משום שאני סבור שההנחה הבסיסית שעליה אתה בונה את כל הטיעונים הינה שגויה, וכבר אמרתי לך את זה כמה וכמה פעמים אבל אתה בחרת להתעלם מדברי באופן שיטתי גורף. לעומתי, ר.ח עושה את הדבר הנכון. הוא מפרק את הטיעונים שלך אחד לאחד ומראה לך מהיכן אתה משתין. היתרון שלו עלי הוא הסבלנות הבלתי נדלית שלו. שלי פקעה לא אחת.

  237. ר.ח.

    “מסקנה: יש הבדל בין המערכות של ג’ק וג’יל. ———-> נכון”

    למה אפליה?

    1. כלום אינך רואה ששתי המערכות סימטריות לחלוטין? שכמו שג’יל נעה יחסית לג’ק, ככה ג’ק נע יחסית לג’יל, באותה מהירות בדיוק רק בכיוון ההפוך?

    2. האם לא זה בדיוק מה שיגידו גלילאו, ניוטון ואיינשטיין?

    3. בגלל הסימטריה המוחלטת, האם אין זה מתחייב שאם שעונה של ג’יל מפגר יחסית למערכת של ג’ק, אז גם שעונו של ג’ק יפגר יחסית למערכת של ג’יל?

    4. האם אין זה בדיוק מה שאומרת היחסות? שכל מערכת תראה את הזמן של השניה כנע לאט יותר?

    בוא נעשה תרגיל מחשבתי קטן. נאמר שזמן רב לפני שג’ק פגש בג’יל, חלף לידו שעון שמסונכרן עם שעונה של ג’יל, ושעונו של ג’ק מראה את אותו הזמן בשעון החולף + קבוע מסוים.

    שים לב שכל מה שעשיתי פה הוא היפוך תפקידים: קיבלנו את נתוני הבעיה המקורית, רק בכיוון ההפוך. הפעם ג’ק הוא זה שנע יחסית למערכת של ג’יל, ולכן שעונו מפגר יחסית אליה. לכן, בגלל הסימטריה המוחלטת, ניאלץ להסיק שבעת המפגש בין ג’ק וג’יל יראה שעונו של ג’ק פיגור של מיליארד שנה יחסית לג’יל, + אותו קבוע מקודם.

    ולכן, אם נשתמש באותה אנלוגיה, יראה הוידאו של ג’ק יחס סיבובים של 1,000,000,000:1, ואילו של ג’יל 1:1.

    וזאת בסתירה לוידאוס שבידינו, שמראים את ההיפך.

    אולם מה עשינו בסך הכל? הכנסנו שעון נוסף, שאף אחד לא יודע אפילו על קיומו?

    חוץ מזה שיובל רומז שאני מנסה להתחמק ממשהו, או שאני טועה, וזה כשלעצמו ראיה חזקה שאני כנראה צודק ב100%.

    יובל.
    נטל הוכחה של מה? שמהירות אור יחסית למקור מסבירה את תוצאות ניסוי מ-מ? הסתכל על הניסוי שנית, או עיין ערך emission theory of light.

    אינני מבין גם למה איתך הכל נגרר תמיד לעימות אישי. למה שלא תודה, שפשוט אין לך מושג על מה אני ור.ח. מדברים? בכלל, אם אינך יכול להפסיק עם כל המניירות האישיות, אולי עדיף לסתום וזהו? מלחמה יודעים איך מתחילים, לא איך מסיימים.

  238. ר.ח,
    מנסיוני עם הלז אני אומר בוודאות גבוהה שאין טעם לחכות לתשובה סבירה ממנו. הוא נהנה לחולל מהומות אך יודע להסתלק בדיוק כשמגישים לו מטאטא ויעה

  239. ג’ק רואה יחס של 1:1
    1) ג’יל רואה יחס של 1,000,000,000:1.

    מסקנה: יש הבדל בין המערכות של ג’ק וג’יל. ———-> נכון

    ג’ק במערכת לא מואצת.
    ג’יל במערכת לא מואצת.

    מסקנה: אין הבדל בין המערכות של ג’ק וג’יל.————> תספר את זה לבנאדם שיושב בתחנת אוטובוס שעתיים ואז מגיע סוף סוף אוטובוס במהירות קבועה (מנוחה) ולא עוצר לו. נראה איזו שטוזה תחטוף כשתגיד שבעצם אין שום הבדל מבחינתו אם הוא בתחנה או באוטובוס, בכל מקרה הוא במנוחה.

    2) אבל אם כוכב פינקפלוידג’וס נע יחסית לביטלג’וס כמעט במהירות האור, אז ג’יל יחסית אליו במנוחה וג’ק בתנועה, לא? האם זה ישנה את יחס הסיבובים בין שעוניהם?

    ישראל איזה חשיבה מעוותת זו?מה הקשר מה עושים או לא עושים ביטלג’וס פינקפלוידג’וס או לד זפלינג’וס ליחס בין השעונים של ג’יל?? אתה מבלבל פה בין סיבה למערכת ייחוס. מערכת הייחוס לא קובעת כלום. אם אנו סופרים את התאריך לפי בריאת העולם , הולדת ישו או ההיג’רה לא משנה כלום לקצב התקדמות הזמן. זה בסך הכל מערכת ייחוס או מערכת צירים. זה נראה לי כל כך טריוויאלי ואני מעריך אותך מאד ולכן אני בטוח שאני מחמיץ פה משהו ממש עקרוני. רק תגיד לי מה?

  240. ר.ח.

    ג’ק רואה יחס של 1:1
    ג’יל רואה יחס של 1,000,000,000:1.

    מסקנה: יש הבדל בין המערכות של ג’ק וג’יל.

    ג’ק במערכת לא מואצת.
    ג’יל במערכת לא מואצת.

    מסקנה: אין הבדל בין המערכות של ג’ק וג’יל.

    נא ליישב את הסתירה.

    הצעה:

    אתה אומר ג’ק במנוחה יחסית לביטלג’וס, ג’יל קרובה למהירות האור. מכאן ההבדלים.
    אבל אם כוכב פינקפלוידג’וס נע יחסית לביטלג’וס כמעט במהירות האור, אז ג’יל יחסית אליו במנוחה וג’ק בתנועה, לא? האם זה ישנה את יחס הסיבובים בין שעוניהם?

    אל תשכח ממה התחלנו: ג’יל נעה רק יחסית למערכת שעונים שמסונכרנת במערכת של ג’ק. יחסית אליה, ג’ק נע בדיוק כמו שהיא נעה יחסית אליו. אילו היינו בודקים את התנועה של ג’ק יחסית למערכת של ג’יל, היינו רואים שהזמן אצלו מאט את קיצבו, ולא אצל ג’יל. אז מה, פתאום היחס אצליהם יתהפך רק בגלל שאנחנו החלטנו שהמערכת שונה?

    חשוב על מערכת מסונכרנת כרכבת ארוכה עם קרונות במרחק קילומטר זה מזה. איינשטיין הציע במאמר המקורי שלו על היחסות איך לסנכרן את השעונים באמצעות קרני אור.

    אם מכונית נעה במהירות לאורך הרכבת, היא תראה בכל שעון בקרון עוקב זמן קצת יותר ויותר מתקדם מזה שלה. זוהי ג’יל שלנו.

    אך אותו הגיון פועל גם בכיוון ההפוך. אם ג’יל יושבת בקטר, ומאחריה קרונות רבים ומסונכרנים זמנית, וג’ק נוהג בקטר רכבת הנוסעת בכיוון ההפוך, הרי שאם בזמן שהקטרים חולפים זה על פני זה השעונים מראים את אותו הזמן, גם ג’ק וגם ג’יל ימצאו שהזמן בקרונות הבאים מאיץ יותר ויותר יחסית אליהם (זה בגלל שיחסית לרכבות הקטרים הם אלו שבתנועה, ולכן הזמן בהם מאיט).

    בכל אופן, הסימטריה כאן מושלמת. שתי המערכות זהות לחלוטין, ומבחינת ג’ק, או כל קרון ברכבת שלו, ג’יל בתנועה, ואילו מבחינת ג’יל, וכל קרון ברכבת שלה, ג’ק בתנועה.

    לכן לא יתכן שיהיה הבדל כלשהו ביחס סיבובי השעונים. כוכבים חיצוניים, או זברבירי מסע, אינם קשורים לעניין כלל.

    אם מהירות האור היא יחסית למקור האור, זה מסביר במדוייק את ניסוי מ-מ. זה הדבר היחיד שאמרתי. ברור שיש עם זה בעיות אחרות, אבל כך גם עם תאוריית לורנץ (התכווצות) או המודל שאני פטור מלדבר עליו.

    קזינואים – מטעמים מובנים (ילדים שיכולים לעיין באתר לדוגמה) אינני יכול להרחיב ללא אישורו של אבי. המייל שלי אצל יובל, אם תרצה אשלח לך פרטים.

  241. פעם, כשתגובה שלי היתה מעוכבת, לפחות אני הייתי יכול לראות אותה (עם הערה שהיא ממתינה לאישור). כעת אפילו זה לא. נפלאו דרכי המערכת

  242. תוספת קטנה, תמדד מהירות C על ידי כל מודד בואקום ביקום שלנו
    יש מצב שביקום אחר מהירות האור תהיה שונה אבל את זה צריך לשלול או להוכיח.

  243. ישראל ויובל,

    אני מסכים עם יובל כאן ש הפירוש הפשוט עוד יותר, שמהירות האור היא פשוט יחסית למקור האור, אינה לעניין והיא לא עובדת.

    לפי ההוא מהשטר הישן של 5 לירות שאסור להזכיר את שמו באתר הידען מהירות האור קבועה לכל צופה, מפעיל או מקור אור. לא משנה מה יעשה או לא יעשה, גם אם יעמוד על הראש או יעשה פליק פלאק לאחור הוא תמיד ימדוד את מהירות האור כ- C. כנ”ל יעשו גם כל אלה שסביבו על תאוצותיהם, מהירותיהם ופליקפלקיהם כולם ימדדו כל אור ולא משנה מהיכן יצא ומתי כמהירות C (בתנאי אחד ויחיד, בואקום).

  244. אתחיל מן הפטור. אינני נוהג לקחת מתנות בחזרה. משמעות המילה “פטור” לענייננו היא שאינך צריך להרגיש מחויב להגיב למה שאני אומר.
    האם אתה אומר שמזה שמהירות האור היא יחסית למקור האור אפשר להסיק את תוצאות ניסוי מייקלסון מורלי? אם כן, נטל ההוכחה עליך.

  245. יובל.
    “ההצעה שלך, “הפירוש הפשוט עוד יותר, שמהירות האור היא פשוט יחסית למקור האור”, אינה לעניין והיא לא עובדת.”
    היא עובדת יפה כדי להסביר את תוצאות ניסוי מ-מ.
    בקשר למודלך וביטול הפטור – התנאי מצידי היה ונשאר בקשה מפורשת ממך, והתחייבות מצד שנינו לדבוק בענייניות בלבד.

  246. ישראל,
    הויכוח הזה הולך סחור סחור.
    1) אתה אומר ” זכור, שניהם מבחינתם במנוחה מוחלטת. ההסתייגות היחידה הבאה בחשבון הידועה לי, היא המהירות היחסית למערכת הקרינה הקוסמית. אך זה לא הנושא שלנו, וגם לא רלוונטי כרגע. אנחנו ב1905, אין קרינה.”

    לא חייבים דוקא קרינה קוסמית. ג’יל וג’ק יכולים לקחת כל עצם שלישי שיוחלט בינהם שהוא מערכת ההתייחסות. נניח כוכב ביטלג’וס. גם ג’ק וגם ג’יל יסכימו שג’יל נעה במהירות קרובה למהירות האור ביחס לביטלג’וס ואילו ג’ק נע לאט יותר (מהירות שלילית).

    היוצא מן הכלל היחידי הוא האור. כאן נראה לי שאתה מפספס משהו. נניח שג’יל תשלח קרן אור וג’ק ימדוד את מהירותה הוא יקבל C. ג’יל שתמדוד את מהירות אותה קרן תקבל גם היא C וגם יורגנסון שעל כוכב ביטלג’וס ייקבל C מאותה קרן.

    2) אתה אומר : “בכל אופן, אם נחזור לדיונינו, אין מנוס מן המסקנה שצילום של שעוני הטמפ’ והצזיום יראה את אותו הזמן בשניהם, גם אצל ג’ק וגם אצל ג’יל. אם לא, הראה לי איפה.”

    למה אין מנוס? כי אתה החלטת? כאמור ג’יל תראה התקררות בקצב של 1000 מעלות בשעה ואילו ג’ק יראה 1000 מעלות במיליארד שנה. ברור שבכל זמן ששניהם יצלמו את השעונים תימדד אותה טמפ’ בדיוק. שוב כמו בדוגמת הרכבת. אם שני נוסעים, יושב ורץ יצלמו את הנוף בחוץ תמיד יהיה אותו נוף למרות שאחד עבר קילומטראז הרבה יותר גדול. קצת לעסנו את זה, לא?

    3) מה הקשר עכשיו למפץ הגדול? המפץ מוגדר כנקודה סינגולרית שאין שום דרך כרגע לדעת מה קרה מהעבר השני שלה, כן זמן, לא זמן, כן מרחב, לא מרחב הן שאלות לא רלוונטיות כי אין שום כלי כרגע לבדוק אותן. באופן כללי אין זמן לפני המפץ וזו שאלה לא רלוונטית. זה בערך כמו הקול שמשמיעה מחיאת כף של יד אחת.

    4) קזינו – עזוב אותך מהשטיויות של איינשטיין מאך וכל אלה. זה הרבה יותר מעניין. איך זה ייתכן שספירה של 6 חבילות פשוטה כמו ספירה של חבילה 1? תן איזה Casino for dummies.

  247. ישראל,
    את ההסבר של לורנץ הבאתי רק כדוגמה להסבר אפשרי. ההסבר שלי הוא אחר. אמרתי את זה כאן כמה וכמה פעמים, ואף הבאתי פירוט על פי המודל שלי:
    https://www.hayadan.org.il/astronomers-reach-new-frontiers-of-dark-matter-130112/#comment-329898
    ההצעה שלך, “הפירוש הפשוט עוד יותר, שמהירות האור היא פשוט יחסית למקור האור”, אינה לעניין והיא לא עובדת.

  248. יופי, חלק א’ עבר. ננסה את ב’.

    אל תשכח שאיני מנסה לפסול את התארכות הזמנים – אני רק מצביע על ההתנגשות שלה עם תאוריית המפץ הגדול. נסה לראות זאת בצורה הבאה:

    1. האם לפי המפץ, יש בכלל משמעות לשאלה מה היה 1000 שנים לפני המפץ? מה קרה לפני 20 מיליארד שנה? או שעצם מושג “זמן” איננו מוגדר לפני המפץ?

    2. האם לפי היחסות ב1905, יש בעייה עם השאלה מה קרה לפני 100 מיליארד שנה?

    בכל אופן, אם נחזור לדיונינו, אין מנוס מן המסקנה שצילום של שעוני הטמפ’ והצזיום יראה את אותו הזמן בשניהם, גם אצל ג’ק וגם אצל ג’יל. אם לא, הראה לי איפה.

    קזינואים: אין בעיה מיוחדת לסופר לספור 4, 8, או 800 חפיסות. הבעיה היא בכדאיות. למעשה, קיימת שיטה שמותאמת במיוחד למקומות שבהם רק שולחנות עם 6 חפיסות ומעלה.

    בשורה התחתונה: זה לא נכון שהקזינו תמיד מנצח, אבל זה נכון שהמהמר תמיד מפסיד. אפילו עם יש לו יתרון על הבית.

    אלא אם כן אתה מעוניין להקדיש את חייך לקיזוזי אסים, רביצה ליד שולחנות, ושאיפת עשן לרוב.

    יובל.

    “לא צריכים להיות צ’ילבאות בשביל לתת פייט טוב”.
    אפשר בכלל לוותר גם על נושא הפייט, ולגשת ישר למדע. הסברתי לך את הבעייתיות שקיימת בפירוש שלך לניסוי מ-מ. (אני מאמין שהתכוונת לפירוש של לורנץ, התכווצות).

    שאלה: למה שלא תיקח את הפירוש הפשוט עוד יותר, שמהירות האור היא פשוט יחסית למקור האור? זה יסביר את תוצאות ניסוי מ-מ בקלות, לא?

  249. ננסה עוד הפעם אינטגרציה בחלקים, נראה אם יעבור.

    ר.ח. יקיר.
    רונן, אה? לאט ובזהירות, כמו קיפודים, נחשוף את פרצופך האמיתי.

    3. מנוחה: גלילאו, ניוטון, איינשטיין. כל גוף שאינו בתאוצה נמצא במנוחה. ייתכן שהוא נע יחסית למערכות אחרות, אך מבחינת המערכת שלו הוא במנוחה.

    אור: תמיד נעה יחסית אליך במהירות אחת: של האור. אם אתה נוסע על כביש החוף צפונה במהירות 200 מ/שנ, אופנוע נוסע לכיוון דרום במהירות 300 מ/שנ, וברגע שאתם חולפים זה על פני זה אתה מדליק את הפנסים, הרי שהאור מתרחק משניכם באותה מהירות, מהירות האור, גם אם אתה תמדוד וגם אם האופנוע. זאת למרות שהמהירות היחסית בינכם היא 500 מ/שנ. זה יהיה אותו הדבר גם אם המהירות ביניכם היא 0.9C.

    לכן, ג’ק תמיד במהירות C יחסית לאור וגם ג’יל. אך שניהם נמצאים במנוחה מוחלטת מבחינתם.

    4. ג’יל נעה יחסית לג’ק קרוב למהירות האור, אך מבחינתה היא במנוחה, וג’ק הוא זה שנע. אי אפשר להעדיף את המערכת של ג’ק על זו של ג’יל, או להיפך. זהו בדיוק הפוסטולט הראשון של היחסות.

    5. עקרוני מאוד. כי אם אין הבדל בין המערכות, ויחס סיבובי השעונים אצל ג’ק הוא 1:1, אז זה יהיה גם היחס אצל ג’יל.

    זכור, שניהם מבחינתם במנוחה מוחלטת. ההסתייגות היחידה הבאה בחשבון הידועה לי, היא המהירות היחסית למערכת הקרינה הקוסמית. אך זה לא הנושא שלנו, וגם לא רלוונטי כרגע. אנחנו ב1905, אין קרינה.

    6-7. אין מהירות במרחב למערכת שאינה מואצת, אלא רק מהירות יחסית למערכת אחרת. מה שהוכח בניסוי המטוסים הוא שהזמן מתארך במערכת מואצת. דבר זה אפשרי אם תתייחס לזמן כמספר חלקיקים ליחידת נפח, ע”ע הדיון שלי עם סטודנט.

  250. ר.ח. יקיר.
    רונן, אה? לאט ובזהירות, כמו קיפודים, נחשוף את פרצופך האמיתי.

    3. מנוחה: גלילאו, ניוטון, איינשטיין. כל גוף שאינו בתאוצה נמצא במנוחה. ייתכן שהוא נע יחסית למערכות אחרות, אך מבחינת המערכת שלו הוא במנוחה.

    אור: תמיד נעה יחסית אליך במהירות אחת: של האור. אם אתה נוסע על כביש החוף צפונה במהירות 200 מ/שנ, אופנוע נוסע לכיוון דרום במהירות 300 מ/שנ, וברגע שאתם חולפים זה על פני זה אתה מדליק את הפנסים, הרי שהאור מתרחק משניכם באותה מהירות, מהירות האור, גם אם אתה תמדוד וגם אם האופנוע. זאת למרות שהמהירות היחסית בינכם היא 500 מ/שנ. זה יהיה אותו הדבר גם אם המהירות ביניכם היא 0.9C.

    לכן, ג’ק תמיד במהירות C יחסית לאור וגם ג’יל. אך שניהם נמצאים במנוחה מוחלטת מבחינתם.

    4. ג’יל נעה יחסית לג’ק קרוב למהירות האור, אך מבחינתה היא במנוחה, וג’ק הוא זה שנע. אי אפשר להעדיף את המערכת של ג’ק על זו של ג’יל, או להיפך. זהו בדיוק הפוסטולט הראשון של היחסות.

    5. עקרוני מאוד. כי אם אין הבדל בין המערכות, ויחס סיבובי השעונים אצל ג’ק הוא 1:1, אז זה יהיה גם היחס אצל ג’יל.

    זכור, שניהם מבחינתם במנוחה מוחלטת. ההסתייגות היחידה הבאה בחשבון הידועה לי, היא המהירות היחסית למערכת הקרינה הקוסמית. אך זה לא הנושא שלנו, וגם לא רלוונטי כרגע. אנחנו ב1905, אין קרינה.

    6-7. אין מהירות במרחב למערכת שאינה מואצת, אלא רק מהירות יחסית למערכת אחרת. מה שהוכח בניסוי המטוסים הוא שהזמן מתארך במערכת מואצת. דבר זה אפשרי אם תתייחס לזמן כמספר חלקיקים ליחידת נפח, ע”ע הדיון שלי עם סטודנט.

    אל תשכח שאיני מנסה לפסול את התארכות הזמנים – אני רק מצביע על ההתנגשות שלה עם תאוריית המפץ הגדול. נסה לראות זאת בצורה הבאה:

    1. האם לפי המפץ, יש בכלל משמעות לשאלה מה היה 1000 שנים לפני המפץ? מה קרה לפני 20 מיליארד שנה? או שעצם מושג “זמן” איננו מוגדר לפני המפץ?

    2. האם לפי היחסות ב1905, יש בעייה עם השאלה מה קרה לפני 100 מיליארד שנה?

    בכל אופן, אם נחזור לדיונינו, אין מנוס מן המסקנה שצילום של שעוני הטמפ’ והצזיום יראה את אותו הזמן בשניהם, גם אצל ג’ק וגם אצל ג’יל. אם לא, הראה לי איפה.

    קזינואים: אין בעיה מיוחדת לסופר לספור 4, 8, או 800 חפיסות. הבעיה היא בכדאיות. למעשה, קיימת שיטה שמותאמת במיוחד למקומות שבהם רק שולחנות עם 6 חפיסות ומעלה.

    בשורה התחתונה: זה לא נכון שהקזינו תמיד מנצח, אבל זה נכון שהמהמר תמיד מפסיד. אפילו עם יש לו יתרון על הבית.

    אלא אם כן אתה מעוניין להקדיש את חייך לקיזוזי אסים, רביצה ליד שולחנות, ושאיפת עשן לרוב.

    יובל.

    “לא צריכים להיות צ’ילבאות בשביל לתת פייט טוב”.
    אפשר בכלל לוותר גם על נושא הפייט, ולגשת ישר למדע. הסברתי לך את הבעייתיות שקיימת בפירוש שלך לניסוי מ-מ. (אני מאמין שהתכוונת לפירוש של לורנץ, התכווצות).

    שאלה: למה שלא תיקח את הפירוש הפשוט עוד יותר, שמהירות האור היא פשוט יחסית למקור האור? זה יסביר את תוצאות ניסוי מ-מ בקלות, לא?

  251. אינשטין
    אינשטיין
    איינשטין
    הםםם… צודק
    אני מציע שהמערכת תשלח לנו, המגיבים שעברו את מבחן הלשון הנקיה, את רשימת המילים הבעייתיות, ואנחנו נתחייב לשמור אותה בסוד 🙂

  252. אבי, ברוך שובך. אם יש חוויות, אנא חלוק.

    ר.ח,
    התגובות שלי נחסמות הרבה פעמים, אבל ישראל איננה מילת מפתח רלוונטינית. לעומת זאת, חשוד בעיני קרובו של זמר ותיק שנתן את שמו למעבד תמלילים עתיק.

    ישראל,
    ממך לעולם לא אתאכזב. אתה תמיד תדע להחזיר מלחמה שערה. והקטע העוד יותר טוב הוא שאנחנו בכלל לא צריכים להיות צ’ילבאות בשביל לתת פייט טוב.
    מאז שפתחתי בלוג עצמאי (בעצת סטודנט, טכניון הזכור לטוב) תפשה אותי נינוחות עצלה ולא העליתי שום דבר חדש. אבל בקרוב, אינשאללה, בקרוב. אני עובד על הדברים

  253. ספקן, ליובל יש מילון מיוחד וצריך להבדיל בין מה שיובל אומר למה שקורה באמת.
    כשיובל אומר: “רק אתה לא הבנת” – הוא מתכוון ל”אף אחד לא הבין”.
    “אתה מתחמק” – הכוונה ל”לא ענית מה שרציתי שתענה”.
    “תהנה לך מאור הזרקורים” – הכוונה היא “למה אף אחד לא מתייחס לנושא החשוב באמת: המודל של יובל”.
    “שוטף מוחות” – אתה עלול רחמנא ד’ליצלן לשכנע מישהו במה שיובל לא מאמין או לא מבין.
    “סרסור מילים” – כנ”ל.

    “נונקונפורמיזם” – דיסלקציה.

    ובאופן כללי, לא רק לגבי יובל: מי שמתחיל להתעסק איתך אישית, במקום עם דעותיך, לא ממש מחפש מודל מדעי, אלא מודל אגו. אחרת היה עוזב אותך לנפשך ומדפדף לתגובה הבאה.
    וחוץ מזה, יובלי די חמוד למרות הכל.

  254. אבי בליזובסקי,
    אם אכן החשד שלי ושל ישראל נכון ששמו, שם מדינתנו ושמו השני של יעקב אבינו היא מילה אסורה בידען, אני מוחה בתוקף!

    ישראל,
    1) מסכימים
    2) מסכימים
    3) לא יודע מה זה מנוחה. אנחנו הגדרנו את ג’ק כנע במהירות 0 ביחס למהירות האור, ג’יל, כוכב שבתאי או הזבוב שחג מסביבו.
    4) לא יודע מה זה מנוחה. אנחנו הגדרנו את ג’יל כנעה במהירות קרובה למהירות האור עד כדי כך שחלפו להן מילארדי שנים של ג’ק בשעה שלה.
    5) לא יודע מה זה עקרוני. ג’יל נעה יותר מהר מג’ק. זה עקרוני? חוץ מזה היא בלונדינית והוא אזיאט. זה עקרוני?
    6-7) לא מבין את השאלות. כאמור ההבדל במהירות במרחב שמביא להאטה במהירות הזמן שחלף. לא מבין מה אתה מתעקש כל כך על זה הרי זה הוכח בניסוי שעוני הצזיום.

    לינקים לגלילאו? יש כאלה שיודעים לספור קלפים ויש כאלה שמוצאים כל דבר, גם מה שקבור במרתפים החשוכים של האינטרנט. דרך אגב, חשבתי שמאז ביל קפלן מ- MIT אי אפשר יותר לספור קלפים כי הקזינואים משתמשים במספר רב של חבילות. הלא כן?

  255. זמן הוא רלבנטי / משמעותי כאשר ישנה תנועה כלשהיא של גופים, אטומים או חלקיו הקטנים יותר.
    לגבי האנטרופיה, היא רלבנטית כל עוד ישנו מבנה מולקולרי כלשהו ביקום, כי ברגע שהמרק הקוסמי יהיה זהה ומפוזר באופן שווה, לא תהיה לו משמעות.

  256. זמן הוא מימד העומד בפני עצמו,
    והוא מוצמד לכל מערכת מימדים בדרך שונה.

    עד כה היכרנו זמן אשר נע קדימה,
    בהמשך נגלה מערכות בהן הזמן מתנהג בדרך שונה.

  257. נסיון הברחת שבר תגובה לפני שתתישן:

    ר.ח.

    רק עכשיו ראיתי את התגובות שלך. נראה לי שהזכרת שמי מעלה את חמתה של האינקוויזיציה. להבא השתדל להשתמש במחלות קוד. נסה “הצבי” או “ישורון” או בקיצור נמרץ “צבא הגנה ל..” אני כבר אבין שהכוונה אלי ואדלג ישר לתגובה שאחרי.

    נראה לי שאני מתחיל להבין את שורש אי ההבנה ביננו. הטריגר הוא המשפט ” היחס בין השעונים של ג’יל יהיה תמיד זהה כתלות במהירות שלה ביחס למצב מנוחה או אם תרצה למהירות האור.”

    הנה השאלות הסוקרטיות הרגילות:

    1. האם אנו מסכימים על כך שיחס הסיבובים בין שעוני הטמפ’ והצזיום אצל ג’יל הוא 1,000,000,000:1?

    2. האם אנו מסכימים על כך שיחס הסיבובים בין שעוני הטמפ’ והצזיום אצל ג’ק הוא 1:1?

    3.האם לדעתך ג’ק נמצא במנוחה?

    4. האם לדעתך ג’יל נמצאת במנוחה?

    5. האם יש איזה שהוא הבדל עקרוני בין המערכת של ג’ק לזו של ג’יל? כן/לא

    6. אם התשובה ל5 היא לא, אז איך זה שיש כזה פער ביחס? למה אין המצב הפוך ביחס הסיבובים, למה אין היחס אצל ג’יל 1:1או משהו באמצע, 100:1 לדוגמה, איך זה שהיחס אצל ג’יל בדיוק מתאים למהירותו של ג’ק יחסית אליה?

    7. אם התשובה ל5 היא כן, הסבר מה ההבדל. מה אתה מתכוון כשאתה אומר: “במהירות שלה ביחס למצב מנוחה או אם תרצה למהירות האור.” מדוע מצבו של ג’ק שונה משלה?

    דבר גלויות, כולנו פה אחים. אל תכהה ואל תכחד. רק זכור, לא להזכיר את השם המפורש.

    אגב – מאיפה הבאת את הלינקים לגלילאו? לא ידעתי על קיומם. כרגיל, לא מספרים לי כלום.

  258. ר.ח.

    רק עכשיו ראיתי את התגובות שלך. נראה לי שהזכרת שמי מעלה את חמתה של האינקוויזיציה. להבא השתדל להשתמש במחלות קוד. נסה “הצבי” או “ישורון” או בקיצור נמרץ “צבא הגנה ל..” אני כבר אבין שהכוונה אלי ואדלג ישר לתגובה שאחרי.

    נראה לי שאני מתחיל להבין את שורש אי ההבנה ביננו. הטריגר הוא המשפט ” היחס בין השעונים של ג’יל יהיה תמיד זהה כתלות במהירות שלה ביחס למצב מנוחה או אם תרצה למהירות האור.”

    הנה השאלות הסוקרטיות הרגילות:

    1. האם אנו מסכימים על כך שיחס הסיבובים בין שעוני הטמפ’ והצזיום אצל ג’יל הוא 1,000,000,000:1?

    2. האם אנו מסכימים על כך שיחס הסיבובים בין שעוני הטמפ’ והצזיום אצל ג’ק הוא 1:1?

    3.האם לדעתך ג’ק נמצא במנוחה?

    4. האם לדעתך ג’יל נמצאת במנוחה?

    5. האם יש איזה שהוא הבדל עקרוני בין המערכת של ג’ק לזו של ג’יל? כן/לא

    6. אם התשובה ל5 היא לא, אז איך זה שיש כזה פער ביחס? למה אין המצב הפוך ביחס הסיבובים, למה אין היחס אצל ג’יל 1:1או משהו באמצע, 100:1 לדוגמה, איך זה שהיחס אצל ג’יל בדיוק מתאים למהירותו של ג’ק יחסית אליה?

    7. אם התשובה ל5 היא כן, הסבר מה ההבדל. מה אתה מתכוון כשאתה אומר: “במהירות שלה ביחס למצב מנוחה או אם תרצה למהירות האור.” מדוע מצבו של ג’ק שונה משלה?

    דבר גלויות, כולנו פה אחים. אל תכהה ואל תכחד. רק זכור, לא להזכיר את השם המפורש.

    אגב – מאיפה הבאת את הלינקים לגלילאו? לא ידעתי על קיומם. כרגיל, לא מספרים לי כלום.

    יובל.

    “שוב אתה מתחמק, צלופח קטן, אבל כבר התרגלתי.”
    בגלל זה חשבת שאני מזל דגים, בגלל הצלופח, אה?

    לא. נראה לי שאתה הוא זה שמתחמק. הסבר לנו מדוע הזמן וחוק ב’ הם היינו הך. אתה העלית נקודה זו.

    ספקן.

    נקודה מעניינת, אך אחרי חודשיים ו900 תגובות, לא עדיף שנתמקד בנושאים הפתוחים לפני שנעבור לחזיתות נוספות כגון פרספציה? נשאיר זאת לכתבות הבאות. גם ככה לוקח לי 5 דקות להגיע לתגובות מרוב העומס.
    אבל אתה מוזמן בהחלט להשאר איתנו בקוסמו. נראה שיש לך מה להגיד.

  259. safkan yakar, תודה על התייחסותך.
    י.ש עושה לעצמו מנהג קבוע להתחמק משאלות שאינן מתאימות לסדר היום שלו. מן הסתם הוא מבין שניסיתי לטמון לו פח, כי אינטליגנציה לא חסרה לו בכלל. כל מה שניסיתי להראות, בהפוכה, זה שהזמן הוא הגודל הקבוע שעל פיו יישק כל הסדר (והאי-סדר) העולמי. מזה חודשים רבים הוא משחק עם משוואות היחסות המציגות את הזמן כגודל משתנה ומגיע לפרדוקסים שאותם הוא מגיש לפתחנו בקול ענות גבורה תוך התעלמות “פעוטה” מן האפשרות הפשוטה שלתוצאות ניסוי מייקלסון מורלי יתכן הסבר אחר מזה המקובל.
    מאליו מובן כי הדברים שאתה אומר הינם יפים בעיני ומקובלים עלי.

  260. י.ח. ישראל וכל השאר.

    זמן , כמו ממדי המרחב, הם מושגים שהפרספציה שלנו ייצרה באופן אבולוציוני לצורך שרידות.
    זה שהפיזיקה *תפסה בעלות* על מושג הזמן לצורך המודלים שלה לא אומר שיש לה בעלות של ממש עליו.

    לפיכך מושג הזמן, כאמצעי של תפיסת המציאות, אינו בהכרח מוגדר על ידי אנטרופיה בלבד, או על ידי מושג פיסיקלי אחר. כל תהליך (למשל תנועת עצמים בסביבה של ייצורים חיים) מחייבת פרספציה של הזמן, כמו שהיא מחייבת פרספציה של המרחב. (בלי הפרספציות הללו צורות החיים לא ישיגו את משאבי החיים הדרושים להם וכן לא יוכלו לחמוק מאיומים על קיומם).

    לדעתי, צריך להתייחס למושג הזמן כאל מושג פרימיטיבי שאינו מחייב הנמקה, ממש כמו שמושגי המרחב התלת ממדי הם מושגים פרימיטיביים נטולי הנמקה. להערכתי, כל נסיון להגדיר מושגים פרימיטיביים של מרחב וזמן יביא לפרדוקסים בהגדרות, סוג של בוט-סטראפינג בהגדרות, שכן תופעות פיסיקליות שונות משתמשות במובלע במושגי המרחב והזמן (על כן אי אפשר להשתמש בהן לצורך הגדרת המושגים האחרונים). העובדה שלא מגדירים את הזמן (כלומר מתייחסים אליו כאל מושג פרימיטיבי) לא מונעת את האופציה למדוד אותו, לפחות למטרות פרקטיות (בעיני, הקוסמולוגיה של היקום היא היפותזה לא פרקטית אבל זה כבר סיפור אחר) .

    לגבי גידול האנטרופיה. זה קיים כנראה בכל המערכות הדינמיות שניתנות להבנה, שכן היפוך כיוון האנטרופיה משמעותו תהליכים שקשים לפרספציה (ובוודאי שקשה לנתח אותם במודלים מתמטיים).
    אין לזה קשר לחוק פיזיקלי כזה או אחר, אפשר להגדיר אנטרופיות גם על מערכות לא פיסיקליות.

    אני עצלן מכדי לאמר יותר בעניין האנטרופיה, זה מסובך ופילוסופי מדי, גם אי אפשר לתת הסבר מלא למה שאני מתכוון. אומר רק זאת: גידול אנטרופיה על ציר הזמן מתאים לתיאור סיבה-תוצאה מהסוג “סיבה *אחת* גוררת תוצאות רבות”, מאידך, קיטון אנטרופיה על ציר הזמן מתאים לתיאור סיבה-תוצאה מהסוג “סיבות רבות *מסונכרנות יחד* גוררות תוצאה יחידה”. תיאור סיבה-תוצאה מהסוג השני קשה מאוד לפרספציה, אם בכלל אפשרי.

  261. אריה – שמח שירדת לסוף דעתי.
    מי יודע אולי במרכז, יש מבוע שמייצר יקומים וזה מה שנותן דחיפה להתפשטות היקומים.

  262. רובי – עכשיו ברור מאיפה הבאת את הבצל. אתה אומר לגבי הבלון בתוך בלון בתוך בלון “האם יש ראיה לכך שזה לא המצב?” את המשפט הזה אפשר לומר לגבי כל הנחה…יש מפלצת ספגטי מעופפת; האם יש ראיה שלא?. ואני אומר: אם יש יותר מבלון אחד – אז כל בלון הוא יקום אחר (גם אם הם אחד בתוך השני)!

  263. שוב אתה מתחמק, צלופח קטן, אבל כבר התרגלתי.
    וְלָמָשָׁל מן האגדות: לא! זמן ומהירות אינם אותו דבר כי אם דבר והיפוכו. אך לא כך הם פני הדברים עם הזמן והאנטרופיה, שהם מתקדמים ביחד.

  264. סליחה שהתחמקתי.
    כמו שדונקי אומר לשרק,
    דוגמה? (למה אתה מתכוון).
    הרי אם תסלק את הזמן מהנוסחאות, לא תהיה לך מהירות. האם זמן ומהירות הם אותו הדבר?

  265. י.ש
    היות שאתה מתחמק מלהרים את הכפפה, אעשה זאת במקומך. הטענה שלי היא כי “הזמן” ו”החוק השני של התרמודינמיקה” אלה שני שמות של אותו דבר בדיוק. תסלק את הזמן מן הנוסחאות ולא יהיה לך החוק השני של התרמודינמיקה. תתעלם מן השינויים במצב הסדר ביקום ולא תוכל למדוד זמן

  266. -לקראת סיום המאה התשע עשרה – פתח הפרופסור – חשבו הפיסיקאים כי מלאכתם כמעט ותמה.
    -חוקי המכניקה של ניוטון האנגלי, והאלקטרומגנטיות של מקסוול הסקוטי, נתנו תיאור כמעט מושלם של העולם הפיזי כקומפוזיציה של גלים, חומר..
    -הכל גלים – מלמל החוק.
    -גלים וגופים – תיקן הפרופסור.
    – גלים וגלים דחוסים- התעקש החוק.
    -התרמודינמיקה, המשיך הפרופסור, עם שני חוקיה רבי העצמה, צומצמה עם גילוי האטומים לענף של המכניקה הקינטית, ועכשיו, קיוו כולם, נוכל כולנו להתרווח בעולם הנוח והמסודר שארגנו עבורנו שני הג’נטלמנים הבריטים, וללגום כוס תה של אחר הצוהריים.
    -אה, אלו היו הזמנים, זו הייתה תקופה של ממש- נאנח הפרופסור ומחה אגלי נוסטלגיה ממצחו. -שערו בנפשכם את גודל האידיליה, את קסמו של יקום שבו המרחב מוחלט והזמן מוחלט והשוו אותם לעקה והמבולקה ששוררים כיום.
    -סבי מספר, שבעיר הולדתו גל היה גל, חלקיק היה חלקיק, ושום גל לא העז להתנהג כמו חלקיק או להפך. אלקטרונים היו חגים בחדווה סביב אמאלה גרעין, ברורים, חדים, מדידים, במסילות אליפטיות סגלגלות ומושלמות. נותרו עדיין כמה עניינים פעוטים להסדיר. קרינת גוף שחור, מהירות כדור הארץ דרך האתר – באמת לא ציד דובים ואריות, אלא לכל היותר מרדף עצל אחר עכברי שדה.
    -נו, אז מה קרה? התעניין החוק.
    -מהירות האור סירבה לשתף פעולה – ענה הפרופסור בחמיצות.
    -מה פירוש?
    -היא לא הסכימה להסתכם.
    -אולי תואיל להסביר לחוק מסכן וטיפש למה אתה מתכוון? התעצבן החוק.
    נהמת הסכמה נשמעה מן הקהל.
    -היא לא הסכימה להסתכם בצורה וקטורית ככל מהירות אחרת. ניסוי מייקלסון ומורלי הראה שמהירות האור קבועה תמיד, ללא קשר למהירות מקור האור, ולמעשה ללא קשר לכל גורם שהוא.
    – ומה כל כך נורא בכך? תמה החוק.
    -מה נורא? עכשיו הפרופסור הוא זה שהתעצבן. -כלום אינכם מבינים? הכל כבר היה מוכן ומזומן לתיאוריה מאוחדת גדולה פשוטה ומובנת של הפיסיקה הקלאסית. כל מה שרצינו לדעת הוא את מהירותנו דרך האתר. האם זה כל כך הרבה לבקש? פנה אל הקהל בקול בוכים.
    יבבות חנוקות עלו מן הקהל.
    -אבל לא, הפרימדונה לא הסכימה להסתכם בשום תנאי. אתה יודע כמה ביקשנו, כמה התחננו.. בסוף עלה הרעיון להעמיד את המנוולת לדין, אבל זה לא יצא אל הפועל.
    -מה קרה?
    -איינשטיין הודיע שעקב הנסיבות העגומות הוחלט לבטל את האתר, וסוכם שמהירות האור היא הקבוע המוחלט היחידי ביקום. כל השאר יחסי.
    -מה בדיוק יחסי?
    -כל הגורמים הפיזיקליים: זמן, מרחק, מסה – הכל.
    -פרופסור בולצמנסקי – אמר החוק בהדגשה. האם ניתן לעבור את מהירות האור?
    -לא. היא הגבול העליון.
    -מדוע?
    -כי מהירות גדולה ממהירות האור תהווה סתירה ל.. הפרופסור השתתק.
    -סתירה למה? חשבתי שאמרת שהיא המוחלט היחידי.
    הפרופסור מלמל מילים לא מובנות.
    -אתה מתכוון לענות לשאלה או שנכריז עליך כעד עוין?
    -סתירה לחוק השני..
    -איזה חוק שני? של המכניקה? של האלקטרומגנטיות?
    -של התרמודינמיקה – לחש הפרופסור.
    -והזמן? הצליף החוק ללא רחמים, -כיצד מוגדר כיוון הזמן?
    -כוון הגדלת האנטרופיה – לעלע הפרופסור.
    -הגדלה שהיא? דרש החוק.
    -החוק השני של התרמודינמיקה..
    -פרופסור בולצמנסקי – החוק דיבר כעת ברוך – אינך נאשם יותר, ואני עומד לשחרר אותך מהדוכן, אך עליך לזכור דבר אחד לתמיד:
    -קיים רק קבוע אחד בפיזיקה והוא החוק השני של התרמודינמיקה.
    כל השאר יחסי.
    הפרופסור חייך בברק של הבנה פתאומית.
    -האם למדנו את הלקח שלנו היום?
    הפרופסור הנהן.
    -כי אני לא רוצה שנאלץ לחזור על השיעור הזה פעם נוספת.
    הפרופסור הניד בראשו לעבר החוק וירד מן הדוכן לקול תשואות הקהל.

  267. בהנחה שתאורית המפץ הגדול נכונה, היקום מתפשט מהמרכז החוצה.
    אנלוגית הבלון מתארת ספירה חד שכבתית שמתנפחת עד אין סוף תוך כדי תאוצה.
    לדעתי יש לדופן הבלון עובי ואולי מדובר במספר בלונים מתנפחים אחד בתוך השני שהחיצוני מתפשט מהר יותר מהפנימי וכך כלפי המרכז.
    האם יש ראיה לכך שזה לא המצב?

  268. יובל.
    פילוסופיה אינה פטפוט. השילוב של פיזיקה ופילוסופיה הוא די מוצלח לדעתי. פשוט, זה מעבר לסקופ שלנו כרגע. רק זה חסר לנו עכשיו לקראת התגובה 900, עוד כיוון להתבדר אליו.

    לפי דעתי אם אנו מקבלים את תאוריית המפץ הגדול, אכן הזמן הוא גורם פיזיקלי מוחלט, המושפע מתאוצה אך לא מתנועה שאינה מואצת, וניתן לכימות כפונקציה של התפשטות היקום. לפני המפץ אין משמעות למושג הזמן. אין דבר כזה “מה היה לפני 20 מיליארד שנה”.

    לעומת זאת אם נחזור ל1905, הרי שתמונת המצב הוא של יקום נצחי, או כמעט נצחי. עבור פואנקרה 100 מיליארד שנים הן משחק ילדים, וזה רק שאלה של זמן עד שכל מצב יחזור על עצמו, והאנטרופיה תרד. לכן ביחסות הזמן הוא סובייקטיבי ולכל מודד יש את הזמן שלו, ושום זמן אינו מוחלט.

    אגב, טענתך שאנו במרכז היקום אינה נתמכת עיי תאוריית המפץ. לפי התאוריה אין דבר כזה מרכז היקום. כל נקודה רשאית לראות עצמה כמרכז.

  269. רובי – לא שאני מומחה גדול, אך מודל הבצל שלך אינו מובן ואינו מתאים לרדוקציה בממדים. אתה מנסה להביא את הבצל כמודל תלת ממדי וזה מה שלא מובן. המודל הראוי להבנה הוא מודל דו ממדי (משטח הבלון) רק שהמשטח עקמומי. היקום שלנו זה אותו הדבר רק בתוספת ממד (ולא בהכרח שצורתו כדורית). כלומר משטח הבלון הוא אנלוגי למרחב תלת ממדי.
    יובל – ככל שאני יודע, אף אחד לא יודע בבירור מה גורם לאנרגיה האפלה. דומני שאחת המחשבות זו אנרגיית הריק (ע”ע)?

  270. אריה סתר, בהתנצלות על הניסוח הלא ברור.
    לא התכוונתי לערוך לך בוחן בידע כללי אלא ביקשתי את דעתך. מה, לדעתך, גורם לאנרגיה האפלה?

  271. אריה – לדעתי אנלוגית הבצל טובה יותר לחישובים אינפיטיסימליים של התרחקות השכבות של קליפות הבצל כלומר התפשטות מואצת של היקום.
    האם מתצפיות אסטרונומיות יש מעבר של גלקציות מקליפה לקליפה אם נמשיך עם אנלוגית הבצל? אם לא אז אפשר להשאר אם אנלוגיה זו אם כן, צריך לבדוק מה הסיבה.

  272. יובל – התמדת המפץ הגדול בתוספת האנרגיה האפלה; לא כן? רובי – למה בצל? הבלון הוא מודל שלנו בהפחתת ממד אחד כדי שיקל עלינו להבין. הגלקסיות לא נעות יחד עם מעטפת הבלון (המרחב המתנפח), רק המרחק ביניהן גדל עם התנפחות המרחב.

  273. אריה- הייתי מדמה את היקום לקליפות בצל שמתנפחות ולא לבלון מאחר והיקום הוא תלת מימדי בתוספת וקטור הזמן.
    עדיין הגלקציות שנמצאות על אותן קליפות נעות יחד איתן באותה מהירות (גדולה ממהירות האור?) ואז זה סותר את האקסיומה של איינשטיין כפי שהמדע אימת נכון לעכשיו מלבד סטיות במקרים מסויימים.

  274. רובי – הגלקסיות עצמן לא נעות; היקום הנושא אותן הוא זה שמתנפח. תאר לך בלון ועליו כתמים המייצגים את הגלקסיות. הבלון מתנפח והכתמים הרחוקים אחד מהשני מתרחקים זה מזה במהירות העולה על מהירות האור למרות שהכתמים עצמם אינם נעים.

  275. אריה,
    חשבתי שמהירות האור היא מכסימלית. איך זה יכול להיות שגלקציות שהן חומר ולא אנרגיה או פוטונים יכולות לנוע במהירות גדולה ממהירות האור?
    לגבי גיל היקום הידוע הוא כ 15 מ’ שנה אם יש גלקציות מעבר למרחק של 15 מ’ שנות אור ותאורית המפץ הגדול נכונה, זה אומר שההתפשטות שלהן היתה באמת מעבר למהירות האור.

  276. רובי – הגלקסיות הכי רחוקות שאפשר לראות הן מתחילת קיום היקום. גם הגלקסיות שאורן לא יגיע אלינו אף פעם ואפילו אלה הכי רחוקות מאיתנו, גילן אינו עולה על גיל היקום; הם פשוט התרחקו (ומוסיפות להתרחק) מאיתנו במהירות העולה על מהירות האור.

  277. יובל שלום,
    אתה צודק, בילבלתי בין זמן למרחק אבל עדיין נשאלת השאלה:
    הקרניים שנצפות מהגלקציות הרחוקות עברו מרחק של 14.5 מ’ שנות אור. בזמן שהן עשו מרחק זה, אותן גלקציות המשיכו להתקיים ולהתרחק במשך אותו פרק זמן שאינני יודע מהו )אשמח לדעת מהו) אבל הו חייב להיות זמן כפול מהזמן שלקח לקרני האור להגיע לכאן.
    קוטר היקום חייב להיות גדול מ 58 מ’ שנות אור בשל האצת ההתפשטות כפי שנצפה ופורסם.
    איך הגעתי ל 58 מ’ שנות אור? 14.5 מ’ שנות אור מרחק ראשוני ועוד מרחק כזה לפחות בשל ההאצה כפול שניים בשל הקוטר.
    רובי

  278. יובל – אכן הגלקסיות מעבר ליקום הנראה מתרחקות מאיתנו במהירות האור ויותר ממנה – הלא כן? (מיקומן ביקום מתרחק מאיתנו במהירויות אלה)

  279. רובי
    מן הטענה שלך אפשר להבין כי ברגע בו יצא האור לדרכו מן הגלקסיות הרחוקות אלינו, גיל הגלקסיות היה 14.5 מש”א.
    אתה גם מערבב את המושגים “זמן” ו”מרחק” (שנת אור היא יחידת מרחק, לא יחידת זמן) ואפשר להבין כי אתה סבור שהגלקסיות מתרחקות מאיתנו במהירות האור.
    ולבסוף, לא נהיר לי די הצורך איך הגעת למספר 59. האם התכוונת למספר 58?

  280. אריה שלום,
    לגבי היקום הנראה, אם הגלקציות הרחוקות ביותר שלחו אלינו את קרני האור שלהן לפני 14.5 מ’ שנות אור, אני טוען שגיל אותן גלקציות הוא כפול כלומר 29 מ’ שנות אור כי בזמן שלקח לקרניים להגיע לכאן הן התקיימו ואף התפשטו בדיוק אותו משך זמן כלומר 14.5 מ’ שנות אור נוספות.
    כלומר קוטר היקום נכון לעכשיו (כפי שיראה בעוד 14.5 מ’ שנות אור) הוא 59 מ’ שנות אור.
    האם אני טועה?
    רובי.

  281. אריה סתר
    על פי המודל הרווח של “המפץ הגדול” גיל היקום וגודלו הינם פונקציות הפוכות אחת של השניה (בהינתן גיל היקום אפשר לחשב את גודלו, ובהינתן גודלו אפשר לחשב את גילו).
    מאידך, אני שותף לדעה שביטאת במשפט “זה שהיקום גדול בהרבה מהיקום הנראה, לא אומר שגיל היקום בשנות אור הוא כגודל היקום”. הסיבה לכך שאינני אוהב את מודל “המפץ הגדול” היא כי ההנחה שאכן היה מפץ גדול אשר ממנו התפזר החומר ביקום לכל הכיוונים ללא העדפת כיוון אחד על אחרים, יחד עם התצפיות המראות את אופק היקום במרחק קבוע מאיתנו לכל כיוון, מציבות אותנו במקום טוב באמצע היקום בדיוק היכן שהמפץ הגדול פרץ.

  282. ישראל,
    נראה ששוב לא הובנתי, כרגיל. לא התכוונתי לפטפט פילוסופיה. למדנו מן היחסות כי לא הזמן הוא הקבוע (כי אם מהירות האור), ולכן אפשר לראות בו גורם פיסיקלי משתנה. אם עד היום אמרנו כי האנטרופיה (במאקרו) גדלה עם הזמן, וההנחה הנוכחית היא כי הזמן הוא יישות פיסיקלית, הרי הזמן בעצמו הוא זה שעושה את כל הבלגן. עד היום נוהגין הבריות להגדיר מהירות כנגזרת של משהו (למשל מרחק) על פי זמן, אך היות שלא הזמן הוא הקבוע האמיתי (אלא מהירות האור, כאמור), יש לשנות את נושא הנוסחה ולהגדיר מחדש בהתאם. האם, בתנאים החדשים האלה תוכל להראות כיצד הזמן (“שיני הזמן”) מכרסם בסדר הטוב?

  283. חברים; לא עניתם על מה שרובי שאל. הוא מבלבל בין גיל היקום לגודל היקום. היקום הנראה משתרע למרחק של כ-14 מיליארד שנות אור לכל כיוון – כי אי אפשר לראות משהו שרחוק בשנות אור יותר מגיל היקום כי מה שמעבר לכך – קרני האור לעולם לא יגיעו אלינו – בגלל האצת ההתפשטות. אבל זה שהיקום גדול בהרבה מהיקום הנראה, לא אומר שגיל היקום בשנות אור הוא כגודל היקום.

  284. בגלל המתיחות בדרום, מתארכת תקופת ההמתנה של תגובות. ננסה לעשות אינטגרציה בחלקים. אם אותם דברים יופיעו שנית בקרוב, האשמה היא בהבלגה ובהמתנה.
    מזכיר קצת את ההמתנה לפני מלחמת ששת הימים.

    יובל
    “החוק השני של התרמודינמיקה הוא בבת עינך, מסתבר.”

    החוק השני הוא אבו נזק לא קטן. אם תבדוק טוב, בשורה התחתונה הוא זה שאחראי לכל צרה שהיא.
    אין טוב ורע. יש רק סדר ואנטרופיה. כל אדם, טוב או רע, אם הדבר תלוי בו, יבחר בטוב בשביל בנו. אם אדם בוחר ברע בשביל עצמו או אחרים, זה אך ורק בכורח הנסיבות העגומות, שבבסיסן החוק השני של התרמודינמיקה.

    “האם אתה יכול להראות כיצד הזמן איננו סתם מושג ערטילאי שתהליכים נמדדים לפיו (“עם הזמן”, “לאורך זמן”, וכו’) אלא הוא גורם להם באופן עצמאי (למשל, “הזמן יעשה את שלו”)?”

    הגורם הפילוסופי מעניין אותי הרבה פחות מהגורם הפיזיקלי. קשה מאוד להגדיר זמן בפיזיקה. אני מאמין שההתיחסות לזמן שונה בשתי התאוריות המקובלות כיום, המפץ הגדול והיחסות. לפי המפץ הגדול, לא היה זמן בכלל לפני 13.7 מיליארד שנה (מה היה קיים מיליארד שנה לפני המפץ? לא שאלה חסרת משמעות?). לעומת זאת כשאיינשטיין הגה את היחסות ב1905 תמונת היקום הייתה של יקום אינסופי ונצחי. לכן ההתנגשות.

    בדיון שלי עם ר.ח. הוא חוזר ומעלה את הטיעון הבא: “זה מד טמפרטורות שמודד פרמטר מחוץ למערכת.” אני מנסה להביא אותו לראות את הסתירה שקיימת בין הנתונים השונים, ושהפתרון הוא בכך שהפרמטר נמצא בתוך המערכת.

    בקשר לפטור – ביטול איננו בקשה מפורשת. נראה לי שאתה מסתדר די טוב עם כל החברה האחרים (או שאולי לא ממש? שבהשוואה לדבריהם אני צפיחית בדבש). אבל עדיין אני מאמין שכולם פה מעוניינים לשמוע את המשך המודל, ומתי הוא יוצא משלב ההגדרות לשלב התאוריה, ההוכחות, הנוסחאות ומבחני ההפרכה.

  285. יובל
    “החוק השני של התרמודינמיקה הוא בבת עינך, מסתבר.”

    החוק השני הוא אבו נזק לא קטן. אם תבדוק טוב, בשורה התחתונה הוא זה שאחראי לכל צרה שהיא.
    אין טוב ורע. יש רק סדר ואנטרופיה. כל אדם, טוב או רע, אם הדבר תלוי בו, יבחר בטוב בשביל בנו. אם אדם בוחר ברע בשביל עצמו או אחרים, זה אך ורק בכורח הנסיבות העגומות, שבבסיסן החוק השני של התרמודינמיקה.

    “האם אתה יכול להראות כיצד הזמן איננו סתם מושג ערטילאי שתהליכים נמדדים לפיו (“עם הזמן”, “לאורך זמן”, וכו’) אלא הוא גורם להם באופן עצמאי (למשל, “הזמן יעשה את שלו”)?”

    הגורם הפילוסופי מעניין אותי הרבה פחות מהגורם הפיזיקלי. קשה מאוד להגדיר זמן בפיזיקה. אני מאמין שההתיחסות לזמן שונה בשתי התאוריות המקובלות כיום, המפץ הגדול והיחסות. לפי המפץ הגדול, לא היה זמן בכלל לפני 13.7 מיליארד שנה (מה היה קיים מיליארד שנה לפני המפץ? לא שאלה חסרת משמעות?). לעומת זאת כשאיינשטיין הגה את היחסות ב1905 תמונת היקום הייתה של יקום אינסופי ונצחי. לכן ההתנגשות.

    בדיון שלי עם ר.ח. הוא חוזר ומעלה את הטיעון הבא: “זה מד טמפרטורות שמודד פרמטר מחוץ למערכת.” אני מנסה להביא אותו לראות את הסתירה שקיימת בין הנתונים השונים, ושהפתרון הוא בכך שהפרמטר נמצא בתוך המערכת.

    בקשר לפטור – ביטול איננו בקשה מפורשת. נראה לי שאתה מסתדר די טוב עם כל החברה האחרים (או שאולי לא ממש? שבהשוואה לדבריהם אני צפיחית בדבש). אבל עדיין אני מאמין שכולם פה מעוניינים לשמוע את המשך המודל, ומתי הוא יוצא משלב ההגדרות לשלב התאוריה, ההוכחות, הנוסחאות ומבחני ההפרכה.

    משפטו של החוק השני.

    1. בולצמנסקי.

    אולם המשפט המה מפה אל פה.
    אכן, אין זה מחזה נפוץ שחוק עומד למשפט פלילי.. ולא סתם חוק: החוק השני של התרמודינמיקה, אולי הבסיסי ביותר בין כל חוקי הפיזיקה.
    לא היה זה משפט במובן הרגיל של המילה. החוק השני לא הובל לאולם כשהוא כבול באזיקים, לא הוצאה פקודת מעצר, והחשוב מכל – גם שופט לא נכח במשפט!
    החוק השני העמיד דברים על דיוקם מן ההתחלה: כתנאי להסכמתו לעמוד למשפט, דרש החוק- וקיבל- זכויות מלאות של סניגור ושל קטיגור כאחת. משפט אכן יהיה, אך הוא יהיה דו כיווני, כשלכל צד זכויות וחובות שוות. מיהו השופט ומיהו הנשפט – זאת, טען החוק, יתברר במהלך המשפט.
    תשאל אפוא: מה הטעם במשפט מעין זה? מי יפסוק את גזר הדין? ולמי סמכויות האכיפה?
    התשובה לשאלה הראשונה הנה שהמשפט חשוב מאין כמותו, אולי החשוב ביותר שנערך אי פעם, וזה יתברר במהלך המשפט. התשובה לשאלה השניה: מושבעים. מי? גם זה יתברר במהלך המשפט. ולשלישית, סמכויות האכיפה, בודאי כבר ניחשת..זה יתגלה במהלך המשפט.

    ולכן לא נכביר מילים, וניגש מיד אל מהלך המשפט.
    ראשון דיבר התובע מטעם המדינה.
    -כבוד בית הדין.
    ברצוני להציג בפניכם פושע מיוחד במינו.
    הנאשם שניצב היום לפניכם, הנאשם שנקרא “החוק השני של התרמודינמיקה” וידוע גם בכינויו “אנטרופיה”, אינו רק הפושע הגדול ביותר של כל הזמנים – הוא למעשה גם הפושע היחיד של כל הזמנים.
    המיה נרגנת עלתה מן הקהל.
    -פשעו, הרעים התובע בקולו, הוא גרימת כל פשע!
    כל תקלה , כל מכאוב, כל עצב וכל רוגז – מקורם הנו החוק השני, הוא ורק הוא.
    אני טוען כי לחוק השני יש והייתה אך ורק תכלית אחת מיום הולדתו לפני ארבע עשרה וחצי מיליארד שנים ועד עצם היום הזה: הגדלת אי הסדר ביקום.
    עצם המילה עצמה – אנטרופיה – פירושה כאוס, אנרכיה, התפרקות כל סדר. התביעה תוכיח ללא כל שמץ של ספק את חוסר מוסריותו של החוק, את הרשע המושרש שבו, את האידיאולוגיה המעוותת…..

    התובע המשיך לתאר בציוריות עד כמה איום ואכזר הוא החוק. במקום מושבו העווה החוק את פניו בבוז, ופלט גיחוכים והערות ציניות שבינו לבין עצמו. בשלב מסוים איבד החוק את סבלנותו וקטע את דברי התובע:
    -אולי תפסיק כבר לקשקש ותתחיל בהצגת העובדות?
    המולת קולות נרעשת נשמעה בקהל. התובע החוויר. ניסיונו רב השנים היה כעת בעוכריו: מאז ומתמיד הורגל במשמעת הקפדנית של בתי המשפט, שהיו קוראים לסדר מיידי כל מי שהעז לחרוג מן הנוהל המשפטי התקין.
    מתוך הרגל שלח מבט חסר ישע אל מקום מושבו הריק של אב בית הדין, הבין שמשם ישועה כבר לא תגיע היום, וקיבל על עצמו את הדין.
    -אני קורא אל הדוכן את העד המומחה הראשון, פרופסור בולצמנסקי, הפיזיקאי הבכיר במדינה.
    פניו של החוק השני אורו. -פיזיקאי! הריע בקול. כמה נפלא! עובדות! הוכחות! הגיון! הוא הנמיך את קולו כממתיק סוד עם הפרופסור.- אני אוהב פיזיקה ופיזיקאים. אתה בטח יודע שגם אני עצמי חוק פיזיקלי? אשמח להעמיד את עצמי לשירותך, אדוני הפרופסור. תוכל לצפות ממני לשיתוף פעולה מלא.
    פרופסור בולצמנסקי לכסן מבט מחמיר לעבר החוק כאומר: “נראה בגמר העדות עד כמה תאהב פיזיקאים, ולכמה שיתוף פעולה נוכל לצפות ממך”.

    -החוק הראשון של התרמודינמיקה , פתח הפרופסור, הוא בעצם חוק שימור האנרגיה: אנרגיה אינה יכולה להווצר או להעלם מעצמה. תמציתו של החוק הנה: במערכת סגורה סכום האנרגיות – מכנית, חשמלית, פוטנציאלית, חומנית וכו -תמיד קבועה.
    -אח גדול – למלם החוק השני בערגה כשהוזכר החוק הראשון – חוק מקסים, אבל מאד מאד שמרני.

    -חוק זה- המשיך הפרופסור, מתעלם מן ההפרעה, הינו הוגן ומוסרי ביסודו. הוא מעניק תמורה הולמת לעבודה, ומעמד שוויוני לכל צורות האנרגיה. כל אנרגיה יכולה להתחלף באחרת, וכולן שוות בפני החוק.

    פניו של החוק השני נתכרכמו. “עבודה?”, צנף, -מאיפה הבאת את המונח הזה פתאום? איפה בכלל תמצא עבודה במערכת תרמודינמית סגורה? אתה פיזיקאי בכלל?
    -זו בדיוק הנקודה שעליה אתה עומד היום למשפט, ענה הפרופסור בצינת קרח. -למרות שעבודה אפשרית מבחינה תיאורטית בכל מערכת שהיא – ומעולם לא שמענו למשל את החוק הראשון מתנגד לה – הרי שבגלל קיומו של החוק השני העבודה מוגבלת אך ורק למערכות שכוללות אלמנטים ביולוגיים.
    התובע התערב, שמח לקחת נקם על כבודו שהושפל. -אתה רוצה לאומר- פנה אל הפרופסור, -שבכל היקום הגדול לא תמצא עבודה בשום מקום מלבד בכדור הארץ או במקומות אחרים, אם הם קיימים, בהם יש חיים? וכל זאת בגלל הנאשם??
    הצעקות מן הקהל הופנו אל החוק השני. -בטלן! עצלן! לך לעבוד במקום להפריע לחיים!
    -שתקו! צעק החוק בחזרה,- ותגידו תודה על כך שאתם בכלל חיים, ובזכותי!
    -אני רוצה לאמר- הטעים פרופסור בולצמנסקי כל הברה, -שמאז הווצרות היקום ועד הופעת החיים, לא בוצעה בו כל עבודה – במובנה הפיזיקלי – וזאת בזכותו של ידידנו כאן, מר אנטרופיה, החוק השני של התרמודינמיקה.
    -נו אז מה? היתמם החוק. -את מי בכלל זה מעניין? למה שלא תשחררו אותי לנוח, ולא תעשו את אותו הדבר?
    התובע, שחש כי הגה המשפט נשמט מידיו, מיהר להתערב. -ספר לנו על טבעו ואופיו של הנאשם- אמר לפרופסור, ובלחש הוסיף: -הפוך אותו לעפר ואפר.
    פרופסור בולצמנסקי הניד קלות בראשו, ופתח: -אם החוק הראשון עוסק בשימורה של האנרגיה, הרי שהשני מתווה את כוון תנועתה בתוך המערכת.
    החוק השני קובע שאנרגיה תנוע תמיד לכוון שיהפוך אותה לפחות מרוכזת.
    -ו.. פיהק החוק.
    חום תמיד יזרום מגוף חם לגוף קר יותר. לעולם לא להפך.
    -אם,- המשיך הפרופסור תוך שהוא אוחז בדסקית עץ שהביא לצורך ההדגמה -ארפה מן הדסקית-
    הוא הרפה -תיפול הדסקית לארץ.
    הדסקית נפלה בקול נקישה.
    -אם אגלגל אותה, היא תמיד תתגלגל במורד.
    -אם אדליק אותה, היא תישרף.
    אבל התהליך לעולם לא יקרה מעצמו בכוון ההפוך, למרות שאין מניעה תיאורטית לכך.
    -לא בדיוק… מלמל החוק.
    -הפסק להפריע- פקד הפרופסור. -אתה יודע כמוני שכך בדיוק מתנהלים הדברים.
    -נו טוב , התרצה החוק, אז יש כוון מועדף לזרימת האנרגיה במערכת. האם על כך אני עומד למשפט? אני מודה מיד באשמה. אתם אנשים עסוקים, אני חוק עסוק, אולי נסיים ונתפנה לע..
    -הסיום עודנו רחוק- הרים הפרופסור את קולו. -למעשה אנחנו רק בהתחלה. לאחר שתיארתי את אופיו הפיזיקלי של החוק, אגש לתאר את ההשלכות ההרסניות שלו לחיי היומיום.
    הקהל נדרך בציפייה.
    -אולם ראשית עלי לציין עובדה פיזיקלית הנובעת מן החוק: בכל מערכת פעילה יש תמיד אבדן מסוים של אנרגיה השמישה לצורך בצוע עבודה-
    -עוד הפעם אתה והעבודה שלך-
    – כל צורות האנרגיה שואפות תמיד לרדת לאנרגית חום. החום- קולו של בולצמנסקי נשבר- החמימות הנעימה שעליה אנו מתרפקים – הנה צורת האנרגיה הנמוכה ביותר – קולו הפך ליבבה – הבזויה ביותר-
    הקהל געה בבכי.

  286. רובי
    אם תפשפש בתגובות בתחילת הדיון תגלה שכבר ענו על השאלה ששאלת.
    בגדול, גילויו של האור המגיע מגלקסיות רחוקות תלוי ביכולות קליטת האור של המכשור הצופה אל הגלקסיות ההן. כלל שהיכולות של המכשיר יותר מתקדמות ברמה טכנולוגית, ניתן יהיה לצפות למרחק רב יותר באמצעות המכשיר. הטלסקופים של היום יכולים לקלוט אור ממרחק רב, אבל עדיין קיימת בעיה של לקלוט אור בזמן אמת. ולמה הכוונה: האור שהטלסקופ קולט ממרחק של 14 מ’ שנות אור, מראה את תמונת המצב של האיזור כפי שהיה בזמן שהאור יצאה לדרכו מאותו האיזור שעליו צפה הטלסקופ.
    ידוע (ומוסכם על רוב המדענים) שקצב התפשטות היקום- מרחיק את הגלקסיות אחת מהשניה.
    התופעה הזאת מצביעה על מקרה ביקום שלנו: שהגלקסיות הרחוקות עליהן צופים דרך הטלסקופ אינן באמת נמצאות באותו האיזור בזמן (האמיתי) שבו צופים עליהן. זה תקף על כל גלקסיה. הגלקס’ הכי קרובה אלינו נמצאת כמה שנות אור בודדות מאיתנו – וזה אומר שכאשר מתקבלות תמונות סטילס מהטלסקופ שמראות את הגלקסיה, אזי התמונות מראות את מצבה כפי שהיה לפני כמה אותם שנים בודדות.

    היום- אותן גלקס’ הכי רחוקות מראות על מצבן כפי שהיה לפני 14 מ’ שנים.
    זאת אומרת שהיום (בגלל תופעת האצת התפשטות היקום) הגלקסיות אמורות להיות באיזור שהמרחק אליו מאיתנו הוא גדול יותר מ14 מ’ שנות אור. לפי הספקולציה, הגורם לכך שהגלקסיות מתרחקות הוא האנרגיה האפלה. לכן לפי החישובים (אם מתחשבים באנרגיה אפלה) – הגלקס’ הרחוקות ביותר אמורות להימצא היום במרחק של 46מ’ שנות אור. כלומר בפועל, בזמן אמת (כרגע), הגלקס’ צריכות להיות כ46 מ’ שנות אור מכדור הארץ.
    מכאן שהקוטר האמיתי (נכון להיום) של היקום הוא לא 29 מ’ שנות אור, אלא 93 מ’ שנות אור.

  287. רובי,
    למה אתה חושב שאתה טועה?
    לכל כיוון אליו נסתכל האופק נמצא במרחק 14.5 מיליארד שנות אור מאיתנו. אם נראה את עצמנו יושבים במרכז היקום (וקודם צריך עוד להניח שליקום יש מרכז), הרי קוטר היקום הוא אכן 29 מיליארד ש”א. אבל ההנחה הזאת אין לה על מה להסתמך. לא מן הנמנע שהמספר הזה נובע ממגבלת המיכשור שלנו ולא מן המצב האמיתי של היקום.

  288. ישראל,
    “חשוב רק על המערכת של ג’יל. היא רואה אצלה את שעון הטמפ’ מסתובב פי מיליארד משעון הצזיום. וזאת למה? בגלל שג’ק נוסע יחסית אליה במרחק של מיליארדים רבים של קילומטרים? מה יקרה אם ג’ק ישנה כיוון ויטוס במהירות 0 יחסית אליה, האם גם שעון הטמפ’ של ג’יל יאט פתאום את מהלכו ויסתובב באותו קצב כמו שעון הצזיום?”

    מה הקשר בין ג’יל לג’ק ??? היחס בין השעונים של ג’יל יהיה תמיד זהה כתלות במהירות שלה ביחס למצב מנוחה או אם תרצה למהירות האור. גם אם ג’ק חס וחלילה יושמד היחס בין שעוניה של ג’יל לא ישתנה. אין שום תלות בינם. האם לדעתך יש תלות שכזו??

  289. ישראל,
    הענקתי לך פטור כדי שתוכל להקדיש את זמנך לדברים שאתה אוהב באמת. אבל פטור איננו איסור. אם בוער לך לרדת מתאוריות-העל לרמה הפרימיטיבית שלי, הרגש חופשי.
    החוק השני של התרמודינמיקה הוא בבת עינך, מסתבר. האם אתה יכול להראות כיצד הזמן איננו סתם מושג ערטילאי שתהליכים נמדדים לפיו (“עם הזמן”, “לאורך זמן”, וכו’) אלא הוא גורם להם באופן עצמאי (למשל, “הזמן יעשה את שלו”)?

  290. יש לי שאלה לגבי גיל היקום למומחים:
    אם הציוד הקיים שיש ברשות האסטרונומים מזהה גלאקציות שהאור שלהם הגיע מלפני 14.5מיליארד שנות אור הווה אומר שמרגע שיציאת הקרניים לכוון כדור הארץ אותם גלאקציות המשיכו להתרחק ולהתקיים זמן זהה של 14.5 מיליארד שנות אור, זאת אומרת שגיל היקום (לפחות של אותן גלאקציות) צריך להיות 29 שנות אור נכון לעכשיו ולא כמו שנאמר.
    מישהו יכול להסביר לי איפה אני טועה?
    תודה
    רובי (ראובן)
    חג פורים שמח ושבת שלום

  291. ר.ח.

    אם ענית כן ל2 השאלות, הרי מה שמשתמע הוא (ותקן אותי אם אני טועה):

    1. אם ג’ק סופר משווה את מס הסיבובים בין 2 השעונים (ואפשר בהחלט לסדר ששעוני הצזיום והטמפ’ יראה זמן בסיבובי מחוגים) הוא יראה יחס של 1:1. הוא איננו צריך למעשה לחכות כל הזמן עד למפגש. בכל פרק זמן נתון שהוא בודק, היחס יהיה זהה.

    2.אם ג’יל סופרת, היא תקבל יחס של נגיד, 1,000,000,000:1

    הבעיה היא זו:

    1. גם ג’ק וגם ג’יל נמצאים מבחינתם במנוחה. המערכות שלהם אינן מואצות, ונקראות בשפת היחסות “אינרציאליות”. אילו היה ניתן להבדיל בינהן עיי פעולה של ספירת סיבובים לדוגמה, זה היה סותר את הפוסטולט הראשון של היחסות.
    ההסתייגות הזאת תתפוס גם אם תאמר שאחד מהשניים נע יחסית למערכת הCMBR, למרות שאפשר להתווכח עם הסתייגות כזו. (לא בהצלחה לדעתי. נגיע האליה אם נגיע לCMBR).

    2. חשוב רק על המערכת של ג’יל. היא רואה אצלה את שעון הטמפ’ מסתובב פי מיליארד משעון הצזיום. וזאת למה? בגלל שג’ק נוסע יחסית אליה במרחק של מיליארדים רבים של קילומטרים? מה יקרה אם ג’ק ישנה כיוון ויטוס במהירות 0 יחסית אליה, האם גם שעון הטמפ’ של ג’יל יאט פתאום את מהלכו ויסתובב באותו קצב כמו שעון הצזיום?

    יובל – כמובן שאתה יכול להמשיך, אבל זכור שיש לי פטור.

    רפאים לאן נעלמת? התגעגעתי להתקפות הפראיות של המעריץ הסודי שלי.

    אגב, האם תגובה זו:

    https://www.hayadan.org.il/confusion-about-evolution-3008116/#comment-319326

    תגרום לך לסתום את פיך הגדול והמטופש?

    או שמה הערות אילו מפי העורך והעורך המדעי של הירחון בחסות האוניברסיטה העברית?

    שלום ישראל,
    תשמע – מרגע שהתחלתי לקרוא את הפרק שלך פשוט לא יכולתי להפסיק… מרתק.
    גם העורך המדעי צבי עצמון התלהב. אנא קרא את תגובתו, יש בה הערות חשובות מאוד.
    מצורף גם הטקסט שלך עם הערותיו הספציפיות של צבי.
    (אחרי שנגמור עם הפרק הזה נדבר על ההמשך… נראה לי שבפרקים המתוכננים הבאים מחריפה הבעיה שקיימת גם בפרק זה: אתה מניח שכל הקוראים בקיאים לגמרי במטריה ולפיכך יכולים לעקוב בקלות אחרי ההתכחמויות וההערות הציניות. כאן זה אולי לא נורא, אבל כשמתעסקים עם גדל ויעקוביאנים וכו’ עלולים לאבד את הקורא הממוצע לגמרי…)
    מיקי

    ראשית, אנחנו מנוולים, ואותנו צריך להעמיד למשפט, אם לא נפרסם. חד משמעית.
    שתקום צעקה, לא נורא. זה שווה ויכוח.
    שנית, לא את הכל הבנתי, אני מודה. יש גם בעיה בסוגה (ז’נר) זה – קשה לדעת בביטחון מי ומתי כל דובר אומר אמת או ציני או מתחכם. וזה עלול להטעות את הקוראים. לכן צריך להקפיד לתת רמזים שקופים מתי הטענה היא מה שהדובר באמת חושב, או מתי הוא ציני או מנסה להוליך שולל. זו בעיה שצריך להקפיד בה.
    שלישית, יש דברים שבהנחה שהבנתי אינני מסכים אתם, או שאני תוהה למידת דיוקם. החלק של העניין הסוציולוגי-מדיני: לא לגמרי ברור לי באיזו מידה הדברים הכתובים הם דעת המחבר, או הצגה של גישות שהוא מתנגד להן.
    כל זאת סימנתי.
    צבי

  292. האחריש או אויל אחשב? 🙂
    התגובות “הקוטלות” שקיבלתי עד כה, גם אם מוצדקות, חוזרות על עצמן וכבר אינן מועילות לי. גם התגובות המעודדות לא מועילות הרבה מעבר לעשיית מצב רוח טוב.
    ראיתי כאן הרבה דיונים במודלים מתקדמים, אך לא ראיתי התעניינות רבה בבסיס הפיסיקה. יש מצב שאמשיך?

  293. ישראל, “אם אין לך מה להגיד – אל תגיד את זה בקול רם.” (משלי, יז כח)

  294. שמעתי על שירות חדש של הידען: תגובה ממתינה.

    בינתיים, עוד מעלילותיו של החוק השני.

    פרופסור בולצמנסקי ישב על כסא הנאשם, מרעיד קלות מצינה ומחשש מפני הבאות.
    החוק השני של התרמודינמיקה סקר אותו במבט שארך, כך נדמה לפרופסור, נצח נצחים.
    -פרופסור בולצמנסקי- דיבר החוק במהירות, – מדוע אתם המדענים נטפלים אל חוקי הפיזיקה ומחרידים אותם ממקום רבצם?
    -מה? נזקף הפרופסור בכיסאו. -מי זה נטפל אל חוקי הפיזיקה? חוקי הפיזיקה הם טובים ישרים וחברותיים לחיים. אתה החבלן היחיד!
    -באמת כך? שאל החוק בביטול.
    -בדיוק כך! החוק הראשון של התרמודינמיקה הוא חוק נפלא. חוקי התנועה – מלאכת מחשבת. חוק המשיכה – גאוני. חוקי החשמל, המגנטיות, דינמיקת הנוזלים…
    -ומכניקת הקוונטים? קטע אותו החוק.
    הפרופסור שתק.
    -מבנה האטום? חלקיקי היסוד? דעיכה רדיואקטיבית?
    הפרופסור מילא את פיו מים.
    -קרה משהו? העמיד החוק פנים מודאגות.
    עכשיו הפרופסור דיבר במהירות. -אינני אוהב את מכניקת הקוונטים. האמת היא שאינני אוהב את כל הפיזיקה המודרנית. חלקיקים שהם גלים, אלקטרונים קופצים ממקום למקום בלי להיות באמצע, הכוח החזק שמוערך רק בקירוב ומשנה את כוונו ללא הודעה מוקדמת.. אי ודאות… אי ודאות…
    -ותורת היחסות? חקר החוק.
    -זה כן! שמח הפרופסור. זוהי תיאוריה מקסימה שבמקסימות! אולם תורת היחסות הנה מתמטית ביסודה, מתמטיקה נפלאה ושמיימית.. הפרופסור נמלא התרגשות כשעסק בנושא האהוב עליו.
    -עקרונותיה, במיוחד של היחסות הכללית, קשים במקצת להבנה, אולם מהרגע שהובנו הנם עקביים וחד משמעיים, זאת למרות -הפרופסור צחקק לעצמו- שהגיאומטריה שלה קצת עקומה.
    -משהו מצחיק? תבע החוק.
    -לא.. שום דבר..בדיחה פרטית..חשבתי על רימן…
    -כלום תוכל לצטט לנו ממשוואותיה המפורסמות של תורת היחסות?
    -כמובן! ענה הפרופסור בידענות. -הטרנספורמציות של לורנץ, הקושרות את מסתו ואורכו של גוף למהירותו, או הנוסחה הידועה: E=MC ברבוע, האנרגיה של גוף שווה למסתו מוכפלת במהירות האור ברבוע..
    -נאשם!- קטע אותו החוק ביבושת -האם נכון הדבר שבעבר רציתם אתם הפיזיקאים להעמיד לדין את מהירות האור?
    תדהמת הזדעזעות באולם המשפט על שם סולוניו!
    -לא!!! נזדעק הפרופסור. לא בדיוק!! אתה מוציא דברים מהקשרם!
    -לא? האם אתה רוצה שנזמן את מהירות האור לעדות? היא ידידה ותיקה שלי, אתה יודע.
    -אני… אנחנו…הפרופסור התקשה לעכל את התפנית המהירה במשפט, ואת בוגדנותו של הקהל שעכשיו שרק לו בוז. -בכלל, מה פתאום אני נאשם? זהו משפטו של החוק השני! אני מוחה! אני רוצה ללכת הביתה! הוא פנה לעזוב את דוכן הנאשמים אך נעצר עיי שני סדרנים חסונים.
    – ידעת שזהו משפט דו כווני כשהתנדבת להעיד, ועכשיו תציית לחוק – אמר החוק בחומרה. שב במקומך. אני רוצה להציג את מוצג מס 1 – הוא שלף פנס – ולזמן לעדות את עדת התביעה מס 1, את הזריזה שבזריזות -הוא לחץ על כפתור הפנס, -מהירות האו..
    -לא לא! קרא הפרופסור. אני מודה! תנו לי רק לשתות כוס מים ולהתאושש, ואני אספר הכל.
    -החוק השני כיבה את הפנס, ארשת של ניצחון שפוכה על פניו.

  295. שוב התגובה ממתינה לאישור. פשוט לא יאומן.
    נראה לי שהשם שלך ישר**אל הוא המילה הבעייתית.

    הנה התגובה הקודמת ללא השם המפורש

    עד שהתשובה הקודמת שלי תשתחרר בוא נעשה את זה פשוט.
    1) כן
    2) כן

    הערה: על מנת לחשב את הזמן מטמפרטורה או התפרקות רדיואקטיבית (קרי צזיום) אתה חייב להוסיף לנוסחא את המהירות של המודד. במהירויות נמוכות זה זניח אולם במהירות קרובות לאור זה משמעותי. לא אני אמרתי, זה איינשטיין.

    עדיין לא הבנתי, היכן הסתירה?

  296. ישראל,
    עד שהתשובה הקודמת שלי תשתחרר בוא נעשה את זה פשוט.
    1) כן
    2) כן

    הערה: על מנת לחשב את הזמן מטמפרטורה או התפרקות רדיואקטיבית (קרי צזיום) אתה חייב להוסיף לנוסחא את המהירות של המודד. במהירויות נמוכות זה זניח אולם במהירות קרובות לאור זה משמעותי. לא אני אמרתי, זה איינשטיין.

    עדיין לא הבנתי, היכן הסתירה?

  297. ישראל,
    קודם כל “שעון הטמפ” לא עושה סיבובים. הוא יראה בשני המקרים ירידה מ 6000K ל3. אצל ג’יל זה ייקח שעה ואצל ג’ק מיליארד שנה.
    נראה לי ששנינו מסכימים, הלא כן?

    עכשיו אתה הקיפוד,
    1) מה הסתירה אם כן?

    2) מה לדעתך יראו השעונים, צזיום וטמפ’ של ג’יל וג’ק ברגע המפגש בינהים?

    לגבי ה- GPS. מה שניסיתי לעשות זה להראות על מימד “רגיל” את מה שקורה בזמן. שים לב של GPS יש שלוש פונקציות (יש יותר, אבל לצורך העניין שלוש חשובות):
    1) מד מיקום – נותן קואורדינטות — אנלוגי לשעון הטמפרטורות שלך
    2) מד מרחק – אומר כמה נסעת —— אנלוגי לשעון הצזיום
    3) ספידומטר, אומר כמה מהר נסעת ביחס למערכת קואורדינטות כלשהיא (בדרך כלל WGS84)

  298. ר.ח.
    את דוגמת הGPS ניתן לדעתי לנסח כך: ג’יל נוסעת הלוך ושוב באופניים, וכל פעם שהיא חולפת על פניו של ג’ק הם משווים את הספידומטרים. ג’יל דופקת מיילים, והספידומטר של ג’ק העצלן מראה 0, או מעט, או מה שלא יהיה: פחות מג’יל. זאת למרות העובדה ששניהם מסכימים שהם אותו מרחק מאורי. הבנתי נכון?

    שים לב שמערכת זו היא ניוטונית לחלוטין, בלי קשר למה”א. שים לב גם שהיא מחמיצה את הטיעון החשוב הבא: הקשר בין הטמפרטורה והזמן נתון בנוסחת פרידמן, שהיא פונקציה רציפה וחד-חד ערכית. את הטיעון שלך: “זה מד טמפרטורות שמודד פרמטר מחוץ למערכת.” נבדוק אם נצליח להגיע להסכמה שאנו מגיעים לסתירה בין ההנחות של התארכות הזמנים ביחסות ותאוריית המפץ הגדול, או שאני אודה וארוחם, או ששנינו נתייאש ונתחרבש. אך זאת נוכל לעשות רק אם נעבוד כקיפודים: שאלה מדוייקת – תשובה מדוייקת.

  299. ר.ח.
    הטיעון שלי בנוי משלבים. אם הסכמנו על שלב אחד, אפשר לעבור לשני. בתגובה זו:

    https://www.hayadan.org.il/astronomers-reach-new-frontiers-of-dark-matter-130112/#comment-331078

    שבה נאמר:

    מסקנה א’:
    עיי מדידת הטמפרטורה בלבד, ניתן לדעת את הזמן שחלף מאז המפץ הגדול בכל מקום ביקום, בכל דיוק שנרצה ושיתאפשר לנו טכנית.

    השלב הבא: התקנת שעוני טמפ’.
    ניתן לעשות זאת עיי חיבור מחשב למדחום. (אין זה משנה כרגע אם זוהי טמפרטורת הCMBR).

    מסקנה ב’:

    ניתן לצייד את כל מי שנרצה בשעוני טמפ’.

    הסכמנו על נושא שעוני הטמפ’. קשה עם כן לבוא ולטעון:

    “זה שאתה קורא לזה “שעון טמפ'” עוד לא הופך את זה לשעון שמודד זמן.”

    יש דבר קטן ופשוט שתוכל לעשות, אבל כמו קיפוד, שיחדד את הנושא:

    א. נאמר שאצל ג’יל היה המתקן לו אותו אני מכנה “שעון טמפ”. לצד שעון זה יש שעון צזיום. וידאו מצלם את שניהם. בסוף המסע בן השנה של ג’יל, שבו ירדה הטמפ’ מ6000K ל3K, אתה מריץ את הוידאו במהירות, סופר את מס הסיבובים שעשה שעון הטמפ’ ומחלק אותו במס הסיבובים שעשה שעון הצזיום.

    1. האם אתה מסכים שנקבל מספר שהוא בסדר גודל של מיליארדים? כן/לא?

    ב. אותו הסידור קיים גם בחללית שלעברה טסה ג’יל. על החללית ההיא עברו מבחינתה 13.6 מיליארד שנים. גם פה, אתה סופר את מס הסיבובים שעשה שעון הטמפ’ ומחלק אותו במס הסיבובים שעשה שעון הצזיום.

    2. האם אתה מסכים שנקבל מספר שהוא בסדר גודל של 1? כן/לא?

    אם תשובתך על 1 או 2 היא לא, הסבר למה.

  300. ישראל,
    זה שאתה קורא לזה “שעון טמפ'” עוד לא הופך את זה לשעון שמודד זמן. זה מד טמפרטורות שמודד פרמטר מחוץ למערכת.
    באותה מידה ג’יל וג’ק היו יכולים להסתכל על קצב ההתרחקות שבין אנדרומדה לשביל החלב.
    אם היית שואל את ג’ל (שנעה במהירות) מה קצב ההתרחקות? היא תגיד שתוך שעה הגלקסיות התרחקו (נניח) 100 שנות אור. אם תשאל את ג’ק הוא יגיד שהקצב הוא 100 שנות אור במיליארד שנה.

    לגבי מד הטמפ’ שיודע או לא יודע שייפגש במד אחר, לדעתי אתה מחמיץ פה נקודה עקרונית. אין שום קשר בין המדידות של ג’יל וג’ק. שניהם מודדים את הטמפ’ בזמן המדידה. השאלה היא באיזה מהירות הגעת לנקודה.

    תחשוב על הזמן כעל מימד. אולם, בניגוד למימדי המיקום אנחנו כל הזמן נעים במימד הזמן בקצב קבוע.
    תאר לך שוב ג’יל וג’ק ברכבת במהירות קבועה. ג’יל רצה הלוך ושוב ודופקת קילומטראז ברכבת וג’ק התאבן יושב. כל פעם שהם נפגשים הם מסתכלים ב – GPS (אנלוגי למד הזמן שלך) של שניהם. הפלא ופלא ה-GPS יראה שג’יל עברה מרחק עצום (אנלוגי לזמן) ביחס ל-ג’ק, אולם ה GPS גם יראה שהמיקום שלהם ברגע הפגישה זהה למרות המרחק השונה שעברו.

  301. התכוונתי לדיון מלפני חודש, אבל מסכים עם נקו שהתמצאנו.

  302. ישראל,

    שאלתי הייתה כתגובה לשאלה שכתבת בעבר: “מדוע אם נחיל את אותו ההגיון בדיוק, לא תנוע המערכת לקראת אנטרופיה גבוהה יותר גם בעבר? הרי הכל כאן הפיך לחלוטין מבחינת חוקי ניוטון.”

  303. סטודנט
    “שאלה אליך – האם הבנת וקיבלת את ההסבר שניתן במאמרים למונוטוניות האנטרופיה ופתרון “בעית האסימטריה” ששאלת עליה?”

    כמובן שקיבלתי. לא הבנתי גם את שאלתך “לא ברור לי למה אתה ונקודה מתעקשים להמציא הסברים משלכם למה שכבר ניתן הסבר הגיוני למדי (כפי שנטען במאמר) ע”י גדולים מכם (בולצמן, פיינמן, פנרוז…)” .כל העניין היה שאלה שלי אם ניתן בפשטות “לכמת” רגע מסויים. מעולם לא ניסיתי לטעון שאין הסבר, או שהוא איננו ראוי.
    אבל נראה לי שקצת מיצינו את הנושא, ועכשיו שנינו באותו הראש.

    ר.ח. – תגובה אליך בבידוק בטחוני.

  304. ר.ח.
    אתייחס תחילה לבעיית הטרחנות שהעלית.
    מכיוון שאני היחידי בין שנינו שמכיר את עניין ה”התקפה על הממסד” כדבריך, אני יכול להבטיחך נאמנה כי אין דבר הרחוק יותר מן האמת. התקשורת נעשתה באמצעות אימיילים, כשאותם אנשים איתם יצרתי קשר שיתפו פעולה בהתלהבות. באי ההסכמות שהיו ביננו לגבי הנושאים המהותיים, עלה בידי לשכנע אותם בנכונות טענותי. כשהגענו יותר לעומק, ברוב המקרים הובעה התנצלות על כך שאין ביכולתם לפתור את הבעייה שהעלתי.

    הטענה הראשונה שהועלתה דרך קבע, הייתה שכל מערכת רואה את השניה כנעה לאט יותר, כפי שטוענת היחסות.

    טענה זו ניתן להדוף בקלות עיי שימוש במצלמות חדות רזולוציה ובאמצעות וידאו, מכיוון שקשה מאוד להתווכח עם תמונות חד משמעיות, המראות את הזמן המסויים שמראה כל שעון בזמן המפגש.

    ניגש עכשיו לטיעון שלך, וננסה לממש את אותו עקרון של תמונות, בהן ניתן להתבונן זמן רב לאחר מעשה.

    “שעוני הצזיום יראו הבדל, שעה במקרה אחד ומיליארד שנה (לפי ההקצנה שלך) במקרה השני. מד הטמפ’ של שני התאומים יראו טמפרטורה זהה (על זה אנו מסכימים). אם שני החבר’ה האלו היו מציירים גרף של מד הטמפ’ לאורך זמן היינו רואים שקצב ההתקררות במקרה של ההוא שנח הוא איטי (נגיד מ – 6000K ל- 3K במילארד שנה) בעוד שהגרף של השני היה מראה קצב מהיר 6000K ==> 3K בתוך שעה. די פשוט, לא? אלא אם כן אני מחמיץ משהו מאד עקרוני ואשמח אם תאיר את עיני”

    הנה ההארה:

    במקרה שדנו בו, מדובר במערכות שאינן מואצות. נערה נחמדה בשם ג’יל נמצאת בחללית שמבחינתה נמצאת במנוחה מוחלטת. לצידה שני שעונים: אחד שעון צזיום והשני שעון טמפ’. וידאו מצלם את שני השעונים ביחד.

    1. אם לטענתך “הגרף של השני היה מראה קצב מהיר 6000K ==> 3K בתוך שעה.” הרי שהוידאו של שני השעונים היה מראה את שעון הטמפ’ מסתובב מהר יותר בהרבה משעון הצזיום. למעשה, על כל סיבוב אחד של שעון הצזיום, היה שעון הטמפ’ מסתובב מיליארדים רבים של סיבובים.

    וכל כך למה? מפני שאי שם בהמשך הדרך עתידה ג’יל לפגוש איזה שעון שנע יחסית אליה? כיצד יודעים השעונים בחללית של ג’יל באיזה מהירות נע אותו שעון מרוחק, שבגללו עליהם לשנות את קצב הסיבוב שלהם?
    ההגיון מחייב שכל עוד המערכת אינה מואצת שעון הצזיום ושעון הטמפ’ נעים באותו הקצב וכל צילום יראה את אותו הזמן בשניהם, כי המערכת למעשה במנוחה.

    נמשיך. בשעת המפגש חולפת ג’יל במהירות עצומה ליד שעון צזיום אחר, אשר צמוד אליו שעון טמפ’ אחר. זה אינו מונע מהמצלמה חדת הרזולוציה שבחלליתה של ג’יל לצלם את 4 השעונים ביחד. נניח שגם בנקודה בה נמצאים השעונים המרוחקים נמצאת מצלמה, הנחה יחסית לשעונים במקום, והיא מצלמת את 4 השעונים גם כן.

    2. האם התמונות שצולמו עיי 2 המצלמות ברגע המפגש תהיינה זהות? או שדעתך היא שהצילומים יראו שכל מערכת מפגרת יחסית לשניה?

    3. מה יראו הצילומים החד משמעיים?

    כי זו הרי הנקודה המרכזית: כל מערכת מבחינתה נמצאת במנוחה, ולכן קצב ההתקדמות של שעוני הצזיום והטמפ בכל מערכת זהה. מכיוון שהסכמנו שבעת המפגש יראו שעוני הטמפ’ את אותו הזמן, וכל שעון טמפ’ מראה את אותו הזמן של שעון הצזיום הצמוד אליו (מערכת שאינה מואצת), ומכיוון שאם א=ב, ג=ד, וא=ג אז א=ב=ג=ד הרי שכל השעונים חייבים להראות את אותו הזמן ברגע המפגש.

    וזאת בניגוד לטיעון המקורי שבו קיים הפרש של 2 מיליארד שנים בין 2 שעוני הצזיום.

    נסה לעשות זאת כמו קיפוד, לאט ובזהירות, ורשום את כל הזמנים וקצבי הוידאו, תראה אני מאמין שזו מסקנה בלתי נמנעת.

    מצד שני אם העניין קצת נמאס עליך, אקבל זאת בהבנה. הדבר האחרון שאני מבקש הוא להיות לטורח.

  305. ישראל,

    “היש איזו פסקה במאמר האחרון או באיזה מאמר אחר, שבה ניתן לתת לכל רגע בהסטוריה ערך מספרי מסויים יחודי, מדוייק ככל שרצה, וגם לקשור זאת לגורמים פיזיקליים?”

    – אנטרופיה? חפש לקראת סוף המאמר הראשון שקישרתי, מוזכרים שם ערכים מקורבים (ע”י פנרוז) עבור האנטרופיה במצבים שונים של היקום. (אני לא חושב שזה רציני לבקש “מדויק ככל שנרצה” עבור נתון כזה)

    בקשר לשאלה השניה – אני לא מכיר את ניסוי ווילר. אני חושב שפרופ’ אהרונוב חקר/חוקר נושאים כמו השפעה על העבר/הווה מהעתיד (או וריאציות כאלה), אולי תרצה לחפש מאמרים שלו בנושא. עד כמה שזכור לי, זה מתקשר למה שנקרא מדידות חלשות בתורת הקוונטים.

    שאלה אליך – האם הבנת וקיבלת את ההסבר שניתן במאמרים למונוטוניות האנטרופיה ופתרון “בעית האסימטריה” ששאלת עליה?

  306. הפיתרון של תנאי התחלה לא מספיק.
    לדעתי הפיתרון הוא שיש ליקום הרבה מצבים אפשריים, והתודעה שלנו מטיילת האותם מצבים שמאפשרים את קיומה, כלומר, באותם מצבים שבו אנו רואים שהאנטרופיה עולה. קיימים מצבים (יקומים?) אחרים שבהם זה לא קורה, והתודעה שלנו לא נמצאת שם.

    במובן זה ניתן לומר שכל המצבים האפשריים קורים, ואנו נמצאים רק באותם מצבים המאפשרים את המצאותנו. לצופה מבחוץ זה יראה שתודעה גורמת לפונקציית גל לקרוס בצורה מסוימת.

  307. ישראל,
    מאחר והתגובה לא שוחררה עד עכשיו העתקתי אותה ערוכה ללא הלינק שהיה מצורף ונקווה שהפעם תעבור.

    ישראל,

    הדיון הזה חוזר על עצמו בצורה מעגלית. אני לא רואה שום סתירה ואני לא מצליח להבין איזה סתירה אתה רואה.
    לשאלותיך התשובה פשוטה וקוהרנטית עם מושג האצת הזמן לפי המהירות
    שעוני הצזיום יראו הבדל, שעה במקרה אחד ומיליארד שנה (לפי ההקצנה שלך) במקרה השני. מד הטמפ’ של שני התאומים יראו טמפרטורה זהה (על זה אנו מסכימים). אם שני החבר’ה האלו היו מציירים גרף של מד הטמפ’ לאורך זמן היינו רואים שקצב ההתקררות במקרה של ההוא שנח הוא איטי (נגיד מ – 6000K ל- 3K במילארד שנה) בעוד שהגרף של השני היה מראה קצב מהיר 6000K ==> 3K בתוך שעה. די פשוט, לא? אלא אם כן אני מחמיץ משהו מאד עקרוני ואשמח אם תאיר את עיני.

    מעבר לזה אני ממש לא אוהב את ההתייחסות שלך לזה שפנית לפרופסורים והמדענים מוסמכים והם לא מבינים. היה פה למעלה דיון על טרחנות מתימטית ומדעית והתקפה שהופמתה כנגד יובל. אחד המאפיינים העיקריים של טרחן מדעי הוא שטרחן בדרך כלל יתקוף את “הממסד המדעי” ויזלזל במדענים הנוכחים שלא מצליחים לראות מה שהטרחן טוען בלהט. אני מציע שכדאי שלא תלך לכיוון הזה פן תהפוך לכזה. מה אתה צריך להתקיף את ה”ממסד”? תיצמד למדע שלך ותאמין לי שאם תוכיח התאוריה שלך תתקבל ללא קונספירציות או אטימות ממסדית.

  308. אולי פספסתי משהו.
    היש איזו פסקה במאמר האחרון או באיזה מאמר אחר, שבה ניתן לתת לכל רגע בהסטוריה ערך מספרי מסויים יחודי, מדוייק ככל שרצה, וגם לקשור זאת לגורמים פיזיקליים?
    האם אתה מכיר דבר דומה?
    אשמח לקבל הסברים או הפניות.
    שאלה נוספת: האם אתה מכיר את ניסוי וילר שבו ניתן לכאורה להשפיע על העבר מן ההווה?

  309. נקודה,

    “מה העניין סטודנט, הדברים והבעיות שהזכרתי אלו בדיוק הבעיות שהמאמרים הללו מתיחסים אליהם.
    לכן אני מניח שלא קראת אותם.”
    – “לכן”? המאמר מציג את ה”בעיה” ומציע פתרונות, כפי שטענתי שקיימים ושללת (וזו הסיבה שקישרתי אליו).

    ישראל,

    אוקיי. שים לב שזה נמצא בפסקה העוסקת בפן הפילוסופי של מושג הזמן (אליו הוא לא נכנס במאמר השני שקישרתי – בין היתר הסיבה שטענתי שהמאמר השני יותר מתומצת).

    “מכאן דיונינו הקודם. לפי הרעיון שלי, אפשר להגדיר בצורה מוחלטת מה פירוש הביטוי WITH TIME.
    הזמן מתקשר ישירות לכמות החלקיקים, או מה שלא יהיה שם, ביחידת נפח. כל רגע בזמן מוגדר כך באופן חד משמעי, והעתיד מופרד מן ההווה והעבר עיי מספר מונה פשוט.”
    – הבנתי למה אתה (ונקודה) מתכוונים ב”בעיה” עליה אתם מדברים החל מאחת התגובות האחרונות שלך, לכן קישרתי שוב למאמר הנ”ל.
    לא ברור לי למה אתה ונקודה מתעקשים להמציא הסברים משלכם למה שכבר ניתן הסבר הגיוני למדי (כפי שנטען במאמר) ע”י גדולים מכם (בולצמן, פיינמן, פנרוז…).

  310. סטודנט.
    מתוך הפנייתך בתגובה:

    https://www.hayadan.org.il/astronomers-reach-new-frontiers-of-dark-matter-130112/#comment-325851

    In a book entitled Time’s Arrow and Archimedes’ Point”

    מתוך הפנייתך האחרונה:

    Once we accept Boltzmann’s explanation of why macroscopic
    systems evolve in a manner that makes SB increase
    with time, there remains the nagging problem (of which
    Boltzmann was well aware) of what we mean by “with
    time

    מכאן דיונינו הקודם. לפי הרעיון שלי, אפשר להגדיר בצורה מוחלטת מה פירוש הביטוי WITH TIME.
    הזמן מתקשר ישירות לכמות החלקיקים, או מה שלא יהיה שם, ביחידת נפח. כל רגע בזמן מוגדר כך באופן חד משמעי, והעתיד מופרד מן ההווה והעבר עיי מספר מונה פשוט.

    נקודה.
    תוכל להרחיב על נושא חץ הזמן החד כיווני כתוצאה של קריסת פונקציית הגל? מדוע היא מונעת את עליית האנטרופיה גם לכיוון עבר?

    ר.ח. ???!?

  311. מה העניין סטודנט, הדברים והבעיות שהזכרתי אלו בדיוק הבעיות שהמאמרים הללו מתיחסים אליהם.
    לכן אני מניח שלא קראת אותם.

  312. נקודה,

    אני יודע. כתב אותו פרופ’ בעל שם עולמי בתחום הפיזיקה הסטטיסטית.
    עושה רושם שאתה מנסה להפתיע אותי במה שכתוב במאמר, שזה די משעשע. בכל מקרה, ממליץ לך לקרוא את כולו מתחילתו ועד סופו – אולי תשנה את דעתך. כמובן, אתה יכול להישאר/להמציא איזה הסברים שאתה רוצה, לפתח תאוריות מפה ועד קאלטק, אך כנראה שבסופו של דבר אתה “תפסיד” מזה.

    ישראל,

    אוקיי, הבנתי את שאלתך. אני מציע לך לקרוא שוב (רוצה להאמין שקראת כבר פעם אחת) את המאמר של פרופ’ לייבוביץ’, אליו כבר קישרתי אותך בעבר ופה את נקודה: http://www.scholarpedia.org/article/Time%27s_arrow_and_Boltzmann%27s_entropy – הוא מדבר בין היתר על השאלה שלך.

    בקשר לספר הנ”ל – אני לא מכיר אותו, ולא ראיתי אותו בקישורים שנתתי לך.

  313. יופי נקודה. אנחנו בראש אחד בעניין דו כיווניות האנטרופיה ביחס לחץ הזמן.
    האם הקריסה יכולה גם להסביר את עניין ההשפעה כביכול של ההווה או העתיד על העבר?

  314. סטודנצ’יק, המאמר ששלחת הוא מאמר טוב. ואני מציע לך לקרוא אותו גם.

    Boltzmann: “That in nature the transition from a probable to an improbable state does not take place as often as the converse, can be explained by assuming a very improbable [small SB] initial state of the entire universe surrounding us. This is a reasonable assumption to make, since it enables us to explain the facts of experience, and one should not expect to be able to deduce it from anything more fundamental”

    Once we accept the statistical explanation of why macroscopic systems evolve in a manner that makes SB increase with time, there remains the nagging problem (of which Boltzmann was well aware) of what we mean by “with time”: since the microscopic dynamical laws are symmetric, the two directions of the time variable are a priori equivalent and thus must remain so a posteriori.

    ואני אומר שזה לא עניין של תנאי התחלה, אלא של קריסה של פונקציות גל.

  315. נקודה,

    1. אם חוק פיזיקלי מוסבר באמצעות תורה מתמטית הוא לא יכול להיחשב חוק טבע יסודי? עובדה שחולקים עליך אנשים שסביר להניח דעתם בנושא עולה על שלך. גם ברוב ההקשרים שאני למדתי, חוקי התרמודימיקה הוזכרו כ-“fundamental laws of physics”. גם בויקיפדיה האנגלית הם מוזכרים ככאלה.

    2. אמרת זאת מספר פעמים והגבתי, אך לא אחזור על דבריי.

    3. אתה צודק שאכן ההתפלגויות שציינתי מתקבלות מהנחות מסוימות, אך הנחות אלו הן מה שמונח, לא ההתפלגויות עצמן.

    למה הכוונה ב”…אין שום סיבה מצד חוקי הפיסיקה שאכן מערכות בשיווי משקל כאלו יהיו קיימות במציאות.” אני לא מבין, אבל בפיזיקה ומכניקה סטטיסטית “מראש” מוותרים על תיאור מדויק. בכלל, יש מעט מאוד מערכות פיזיקליות שיודעים לתאר באופן מדויק. ועובדה, פיזיקה סטטיסטית עובדת מצוין. המחשב שאתה כותב עליו מבוסס בין היתר עליה.

    4.
    “החוק הפיזיקלי הוא משוואת שרדינגר לצורך העניין, והחוק הסטטיסטי הוא החוק התרמודינמי. נראה לי שזה ברור.”
    – ברור? מעולם לא שמעתי על חלוקה כזו.

    “החוק הסטטיסטי לא מפריד בין אוסף מולקולות שמרכיבות גז, לאוסף מולקולות שמרכיבות תא זיגוטה. למרות שהתוצאות הפיזיקליות של שתי אלו שונה לחלוטין.”
    – מצטער, לא הבנתי על מה אתה מדבר. אין הבדל בין מולקולות גז לתא ביולוגי? אילו תוצאות פיזיקליות שונות?

    “לוקלית החוק הסטטיסטי לא תקף, חוקי הפיזיקה תקפים, אך כשמסתכלים בגדול ודאי שהוא תקף בעולמינו, אחרת לא היה לנו חץ זמן פיסיקלי. והשאלה היא איך אוסף של חוקי פיזיקה הפיכים, הופכים למשהו לא הפיך. ואני חושב שזה קשור למנגנון של קריסת גל.”
    – היות ואני לא דוגל בחזרה על עבודה של אחרים (וגדולים ממני), אפנה אותך למאמר:
    http://www.scholarpedia.org/article/Time%27s_arrow_and_Boltzmann%27s_entropy, למקרה שתרצה. אני סבור שזה יתרום לך.

  316. ישראל
    החוק השני של ניוטון מגדיר כוח כשינוי מהירות של מסה, ולאומת זאת אינרציה (התמדה) מתייחסת למהירות
    קבוע או למנוחה קרא את הקישור מהפעם הקודמת.

  317. נקודה.
    האם ידוע לך מהי מסת החומר הבריוני ביקום? מהו רדיוס היקום? אם כן, כתוב בבקשה פה, או קשר ללינק.

  318. א.ש.
    איך היית מגדיר את F=MA? לא חוק אינרציה?

    סטודנט.
    בוא נוודא שאנחנו מדברים על אותו הדבר.
    אם יש לך מערכת במצב אנטרופי מסויים, הרי בשל עקרונות סטטיסטיים היא תנטה לנוע למצב אנטרופי גבוה יותר בעתיד. על כך אין שום ויכוח אני מאמין, ועל זה מדבר גם הלינק שנתת.

    השאלה היא זו:
    מדוע אם נחיל את אותו ההגיון בדיוק, לא תנוע המערכת לקראת אנטרופיה גבוהה יותר גם בעבר? הרי הכל כאן הפיך לחלוטין מבחינת חוקי ניוטון.

    ואין כאן שום קשר להליכים הפיכים תרמודינמיים. זהו נושא שונה.

    הנושא נדון בספר באחד הספרים שבלינק ששלחת לי “חץ הזמן ונקודת ארכימדס” ע’ 56. מתואר שם גם ערעורו של פואנקרה, שטען שלא רק שירידת האנטרופיה אפשרית, אלא היא בלתי נמנעת.

    ר.ח.
    צריך לעשות סדרות שבי לכל התגובות לפני השלח תגובה, להכין אותן לנורא מכל.
    כתבה זנוחה עלק…
    כל האקשן בקוסמו!

  319. ישראל, לא ברור לי בכלל מה אתה רוצה. אמרתי לך מה הנוסחא לרדיוס שוורצילד של חור שחור.
    את הM של היקום, אתה לוקח מאיפה? מנתונים של WMAP? הרי אלו אותם נתונים שמראים שהחומר הבריוני מהווה רק 5%.
    או שאולי לשחק ביחידות אתה רוצה? אתה יכול להתחיל מהמספר 1 וע”י שימוש ביחידות להגיע לכל משוואה פיזיקלית שרק תרצה. הרי הפיזיקאים דואגים לכך שהמשוואות יהיו הגיוניות (מאוזנות ביחידות משני האגפים).

  320. 1) חוקי התרמודינמיקה אינם יסודיים, ועובדה שלמדת מכניקה סטטיסטית שמסבירה כיצד מקבלים את חוקי התרמודינמיקה.

    2) אין תשובה כיוון שהחוקים היסודיים המיקרוסקופיים הם סימטריים בזמן, וחוקי התרמודינמיקה המאקרוסקופיים לא. ועדיין שניהם מתקיימים בעולמינו. ולא קיים הסבר של מעבר מעולם המיקרו לעולם המאקרו וסביב זה נסוב כל הויכוח. אולי כשיתחילו עם המחשבים הקוונטיים נדע יותר על העניין הזה.

    3)ההתפלגויות באותן המערכות שאתה מתאר הן מבוססות על כמה וכמה הנחות. וזה בדיוק העניין, שאין שום סיבה מצד חוקי הפיסיקה שאכן מערכות בשיווי משקל כאלו יהיו קיימות במציאות.
    ולמה משתמשים במערכות כאלו? כי לא ניתן לפתור את משוואת שרדינגר(או מקבילותיה היחסותיות) למספר רב של חלקיקים. לו היה ניתן, לא היו צריכים להשתמש במכניקה סטטיסטית כלל.

    4)החוק הפיזיקלי הוא משוואת שרדינגר לצורך העניין, והחוק הסטטיסטי הוא החוק התרמודינמי. נראה לי שזה ברור. החוק הסטטיסטי לא מפריד בין אוסף מולקולות שמרכיבות גז, לאוסף מולקולות שמרכיבות תא זיגוטה. למרות שהתוצאות הפיזיקליות של שתי אלו שונה לחלוטין.
    לוקלית החוק הסטטיסטי לא תקף, חוקי הפיזיקה תקפים, אך כשמסתכלים בגדול ודאי שהוא תקף בעולמינו, אחרת לא היה לנו חץ זמן פיסיקלי. והשאלה היא איך אוסף של חוקי פיזיקה הפיכים, הופכים למשהו לא הפיך. ואני חושב שזה קשור למנגנון של קריסת גל.

  321. התגובה למעלה היא לנקודה.

    ישראל,

    “וגם בפעם שעברה עניתי לך שדווקא נראה לי שאני מבדיל. ע”ע מחזור קרנו. אין פה שום קשר להפיכות התהליך.”
    – אתה הכנסת את ההפיכות, לא אני. אם אתה רוצה להשמיט את המושג הזה ולחזור למה שכתבת:
    “מדוע, אם הכל הפיך וסימטרי, וחוקי ההסתברות תקפים לשני הכיוונים, לא תעלה האנטרופיה גם לכיוון העבר?”
    – למה שהאנטרופיה תעלה אם תהפוך את כיוון החץ (לעבר)? הרי שמהגדרת האנטרופיה תקבל שבהקטנת המשקל הסטטיסטי היא תרד.

    “מה שאני מדבר עליו, ואני מאמין שגם נקודה, הוא על כך שתאורטית, מבחינת חץ הזמן, האנטרופיה צריכה לגדול גם לכיוון עתיד וגם לכיוון עבר, בשעה שבפועל היא גדלה אך ורק לכיוון עתיד.”
    – אני אכן לא מבין מה אתה (או אתם) לא מבינים. אני חושב שבקישור שנתתי לך הביאו הסבר די ממצה למה שאתה קורא כאן בעיה. אולי אם תיתן דוגמה ספציפית תוכל להבהיר על מה אתה מדבר.

    “אם נראה לך שאין כאן בעיה, אתה במחלוקת עם פואנקרה ובולצמן.”
    – do tell.

  322. “בכלל בלינק שהבאת התשובה היא לא רצינית ולא מדוייקת כלל.”
    – ולזרוק טענה כזו בלי לציין אפילו מה – זה מאוד רציני ומדויק. בכלל, אותו דבר אפשר להגיד לגבי דבריך. היות והכותבים של הקישורים שנתתי הם סטודנטים לתארים גבוהים ודוקטורים ממכון ויצמן, האמינות, עד שתוכיח אחרת, נשארת עימם.

    “חוקי התרמודינמיקה אינם יסודיים כלל.”
    – הם כן, וזה לא נאמר רק בקישורים שנתתי, אלא ברוב ההקשרים שלמדתי עד היום.

    “והתשובה לדעתי קשורה למעבר מעולם המיקרו הקוונטי לעולם המאקרו הקלאסי וזה קשור לקריסות גל באיזשהו אופן נסתר.”
    – כלומר, אין לך תשובה ואתה מסרב לקבל את זו שקיימת?

    “מכניקה הסטטיסטית מניחים פונקציות התפלגות של חלקיקים. אף אחד לא פתר את משוואות שרדינגר כדי לקבל את הפונקציות הללו.”
    – לא הבנתי מה הקשר בין פונקציות ההתפלגות במכניקה סטטיסטית (שכלל לא מניחים אותן, הן מתקבלות מניתוח של המערכת – בין אם היא קלאסית שמובילה למקסוול-בולצמן ובין אם היא קוונטית שמובילה לפרמי-דיראק, בוז-איינשטיין או אלנטרנטיבות כמו parastatistics), לבין הקשר של משוואת שרדינגר וקריסת פונקצית הגל, לדיון הזה.

    “ואם מישהו היה מנסה לתור עדיין הוא לא היה מגיע לפונקציות ההתפלגות הללו, כי מנגנון הקריסה הוא לא חלק ממשוואת שרדינגר. זה מנגנון אחר שהוא אינו ברור.”
    – שוב, אני לא מבין את הטענה או למה אתה מתכוון. הרי משוואת שרדינגר לא נחוצה בכלל (אך כן להתחשב באופי הקוונטי של החלקיקים) כדי להגיע להתפלגות פרמי-דיראק (למשל), אז איפה החיבור?

    “הדיון בשאלה הוא כאילו פילוסופי. זאת שאלה פיסיקלית לכל דבר. זה שאין לה מענה כרגע לא הופך אותה לפילוסופי.”
    – אמרתי שהדיון בשאלה מהם “חוקי הפיזיקה” או מהי הסתברות ולמה היא עובדת, הוא פילוסופי.

    “ושוב לגבי האבולוציה, אתה עדיין מתבלבל. האבולוציה היא דוגמא לכך שמה ששולט בטבע הוא החוק הפיסיקלי, ולא החוק הסטטיסטי, זה לא אומר שחוקי התרמודינמיקה הופרו, זה רק אומר שהם כלל לא יכולים לחזות משהו כמו חיים.”
    1. מה/מי זה החוק הפיזיקלי והחוק הסטטיסטי?
    2. למה אתה חושב שהם לא יכולים לנבות? כי אין לך דרך לתאר מערכת מורכבת כמו זו שהובילה להתפתחות החיים?
    3. אני חושב שאתה סותר את עצמך. מצד אחד אתה אומר שחוקי התרמודינמיקה תקפים (לא הופרו) ומצד שני אתה טוען שהם לא יכולים לחזות איזה תרחיש יקרה (כמו יצירת חיים) – שזו חלק ממהות החוק השני.

  323. ישראל,

    התגובה שלי ממתינה לאישור במרתפי השטאזי, מסכנה קרעו אותה בעינויים ולמרות זאת היא לא הודתה בטרוליות או כל פגיעה אחרת. אני מניח שהיא תשוחרר בקרוב.

  324. נקודה.

    G = קבוע הגרביטציה.
    C = מהירות האור.
    M = מסת היקום המשוערת. (אפשר למצוא בוויקיפדיה).
    R = רדיוס היקום המשוער. (כנ”ל).

    אז: GM=RC^2 בקירוב.

    אם נשתמש ב dimensional analysis, אחרי כל הצמצומים משני צידי המשואה, נשאר רק עם:

    F = MA

    החוק השני של ניוטון, חוק האינרציה.

    השאלה:

    איך זה מסתדר איפוא עם הרעיון שיש למעשה פי 5 לפחות מסה מן המקובל? לא היינו אמורים להוסיף פקטור נוסף בנוסחה כה אלגנטית? האין חוסר התאמה זה לבדו אמור לחסל את כל רעיון המסה האפלה? הרי אין כמעט ספק בערכים של G, R וC.

    או אולי M בנוסחה כבר כולל בתוכו את המסה האפלה?

    סטודנט.

    “- למה ואיך הגעת למסקנה שהתהליך המתואר בקישור שנתתי הוא הפיך? גם בפעם הקודמת שדנתי איתך קיבלתי רושם שאתה לא מבדיל בין תהליך הפיך ולתהליך לא הפיך.”

    וגם בפעם שעברה עניתי לך שדווקא נראה לי שאני מבדיל. ע”ע מחזור קרנו. אין פה שום קשר להפיכות התהליך. מה שאני מדבר עליו, ואני מאמין שגם נקודה, הוא על כך שתאורטית, מבחינת חץ הזמן, האנטרופיה צריכה לגדול גם לכיוון עתיד וגם לכיוון עבר, בשעה שבפועל היא גדלה אך ורק לכיוון עתיד.

    אם נראה לך שאין כאן בעיה, אתה במחלוקת עם פואנקרה ובולצמן. יש כמה הסברים לתופעה, והקוואנטים הוא אחד מהם.

    אני ניסיתי להראות שייתכן שתאוריית המפץ הגדול יכולה לשפוך אור על הבעיה, אבל אנחנו קצת מתבדרים, אז נרד מזה.

    ר.ח.??? שעונים, תאומים, עמייאת.

  325. כל צורות האנרגיה שואפות תמיד לרדת לאנרגית חום. החום- קולו של בולצמנסקי נשבר- החמימות הנעימה שעליה אנו מתרפקים – הנה צורת האנרגיה הנמוכה ביותר – קולו הפך ליבבה – הבזויה ביותר-
    הקהל געה בבכי.
    -וואו, וואו- חיקה החוק. -חום, אנרגיה, עבודה – מה לכל זאת ולי? חם לך? תפעיל מזגן!
    – ומה עושה המזגן? שאג הפרופסור -מקרר מערכת סגורה אחת על חשבונה של אחרת? -עליכם להבין, פנה אל הקהל, שמזגן, בניגוד לתנור למשל, אינו יכול להיות כלול במערכת סגורה. זו הסיבה לכך שהוא בדרך כלל קבוע על הקיר – חציו במערכת סגורה, הבית, וחציו השני באוויר החופשי, ותמיד יש צורך לספק לו אנרגיה ממקור חיצוני. אתם מבינים את האבסורד? זקף הפרופסור אצבע בתוכחה- עלינו להשקיע אנרגיה כדי להוציא אנרגיה מן החדר. באמת, כמה נמוך אפשר לרדת?
    -מאותה הסיבה בדיוק איננו יכולים להפיק עבודה מועילה מחום בצורה ישירה. מטוס לא יוכל להתקדם על ידי כך שיקרר את האוויר דרכו הוא עובר. כל המערכות המפיקות עבודה מחום – מנוע קיטור למשל – חייבות להיות חמות יותר מן הסביבה. נצילות האנרגיה שלהן, הנמוכה מאד, מוגבלת להפרש הטמפרטורות בין המערכת לסביבה, מחולקת בטמפרטורה המוחלטת..
    -פרופסור בולצמנסקי- התערב התובע בעדינות, -אין בכוונתי להגן על הנאשם, אולם האם לא סטית מעט מן הנושא? אחרי הכל אין הנאשם עומד לדין על פעילותו התרמודינמית. כלום תוכל לתאר לנו את הנזק הנגרם מפעילותו לחיים?
    -נכון מאד- קרא החוק – אני, חברי החדש, התובע, וידידי בקהל אותם הרדמת כה יפה רוצים סופסוף להבין: מה עשיתי? למה אני פה?
    -בלגן. ענה הפרופסור.
    -מה? שאלו החוק, התובע ועוד כמה קולות בקהל.
    -עשית בלגן. אנרכיה. אי סדר. בגלל הנטייה של האנרגיה להתפזר, כוון הדברים בטבע הנו תמיד להגדלת האנטרופיה, או אי הסדר. הבית תמיד מתבלגן מעצמו – אולם כדי שיסודר עלינו להשקיע עבודה. ספל חרסינה יתנפץ אם ייפול, אולם השברים לעולם לא יתאחו מעצמם. מכונית תחליד ותתישן, ולא תיהפך לחדשה יותר ללא עבודה חיצונית.
    -לא בדיוק.. מלמל החוק שנית.
    -כן בדיוק. כבר הודית בכוון זרימת האנרגיה – לחום, האנרגיה הנמוכה ביותר -הודה עכשיו בכוון אליו אתה מוביל את כל הדברים בעולם: התפרקות ואי סדר. אילולא החיים – כל הקיים ביקום היה הולך ושואף בהתמדה לצורה הנמוכה ביותר האפשרית של סדר.
    עצם מושג העבודה – המוגדר בפיזיקה כמכפלה של כוח בדרך – הנו למעשה היפוך האנטרופיה. עבודה רציפה קיימת אך ורק במערכות ביולוגיות או במערכות שנוצרו על ידי מערכות ביולוגיות. בטבע הדומם אין בנמצא מזגנים, או כל מכונה אחרת, ומערכת יכולה להקטין את האנטרופיה המקומית שלה רק בצורה אחת: התפשטות.
    במערכות חיות לעומת זאת ניתן להקטין את האנטרופיה המקומית עיי קליטת אנרגיה חיצונית -כפי שעושים הצמחים בתהליך הפוטוסינתזה. גם מכונות מעשה ידי אדם עושות זאת. למעשה החיים מהווים סתירה מוחלטת לחוק השני ולמטרותיו הנפסדות, ניצחון הטוב והנאצל על הרע והמשחית..
    -אם כל כך טוב למה כל כך רע? שאל החוק ברוסית.
    -חדל, ציניות לא תעזור לך. אתה מנסה להציג את עצמך כתמים ושובב, אבל אנו נחשוף את פרצופך האמיתי: חבלן! משחית! אתה הסיבה לכל צרותינו ותחלואינו! אתה הסיבה לתקלות בחיינו, להזדקנותנו, רוצח! אתה גורם לכולנו ל.. למ.. הוא התקשה לבטא את המילים – ללכת בדרך כל בשר-
    הקהל התייפח מרות.
    -ממש שובר לבבות – רטן החוק, -וגם אתה תלך בדרך כל בשר-
    -אני סיימתי, ניגב הפרופסור את עיניו הדומעות בממחטה. -אפשר להמשיך לתאר את מעלליו הנפשעים של הנאשם. אבל העיקרון יישאר זהה: עד אשר לא יבוטל החוק השני, או ישונה אופיו ההרסני – ימשיכו החיים לסבול לנצח מהפרעותיו והתנכלויותיו.
    -או אנחנו – או הוא.

  326. בכלל בלינק שהבאת התשובה היא לא רצינית ולא מדוייקת כלל.
    חוקי התרמודינמיקה אינם יסודיים כלל. אלו חוקים סטטיסטיים החלים בעולם המאקרו, כאשר יש מספר רב של חלקיקים. וכל השאלה היא כיצד זה קורה. והתשובה לדעתי קשורה למעבר מעולם המיקרו הקוונטי לעולם המאקרו הקלאסי וזה קשור לקריסות גל באיזשהו אופן נסתר.
    בודאי שבקורס לא לומדים על קריסה של פונקציות גל. במכניקה הסטטיסטית מניחים פונקציות התפלגות של חלקיקים. אף אחד לא פתר את משוואות שרדינגר כדי לקבל את הפונקציות הללו. ואם מישהו היה מנסה לתור עדיין הוא לא היה מגיע לפונקציות ההתפלגות הללו, כי מנגנון הקריסה הוא לא חלק ממשוואת שרדינגר. זה מנגנון אחר שהוא אינו ברור.
    הדיון בשאלה הוא כאילו פילוסופי. זאת שאלה פיסיקלית לכל דבר. זה שאין לה מענה כרגע לא הופך אותה לפילוסופי.

    ושוב לגבי האבולוציה, אתה עדיין מתבלבל. האבולוציה היא דוגמא לכך שמה ששולט בטבע הוא החוק הפיסיקלי, ולא החוק הסטטיסטי, זה לא אומר שחוקי התרמודינמיקה הופרו, זה רק אומר שהם כלל לא יכולים לחזות משהו כמו חיים.

  327. ישראל,
    סליחה במבט שני ראיתי שאת ההערה על האבולוציה נקודה אמר לכן התשובה בעניין הזה מופנית אליו.

  328. נקודה,

    אתה טועה. יש הסבר לחץ הזמן, והוא ניתן באופן די ממצה בשתי הכתבות אליהן קישרתי. אני לא רואה את סיבת ההתעקשות שלך לכך שאין הסבר. אני בכל אופן, כמובן לא יכול לקרוא במקומך.

    מספר הערות:
    1. חוקי התרמודינמיקה נחשבים לחוקי טבע יסודיים.
    2. ההסבר לחוק השני ניתן במסגרת פיזיקה/מכניקה/תרמודינמיקה סטטיסטית. מכניקת הקוונטים כן נכנסת בפיזיקה סטטיסטית, אבל אני לא שמעתי על קריסת פונקצית הגל בקורס, ולמען האמת אני לא יודע כל כך על מה אתה מדבר בהקשר לדיון.
    3. אתה מושך את הדיון לכיוון פילוסופי ואני לא מעוניין להיכנס לשם. מהי הסתברות ומהם “חוקי הפיזיקה” אלה שאלות פילוסופיות. בפועל אין פונקצית גל, אין חוקים ואין פיזיקה – יש רק מה שאנחנו מסיקים מחקירת העולם סביבנו.

    “ואת האבולוציה הבאתי כדוגמא לכך שאיננו מקבלים תמיד את המצב האנרגטי הנמוך ביותר.”
    – ומזגנים לא אמורים לעבוד? אין באמירה הזו שום דבר. כתבת:
    “אין שום גורם סטטיסטי שיגרום לכך שדוקא הרכיב בעל מספר המצבים הרב ביותר יתרחש. הדוגמא הכי טובה לכך היא האבולוציה.”
    וזו פשוט לא דוגמה טובה. יצירת החיים, גם אם אינך מבין או רואה זאת, הגדילת את אנטרופית היקום.

  329. ישראל,

    הדיון הזה חוזר על עצמו בצורה מעגלית. אני לא רואה שום סתירה ואני לא מצליח להבין איזה סתירה אתה רואה.
    לשאלותיך התשובה פשוטה וקוהרנטית עם מושג האצת הזמן לפי המהירות
    שעוני הצזיום יראו הבדל, שעה במקרה אחד ומיליארד שנה (לפי ההקצנה שלך) במקרה השני. מד הטמפ’ יראו בדיוק אותו דבר. אם שני החבר’ה האלו היו מציירים גרף של מד הטמפ’ לאורך זמן היית רואה שקצב ההתקררות במקרה של ההוא שנח הוא איטי (נגיד מ – 6000K ל- 3 במילארד שנה) בעוד שהגרף של השני היה מראה קצב מהיר 6000K ==> 3 בתוך שעה.

    מה הסתירה??? מה הפרופסורים והמדענים המוסמכים שלדעתך לא מבינים, לא מבינים?
    ישראל, היה פה דיון על טרחנות מתימטית ומדעית שהופנה ליובל. אחד הסממנים העיקריים של טרחן מדעי היא התקפה על הממסד וזלזול במדענים הנוכחים שלא ראו מה שהוא רואה ככה שכדאי שלא תלך לכיוון הזה פן תהפוך לכזה.
    ראה למשל כאן:
    http://www.gadial.net/?category_name=%d7%94%d7%91%d7%9c%d7%99%d7%9d-%d7%a4%d7%a1%d7%90%d7%95%d7%93%d7%95-%d7%9e%d7%aa%d7%9e%d7%98%d7%99%d7%99%d7%9d

    לגבי האנטרופיה, למרות שאתה מתמצא בכמה מונים יותר ממני וקראתי את המאמר שלך בגלילאו שאתה מצטט למעלה, אני חושב שאתה טועה כשאתה אומר “אין שום גורם סטטיסטי שיגרום לכך שדוקא הרכיב בעל מספר המצבים הרב ביותר יתרחש.”. בודאי שיש. תשחק 5 דקות עם קוביה הונגרית ותראה שיש.

    “מבחינה סטטיסטית אנרגטית המצב ללא חיים הוא עם אנרגיה נמוכה יותר ממצב עם חיים. ובכל זאת החיים התרחשו. זה רק מראה שאין שום גורם סטטיסטי שאוכף את עצמו על היקום.”

    – זה אחד הטיעונים הבריאתניים הישנים והלעוסים. כדור הארץ הוא לא מערכת סגורה ויש הזרקה של אנרגיה מהשמש ולכן מקומית יש כאן היפוך של חץ האנטרופיה. באותה מידה אפשר היה לשאול איך נדלקת אש? הרי המצב האנרגטי הנמוך ביותר הוא עץ כבוי, אולם תזריק לו אנרגיה והוא יידלק והאש תחזיק ואף תתפשט.

  330. ואת האבולוציה הבאתי כדוגמא לכך שאיננו מקבלים תמיד את המצב האנרגטי הנמוך ביותר.

  331. היי סטודנט,

    תראה, מבחינת החוקים הפיסיקליים, הזמן הוא סימטרי, ואנו לא אמורים לקבל שום הבדל בין העבר לעתיד.
    החוק השני של התרמודינמיקה הוא לא חוק טבע בסיסי (למרות שהוא מתקיים תמיד בעולמינו), הוא תוצאה של ניסויים ושל עקרונות שימור.
    הפיסיקה הקלאסית שהיא סימטרית בזמן אינה יכולה להסביר תוצא כמו החוק השני. הגורם היחידי שאנו מכירים, שיכול להסביר משהו שדומה לחוק השני זה מנגנון הקריסה (הלא מוכר) של פונקציית גל, כי זהו מנגנון לא סימטרי שבו אנו עוברים מסופרפוזיציה למצב יחיד.

    סטודנט, ההסתברות אינה חלק מחוקי הפיסיקה, ולכן אין היא אמורה לעבוד בייקום שלנו סתם כך. המקום היחידי שמכניס את ההסתברות לייקום זה אינטרפטציה מסוימת למכניקה הקוונטים.

  332. ישראל,

    “מדוע, אם הכל הפיך וסימטרי, וחוקי ההסתברות תקפים לשני הכיוונים, לא תעלה האנטרופיה גם לכיוון העבר?”
    – למה ואיך הגעת למסקנה שהתהליך המתואר בקישור שנתתי הוא הפיך? גם בפעם הקודמת שדנתי איתך קיבלתי רושם שאתה לא מבדיל בין תהליך הפיך ולתהליך לא הפיך.

    נקודה,

    “אין שום גורם סטטיסטי שיגרום לכך שדוקא הרכיב בעל מספר המצבים הרב ביותר יתרחש.”
    – אין גורם כזה, זה פשוט קורה. אתה תוהה למה הסתברות עובדת? למה מתקיים החוק השני? זו כבר שאלה פילוסופית. לרוב חוקי היסוד בטבע אין הסבר ל”למה”. קישור בנושא:
    http://davidson.weizmann.ac.il/online/askexpert/physics/%D7%9E%D7%93%D7%95%D7%A2-%D7%9E%D7%AA%D7%A7%D7%99%D7%99%D7%9D-%D7%94%D7%97%D7%95%D7%A7-%D7%94%D7%A9%D7%A0%D7%99-%D7%A9%D7%9C-%D7%94%D7%AA%D7%A8%D7%9E%D7%95%D7%93%D7%99%D7%A0%D7%9E%D7%99%D7%A7%D7%94-%D7%A9%D7%A8%D7%95%D7%9F

    “מבחינה סטטיסטית אנרגטית המצב ללא חיים הוא עם אנרגיה נמוכה יותר ממצב עם חיים. ובכל זאת החיים התרחשו. זה רק מראה שאין שום גורם סטטיסטי שאוכף את עצמו על היקום.”
    – לא. יצירת חיים לא סותרת את החוק השני של התרמודינמיקה ושאנטרופית היקום גדלה בתהליך האבולוציה.

    “לכן, אין הסבר קלאסי אמיתי לתופעת האנטרופיה.”
    – אני חושב שיש.

    “רק מכניקת הקוונטים יכולה להסביר זאת.”
    – do tell.

  333. ישראל לא הסברת מה הם הקבועים בנוסחא שלך. אתה יודע שרדיוס של חור שחור הוא R=2Gm/c^2 ?

    מבחינה פיסיקלית פרופר אין שום דבר מיוחד בזמן מסויים. זה שהתודעה שלך חווה זמן מסויים אין לכך משמעות פיסיקלית (כפי שאנו מבינים היום את הפיסיקה, לתודעה אין שום פונקציה). לכן אפשר לומר שהתודעה שלנו בסה”כ שטה ע”פ הזמן. זה כמו שבסרט לא ניתן לשאול אם מה קרה עכשיו ישפיע על מה שיקרה בעתיד או בעבר של הסרט (כי כל הסרט כבר ערוך).

    נראה לך שאני הולך לעבור על כל מה שכתבתם.
    תשאל שאלה ספציפית (ולא שאלה תחמנית שכאילו מנסה לגרור אותי לדיון מסויים).

  334. אל מערכת הידען
    לאחרונה מעוכבות הרבה תגובות שלי למשך זמן ארוך יותר מן הרגיל. זה גורם לי להתחיל לחשוד שאין מדובר בצרוף מקרים אקראי. התגובה המדוברת הנוכחית היא
    https://www.hayadan.org.il/astronomers-reach-new-frontiers-of-dark-matter-130112/#comment-331370
    אנא בדקו ואמרו לי מה גורם לעיכובה וכיצד אוכל לחסוך עיכובים דומים להבא. כמן כן, מובן מאליו שאודה לכם אם זו תשוחרר

  335. נקודה.
    אם נדבר פה רק על דברים שאינם ידועים, לא נדבר כמעט בכלל.
    אני חושב שעלולה להיות אולי סיבה נוספת לחץ הזמן, אבל לא כדאי כרגע להתבדר לכיוונים נוספים.
    התגובות האחרונות שלך הן הגיוניות ועוזרות ומראות על ידע רב. הייתי מעוניין שתתן את דעתך על כמה נושאים שהועלו לאחרונה:

    1. אם ניקח את הנוסחה GM=RC^2, הרי שקיבלנו נוסחה יפה שמתארת קשר בין קבועים, ובינהם מסת היקום אך ל ל א מסה אפלה.

    איך זה מסתדר איפוא עם הרעיון שיש למעשה פי 5 לפחות מסה מן המקובל? לא היינו אמורים להוסיף פקטור נוסף בנוסחה כה אלגנטית? האין חוסר התאמה זה לבדו אמור לחסל את כל רעיון המסה האפלה? הרי אין כמעט ספק בערכים של G, R וC.

    או אולי M בנוסחה כבר כולל בתוכו את המסה האפלה?

    2. דבריך אתה: “לגבי השפעה מההווה לעתיד וכדומה. אין דברים כאלו. מבחינה מסוימת העבר והעתיד כבר קיימים ואנו רק חולפים דרכם. כמובן גם פה נכנסות שאלות ממכניקת הקוונטים.”

    לא מובן לי. נשמע קצת פילוסופי. תוכל להרחיב?

    3. אם תוכל לעבור ולמצוא שגיאה בדיון שאני עורך עם ר.ח. בנושא הסתירה כביכול בין התארכות הזמנים ביחסות ותאוריית המפץ הגדול. החל מ:

    https://www.hayadan.org.il/astronomers-reach-new-frontiers-of-dark-matter-130112/#comment-330884

    אם אפשר, רק תשובות פרטניות לעצם העניין, לא הכללות.

    תודה, ולילה טוב מלוס אנג’לס.

  336. סטודנט, המושג הזה של כאוס מולקולרי לא יכול להסביר אי סדר. אילולי מכניקת הקוונטים, אין שום גורם סטטיסטי שיגרום לכך שדוקא הרכיב בעל מספר המצבים הרב ביותר יתרחש. הדוגמא הכי טובה לכך היא האבולוציה. מבחינה סטטיסטית אנרגטית המצב ללא חיים הוא עם אנרגיה נמוכה יותר ממצב עם חיים. ובכל זאת החיים התרחשו. זה רק מראה שאין שום גורם סטטיסטי שאוכף את עצמו על היקום.
    לכן, אין הסבר קלאסי אמיתי לתופעת האנטרופיה. רק מכניקת הקוונטים יכולה להסביר זאת.

  337. ישראל העניין של בעיית חץ הזמן ידועה.

    ככל הנראה הפתרון מצא בגורם מאוד חשוב במכניקת הקוונטים שאינו ברור כלל, קריסת פונקציות הגל. רק גורם לא ברור זה יכול להסביר כיווניות לא ברורה של הזמן (וזה בעצם הטלת הקוביות המפורסמת).

    לגבי השפעה מההווה לעתיד וכדומה. אין דברים כאלו. מבחינה מסוימת העבר והעתיד כבר קיימים ואנו רק חולפים דרכם. כמובן גם פה נכנסות שאלות ממכניקת הקוונטים.

    בכל מקרה, אלו נושאים ידועים ואין פה משהו חדש.

  338. בצעדים מלאי בטחון עלה התובע לדוכן ופתח בחקירתו של החוק השני:

    ש. נאשם, האם תוכל להגיד לבית המשפט מה הסיבה שאתה מגדיל תמיד את האנטרופיה?

    ת. שום סיבה. זה פשוט קורה.

    ש. אבל מדוע? האם זה בלתי אפשרי להשאיר אותה כפי שהיא? להקטין אותה?

    ת. אפשרי, אבל לא סביר.

    ש. מה פירוש?

    ת. חוקי האקראיות המתמטיים מעניקים הסתברות נמוכה ביותר להקטנת האנטרופיה.

    ש. אינני מבין. אתה טוען שהמתימטיקה, המבנה ההגיוני, והמסודר ביותר היא הגורמת להגדלת אי הסדר? זה נשמע לכם הגיוני? פנה התובע אל הקהל בסרקזם.
    הקהל שרק בבוז.
    ת. כן.

    “אני לא מאמין למשמע אוזני,” אמר התובע בסלידה. “אך אני אינני מתמטיקאי מקצועי. אני רוצה לזמן לעדות את פרופסור לייבנוביץ, ראש מכון גאוס לחקר האריתמתיקה.”

    החוק נמלא עליצות. “יופי! מתמטיקאי לבטח יוכיח את טענותי ואת צדקתי. אני אוהב הוכחות מתמטיות. הלוואי שזו תהיה הוכחה בדרך השלילה, אלו אהובות עלי במיוחד.” הוא נשען על מסעד הכסא בציפייה.
    פרופסור לייבנוביץ תפס את מקומו אצל הדוכן ופתח. “לכאורה, החוק צודק. מבחינת ההסתברות המתמטית גרידא, קיימים הרבה יותר מצבי אי סדר למערכת מאשר מצבי סדר.”
    החוק הצניע חיוך קטן כאומר: “נו, לא אמרתי לכם?”
    הפרופסור שלף חבילת קלפים חתומה ופתח אותה מול הקהל. “רואים? הקלפים נמצאים במצב של סדר מקסימלי. בתחילה סדרת העלה, כאשר הקלפים מסודרים בצורה עוקבת מן הקטן אל הגדול. אחריה סדרת הלב, אחריה התלתן ולבסוף סדרת היהלום כשכולן מסודרות באותה צורה, מן הקטן אל הגדול.”
    “עכשיו, אם אערבב את החפיסה ערבוב קל, יקטן הסדר בחפיסה. תשארנה אמנם סדרות קטנות של קלפים ש”יידבקו” זה לזה, אך הסדר בחפיסה ירד, וימשיך לרדת ככל שאמשיך בערבוב, עד אשר יגיע למצב של אי סדר, או אנטרופיה, מקסימלית.”
    בדברו, ערבב הפרופסור את הקלפים מול הקהל, בעוד החוק מתמוגג מנחת בכיסאו.
    “אותו הדבר יקרה לחדר מסודר אשר רוח פרצים נושבת בו. רמת הסדר תלך ותקטן עם הזמן. אם נצלם את החדר במצבים שונים ואחר כך נערבב את התמונות, נוכל תמיד לסדר אותן חזרה בסדר כרונולוגי לפי מצב אי הסדר שבתמונה: ככל שגדל הבלגן או האנטרופיה שבתמונה, כך מאוחרת יותר התמונה. מסכימים?”
    הקהל המהם בהסכמה, ואילו החוק זרח באושר.
    “זו גם הסיבה שחץ הזמן מוגדר ככוון הגדלת האנטרופיה. בטבע, במערכת סגורה, ככל שחולף הזמן כך גדלה האנטרופיה, או למצער נשארת קבועה. רק במקרים נדירים ומיוחדים היא יורדת.
    הסיבה לכך הינה מתמטית טהורה. פשוט קיימים הרבה יותר מצבים אפשריים שהם עתירי אנטרופיה, מאשר מצבים מסודרים.”
    “מ.ש.ל.” אמר החוק בהנאה. ידעתי שהמתמטיקה תוציא לאור את חפותי!”
    “לא נכון!” אמר הפרופסור. המתמטיקה מוכיחה בדיוק את ההפך המוחלט! את אשמתך!”
    “מה?” נזעק החוק “הרגע הוכחת לנו שהאנטרופיה חייבת תמיד לגדול מסיבות מתמטיות טהורות של הסתברות!”
    “נכון, אולם זוהי תמונה חלקית בלבד! עדיין נותרה פתוחה השאלה: למה?”
    “נדמה לי” אמר החוק בקרירות, “שהסברת זאת יפה בדוגמת חפיסת הקלפים. “קיימים הרבה יותר מצבים עתירי אנטרופיה מאשר מצבים מסודרים” – ציטט.
    נכון, אבל מדוע פונה האנטרופיה לכוון בו היא בוחרת עם הזמן? מדוע אין הערבוב מסדר את הקלפים בחפיסה במקום לבלגן אותם? מדוע אין הרוח מסדרת את החדר? מדוע איננו נהיים צעירים יותר עם הזמן במקום להזדקן?”
    “הכל באשמתי, כרגיל”, מלמל החוק.
    “בדיוק” הסכים הפרופסור. “אתה בלגניסט לא קטן. רק צרות צרורות אתה יודע לגרום. כלום יש הסבר אחר לכך שהאנטרופיה תמיד גדלה עם הזמן? חוקי התנועה של ניוטון אינם מחייבים זאת! אם נזין מחשב בנתונים של מערכת כוכבים מסוימת, הוא יוכל לחזות בדיוק רב את מצבה בעוד אלף שנים – אבל גם את מצבה לפני אלף שנים! מדוע אין עקרון שקילות זה חל גם על מערכת תרמודינמית?”
    “כמו שאמרת, המתמטיקה..”
    “אה!” השתלהב הפרופסור. “בעניין זה טעות בידך, ואת זאת אוכל להדגים אם יובא לי לוח וגיר.”
    הקהל נמתח במושביו בציפייה מתוחה, בעוד השמש מטלטל לאולם לוח שחור נייד.

  339. סטודנט
    מתוך הלינק ששלחת לי:
    “אפשר לשאול את עצמנו את השאלה הבאה: מצד אחד, חוקי ניוטון אינם מבחינים בין עבר ועתיד – משוואות התנועה הן סימטריות ביחס לזמן (כלומר אפשר להחליף במשוואות את t ב- t- , והן יהיו נכונות באותה המידה).”

    הסתכל על התמונה המצורפת ללינק. יש בה 3 מצבי אנטרופיה וחץ זמן המצביע ימינה, לעתיד.
    המצב השמאלי הוא מצב של אנטרופיה נמוכה, האמצעי של אנטרופיה בינונית, והימני של אנטרופיה גבוהה.

    אם חזינו במצב האמצעי בזמן הווה, נוכל לקרוא לשמאלי עבר ולימני עתיד.

    1. הסתכל על המצב האמצעי בלבד, והפוך את כיוון החץ, לכיוון עבר.
    2. הסתכל על הציטוט למעלה.
    שאלה: מדוע, אם הכל הפיך וסימטרי, וחוקי ההסתברות תקפים לשני הכיוונים, לא תעלה האנטרופיה גם לכיוון העבר?

  340. ישראל,

    http://davidson.weizmann.ac.il/online/askexpert/physics/%D7%9E%D7%94%D7%95-%D7%9B%D7%90%D7%95%D7%A1-%D7%9E%D7%95%D7%9C%D7%A7%D7%95%D7%9C%D7%A8%D7%99-%D7%95%D7%9B%D7%99%D7%A6%D7%93-%D7%94%D7%95%D7%90-%D7%9E%D7%A9%D7%A4%D7%99%D7%A2-%D7%A2%D7%9C-%D7%AA%D7%A4%D7%99%D7%A1%D7%AA-%D7%97%D7%A5-%D7%94%D7%96%D7%9E%D7%9F-%D7%91%D7%A2%D7%91%D7%95%D7%93%D7%AA%D7%95-%D7%A9%D7%9C-%D7%91%D7%95%D7%9C%D7%A6%D7%9E%D7%9F-%D7%9E%D7%99%D7%9B%D7%90%D7%9C-0

    בקשר לשאלתך הזו:
    “האם לדעתך ניתן, באופן כלשהו, להשפיע מן ההווה או העתיד על העבר?
    …”
    – אין לי תשובה (או דעה) בנוגע לשאלה הזו ואלו שאחריה. אני חושב שפרופ’ יקיר אהרונוב חקר/חוקר תחומים הנוגעים בשאלות כמו זו.

  341. סטודנט.
    בזמנו, כאשר דנו בנושא האנטרופיה, הנקודה שרציתי שנגיע אליה ולא הספקנו, הייתה של הפיכות האנטרופיה בשני כיווני הזמן.

    הרעיון הוא כזה: אם חוקי ניוטון הפיכים לגבי הזמן, אם מחשב יוכל לחשב את מקומם של גרמי השמים בעוד 1000 שנים, אך גם לפני 1000 שנים, אם מערכת תרמודינמית ביסודה איננה שונה מכדורי ביליארד על שולחן, אם חוקי ההסתברות מנבאים שהאנטרופיה חייבת לעלות עם הזמן, מדוע היא אינה חייבת לעלות כאשר נהפוך את כיוון הזמן לכיוון העבר?

    אם נושבת רוח פרצים בחדר ואנחנו יכולים לדעת מה מוקדם ומה מאוחר בתמונות מן החדר רק לפי מידת הבלגן בתמונות, וזה נובע אך ורק משיקולי הסתברות, מדוע אין אותם חוקים בדיוק ינבאו שהאנטרופיה תעלה גם לכיוון העבר? אם הקרח באגם נמס באביב לכיוון העתיד, מדוע שהוא לא יעשה את אותו הדבר גם לכיוון העבר, וזה בדיוק מאותם שיקולי הסתברות?

    אך איני רוצה לפתוח חזית חדשה. אני מספיק שקוע בנושא היחסות כרגע. הייתי רוצה רק להציג לך כמה שאלות, כמי שמבין באנטרופיה ובקוואנטים.

    1. האם לדעתך ניתן, באופן כלשהו, להשפיע מן ההווה או העתיד על העבר?

    2. אם התשובה על 1 היא לא, האם אתה מסכים שזה בדיוק (השפעה על העבר) מה שמנבאת מכניקת הקוואנטים? (ע”ע ניסוי וילר).

    3. במידה והתשובה על 2 היא כן, האם אין זה סותר את כל מה שאמרנו קודם על אי הפיכות האנטרופיה בזמן?

    4. האם אתה מכיר איזה הסבר לניסוי וילר שאינו מצריך חזרה בזמן?

    5. אינני יודע אם עקבת על אחרי הדיון ביני לר.ח., אך במידה והתשובה על 1 היא לא, והתשובה על 2 היא כן, והתשובה על 3 היא כן, האם אתה רואה מוצא מן הנורא מכל (השפעה על העבר) באמצעות מה שהועלה בדיון?

    תודה, וסליחה על השאלות הרבות.

    מאיר.

    בכוונה התעלמתי כרגע מהCMBR, אך זה לא קריטי לשאלה המרכזית: האם תאומים שנפרדו כשטמפרטורת היקום הייתה 6000K יסכימו בינהם על הטמפרטורה כיום, 3K, למרות שעל אחד מהם חלפה רק שנה ביולוגית אחת?

    מה להדליק? חימום או מיזוג?

  342. ישראל, לא תחטוף. לא זו הפעם.
    אינני זוכר מתי בדיוק הגיתי את הרעיון שהחלל הריק אינו ריק, אבל אני זוכר שכבר בכיתה יו”ד (1969) החלטתי שזה מחויב המציאות.
    את משפט החלקיק היסודי ניסחתי כמעט סופית, כשלמדתי שנה א ביולוגיה באוניברסיטה העברית בהר הצופים בשנה”ל 1977-78. את הקשר בין אסימפטוטת ההיפרבולה לבין האופטיקה הגיאומטרית גיליתי כשכבר הייתי שקוע חזק במודל (התגלית המסוימת הזאת, אגב, נפלה עלי בערב בו ייביזהבר כובוהבן כבש את הארוויזיון. אז לא החזקתי טלויזיה בבית בפרינציפ. את השיר שמעתי שוב ושוב מן השכנים. מעניין אם הייתי מגלה את זה אילו כן היה לי מכשיר טלויזיה..).
    אחרי שנה ב עזבתי את הלימודים לטובת משפחה, ילדים ומס הכנסה. מדי פעם הצצתי במודל לצורך ליטושים קלים. חזרתי לאוניברסיטה בשנת 1983 ועשיתי פילוסופיה, מתמטיקה ומדעי המחשב במשך שלוש שנים נוספות. הרעיון שה”יש” התחיל מן ה”אין” בא אלי ב-1983 בחלום, בעקבות דיון בפילוסופים הפרה-סוקראטים, אך אז לא קישרתי בינו לבין המודל. אחר כך עזבתי אותו הרבה הרבה זמן וחזרתי אליו רק לפני שנה בערך כשעשיתי שנה א פיסיקה בגלזגו והתוודעתי לעומק לחומר האפל. החלטתי לחזור אליו כי ההתנהגות של החומר האפל דומה עד כדי זהות לחלקיקים במודל שלי. במהלך השנים החלפתי מעבדי תמלילים (עורך פרימיטיבי על המיינפריים באוניברסיטה, איינשטיין, קיוטקסט דוס, קיוטקסט חלונות, וורד 2, וורד 6…) ופעמיים נהרס לי דיסק קשיח בלי שהספקתי לעשות גיבוי. התחלתי לספר עליו כאן בגלל הקוריוז, ובינתים עלה בי הרהור מגלומני שאולי יש בו פוטנציאל למדליה מוזהבת מידיו של מלך שבדיה. כעת אני משחזר ועורך, ובינתים משתף אתכם כאן בהידען.

  343. יובל.
    אני איני מבקש להטריח. אני מאמין גם שלמדתי את הרקע לנושאים בהם אני דן בצורה ריגורסית, ובמוסדות האקדמים המתאימים. פשוט, משהו לא מסתדר, ופשוט, אני מאמין שזה גם משהו פשוט.

    את השאלות שהעלתי ואני מעלה פה, העלתי גם בפני פרופסורים ומדענים מוסמכים ביותר. מעולם לא קיבלתי תשובה משביעת רצון. למעשה, הגעתי למסקנה (הסובייקטיבית כמובן), שהם לא ממש מבינים את הנושא.

    הבעיה שלי עם דבריך קצת שונה: לא נראה לי, ותקן אותי אם אני טועה, שלמדת את הדברים עליהם אתה מדבר בצורה ריגורסית ( מוסד אקדמי מוכר, שיעורי בית, בחינות, מעבדות).

    הסיבה שאני אומר זאת היא שאריתמטיקה פשוטה תראה לך שהמודל היה קיים אצלך עוד לפני שיכולת אפילו ללמוד פיזיקה באוניברסיטה. פשוט, היית צעיר מדי.

    זה לא אומר שהוא שגוי – אך תצטרך לדעתי להביא נוסחאות יפות, או ניסוי משכנע, כדי לסבר את אוזן שומעיך.

    אם למדת אלגברה ליניארית, בוודאי ידוע לך שניתן לבנות “מודלים” או “שדות וקטורים” שהם סגורים לחלוטין מבחינה מתמטית, וללא סתירות, אך זה אינו הופך אותם למציאותיים.

    הדוגמה הפשוטה ביותר הוא עולם בעל יותר מ3 ממדי מרחב. אין הוא שונה במאומה מהבחינה המתמטית מהעולם המוכר לנו, אך העולם שלנו, לפחות אצל ניוטון, הוא בעל 3 מימדים.

    אז בשביל מה אם כן לבנות מודל? בשביל האגו? בשביל אמא’לה שתהיה גאה כל כך ביוסינקה החכם שלה שהנה אפילו יש לו מודל משלו באינטרנט?

    פעמים רבות שאלת אותי למה אני לא כותב ספרים. התשובה די פשוטה: למה כן? חסרים ספרים בעולם שעולים בהרבה על כל מה שאני מסוגל להפיק?

    זהו. מקווה שלא אחטוף יותר מדי על מה שאמרתי.

  344. אחת שתים שלוש ורבע, ניסיון
    הרגע יסדתי אתר:
    https://sites.google.com/site/darkmattermodel/
    יש שם מאמר שאפשר להוריד: Pattern of returning waves.pdf
    המאמר הזה הוא “stand alone” דהינו הוא נכון מתמטית בלי קשר לשאר המודל
    נא לקרוא בעדינות ולהביא בחשבון שזה נכתב לפני הרבה שנים
    ר.ח וישראל, אתם כבר קיבלתם אותו באימייל

  345. ישראל,

    רק עכשיו ראיתי:
    “איך מוסברת אצלך טמפרטורת היקום? ומערכת הCMBR? והקשר בין השתיים?”

    אין לי הערות בקשר לקשר המובנה בין השתיים. יש לי דברים משמעותיים לומר בקשר ל”אינטרפולציה” (המרכאות מבטאות את דעתי לגביה), שממנה מסיקים (תרתי משמע) את הטמפרטורות בתת-חלקיקי השניה הראשונה של המפץ. אני אגיד אותם במאמר מסודר.

    לגבי שעוני הטמפ’ שאתה מציע, אני רוצה להעיר (בלי קשר לתאורית MCS) שלכאורה אמורה להיווצר במדידות שלהם אנאיזוטרופיות לאורך ציר התנועה, כלומר הפרש קטן בטמפרטורה הנובע מדופלר הפרשי באורכי הגל של ה-CBR הנמדדים עם ונגד כיוון התנועה (וכמובן גם חצאי הפרשים בהשוואה למדידות בניצב לכיוון התנועה). ההפרש הזה אמור ללמד על מהירות אבסולוטית ביחס למערכת המנוחה של ה-CBR. זה צריך לעורר תהיות בקשר ליחסיות הזמן, מכיוון שזה אמור להכריע בין שני נוסעים אינרצייאליים מי נוסע ובאיזו מהירות.

  346. ישראל,

    “אם נראה לך שיובל, יהודה, או אני, או מי שלא יהיה, הם בני דמותו של פינלייסון, מדוע שלא תדפדף?”
    – אני כבר עושה זאת. שים לב שהיחיד שנכנסתי איתו לדיון מדעי הוא אתה – וזה היה רק בנושאים שלטעמי היה טעם לדון עליהם איתך. אחד הדברים החשובים שלומדים בטכניון הוא איך לנצל זמן, אתה יכול להיות בטוח שאני לא אבזבז אותו על דיונים עם הטרחן התורן.

  347. ישראל שפירא, תודה על ההגנה
    כשהם מוקיעים אותי כטרחן זה לא מזדון לב, היות שאני באמת כזה, ולא רק מנקודת מבטם. ר.ח העיר לי, ובצדק רב, שאני מתנהג כמו טרחן (לא באלה המילים, כמובן 🙂 ) והציע לי ניסוח מוצלח יותר שאני מאמץ לחיקי בשתי ידיים. לי אין בעיה רצינית עם זה שקוראים לי טרחן. אני יודע שאני כזה, וזה לא מהיום. ההערות של סטודנטכניון, נקודה ור.ח רפאי.ם (היכן הוא? אנא הבטח לו שאנחנו אוהבים אותו וקרא לו שיחזור), מעבר ללשון הדוקרנית משהו, הינן מועילות בהחלט ואני מברך עליהן. בסופו של דבר אוציא מתחת ידי מודל יפהפה המנוסח כהלכה – והרבה מזה בזכותם.
    אגב, המשפט שלך “אני מייחל לרגע שבו אגלה שיש לי טעות, ואבין איפה” שגור על פי כבר הרבה מאד שנים.
    בב”ח

  348. סטודנט.

    בעיית הטרחנים בפיזיקה היא בעייה ישנה ומוכרת, בעיקר בנושא של כבידה. פיינמן מזכיר אותה בספרו “מה אכפת לך מה חושבים האחרים”. גגל גם PHYSICS CRACKPOTS.
    אולם לבני דורינו יש את היתרון שלא היה בעבר. אז היית חייב לשבת בכינוסים ולהקשיב לכל נודניק. היום בעידן האינטרנט אתה יכול פשוט לדפדף הלאה. יסלחו לי מגיבים מסויימים, אך זה מה שאני עושה עם תגובותיהם.

    אם נראה לך שיובל, יהודה, או אני, או מי שלא יהיה, הם בני דמותו של פינלייסון, מדוע שלא תדפדף? אילו הייתי ממשיך להתנצח עם דב הניס, השאלה הראשונה שהייתי שואל את עצמי היא מה המוטיבציה שלי בעשותי זאת. רגשי עליונות? שעשוע סדיסטי?
    ויסלח לי דב. יתכן שהוא צודק לחלוטין בדבריו. פשוט, לי אין עניין.

    אני יכול לדבר רק בשביל עצמי: אני מייחל לרגע שבו אגלה שיש לי טעות, ואבין איפה. זה מה שאני עושה בדיון הנוכחי עם ר.ח.

    אולם יקשה עלי מאוד להתדיין עם מי שנקודת המוצא שלו היא שהוא לבטח צודק ואני בטוח טועה. אפילו אם אדע שהפסיכומטרי שלו גבוה משלי, והרקע האקדמי שלו בתחומים בהם אני דן עשיר משלי.

    ר.ח.

    בלינק ששלחתי, כשג’יל מגיעה לשעון C2, הוא מראה על 10 שניות ושלה מראה על 8.

    בוא נמתח את זה קצת, כדי שיהיה נוח לעבוד: 10 מיליארד שנים ו8 מיליארד.

    שאלה1: אילו בצמוד לשעון C2 היה גם שעון טמפ’, P2, וC2 מראה 10 מיליארד, מה יראה P2?
    שאלה 2: אילו בצמוד לשעון ‘C של ג’יל היה שעון טמפ’ ‘P, ו’C מראה 8 מיליארד, מה יראה ‘P?

    אל תשכח שאילו מערכות שאינן מואצות, שמבחינתן נמצאות במנוחה.

    והכי חשוב:

    שאלה 3:

    נאמר שיש לנו מצלמה בעלת רזולוציה גבוהה מאוד. בזמן המפגש, המצלמה מסוגלת לצלם את כל 4 השעונים ביחד. אילו זמנים בכל שעון יראה הצילום החד משמעי המשותף?

  349. יובל,

    אני מעדיף שזה ישאר במסגרת של דיון כאן. קבצים אפשר להעלות לאינטרנט דרך כל מיני אתרים. אם תרצה, רשום “file upload” בגוגל ואני מאמין שמשם תסתדר.

  350. ישראל,
    אני לא מבין מה אתה רוצה. הרי הלינק ששלחת בדיוק מראה שאין סתירה בדברים.
    ואני גם לא מסכים להגדרה הזו “זמן אמיתי”, מה זה זמן אמיתי? אתה מתכוון שתאום 2 שכביכול נח זה זמן אמיתי? אז תקרא לו “זמן מנוחה” לעומת זמן תנועה. לכולם היה זמן אמיתי, רק שהקצב שלו השתנה בין תאום לתאום עקב התנועה שלהם. תשאל את איינשטיין.

    אני חושב שמה שמבלבל כאן זה שבניגוד למימדים האחרים אנחנו לא מכירים זמן “נח” אלא רק זמן שנע בקצב של שעה בשעה. דומה הדבר שאנשים שנעים כל הזמן ברכבת חלקם יושבים וחלקם רצים. האם לאלו שיושבים היית מגדיר מרחק “אמיתי” ואלה שרצים ברכבת זה לא אמיתי?

  351. יובל,

    את ה”גרפיקה” שאתה צריך המציאו מזמן – היא נקראת מתמטיקה. לא לחינם סטודנטים למדעים לומדים אותה באוניברסיטה.

    ועצה: אתה יכול להכין מה שאתה רוצה (תמונות, משוואות, איורים וגרפים) בתוכנות כמו word ו-excel או חבילת openoffice, להפוך את הקבצים ל-PDF ולעלות לאינטרנט. אל תגביל את עצמך למילים בלבד בגלל מערכת התגובות כאן.

  352. ר.ח.
    למעשה, אל תשכח שלפני שהתאומים נפרדו הם היו ביחד באותה מערכת לא מואצת, וגם אחרי ששבו ונפגשו הם שנית במערכת לא מואצת. אילו היו 10 תאומים במקום 2 שהיו נפרדים ונפגשים חזרה, אפשר היה לכוון זאת כך שעל כל התאומים המטיילים חלפה שנה אחת בדיוק, וזאת למרות המסלולים השונים שהם עשו. כמובן שאילו היה מחשב על בספינות התאומים המטיילים, הרי שהאינטגרל הקווי של סיכום המהירויות חייב לכלול חישובים יחסותיים.

    בכל אופן, ניתן לדלג על כל הבעיה שהעלית אם נעבור למערכות שאינן מואצות. בשביל זה אם אפשר, הצץ בלינק הבא:

    http://galileoandeinstein.physics.virginia.edu/lectures/time_dil.html
    שים לב שבדוגמה שם אף מערכת אינה מואצת, ולכן מבחינתה נמצאת במנוחה.

    (מצטער על הטירטור, אם זה מסבך, אפשר גם להמשיך בלי הלינק, למרות שזה חשוב). אבל אני חייב לוודא שהסכמנו על הנקודה העיקרית: בזמן הפגישה, אפילו עם 10 תאומים שנמצאים כעת ביחד באותה חללית, כולם יסכימו שהזמן ה”אמיתי” הוא זה של תאום מס’ 2, והראיה היא החימום הפועל בחללית.

  353. ר.ח,
    תודה על העצה הנבונה. האמת היא שנהגתי לפיה בחלק מתגובותיי, אך כנראה לא הדגשתי זאת די הצורך. כפי שנקודה הבחין, ואני אישרתי, אני בונה עולם “כמו במנוע גראפי של משחקי מחשב”. אולם, שלא כמו במשחקי מחשב, אבן הבניין מוגדרת היטב מראש ואינני מוסיף הגדרות אד-הוק בהמשך.

    נקודה, סטודנט, טכניון ושותפיהם לדעה,
    העדר אפשרויות גרפיות במערכת התגובות של הידען מקשה עלי להמחיש את דבריי, ומאלץ אותי להסביר בעזרת הרבה “נפנופי ידיים”. אני רושם לפניי את הערותיכם, אף על פי שהן “קטלניות”, ולומד מהן בתקוה שאצליח לנסח יותר טוב להבא.

  354. נקודה,
    אני מצטרף לביקורת. אתה לא יכול סתם לקטול את מה שיובל כותב כ”שטויות”.
    יובל אומר “בואו נניח אקסיוטמטית שיש א’ ב’ ג’.” ומכאן נלך ונראה מה המסקנות הנובעות מאקסיומות אלה. אם הן תסברנה ללא סתירות דברים שנויים במחלוקת בפיזיקה הנוכחית הרי מה טוב ומה נעים.
    אתה יכול לקטול את המשפטים ואת המסקנות העולות מההנחות המוצא אולם אינך יכול לקטול את האקסיומות.

    לידיעתך תורות המיתרים בנויות בדיוק באותה צורה. “בואו נניח שיש מיתר אחד שמרכיב את כל החומר ביקום שרוטט באורכי גל שונים”. מזה הכל מתחיל.

    יובל,
    הביקורת גם אליך וכבר כתבתי לך את זה קודם. אתה חייב להתחיל במשפט “בואו נניח ש… ונראה מה יוצא מזה” ו-ל-א במשפט “יש חלקיקים צפידים וחלל ריק ואיין וכו'” כאילו שזו עובדה קיימת.

  355. יובל,

    לצערי ככל שמתקדם הדיון אני מתכנס לדעתו של נקודה, ומקבל רושם שאתה בסה”כ עוד טרחן מיני רבים.

  356. ישראל,

    מסכים עם כל מה שאמרת. אבל חסר כאן משהו.
    מה שחסר בתאור המחשבון שהצגת הוא המשפט שהמחשבון הנ”ל נכון למנוחה בלבד.
    למעשה היה צריך להכניס פקטור של רכיב מהירות יחסותי. לכל מהירות החישוב יהיה שונה (ואל תשאל אותי באיזה נוסחא) ואז הכל מסתדר ואין סתירה.
    כלומר בדוגמא שלנו ירידת הטמפ’ ב 13.6 מיליארד שנה במנוחה שווה לאותה ירידה במשך שעה במהירות X.

  357. ישראל ויהודה! חן חן על הפירגון 🙂
    לרגע חשבתי לסגור את הבאסטה וללכת הביתה, אבל בגללכם איאלץ להמשיך לטחון פה את המוח 😀

    נקודה,
    התגובות “הקוטלות” שלך מועילות מאד. אשתדל להביא להבא יותר “בשר”. תודה.

  358. ליובל חייקין
    מסכים אתך. תיאוריה אמורה להסביר רק את מה שידוע בצורה הפשוטה ביותר והיא מושלמת אם היא עשתה זאת!. הנבואה ניתנה רק לנביאים ומדענים לא אמורים להיות נביאים. ניוטון היה מושלם בתיאוריות שלו והוא לא היה אמור להסביר בתיאוריות שלו מה שפעם ידעו על תורת היחסות.
    מציע לקרוא את מאמרי כאן בידען בשם “אבולוציה של תיאוריות” שבו אני מראה דימיון רב בין אבולוציה של חיים לאבולוציה של תיאוריות.
    מצטער שאני עסוק מאוד בעינייניי ובעיותי וקשה לי למצוא את הזמן לתגובות.
    יום טוב
    סבדרמיש יהודה

  359. אינני בטוח שקלטת את הראש של יובל.
    לדעתי יובל אינו מנסה להסביר את הפיזיקה הקיימת בצורה שאנו רגילים לה או מבינים אותה, אלא הוא מנסה ליצור פיזיקה אלטרנטיבית. לכן, מבחינתו לפחות, הוא חייב קודם להגדיר את מושגי היסוד למען יוכל להמשיך לקשר בין הפיזיקה שלו לפיזיקה המקובלת.
    למתבונן מהצד זה באמת יכול להיראות כזיבול מוח, ואולי זה באמת מה שזה. אבל אותו דבר ניתן היה לאמר על תאוריית האתר של מקסוול – אלא שמקסוול הביא את המשוואות.
    שער בנפשך איזה כובע גדול תיאלץ לבלוע, אם יובל יוכל לחלץ את משוואת שרדינגר הבסיסית מתוך המודל שלו. כלום לא יתחילו כולם לדבר אך ורק על חלקיקים צפידים וחללים ריקים?

    אז אל תחסום שור בדישו. לא לפני שלב הנוסחאות, שיובל הבטיח.
    ( ונכון, לי קל לדבר. לי יש פטור).

  360. תפקידי הוא לרדת על מי שעושה הצגות. וכל מי שממלא מיליון תגובות בלי לומר כמעט כלום, עושה הצגות.

  361. נקודה, מה אתה מתנפל על יובל? חשבת פעם על איך היית מגיב אם מישהו היה מעלה פה לראשונה את הרעיון ההזוי של יקומים מקבילים? או מימדים מכורבלים? או חורי תולעת? הצצת פעם בתאוריית מקסוול, עם כל המנהרות, וגלגלי התנופה, והזרמים, והגלים המאונכים?
    תוכל להראות לנו אפילו תגובה אחת שלך שאינה מסתכמת ב: “הכול שטויות!” או “לך תלמד!” ניסית פעם לפתור בעצמך את הבעיות הלא פתורות של הפיזיקה, או שתפקידך היחיד הוא לרדת על מי שמנסה?

    הרי הבטחת לנו קישורים לפתרון בעיית האי לוקליות בשזירה קוואנטית. איפה הם? העולם תוהה ומשתומם.

  362. נקודה,
    היות שקיבלתי כאן מעט מאד פידבקים, קפצתי על ההתכתבות איתך כמוצא שלל רב. אני מודה לך על תגובותיך ומצטער על שבגללי בזבזת זמן היקר.

  363. יובל, אתה בעצם טוען שהמבחן הוא בנאיביות של הבוחן. ככל שהבוחן נאיבי יותר, ומתעלם מיותר דברים כך המודל שלו פשוט יותר ולכן נכון יותר.

    זה בערך המודל שאתה מציג. כתבת פה כבר עשרות אלפי מילים, וחוץ מחלקיק צפיד וחלל ריק שאין לך שמץ של מושג מה בלל המילים הזה אומר. לא אמרת פה כלום.
    ושים לם כמה מילים אתה מבזבז פה באתר רק כדי לומר שאצלך הכל בא לאט.

    שמתי לב שמה שמגיע אצלך לאט זה שום דבר. רק חלקיקים צפידים וחללים ריקים שאין להם שום משמעות.

    בקיצור שיניתי את דעתי עליך וכעת אחרי ההתחמקויות וההתעלמויות שלך משאלות פשוטות ששואלים אותך כאן, אני מכיא בך כזבלן מוח מן השורה.

  364. ניסיון אחרון לעבור את מחסום העיכוב (12 שעות):
    נקודה, הצדק עמך,
    זה בדיוק מה שאני עושה: בונה עולם כמו במנוע גראפי של משחקי מחשב. אבל לא זו הנקודה. הנקודה היא שאני משתמש באבן בנין מאד פשוטה, ומנסה להראות כי די בה כדי לבנות את כל העולם כך שיהיה זהה לעולם המוכר לנו מן הפיסיקה.
    הגדרת אבן הבנין הזו מורכבת מארבעה סעיפים (למעשה, הגעתי אליה מתוך הגדרה פשוטה אף יותר אך אינני מביא אותה כאן משום הקשיים הטכניים: היא מצריכה המחשה גראפית):
    א) העולם בנוי ממספר גדל והולך של חלקיקים ומאינסוף חלל ריק.
    ב) בהינתן די חלל ריק, משרים החלקיקים מעצמם על סביבתם ליצירת חלקיקים כמותם. חלקיק המוקף בחלקיקים בצפיפות גדולה מאד, משנה את מצבו לפיסת חלל ריק.
    ג) חלקיק בודד תופס נפח מוגדר במרחב אשר שום חלק ממנו אינו תפוס בוזמנית על ידי חלקיק אחר.
    ד) החלקיק הינו בעל תנועה עצמית עצמונית המשתנה באורח אקראי בכיוונה ובמהירותה, להוציא הגבלה אחת הנובעת מן הסעיף הקודם: תנועתו של חלקיק אינה מתקיימת אל תוך נפח התפוס על ידי חלקיק אחר.
    שלל ההתנהגויות של אוסף החלקיקים נובע מן ההגדרה הזאת בלבד, ללא תוספות.
    ל ד ע ת י, מ בח ן ש ל מ ו ד ל א י נ נ ו ב ת ח ז י ו ת ש ל ת ו פ ע ו ת ש ל א נ צ פ ו ק ו ד ם (ג ם א ם י ש ב א ל ה א פ ק ט ד ר מ ט י מ ש כ נ ע ), כ י א ם ב פ ש ט ו ת ו ו ב י כ ו ל ת ו ל ה ס ב י ר ה ר ב ה. ה מ ו ד ל ש ל י מ ו ג ד ר ב פ ש ט ו ת, כ ד ל ע י ל, ו ה ו א מ ת י י מ ר ל ב נ ו ת ד ג ם ש ל כ ל ה פ י ס י ק ה ה י ד ו ע ה. א ם ה ו א י צ ל י ח ב מ ש י מ ת ו, ה ר י ש ה ו א ע ב ר א ת ה מ ב ח ן ב ה צ ל ח ה ג ם ל ל א ת ח ז י ו ת.

    התגובה ממתינה לאישור.

  365. סטודנט, טכניון וכל מי שעוקב,
    לאחר מספר ביקורים בבלוג של מאיר, שוכנעתי כי המודל שלו הינו המשך מיידי של המודל שלי. אני מביא תאור פלסטי, ומאיר מציב בו נוסחאות. יש בכוונתי לאמץ את הנוסחאות של מאיר כבסיס לחישובים הכמותיים במודל שלי. עד שאסיים את המלאכה, אתם מוזמנים לגלוש את הבלוג שלו. אנא גגלו “פיזיקת MCS”.

  366. סליחה על ההתחזות : )

    ההודעה הקודמת היא כמובן ממני לישראל (ישראל לא נוהג לדבר אל עצמו)!

  367. ישראל,

    ה-spacents הם פרודות צפופות שהרצף הדביק שלהן מהווה את ה”אריג” של המרחב. כלפי החלקיק ברגע הפעולה שלו, האריג הוא סטציונרי. הEMP “זוחל” בתוך האריג הזה בצעדים מדודים שאורך כל אחד מהם הוא spacent בודד בכיוון אקראי, התלוי במיקום היחסי בו מתעורר הEMP בין פרודות האריג ברגע הפעולה. בין פעולה לפעולה הEMP והאריג שקופים זה לזה. מרווח הזמן בין פעולה לפעולה (זמן המחזור של החלקיק) קבוע, וגדול מאוד ( >>) מזמן האינטראקציה, שהיא למעשה “זינוק קוונטי” למרחק של spacent בודד, במהירות “~אינסופית”, שמתמצעת למהירות האור כפול שורש שתיים כשמחלקים את אורך הקפיצה לזמן המחזור.

    על מנת להבין אנרגיה “פוטנציאלית” כתנועה, נוכל להתייחס לאנרגיה פוטנציאלית גרביטציונית (MCS היא תורת כבידה, ובמתח של קפיץ מעורבת אנרגיה “פוטנציאלית” חשמלית) :

    אנרגיה פוטנציאלית גרביטציונית היא אסימטריה בשדה הכבידה שאליו חשוף חלקיק אלמנטרי, הבאה לידי ביטוי כתנועה של החלקיק (תמיד, גם כאשר אנו מתייחסים אליו כאל “חלקיק במנוחה”). אפשר להתווכח ולטעון ש”אסימטריה” מקומית של שדה כלפי חלקיק היא “אנרגיה פוטנציאלית”. כנגד זה אפשר להתווכח ולטעון שעל מנת שתהיה משמעות לאסימטריה המקומית הזו, צריך שיהיה שם חלקיק בעל פוטנציאל פעולה על השדה, ולכן גם החלקיק מבטא “אנרגיה פוטנציאלית” שהיא הכושר שלו לעשות “משהו” לשדה.

    לכן, אני חושב שהנכון הוא שאנרגיה היא האינטראקציה עצמה, כלומר ה”מימוש” של הפוטנציאל (משני צדי המתרס) שהוא תמיד קינטי, וכך להישאר עם סוג אחד של אנרגיה: אינטראקציה בין חלקיק לבין המרחב, הבאה לידי ביטוי בתנועת החלקיק כלפי המרחב במהירות האור.

  368. דעתי היא כי מבחן של מודל איננו בתחזיות של תופעות שלא נצפו קודם, כי אם בפשטותו וביכולתו להסביר הרבה

    התגובה ממתינה לאישור.

  369. ובאותה הזדמנות אציין שהסבר (ובכלל, מודל) צריך להיות גם ריגורוזי ומתמטי.

  370. נקודה, בקיצור עד שתגובה מעוכבת תאושר:
    דעתי היא כי מבחן של מודל איננו בתחזיות של תופעות שלא נצפו קודם, כי אם בפשטותו וביכולתו להסביר הרבה

  371. סטודנט, טכניון
    אני מבין ללבך. אילו הייתי במקומך הייתי מתפקע מחוסר סבלנות.
    אבל אם אכתוב כעת משהו כמו: “מסלול תנועת האור הוא היפרבולי, אך אנחנו מתייחסים אל האסימפטוטה ולכן הוא נראה כנע בקו ישר; זה מסביר מדוע זוית הפגיעה שוה לזוית ההחזרה” היית מרים גבה בחוסר הבנה או בזלזול מוחלט.
    אתמול הצלחתי לשחזר חלקים מדיסק-קשיח שנהרס לפני 15 שנה ומצאתי בו כמה דברים שנשתכחו ממני. צר לי על האיטיות. זה כל מה שאני יכול לענות לך לעת עתה

  372. יובל,

    אני בכל זאת לא מבין משהו. אם יש לך את ההסברים למה שרשמתי, למה אתה לא מפרסם אותם?

  373. אבי, מערכת הידען:
    עיכוב התגובות, מוצדק ככל שיהיה, מקלקל את הדינמיקה של הדו-שיח. האם אין מקום לשכלל את השיטה?
    בהזדמנות זו, אנא שחרר לפרסום את תגובה 331216
    תודה

  374. סטודנט, טכניון
    אצלי ההסברים מגיעים לאט. מצטער, אך זה מה שיש. לתופעה אחת לא מצאתי הסבר, אך אני סבור שזה רק משום שלא חיפשתי. זו שאלת האי-לוקאליות בשזירה קואנטית שהעלה כאן עמיתנו. בהזדמנות קרובה אתעמק בה, ואם אצליח להסביר אותה ללא הוספת סעיפים להגדרה המקורית של החלקיק, אביא את ההסבר ללא דיחוי.

  375. נקודה, הצדק עמך,
    זה בדיוק מה שאני עושה: בונה עולם כמו במנוע גראפי של משחקי מחשב. אבל לא זו הנקודה. הנקודה היא שאני משתמש באבן בנין מאד פשוטה, ומנסה להראות כי די בה כדי לבנות את כל העולם כך שיהיה זהה לעולם המוכר לנו מן הפיסיקה.
    הגדרת אבן הבנין הזו מורכבת מארבעה סעיפים (למעשה, הגעתי אליה מתוך הגדרה פשוטה אף יותר אך אינני מביא אותה כאן משום הקשיים הטכניים: היא מצריכה המחשה גראפית):
    א) העולם בנוי ממספר גדל והולך של חלקיקים ומאינסוף חלל ריק.
    ב) בהינתן די חלל ריק, משרים החלקיקים מעצמם על סביבתם ליצירת חלקיקים כמותם. חלקיק המוקף בחלקיקים בצפיפות גדולה מאד, משנה את מצבו לפיסת חלל ריק.
    ג) חלקיק בודד תופס נפח מוגדר במרחב אשר שום חלק ממנו אינו תפוס בוזמנית על ידי חלקיק אחר.
    ד) החלקיק הינו בעל תנועה עצמית עצמונית המשתנה באורח אקראי בכיוונה ובמהירותה, להוציא הגבלה אחת הנובעת מן הסעיף הקודם: תנועתו של חלקיק אינה מתקיימת אל תוך נפח התפוס על ידי חלקיק אחר.
    שלל ההתנהגויות של אוסף החלקיקים נובע מן ההגדרה הזאת בלבד, ללא תוספות. לדעתי, מבחן של מודל איננו בתחזיות של תופעות שלא נצפו קודם (גם אם יש באלה אפקט דרמטי משכנע), כי אם בפשטותו וביכולתו להסביר הרבה. המודל שלי מוגדר בפשטות, כדלעיל, והוא מתיימר לבנות דגם של כל הפיסיקה הידועה. אם הוא יצליח במשימתו, הרי שהוא עבר את המבחן בהצלחה גם ללא תחזיות.

  376. מאיר.
    מאיפה האנגלית המצויינת? אבא אבן יכול לקחת דוגמה.

    שאלות על פרק 4.

    להבנתי, אתה טוען ששקילות מסה אנרגיה נובעות מאינטראקציית חלקיקי החלל ( שאתה מכנה Spacent) עם חלקיקי היסוד אותם אתה מכנה EMP.

    מכיוון שהEMP נעים במהירות של שורש 2 כפול C, אנחנו מקבלים את השקילות כאנרגיה קינטית.

    1. האם הבנתי נכון?
    2. האם זה לא אמור להיות ההפך? הSpacent הם אלו שנעים, כמו מולקולות אויר, והEMP נעים דרכם?
    3. לטענתך, כל צורות האנרגיה מקורן בתנועה (נשמע סביר מאוד). מה עם אנרגיה פוטנציאלית? קפיץ דרוך לדוגמה?
    אינטואיטיבית נראה לי שלפי המודל שלך זה די ברור למה גם פה, אבל הייתי מעוניין לשמוע את ההסבר שלך.

  377. יובל אני לא מאמין שכתבת את מה שכתבת ברצינות.
    באותה מידה אתה יכול לטעון שעצם העובדה שהעולם קיים זה הוכחה למודל שלך. זה פשוט מגוחך.

    תציע ניסוי שמנבא תוצאות אחרות ממה שחוזות תאוריות שונות.

    העניין הוא שאינך מסוגל לעשות זאת, כיוון שהמודל שלך אינו חוזה שום דבר, זהו מודל גמיש המאפשר התאמה לכל יקום שיעלה על הדעת.
    זה מה שעושים במנועים גרפיים של משחקי מחשב, אפשר לבנות איתן כל מיני “עולמות”.

  378. יובל,

    אה וכן, אני (ומאמין שעוד כמה שדנים איתך) עדיין מחכה להסברים על הנושאים שרשמתי לא מזמן. אם אין לך כאלה, אנא אמור זאת.

  379. סטודנט, טכניון, הצדק עמך. יצא המרצע מן השק 😛 אז נמשיך:
    אמנם ברמת החלקיק לא מתקיים שימור תנע, אך ראינו כי ברמת הפרוטון התופעה הזו מתקיימת. תנועתו של פרוטון היא ביטוי של יחסי צפיפויות בין צד אחד שלו לצד הנגדי, ויחס הצפיפות הזה נשמר לאורך זמן. תנועה של פרוטון היא למעשה תנועה של גל. שני פרוטונים אשר במהלך תנועתם נתקלים זה בזה, מחליפים ביניהם את יחסי הצפיפויות, ובדיוק כך נראית התנגשות אלסטית בפיסיקה. במהלך התנגשויות אלסטיות מחליפים פרוטונים תנע. אך פרוטונים לא רק מקבלים תנע זה מזה אלא גם מייצרים תנע מתוך עצמם. מה גורם לתנע העצמי של פרוטונים?
    נבדוק את המתחולל ב”לבו” של פרוטון: צפיפות החלקיקים במרכזו של הפרוטון הינה גדולה. חלקיק הנמצא בצפיפות גדולה עשוי “למות” ולהיהפך לפיסת חלל ריק – חור. אל החור הזה יכול חלקיק כלשהו להיכנס ולהותיר אחריו חור אשר גם אליו נכנס מתישהו חלקיק שמותיר מאחוריו חור. כך, בגלל הצפיפות הגדולה של הפרוטון נוצר באזור המרכז שלו חור, וזה מפעפע החוצה. היות שבין חלקיקים לא קיימת התנגשות אלסטית, הרי הצפיפות במרכז הפרוטון שבה לגדול. ברגע מסוים נוצר שם חור נוסף וגם זה מפעפע החוצה. התהליך הזה עשוי לחזור על עצמו ללא הפסק. חור כזה, הנוצר בלב הפרוטון ומפעפע החוצה, גורם לכך שברגע מסוים הצפיפות המקומית באזור אחד של מעטפת הפרוטון קטנה יותר מן הצפיפות באזור הנגדי. הפרש הצפיפויות גורם לפרוטון לנוע אל האזור הצפוף יותר. חור חדש הנוצר במרכז הפרוטון מפעפע החוצה ללא העדפת כיוון ועשוי לגרום לפרוטון לנוע בכיוון חדש.

  380. ר.ח.
    או קי, הסכמנו שהם יסכימו על הטמפ’.
    השלב הבא: הולכים לאתר:
    http://hyperphysics.phy-astr.gsu.edu/hbase/astro/expand.html#c3
    יש שם למטה נוסחה – נוסחת פרידמן לקשר בין הטמפ’ והזמן שחלף מהמפץ הגדול.
    יש שם גם מחשבון – תוכל להכניס טמפ’ ולקבל זמן, בכל דיוק שתרצה, או להפך.

    מסקנה א’:
    עיי מדידת הטמפרטורה בלבד, ניתן לדעת את הזמן שחלף מאז המפץ הגדול בכל מקום ביקום, בכל דיוק שנרצה ושיתאפשר לנו טכנית.

    השלב הבא: התקנת שעוני טמפ’.
    ניתן לעשות זאת עיי חיבור מחשב למדחום. (אין זה משנה כרגע אם זוהי טמפרטורת הCMBR).

    מסקנה ב’:

    ניתן לצייד את כל מי שנרצה בשעוני טמפ’.

    השלב הבא: שעוני צזיום.

    שעונים אלו מראים את הזמן היחודי למערכת, ומושפעים מתאוצות.

    מסקנה ג’:

    אם על תאום 1 חלפה רק שנה ביולוגית מאז פרידתו מתאום 2, גם שעון הצזיום שלו יראה שחלפה רק שנה.

    ואם על תאום 2 חלפו 13.6 מיליארד שנים, גם שעון הצזיום שלו יראה שחלפו 13.6 מיליארד שנים.

    ( אני מניח שלזאת התכוונת ב: “הויכוח בינהם יהיה מה קצב התקררות היקום. הראשון יגיד שהיקום מתקרר מ X התחלתי ל – 2.75 בשעה והשני יגיד – לא, הקצב הוא X-2.75 חלקי 13.6 מיליארד שנה.”).

    מסקנה ד’:

    אם נצייד את שניהם בשעוני טמפ’ + שעוני צזיום, הרי וידאו של שני השעונים ביחד יראה שאצל תאום 2 קצב השעונים זהה או כמעט זהה, ואילו אצל תאום 1 הקצב של שעון הטמפ’ מהיר בהרבה מזה של שעון הצזיום.

    מסקנה ה’:

    בעת הפגישה בינהם, יראו שני שעוני הטמפ’ את אותו הזמן (13.7 מליארד שנה) ואילו שעוני הצזיום יראו זמן שונה לחלוטין (100,001 שנים ו13.7 מיליארד שנים).

    עבור בבקשה על המסקנות והגד לי אם אתה מסכים, ואם לא איפה.

  381. ישראל,
    מסכים, שניהם יראו את אותה טמפ’ במפגש הראשון – נניח X (כמה אלפי מעלות) ובמפגש השני היום נניח 2.75K.
    הויכוח בינהם יהיה מה קצב התקררות היקום. הראשון יגיד שהיקום מתקרר מ X התחלתי ל – 2.75 בשעה והשני יגיד – לא, הקצב הוא X-2.75 חלקי 13.6 מיליארד שנה.
    אין טועה וצודק. שניהם צודקים, בגלל זה יש מלחמות בעולם.

  382. יובל,

    אתה יותר מדבר על ה”מודל” שלך מאשר מפרסם אותו בפועל. זה יוצר תדמית של קשקשן – סתם שתדע.

  383. מן הסתם, במערכת הידען לא עובדים בשבתות. אחרת, לא ברור לי מה גורם לכך שתגובה תמימה שלי ממתינה לאישור כבר כמעט יממה.

  384. ר.ח.
    אנחנו ממש קרובים לשורש הבעיה. ברשותך, אדלג ישר לשאלה 2 שלך כדי לראות אם נוכל להגיע להסכמה. נראה בעצמנו לאן זה מוביל. אנסה גם להשתמש במיתודה הסוקרטית של שאלות ותשובות כדרך לחקר האמת.

    2) לגבי “שעון הטמפרטורה” אני עדיין חושב שאתה טועה ואין סתירה. שני תאומים, אחד טס וחוזר אחרי שנה. הוא יהיה מאד מופתע לראות שאחיו שנשאר הזדקן ב – 10 שנים. הוא גם יהיה מאד מופתע שלפי שעון הטמפ’ ע”ש שפירא גם עברו 10 שנים. אם הוא היה מסתכל בשעון דומה שהיה איתו בחללית הוא היה רואה שהשעון כל הזמן ממהר פי 10 מהצפוי. היכן הסתירה?

    בכתבה עם חסוי, נתתי דוגמה קיצונית שהפרש הזמנים בין התאומים הוא 13.6 מיליארד שנה. תאום 1 טוען שחלפה רק שנה מזמן הפרידה, ואילו 2 טוען שחלפו 13.6 מיליארד.

    ציינתי גם שלדעתי הצדק עם 2, ואחד טועה. הראיה: קר, קר מאוד בחוץ, ושניהם מסכימים על כך. אילו 1 היה צודק, היו צריכים להפעין מזגן. זה שהחימום פועל, מראה ש2 צודק.

    שאלות:
    1. האם אתה מסכים איתי ששני התאומים בזמן מפגש יסכימו בינהם על הטמפרטורה?

    2. האם אתה מסכים איתי ששניהם יסכימו שבעניין הטמפרטורה, קר מאוד בחוץ (3K), ולא כמה אלפי K כמו שהיה בזמן שנפרדו?

  385. נקודה, תגובתי אליך “מחכה לאישור” כבר 12 שעות בערך.
    בקיצור, הצעתך מקובלת עלי אך אני מוסיף כי ניסוי שמאשש הוא אמנם משכנע, אך הוא לא אָמָת המבחן היחידה. כלים נוספים הם מבחן הפשטות ומבחן העומק. אני טוען שהמודל שלי פשוט יותר ומסביר יותר מן המודלים הקיימים. כמו כן, אני מתנצל על זה שאני מביא את הדברים טיפין טיפין.
    תודה

  386. ישראל,

    1) אתה ענית לשאלתי על חסם תחתון למהירות האור : “. נקודה יפה. אבל אל תשכח שני דברים: קודם, שאינני יודע אם לפי המודל האור נע בכל המהירויות. יתכן שהוא נע בקשת מהירויות מסויימת, בהתאם למהירויות המתנד במקור. והשניה, החשובה, שמה שחשוב הוא, תחום המהירויות להם רגיש הגלאי, או הצופה. כמו שאם אתה מנסה למדוד את המרחק לקשת בענן, תראה תמיד שהיא במרחק מסוים וקבוע ממך הצופה, ולא משנה איפה אתה נמצא.”

    תסלח לי אבל זו לא תשובה אלא נפנוף ידיים שלא אומר הרבה. לאמר שהאור נע בקשת מוגבלת של מהירויות זה בדיוק כמו לאמר שהוא נע במהירות אחת. ודבר שני האם ייתכן שהגלאי רגיש רק לאור במהירות יחסית אחת מדוייקת כל כך? אם כן זו שאלה מאד מעניינת, איך זה שאף אחד לא בנה גלאי שמסוגל למדוד אור במהירויות משתנות? (אולי זה הכיוון שאתה צריך ללכת,יש להניח שמי שבנה גלאי כזה זרק אותו לפח כי הוא חשב שהוא לא עובד?)

    2) לגבי “שעון הטמפרטורה” אני עדיין חושב שאתה טועה ואין סתירה. שני תאומים, אחד טס וחוזר אחרי שנה. הוא יהיה מאד מופתע לראות שאחיו שנשאר הזדקן ב – 10 שנים. הוא גם יהיה מאד מופתע שלפי שעון הטמפ’ ע”ש שפירא גם עברו 10 שנים. אם הוא היה מסתכל בשעון דומה שהיה איתו בחללית הוא היה רואה שהשעון כל הזמן ממהר פי 10 מהצפוי. היכן הסתירה?

  387. ניסוח מחודש של תגובה הממתינה לאישור כבר 9 שעות.
    נקודה, על דבריך אלה: “תציע ניסוי שכל אחד יכול לבצע שמוכיח שההיגיון הפנימי שלך אכן מדבר על המציאות… כך עובד המדע, וזה משמעו של הניסוי”.
    מה שאתה אומר הוא יפה ועשוי אפילו לשכנע, אבל זה לא מספיק כדי לשמש כהוכחה. בפסקה שכותרתה “התכווצות לורנס ועידוש כבידתי” (https://www.hayadan.org.il/astronomers-reach-new-frontiers-of-dark-matter-130112/#comment-329898) הצעתי, למעשה, לבצע את ניסוי מייקלסון מורלי ואת ניסוי אדינגטון וחזיתי (בדיעבד) את תוצאותיהם. אילו הייתי מפרסם את המודל שלי באמצע המאה ה-19, בטרם עידן תורת היחסות, היו הניסויים ההם מאששים אותו. המבחן לתאוריה מדעית כולל לא רק ניסוי עם תחזיות שמתממשות אלא גם הסבר פשוט יותר מן הקיים. אני טוען שהמודל שאני מביא עדיף על המודלים הפיסיקלים הקיימים היום בשני דברים: הוא פשוט יותר ומסביר יותר. אך אפילו זה עדיין לא אומר שהמודל שלי נכון; זה רק מגדיל את הסבירות לכך.
    הריני מתנצל על כך שאני מביא את הדברים בטפטוף דליל. כאמור, מדובר בהרבה חומר ישן שעליי לשלוף מאמצעי איחסון “עתיקים”, לפענח ולערוך אותו.
    הוצע לי לפתוח בלוג ולרכז בו את הדברים, ואני שוקל זאת ברצינות.

  388. נקודה, על דבריך אלה: “תציע ניסוי שכל אחד יכול לבצע שמוכיח שההיגיון הפנימי שלך אכן מדבר על המציאות… כך עובד המדע, וזה משמעו של הניסוי”.
    מה שאתה אומר הוא יפה ועשוי אפילו לשכנע, אבל זה לא מספיק כדי להוות הוכחה. בפסקה שכותרתה “התכווצות לורנס ועידוש כבידתי” (https://www.hayadan.org.il/astronomers-reach-new-frontiers-of-dark-matter-130112/#comment-329898) הצעתי, למעשה, לבצע את ניסוי מייקלסון מורלי ואת ניסוי אדינגטון וחזיתי (בדיעבד) את תוצאותיהם. אילו הייתי מפרסם את המודל שלי באמצע המאה ה-19, היו הניסויים ההם מאששים אותו. אולם היות שאיינשטיין הביא את תורות היחסות עם אותן תחזיות, נעשו אלה מקובלות. המבחן לתאוריה מדעית כולל לא רק ניסוי עם תחזיות שמתממשות אלא גם הסבר פשוט יותר מן הקיים. אני טוען שמודל החלקיקים של החומר האפל שאני מביא הינו גם פשוט יותר מן המודלים הפיסיקלים הקיימים היום וגם מסביר יותר מהם. אך אפילו זה עדיין לא אומר שהמודל שלי נכון לחלוטין; זה רק מגדיל את הסבירות לכך.
    הריני מתנצל על כך שאני מביא את הדברים בטפטוף דליל. כאמור, מדובר בהרבה חומר ישן שעליי לשלוף מאמצעי איחסון “עתיקים”, לפענח ולערוך אותו.
    הוצע לי לפתוח בלוג ולרכז בו את הדברים, ואני שוקל זאת ברצינות.

  389. טו-אוב.
    יש שורת מחץ ששום צפונית שעושה את הפוסט דוקטורט במדעי הדשא אינה מסוגלת לעמוד בפניה:

    “חליק דאווין, מספיק עם האוונטה”.

  390. תודה יובל. הנה ההמשך:

    רנדו סיפר את סיפורו בקולו המתכתי, הדיגיטלי. ואנשים רבים שעקבו באמצעי התקשורת אחר “מרד הנפלים” – פאראפרזה על סיפור ידוע בו קרה בדיוק ההפך – מאות מיליוני אנשים אשר שאלו את עצמם יום יום מה התכלית והטעם של חייהם התפלים, אנשים שלא היו יפים (יחסית למי?), לא גבוהים (יחסית למה?), לא עשירים (יחסית לכלבלבון חסר כל אך מאושר?), לא חכמים (יחסית לקוף? לארנב? או לאנשים אחרים), מרי נפש (כלום באמת יכלו אחרת?), שעבדו, אם בכלל, בעבודות אותן תעבו בשכר מועט ללא יכולת אמיתית להתקדם, קיבלו סופסוף את ההסבר לתכלית חייהם חסרי הסיפוק והתקווה: להיות מאגר החום הנמוך המאפשר לכל המערכת הפסיכומכנית הגדולה להתגלגל.

    מאיר, לא שכחתי אותך, אני מנסה לשחרר כמה שעות כדי שאוכל לעמת את התאוריה שלך עם שיימה.

    שאלה: איך מוסברת אצלך טמפרטורת היקום? ומערכת הCMBR? והקשר בין השתיים?
    אגב, האם אתה או מישהו יודע אם ניתן למדוד את טמפרטורת החלל במדחום קלווין פשוט? ואם כן, מה הוא יראה באיזור מוצל?

  391. יובל, פשוט מאוד, תציע ניסוי שכל אחד יכול לבצע שמוכיח שההיגיון הפנימי שלך אכן מדבר על המציאות, ולא על אילוזיות שלך. כך עובד המדע, וזה משמעו של הניסוי.

  392. נקודה,
    מה שאתה אומר הינו נכון מאד. לכל אחד יש ההגיון הפנימי שלו, והמבחן הוא המציאות החיצונית. ההגיון הפנימי שלי עובר בהצלחה את מבחן המציאות (על פי השיפוט שלי 😛 ), אך הבעיה שאני נתקל בה כבר למעלה מיובל שנים היא איך להפוך את ההגיון הפנימי שלי לנחלת הכלל.

  393. ר.ח, תודה 🙂
    דוקא בשלילת השלילה אין מיסטיקה. כולנו מכירים את הפעולה הזאת מן היומיום כ”חיוב”. ה”מיסטיקה” היא בחידוש של ספנסר שאותו אני מאמץ אל חיקי בחום. כפי שהמספרים “הדמיוניים” נמצאים בקבוצה החיצונית למספרים “הממשיים”, כך גם “שורש השלילה” נמצא בקבוצת ערכי אמת החיצונית לקבוצה {“אמת”, “שקר”}. נזקקתי לו כדי להימנע מהגדרה מעגלית של “יש” מתוך “יש” (כי גם פעולת השלילה שאנחנו מכירים קיימת בעולם ה”יש”).
    בשעתו העלה ישראל שפירא את השאלה מהו שורש i. התשובה לזה היא ± שורש חצי כפול (1+i) והיא אינה מצריכה מספרים דמיוניים נוספים. באופן הזה גם השורש של שורש השלילה הוא ביטוי המורכב מערכי האמת בקבוצה {“שלילה”, “שורש השלילה” } ואינו מצריך ערכי אמת נוספים.
    לדעתי, המכשול היחיד בהבנת העניין הזה הוא הסתמכותנו היתירה על האינטואיציה.

  394. אולי אין זה נעים במיוחד, אך חייבים לחשוף בפרהסיא את אחת הפרשיות העלומות ביותר של ממלכת המתמטיקה…אולי הפרק האפל ביותר… מרידת המספרים הפשוטים בהנהגתו האמיצה של רנדו, המספר הצנוע והאקראי מכל מספר.
    כאשר i הקטן הצטרף למשפחת המתמטיקה, ערכו לכבודו משתה כדת וכדין, שנועד, כך הבטיחו, לכל עולם המספרים. מי לא היה שם? כל שמנה וסלתא של המתמטיקה, כל דוכס וכל רוזן שהוא, וכל מי שהוא קצת משהו. טורים אינסופיים, אשר התכנסו במיוחד לכבוד המאורע, צעדו בסך. בימות בידור הוקמו לסדרות. מובן שבכל הרחובות הוצבו חסמים עליונים ותחתונים כדי למנוע מן הפרולטריון להתחכך בשועים והאצילים. ואז, כאשר ניתן האות, הכריז הכרוז: “גבירותי ורבותי, אני מבקש מכולם לכרוע ברך,והרשו לי להציג לפניכם את חמשת נסיכי המתמטיקה, 1, 0, i,e,ו- π.”
    את 1, 0 ו-i אתם כבר מכירים. π הוא כמובן היחס בין קוטר המעגל והקיפו, בערך 3.14. ערכו המספרי של e הוא בערך 2.72, ומוגדר בקלקולוס כמספר שהלוגריתם הטבעי שלו שווה ל-1.
    חמישתם ניצבו על במת הכבוד, בני אלים מורמים מעם, בעוד הכרוז מפרט את יחוסו ומעלותיו של כל אחד מהם. “והנה, הגענו לרגע הגדול, לנוסחה הגואלת אשר תאחד לנצח את תחומי האלגברה, הקלקולוס והגיאומטריה!”
    האורות התעמעמו, ולקול חצוצרות ותופים נדלקה מעל הבמה כתובת אש ענקית שהאירה את שמי הלילה ואת ההמון המריע:

    e^iπ+1=0

    ורק אותם מספרים שלא הוזמנו למסיבה, 1995 המספר המכונה רנדו, וחברתו 763 המכונה ערבה, הסתובבו והתחילו לצעוד לעבר תחום הדמדומים שבין המספרים הסופיים לאינסוף, לערבות הנצחיות בהן ניתן לחלוף על פני מיליונים רבים של מספרים שלמים עוקבים בלי לפגוש אפילו במספר ראשוני אחד. עד מהרה השתרך מאחוריהם טור הולך וגדל באופן גיאומטרי של מספרים פשוטים, מספרים אלמוניים וקשי יום, שמעולם לא הוזכרו בשום ספר, מספרים שלא היו יפים, לא מושלמים, לא ראשוניים, אפילו לא בהכרח חיוביים.
    כל התהלוכה הענקית הזו התפתלה לאיטה מול בימת הכבוד שעליה ניצבו עדיין נסיכי המתמטיקה, מבט מבועת בעיניהם, מכיוון שכאשר החלו כל המספרים הפשוטים להחלץ מציר המספרים, איבדו כל המספרים המיוחסים את התמיכה שהייתה להם תמיד מימין ומשמאל, ועד מהרה הצטמצמו כולם לנקודה סינגולרית אחת: 0.

    כי זהו טבעו של ציר המספרים: כל מספר כשלעצמו, חשוב ומיוחס ככל שיהיה, איננו אלא נקודה חסרת ממדים, אך צירוף עוקב של כולם הופך אותם לקו בעל מימד של אורך.

  395. יובל המדע מבוסס על דברים שאחרים יכולים להבין. אם אף אחד לא מבין אותך זה מכיוון שיש לך את ההיגיון הפנימי שלך, ואתה חושב אותו לנכון (באופן טבעי). החלק הפסיכולוגי הקשה במדע הוא לוותר על תקיפות אותו היגיון פנימי, כי מה שקובע זה לא ההיגיון אלא המציאות. והמציאות מוגדרת כאותו דבר שמשותף לכולם.

  396. יובל,
    אל תעשה פרצוף כזה עצוב. אני מנסה להבין. גם אם אני לא מסכים עם הרבה דברים שאתה אומר.
    למשל לגבי שורש של מינוס אחד (i) הוא נובע פשוט אם אתה מוסיף ציר מספרים אנכי לציר הממשי. כלומר אם אתה יוצר מערכת קרטזית שבה ה – X הם המספרים הממשים ככפולות של 1 ועל ה Y המספרים המדומים ככפולות של i. כל מספר ברשת מוצג ע”י שתי קורדינטות של הרכיב הממשי והרכיב המדומה שלו, לכן הוא מכונה מספר מרוכב.. אין כאן שום דבר מיוחד או מיסטי של שלילת השלילה וכו’.
    יתירה מכך אפשר לסבך ולהוסיף עוד צירים ומימדים (למרות שמעבר לציר השלישי כבר באמת קשה לדמיין מה קורה).

  397. למי שעדיין עוקב, השלמה לקטע ה”מיתולוגי” בתגובה הבאה: https://www.hayadan.org.il/astronomers-reach-new-frontiers-of-dark-matter-130112/#comment-329001
    קל היה לי להשליך מן השלילה שאנו מכירים בחיי היומיום על המנגנון של המהות שקדמה לכל ה”יש”. אולם באופן הזה אני קובע שרירותית כי ה”אין” שקדם לכל ה”יש” הינו בעל תכונות הקיימות במציאות ה”ישית” שלנו. זו הגדרה מעגלית ולכן שלילה כזאת אינה יכולה להיות הגורם הבראשיתי. בחיי היומיום איננו מכירים קָשָׁר לוגי חד מקומי אשר החלתו על עצמו יוצרת שלילה. המתמטיקאים נתקלו בבעיה דומה כאשר ניסו למצוא בין המספרים הידועים מספר שתוצאת הכפלתו בעצמו היא מספר שלילי. במהלך התפתחות החשיבה האנושית בד בבד עם העמקת הבנת דרך העולם התברר כי קיומו של מספר כזה הינו מחויב המציאות אף על פי שבמופעו הטהור הוא איננו בא לביטוי במציאות היומיומית המוכרת אינטואיטיבית. למספר הזה הוענק הכינוי, הלא כל כך מוצלח, “דמיוני”. בהשלכה מן המספרים אל הלוגיקה, מקבילו הלוגי של המספר הזה עשוי לשמש כשורש השלילה. כמו המספר “הדמיוני” של המתמטיקה גם אותו איננו מכירים אינטואיטיבית ואין לנו שימוש בו בחיי היומיום; אולם בשימושו כמקור השלילה הוא איננו השלכה מן המציאות היומיומית שלנו וכך הגדרתו את ה”יש” איננה מעגלית.
    גילוי נאות: הרעיון של שורש השלילה אינו מקורי שלי כי אם של מתמטיקאי בריטי בשם ג’ורג’ ספנסר בראון. פרטים על משנתו אפשר למצוא בספר
    LAWS OF FORM

  398. ישראל,

    תודה על הקומפלימנט.

    “או אולי M בנוסחה כבר כולל בתוכו את המסה האפלה?”

    לפי שיעורי בית שעשיתי M לא כולל את המסה האפלה, ולא את האנרגיה האפלה (קיבלתי ש-M קטן פי 16.4 מהצפיפות הקריטית).

    לשיטתי אמנם המסה היקומית כולל האפלה היא כנראה בסביבות 10% מהצפיפות הקריטית, כך שמספרית הקשר די מכוון. אבל מבחינת הפיסיקה, הקשר הזה מיותר לי. בעוד כמה עשרות מילירדי שנים, הרבה אחרי שתכבה השמש, מסת היקום הנצפה תהיה קטנה ושווה למסה הגלקטית, ועדיין G יהיה R, G יהיה R ו-C יהיה C, כלומר, הקשר יהיה חסר קשר.

  399. מאיר.
    התחלתי לעבור קצת על הכתבות שלך. ראשית, קבל קומפלימנט: אין ספק שהן ערוכות בצורה טובה בהרבה מרוב הכתבות שנתקלתי בהן במה שאני מגדיר “תאוריות אלטרנטיביות”.

    יקח לי קצת זמן לעבור על כל הפרטים, אני די עסוק וגם צריך להתעסק ברעיון שלי. אבל יש לי שאלה שנוגעת לפרק 3 שלך, בעניין האנרגיה האפלה, ולמעשה גם לכתבה זו בעניין החומר האפל.

    אם ניקח את הנוסחה של חברתינו מהבלוג, שאפשר כאמור לכתוב כ GM=RC^2, הרי שקיבלנו נוסחה יפה שמתארת קשר בין קבועים, ובינהם מסת היקום אך ל ל א מסה אפלה.

    איך זה מסתדר איפוא עם הרעיון שיש למעשה פי 5 לפחות מסה מן המקובל? לא היינו אמורים להוסיף פקטור נוסף בנוסחה כה אלגנטית? האין חוסר התאמה זה לבדו אמור לחסל את כל רעיון המסה האפלה? הרי אין כמעט ספק בערכים של G, R וC.

    או אולי M בנוסחה כבר כולל בתוכו את המסה האפלה?

    הולך לישון, נדון יותר מאוחר.

  400. מאיר
    אני חייב לטוס לעבודה. תן לי קצת זמן, יתכן שהפתרון שלך הוא בדיוק מה שצריך. אבל אתה מבין, אחרי הכל, איינשטיין… שיימה… מאך…
    תהנה מהשלג. פתיתי שלג לא קצת מקשות על החוק השני של התרמודינמיקה?

  401. עוד דבר. כששיימה טען ש-G משתנה, הוא לא אמר את זה על מנת להציל את מאך מהטענה של מאיר עמירם.
    ונשאלת השאלה, איך זה קורה שאחרי מאה שנות דישה בעקרון מאך, צריך אזרח מהשורה לגלות שבעצם כדי שהקעירות של המים תשתנה כפונקציה של המסה היקומית לפי עקרון מאך, צריך בהכרח ש G ישתנה עם המסה היקומית.

  402. חבל לטרוח. זה לא הנושא. אני יודע שדניס שיימה טוען לזה, ואני מניח שכשאתה מדבר על פורמליזם, אתה מדבר על האינטרפרטציה של שיימה לעקרון מאך.

    השאלה היא אם מאך עצמו טען את זה. ברור שלא, כי שיימה המציא את זה אחרי שמאך כבר היה ביקום חסר גרביטציה.

    הרי הכל התחיל מכך ששאלת אותי למה כתבתי שלפי מאך המים יתקערו בגלל אטום מימן במרחק עשרה מיליארד שנות אור. הסברתי למה. ואיך שלא נסתכל על זה זה מביך שמאך לא שם לב שהמים ישמרו על קעירות (שהרי הוא לא אמר ש-G משתנה. אם הוא היה רוצה לומר את זה הוא היה קורא לילד בשמו).

    אז נכון שלפי שיימה יש לו תקומה (מוזרה, ושהנסיון לגלות אותה במדידות נכשלה), אבל האם עקרון מאך ההיסטורי כפוף להיפותיזות השנויות במחלוקת של שיימה?

  403. מאמין שזה כשהמסה היקומית מרוכזת יותר. תן לי קצת זמן, אני אנסה להביא לך את כל הפורמליזם.

  404. יש איזה ציטוט של מאך שזה מה שהוא התכוון לומר (ש-G עולה ככל שהמסה היקומית קטנה יותר)?

  405. לא בהכרח. מה שהשתנה הוא G, קבוע הכבידה האוניברסלי. המסה היא עדיין אותה המסה.

  406. ישראל,

    “פשוט מאוד: לפני 5 מיליארד שנה, על אותה מהירות סיבובית היית מקבל התקערות רבה יותר. הסיבה: יותר מסה משפיעה, יותר קרוב.”

    מה בנוגע לכח הכבידה? הוא לא שחקן במשחק הזה?

    צייר לעצמך את וקטורי הכוחות הפועלים על מולקולת מים אחת שתפסה גובה בדלי.

    הרי אם היא מסתובבת באותה מהירות כמו היום, התאוצה הרדיאלית שלה היא אותה תאוצה כמו היום. אם אתה טוען שהכח הצנטריפוגלי שפועל עליה הוא נניח כפול מהיום, אזי הדבר היחיד שהשתנה הוא המסה שלה.

    אותה תאוצה, כח צנטריפוגלי כפול = מסה כפולה.

    אם המסה שלה כפולה, כח הכבידה שפועל עליה כפול (ועשיתי לך כאן הנחה).

    עכשיו תסביר מה קורה למולקולת מים הנמצאת בשיווי משקל בין שני כוחות, כשמגדילים את עוצמת הכוחות באותו יחס?

  407. נקודה, נו באמת.
    אולי תוכל כבר להתמקד באיזה שהוא נושא ספציפי?
    אם “שטחיות” מתאר שני מימדים, אז מה יתאר “נקודתיות”? 0 מימדים?
    מה עם הלינקים לאי לוקליות? העולם עוצר את נשימתו.
    אם יש לך משהו ספציפי לאמר מההתחלה ועד הסוף, זה המקום. אחרת, בבקשה להמנע מההכללות המטופשות, ולעבור לכתבות אחרות. יש כתבה פתוחה על קופים. לא נראה לך ששם מקומך הטבעי?

  408. מאיר.
    ראשית, אבהיר את עמדתי:
    1. אני לא סגור על מאך.
    2. למיטב ידיעתי, איינשטיין התלהב ממאך בהתחלה, והתאכזב ממנו בהמשך. איינשטיין טען ביחסות הכללית, למעשה כמוך, שגם ביקום ריק הדלי יסתחרר. אני לא מבין איך.
    3. אינטואיטיבית – אני מסכים עם מאך.
    4. לוגית – אם איינשטיין התכחש למאך לבסוף, חייבים להבין למה, ועלי לצאת מתוך הנחה שהטעות אצלי.
    5. אתה טוען את מה שטען איינשטיין. המים יסתחררו בדלי, לא משנה מה.
    6. ומכאן דיונינו.

    “מידת התאוצה הרדיאלית לא מושפעת מגודל המסה.”. לא יודע אם כוונתך היא למסת המים או מסת היקום. אני מניח שהתכוונת למים. לפי מאך, מידת מסת היקום היא הקובעת את כוח האינרציה על יחידת מסה של המים, כך שאין במשפט סתירה למאך.

    “אם מולקולות המים בדלי הן כמעט חסרות אינרציה (= מסת התמדה זעירה), מה ימנע מהן לטפס וליצור את ההתקערות?”

    כלום לא ימנע מהן, אך גם כלום לא יגרום להן.

    ” הן ישארו בדיוק באותו שיווי משקל בין כח המשיכה שמפעיל עליהן כדור הארץ כלפי מטה לבין הכח הצנטריפוגלי המאיץ אותן כלפי הדופן.”

    בדוגמת היקום הריק, אין כדור ארץ ואין כוח צנטריפוגלי. אם התכוונת ליקום ריק שבו רק הדלי וכדה”א, זו מערכת שונה ממה שדוסקס עד כה.

    “האם אתה רוצה לטעון את הטענה ההזויה שהמסה האינרציאלית מכווננת למסה הגרביטציונית דווקא ביקום שמסתו בדיוק כמסת היקום שלנו?”

    לא מבין את הטענה. אם אפשר, פירוט.

    “מסת ההתמדה של הערפילית שממנה נוצרנו היתה כבדה הרבה יותר מכיוון שהיא היתה חשופה למסה יקומית גדולה פי שמונה.”

    אני מניח שלפי עקרון מאך הכוח הפועל על הדלי באותם זמנים היה גדול פי שמונה. לא רואה מה הבעייה.

    ” האיש היה כל כך עסוק בשאלת האינרציה עד שהוא שכח להביא בחשבון את שאר הכוחות שאחראים על הצורה הקעורה של מים המסתובבים בדלי.”

    אילו כוחות? קוריליוס? לפי מאך לא אמורים להיות עוד כוחות.

    ” כיצד ומדוע תושפע הקונטורה של קעירות המים בדלי המסתובב על פלנט X במשך 14 מיליארד שנה ביקום היפוטתי בו קיימת דעיכה לינארית של מסה”

    פשוט מאוד: לפני 5 מיליארד שנה, על אותה מהירות סיבובית היית מקבל התקערות רבה יותר. הסיבה: יותר מסה משפיעה, יותר קרוב.

  409. ישראל אתה סתם מרחיב ומנפח דברים שנידונו ונטחנו בעבר עד דק. ואתה חושב שאתה מגלה משהו חדש. זה קצת מגוחך.

  410. ישראל,

    תבדוק את עצמך. מידת התאוצה הרדיאלית לא מושפעת מגודל המסה.

    מאך טען שאין משמעות לסיבוב של הדלי אם אין למה ליחס אותה. לצורך זה ורק לצורך זה הבאתי את אטום המימן. הנה יש לנו אטום מימן במרחק 10 מיליארד קילומטר, והנה יש לנו למה ליחס את סיבוב הדלי.

    האם דלי שמסתובב ביקום ריק (שיש בו רק צופה זניח במרחק 10 מליארד ק”מ) יתקערו מימיו או לא?
    אני טוען שהם יתקערו באותה מידה בה הם מתקערים ביקום שלנו. מאך ואתה טוענים שהם לא יתקערו.

    האם דלי שמסתובב ביקום ריק (שיש בו רק דלי מים תלוי על חצובה, וכדור ארץ אחד בודד באפלה שעליו היא מוצבת, ושהוספתי אותו מטעמים פרקטיים:שלא נכנס לשאלה יחסית למה הדלי מסתובב, על מה הוא תלוי, ולמה המים נשארים בדלי ולא מרחפים בתנאי חוסר משקל), יתקערו מימיו או לא? אני טוען שהם יתקערו באותה מידה בה הם מתקערים ביקום שלנו. מאך ואתה טוענים שהם לא יתקערו.

    שאלה:
    אם מולקולות המים בדלי הן כמעט חסרות אינרציה (= מסת התמדה זעירה), מה ימנע מהן לטפס וליצור את ההתקערות? באותה מידה בה הן איבדו מסת התמדה (כתוצאה מאבדן השפעת המסה היקומית) הן גם איבדו מסה גרביטציונית. הכח הגרביטציוני הפועל עליהן קטן באותה מידה בה קטן הכח הצנטריפוגלי. הן ישארו בדיוק באותו שיווי משקל בין כח המשיכה שמפעיל עליהן כדור הארץ כלפי מטה לבין הכח הצנטריפוגלי המאיץ אותן כלפי הדופן.

    האם אתה רוצה לטעון את הטענה ההזויה שהמסה האינרציאלית מכווננת למסה הגרביטציונית דווקא ביקום שמסתו בדיוק כמסת היקום שלנו?

    אגב, שים לב שלפי טענתך (וכאדם ההולך בתלם שהתווה אלברט) חוקי הפיסיקה הולכים ומשתנים עם הזמן, שהרי המסה של היקום הנצפה הולכת וקטנה. בכל יום של התפשטות היקום, אנו מאבדים גלקסיות יקרות. לפני 7 מליארד שנים תכולת המסה של היקום הנצפה היתה בערך פי שמונה מתכולתה היום, כלומר מסת ההתמדה של הערפילית שממנה נוצרנו היתה כבדה הרבה יותר מכיוון שהיא היתה חשופה למסה יקומית גדולה פי שמונה.

    את עקרון מאך אני יכול להוציא בכבוד יחסי מהפלונטר הזה, אבל אני לא יכול להציל אותו מהטעות המביכה ביחס לדלי. האיש היה כל כך עסוק בשאלת האינרציה עד שהוא שכח להביא בחשבון את שאר הכוחות שאחראים על הצורה הקעורה של מים המסתובבים בדלי.

    אם אתה חושב שזו לא טעות מביכה שלו, אל תשלח אותי לדיסקיות מסתובבות (שכבר דיסקסנו והותר), אלא תסביר לי עם כושר ההסבר הרגיל והמשובח שלך, כיצד ומדוע תושפע הקונטורה של קעירות המים בדלי המסתובב על פלנט X במשך 14 מיליארד שנה ביקום היפוטתי בו קיימת דעיכה לינארית של מסה.

  411. מאיר
    לפי עקרון מאך, אם הדלי נמצא לבדו ביקום ריק המים לא יתקערו, לא משנה באיזו מהירות מסתובב הדלי. זה שהוספת פרוטון בודד איפה שהוא ישפיע באופן כה זניח, שפשוט אפשר להתעלם מההשפעה.
    אם תוסיף את כל המסה שקיימת היום ביקום בדיוק באותו מקום שהיא נמצאת אך מחולקת ב2 – מידת ההתקערות תקטן ביחס המתאים. תשלש את כמות המסה – תגדל ביחס המתאים. (לא פי 3, אפשר לחשב בדיוק כמה).

    זו בדיוק השאלה האוריגינלית שלי: אם 2 דסקיות מסתובבות אחת יחסית לשניה – מיהי המסתובבת האמיתית – יחסית למה?
    אליבא ד’מאך – יחסית לחומר ביקום.

  412. ישראל,

    “אני רק שאלה: עברתי קצת על הבלוג שלך. למה נראה לך שלפי עקרון מאך (השגוי מבחינתך) אם יש ביקום ריק פרוטון בודד המקיף את הדלי במרחק מליוני קילומטרים ממנו, יתקערו המים בדלי? תיאורטית כן, אך במידה כה אפסית שאין כאן שום סתירה לעקרון עצמו”

    למה במידה אפסית?
    תאוצה רדיאלית היא תמיד Vsquared/R, ללא תלות במסה (לענייננו מסת ההתמדה). לכן הקטנת מסת ההתמדה לא תשנה את מידת התקערות המים בדלי.

    ניסוי מחשבתי: בהינתן מהירות זויתית מסויימת כלשהי, האם הקעירות של כספית המסתובבת בדלי תהיה שונה מהקעירות של iso-pentane או של מים או של כל נוזל אחר (בהתחשב במסה, ובהתעלם מהבדלים הנובעים מצמיגות שונה)?

    כמובן שלא, מכיוון שככל שהמסה קטנה יותר כך היא קלה יותר להסטה מהמרכז ולהיערמות בהיקף.

  413. יובל

    משקפיים ולוגיקה: אתה לא יכול להגדיר מילה שמכילה את המילה המוגדרת.

  414. ר.ח רפאי.ם,
    השאלה אם אני מתבייש או לא איננה רלוונטית. אם כל מה שאתה רואה בדברים שלי הוא חומר גלם לדאחקות, אני יכול רק להצטער עבורך. אינני טוען בוודאות מוחלטת שהיקום בנוי מן החלקיקים שאני מציג כאן. כל מה שאני מתיימר לעשות זה להראות כיצד באמצעים פשוטים אפשר לבנות משהו שהוא כמו היקום שאנחנו מכירים. אולי החלקיקים שאני מדבר עליהם הם בדיוק החלקיקים מהם בנוי היקום ואולי לא. אבל זו לא הפואנטה.

  415. התכווצות לורנץ ועידוש כבידתי:
    הניידות של החלקיקים תלויה תלות הפוכה בצפיפות בה הם מסודרים, ובהתאם לכך גם הניידות של החורים. ככל שגדלה הצפיפות בה מסודרים החלקיקים, קטן נפחם של החורים שביניהם וקטנה ניידותם. זה מתבטא, בין היתר, בהקטנת מהירות התקדמותם של החורים. נסתכל שוב על פרוטון. ככל שמהירות תנועתו גדלה, גדל גם היחס בין צפיפות החלקיקים שלפניו לבין צפיפות החלקיקים שמאחוריו. תנועתו של חור (או גל של חורים) הנקלע לסביבה צפופה תהיה אטית יותר מתנועתו בסביבה פחות צפופה.
    אם נדמה את הפוטונים של הפיסיקה לחורים של המודל נראה כי מהירות האור משתנה בהתאם לצפיפות הסביבה בה הוא נע. משום כך, מהירות האור כלפי גוף הנמצא בתנועה זהה למהירותו כלפי גוף הנמצא במנוחה.
    פרוטון מחזיק בסביבתו הקרובה חלקיקים בצפיפות הגבוהה יותר מן הצפיפות בסביבה הרחוקה ממנו. הסביבה הצפופה מזמנת יותר חלקיקים מאשר הסביבה הדלילה ולכן מושכת אליה יותר חורים. כך, הסיכוי של חור הנע בקרבת פרוטון לנוע בכיוון אל הפרוטון הינו גדול יותר מאשר סיכויו לנוע בכיוון ההפוך.
    גם כאן, אם נדמה את הפוטונים של הפיסיקה לחורים של המודל נראה כי מה שהפיסיקה מכנה “עיקום המרחב” אינו אלא שינוי צפיפות חלקיקי החומר האפל בהתאם לקרבתם אל הגרם השמימי.

  416. יובל!!

    התחלת תגובתך: “עוד על חלקיקים וחלל ריק:
    המונח “פיסת חלל ריק” לא הוגדר עד כה בפירוט: זהו אזור רצוף של חלל ריק…”

    תגיד לי, אתה לא מתבייש?
    אתה רוצה שנמשיך להריץ עליך דאחקות?

  417. נקודה, תודה על העצה.
    באמת אני זקוק למודל ממוחשב שיעזור לי להמחיש את הדברים ואולי גם לפתור לי כמה שאלות פתוחות. אני מניח שבמילה “נאיבי” אתה מתכוון למה שאני מביא כאן. זה נכון, בעיקר בגלל האופן הנאיבי בו אני מביא את הדברים ללא נוסחאות מתמטיות, לעת עתה. גם כשיגיע תורן של הנוסחאות המתמטיות, הן לא יהיו מסובכות. הצורך במתמטיקה אוניברסיטאית (מטריצות, וקטורים, משוואות דיפרנציאליות וכו’) מתעורר כאשר מחברים מספר רב של מבנים בודדים אלא לאלה. אך כעת אני עדיין ברמה של המבנים הבודדים.

  418. יובל, תלמד OpenGL תוכל להעלות את היקום שלך על מחשב ולהתפאר מיצירתך.
    היקום הנאיבי לא קשור לפיסיקה של היקום שלנו.

  419. עוד על חלקיקים וחלל ריק:
    המונח “פיסת חלל ריק” לא הוגדר עד כה בפירוט: זהו אזור רצוף של חלל ריק שחלקיק צפיד יכול לנוע לתוכו בחופשיות. באופן הזה ניתן להסתכל גם על החלל הריק כאילו הוא מורכב ממעין חלקיקים בדידים ולא רק כ”ענן”. להלן, לצורך קיצור, נכנה את פיסות החלל הריק בשם “חורים” ואת החלקיקים הצפידים “חלקיקים”. נביט כעת על אופן תנועתו של חור: כאשר באזור מסוים נוצר חור (למשל בעקבות הפיכתו של חלקיק לחלל ריק בגלל צפיפות גבוהה) חלקיקים מן הסביבה יכולים לנוע לתוכו. כאשר חלקיק נע אל תוך חור, הוא מפנה חלל ריק בכיוון הנגדי ומביא ליצירת חור במקום חדש. גם אל החור החדש הזה יכולים חלקיקים להיכנס והחור עובר הלאה. כך יכול חור לנוע מרחקים גדולים בעוד שכל אחד מן החלקיקים שתרם לתנועתו נע רק מרחק קצר.
    כעת נראה כיצד יש לחלל הריק התנהגות דואלית:
    אם קיים אזור המייצר חורים בהתמדה, שלהלן נקרא לו “מוקד” (ובהמשך נראה כיצד יכול דבר כזה להתקיים), הרי החורים יתפזרו ללא העדפת כיוון מן המוקד החוצה. כל חור הוא יחידה בודדת, אבל ביחד אוסף החורים יוצר חזית של גל.

  420. ר.ח.
    קראתי מחדש את תגובת הדג האוריגינלית. אין שם התייחסות לאור. יש התייחסות לדג. דג זה לא אור. דג זה דג. הרבה אנשים יחשבו שכשכותבים דג מתכוונים לדג. קוראי מחשבות יוכלו לדעת שכשכותבים דג מתכוונים לאור. אני לא קורא מחשבות. אני מתייחס רק למה שכתוב. היה כתוב דג. התייחסתי לדג. אילו היה כתוב “תקרא שוב את מה שכתבתי ובמקום דג תקרא אור ואז תחווה דעתך” כמו בתגובתך האחרונה, הייתי עושה זאת. חשדתי שלזאת התכוונת, אך לא הייתי בטוח. לכן התייחסתי רק לדג. לדג.

    עניינית.
    נראה לי שעברת מניסוי מ-מ למסקנתו של איינשטיין. זהו נושא חדש, שנוכל להגיע אליו בהמשך. אולם אני חולק על קביעתך ש” ניסוי מ-מ רצה לבדוק את מהירות הדג (האור) ביחס למצוף (כדור הארץ) ומה שמצא הוא שלא משנה איך ומהיכן תמדוד, הדג שוחה באותה מהירות.” זה לא מדוייק, אפילו עד מאוד. את תוצאות ניסוי מ-מ ניתן להסביר בקלות עיי ההנחה שהאור נע במהירות אחת ויחידה (מהירות האור) ביחס למקור האור, כמו כדור רובה הנע במהירות אחת ויחידה ביחס לרובה. אם אינך מאמין, עבור על הניסוי, תראה שזה די ברור.

    לכן גם רבים אכן האמינו ב Emission theory ע”ע, שהייתה תאוריה מתחרה ליחסות, ולפיה האור נע במהירות קבועה יחסית למקור. אולם יש בעיות רבות עם תאוריה זו כפי שתוכל לקרוא אצל ויקי (כנפו). בכל אופן – לא קבלתי עדיין תשובה לשאלת תקפותו הלוגית של ניסוי מ-מ.

    אם תרצה, נוכל כרגע להשאיר שאלה זו פתוחה ולעבור על מסקנתו של איינשטיין, מדוע אני רואה בה בעייה, וכיצד הרעיון שלי יכול לעזור לפתור אותה.

    ובעניין ניסוי דורותי/אישפח, הרי מן המפורסמות היא שהדחליל היה מכריז תמיד “ועכשיו כלי זמר בחיצרוץ, בשרו בואו של מלך עוץ!” לפני כניסתו של איוב הג’ובניק.

    מאיר.
    אני רק שאלה: עברתי קצת על הבלוג שלך. למה נראה לך שלפי עקרון מאך (השגוי מבחינתך) אם יש ביקום ריק פרוטון בודד המקיף את הדלי במרחק מליוני קילומטרים ממנו, יתקערו המים בדלי? תיאורטית כן, אך במידה כה אפסית שאין כאן שום סתירה לעקרון עצמו

    יובל…
    SIEMPRE PROBLEMAS CONTIGO!

    אינני חושד בכלום. פשוט, כתבת שרבים הבינו כיצד מתקיימת גרביטציה במודל שלך גם ללא התגובה שציינתי, ושאי ההבנה היא רק אצל ישראל הדביל, כמו תמיד. מכיוון שלדעתי לא ניתן להבין איך יש גרביטציה במודלך ללא תגובה זו, הסקתי ששלחת אותה ליודעי חן בלבד. עכשיו, אני יודע את התשובה: אף אחד כנראה לא קיבל, כיוון שאף אחד לא הבין.

    אבל עזוב שטויות, אחינו. נראה לי שאני מתחיל להבין לאן אתה חותר, אך מכיוון שהרופא נתן לי פטור, אני מתכוון לנצל אותו עד הסוף.
    אך ראה הוזהרת: נראה לי שהמודל, לפחות מבחינת הגרביטציה, הוא רק לסאז’ בתחפושת.
    אולי עדיף לא לבטל את הפטור…

  421. סטודנט, טכניון (וכל שאר חסרי הסבלנות),
    צר לי על כך שאינני מספק את הביקוש בקצב המתאים לכם. אני מתמודד עם שחזור חומר ממעבדי תמלילים פרה-היסטוריים והגהתו. בנוסף, אני מאד עסוק במשך היום.

  422. ישראל,

    אני מצטער אבל אתה לא הבנת את משל הדג. או שלא הייתי ברור מספיק או שקראת בחופזה. בסוף אני אחשוב כמו יובל שאתה לא באמת מתייחס לתגובות אלייך.
    תקרא שוב את מה שכתבתי ובמקום דג תקרא אור ואז תחווה דעתך.
    אתה גם כל הזמן חוזר “האוקיינוס יחסית לארץ”. תשכח מהארץ, במשל הנ”ל אין ארץ. האוקיינוס הוא החלל מלא מים (אתר). ניסוי מ-מ רצה לבדוק את מהירות הדג (האור) ביחס למצוף (כדור הארץ) ומה שמצא הוא שלא משנה איך ומהיכן תמדוד, הדג שוחה באותה מהירות. מכאן מסקנתו של איינשטיין שהתקבלה על כלל קהילת הפיזיקאים היא שמהירות האור קבועה ולא שבמהירות גבוהה הדג הפך לשקוף למכשירי המדידה. אני לא בטוח איזו מן המסקנות מוזרה יותר אולם לעניות הבנתי בנושא הרבה מאד תחזיות של תאוריית היחסות התממשו עד היום. רגע, שנייה, אני אחבר את ה GPS, שנייה, אוי, עבד! עוד ניסוי שאישש את התחזיות של תורת היחסות עבר בהצלחה.

    אהבתי את הדיר בלק באנגלית 🙂 ולגבי איוב אני לא מבין מהיכן הם הביאו שהיה יועץ לפרעה, מקסימום היה יועץ של דורותי.

  423. יובל, בקיצור, נכשלת בהסברים שלך ובלגוקיה של ההסברים. מצד אחד לא התחלת מההתחלה (הזמן והמרחב אצלך הם נתונים ללא פשר והסבר מדוע בחרת 3 מימדים של מרחב), ומצד שני עדיין לא הבאת שום מסקנה.

    ה”עולם” שאתה מנסה לבנות מתאים יותר לאותם עולמות וירטואליים הקיימים במשחקי מחשב, הם מורכבים מחלקיקים צפידים (המשלושים) שנעים בתוך מרחב ריק (המרחב בין המשולשים) וככה אנו מקבלים הצגה של עולם תלת מימדי על מסך המחשב שלנו.
    ואין בין זה לפיסיקה, שום קשר. מדובר בדימיון בלבד.

    חיפשתי אולי יש לך אילו שהם עקרונות פילוסופיים מענינים, התחלת עם “היש נוצר מהאין”, שזו אמירה שמכילה סתירה. לא הסברת אותה, סתם רצתי כאילו להסביר הכל, אבל לא הסברת כלום, רק קבעת שזה כך. לא הסברת איך מתוך האין נוצר דוקא יש שכזה ולא יש אחר.. בקיצור נכשלת בניסיונות שלך.

  424. יובל,

    נו, מתי נראה את החיבור לפיזיקה קוונטית? את החישובים לאורך הקשר הכימי, המשוואות הנחשקות והקישור לספקטרוסקופיה? האם נזכה להסברים על NMR, ESR ועיבוי בוז אינשטיין? להזכירך, בשביל להיכנס לכתב עת מדעי תצטרך יותר מסיפור בדיוני על חלקיקים צפידים, חללים ריקים, אלקטרונים של שכבה דקה ופוטונים עם קוטר.

  425. ישראל!
    האם אתה מנסה לרמוז שאתה יודע (ולא רק סתם חושד) שאני מסתיר משהו? אם כן, אנא חשוף את מקורותיך. ואם אין, אז בבקשה שמור את חשדותיך לעצמך.
    בוקר טוב לעיר המלאכים

  426. נקודה,
    התהיה שלך על מרחב וזמן היא אכן במקום. לא הבאתי את הנימוקים לקיומו של מרחב תלת-מימדי הדיר (שאפשר לנוע בו הלוך ושוב) ולמרחב זמן חד-מימדי חד-כיווני. הדברים האלה שייכים לשלב קדום יותר במודל שהעדפתי לדלג עליו כאן.
    נכון גם שאני בונה דגם של העולם מ”אבני לגו”.
    אתה צודק גם בהערתך הכללית יותר שאוסף העובדות שהבאתי עד כה אינו מוכיח דבר. הבעיה היא שאני כותב לאט ועדיין לא הגעתי לדברים ה”דרמטיים”.
    ואם ירצה אתר הידען למחוק את דבריי, לא אכהה בעדו. אולם נושא המאמר הנוכחי הוא החומר האפל, והמודל שלי מתייחס בדיוק לזה.

  427. נקודה.
    אינך עונה לשאלה ששאלתי. עזוב כרגע עם השאלה נכונה או לא.

    אם עקבת אחרי הדיון פה, לדעתי מודל האתר דווקא כן נכון.

    לילה טוב.

  428. יובל, אוסף ה”עובדות” שאתה טוען אינם מסבירות דבר. בכלל, באופן בלתי מודע זה נראה שאתה מניח “מרחב” ו”זמן” כאובייקטים שקיימים, לא ציינת ממה מורכב אותו חלל ריק וכו’. בקיצור אתה סתם מנסה לבנות מחלקי לגו פשוטים עולם שאין קשר בינו לבין הייקום שלנו.

    ניסיונות כאלו לא קשורים לאתר הידען.

  429. ומה עם תגובה זו

    https://www.hayadan.org.il/astronomers-reach-new-frontiers-of-dark-matter-130112/#comment-329304

    האם לא חלקת אותה עם אף אחד לפני שפרסמת באתר?

    ואיך אתה יודע מה אני עושה עם חיי? לא שאני יודע, אבל איך אתה?
    לילה טוב יובלי. עדיין לא קיבלתי בקשה מפורשת. בינתיים, אנצל את הפטור ואצא לאפטר.

  430. ישראל, אני לא מבין לאן אתה חותר. אני לא מבין מה אתה לא מבין.

    מבחינה היסטורית פעם חשבו שאנו נעים בתוך האתר. הרעיון הוא שהכוכבים סטאטיים (זה לכשעצמו לא יכול להיות נכון בגלל כוח המשיכה) בתוך האתר, ומערכת הייחוס הם הכוכבים הסטאטיים (כמו השמש).

    כל זה התברר כלא נכון.

    מה יש לך לחפור בדבר שהתברר כלא נכון, היו הרבה מאוד בעיות שהיו קימות אז בתפיסות וגם היו מודעים אליהם וניסו למצוא את הפיתרון הכי סביר לפי הידע שהיה אז.

  431. ישראל,
    אתה מוכשר דייך לכתוב ספרים מתוצרת עצמית. הלא מוטב שתחדול מלצטט ולתת פרשנויות מגוחכות ותתחיל לעשות משהו מועיל ומשמעותי בחיים שלך?

  432. יובל.
    כזכור, עליך ליידע אותי אם אתה מעוניין בתגובות ממני בקשר למודל.

    איך היו הציטוטים מאיוב? ספר גדול, אה? החברה של הקבלה טוענים שהוא היה יועץ של פרעה.

    ואיך אהבת את הדרש על פרשת בלק? שזו הייתה בסך הכל אי הבנה, שמשה ויהושע המסכנים פשוט לא דברו מואבית טובה, לכן שלחו לבלק מכתב באנגלית. ההוא פתח את המכתב, קרא את השורה הראשונה: דיר בלק! תפס חופה ושלח להם את בלעם, שילמדו לקח ולא יעלו עוד פעם על הדשא עם כל העם וילחכו אותו כלחוך השור.

  433. רפאים
    תגובתך אכן הייתה לעניין, ואם תתחייב להמשיך להגיב לעניין, לא לגלוש לפסים אישיים, ואם קוראים אותך לסדר על שעשית זאת להתנצל ולהפסיק, נוכל להמשיך. ע”ע תגובותיהם של מאיר ואריה סתר.

    אני מתחייב לעשות אותו הדבר.

    אבל עליך להתחייב. בפירוש.

  434. ישראל
    התגובה שלי היתה לעניין.
    אתה לא פעם כתבת משהו בסגנון: ” אך בכל הכנות, אני מאמין שזו בעיה אצלי…”.
    ואני מתייחס עניינית לבעיה הזאת שלך, ועונה לך שהבעיה היא באמת אצלך. לא ברעיון שלך. הרעיון שלך מופרך. הבעיה היא אצלך והיא שאתה לא מבין שהרעיון שלך מופרך.
    וכל הנסיונות הם לעזור לך להבין זאת.

    אתה כותב:
    “לפי תמונת העולם ב1887, היקום הוא אינסופי, הומוגני ואיזוטרופי. האם יכולה ביקום כזה בכלל להיות מערכת מנוחה ל”אוקיינוס” האתר?…”

    וכולנו שואלים אותך:

    מר. ישראל שפירא, מקריית ענבים, מדוע לא רשמת: “לפי תמונת העולם ב2012, היקום הוא…”?
    ו- “האם יכולה ביקום כזה בכלל להיות מערכת מנוחה ל…”?

  435. ר.ח.

    התגובה אליך ממתינה לאישור.

    רפאים (אני קורא לך כך כי זה פשוט יותר לכתוב).

    כפי שציינתי כמה פעמים, אם תרצה שאגיב לתגובותיך, עליך להתחייב להשאר בתחום הענייניות בלבד, ולא להכנס למניירות אישיות.
    אני ויובל כבר התחייבנו. זה תלוי בך.

    נקודה.

    בחודשים האחרונים עסקנו בכתבה זו, וגם באחרות, בנסיונות להסביר את אותו דבר מוזר שהתייחסת אליו.

    אולם כדי שנבין מהו אותו דבר מוזר, נסה לראות אם אתה יכול לענות על השאלה שהצגתי:

    לפי תמונת העולם ב1887, היקום הוא אינסופי, הומוגני ואיזוטרופי. האם יכולה ביקום כזה בכלל להיות מערכת מנוחה ל”אוקיינוס” האתר? האין מציאת מערכת מנוחה כזו מפרה את הנחת ההומוגניות והאיזוטרופיות? אילו נמצאה המערכת, כלום לא היינו מחוייבים לשאול: מדוע דווקא זו? האין זה משול למציאת מרכזו של ישר אינסופי?

  436. ישראל, מה זה משנה למה נועד ניסוי מייקלסון מורלי, כוונות הניסוי אינן חשובות. מה שחשוב הוא הניסוי עצמו.
    והניסוי הראה שמהירות האור בשני כיוונים ניצבים הם זהות. ולכן או שכדור הארץ נח, או שמשהו מוזר קורה. מוזר ברמות שרק מישהו כמו איינשטיין יוכל לפענח.

  437. “נוכל לחזור על כל הרעיון של הניגוד לכאורה עם המפץ הגדול.” – מי נגד מי, לא הבנתי?

  438. מאיר.

    לסאז’ מבחינתי הוא הצגת משנה. זה יותר התחום של יהודה הנעלם.
    קצת קשה לוותר על היחסות, לא? פצצות גרעיניות, עידוש, ניסויי מטוסים עם התארכות זמנים, GPS, מואונים?

    הבעייה שלי עם היחסות היא שונה, אם מישהו יהיה מעוניין, נוכל לחזור על כל הרעיון של הניגוד לכאורה עם המפץ הגדול. אך בכל הכנות, אני מאמין שזו בעיה אצלי ולא ביחסות.

    תן לי קצת זמן להתעמק במודל שלך. אצלך לפחות, הכל מאורגן יחסית, לא בלגן כמונו.

  439. יובל.
    זכור שהמודל ערוך ומסודר רק בראשך. לנו, כמגיבים, אין אפשרות לדעת מה אתה חושב. לכן אם אתה משחרר אותו טיפין טיפין, עלינו לעבור על כל השירשור כדי לנסות להבין לאיזו תגובה מסויימת משוייך נושא מסויים.

    לדוגמה: אני יודע בדיוק איפה למצוא את התשובה לשאלה האחרונה של ר.ח., כי אני שולט במודל שלי. האם תוכל אתה? סביר שלא.

    לעומת זאת, אם הכל היה ערוך ומסודר בבלוג, בתגובה, או במקבץ תגובות מסויים, היינו יכולים תמיד לגשת לשם ולדעת במה דברים אמורים.

    את הכתוב בתגובתך זו:

    https://www.hayadan.org.il/astronomers-reach-new-frontiers-of-dark-matter-130112/#comment-329304

    ראיתי לראשונה באתר לפני 3 ימים. יש לי הרגשה שאחרים קבלו זאת קודם.

    ר.ח.
    צללת פעם בים הפתוח?
    אתה בתוך כחול עמוק, 30 מטר מתחת לפני הים, אין ארץ ואין שמיים רק אוקיינוס. לכל צורך ועניין הוא אינסופי. אם לא עשית תדמיין.
    דג שוחה לבד. ביחס למה???? ביחס לאוקיינוס עצמו. אם נסמן נקודה במים, נניח מוליקולת מים רדיואקטיבית נדע בדיוק את תנועת הדג. (בסדר, אבל מהירות האוקיינוס עצמו היא 0 יחסית לארץ. היא די גבוהה יחסית לשמש, או כוכב שביט חולף).
    עכשיו אתה צוללן. אתה מסתכל על דג שהולך ומתרחק ממך ומודד את מהירותו.
    אתה מודד את מהירותו ביחס לבלון שצף במים ואתה מגלה שהדג שוחה באותה מהירות שמדדת קודם. (נכון. כי הבלון במהירות 0 יחסית לאוקיינוס).
    עכשיו אתה צוללן ששוחה מהר, אתה מודד את מהירותו של הדג ולהפתעתך הוא עדיין שוחה באותה מהירות. (לא מדוייק. הדג שוחה באותה מהירות יחסית לאוקיינוס, אבל לא יחסית אלי, הצוללן. אם מהירות הדג 10 מ/ש יחסית למים, ומהירותי 1000 מ/ש יחסית למים בכיוון ההפוך, מהירותי יחסית לדג 1010 מ/ש).
    אתה עולה על אופנוע תת ימי מודד את מהירות הדג ועדיין הוא שוחה באותה מהירות. (כנ”ל).
    עכשיו אתה שם את כלאבי שלך על מצוף תת ימי עם מד מהירות ואתה שוחה נורא מהר, שניכם מודדים את מהירות הדג בו שמנית ולהפתעתך בשתי המדידות הוא שוחה באותה מהירות! (אותו כנ”ל. עם כלאבי זה קצת שונה, כי כלאבי יאכל את הדג).
    יתירה מכך הדג התרחק ממך ונע לצידו של כלאבי, אותה מהירות!

    מה מסקנתך?

    1) שהדג תמיד שוחה באותה מהירות ומהירותו לא יחסית לכלום? (מהירות הדג תמיד אותה מהירות יחסית לאוקיינוס. אם לאוקיינוס עצמו מהירות יחסית לשמש, אזי מהירות הדג יחסית לשמש היא הסכום הווקטורי של שתי המהירויות: מהירות הדג יחסית לאוקיינוס + מהירות האוקיינוס יחסית לשמש.

    מהירות האור, להבדיל, תמיד תהיה אותו הדבר, לא משנה מה מהירות הדג או האוקיינוס, עבור כל מודד).

    2) שמוליקולות המים נעות במהירויות שונות מ-0 לאינסוף והדג שלנו נסחף בהן. למעשה הוא נע בכל המהירויות מ – 0 ועד אינסוף אולם מסיבה עלומה כלשהיא הוא שקוף למכשירי המדידה שלנו מעל ומתחת למהירות הנמדדת ולכן נראה לנו שהוא במהירות קבועה. ( למה? המהירות הממוצעת של מולקולות המים יחסית לארץ היא בסדר גודל של כמה קמ/שנ. אולם הן מקזזות זו את זו, כך שהסכום הווקטורי של כולן הוא 0 יחסית לאוקיינוס. לדג יש מהירות יחסית לאוקיינוס, שהיא וקטור נוסף. אין פה שום שאלה של אינסוף).

    בוא נעבור רגע לאויר. המולקולות נמצאות במהירות ממוצעת של כ480 מ/שנ יחסית לארץ, אך האויר עצמו במהירות 0 יחסית לארץ. אם יש רוח, זהו וקטור נוסף. אם יש ציפור שעפה, זהו עוד וקטור.

    אך מהי מהירותו של אוקיינוס האויר עצמו? היא שונה לכל מודד. לגבי הארץ (אם אין רוח) 0. לגבי אווירון, מהירות האווירון. לגבי השמש, מהירות השמש.

    אבל ניסוי מ-מ ניסה למצוא את מהירות “אוקיינוס” האתר יחסית לארץ. אילו מצא אותה, איפה ההומוגניות? למה דווקא זו? במה היא עדיפה על כל מהירות אחרת?

    השקיפות איננה הזיה פרועה. זה דבר די מקובל אצל חלקיקי יסוד מהירים. ע”ע אורניום 235 וחדירותו לנייטרונים, נייטרינו, וטכיונים (שבמידה והם קיימים, אי אפשר לגלות אותם בדיוק מהסיבה הנדונה: שהם מהירים מהאור).

  440. ישראל,
    הבשורה המשמחת היא שכתבתי תגובה. הבשורה השניה היא שהתגובה ממתינה לאישור.

    השורה התחתונה הייתה:

    “בקיצור, זה לדעתי לא במקום למדוד את MCS לפי הקריטריונים של שיימה, אבל זכותך : ) ”

    ליל מנוחה מירושלים

  441. ישראל,

    “אם אתה מקבל את רעיון היקום האינסופי, ההומוגני והאיזוטרופי”
    אני מקבל את הרעיון רק לטווח מספיק גדול כדי לא להפריע לעשות את הפיזיקה המדידה. מה איכפת לי מה שמעבר?

    “נקודת המוצא של כל הרעיון הוא המטוטלת הבליסטית – שמעל מהירות מסויימת, הרבה פחות מאינסוף, כל החלקיקים נהפכים לשקופים” אני לא חוזר בי מההצהרה שהרעיון גאוני. בכל זאת, בעוכריו שהוא נולד על מנת לתרץ מכניזם (לה סאז’) שצורך משאבים רבים מדי ונותן תפוקה קטנה מדי. איך שלא יהיה, לא הבנתי מה הקשר לשאלה ששאלתי: החיכוך עליו דיברתי בין מערכת א’ למערכת ב’ הוא וירטואלי. פשוט שאלתי מה מפריע לשתי מערכות היפותתיות אין סופיות להיות בתנועה יחסית?

    “הנקודה העיקרית היא השאלה אם אתה מקבל או שולל את מה שנכתב לגבי איינשטיין:
    אם אינך מקבל”

    איני מקבל.

    “אשתדל לעבור על כל ההוכחה של שימה (אין לינקים, זה ספר מלפני כמעט 50 שנה) כדי לראות איך זה מסתדר עם התאורייה שלך.”

    שאתה, ישראל, תסתמך על תורת היחסות כדי לבחון את הבייבי שלי?! למה אתה צריך הסכמה מתאוריות אחרות על מנת לשפוט תאוריה חדשה? אני מראש אומר לך שזה לא מסתדר עם התאוריה שלי. הרעיון של שיימה מבוסס בין היתר על השפעת התנועות העתידיות של המסות ביקום על ההתמדה של מסה כאן ועכשיו. מתאים לתורת היחסות ולהתייחסות אל זמן כאל מימד שאפשר להשתעשע בו. בעיני: סיפורי פיות.

    המבחן של תאוריה הוא ההתאמה שלה למציאות, למדידות, ולמגוון התופעות שהיא מתיימרת להסביר. על מנת לבחון את ההתאמה הזו, צריך לבחון את המציאות מתוך הנחות היסוד של התאוריה הנבדקת, ולא מתוך הנחות היסוד של התאוריות שאותן היא באה להחליף. אני טוען שתאורית הכבידה שלי מספקת יותר התאמות למציאות מתורת היחסות. ממה נובעת ההצלחה של תורת היחסות? מההנחה הנוחה והשגויה ש”צופה במנוחה” הוא מציאות פיזיקלית. ההנחה הזו מאפשרת להתייחס אל מכשיר המעבדה כאל צופה במנוחה, ולטעון שהוא מתקדם בזמן מכיון שהוא נח במרחב, בהשוואה לקרן אור ש”מתקדמת במלוא המהירות האפשרית במרחב”, ולכן נחה בזמן. הבעיה היא שסטטיסטית ה fairy-tale הזה המתורגם לטרנספורמציות ומשוואות הוא נכון, ופיזיקלית הוא שטות גמורה. לכן, כמו סטטיסטיקה, זה ברוב המקרים עובד לא רע במספרים גדולים ובמרחקים לא יותר מדי קצרים, ונופל באינדיבידואל.

    כפי שאתה שם לב, נתתי את הקרדיט לתורת היחסות, אבל רק את אותה כמות קרדיט שהיא ראויה לה.

    במאמר על אינרציה כתבתי את סוג הניסויים והתצפיות שיכולים להבחין בין התחזיות של תורת היחסות לבין התחזיות של פיזיקת MCS בכל הנוגע לאינרציה. חלקם בוצעו בפועל ותוצאותיהם מוגדרות כאנומליות על פי תורת היחסות.

    בנוסף לזה, ישנה תופעה מסויימת שבה פיזיקת MCS מנבאת תוצאה הפוכה לניבויי תורת היחסות (כלומר בעוד לפי תורת היחסות ההשתנות של ערך ניסויי מסויים הוא פונקציה ישרה של ההשתנות בערך אחר, לפי פיזיקת MCS הערך הניסויי הזה ישתנה כפונקציה הופכית של הערך האחר), מה שנקרא “מבחן יום הדין”. ניסוי קל לביצוע, ועוד ידובר בזה.

    בקיצור, זה לדעתי לא במקום למדוד את MCS לפי הקריטריונים של שיימה, אבל זכותך : )

  442. ישראל,

    צללת פעם בים הפתוח?
    אתה בתוך כחול עמוק, 30 מטר מתחת לפני הים, אין ארץ ואין שמיים רק אוקיינוס. לכל צורך ועניין הוא אינסופי. אם לא עשית תדמיין.
    דג שוחה לבד. ביחס למה???? ביחס לאוקיינוס עצמו. אם נסמן נקודה במים, נניח מוליקולת מים רדיואקטיבית נדע בדיוק את תנועת הדג.
    עכשיו אתה צוללן. אתה מסתכל על דג שהולך ומתרחק ממך ומודד את מהירותו.
    אתה מודד את מהירותו ביחס לבלון שצף במים ואתה מגלה שהדג שוחה באותה מהירות שמדדת קודם.
    עכשיו אתה צוללן ששוחה מהר, אתה מודד את מהירותו של הדג ולהפתעתך הוא עדיין שוחה באותה מהירות.
    אתה עולה על אופנוע תת ימי מודד את מהירות הדג ועדיין הוא שוחה באותה מהירות.
    עכשיו אתה שם את כלאבי שלך על מצוף תת ימי עם מד מהירות ואתה שוחה נורא מהר, שניכם מודדים את מהירות הדג בו שמנית ולהפתעתך בשתי המדידות הוא שוחה באותה מהירות!
    יתירה מכך הדג התרחק ממך ונע לצידו של כלאבי, אותה מהירות!

    מה מסקנתך?
    1) שהדג תמיד שוחה באותה מהירות ומהירותו לא יחסית לכלום?
    2) שמוליקולות המים נעות במהירויות שונות מ-0 לאינסוף והדג שלנו נסחף בהן. למעשה הוא נע בכל המהירויות מ – 0 ועד אינסוף אולם מסיבה עלומה כלשהיא הוא שקוף למכשירי המדידה שלנו מעל ומתחת למהירות הנמדדת ולכן נראה לנו שהוא במהירות קבועה.

    בחר את התשובה הסבירה לדעתך.

  443. ישראל,
    אני מקוה שהאופן בו אני משחרר כעת את המודל טיפין טיפין, מסך אחד בכל פעם, עוזר להבנתו וללמידתו. אני מעדיף כך, כי זה גם עוזר לי בניסוח מחודש ומתוקן של דברים שנכתבו לפני עשרות שנים. כשאסיים להעביר הכל ואקבל כאן די ביקורת עמיתים, כעצתך המועילה, אפרסם הכל במאמר מסודר בעיתון מדעי או בבלוג פרטי.

  444. ניקח לרגע הפסקה קצרה מן הפרוטונים ונבדוק מה קורה לחלל הריק.
    החלל הריק הוא אמנם בלתי צפיד, אך במובן מסוים יש לו התנהגות של חלקיקים צפידים. כאשר חלקיק צפיד נע אל תוך פיסת חלל ריק, הוא מותיר אחריו פיסת חלל ריק. אמנם פיסת החלל הריק שהוא נכנס אליה איננה זו שהוא הותיר מאחוריו, אך שתיהן חלל ריק ולכן דומות בתכונותיהן. תנועתו של חלקיק צפיד מנקודה א לנקודה ב מתרחשת במקביל לתנועתה של פיסת חלל ריק ממקום ב למקום א. אך בעוד החלקיק הצפיד נע בשלמותו, פיסת החלל הריק שהוא נע לתוכה קטנה והולכת במהלך התנועה ובאותו זמן פיסת החלל הריק שמאחוריו הולכת וגדלה. באופן הזה, אם חלקיק צפיד נע רק חלק מן הדרך, הרי פיסת החלל הריק התפצלה לשתי פיסות קטנות. מאידך, אוסף של פיסות קטנות של חלל ריק הינו למעשה פיסה אחת גדולה. בניגוד לחלקיק הצפיד, שיש לו צורה קבועה וגודל קבוע, לחלל הריק אין צורה קבועה והוא משתרע על פני מרחב גדול. על כן, בעוד שאת החלקיקים הצפידים ביחידת מרחב אפשר למנות גם כמספר שלם אי-שלילי וגם כנפח, את החלל הריק אפשר למדוד רק ביחידות של נפח. נפח החלקיקים הצפידים ביחידת מרחב ועוד נפח החלל הריק באותה יחידת מרחב הינו גודל קבוע. צפיפות החלקיקים הצפידים ביחידת מרחב הינה מדד לדלילות נפח החלל הריק באותה יחידת מרחב.
    כאמור בסעיף קודם, תנועתו של פרוטון במהלך גרביטציה או אינרציה היא תוצר של יחסי צפיפויות בין צדו הקדמי וצדו האחורי. אפשר לראות כאילו החלל הריק דוחף את הפרוטון בעוד שהחלקיקים הצפידים מושכים אותו. ככל שיחס הצפיפויות גדול יותר, כך מהירות תנועתו של הפרוטון גדולה יותר. אינטואיטיבית יש כאן רמז עבה למדי: בעוד שהחלקיקים הצפידים עשויים להיתפס בתודעתנו כאחראים ל”מאסה” (המתבטאת בגרביטציה ובאינרציה) החלל הריק אחראי ל”אנרגיה”; כאמור, זו רק נקודת מבט אינטואיטיבית; בהגדרות המפורטות והמדויקות של מאסה ואנרגיה נטפל מאוחר יותר.

  445. לא לוקח אחריות על מה שאיני אחראי לו. אין לי דיסלקציה.

    עברתי על כל האימיילים. רק פרק א’ ותבניות גאומטריות.
    לא משנה. אם אתה רוצה לבטל לי את הפטור, שלח הכל או פרסם באתר.

  446. ישראל שפירא, רוצה מחילה? קבל גלגול מחילות!
    מה שהעברתי לאחרים, כולל ר.ח, באימייל זה בדיוק מה שהעברתי לך. חלק השתמטו מקריאה, כמוך, וחלק טרחו להתעמק. על מושבות החלקיקים דיברתי כאן באחדות מתגובותיי, ולא רק ר.ח התייחס אליהן. ככלל, אני סבור שיש הרבה קווי דמיון משותפים בינך לביני, ודיסלקסיה היא אחד מהם. מה דעתך שנגיש תביעת מחילה יצוגית מיוסד הארץ?

  447. ר.ח.
    ברור שבלוג הוא רעיון טוב, אחרי שעבר את המסננת הראשונית של ביקורת עמיתים.

    אוקיינוס. לאוקיינוס יש מערכת מנוחה ברורה – כדה”א. אולם איזו מערכת מנוחה יכולה להיות לאוקיינוס ביקום אינסופי? יחסית למה?

    בוא נאמר שכדה”א מתנגש עם תואם שנע יחסית לו במהירות של 100 קמ/שנ לכיוון השמש ומתמזג עימו. מה שנקבל זו מערכת מנוחה חדשה לאוקיינוס המוגדל.

    אך מה תהייה מערכת המנוחה של אינסוף תואמים בכל המהירויות לכל הכיוונים המתנגשים ומתמזגים?

    מאיר.

    “אני מפקפק באינסופיות של מערכות אינסופיות, אבל נזרום. תסביר לי מדוע למערכת אינסופית א’ לא יכולה להיות מהירות יחסית למערכת אניסופית ב’, בהנחה ששתי המערכות מורכבות מאינסוף פריטים בדידים. איזו לוגיקה תשבר אם כל פריט במערכת א’ יחווה התחככות בקצב קבוע עם פריטים ממערכת ב’ המקדמים את פניו בכיוון נתון, וכן להפך.”

    אם אתה מקבל את רעיון היקום האינסופי, ההומוגני והאיזוטרופי, איך תוכל לדבר על סופיות של מערכות? האין זה מחייב העדפת מערכת אחת על השניה, או של איזור אחד בחלל על אחר?

    נקודת המוצא של כל הרעיון הוא המטוטלת הבליסטית – שמעל מהירות מסויימת, הרבה פחות מאינסוף, כל החלקיקים נהפכים לשקופים. בעניין החיכוך, אותו הגיון של חיכוך יכול לתפוס בגז במיכל של אמישראגז. הסיבה שזה לא קורא היא שכל ההתנגשויות הן אלסטיות לחלוטין. (בניגוד אגב, למודל לסאז’ המקורי שמחייב התנגשויות א-אלסטיות ומכאן חיכוך).

    בעניין המודל שלך:

    הנקודה העיקרית היא השאלה אם אתה מקבל או שולל את מה שנכתב לגבי איינשטיין:

    To test the idea, Einstein considered a small body at rest inside a rotating shell of
    material in otherwise empty space. Using his theory of relativity, he calculated what would happen. It turns out that the body should indeed feel a tiny gravito-magnetic force

    אם אינך מקבל, אשתדל לעבור על כל ההוכחה של שימה (אין לינקים, זה ספר מלפני כמעט 50 שנה) כדי לראות איך זה מסתדר עם התאורייה שלך.

    תבין, אינני טוען שיש לך איזו טעות – להיפך. גם איינשטיין נסוג מעקרון מאך. פשוט אף פעם לא התחוור לי למה, וייתכן שהתאוריה שלך היא ההסבר.

    נקודה, אריה, או מי שקורא:

    מה דעתכם על השאלה הבאה:

    ניסוי מייקלסון מורלי בא כדי לנסות למצוא את מערכת המנוחה של האתר.

    שאלה:

    לפי תמונת העולם ב1887, היקום הוא אינסופי, הומוגני ואיזוטרופי. האם יכולה ביקום כזה בכלל להיות מערכת מנוחה ל”אוקיינוס” האתר? האין מציאת מערכת מנוחה כזו מפרה את הנחת ההומוגניות והאיזוטרופיות? אילו נמצאה המערכת, כלום לא היינו מחוייבים לשאול: מדוע דווקא זו? האין זה משול למציאת מרכזו של ישר אינסופי?

    אם אפשר, התייחסות לשאלה ולא לשואלה.

    יובל
    “התגובה ממתינה לאישור.” כנראה בגלל שכתבתי איוב.

  448. יובל.

    “ישראל!
    מה נפל עליך שבגללו אתה מצטט פתאום את איוב?!”

    דווקא ממך ציפיתי שתבין.

    זה לא איוב, זו תשובת אלוהים לאיוב. תשובה מאוד לא במקום, יש לציין. המסכן מתגרד בחרס, הילדים מתו, האשה ברחה, הבקר קבב, הכל בגלל שהבוס עשה התערבות עם הס’ ולא רצה לצאת פרייר. כשאיוב זועק בצדק “ארץ נתנה ביד רשע!” ותובע הסבר, מתגלה לו אלוהים מתוך הסערה, ועושה לו בוחן פתע בבריאת עולם, כאילו שאיוב המסכן התיימר אי פעם לדעת איך בוראים או מנהלים עולמות.

    הזכיר לי קצת את כל מטר השאלות המונחתות עליך בקשר למודל שלך.

    “ישראל, להצתתך המאוחרת:
    על שאלת ייסוד הארץ התעכבתי כאן רבות ונתקלתי בחומת חוסר סבלנות אשר גם אתה היית אחד מנדבכיה. הדברים אינם מסובכים והם מצריכים הבנה מאד בסיסית ומינימלית של הגיון יומיומי, ובכל זאת התעלמת. הנקודה השחורה שסימנתי ביומני על יד שמך נובעת מזה ששמתי לב שאתה נוהג להתעלם מדברים שאינם תואמים את האג’נדה שלך. אולי אתה סתם לא מבין, או שמא בכלל אינך קורא. אם ניסית להבין ולא הצלחת, אשים טיפקס ואקח את האשמה על עצמי.”

    קח אחריות. הייתה לי טענה אחת יסודית כלפיך כל הזמן: שאינני יודע מהו המודל שלך בכלל. אתה העברת אותו באימייל פרטי לאחרים, אך לא לי. אתה שאלת אותי כל הזמן שאלות על המודל, ואני כל הזמן עונה לך שאינני יכול לענות לך אם אינני יודע מהו.

    איך אני יודע שהעברת אותו ל ר.ח. לדוגמה ולא לי? ר.ח. התייחס למושבות החלקיקים שלך שהזכירו לו את מושבות החיידקים. אני מעולם לא קיבלתי דבר על מושבות אלו, מ.ש.ל.

    בקיצור, חראם עליך יובלי. האשמת אותי לשווא בגין הרשלנות הפושעת שלך עצמך. עליך ליפול על פניך, לבקש ממני מחילה, ולהניח לי לגלות את הצדדים הרחומים שבאופיי.

  449. ישראל, להצתתך המאוחרת:
    על שאלת ייסוד הארץ התעכבתי כאן רבות ונתקלתי בחומת חוסר סבלנות אשר גם אתה היית אחד מנדבכיה. הדברים אינם מסובכים והם מצריכים הבנה מאד בסיסית ומינימלית של הגיון יומיומי, ובכל זאת התעלמת. הנקודה השחורה שסימנתי ביומני על יד שמך נובעת מזה ששמתי לב שאתה נוהג להתעלם מדברים שאינם תואמים את האג’נדה שלך. אולי אתה סתם לא מבין, או שמא בכלל אינך קורא. אם ניסית להבין ולא הצלחת, אשים טיפקס ואקח את האשמה על עצמי.

  450. נקודה,
    אמנם אני מביא גם חישובים כמותיים, אך לכל מי שמצפה למצוא אצלי משוואות מסובכות צפויה אכזבה. אינני ממציא מתמטיקה חדשה אלא משתמש במתמטיקה מאד בסיסית. לא טרחתי על משוואות דיפרנציאליות, אבל אם תרצה אני מאמין שתוכל למצוא בפיסיקה די חומר גלם גם לאלה. המודל שאני מביא כאן מתיימר לבנות את משפטי היסוד הרבים של הפיסיקה מחומרי גלם פשוטים ומעטים. אם אתה מצפה למצוא אצלי גדולות ונצורות, אנא קבל מראש את התנצלותי העמוקה וכנה.

  451. ישראל,

    אני מפקפק באינסופיות של מערכות אינסופיות, אבל נזרום. תסביר לי מדוע למערכת אינסופית א’ לא יכולה להיות מהירות יחסית למערכת אניסופית ב’, בהנחה ששתי המערכות מורכבות מאינסוף פריטים בדידים. איזו לוגיקה תשבר אם כל פריט במערכת א’ יחווה התחככות בקצב קבוע עם פריטים ממערכת ב’ המקדמים את פניו בכיוון נתון, וכן להפך.

    הצצתי בלינק. תודה. לא הכרתי את ריימונד ספציפית, אבל נתקלתי בכמה כמוהו שמגששים באפלה סביב הדבר הנכון. אפשר לומר שהוא רחוק מהאמת כרחוק מזרח מצפון מערב.

    לשמחתי אני פטור ממחלות הילדות שעימן הוא מתכתש, מכיוון שבניגוד אליו אני הצלחתי לבסס תורת כבידה חלקיקית שלמה בטרם התברר לי שבלי כוונה ובלי לשנות מאומה קיבלתי את האינרציה כבונוס, בעוד הוא (והוא לא היחיד במצב הזה) מובל לפי אינטואיציה (נכונה) שמקור האינרציה הוא בכבידה עצמית ומנסה לפלס באמצעותה דרך להבנת הגרביטציה.

    המפתח הוא בגאומטריה המודגמת בשרטוט שהבאתי למעלה. אני יודע שהיא נכונה כי אין מצב שהייתי מקבל את כל הדברים שקיבלתי דרכה כבונוס אם היא היתה שגויה. מי שאין לו את השרטוט הזה בראש ימשיך לחפש את המחט בערימת השחת. שים לב שבלי השרטוט הזה, בלי הגאומטריה המיוחדת הזו, כאשר אתה מסיט חלקיק ממקומו אתה מרחיק אותו ממרכז השדה העצמי, וככל שאתה מרחיק אותו יותר, יותר קל לך “לדרדר” אותו במורד השדה. ריימונד מבין שכך לא מייצרים אינרציה (ואני משער שזו הסיבה שמאך עצמו לא התייחס לאפשרות של כבידה עצמית), ולכן הוא ממציא שדחיפת החלקיק דוחסת את השדה. אם היתה לו מול העיניים הגאומטריה הנכונה, ואם הוא היה מבין שחלקיק עובד במחזורים, הוא היה קולט שאין “חיבור” בין החלקיק לשדה. חלקיק לא יכול “לדחוס” שדה, אבל הוא יכול לייצר אותו מחדש בתנאי פתיחה חדשים.

    ההבדל הוא בראש ובראשונה בין גאומטריה של כדור שיושב על חוד של הר (ריימונד ודומיו), לבין גאומטריה של טבעת שיושבת על אמצע הצלע של הר, מקיפה אותו מכל צדדיו (מאיר עמירם).

  452. ישראל יקירי,

    מי אמר לך לסתום או להחריש? אבל יש דרך. אם אתה שואל אותי, אני הייתי אומר:
    “מאחר ויש היום בעיות בפיזיקה שבה תאוריה אחת לא מסתדרת עם תאוריה אחרת ברור שיש לחפש הסבר נוסף (זה טריוויאלי וידוע). אני מציע לצאת מאקסיומות שונות מאלה המקובלות ופרשנות שונה לתוצאות של ניסוי מ-מ המבוססת על האפשרות שלא ל-ד-ע-ת-י למצוא מערכת מנוחה לאתר ביקום אינסופי והומוגני . באם נקבל הנחות אלה אני אראה שבעיות א’ ב’ ואפילו ג’ נפתרות ומתקבלת תורה שלימה ללא סתירות. על מנת לאשש את התאוריה שלי ואת הפרשנות שלי לתוצאות של מ-מ אני מציע ניסוי שבו נעשה א’ ב’ ג’. אם תוצאותיו יראו X אזי התאוריה שלי תאושש ואם לא נצטרך לחזור לשולחן העבודה.”

    אבל באמת שזה בסך הכל עניין של ניסוח וסגנון. מה שאני יודע שאם אני מקבל מאמר לרוויו שייקטול נניח את ניסיון מסלסון-שטאל (שהראה שכפול סמי קונסרבטיבי של DNA) כ “לא לוגי” אני מייד אהיה אנטי ותיווצר לי בעל כורחי דעה לא כל כך טובה על מחבר המאמר.

    ולגבי בלוג? זו הייתה הצעה ידידותי קונסטרוקטיבית, לא רוצה אל תיקח.

    ולעצם העניין, אתה כל הזמן חוזר על המנטרה “נניח שאפילו היו מוצאים אותה. נניח שהיא נעה יחסית למערכת השמש במהירות של 345 קמ/שנ לכיוון קונסטלציית ליאו.”. אתה לא מבין שמערכת המנוחה לא נעה ביחס לעצמים שבה אלא הם ביחס אליה? דומה הדבר לדגים באוקיינוס שתבוא ותטען שהאוקיינוס כולו נע ביחס לדג נמו, אולם מה נגיד לגבי אבא של נמו שנע בדיוק לכיוון ההפוך? לא. כל הדגים נעים ביחס לאוקיינוס.

  453. יובל, יש לך משוואות. כי לספר סיפורים כל אחד יכול, על חלקיקים צפידים ועל פיות טובות.
    משוואות דיפרנציאליות יש?

  454. נקודה, בתודה על תשומת הלב,
    עד כה התמקדתי בחלקיקים הצפידים. אך גם לחלל הריק שביניהם יש תפקיד חשוב בפיסיקה. אני עוצר כעת מפאת חוסר זמן. אשוב אחה”צ ואפרט.

    ר.ח וישראל, נראה לי שאנחנו חולקים במשותף יותר מעניין אחד 🙂

  455. ר.ח.
    בוא נאמר שאתה היית רוצה להסביר את הרעיון שלי, ללא יהירות, ללא גאוותנות, למעשה אם אפשר בכלל באופן אובייקטיבי לחלוטין. האם היית יכול למצוא איזה שהוא ניסוח שהיה משמיט את הנקודה המרכזית, אי תקפותו של ניסיון מ-מ בלי שזה ישמע מזלזל ומתנשא?

    אז מה האלטרנטיבה – לא לאמר את העיקר?

    הנה הניסוח המדוייק שאני בחרתי:

    “לכן, אם נאמר שהיה מייקלסון מצליח למצוא את מערכת מנוחת האתר, ונאמר שהיא נעה יחסית אלינו במהירות של 1887 קמ/שנ לכיוון קונסטלציית האריה, היינו צריכים לשאול: למה דווקא זו? מה מיוחד בה? איפה ההומוגניות?

    זה כמו למצוא את נקודת מרכז היקום האינסופי. למה דווקא זו?

    וכאן אני יודע שחייבת להיות לי טעות. אין, אין מצב שאני חשבתי על זה ולורנץ לא. נכון, התקבלתי להרווארד, אבל רק לתואר ראשון. ז”א התקבלתי למשרת מנקה השרותים בהרווארד, אבל רק של הסטודנטים לתואר ראשון. לשרותי הסגל לקחו מישהו יותר מתאים.

    אז זה שלא ידוע לי שמקסוול ולורנץ העלו הסתייגות זו, מראה לי לוגית, שיש לי טעות.

    אבל עד שאדע איפה הטעות, אני חייב לנסות לפתור את התעלומה, כדי שאוכל למות מאושר.”

    אני קורא את זה, ואינני מרגיש שיש פה איזושהיא יהירות. רק צורת התבטאות האופיינית למחבר הפיסקה.

    לעצם העניין.

    הייתי די משוכנע, שאחרי שאעלה את הנקודה, מישהו, אולי אתה, אולי אריה, יפנה אותי מהר מאוד ללינק שיראה לי איפה הטעות.

    כי ברור (לפחות לי) שקיימות רק שתי אפשרויות: או שיש לי טעות – או שלא.

    אם אין לי טעות, וניסוי מ-מ הוא אכן איננו לוגי – הרי שאין מנוס מהמסקנה שכל האחרים טעו, ולא סתם טעות, טעות קולוסאלית.

    מה שפשוט לא מתקבל על הדעת.

    מצד שני אין גם מנוס מהמסקנה שהמעטים שקראו את התגובה עד כה, שרובם בוודאי אינטלגנטים ושולטים בחומר, לא הצביעו על הטעות. מכאן אני מסיק שאני לא הטמבל היחיד שאינו רואה את הברור מאליו.

    מה היית מייעץ לעשות במקרה כזה? פשוט לסתום מחשש להראות יהיר? אני יודע היטב שהתמונה חשובה לי הרבה הרבה יותר ממקומי בה. אין ברירה, אני אמשיך לשאול את אותה שאלה, ולחטוף כמו תמיד.

    יש לי עוד הרבה מה לאמר, על הביטול כלאחר יד לדעתי של תאוריית מקסוול, ועל הסתירה שקיימת אולי בין תאוריות היחסות והמפץ הגדול, אך הסכמנו כבר שהתמצינו.

    שמעתי על כתובת בלעם. על “דיר בלק!” כבר שמעת?

    ויובל, אם אתה קורא, למרות שנתת לי פטור: האם אפשר לפי המודל שלך לדעת את מועד המלטתן של יעלות הסלע? ומי הוליד אגלי טל? ובכלל, איפה היית ביסדי ארץ? הגד אם ידעת בינה!

  456. ישראל,
    אני מתנצל אם נעלבת. חס וחלילה שאני אמנה על מחנה שונאי ישראל. אני גם לא מבין מה אתה כועס, אתה רצית ביקורת על המודל, קראתי מה שכתבת, ניסיתי להבין והעליתי סייגים ושאלות, לא זו הייתה הכוונה?

    הנקודה שחורה לי בסעיפים שציינת היא 3, אני עדיין חושב שלבוא ולאמר בפסקנות שכזו שהנסיון של מייקלסון מורלי הוא לא לוגי זו יהירות. אין לי בעיה עם תאוריות מהפכניות, רעינות חדשניים, אני מסכים שימי הבניים נבעו מקיבעון מחשבתי עמוק בעיקר של הכנסייה הקתולית ושחשיבה מחוץ לקופסא זה מה שמקדם את העולם.
    אבל, וזה אבל גדול, לא מתחילים תאוריה ב”הנסיונות הקודמים שכולכם לומדים ומצטטים במשך שנים הם שטויות (כלומר לא לוגים)”. שים לב שזו בכלל לא ביקורת על המודל או הנסיון המתוכנן שלך אלא על הגישה.
    אבל אם מפריעות לך הערות שכאלה אני מבטיח להיות ענייני בלבד מעתה ואילך.
    דרך אגב תשובה יפה לשאלה שלך “אגב, לא נראה לך שסעיף 3 שלי רלוונטי מאין כמוהו? ואם כן, האין זה תמוה שכל הקוראים האינטלגנטים והשולטים בחומר (כולל אתה) שקראו את השאלה שלי עדיין לא ענו עליו?”
    נתן לך כבר ב-1905 אחד בשם א. איינשטיין שביסס על הניסוי הלא לוגי הזה תורה שלימה שבעזרתה הנחיתו אנשים על הירח ואתה מוצא דרכך ברחובות L.A גם אם אתה בשכונות הלא מוכרות שלה. אז מה אתה רוצה ממני?

    דרך אגב, אם אתה כבר בעניין בלעם,על כתובת בלעם שמעת?

  457. רפאים.
    כשיהיה לך משהו ענייני לכתוב לי, במקום ההתעסקות האובססיבית במי אני ומה אני, יהיה אפשר לדבר איתך.
    עד אז, קישטה.

  458. ישראל
    עם כל הכבוד, אתה לא פרופסור לפיזיקה גרעינית, וגם לו היית אתה לא ברמה של המדענים שכן יכולים לדון על הנושא.
    אולם, אתה דווקא מנסה להתיימר ככזה.
    “אתה מרביץ לסוס מת” כמו שאומרים אצלכם.

  459. מאיר.
    קח ישר אינסופי. האם תוכל לאמר מהי נקודת האמצע שלו? למה דווקא זו?
    קח יקום אינסופי. כנ”ל.
    קח יקום אינסופי, הומוגני ואיזוטרופי. נניח שיש לו מערכת מנוחה מוחלטת כפי שניסו מייקלסון ומורלי למצוא. נניח שאפילו היו מוצאים אותה. נניח שהיא נעה יחסית למערכת השמש במהירות של 345 קמ/שנ לכיוון קונסטלציית ליאו.
    שאלה: למה דווקא זו? איפה ההומוגניות?
    זה כל הסיפור.

    לפני שנחזור לאינרציה, תוכל אולי להציץ בלינק

    http://www.reocities.com/perfectfluid/

    תודה.

  460. ישראל,

    אתה יכול להרחיב קצת יותר (לטובת בלתי אינטליגנט שמנסה לעקוב) מדוע יקום אינסופי והומוגני לא יאפשר מערכת מנוחה לאתר, ולו רק מקומית?

    בהפוך על הפוך: הרי אחת הבעיות שמילר ניסה להתגבר עליהן בניסויים החוזרים זה ה”אולי” גרירה של האתר עם המערכת המקומית. כלומר הוא שיער שיכולה להיות לאתר מערכת בתוך מערכת.

  461. ר.ח. ידידי.
    בוא ניקח את דבריך אחד לאחד ונראה אם הם עומדים במבחן המציאות.

    1. “יובל (וגם יהודה ישראל וכל מי שחושב על מודלים ומת שיקראו אותם ויגיבו עליהם)”

    אינני יכול לדבר בשביל אחרים אלא רק בשביל עצמי. מקריאת השירשור בכתבה זו, דווקא נראה יותר שאתה הוא זה שלחצת שאומר לך את הרעיון שלי ולא להיפך. ראה תגובה זו ואילך:

    https://www.hayadan.org.il/astronomers-reach-new-frontiers-of-dark-matter-130112/#comment-324601

    אני מעולם לא ניסיתי “לשווק” איזו תאורייה. להיפך. די עמדתי על כך שאם אתה מעוניין ברעיון, עליך לזחול ולקרוא לינקים מסויימים הקשורים לנושא, ואף דיברתי על “סחר” ברעיונות (ע”ע חיסון לשפעת).

    להעמיד את זה כאילו שאני “מת” שיקראו את הרעיונות שלי ויגיבו עליהם, חוטא לאמת ולחוש הצדק. נכון, אני רוצה להעמיד את רעיונותי לביקורת, אך רק עם מי שמעוניין בכך ובעל שליטה סבירה בחומר. אתה הוכחת את שניהם, בכך שביקשת במפורש לדון ברעיון, ובכך שקראת את הכתוב בלינק למקסוול (מש’ 136, זוכר?).

    אני מוכרח להודות שהפתעת אותי בתפיסת הפרטים החדה, וגם הודיתי לך בעבר על כך שעברת על הרעיון בפרטנות. אך זו היתה בחירה שלך, נא לא להפיל תיקים.

    2. “למה שלא תקימו לכם בלוגים כמו מאיר עמירם או אותה בחורה ושם תפרשו את משנתכם בצורה מסודרת וברורה ולא מעל גבי פוסטים מעורבבים ומעורבלים המהווים תגובה למאמר שנשכח מזמן באתר הידען?”

    הסברתי לך מקודם. אין הגיון להקים בלוג אם אינך בטוח שמה שיש לך לאמר נכון בכלל. עדיף לשאול קודם, כדי לחסום טעות באיבה. זה מה שעשיתי.

    ואם נאמר שהיינו לוקחים כתבה אחרת, נגיד איזו כתבה זנוחה של נחמני מלפני 5 שנים ומגיבים בה, האם זה היה משנה משהו? מי מחייב אותך, או נקודה, או מישהו אחר שאינו מעוניין אפילו להציץ בכתבות אלו? והאם אין העובדה שרק כתבה זו זוכה בימי חייה למספר תגובות בסדר גודל של כל הכתבות האחרות באתר, מעידה על ההתעניינות הרבה בנושאים אלו?

    3. “לגבי 2, 3 כאמור קטונטי לקבוע אולם, שוב, אם כל הקהילה המדעית כן החליטה “פתאום” כדבריך שאין אתר ושניסוי מ-מ קביל לוגית אז אולי תבדוק את ההנחות היהירות שלך שוב? וזו לא התקפה אישית, לדעתי זו יהירות ועזות פנים להטיל ספק בכל מה שמקובל היום בפיזיקה הנוכחית שנחקרת ונבדקת ללא לאות ע”י מיטב המוחות בעולם ללא שום ממצא ניסיוני או בדל של עדות. יתירה מכך, לבוא ולהגיד שכ-ל הפיזיקאים המלומדים, ללא ציניות, לא רואים שניסוי מ-מ הוא לא לוגי???
    מכיר את הבדיחה העתיקה על דוד לוי שאישתו מתקשרת אליו ואומרת “תיזהר, אמרו בחדשות שיש איזה נהג שנוהג נגד כיוון התנועה בכביש מס’1”. “איזה אחד?” הוא שואל? “אלפים!”

    יהירות? עזות פנים? בזה ששאלתי שאלה ואמרתי מראש שכנראה יש לי טעות, אך אני רוצה לדעת איפה היא? אני ציפיתי דווקא לתשובה בנוסח: ראה ישראל, הנה לינק זה וזה, השאלה ששאלת נחקרה כבר והנה התשובה. ראה דוד לוי, העיגול האדום עם הפס הלבן פירושו “אין כניסה”.

    שלא לדבר על הסתירה הפנימית עם דבריך שלך:

    “בנוסף לא ברור מה הטיעון שלך לגבי מקסוול ולורנץ. זה שהם לא חשבו על זה קודם אומר שזה לא נכון? בגלל הלך מחשבה כזה העולם נתקע בימי הבניים מאות שנים כי “אם אריסטו לא אמר את זה קודם אז זה כנראה לא נכון”. תתבייש לך דקה ותמשיך מייד עם קו המחשבה שלך. רוב התגליות וההמצאות הן כאלה שאח”כ אומרים “וואלה, איך לא חשבו על זה קודם??” עובדה. לא חשבו על זה קודם!”

    שים לב גם שניסוי החשיבה של גלילאו בעניין מהירות הנפילה השווה של כל הגופים הוא ניסוי חשיבה פשוט וברור מאין כמוהו, ובאמת מפליא הדבר שהיה צריך אלפיים שנה כדי לחשוב עליו.

    בקיצור, ר.ח., כמו תמיד אני מזמין ביקורת על הרעיונות שלי. אבל אם לשאלתו של ישראל: האם ניסוי מ-מ קביל לוגית, האם יכולה להיות מערכת מנוחה לאתר ביקום אינסופי והומוגני, כמו שהאם יכולה להיות נקודת מרכז לאינסוף? עונה ר.ח.: למה מי אתה בכלל ומה בדיוק הסיפור שלך? אני נאלץ להסיק בצער שעברת, בלי כל סיבה הגיונית, למחנה של שונאי ישראל.

    אגב, לא נראה לך שסעיף 3 שלי רלוונטי מאין כמוהו? ואם כן, האין זה תמוה שכל הקוראים האינטלגנטים והשולטים בחומר (כולל אתה) שקראו את השאלה שלי עדיין לא ענו עליו?

    יאללה, יה בלעם, התהפך חזרה ותחזור להיות ר.ח. הנחמד.

    מי יודע מה הסיבה האמיתית שבלק שלח את בלעם לקלל את ישראל?

  462. רשמת “רוב הדברים שפירטת מוצאים אצלי הסבר.” אז חשבתי שההסבר קיים ואתה רק צריך לפרסם אותו.

  463. הי יובל,

    מרתק! תמשיך עם זה, נראה לאן תגיע בסוף.
    אל תשים לב לכל הספקנים ולמרפי ידיים, שיהיה
    לך המון בהצלחה וכל הכבוד על המאמץ והאומץ.

    🙂

  464. נקודה,
    כן. תיאוריה של הכל. האקסיומה הבסיסית היא “היש נוצר מן האין”. בשלב מסוים של הפיתוח שלה הגעתי אל משפט החלקיק של החומר האפל שאומר בערך כך:
    א) העולם בנוי ממספר גדל והולך של חלקיקים צפידים ומאינסוף חלל אדיש (או חלקיקים בלתי צפידים).
    ב) בהינתן די חלל אדיש, משרים החלקיקים הצפידים מעצמם על סביבתם ליצירת חלקיקים צפידים כמותם. חלקיק צפיד המוקף בחלקיקים צפידים בצפיפות גדולה מאד, משנה את מצבו לחלקיק בלתי צפיד (דהינו הופך לפיסת חלל ריק, או “מת”).
    ג) חלקיק צפיד בודד תופס נפח מוגדר במרחב אשר שום חלק ממנו אינו תפוס בוזמנית על ידי חלקיק צפיד אחר.
    ד) החלקיק הצפיד הינו בעל תנועה עצמית עצמונית המשתנה באורח אקראי בכיוונה ובמהירותה (להוציא הגבלה אחת: תנועתו של חלקיק צפיד אינה מתקיימת אל תוך נפח התפוס על ידי חלקיק צפיד אחר).

  465. יובל,

    “רוב התופעות שהבאת מוסברות, מחושבות ומכומתות על ידי מודלים פיסיקליים קיימים. המודל שלי מתיימר לבנות את אותם מודלים פיסיקליים.”
    – זו לא רוח דבריך בתגובות האחרונות. קיבלתי את הרושם שהמודל שלך יכול להסביר תופעות שמסבירה פיזיקה קוונטית. אם אתה טוען שהתופעות מוסברות, מה אתה מנסה להסביר? גם לא הבנתי מה זה מודל שבונה מודל.

    “כתרגיל, אפשר להגיע אל התופעות השונות ישירות מהמודל שלי.”
    – נא הראה לנו.

  466. יובל, לא עקבתי אחרי כל השיחה פה, אך מכל התכתובות אני מבין שאתה מנסה לעשות משהו מעניין.

    תקן אותי אם אני טועה אך אתה מנסה לבנות תאוריה כלשהי…האם אתה מנסה לבנות תאוריה של הכל? או שאני סתם מתבלבל?

    ואם כן, מהן ההנחות או האקסיומות הבסיסיות עליהן אתה משתית את הרעיונות שלך?

  467. סטודנט, טכניון,
    רוב התופעות שהבאת מוסברות, מחושבות ומכומתות על ידי מודלים פיסיקליים קיימים. המודל שלי מתיימר לבנות את אותם מודלים פיסיקליים. כתרגיל, אפשר להגיע אל התופעות השונות ישירות מהמודל שלי. אך, באנלוגיה למדעי המחשב, זה כמו לכתוב קוד מורכב תוך שימוש בשפת מכונה בלבד.

  468. יובל,

    אפשר למלא כמה מדפים בספריה בנושאים שכתבתי לך עד עכשיו. אמרת שיש לך הסבר לרוב מה שפירטתי – אני אשמח לראות את ההסברים האלה.

  469. ישראל,
    במאמר נשכח התכוונתי שהדיון שהתפתח אינו קשור למאמר (אולי באיזו צורה עקיפה) וכבר קיבל הרבה מאד תפניות משל עצמו. אין אפשרות לעקוב אחריו והוא לא מסודר ואסוציאטיבי.
    אם היית כותב מאמר רק על המודל שלך ומרחיב את הנקודות שכתבת למעלה ויובל היה מתאר את המודל שלו ויהודה ומאיר כל אחד בנפרד הדיון היה הרבה יותר ממוקד וברור.
    כך זה סלט שלם.

    לגבי הסעיפים שלך אין לנו ויכוח על 4.
    לגבי 2, 3 כאמור קטונטי לקבוע אולם, שוב, אם כל הקהילה המדעית כן החליטה “פתאום” כדבריך שאין אתר ושניסוי מ-מ קביל לוגית אז אולי תבדוק את ההנחות היהירות שלך שוב? וזו לא התקפה אישית, לדעתי זו יהירות ועזות פנים להטיל ספק בכל מה שמקובל היום בפיזיקה הנוכחית שנחקרת ונבדקת ללא לאות ע”י מיטב המוחות בעולם ללא שום ממצא ניסיוני או בדל של עדות. יתירה מכך, לבוא ולהגיד שכ-ל הפיזיקאים המלומדים, ללא ציניות, לא רואים שניסוי מ-מ הוא לא לוגי???
    מכיר את הבדיחה העתיקה על דוד לוי שאישתו מתקשרת אליו ואומרת “תיזהר, אמרו בחדשות שיש איזה נהג שנוהג נגד כיוון התנועה בכביש מס’1”. “איזה אחד?” הוא שואל? “אלפים!”

  470. המשך, אינרציה:
    כאמור, אם צפיפות הסביבה בצדו האחד של הפרוטון נמוכה מצפיפות הסביבה בצדו האחר, הפרוטון ינוע לכיוון המקום הצפוף יותר. היות שהפרוטון צפוף יותר מסביבתו, הוא מייצר יותר חלקיקים מאלה שסביבתו מייצרת. החלקיקים שמייצר הפרוטון מתפזרים לכל כיוון ללא העדפה, וכך יחס צפיפויות הסביבה בין שני צדי הפרוטון נשמר ואיתו נטיית הפרוטון להמשיך בתנועתו.

  471. ישראל,

    לא ברור לי מה מלהיב בזה שמקבלים מנוסחת מהירות ההימלטות את החוק השני של ניוטון. קח את חוק היפוך הריבועים ותגלה ש F=m*g וש g=M*G/R^2
    כל חוקי התנועה של ניוטון קשורים זה בזה ובחוק היפוך הריבועים.

    חלק את שני אגפי המשוואה שהצגת בריבוע הרדיוס ותקבל תאוצה משני הצדדים. תכפיל במסה של חלקיק אלמנטרי ותקבל את הכח שהמסה היקומית מפעילה על חלקיק אלמנטרי הממוקם בקצה היקום. האם היא מפעילה עליו את הכח הנתון הזה בגלל שהיא המסה היקומית, או בגלל שהוא חלקיק אלמנטרי עם המסה הנתונה? האם משהו במשוואה הזאת היה משתנה אילו המסה היקומית היתה עשירית ממה שהיא (או מאית, או מיליונית)?

    אין שום דבר מפתיע בכך שיש קשר בין גבול היקום הנצפה לבין מהירות האור, ואין שום דבר מפתיע בכך שהמסה בתוך גבול היקום הנצפה היא סך המסה היקומית של היקום הנצפה. אלו בסך הכל דרכים שונות להגיד את אותו דבר.

    אז מה מפתיע בכך שהמסה היקומית מפעילה כח נתון על חלקיק בעל מסה נתונה במרחק נתון? איך זה יותר קשור לתכונת האינרציה של החלקיק, משקשורה לכך המסה העצמית של החלקיק הזה?

    כל עוד לא נמצא בכתובים אחרת, דבר אחד ברור: לא מאך ולא דניס שימה הביאו בחשבון את הפתרון של מאיר עמירם בטרם טרחו להמציא אינטראקציות מאגיות כדי להסביר תופעה שניתן להסביר אותה באמצעות חוק היפוך הריבועים של ניוטון ועוד לקבל מתוך זה כבדרך אגב את שלושת חוקי התנועה שלו. דבר מביך קצת, אבל חתיכת דבר.

  472. ר.ח.

    “יובל (וגם יהודה ישראל וכל מי שחושב על מודלים ומת שיקראו אותם ויגיבו עליהם),”
    האם לא נראה לך שלפני שמקימים בלוג, עדיף לבדוק עם אנשים חושבים אחרים באם אין טעות בסיסית ברעיון אותו מנסה אדם להציג?

    לי נראה בהחלט. זה מה שעשיתי פה. שים לב שהצגתי כמה סעיפים לביקורת:

    1. תאוריית האתר של מקסוול הייתה נחשבת להזויה למדי, בלא הנוסחאות המפורסמות וניסוי הרץ.
    2. מכיוון שהיא הוכחה, אי אפשר להתעלם ממנה ולהחליט פתאום שאין אתר.
    3. ניסוי מ-מ אינו קביל לוגית. לא יכולה להיות מערכת מנוחה לאתר ביקום אינסופי והומוגני, כמו שלא יכולה להיות נקודת מרכז לאינסוף.
    4. מספיק שיתברר שניטרינו אחד ויחיד הצליח לעבור את מהירות האור כדי למוטט את היחסות.
    5. בניגוד לשזירה הקוואנטית, שבה אין מנוס מהמסקנה שאינפורמציית הספין עוברת מיידית, במקרה של האור קיימת לפחות אפשרות תאורטית לכך שמהירות האור זהה בכל מערכות היחוס, וזאת ללא התארכות זמנים במערכות שאינן מואצות.

    עד עכשיו לא קיבלתי תשובה.

    הנה דבריך לגבי סעיף 3, לב הטיעון:

    “רק הערה אחת לגבי סעיף 3 שלך. אתה טענת בעבר שאתה מרכין ראש מול מקסוול לורנץ ואחרים. טענת גם שהתאוריה האקזוטית שלא לאמר מופרכת של ל.ס ראלית כי הרבה אנשים כולל ניוטון הבינו שיש כאן מעין גרויטציה ורק בעיית החיכוך נשארה. אולם מצד שני אתה בא כאן ובמחי יד קוטל את נסיון מ-מ שנחשב לניסוי מספר אחד בכל הזמנים בפיזיקה (טוב, לפחות בעשיריה הראשונה) וכל הקהילה המדעית פיזיקלית אימצה וקיבלה אותו ובעקבותיו את תורת היחסות. קטונטי לשפוט בנושא אולם יש לי הרגשה שלא סביר שכל הפיזיקאים במאה ה-20 מטומטמים ולא ראו שזה ניסוי לא לוגי כמו שאתה טוען.”

    ותשובתי: “וכאן אני יודע שחייבת להיות לי טעות. אין, אין מצב שאני חשבתי על זה ולורנץ לא.”

    זו בדיוק הסיבה שהצגתי את הרעיון פה. כדי לראות אם מישהו יוכל להצביע על טעות, או על איזה הסבר, או לינק למאמר שלי לא ידוע עליו.

    אגב, אם לא עשיתי זאת במפורש עד עכשיו, תודה על ההשקעה והביקורת.

    ונראה לי שאתה קצת טועה בקשר לנשכחות המאמר. זה אחד המאמרים הנקראים והמוגבים ביותר בתולדות האתר.

  473. יובל,

    “ס,ט: הרשימה ארוכה. רוב הדברים שפירטת מוצאים אצלי הסבר. כרגע אני מתעמק באתגר שהציב לי י.ש שעניינו אי-הלוקליות בשזירה קואנטית.”

    אני סבור שמשתתפי הדיון ישמחו לראות את ההסברים האלה (כשהראשון יהיה חישוב אורך הקשר הכימי של HCl מהספקטרום שקישרתי).

  474. יובל (וגם יהודה ישראל וכל מי שחושב על מודלים ומת שיקראו אותם ויגיבו עליהם),

    למה שלא תקימו לכם בלוגים כמו מאיר עמירם או אותה בחורה ושם תפרשו את משנתכם בצורה מסודרת וברורה ולא מעל גבי פוסטים מעורבבים ומעורבלים המהווים תגובה למאמר שנשכח מזמן באתר הידען?
    תוכלו להמשיך את הדיון כאן אולם אז תוכלו לשלוח לינקים ולא להפנות לתגובות עלומות ונשכחות שלכם מן העבר.
    קצת עבודה, נכון אבל שכרה במקומה.

  475. יובל,
    אם אכן תוכל לענות לאתגרים של סטודנט טכניון ולעוד כמה אתגרים מרושעים שניתן בקלות לחשוב עליהם וגם תציע הסבר לתופעות שאינן ברורות כיום כגון שזירה, חומר אפל וכו’ אזי המודל שלך יתפוש תאוצה. אבל מה אני מספר לך? הרי אתה יודע את זה.

    בהצלחה

  476. לכל המגיבים,
    ס,ט: הרשימה ארוכה. רוב הדברים שפירטת מוצאים אצלי הסבר. כרגע אני מתעמק באתגר שהציב לי י.ש שעניינו אי-הלוקליות בשזירה קואנטית.
    י.ש: אולי תתנחם בכך שהכל עובר עם החיים.
    ר.ר: אתה בחברה טובה ורחבה של אנשים שלא מתרכזים בהסברים שלי. כאמור, חסכתי מכם את ההגדרות הבסיסיות (שמתחילות מה”אין”) והצעתי לקבל את ההגדרות הנוכחיות כאקסיומה. כמו כן, אתה אמנם מדייק לחלוטין בהגדרות הפיסיקליות שאתה מביא, אך אני לא עובד עם פיסיקה ולכן ההגדרות שלי אחרות. בסופו של דבר ההגדרות שלי אמורות להסתדר עם ההגדרות הפיסיקליות. ולשאלותיך האחרונות: לפוטון יש שני מופעים, גל וחלקיק; כגל, קוטרו מאד גדול; כחלקיק, קטן. “חלל ריק” הוא נפח שאינו תפוס על ידי חלקיק צפיד.

  477. סטודנט
    עד שהוא יסיים לבדוק כבר יוכלו למדוד את הפוטונים באנדרומדה.

    יובל
    במתמטיקה- משתמשים באותיות כדי לסמל משהו. “a” בתור איבר למשל, “i” בתור מספר מדומה, “E” בתור מושג כלשהו. סתם לדוגמה.
    במתמטיקה קיימת סמנטיקה שונה מזו שקיימת בספרות למשל, וחבל שאינך מבדיל בינהן.
    “מה שקובע את התדירות של פוטון” הוא היחס בין מהירות הפאזה לבין אורך הגל שלו.
    ולא “הקוטר (הממוצע) של חלקיק” (אפילו שבשבילך זה מסתדר יופי עם חלקיקי הצפפחת : )).
    דרך אגב, מהו הקוטר של פוטון, אתה יודע?
    עוד כתבת “האלקטרון הוא שכבה דקה של חלל ריק”.
    שוב פעם, מהו “חלל ריק”?
    איך אתה מגדיר “חלל ריק”?

  478. יובל
    אוי סליחה, חשבתי שאיתי דברת ביומיים האחרונים, אבל עכשיו שמתי לב באמת שזה היה עם רפאים. התבלבלתי, כנראה בגלל האלצהיימר.

    בכל אופן – פטור זה פטור. אם תידרש לי, ציין. למרות שכפי שבטח כבר הבנתם שניכם מזמן, אני לא ממש מבין על מה אני שח, כך שאין פה אבידה גדולה.

    ישראל הטיפש מטופש.

  479. ועוד כמה נושאים שכדאי לבדוק אם המודל שלך מסביר:
    1. הטבלה המחזורית.
    2. כיצד פועלת תהודה מגנטית גרעינית (NMR) ותהודה מגנטית אלקטרונית (ESR)?
    3. כיצד מתרחש עיבוי בוז איינשטיין ולמה פרמיונים לא עוברים אותו?
    4. למה מולקולת מימן מורכבת משני אטומים ולא 20?
    5. איך עובדים מל”מים?
    6. פיזור ראמאן.

  480. עוד דבר שכדאי לבדוק: האם בעזרת המודל שלך אתה יכול לחשב (אפילו בקירוב) מהי התדירות של אור הנבלע ע”י, למשל, מולקולת בנזן.

  481. סטודנט, טכניון,
    יתכן. טרם בדקתי. מה שקובע את התדירות של פוטון הוא הקוטר (הממוצע) של חלקיק, הצפיפות המקומית הממוצעת, תדירות הרטט הממוצעת וכן הלאה. את הגדלים האלה אפשר לחשב על פי תצפיות בודדות ולבדוק אם הם מתאימים גם לספקטרום רחב.

  482. ישראל!
    שכחת שאתה פטור מהתייחסות אל המודל שלי?
    הבעיה שיש לי איתך היא שאתה ישר שועט קדימה בלי להבין את היסודות. אני לא רק קורא לאט. אני גם כותב לאט. אם לא תתאזר בסבלנות לא נוכל לדבר.

  483. ר.ח רפאי.ם,
    אותיות הם סמלים לכל דבר ועניין.
    אל הפיסיקה הקוונטית אגיע בקרוב, אם יסתייע בעדי. אבל ארמוז משהו כבר כעת:
    האלקטרון הוא שכבה דקה של חלל ריק המקיפה את הפרוטון מכל אבריו. כאמור, פוטון, הוא פיסת חלל ריק בגודל מסוים (המשתנה בהתאם לכמה גורמים). כשפוטון נקלע אל אלקטרון הוא תורם להגדלת נפחו. האלקטרון שנפחו גדל “שואף” לשמור על עוביו הדק ולצורך כך מגדיל את קוטרו. אם הקוטר החדש הוא בדיוק כזה שמתאים למעטפת דקה מושלמת, אז האלקטרון מתייצב ברמה החדשה. אם לא, אז הוא משחרר בחזרה פוטון. אנחנו מודדים פוטונים על פי תדירותם (צבעם) ומוצאים שיש תיאום בינה לבין משהו שאנחנו מכנים “אנרגיה”. אבל על זה לא כעת.

  484. יובל
    סופסוף!

    כמה שאלות עולות מהמודל.

    1. האם אין זה מתחייב שכדי שתיווצר גרביטציה, לא יתחילו חלקיקים צפידים מצידה השני של המושבה למות גם כן, ועיי כך יווצר איזון שימנע מהמושבה – הפרוטון – לנוע דווקא בכיוון מסוים?

    2. מה גורם לאותו מוות? איך יוכל פרוטון על הירח להשפיע על פרוטון על הארץ?

    3. האם הייתי אמור לנחש כל זאת?

  485. יובל

    מתמטיקה מנסחים ב-סמלים ולא ב-אותיות.

    בוא ננסה שיטה אחרת.
    תגיד לנו איך הפיזיקה הקוונטית משתלבת עם המודל שלך.
    כלומר, באיזה שלב, מהו הגורם/גורמים שמחברים בין המודל שלך לבין הפיזיקה ברמה הקוונטית?

  486. המודל של יובל, המשך. מכאן ואילך הכל מתמטיקה, בעיקר גיאומטריה והסתברות.
    פרוטון, שני פרוטונים, גרביטציה:
    משום שהחלקיקים הצפידים של החומר האפל (להלן “חלקיקים”) מתרבים בהינתן צפיפות מקומית נמוכה ומתים כאשר הצפיפות המקומית גדולה מעבר לסף מסוים, הצפיפות בה הם מסודרים מתנדנדת בטווח מסוים סביב גודל קבוע. נכנה את הגודל הזה בשם “ממוצע”. בהנחה שחלקיק תופס נפח סופי, הרי במותו “שיחרר” החלקיק מנה (“קוואנט”) של חלל ריק באותו גודל. הצפיפות הממוצעת קטנה בבת אחת בהתאם לנפח ששחרר החלקיק במותו. בצפיפות הנמוכה הזו גדל הסיכוי שייוולד חלקיק חדש וחוזר חלילה. בכל רגע נתון ישנם אזורים בהם הצפיפות גבוהה מן הממוצע ואזורים בהם הצפיפות נמוכה מן הממוצע.
    נתבונן כעת באוסף של חלקיקים הנמצאים בצפיפות גבוהה מן הממוצע. אפשר לדמות את האוסף הזה למושבה (כמו של בני אדם או של חידקים), ולכן גם נכנה אותה כך. הצפיפות המרבית של המושבה מווסתת על ידי מות החלקיקים ואינה יכולה לעלות מעבר לגודל מסוים; מאידך, מעבר למרחק כלשהו מן המרכז הגיאומטרי של המושבה, הצפיפות של החלקיקים הינה הצפיפות הממוצעת. חלקיקים עוזבים את המושבה אל הסביבה וחלקיקים מן הסביבה מצטרפים אליה ומתקיים שיווי משקל דינמי. במרחק מסוים מסביב למרכז המושבה קיימת מעטפת בה המספר הממוצע של העוזבים ליחידת זמן שוה למספר המצטרפים. את המעטפת הזו נגדיר להיות “הגבול של המושבה”.
    אם המושבה סימטרית, דהינו הצפיפות הממוצעת המקומית זהה בכל נקודה סביב המעטפת שלה, שיווי המשקל הדינמי יביא לכך שהמושבה לא תשנה את מיקומה למרות שהיא מחליפה חלקיקים עם סביבתה. אולם אם באזור אחד שלה הצפיפות הממוצעת המקומית גבוהה מזו שבאזור אחר שלה הרי באזור הצפוף יצטרפו אליה יותר חלקיקים מאשר באזור הדליל ובאזור הדליל יעזבו אותה יותר חלקיקים מאשר באזור הצפוף. כך, במהלך חילוף חלקיקים עם הסביבה, המושבה תנוע אל האזור בעל הצפיפות הגבוהה יותר.
    בהינתן שתי מושבות שקיים ביניהן מרחק סופי כלשהו, האזור שבין המושבות יהיה צפוף יותר מן האזורים האחרים. משום כך ינועו המושבות זו לקראת זו ותסתמן ביניהן משיכה. למשיכה הזו נקרא, להלן, “גרביטציה”, ולמושבות נקרא “פרוטונים”.

  487. הדוגמה עם הטבעת היא לא מוצלחת, כי האפקט של השראת הכוח המגנטי על גוף היושב במרכז הטבעת לא מושג בגלל סיבוב הטבעת אלא בגלל שדה חשמלי משתנה שאותו אפשר להשיג גם בסטטורים של מנועים שכפי שמרמז שמם הם סטטיים.
    זכותך להסיק מסקנות רק מן השורה האחרונה של התופעות, אך זה יזכה אותך לכל היותר בפרס נובל לספרות.

  488. יובל.
    אין לי מושג מתי תוכל להבין למה אני מתכוון. אני יכול להסביר כמיטב יכולתי, אך אין לי שליטה על מה מבינים אנשים מדברי.

    אני רק יודע שכשאני אינני מבין משהו, אני מנסה לקרוא שנית, או מבקש הבהרות. אינני מקיש מיד שהצד השני “מנסח את הדברים שלו בצורה כל כך לא מוצלחת”, או “מערב מין בשאינו מינו”.

    לדעתי, הדוגמה עם הטבעת היא מצויינת, וממחישה את הרעיון המרכזי של הקשר בין כוח משיכה ואינרציה. כידוע, הכוח האלקטרומגנטי חזק בהרבה מכוח הגרביטציה. לכן דוגמה זו מהווה הדגמה של “דחיסת” חומר כדי להגביר את כוח הגרביטציה, כפי שהסברתי ב
    https://www.hayadan.org.il/astronomers-reach-new-frontiers-of-dark-matter-130112/#comment-329007

    מבחינתו של גוף, אין הבדל אם הכוח הפועל עליו הוא כוח משיכה, חשמלי או מגנטי. לכן אם יסתובב גוף טעון חשמלית בגלל סיבובה של הטבעת, רק טבעי להסיק שגוף יסתובב בגלל סיבובו של “צמיג” כבד מאד. זה גם מה שטוען אלברט.

    ובעניין המודל שלך. אמרתי כבר כמה פעמים שאינני יכול להגיב עליו כי אינני יודע מהו. אם, כטענתך, רק קשיי תפיסתי עומדים למכשול בפני ההבנה כיצד פועלות הגרביטציה ואי הלוקליות במודלך, ואילו אנשים אחרים דווקא הבינו היטב, אולי יועילו אותם מבינים להסביר לי, ואז אוכל להתייחס.

    בינתיים, עד שתוכל להגיב עניינית לדברי בנושא עקרון מאך, איאלץ להסכים עם איינשטיין (ע”ע) מאך (ע”ע) ושימה (ע”ע).

  489. ישראל! מה יהיה? מתי אוכל להבין למה אתה מתכוון?
    כוחות הם כוחות הם כוחות, אבל מה שנראה לי רלוונטי כאן הוא לא הכוחות אלא המקור שממנו הם נובעים. לפי דעתי, הקשר הקיים בין מטען חשמלי למטען גרביטציוני הינו קשר עקיף בלבד. תוכל להקיש ולהגיד שיש לשניהם מספר מאפיינים דומים, אבל זה לא מצביע בהכרח על מקור משותף. [על פי המודל שלי, הדמיון נובע מכך שהתווכים בהם הם עוברים (תווך החלקיקים הצפידים ותווך פיסות החלל הריק) משולבים זה בתוך זה, אבל אתה פטור מלהתייחס אליו].

  490. “ובהזדמנות שאני כבר כאן, רציתי גם לציין שמטען חשמלי איננו מטען גרביטציוני.”
    מה באמת? איך זה שלא ספרו לי? ומה הקשר? האם איננו מדברים פה על כוחות?

  491. ישראל! אתה מאד יצירתי. שָׁפּו. יכולתך לערב מין בשאינו מינו עולה הרבה מונים על שלי.
    ובהזדמנות שאני כבר כאן, רציתי גם לציין שמטען חשמלי איננו מטען גרביטציוני.

    התגובה הקודמת ממתינה לאישור. שתמתין

  492. ישראל! אתה מאד יצירתי. שָׁפּו. מידת השעטנזיות שלך עולה כמה מונים על שלי.
    ובהזדמנות שאני כבר כאן, רציתי גם לציין שמטען חשמלי איננו מטען גרביטציוני.

  493. בסדר, לא נעלבתי.

    אני דווקא מחפש לשמוע את האיפכא. אבל אם אפשר, קצת הסברים, דוגמאות, נוסחאות, לינקים (לא לחתולים).

    קצת קשה לי לקבל טיעון בנוסח: מאך – יוק. אקסיומטית.

    לא נראה לך שאם נארגן טבעת הטעונה במטען חשמלי, הרי שאם נשים במרכזה גוף טעון הוא יושפע מסיבוב הטבעת? חוץ מנושא הקרינה, מה למעשה ההבדל בין הכוח שמפעילה הטבעת על הגוף והכוח שמפעילות מסות היקום על מסה מסתובבת במרכז, רק בהיפוך תפקידים?

  494. יובל הדיסלקט.
    ביג דיל, דיסלקציה. מה אגיד אני? גם דיסלקציה, גם אמנזיה, גם ADD, גם פרקינסון.
    אנחנו מזדקנים יובלי, עוד מעט אנחנו כבר בני 30.
    עוד הפעם התבלבלתי, בגלל הסניליות?
    נכון, נכון. 20.

    בכל אופן. נוסחת איינשטיין הובאה מכיוון שכתבת: “שמא יואיל השפירא להסביר לבור ועם הארץ שכמותי למה התכוון המשורר באומרו “האינרציה קשורה לכל המסות ביקום בקשר נוסחתי ליניארי פשוט”?”
    והיא דוגמה לקשר כזה. זה הכל. היא איננה קשורה לסיפורינו. רק דוגמה. דוגמה לקשר ליניארי פשוט. דוגמה. לקשר. ליניארי. פשוט. דוגמה.

    מה שכן קשור ועוד איך היא הנוסחה GM=RC^2. היא אפילו אינה ליניארית – היא יותר נקודתית, כמו נקודה שלנו. רק קבועים. אך יש משהו מפליא למדי בנוסחה זו. אם תערוך לה אנליזה מימדית – ז”א תצמצם את כל המטרים השניות הקילוגרמים והמספרים משני צידי המשוואה, תשאר בסוף רק עם ניוטונים מצד שמאל ומכפלה של קילוגרמים במטרים מחולקת בשניות בריבוע.

    או בקיצור:

    F = MA

    החוק השני של ניוטון, חוק האינרציה.

    זה איננו מקרה. (למרות שכמובן ניתן לשנות אברים במשוואה ולקבל לכאורה קשר אחר). הסיבה לכך היא הדרך בה הגענו למשוואה.

    הדרך מתוארת בחלקה בלינק שצרפתי לכתבה של פרופ וודווארד. הוא הפנה אותי בזמנו לספר של דניס שימה ( בריטי ממוצא יהודי סורי, מתלמידיו של דיראק, קיימברידג’, אחד מאבות הקוסמולוגיה המודרנית, זה בסדר, אני יודע שאתה לא שם קצוץ.). שימה היה מחסידיו הגדולים של עיקרון מאך, ולמעשה מבחינתו לפחות, די הוכיח את הזיקה ההדוקה בין המסה ביקום לאינרציה לפי עקרון מאך. ומכאן נוסחתינו.

    ואם נוסחאות, קצת קשה להתווכח. במיוחד נוסחה כזו, שבאגף השמאלי שלה מסת היקום, ובאנליזה המימדית שלה מקבלים את ניוטון חוק ב’, והפיתוח שלה מושתת על עקרון מאך.

    האם זה אומר משהו? לא בהכרח. אבל נראה לי סביר.

    מה שיותר חשוב אולי, שחזרת קצת להיות יובל הנחמד. חששתי שתפתח עלי פה – ויצאת מברך, כבלעם בשעתו.

    זהו. זה כל הסיפור שלי.

  495. ישראל!
    עדיין אינני בטוח שהבנתי. היה נדמה לי שחיפשת קשר נוסחתי ליניארי פשוט בין אינרציה של גוף בודד לבין כלל המאסות ביקום, אבל כעת אתה מכניס למשוואה את E=MC^2.
    התכוונתי להשאיר את מלאכת ההתדיינות איתך למאיר. לא יודע מה עבר עלי שבגללו תחבתי את חוטמי לדיון לא לי. אנא סלח ומחל.

  496. ישראל!

    אם שכחת, הריני להזכירך כי אני דיסלקטי. רק אם מסבירים לי לאט אני מצליח להבין מהר. כדי להמחיש עד כמה נואש מצבי, אבקשך להאמין לי כי המשפט “האינרציה קשורה לכל המסות ביקום בקשר נוסחתי ליניארי פשוט” לא עובר את סף ההבנה שלי. כאמור, התגובה המיידית שלי היתה להגיד לך “לא”. אבל חששתי שבזה אחטא לך, משום שאולי היה בשאלתך משהו חכם שאני החטאתי בגלל המגבלויות הקוגניטיביות הפרטיות שלי. רק בגלל בושה וגאוה יתירה העדפתי להציג את הדברים כאילו האשמה היא בך, אך האמת היא כי אתה כותב רהוט וברור.

    נ.ב עדיין לא הבנתי למה בדיוק התכוונת.

  497. מאיר.
    שים לב למה שכתוב מעל התגובה שלך, בתגובה שלי:

    אם:
    G = קבוע הגרביטציה.
    C = מהירות האור.
    M = מסת היקום המשוערת. (אפשר למצוא בוויקיפדיה).
    R = רדיוס היקום המשוער. (כנ”ל).

    אז: GM=RC^2 בקירוב.

    מדהים לא? ומה שלא פחות מדהים הוא שאם נשתמש ב dimensional analysis, אחרי כל הצמצומים משני צידי המשואה, נשאר רק עם:

    F = MA

    החוק השני של ניוטון, חוק האינרציה.

    הנוסחה GM=RC^2 נגזרה ( על ידי) מפיתוח של פרופ וודווארד ודניס שימה. הפיתוח שלהם נעשה עיי אינטגרציה המבוססת על עקרון מאך. חלק ממנו תוכל למצוא ב

    http://physics.fullerton.edu/~jimw/kill-time/

    אני מבין שלדעתך מסת היקום נמוכה פי 4 מהמקובל. אך עצם העובדה, שאינטגרציה שנעשתה על סמך עקרון מאך, מביאה לחישוב המסה המקובלת של היקום, מראה לדעתי שאי אפשר להתעלם מעקרון מאך.

    האם אתה מכיר את התאוריה של RAYMOND GRAUDIS?

    אגב, חשבת פעם שמעניין לשאול איך זה יכול להיות שנוסחת הקשר בין רדיס למסה זהה אצל חור שחור והיקום? זה בטח לא נראה שאנחנו חיים בתוך חור שחור!

    אני צריך ללכת, נדון כשאשוב.

  498. ישראל,

    כתבתי שם למעלה שאני מפקפק בחשיבות הקשר הזה. כלומר, לא נראה לי שאפשר להפיק ממנו איזושהי אינפורמציה שחסרה לאנושות.

    אם לתמצת אותו למשפט אחד, הקשר הזה טוען שמהירות ההימלטות מהיקום היא בערך מהירות האור.

    אבל את זה שהיקום הוא סוג של חור שחור האנושות כבר יודעת משחר נעוריה : )

  499. יובל, מאיר.
    כרגיל, “התגובה ממתינה לאישור.” איך זה שאני לא מופתע?

    עד תום החקירה, אני לוקח את כלאבי להליכה. בינתיים, אם אפשר הסתכלו ב

    https://www.hayadan.org.il/astronomers-reach-new-frontiers-of-dark-matter-130112/#comment-326922

    והגידו לי מה פשר הנוסחה המופיעה שם. (מאיר, זה הבלוג של החברה שלך).

  500. ר.ח.

    כדאי לחכות עד סוף הבדיקות.
    בינתיים, ראה הערה מלפני שבוע ב:

    https://www.hayadan.org.il/astronomers-reach-new-frontiers-of-dark-matter-130112/#comment-327662

    “לפי הפרוש הזה, יתכן שהבעיה באופרה בסרן היא בכלל בציוד הקצה, המורכב מרכיבי ציוד (חלקיקי יסוד לדוגמה) הנעים במהירות גבוהה בכיוון תנועת הנייטרינו.”

    זה בהחלט כולל מתנדים.

    יובל.

    אתה קרוא לסדר.

    התחייבת ב:

    https://www.hayadan.org.il/astronomers-reach-new-frontiers-of-dark-matter-130112/#comment-328506

    לא לפגוע אישית במתכוון במגיבים, אלא כתגובה על פגיעה אישית בך.

    מה שסותר את דבריך:
    ” הוא מנסח את הדברים שלו בצורה כל כך לא מוצלחת עד שקשה לי להבין מה הוא בדיוק רוצה לגיד.”

    קח אחריות. זה שהתקשת בהבנה, עדיין לא אומר שהדברים מנוסחים בצורה לא מוצלחת. מאיר הבין מיד, כך נראה לי.

    איך היית מנסח אתה את הקשר בין מסה לאנרגיה בנוסחת איינשטיין? תוכל לחשוב על ניסוח טוב יותר מ “קשר נוסחתי ליניארי פשוט”?

    אם כן, זו ההזדמנות. נסח.
    🙂

  501. בוקר טוב לוס אנחלס, אחה”צ טוב עיר הקודש,
    מאיר! מן המותן אני אומר שמהמודל שלי אפשר להסיק את המודל שלך. לכן, על פניו תשובתי לישראל היא חד משמעית “לא!”. אבל הוא מנסח את הדברים שלו בצורה כל כך לא מוצלחת עד שקשה לי להבין מה הוא בדיוק רוצה לגיד. שמא יואיל השפירא להסביר לבור ועם הארץ שכמותי למה התכוון המשורר באומרו “האינרציה קשורה לכל המסות ביקום בקשר נוסחתי ליניארי פשוט”?

  502. ישראל,

    שאלת את יובל לפי המודל של יובל. אני לא בקי במודל שלו כדי שאוכל לענות לפיו.

    דעתי הפרטית ברורה. אין שום קשר, לא לינארי ולא בלתי לינארי, בין התכולה המסתית של היקום לבין אינרציה.

  503. ישראל,

    “1…נוכל לדעת באיזו מהירות זוויתית מסתובבות הדסקיות, אפילו בחדר אטום. השאלה היא עדיין: מסתובבות יחסית למה? איינשטיין אומר – יחסית למרחב זמן המוחלט. אולם אם הבנתי נכון את טענתך, האינרציה מקורה בהזזת הגוף ממרכז הגרביטציה שלו עצמו. אם כך, איך נוכל לאמר שדסקית א’ מסתובבת במהירות של 5 סיבובים לשניה ודיסקית ב’ 7? יחסית למה?”

    ניקח את כל החלקיקים האלמנטרים לאורך קו רדיוס דמיוני כלשהו של דיסקית, שאני טוען שמהירות הסיבוב שלה אבסולוטית, ונמדוד עבור כל חלקיק כזה מה מידת האסימטריה של שדה הגרביטציה העצמי שלו ביחס לנקודת המרכז של החלקיק. בדיסקית שאיננה מסתובבת לא נגלה אסימטריה כזו באף אחד מן החלקיקים לאורך הקו. אם נרצה, נוכל להשתמש כיחוס עבור כל הדיסקיות האחרות סביבה. בדיסקית מסתובבת נגלה שהאסמטריה הולכת וגדלה ככל שמתרחקים מהמרכז לאורך הרדיוס.
    ככל שהאסימטריה במרחק נתון גדולה יותר, כך הדיסקית מסתובבת מהר יותר.

    בהסבר הנ”ל מקופלת גם תשובה להערה של יובל.

    סיבוב אבסולוטי יחסית למה? יחסית למרחב. לא יחסית למרחב-זמן המוחלט (אין חיה כזאת), אלא יחסית למצב הקודם של המרחב המשתנה (שניתן להתייחס אליו כמוחלט בכל רגע נתון, עד השינוי הבדיד הבא שיחול בו, וחוזר חלילה).

    “2. ניקח את הדוגמה (המצויינת) של יובל על צמיג שבמרכזו דלי עם מים. נכון, כשהצמיג במנוחה, הכוח הגרביטציוני בתוך השטח הכלוא על ידו שווה ל-0. אך אני מאמין שזה שונה כשהצמיג מסתובב יחסית לדלי. אם נוכל לדחוס חומר לריכוז של 1000 טונות לסמ”ק, ומהחומר הזה נרכיב את הצמיג, הרי שצמיג מסתובב שכזה שרדיוסו הפנימי נגיד 2 מטר והחיצוני 2.5, ישקול מליונים רבים של טונות, ולכן אם נקבע את הדלי, נראה שהמים מסתובבים ומתקערים. אותו הדבר יקרה אם הצמיג ישאר במנוחה והדלי יסתובב.”

    לא נכון. כל חלקיק לעצמו. כל חלקיק גורר איתו שדה כבידה עצמי בעל אסימטריה בכיוון התנועה, אסימטריה שבמהירויות קונבנציונליות מידתה יחסית למהירותו הקוית. אין כל השפעה הדדית בין המים בדלי לבין הצמיג. אלברט טען שיש frame dragging. אין חיה כזאת. חלקיקים אלמנטריים מכירים רק את עצמם ואת מה שגובל בהם פיזית. בהתעלם מתופעת התפשטות המרחב (לצורך הדיון, ומכיוון שהשפעתה על הסביבה המיידית של חלקיק אלמנטרי היא אפסית), הסביבה המיידית שחלקיק אלמנטרי חווה היא תמיד סטטית. חלקיק אלמנטרי לא יודע ולא יכול לדעת שמשהו במרחק עשרה סנטימטר (או מיקרומטר) זז. הדבר היחיד שהוא “יודע” זה אינפומציה סטטית שהוא רוכש מהסביבה המיידית ברגע נתון במחזור הפעולה שלו. על מנת שאלקטרון נתון E בדלי המים “יֵדע” שהגלגל הדחוס מסתובב, צריך שסך התרומות של כל החלקיקים האלמנטריים המהווים את הגלגל הדחוס, יגרום לסביבה המיידית של אלקטרון E להיות אסימטרית בכיוון הסיבוב. לא תהיה אסימטריה כזו מכיון שסך כל תרומות האסימטריה המקומית בשדות הכבידה העצמיים של חלקיקי הגלגל בתנועתם הוא אפס ביחס למיקום של אלקטרון E וביחס למיקומו של כל חלקיק אלמנטרי אחר המהווה את המים.

  504. מי יבצע את הניסוי, אני? מי אני, מה אני, אנא אני בא, אנא ערף.

    זה איינשטיין הנודניק עוד הפעם עם השטויות שלו. צריך לדבר עם נקודה ואבי, שיקראו אותו לסדר.

    אני צריך ללכת לישון עוד מעט. אבל יש לי שאלה פשוטה וישירה בענייני אינרציה, ואשמח אם אוכל לקבל ממך תשובה פשוטה וישירה חד וחלק בקשר לתמונת המודל שלך:

    1. האם נראה לך שהאינרציה קשורה לכל המסות ביקום בקשר נוסחתי ליניארי פשוט?

    2. אם התשובה על 1 חיובית, האם אין זה בדיוק עקרון מאך?

    אשמח אם אוכל לקבל תשובה של כ ן או ל א.

  505. ישראל,
    יפה. מצאת לעצמך ניסוי פשוט לעריכה. אנא בצע אותו. קבע מצלמה על הדיסק המסתובב, ובודאי תקבל עוד אישוש לתורת היחסות. אני מזמין כובע דל קלוריות.
    וכידוע, חיפוש המטבע האבוד מתחת לפנס זה מבצע קל בהרבה מאשר חיפושו במקום בו באמת אבד.

    אך בדבר אחד אני איתך באש ובמים: הדיון במקור האינרציה עדיין פתוח.

  506. נקודה, שכנע את אבי שיאסור על דיונים בנושא. בינתיים נראה לי שנמשיך.
    מה עם הלינקים שהבטחת להסבר לאי לוקליות בשזירה קוואנטית? העולם מחכה ומצפה.

  507. למי שעדיין סבור שאולי כל הדיון הזה עקר, מכיוון שיש איזו הסכמה כללית על מקור הגרביטציה והאינרציה, וכל מה שצריך זה קצת לשבת וללמוד, מומלץ להסתכל בלינק:

    http://www.enotes.com/topic/Inertia

    ומתוכו:

    Source of Inertia

    There is no single accepted theory that explains the source of Inertia. Various efforts by notable physicists such as Ernst Mach (see Mach’s principle), Albert Einstein, D Sciama, and Bernard Haisch have all run into significant criticisms from more recent theorists.

  508. יובל.

    הצץ ב

    http://www.padrak.com/ine/INERTIA.html

    וראה:

    To test the idea, Einstein considered a small body at rest inside a rotating shell of
    material in otherwise empty space. Using his theory of relativity, he calculated what would happen. It turns out that the body should indeed feel a tiny gravito-magnetic force

    נראה לי שאני מבין בהחלט מה אומר מאיר. פשוט הרבה יותר קל לעבוד עם דוגמאות סיבוב מאשר תנועה ליניארית.
    .

  509. נקודה, צדק רב בדבריך.
    אכן, האינטואיציה שלנו מוגבלת. לכן, כדי להבין את העולם הגשמי עלינו להרחיב את החשיבה שלנו אל מעבר לגבולות האינטואיציה

  510. כל התאוריות הפיסיקליות היום מניחות “שולחן עבודה” שעליו מתרחשות התופעות. אותן תאוריות לא מסבירות את המבנה של אותו שולחן עבודה, כיוון שאין יחסית למה להסביר אותה.
    לכן ברור ששום תאוריה שיש כיום לא יכולה להיות שלמה.

    ושנית, זה גם אומר שאותו הסבר ראשוני מוכרח שהוא יהיה תלוי בכלום, ומכיוון שניתן להוכיח שאנו לא יכולים לחשוב על דבר שכזה (בקצרה, החשיבה שלנו היא אסוציאטיבית ומקשרת בין דבר אחד לשני, ולכן לא ניתן לחשוב על משהו שהוא לא קשור למשהו אחר), אז מזה נובע שלעולם לא נוכל להבין את המציאות כמות שהיא.
    העניין הזה פשוט להבנה, וקאנט סתם בלבל את כולם כשהוא כתב כל כך הרבה על הנושא הזה. העניין פשוט.

  511. ישראל! בסדר, בלעתי את הפתיון [“הדוגמה (המצויינת) של יובל”].
    הדוגמה שהבאתי (בצחוק, אגב) היא ממש לא מצוינת ובכלל לא מוצלחת, כי הגליל, ולא חשוב מה משקלו, יפעיל את אותה גרביטציה בין אם יסתובב ובין אם יתבטל.

    ומכיון שאני כבר כאן, אז אוסיף כי נראה לי שלא הבנת את טענתו של מאיר. הוא דיבר על שדה כבידה עצמי של פרוטון בודד ועל תנועה בקו ישר, ואילו אתה מדבר על דיסקית מרובת פרוטונים. כדי לעבור מתנועה לינארית של פרוטונים לתנועה סיבובית של דיסקית, יש לערוך תיקנון מתאים.

  512. יובל.
    אין בעיה. ידע אותי אם אתה מעונין בתגובות בעתיד. אתה בהחלט מוזמן להמשיך להגיב לתגובותי.

    מאיר.
    אני מבסוט מהענייניות שלך. עזרת לי להבין איך יכול איינשטיין להתכחש לעקרון מאך. אבל עדיין לא ברורים לי כמה דברים ברעיון שלך. ברשותך, נמשיך עם השאלות.

    1. גם לפי דבריך, נוכל לדעת באיזו מהירות זוויתית מסתובבות הדסקיות, אפילו בחדר אטום. השאלה היא עדיין: מסתובבות יחסית למה? איינשטיין אומר – יחסית למרחב זמן המוחלט. אולם אם הבנתי נכון את טענתך, האינרציה מקורה בהזזת הגוף ממרכז הגרביטציה שלו עצמו. אם כך, איך נוכל לאמר שדסקית א’ מסתובבת במהירות של 5 סיבובים לשניה ודיסקית ב’ 7? יחסית למה?

    2. ניקח את הדוגמה (המצויינת) של יובל על צמיג שבמרכזו דלי עם מים. נכון, כשהצמיג במנוחה, הכוח הגרביטציוני בתוך השטח הכלוא על ידו שווה ל-0. אך אני מאמין שזה שונה כשהצמיג מסתובב יחסית לדלי. אם נוכל לדחוס חומר לריכוז של 1000 טונות לסמ”ק, ומהחומר הזה נרכיב את הצמיג, הרי שצמיג מסתובב שכזה שרדיוסו הפנימי נגיד 2 מטר והחיצוני 2.5, ישקול מליונים רבים של טונות, ולכן אם נקבע את הדלי, נראה שהמים מסתובבים ומתקערים. אותו הדבר יקרה אם הצמיג ישאר במנוחה והדלי יסתובב.

    האם לא קיבלנו “עקרון מאך” בזעיר אנפין?

  513. ר.ח רפאי.ם,
    שאלת אם אני הוא זה שהגדיר את החלקיקים הצפידים ואת החלל הריק שסביבם. האמת היא שכן, אך עלי להסביר משהו. בהתחלה, לפני ארבעים שנה, חיפשתי מודל “פשוט”, דהינו שלא יכיל הרבה הנחות (אז בדיוק סיפרו לי על התער של אוקהם, וזה מצא חן בעיני). הגדרתי את החלקיקים באופן שהצגתי כאן. עם השנים הבנתי כי ההגדרה הזאת מסורבלת מאד וחיפשתי הגדרה פשוטה ממנה. לשם כך “גייסתי” את השלילה ואת יצירת ה”יש” מן ה”אין” לבדו ללא התערבות גורם חיצוני. לא ראיתי אותם בשום משוואה. את התיאור (המתמטי) בניתי בעצמי. האמירה “הפיסיקה לא קיימת” איננה תירוץ, משום שבמודל הזה אני מתיימר לבנות את הפיסיקה.
    תורם של הציורים הסכמאטיים עוד יגיע. אני יכול, למשל, להעלות אותם לרשת כמסמך של גוגל ולשתף את כל העולם.
    אתה אומר שאפשר לחשוב שהשפה זה הדבר היחיד שהיה קיים באותו רגע בראשיתי. זה נשמע יפה, אך לשאלה הזו לא נכנסתי. מתמטיקה בכלל זו שפה, והיא מתפתחת בהתמדה.
    ולשאלה “סתם” שלך, שנראית לי חשובה יותר מן האחרות: “איך ניתן להפריך את התאוריה?”:
    היות שהמודל הוא בראש ובראשונה מתמטי, אפשר לבחון אותו בכלים מתמטיים. למשל, אם אני “ממציא” מתמטיקה חדשה הרי זה לא מריח טוב. לאחר שלב התיאור הפלסטי מגיע שלב ההגדרות ואחריו שלב הנוסחאות והשוואה עם התצפיות בטבע. אז כבר יהיה קל למצוא הרבה נקודות תורפה. צפויה לך מלאכה רבה 🙂

  514. ר.ח,
    הקדמה “מיתולוגית” קצרה: היקום התחיל (בלי “נניח ש”) מלא כלום, “אין”. זאת, משום שאם נניח שהיקום התחיל ממשהו, אז גם לאותו משהו יש למצוא התחלה. יש המעדיפים להציג את היקום כדבר נצחי ולומר “החומר תמיד היה”. לטעמי זו לא שגיאה גסה, משום שאת המושגים “נצח” ו”תמיד” אפשר לראות כמהות הקיימת מעבר לזמן, וגם ה”לא כלום” הוא כזה כי כשאין כלום אז גם אין זמן.
    השאלה הראשונה שמיד מזדקרת לעין היא כיצד מן ה”לא כלום” נוצר משהו? וצמודה לה השאלה כיצד יש ביקום שונות כה גדולה? על כן, כדי שהטענה הזו לא תיתפס כאמירה ריקה מתוכן, יש להבין את המנגנונים בהם ה”אין” הופך ל”יש” וה”לא כלום” הופך ל”הרבה”.
    כפי שההנחה כי היקום החל מאיזשהו “יש” מסיטה את השאלה מ”יש” אחד ל”יש” (קודם) אחר, גם ההנחה כי יש בורא לעולם הינה הסטה בדיוק באותו אופן, כי הנחת קיומו של בורא (וזה עוד בלי לערב את השאלה התיאולוגית “תבוני או לא?”) מעלה את שאלת מקורו של אותו בורא. על כן ה”אין” שממנו נוצר ה”יש” מגלם בתוכו גם את המנגנון היוצר את ה”יש” מן ה”אין”, כלומר זה אותו “אין”.
    אין לנו דרך להכיר את ה”אין” הקדם-בראשיתי הזה, משום שאנחנו חיים בעולם של “יש”. אבל אנחנו יכולים לנסות להשליך על ה”אין” הראשוני ממה שידוע לנו מתוך היכרותנו עם תופעות יומיומיות מסוימות. וכאן מגיע ה”נניח ש”: נניח שה”אין” הקדם-בראשיתי הינו שלילה. ההנחה הזו אינה תלושה מהאויר. כשאנחנו מדברים על “אין” אנחנו מציגים אותו כשלילה של “יש”; “אין” הוא “לא יש”. כלומר, הוא לא רק איזשהו “ריק” ניטראלי כמו המספר אפס בפעולת החיסור (או חיבור) אלא הוא ריק פעיל כמו האפס בפעולת החילוק (או הכפל). הוא שולל.
    אין ל”אין” הקדם-בראשיתי מה לשלול חוץ מאשר את עצמו, כי חוץ ממנו אין שום דבר. מחיי היומיום אנחנו יודעים כי שלילת שלילה נותנת משהו חדש שאנחנו מכנים אותו “חיוב”. הנה הצגתי אפשרות למנגנון של יצירת היש מתוך האין ללא התערבות גורם חיצוני כלשהו, אך עדיין נותרה שאלת הרבגוניות הגדולה של התופעות ביקום.
    ה”אין”, השלילה, הוא גורם פעיל בעוד שה”יש” הוא בלתי פעיל. שוב אנו נזקקים להשלכות מן העולם המוכר לנו. אפשר, למשל, לראות את פעולת החלת ה”אין” על עצמו כהתפצלות (בחלוקה או בהכפלה – לא משנה). ה”אין” הקדם-בראשיתי התפצל לשני “אין”-ים. כל אחד מהם בפני עצמו הוא “אין” פעיל, אך שניהם ביחד מופיעים כ”יש” ניטראלי. אין מניעה מכל אחד מן ה”אין”-ים להתפצל כמו ה”אין” הקדם-בראשיתי וכך נוצרים הרבה “אין”-ים ללא הגבלה. לכל צמד “אין”-ם התנהגות השונה מהתנהגותו של “אין” יחיד ואנו מכנים אותו “יש”. שוב, בהשלכה מעולמנו, אפשר להסתכל על צירופים שונים של “אין” ו”יש” כשרשראות של “אין”, וישנן אינסוף שרשראות כאלה.
    מכאן ועד הגדרת תכונות החלקיקים הצפידים של החומר האפל, כמו גם של פיסות החלל הריק, עוד רבה הדרך, אך אני מקוה שהסקרנות האינטואיטיבית מצאה סיפוק מסוים.

  515. ישראל,
    עד להודעה חדשה ראה עצמך פטור מלהגיב לדבריי. גם אני אשתדל לא לענות לשאלות שתשאל כאן, גם למען ההגינות אך בעיקר משום שמאיר מבצע את המשימה היטב (לכל היותר, אוכל למצוא משהו להוסיף לדבריו אך איני רואה צורך להחליף אותו).

  516. ישראל,

    “שאלה 1: האם תוכל רק מהסתכלות בוידאוס לדעת מה מראה מד הכוח בדיסקיות? אל תשכח, למרות שברור שהמהירות הזוויתית היחסית בינן היא 2 סיבובים בשניה, איננו יכולים לדעת אם המהירויות הן 7 ו 5, 17 ו19, או אפילו 1+ ו 1-, או 0 ו2.”

    לא אוכל, אלא אם כן הוידאוס מראים גם תמונה של כוכבים, שידוע לי שהם רחוקים.

    “שאלה 2: האם נוכל לדעת מה מראה מד הכוח רק מהסתכלות בכוכבים?”

    כן

    “שאלה 3: אם התשובה על שאלה 2 חיובית, האם אין זה מראה על קואורלציה בין הכוכבים ואינרציה, או בקיצור, עקרון מאך?”

    זה מראה על קורלציה, אבל בהחלט לא על עקרון מאך. גם ניוטון טען שהכוכבים הרחוקים הם אינדיקטורים למצב המים בדלי, ובכל זאת אלברט קרא לעקרון “עקרון מאך”, מכיוון שבשונה מניוטון מאך טען שהכוכבים הם אלה ש”עושים את האינרציה” באופן מכני, ולא סתם אינדיקטורים המסמנים לנו את המרחב הריק הסטטי הנצחי והקיים גם בלעדיהם של ניוטון.

    אתה יכול להקשות על ניוטון איך הוא העז לחשוב שהכוכבים הרחוקים הסתדרו דווקא במנוחה יחסית למרחב. סתם במקרה?! על כך באה התשובה שלי, שזה לא במקרה. אני לא אהיה הדובר של ניוטון (שלא שיער שחוק הגרביטציה שלו קובע גבול עליון למהירות התנועה של מסות), אלא של עצמי: זה לא במקרה שהכוכבים הרחוקים הם אינדיקטורים של מנוחה יחסית למרחב. זה בגלל שמהירות התנועה שלהם במרחב מוגבלת. כשצופים למרחקים גדולים המהיר ביותר לעומד ייחשב. בכיכר העיר כולם שועטים סביבך. כשאתה צופה ממנה לקצה השדרה שבאופק, הכל שם נראה רגוע ונייח. זו לא פיזיקה זו גאומטריה.

  517. יובלי, אין לי כוח עוד הפעם לכל הריגשי הזה. ענה לי בכנות, כדי שנוכל להשאר ידידים:
    להגיב או לסתום?
    כי לא אוכל להגיב אם חסרים לי פרקים במודל.

    רפאים!
    קבל לייק לשיפור בסגנון הכתיבה והניסוח. ( זאת למרות הכיסוח). די גאה בך הפעם.

  518. ר.ח רפאי.ם,
    תודה. אמרת דברים של טעם ואני מתכונן להתייחס אליהם באריכות, יותר מאוחר היום או הלילה.

    ר.ח,
    את ההקדמה החשובה החלטתי לחסוך משום שהדיון בה גרם עיכוב רציני. אעמיק בעניין מאוחר יותר

    ישראל!
    כבר היינו צ’ילבאות פעם אחת. בוא לא ניגרר לזה שוב. אנא קבל בהבנה את העובדה שיש לי בעיות איתך

  519. מאיר.
    תשובה יפה, וכנראה גם נכונה, כי זה מה שגם אומר איינשטיין.

    חוסר ההבנה הוא אצלי. בוא נחדד את הבעיה.

    נאמר שיש לנו וידאוס משתי דסקיות המצלמות זו את זו בחלל.

    וידאו מס 1 ממצלמה מס 1 מראה את דיסקית 2 מסתובבת במהירות זוויתית של 2 סיבובים בשניה עם כיוון השעון.

    וידאו מס 2 ממצלמה מס 2 מראה את דיסקית 1 מסתובבת במהירות זוויתית של 2 סיבובים בשניה נגד כיוון השעון.

    שאלה 1:

    האם תוכל רק מהסתכלות בוידאוס לדעת מה מראה מד הכוח בדיסקיות? אל תשכח, למרות שברור שהמהירות הזוויתית היחסית בינן היא 2 סיבובים בשניה, איננו יכולים לדעת אם המהירויות הן 7 ו 5, 17 ו19, או אפילו 1+ ו 1-, או 0 ו2.

    שאלה 2: האם נוכל לדעת מה מראה מד הכוח רק מהסתכלות בכוכבים?

    שאלה 3: אם התשובה על שאלה 2 חיובית, האם אין זה מראה על קואורלציה בין הכוכבים ואינרציה, או בקיצור, עקרון מאך?

    נקודה.

    עשיתי עץ או פלי, יצא קוואנטים.

    הנה הלינק שאני מבקש:

    הסבר פיזיקלי לכיצד עוברת אינפורמציית הספין מאלקטרון א’ לאלקטרון ב’ במרחק שנת אור באפס זמן. הסבר פ י ז י ק ל י – לא “קריסת פונקציית הגל” (עצם מתמטי), לא “ראה מה שאמר בוהם” ולא 17 ממדים מדומים ביקום בעל טמפרטורה שלילית.

    אם תצליח למצוא לינק כזה, או להסביר בעצמך, תזכה לכבוד ויקר. כל מגיבי האתר ישיחו בך ורק בך. מפלגה תקום שססמתה: דה דה, נקודה. שלא לדבר על איזה כמה פרסי נובל קטנים מהצד.
    והתמריץ הכי גדול: אני אאלץ לסתום את פי הגדול – ולסתום אותו חזק, לתמיד.

    אך במידה ולא תצליח, יש לי רק בקשה אחת: קצת שקט. שבוע, לא יותר.

    אז מה זה יהיה: דה דה נקודה, או נקודה דא?

    ר.ח.
    מתפנן, אה? משאיר אותנו לעבוד, והולך לגלוש? אתה עושה מסלולים שחורים?

    יובל.
    אני עדיין בהמתנה לאינפורמציה חסרה. תוכל לשלוח בטלפורטציה?

  520. יובל,
    אתה רואה מה כתב רפא*ים? זה בדיוק מה שהתכוונתי לגבי ההקדמה שחסרה למודל שלך. אם אתה זורק אותו לפנים הוא מעורר תמיהה והרמת גבות מוצדקת.

    אתה חייב להקדים ולאמר “נניח שעולם מורכב מככה וככה חלקיקים צפידים ולא צפידים ו…..”
    ואז תיתן את ההנמקה לכל ההנחות המוזרות הללו:
    “באם נניח את כל אלה ונפתח את הרעיון מכאן נראה לפתע שכמה מהשאלות הלא פתורות היום כגון X Y Z מקבלות תשובה מעניינת ופשוטה ולכן יש טעם בכל הסיפור הנ”ל”

    אחרת כל זה הוא מיתולוגיה ולא מודל.

  521. יובל
    גררתם אותי שוב לענות לכם. טוב תראה, בחנתי בקצרה את המודל שלך ומה אני אגיד לך?
    נניח,שאלה “2) החלקיקים הצפידים רוטטים אקראית, דהינו נעים ללא הגדרת כיוון ומהירות אל תוך פיסות חלל ריק שסביבם.”
    או “(כי הוגדרו להיות צפידים).” – מי הגדיר אותם? אתה? אתה ראית אותם איפשהו?באזושהי משוואה? או שתירוץ כמו הפיזיקה לא הייתה קיימת הוא תירוץ מספיק טוב בשבילך?
    זה מסוג הטיעונים המנצחים שלך? גם אם תחליף את התמונה לאריק ובנץ ותקרא למודל שלך תורת המיתרים של יובל חייקין זה לא יעזור לך. אם זה המודל שלך אז נראה לי שהוא יצטרך לשלם שכירות למגירה שלך עוד 40 שנה.
    ואיפה הסכימה שלך? תבקש מהעורך הוא איש טוב בטוח שהוא יפתח בשבילך נישה שבה תוכל להעלות את הסכימה. כולם ישפטו לא רק החברים שלך.
    כשאתה אומר מה שאתה אומר אפשר לחשוב שהשפה זה הדבר היחיד שהיה קיים באותו רגע בראשיתי.
    סתם שאלה אם אתה רוצה אתה מוזמן לענות: (אפילו שעוד לא סיימת לספר לנו על הרעיון שלך) איך ניתן להפריך את התאוריה שלך?

  522. פואנטה חשובה ששכחתי:
    בסעיף הראשון משלושת הסעיפים שכחתי להוסיף שלחלקיקים הצפידים יש יכולת להתרבות וגם למות. בהינתן פיסת חלל ריק גדולה מספיק ליד חלקיק צפיד, נולד בה חלקיק צפיד חדש. אם חלקיקים צפידים מתקהלים בצפיפות רבה, חלק מהם נהפכים לחלל ריק. הגורמים לזה מפורטים בסעיפים קודמים שחסכתי משניים וחצי הקוראים.

  523. ישראל,אתה רוצה שאני אתן לך לינק לספרים בתורת היחסות, או מכניקת קוונטים?

  524. ישראל,
    “מהירות משיקית – יחסית למה? לחומר האפל?”
    מהירות האור היא יחסית לכל צופה. במקרה הזה יחסית לך, שמודד את מהירות סיבוב הדיסקית הגדולה.

    “מדוע לא נוכל להגיד שהמהירות המשיקית של דיסקית ב’, זו המסתובבת, היא מהירות האור יחסית לדיסקית א’, שאינה מסתובבת?”

    כי מהירות סיבוב היא מהירות זויתית. כאשר המהירות המשיקית של הדיסקית הגדולה היא מהירות האור,המהירות הזויתית שלה אפס. זו גם יהיה גורל המהירות הזויתית של כל דיסקית בגודל דומה. צופה היושב על דיסקית א’ (הגדולה) לא יוכל לטעון שהמהירות הזויתית של דיסקית ב’ (צלחת מעופפת, פלנטה, או גלקסיה) היא אפס, מכיוון שהיא איננה עונה להגדרה.

    אני מבין שאתה מאוד רוצה שאני אומר שמה שקובע את הייחוס זו המהירות היקומית הממוצעת, או סכום התנע היקומי. את זה אני לא אומר, מכיוון שזה פשוט לא נכון. זו לא הסיבה לכך שהכוכבים הרחוקים נראים עומדים.

    הסיבה הרלוונטית היא שכשמסתכלים על מרחקים גדולים היקום כולו במנוחה, כי כל העצמים בו מוגבלים למהירות האור. אתה יכול לראות בהם לדים המסמנים את המרחב, אבל הם לא משפיעים ולא קובעים דבר לגבי כושר התנועה של עצמים מרוחקים. גם אם סכום התנע היקומי לא היה אפס, וגם אם כל הכוכבים הרחוקים היו נעים במהירות האור לכיוון מוגדר כבמסדר צבאי, עדיין היית יכול לקבוע על פיהם מתי צלחת ב’ מסתובבת, ובאיזו מהירות זויתית.

    לכן, כמו ניוטון ובניגוד למאך, גם ביקום ריק הצלחת המסתובבת שלך תמשיך להסתובב (אלא שבבדידות זוהרת זו ובהעדר לדים חיצוניים) תצטרך להסתמך על מד הכח הצנטריפוגלי המורכב עליה.

  525. ישראל ויובל,

    אני גאה בכם. עזבתי אותכם ליומיים לצורכי סקי והנה התגשם לו חזון ישעיה “וגר זאב עם כבש…וגו’ יפה!

  526. מאיר,
    עדיין לא הבנתי.

    “נגדיר דיסקית כלא מסתובבת אם כאשר הקוטר שלה אינסופי המהירות המשיקית שלה היא מהירות האור.”

    מהירות משיקית – יחסית למה? לחומר האפל?
    מדוע לא נוכל להגיד שהמהירות המשיקית של דיסקית ב’, זו המסתובבת, היא מהירות האור יחסית לדיסקית א’, שאינה מסתובבת?

    נקודה

    “תפתח ספר לימוד ותלמד.”
    תוכל להצביע על ספר לימוד אחד יחיד, כולל מספר העמוד, שבו נושא זה מוסבר? אולי יש לך איזה לינק לשתף בו את כולנו?

    כי אם אין לך, אולי סופסוף תגיע למסקנה שאין עדיין הסבר למה ששאלתי? ושמה שאנחנו מדברים פה זה נסיונות למצוא הסבר ולא סתם דיבורים?

  527. ישראל, היתי בשמחה מסביר לך, אך העניין שלא מדובר ברעיונות שדנים בהם.
    תפתח ספר לימוד ותלמד. תתאמן. מלדבר על זה לא תלמד כלום.

  528. ישראל,

    אין מהירות גבוהה ממהירות האור, ולכן במרחב שהתפשטותו איזוטרופית לא קיימים עצמים הנעים במהירות גבוהה ממהירות האור בניצב לקו המחבר אותם לעצם מרוחק כלשהו. אם הקוטר האינסופי מציק לך, נתפשר על פחות מזה. די בקוטר סופי התחום על ידי כוכבים (=לדים) מרוחקים כדי שההגדרה שנתתי תשאר תקפה.

    מכיוון שהעצמים ביקומנו חולקים את אותו מרחב, שבין היתר גם קובע להם את מהירות האור כגבול עליון, תמיד דיסקית המוגדרת כפי שהגדרתי איננה מסתובבת, ותמיד לא יורגש בה כח צנטריפוגלי.

  529. יובל.
    בזמן שמחכים לתגובתו של רפאים לשאלתך, תוכל אולי להעביר לי את הפרקים החסרים כדי שאדע במה מדובר?

  530. נקודה, ארצה. אם אתה כזה אלוף במשוואות טנזוריות, בוא הסבר לנו כיצד פועלת הגרביטציה, ואיך עובר ספין האלקטרון בשזירה קוואנטית. אם לא, שחרר אותנו. יש הרבה כתבות נוספות שנוכחותך נדרשת בהן, ועם כל הקושי הכרוך בדבר, נראה לי שנסתדר איכשהו לבד.

  531. ישראל שפירא,
    האי-לוקליות תצטרך לחכות מעט. אתחיל בגרביטציה (ובהתמדה).
    אבל לפני הכל, נחוץ תאור של המודל לטובת מי שלא קיבל ממני חומר באימייל:
    אחרי שדילגתי על כמה שלבים (החל במנגנון בו ה”אין” חל על עצמו ליצירת “יש” והמשך באיך וכיצד נוצר החלקיק של החומר האפל) הגעתי לתאור הבא:
    1) היקום מאוכלס כולו בחומר אפל שצורתו חלקיקים צפידים (כלומר, לא מאפשרים חדירה של חלקיקים אחרים לתוכם) ובחלקיקים בלתי צפידים (שאפשר לצורך קיצור לכנותם “חלל ריק” [אך יש להבחין בין החלל הריק הזה לבין מה שהפיסיקה הרגילה מכנה “חלל ריק”, משום שהחלל הריק של הפיסיקה הוא המרחב המאוכלס בחלקיקים המדוברים]).
    2) החלקיקים הצפידים רוטטים אקראית, דהינו נעים ללא הגדרת כיוון ומהירות אל תוך פיסות חלל ריק שסביבם.
    3) כאשר חלקיק צפיד נתקל בחלקיק צפיד אחר, הריאקציה היחידה ביניהם היא שהם לא נכנסים אחד בשני (כי הוגדרו להיות צפידים). ברמה הזאת לא קיימת פיסיקה, ולכן לא יקרה ביניהם שום דבר המוכר מן הפיסיקה, כמו למשל, התנגשות אלסטית.
    כיצד מוגדרים חלקיקים שהפיסיקה כן מכירה וכיצד נוצרת ביניהם גרביטציה יוסבר בהמשך, אחרי השאלות והתשובות לחלק הזה. ר.ח רפאי.ם: קדימה! הָיֶה נא ראשון השואלים.

  532. יובל

    זב בסדר. אני גם יותר מאשמח לקרוא את המודל שלך. (אפילו שלדעתי הוא שגוי לחלוטין כבר ממה שקראתי, עדיין לדעתי לפחות הוא יותר הגיוני מהמודל של ישראל חברך. וגם זה בהחלט מעביר לי את הזמן בכיף).

    ישראל

    איזה הסכם שלום? אני אומר לך דעתי העניינית. ואתה נעלב. אם אתה רוצה שאפסיק להגיד את דעתי העניינית רק בשביל שתפסיק להיפגע אז כנראה שתמשיך להיפגע.

  533. יופי יובלי. אפשר להחזיר את התמונה הישנה? הרבה יותר טובה (לדעתי).
    רפאים! אתה מוכן להצטרף להסכם השלום ולהפסיק עם הפגיעות האישיות? אני יותר ממוכן.

    יובל.
    חוזרים לשאלות הישנות.
    1. כיצד פועלת הגרביטציה במודל.
    2. כיצד פועלת אי הלוקליות?

    אתה יכול לשלוח אותי ללינקים, או לכתוב את ההסבר פה. אבל בבקשה, לא משפטים כגון: “דיברנו על זה כבר”, או “זה מופיע בפרק 7”. אני שקוע בענייני, כמו שכולנו שקועים בעניינינו, ואיני זוכר כל פרט ופרט או תגובה שכתבת.

  534. מאיר

    “נגדיר דיסקית כלא מסתובבת אם כאשר הקוטר שלה אינסופי המהירות המשיקית שלה היא מהירות האור.”

    האם הגדרת ה”לא מסתובבת” לא יתפוס לגבי כל מהירות משיקית אחרת סופית? מה עם 100C, 2C? גם אז אם הקוטר הוא אינסופי, הדיסקית לא מסתובבת. לא?

    ומודה גם שלא הבנתי איך זה מסביר מי מבין הדסקיות מסתובבת “באמת” ומי לא.

  535. אל! אל! ישראל! חתום חתמתי
    ר.ח מוזמן לפתוח את בקבוק השמפניה (על חשבוני) במעמד תקיעת הכף.
    מ”ם סופית ו-למ”ד סמוכות זו אצל זו במקלדת, ואני מקוה ומאמין שהחתול נולד משגיאת דפוס והתקבע שלא בטובתו של אף אחד.
    ר.ח רפאי.ם, אנא אל תיקח עמוק ללב.
    בקרוב, אם יסתייע בעדי, אמשיך ואפרט את מבנה החומר האפל על פי תפיסת עולמי. מראש אציין כי רב הדמיון לתורתו של מאיר עמירם. אנא המשיכו לעקוב.

  536. “רפש”, ציטוט ישיר מהמאסטרו, אה?

    ונכון, מיד אחרי שהתנצלת, ( על מה אגב? בשביל מה בכלל התחלת?) המשכת. ראה תגובה זו:

    https://www.hayadan.org.il/astronomers-reach-new-frontiers-of-dark-matter-130112/#comment-327515

    נראה לי יותר שיש אנשים שצריכים שיעורים פרטיים מטאוויל פאדיחה, השר הפלסטיני לענייני זעם בלתי נשלט מלאטמה. אילו אנשים אכולי אגו, החייבים להתפרץ בכל פעם שמובעת דעה שונה משלהם, בלי אפילו לטרוח להקשיב עד סוף המשפט או הפסקה.

    ומכיוון שאין לי שום רצון לחטוף פטיש 5 קילו בראש מעצבאנים, שזו הדרך שלהם להוכיח את קיומה של המסה האפלה, אז עד שלא ישתפר פה טון הדיון, אין לי שום כוונה להכנס לויכוחים עם אלו שמה שמנחה אותם זה שיקולי אגו, לא מדע.

  537. נו, מה קורה ישראל? נגמר לך הרפש?
    ולמה בכלל אתה מזכיר אותי? מה מעניינת אותך דעתו של ילד ירוק וטיפש? כואב לך משהו בעין?
    למה אתה לא מתייחס לרעיון שהעלתי (בתגובה ליובל)? למה אתה ממשיך לכתוב שטויות?

  538. ישראל

    “רוצה לבוא על החתול?” ?? הבדיחה הזאת לא הגיונית כמו שלא הגיוני שלאדם יצמחו אשכים על המצח. (יובל אני יודע שאתה צוחק עכשיו 🙂 )

    בכל מקרה,

    זה מאוד נוח לבוא ולהגיד: ישראל הטיפש.
    וזה עוד יותר נוח להגיד אחר כך: אוי, סליחה לא התכוונתי.

    ואם לשפוט על פי הדברים שכתבת: אתה אמור לקבל את התנצלותי. חד וחלק.

  539. עוד הפעם התמונה הכועסת?

    רוקי זה בקולורדו. לא זוכר שכתבתי אי פעם רונית. פייסבוק, אה?

    יהיה קשה מאד להמשיך כשמסתכנים בפגיעה אישית כל פעם שמעיזים למתוח ביקורת. מושבעים, שמושבעים. לכן, ללא הצהרת כוונות, ולאור נסיון העבר אתך ורפאים, יהיה לי קשה להמשיך בדיון רציני אתכם על נושא כלשהוא.

    דוגמה אישית:

    אני, ישראל שפירא, מתחייב בזאת לא לפגוע אישית במתכוון במגיבים, אלא כתגובה על פגיעה אישית בי מכוונת עיי מגיב. במידה ופגעתי בשוגג, והמגיב קרא אותי לסדר, אתנצל ולא אחזור על טעותי.

    ועל כן באתי על החתום:

    ישראל שפירא
    ———————————–

    מה אתך? רוצה לבוא על החתום?

    רפאים? רוצה לבוא על החתול?

  540. ישראל,

    נגדיר דיסקית כלא מסתובבת אם כאשר הקוטר שלה אינסופי המהירות המשיקית שלה היא מהירות האור.
    נציב לדים בנקודות שונות על הדיסקית, כרצוננו.
    דיסקית זו תשמש אותנו למדידת מהירות הסיבוב של כל הדיסקיות האחרות.

  541. צ’ילבה (כלומר, לא בקשה מפורשת שמבקשת תשובה) יקר,
    ראשית, משהו על עקביות:
    כשאתה מזמין כאסח ובאותה נשימה מספר על כלאבי ורונית בהרי הרוקי, אל תתפלא אם גם הם חוטפים.
    שנית, משקפיים:
    בשום מקום לא כתבתי שיש קשר בין אינרציה לכוכבים הרחוקים. אמרתי רק שיש קורלציה בין התצפיות, ואינני אחראי לשום מסקנה שאתה מגיע אליה מיוזמתך.
    שלישית, התייחסות רצינית:
    אתה יכול לשמור על זכות השתיקה, אבל בזה אתה מסתכן שדעתו של חבר הקוראים המושבעים לא תיטה לטובתך.

    ישראל שפירא,
    בשעתו שאלת על השורש של המספר ה”דמיוני” i ובתגובה הראיתי כי אפשר לחשב אותו. אודה לך אם תאשר שקראת והבנת, ואם תרצה הסבר מעמיק יותר אספק לך אותו ברצון.

  542. מאיר
    אני מבין לחלוטין את ההסבר שלך. גם ציינתי זאת כמה פעמים. אך עדיין נשארת בעינה העובדה, שסיבוב הוא יחסית לכוכבים הרחוקים. אם תוכל כאן באתר, להסביר איך מסתדרות נורות הלד במנוחה דווקא יחסית לדיסקה א’ ( מצב מנוחה) ולא לדיסקה ב’ (סיבוב), זה יקל מאד את הדיון.

    לילה טוב.

  543. ישראל,

    שאלת איך אפשר להתכחש לעקרון מאך, ועניתי לך. עקרון מאך קובע שהמסה היקומית קובעת את מידת התנגדותה של כל מסה בדידה לשינוי במצב תנועתה. אלברט הסכים עם זה כשהתבטא שאינרציה היא תופעה של אינטראקציה בין מסות. לאלה אני מתכחש, וגם הראיתי איך אפשר.

    עיקרון מאך איננו הקורלציה, אלא ההשערה שמאך גזר ממנה. את אותה השערה ניתן היה לגזור גם ממקור אור מרוחק וחסר מסה. העניין הוא שהמסה היקומית נראתה למאך דבר מרשים יותר, שאפשר לתלות בו השפעה למרחוק, למרות שהוא לא הראה איך.

    בא מאיר עמירם, וטען שהמסה היקומית, ככל שתהיה מרשימה בכמותה,עשויה מהמסות של החלקיקים האלמנטריים המהווים אותה. לכן במקום לגרד את אוזן ימין עם יד שמאל ולטעון כמו מאך שכל חלקיק אלמנטרי מפזר את השפעתו המאגית לכל החלקיקים האלמנטריים האחרים ביקום, הבה נגרד את אוזן ימין עם יד ימין ונטען שכל חלקיק אלמנטרי שומר את השפעתו המאגית לעצמו.

    עוד טען מאיר עמירם, שאם נעשה כך, נגלה ששלושת חוקי התנועה של ניוטון נגזרים ישירות מחוק היפוך הריבועים של ניוטון. הוא לא רק טען, הוא גם הראה איך.

    לסיכום, יש לנו את מאך שטען שכל חלקיק אלמנטרי “תורם” באמצעות מכניזם בלתי ידוע ובלתי מוכר השפעה על כל החלקיקים האלמנטריים האחרים ביקום.

    מצד שני יש לנו את עמירם שטוען שכל חלקיק אלמנטרי משפיע על עצמו במכניזם ידוע ומוכר.

    כמובן שקבלת העקרון של עמירם היא שלילת העקרון של מאך, מכיוון שהיא מכניסה את הכוכבים המרוחקים לקטגוריה של נורות לד, שגם אם יכולות לשמש כניצבים לצורך צילום וידאו, אינן משתתפות באופן פעיל בעלילה כטענת מאך.

  544. רפאים.
    אם אינך מוצא טעם להגיב, אז נו, טוף, מה לעשות – אל תגיב.
    אתה יכול גם לחזור למקומך הטבעי: האפס המוחלט – ומתחת לו!

    יובל.
    אז עכשיו יש קשר בין אינרציה לכוכבים הרחוקים? חשבתי שכתבנו לא מזמן “פשוט להגיד כי מאך טעה.”

    אבל קצת נסחפנו. בעבר עשיתי את הטעות להגיב לתגובותיך ללא אישור מפורש, וחטפתי על כך חזק, ללא כל הצדקה. כפי שציינתי כבר, לא אחזור על הטעות שנית. ללא ב ק ש ה מפורשת ממך, לא אגיב למה שאתה כותב באופן ישיר. אם אגיב, אחרי אותה בקשה מפורשת, זה יהיה רק בדרך היחידה הידועה לי לחתירה לאמת: המיתודה הסוקרטית של שאלות ותשובות. השאלות תהיינה נוקבות, בלתי אישיות, קונסיסטנטיות והגיוניות. אצפה לתשובות בהתאם. אם לא ברור לך במה המדובר, ראה חקירת השתי וערב שערך לי ר.ח.

    אם ארגיש שאתה עובר לפסים אישיים, נמנע מתשובות, או לדעתי עונה שלא לעניין, אסיק את המסקנות שלי בהתאם.

    זו, אגב, הדרך היחידה לדעתי למנוע ממודלים להפוך ממודלים פיזיקליים למודלי אגו. זו גם הסיבה שאני תמיד מזמין ביקורת כסאח על כל רעיון אותו אני מעלה: כדי לחסום טעות מצטברת באיבה.

    הכדור בידיך. זכור: ב ק ש ה מ פ ו ר ש ת.

    אני יכול לחיות טוב מאד גם בלי שיעלו אותי על המוקד על שניסיתי לעזור.

    ישראל, שוטף המוחות שלא ניתן לבטוח בו.

  545. ישראל שפירא,
    אניו (ולדעתי גם מאיר) לא אומר שאין קורלציה בין שתי התצפיות (וידאו & מד כוח). אך הטענה שלי היא שהקורלציה הזאת היא תוצר משני. המצלמה מראה את התנועה היחסית בין הגוף והכוכבים, ומד הכוח מראה את האינטראקציה בין הגוף לבין החומר שבקרבתו, שממנו אתה ממשיך להתעלם בעקשנות ראויה לציון.

  546. ישראל
    ביקשת להתייחס לתוכן הדברים.
    אם כן, ברשותך, אתייחס לתוכן התגובה ולמשפט שמופיעה בה: ישראל הדמגוג.
    לדעתי היית צריך לרשום: ישראל הליצן.
    לגבי שאר דבריך, איני מוצא טעם להגיב.
    שבוע טוב.

  547. מאיר.

    אינני טוען שיש לי הסבר לעקרון מאך, או שלא ניתן לבנות ניסוי מסויים שיתכן שיסטה מהנורמה. אני רק טוען שחייבת להיות קואורלציה בין הכוח הצנריפוגלי והכוכבים הרחוקים. אינך מאמין? שים מצלמת וידאו הפונה לשמים על צנטריפוגה, שים בה מד כוח, קח אותה לכל נקודה שתבחר ביבשת בה אתה מתגורר, סובב אותה במהירות זוויתית קבועה, ושלח לי את הוידאו ביוטיוב.

    אני מאמין שרק מצפיה בוידאו אוכל לאמר לך מה הכוח שמראה מד הכוח בצנטריפוגה, וכמה אורניום היא יכולה להפריד.

    וכל זאת – מוידאו של כמה כוכבים מסכנים.

    לא קוסם הקוסמוס?

    יתכן כמובן שיש הסבר מדוע הכוכבים נמצאים בדיוק איפה שהם לפי הרעיון שלך.

    ובקשר לניסוי שהוצע פה כדי לסתור את עקרון מאך: אם ניקח נתונים ריאלים, שבהם משקל הנוזל בדלי כ15 ק”ג, והמערכת מוצבת ליתר בטחון בנקודת אי המשקל בין הארץ לירח, הרי שגליל שגובהו מטר אחד ומרכז המסה שלו ברדיוס של מטר מהדלי, יהיה בעל מסה של כ100,000 טונות כדי להוות שווה ערך לשמש לבדה, שלא לדבר על שאר היקום. ניסוי יקר מאוד, מה גם שאינני חושב שמאך היה מתנגד לפירוש השפעת הגליל על הדלי ולהיפך. ט.ל.ח.

    יובל הערקמן.

    פירשת לא נכון את עניין העלבון שלי. אני מתלהב מכל כסאח, כל עוד הוא ברמה. ואותך תמיד ציינתי לטובה בעברית המצויינת, ע”ע אימייל (שלך).

    גם כשציטטתי את ה”מה אתה עשית” די התלהבתי, כי זה הזכיר לי את פלאטו שרון: “מה אתה עשית בזביל מדינה?”

    אך לאור ניסיון העבר העגום, הייתי מסתפק בעמוס, ה’, י”ד.

    ישראל הדמגוג.

  548. סרסור מילים מקרית גנבים – מודה ועוזב ירוחם או טובל ושרץ בידו?
    יו מייד יור פוינט. הראית שאין לי מונופול על העלבון. אם תתייחס פעם אחת ברצינות למה שאני אומר, אקח את כל המילים הקשות בחזרה – עם ריבית.
    הטענה של מאיר (ושלי) היא שבקרבת הגופים (הדיסקיות המסתובבות, למשל) קיים חומר שאמנם איננו רואים אך ישנה אינטראקציה בין הגופים לבינו. מאיר מכנה אותו “שדה הכבידה העצמי של הגופים” (ואני מזהה אותו עם החומר האפל של הגלקסיות ומייחס לו אופי חלקיקי קוואנטי במובהק עם שמץ התנהגות גלית).
    הטענה שאתה דבק בה בעקשנות פִּרְדִית היא שהאחראי להתנהגות המיוחדת של הגופים הינו לאו דווקא (או לא רק) החומר המדובר שבקרבת הגופים כי אם המאסות שבכל היקום כולו. את זה אתה מתרץ בתצפיות המשקפות כוכבים רחוקים ואינן מראות את החומר הזה (שכאמור הוא בלתי נראה).
    השאלה שאני מבקש ממך שתתייחס אליה ברצינות, שלא כדרכך, היא מדוע אתה מתעקש להתעלם ממנו?. אתה יכול אולי לטעון כי קיומו לא הוכח (אושש, ליתר דיוק [תודה לסטודנטכניון]), אך זה לא אומר שהוא לא קיים (בינתיים קיבלנו אישוש לקיומו של חומר בחלל הבינכוכבי שאמנם איננו רואים אותו אך הוא מתערב בחוזקה בתהליכי גרביטציה). אתה יכול להגיד שזה נוגד את האינטואיציה, אך כבר למדנו לדעת שהאינטואיציה איננה המדד היחיד למציאות. אתה יכול להגיד הרבה דברים, אך בבקשה היה עקבי והגיוני והשתדל להימנע ככל האפשר מסיפורים על הכלב והאישה וכו’.

  549. ישראל,

    בעניין הדיסקיות המסתובבות:

    בא נניח שהן מסתובבות ביקום ריק מכל מסה אחרת. עדיין ביקום הזה יש מרחב שהאריג שלו הוא רצף של קצף קוונטי שדרכו מופצת הגרויטציה של כל אחת מהדיסקות.

    שדות הגרביטציה של כל דיסקה אינם “סובבים”, כי לשיטתי לא קיים frame gragging, ועל כן הם נייחים זה ביחס לחברו. אבל הם סכום של שדות גרביטציה מיקרוניים הנוצרים על ידי החלקיקים האלמנטריים המהווים את הדיסקות. אם כוח חיצוני גרם לאחת הדיסקות להסתובב, שדות הגרביטציה המיקרוניים שלה יסבלו מאסימטריה בכיוון הסיבוב, אשר תנציח את תנועת הסיבוב, שמצידה תנציח את האסימטריה הזו.

    דיסקה ששדות הגרביטציה המיקרוניים של חלקיקיה האלמנטריים סובלים מאסימטריה כזו, היא דיסקה מסתובבת. מידת האסימטריה בשדות אלה היא פונקציה של מהירות הסיבוב. כל זאת ללא תלות במצב תנועתה של הדיסקה האחרת. זו מהירות סיבוב אבסולוטית יחסית לאריג המרחב המקומי.
    אם המרחב ביקום הזה מתפשט (עקב תוספת קצף קוונטי), ככל שההתפשטות הזו היא איזוטרופית (עקב תוספת איזוטרופית של קצף קוונטי), לא תיווצר תנועה יחסית בין הדיסקות בעקבות התפשטות כזו, למעט התרחקות הדדית חסרת אפקטים אינרציאליים.

  550. ישראל,

    בא נניח לצורך הדיון שכדור הארץ היה סב על צירו פעם ברבע שעה, ואתה עומד יציב על הקוטב הצפוני ומסתכל בכוכבים המרוחקים החגים סביבך פעם ברבע שעה.
    להסתובב במקום בתדירות של אחת לרבע שעה איננה תופעה האמורה לגרום לך להרגיש תנועה על בשרך, ולכן בהביטך אל הכוכבים לא תוכל לקבוע האם כדור הארץ הוא הסובב על צירו, או שמא היקום סובב סביב הארץ.

    כעת אתה עולה על הסחרחרת המסתובבת סיבוב אחד לשניה יחסית לארץ, ומצלם את הוידאו. עדיין יהיה בידיך וידאו המתעד סיבוב יחסית לכוכבים הרחוקים, בתדר שהוא כמעט ב-100% זהה לתדר הסיבוב שלך יחסית לארץ, כלומר יחסית לכל וידאו אחר בו היית בוחר לצורך תיעוד הסיבוב.
    האם תוכל להסיק רק על סמך הוידאו הזה על קורלציה בין הסחרחרת לבין הכוכבים הרחוקים, יותר מאשר על קורלציה בינה לבין הארץ?

    לעניננו, הכוכבים הרחוקים כל כך רחוקים, שגם אם קיים סיבוב יחסי בין הגלקסיה לבינם לא נוכל לקבוע דרך סיבוב של סחרחרת האם הגלקסיה היא הסובבת או היקום סובב סביבה. במרחקים המדוברים תמיד תהיה התאמה של 100% בין תדר סיבוב תמונה הוידאו של הכוכבים המרוחקים לבין תדר סיבוב תמונת הוידאו שסוקרת את הנוף הארצי.

  551. ליעל
    כדי שלא תהיי עצובה אני מוכן להוסיף עוד התערבות על ההיגז בוזון,
    אז מה דעתך?
    (:))
    יום טוב
    סבדרמיש יהודה

  552. מאיר.
    אינעל זבבור בובלובה. עוד הפעם תגובה ממתינה לאישור. כמעט 4 בבוקר ב ל.א. לילה (בוקר?) טוב.

  553. מאיר.

    איני יכול לדעת. אבל נראה לי הגיוני, שאם אוכל, רק לפי הוידאו של הכוכבים הרחוקים, לדעת בדיוק, בלי אפילו להסתכל, מהו הכוח שימדוד מד כוח צנטריפוגלי בסחרחרת, הרי שכנראה יש איזו שהיא קואורלציה ביניהם.. אחרת הייתי יכול להסיק את הכוח מכל וידאו אחר, לא?

    כמו שאם רוב הילדים בשוודיה בלונדינים עם עיניים כחולות, אז קיימת איזו קואורלציה בין עיניים כחולות ושיער בלונדיני, ויש סיכוי גבוה במקצת לילד שוודי בלונדיני שיהיו לו עיניים כחולות מבוא נגיד, כושי בן שבט זולו. תשאל את ר.ח., הוא מבין בגנטיקה ובשוודיה. לא?

    נראה לי גם הגיוני לחייב את יהודה להזין את יעלי בחלקיקי גלידה אפלה, לא?

    ונראה לי גם הגיוני שאם השעה היא 3.30 בלוס אנג’לס, אז הגיע הזמן לישון, לא?

  554. הי יהודה,

    בדקתי שוב, אתה צדקת ואני טעיתי – התערבנו על חלקיק המסה האפלה… אז כנראה שהלכה לה הגלידה שלי!
    כאשר אני התערבתי איתך בניתי על זה שסגרו על יצור ההיגס בתחום 125 GEV, ומה שנישאר זה רק להגדיל את ודאות המדידות לחמש סיגמאות. בשביל חלקיק המסה האפלה אני אצטרך מזל גדול בשביל לאכול גלידה בשנה הקרובה…

    🙂

  555. ישראל,

    אתה כותב “לא ברור לי איך אפשר להתכחש לעקרון מאך. הרי מספיק שתסתובב בסחרחרת בלונה פארק, ותצלם את הכוכבים למעלה בוידאו. אם תסתכל על הוידאו, האם לא תוכל לדעת רק לפי הוידאו עד כמה הסתובב לך הראש בזמן הסיבוב?”

    איך אתה מוכיח מהסיטואציה הזו שהדבר שביחס אליו מאיצים החלקיקים האלמנטריים המהווים את ראשך הסחרחר הוא מסת הכוכבים הרחוקים (באמצעות מנגנון עלום שאפילו מאך עצמו לא ידע להציע מהו) ולא שדות הכבידה העצמיים של החלקיקים האלמנטריים המהווים את ראשך (באמצעות המנגנון שהציע מאיר עמירם)?

    אגב, כטיעון צדדי: שים לב שאינך יכול להסיק דבר ממצב תנועתם של הכוכבים הרחוקים מכיוון שגם אם היו נעים במהירות האור בניצב לקו המחבר בינך לבינם (או בכל כיוון אחר שתבחר), עדיין הם היו נחזים לך במנוחה בגלל מרחקם העצום.

  556. ליעל
    אנחנו התערבנו על חלקיק המסה האפלה

    כדברייך ב 25.1.2012
    “התחזית היא שהחלקיק האחראי על ה”מסה האפלה” ימצא השנה ב-2012″ סוף ציטוט.
    חלקיקי המסה האפלה הוא חלקיק ה ווינגז
    ההיגז בוזון הוא החלקיק האמור לתת את המסה לחלקיקיי היסוד. לא ניראה לי שלזאת הייתה הכוונה ונידמה לי שאת מנסה בטענות שווא לסחוט ממני הגבר התמים גלידה עסיסית, שלא לדבר בדרכי רמייה.
    אבקש מקהל המגיבים להצהיר האם לדעתכם גילוי ההיגס בוזון יהיה הוכחה גם לגילוי חלקיק המסה האפלה ובשל כך גם לחייב אותי בגלידה?
    מחכה לתגובתכם. מבטיח לקבל את דעת הרוב.
    יום טוב
    סבדרמיש יהודה

  557. יהודה – never say never

    נכון שזה נשמע אבסורד שיהיה למולקולה תנע שלילי (טמפ’), אבל כשממשיכים לחשוב על זה, אז קשה להיות בטוחים. בשנה שעברה נמצא ב-LHC שחלקיקי אנטי-חומר מתנהגים שונה מחלקיקיהם התאומים. איך עיניין הטמפ’ יראה בעולם של אנטי-חומר?
    יש את נושא העל-קרור במעברי פאזות, מים יכולים לקפוא ב-C -15, בתנאים מסויימים. מהם התנאים שמביאים היום לאפס המוחלט? ניתן להשפיע עליהם? לשנות את האפס המוחלט?

    מצורף קישור על אנטי חומר. פעם חשבו שאנטי-חומר הוא ההבדל במטען החשמלי. מאז נמצא אנטי-ניוטרון וההגדרה התרחבה:
    https://www.hayadan.org.il/anti-helium-2704115/

    אגב, קראתי בסיינטיפיק שגם חוקרי הטבטרון (שכבר נסגר) צפויים להגיב על מציאת ההיגס במרץ… יכול להיות שהגלידה של מתקרבת?
    🙂

  558. מאיר.
    האם אתה מזהה את הבעיה שיתכן וקיימת במודל האינרציה העצמית? אם 2 דיסקיות מסתובבות יחסית זו לזו בחלל, רחוק מכל מסה, איך זה שנוכל לקבוע בודאות שדסקית א’ נחה וב’ מסתובבת? ועל אותו משקל: אם גוף א’ מאיץ יחסית לגוף ב’, מדוע לא נוכל להפוך את היוצרות ולהגיד שב’ מאיץ יחסית לא’?
    אני מתאר לעצמי שיש פתרון, אבל הייתי רוצה לשמוע ממך קודם.

    ישראל הסרסור.

  559. יובל,יהודה

    כן, ברור זאת חריגה מהפיזיקה. לדעתי קיים ‘כאן’ (נקרא לזה נקודת הנפיצה- במקרה שבו מתקיים קשר בין אותו חלקיק גזי לפוטון) קשר ישיר לבעיית הסימטריה. לדעתי בין היתר ברגע הקריטי הזה נוצרת שבירת סימטריה.
    בכל אופן, יובל, לגבי בקשתך לפרט, אולי בהזדמנות.

  560. האפס המוחלט מוגדר באופן כללי כחוסר תנועה במולקולות. אז איך תגדיר טמפרטורה יותר נמוכה?, מהירות שלילית? נידמה לי שיש כאן חריגה של הפיזיקה.
    לילה טוב
    סבדרמיש יהודה

  561. אריה סתר,
    התכווצות לורנץ מקובלת על הפיסיקאים והיא גורם מכריע בתורת היחסות אף על פי שאיננו יודעים מה גורם לה.
    הסיפא של דבריך הינו מובן מאליו, אך לא ברור לי הקשר לרישא.

  562. ר.ח רפאי.ם,
    כשאתה מדבר על טמפרטורה הנמוכה מן האפס המוחלט, אתה יוצא אל מחוץ לגבולות הפיסיקה. זה מראה על חשיבה יצירתית שאני כעיקרון נוהג לעודד. קשה לי לראות לאן הרעיון שלך מוליך. אז אם תוכל לפרט יותר, זה עשוי להועיל.

  563. יובל – איני סבור שצריך לתאר מנגנון של התכווצות פיצג’רלד לורנץ, כיוון שאם אתה מודד מהגוף הנע עצמו או מגוף הנע במקביל לו – אין שום התכווצות.

  564. יובל

    אני לא חושב שקיימת בעיתיות בשאר (חוץ מפוטון) הדברים שכתבת.
    מה דעתך על הרעיון הבא: שישנם חלקיקי גז (או במצב צבירה פלזמה- עדיין לא סגור על זה) שהטמפ’ שלהם מתחת לאפס המוחלט? לדעתי קיים חלקיק כזה והוא מקיים אינטרקציה עם פוטון. הקשר בין שני החלקיקים יוצר את הנטרינו. (אומנם גם הניסוח הזה של הרעיון הוא בעייתי ולא מדויק מספיק, אבל הרעיון רק בחיתולים, וגם אילו היה מדויק עדיין קיימת בעיה של להמציא טכנולוגיה בשביל לבצע את הניסוי).

  565. כדי להבין את המנגנון המביא להתכווצות לורנץ במהלך תנועה צריך להבין קודם את המנגנון היוצר את התנועה. מאיינשטיין למדנו כי תופעת עקרון ההתמדה (שנוסח בידי גליליאו ונקרא היום “החוק התנועה הראשון של ניוטון”) ותופעת כח המשיכה בין מאסות הינן למעשה תופעות זהות. לכן יש מקום להניח כי אותו שדה גרביטציה המקיף גוף מסיבי כלשהו הוא זה שאחראי גם להתמדת תנועת הגוף במרחב. על זה כבר דיבר כאן מאיר עמירם, שהציב גם את הנוסחאות המתמטיות המתאימות, ואודה לו אם ישתף אותנו בפרטים.

    כדי להבין מהי תנועה, עלינו להסתכל אל מעבר לאינטואיציה שלנו. מנקודת המבט שלנו, גוף כלשהו מכיל בתוך עצמו את כל מה שנחוץ להגדרת המאפיינים שלו, ובהם מסה. אלא שראיה זו נובעת מן האופן בו נוח לנו לתפוס את המציאות. יחסי הגומלין בין המהות החשובה שלנו לבין המהות שאנחנו מייחסים לה חשיבות במציאות הסובבת אותנו איננה בהכרח הדבר הבסיסי. משל למה הדבר דומה? כשאנחנו מבצעים פעולה גופנית כלשהי אנחנו לא מודעים לפעולתו של כל תא שריר וכל תא עצב המשתתפים בתהליך, כמו גם לא לזרימת הדם או לנוירונים והסינפסות במוחנו שיוצרות את המחשבות; כל מה שמעניין אותנו הוא התוצאה הסופית, שהיא ביצוע הפעולה. כך גם ראייתנו את עצמנו ואת כל הגופים סביבנו כמהויות המוגדרות לעצמן והנפרדות מן המרחב שסביבן איננה בהכרח תפיסה נכונה של הדברים. אם נראה את הגופים כחלק בלתי נפרד מסביבתם ואת סביבתם כשותפה בהם ובתהליך התנועה שלהם, נוכל לחשוב על אופן בו תופעת ההתמדה של גוף אינה נובעת מן הגוף לבדו. אנחנו יודעים כי גרביטציה היא תהליך בו גורם חיצוני (שאנחנו מייחסים לגוף אחר) מעורב בתנועתו של הגוף. על כן אפשר לחפש מעורבות של גורם חיצוני גם בתופעת ההתמדה.
    דיברו כאן הרבה על המודל של לה-סאג’ ועל האתר, וייחסו להם אופי חלקיקי. אולם כפי שליחידת האור יש יותר ממופע אחד, יש מקום לדבר גם על יותר ממופע אחד של חלקיקי לה-סאג’ או האתר. על כך אנסה לדבר בהמשך.

  566. ר.ח רפאי.ם,
    נכון שזה נשמע קצת מעורפל, אבל יש לי הרגשה שהבנתי את רוח הדברים.
    אנחנו מכירים שני אופנים שונים של האור, גל וחלקיק. אף על פי שנראה כי הם סותרים זה את זה, קיום שניהם אושש בתצפיות. אני סבור שבמופעו כגל האור מתקדם במהירות הידועה כמהירות האור, אך בתור חלקיק הוא כפוף למערכת חוקים אחרת. למשל, הוא יכול להילכד בתוך אלקטרון ולגרום לו לשנות את גודלו. אני רואה את האלקטרון כעטיפה דקיקה מסביב לפרוטון. פוטון הנלכד בתוכה גורם לה להתעבות והיא “מחפשת” לחזור לעוביה הדק על יד כך שהיא מגדילה את קוטרה. אך בקוטר המוגדל הזה היא פחות יציבה והיא עוברת למצב יותר יציב תוך שחרור פוטון. הפוטון שמשתחרר, אם אינו נלכד באלקטרון אחר, חוזר לצורתו הגלית. אחת המסקנות שעשויות לנבוע מזה היא שהאלקטרון הוא “מאגר” של פוטונים, ואכן גם האלקטרון מפגין אופי דואלי – חלקיק וגל.
    אמנם לא עניתי בדיוק לדברים שלך, אך כוונתי הייתה להרחיב מעט על הפוטון.

  567. יובל
    “מהירות האור קבועה” זה משפט בעייתי.
    זה מוביל גם לטעויות כמו – “מהירות האור אולי לא קבועה”.
    לדעתי צריך לדייק ולומר שמהירות חלקיק האור קבועה.
    הבעיתיות היא במילים “מהירות האור”. זה לא מהירות האור, אלא אורך הגל של הפוטון. המהירות של הפוטון היא סופית. הבעיה היא כאשר פוטון עובר בתווך כלשהו…
    אורך גל מסוים של פוטון יוצר – אם תרצה – “אשליה אופטית” (כמו אפקט דופלר למשל), בגלל האי-אפשרות של מכשירים קיימים להבדיל או להפריד בין אורך גל של פוטון לבין אורך גל של קרינה לא ידועה 🙂
    (זה נשמע קצת מעורפל אבל מקווה שהבנת הכוונה)

  568. מדוע נקראת תורת היחסות בשם הזה?
    איינשטיין כינה אותה בשם אחר: “תיאוריית הקביעוּת” (בתרגום חפשי מ-Invariance). הוא החליט לקחת את מהירות האור כקבועה לגבי כל מערכת יחוס, ולראות לאן זה יוביל. את השם המקובל היום נתן לה מקס פלאנק, ומספרים כי איינשטיין לא אהב את זה.
    הנדריק לורנץ נתן הסבר לתוצאה המפתיעה של ניסוי מייקלסון מורלי: “העצמים מתכווצים לאורך תנועתם”, אך לא סיפק את המנגנון היוצר את ההתכווצות הזאת. איינשטיין עקף את המשוכה, והשאר היסטוריה.
    ואכן, תורתו של איינשטיין מקבלת את התכווצות לורנץ ואף משתמשת בנוסחאות שלה בדיוק נמרץ. היות שלורנץ יצא מתוך נקודת הנחה כי מהירות האור איננה קבועה אלא היא רק נראית לנו ככזאת בגלל ההתכווצות, והיות שאיינשטיין הסתמך על נוסחאותיו של לורנץ, הרי איינשטיין רק תיקנן את הפיסיקה לפי התופעה הזו.
    היות שהתצפיות שלנו מתבססות הרבה על אופטיקה, והיות שמהירות האור נראית כקבועה, הרי התצפיות מתאימות למסקנותיה של תורת הקביעות/היחסות וזה קיבע את האמונה כי מהירות האור בריק הינה קבועה בכל מקום ביקום.
    מקביעות מהירות האור קיבלנו תחזית יפה ומדויקת של מספר תופעות, אך נותרנו עם פרדוקסים מטרידים, שהאחרון בהם, אך החזק מכולם, הוא האנרגיה האפלה. פתרון לכל הפרדוקסים האלה יינתן כאשר יימצא המנגנון המביא להתכווצות לורנץ

  569. ניסוי מאך:
    לוקחים גליל גדול ממתכת כבדה במיוחד (נסתפק בעופרת, כי מחירי הזהב והפלטינה רק מאמירים). יוצרים בתוכו חלל שניתן להכניס לתוכו דלי. בונים את המערכת הזאת כך שהגליל יוכל להסתובב סביב הדלי אך גם הדלי יהיה עצמאי להסתובב (שרטוט היה מסביר זאת בקלות). בתוך הגליל, על תקרתו של החלל, מציבים מצלמה הרואה את הדלי. הדלי ממולא בנוזל כבד (מן הסתם, כספית) והמצלמה מתעדת את מצב הנוזל בדלי.
    שני שלבים לניסוי: בשלב אחד מקבעים את הדלי ומסובבים סביבו את הגליל. על פי מאך, המאסות בכל היקום משפיעות על הדלי בסיבובן היחסי כלפיו. משום כך, גם הגליל הכבד אמור להשפיע. יותר מזה, השפעתו של הגליל הכבד גדולה מהשפעת כל המאסות ביקום, משום שהוא קרוב לדלי. בשלב השני הופכים: מייצבים את הגליל ומסובבים את הדלי. לאחר מכן משוים בין צילומי פני הנוזל בשני השלבים.
    הערה: אם יתברר שסיבוב הגליל השפיע פחות מסיבוב הדלי, אפשר להניח כי מאסת כה”א התערבה ושיבשה את הניסוי. לפיכך, אפשר לנסות לעשות אותו במקום המרוחק מכה”א. ברור שלכה”א ישנה השפעה גם במרחקים גדולים (ויש אפילו הטוענם כי עד אינסוף), ועל כן יש לבצע את הניסוי כמה פעמים, ובכל פעם במרחקים שונים מכה”א.
    מצד שני, אפשר גם לחסוך את הוצאות הניסוי ופשוט להגיד כי מאך טעה.

  570. תיקון טעות (מן הסתם אף אחד לא עוקב, אבל למען הסדר הטוב…)
    טענתי כאן בלהט כמה פעמים שהאור המגיע אלינו מן הגלקסיות הרחוקות מאיץ בדרכו. קיבלתי מייל מאחד המגיבים כאן שחישב ומצא כי האור אמור דווקא להאט (אם להיות עקביים עם טענתי כי מהירות האור אינה קבועה). חישבתי גם אני ומצאתי כי הצדק איתו. האור, במסעו בן מיליארדי השנים מן הגלקסיות הרחוקות, מאט בדרכו אלינו, וזה מה שגורם לתופעת ההסחה לאדום שקיבלה פירוש כאילו הגלקסיות מאיצות.

  571. ר.ח.

    נכון, אבל להבדיל מיהודה, בדיוק על הנקודה שהעלת, כתבתי:

    “וכאן אני יודע שחייבת להיות לי טעות. אין, אין מצב שאני חשבתי על זה ולורנץ לא.”

    וגם הוספתי:

    “אבל עד שאדע איפה הטעות, אני חייב לנסות לפתור את התעלומה, כדי שאוכל למות מאושר.”

    ואז כתבתי את כל הרעיון פה באתר.

    אז מה דעתכם: איפה הטעות? האם יכולה, לוגית, להיות מערכת מנוחה לאתר ביקום אינסופי ואיזוטרופי?

    אריה.

    יקל לחשוב על כך, אם תדמיין חללית במרחק שעת אור מהמשדר. במקרה כזה, סיגנל א’ יכול להקדים את סיגנל ב’ נניח בדקה שלמה. בשביל הסיגנל, מה שיכול לגלות אותו זה גלאי במהירות ליניארית מסויימת. מכיוון שהאנטנה הראשונה נעה יחסית אליו במהירות גבוהה/נמוכה, תלוי על איזה צד של הדיסק אתה מסתכל, אז המהירות המשיקית היא נמוכה/גבוהה בהתאם, ולפי הרעיון שלי היא יכולה לשמש “מלכודת” לסיגנל, כי יחסית אליו היא נעה במהירות פחותה ממהירות ה”מילוט”. האנטנה מחוברת קווית לטיימר, הרושם את זמן ההגעה של הסיגנל (בדוגמה שהבאתי, אגב, לא צריך טיימר. אפשר לראות או לשמוע את הסיגנל כמו כל אות רדיו אחר).

    דקה אחרי מגיע הסיגנל השני, הנקלט במערכת הרגילה.

    צמצם את הכל, והנה המתקן שלי.

    ניתן לעשות אותו הדבר עם קליטת אותות מפולסרים רחוקים, אך זה יכול להיות מסובך. המערכת שתיארתי אינה שלמה אגב. זהו רק הרעיון הכללי.

    יאללה, עוברים למודלים החשובים באמת.

    ישראל מקריית גנבים.

  572. ישראל שפירא – הרעיון של אנטנת דיסק מסתובב שלדבריך זהה מבחינת השהיית הקליטה לאנטנה המתרחקת מהמשדר במהירות הזהה למהירות המשיקית של הדיסק המסתובב לא נראה לי. אחרי הכל כל אחת מהאנטנות צריכה להתחבר למקלט בו יקלט האות ויימדד הזמן. המקלטים עצמם נמצאים באותו מרחק יחסית למשדר וזה שאחת האנטנות מבצעת סיבוב – זה לא ישפיע. האנטנה המסתובבת צריכה להעביר את הסיגנל הנקלט למקלט דרך ציר שיהווה את החיבור החשמלי של האות הנקלט ואיך שאני מסתכל על זה – הסיבוב שלה לא יגרום להשהיית הסיגנל – כלומר האות ייקלט בצורה מסונכרנת בשני המקלטים.

  573. יובל,
    חבל, הכי קל לכעוס והכי קשה לסלוח לכן האנשים הגדולים בעיני בהסטוריה הם אנשים שקראו להבלגה במצב שהיה הכי קל לשלהב והשתלהב כמו גנדי, מרטין לותר קינג, בגין אחרי אלטלנה, סאדאת ורבין.

    ישראל,
    נראה לי שמיצינו. רק הערה אחת לגבי סעיף 3 שלך. אתה טענת בעבר שאתה מרכין ראש מול מקסוול לורנץ ואחרים. טענת גם שהתאוריה האקזוטית שלא לאמר מופרכת של ל.ס ראלית כי הרבה אנשים כולל ניוטון הבינו שיש כאן מעין גרויטציה ורק בעיית החיכוך נשארה. אולם מצד שני אתה בא כאן ובמחי יד קוטל את נסיון מ-מ שנחשב לניסוי מספר אחד בכל הזמנים בפיזיקה (טוב, לפחות בעשיריה הראשונה) וכל הקהילה המדעית פיזיקלית אימצה וקיבלה אותו ובעקבותיו את תורת היחסות. קטונטי לשפוט בנושא אולם יש לי הרגשה שלא סביר שכל הפיזיקאים במאה ה-20 מטומטמים ולא ראו שזה ניסוי לא לוגי כמו שאתה טוען.
    ואותו דבר גם לגבי סעיף 2. אם זה כל כך ברור שיש אתר ואי אפשר להתעלם מזה כדבריך אז איך זה שכל הפיזיקאים כן מתעלמים ממנו והוא נחשב לדוגמא קלסית לתאוריה זמנית שכשלה?
    אתה מתחיל להישמע קצת כמו יהודה ידידנו שרואה בכל הפיזיקה איזו שהיא קונספירציה סודית שמטרתה לדחוף את המסה האפילה, כאילו שלמישהו יש מניות של מסה אפילה. חבל שאין כי אני הייתי קונה, והרבה.

  574. בסדר, יובל, הבנתי את הרגישות. בוא נראה את שנינו כמשוחררים.

    בסדר, אבי, הבנתי את הרגישות של הוורדפרס. לא אכתוב יותר “איינשטיין”.

    בסדר, נקודה. וגם צריך שיהיה פינה לאלו “שאין להם שום רעיון”.

    ר.ח.

    1. אין בעיה כזו. בוא נקצין את הנתונים: נאמר שהגלאים הם אצל אסרונאוט במרחק שעת אור מהארץ בדיוק. אם אני מצליח לשלוח לו את המספר 7, ולו יש הוראה, להכפיל את המספר שקיבל ב 5 ולשלוח חזרה לכדה”א, וקבלתי 35 תוך שעה וחצי, הרי שהצלחתי לשלוח סיגנל עם אנפורמציה מהר מהאור.
    צמצם את כל הנתונים, וקבלת את הניסוי שלי.

    2. נקודה יפה. אבל אל תשכח שני דברים: קודם, שאינני יודע אם לפי המודל האור נע בכל המהירויות. יתכן שהוא נע בקשת מהירויות מסויימת, בהתאם למהירויות המתנד במקור. והשניה, החשובה, שמה שחשוב הוא, תחום המהירויות להם רגיש הגלאי, או הצופה. כמו שאם אתה מנסה למדוד את המרחק לקשת בענן, תראה תמיד שהיא במרחק מסוים וקבוע ממך הצופה, ולא משנה איפה אתה נמצא.

    3.אינפורמציה חשובה ועוזרת. רק בשביל זה היה שווה כל הרעיון של בקורת עמיתים. תודה.

    אפשר להמשיך כמובן ברעיון, ולהראות מדוע הוא פותר לדעתי בעיות מסוימות בקוואנטים, ועוד ועוד. אך נראה לי שהתמצנו, אלא אם כן תהייה מעוניין להרחיב. הבקשה שלי אליך, ולכולם, היא אם אתה מוצא פגם בנקודות הבאות:

    1. תאוריית האתר של מקסוול הייתה נחשבת להזויה למדי, בלא הנוסחאות המפורסמות וניסוי הרץ.
    2. מכיוון שהיא הוכחה, אי אפשר להתעלם ממנה ולהחליט פתאום שאין אתר.
    3. ניסוי מ-מ אינו קביל לוגית. לא יכולה להיות מערכת מנוחה לאתר ביקום אינסופי והומוגני, כמו שלא יכולה להיות נקודת מרכז לאינסוף.
    4. מספיק שיתברר שניטרינו אחד ויחיד הצליח לעבור את מהירות האור כדי למוטט את היחסות.
    5. בניגוד לשזירה הקוואנטית, שבה אין מנוס מהמסקנה שאינפורמציית הספין עוברת מיידית, במקרה של האור קיימת לפחות אפשרות תאורטית לכך שמהירות האור זהה בכל מערכות היחוס, וזאת ללא התארכות זמנים במערכות שאינן מואצות.

    בנוגע לנקודה 5: האם ידוע על ניסוי על התארכות זמנים במערכות שאינן מואצות או כוללות אפקט דופלר? ניסוי המטוסים ונושא המואונים הם במערכות מואצות.

    מה גם, עכשיו כשברור הרעיון, ואבולוציית נושא הניסוי, האם יש למישהו הצעה לשיפור בניסוי שהצעתי?

    ובכלל: האם יש איזה שהוא פגם לוגי בכל הרעיון שהעלתי שאולי פספסתי?

    זהו. אפשר לעבור לנושאים חשובים יותר, אבל לפחות עוד 9 תגובות אה? חייבים לעבור את ה500.

    מאיר.
    אני מאוד סקרן לדעת מה אתה אומר על עניין הקשר בין הכוח הצנטריפוגלי והכוכבים הרחוקים. אינני מבין איך אפשר להתעלם מזה, אבל אינני מבין גם כיצד נפתרה הבעייה אצל איינשטיין , כך שברור שהבעייה היא אצלי.

  575. אבי, אני ממליץ שתפתח אתר ותקרא לו “יוצרי הרעיונות” שם יוכלו לכתוב כל אלו “שיש להם רעיון”.

  576. לחיצת יד? לא בגלגול הנוכחי.
    נואשתי ממנו לחלוטין. הוא עושה את עצמו קורא ואחר כך מתברר שהוא רק מסתכל על הצורה של האותיות. הוא אפילו גרם לי להרגיש שאני לא בסדר כי אני לא כותב ברור. אז נתתי לאנשים אחרים לקרוא, והם הבינו היטב. היתה לי בטן מלאה לרוקן עליו, והטריגר היה משפט שהוא מצא בחומר ששלחתי לו ולקח אותו כדי לעשות בו שימוש מגוחך. כלומר, לא רק שהוא לא טורח לקרוא דברים כהלכה וגורם לי להרגיש שמשהו פגום ביכולת הניסוח שלי, אלא הוא רואה בדברים שלי לא יותר מאשר חומר גלם לבדיחות.
    שילך לחפש ת’חברים שלו. אני בטוח לא נמנה עליהם.

  577. ישראל,
    1) איך תדע שמה שאתה רואה לא נובע מאפקט יחסותי על הזמן?
    2) אם מהירות האור אינה קבועה ויש פוטונים בכל המהירויות, אנו לא רואים את הפוטונים המהירים כי הם מהירים, אבל היכן האיטיים? למה מהירות האור קבועה גם מלמטה?
    3) תסתכל כאן : http://www.n3kl.org/sun/index.html . יש כאן נתונים שנאסא אוספת על השמש מתחנות מכדור הארץ ומלוויינים. אולי אתה יכול לחלץ משהו מהפרשי מהירות וזמנים בין התחנות? למשל, האם ההתפרצויות נמדדות קודם לכן (לאחר נירמול קורדינטות המיקום) בתחנות המהירות לעומת האיטיות? לא יודע אם זה אפשרי אבל אולי שווה לנסות.

    יובל וישראל,
    נראה לי שהייתה כאן איזו שהיא התנצלות. יאללה תלחצו ידיים וירטואליות ונמשיך הלאה. ישראל יהיה קצת יותר רגיש ויובל יהיה קצת פחות רגיש. חבל, עד שיש כאן קבוצת דיון מעניינת ואינטילגנטית שדנה בהרבה נושאים אתם מתחילים עם השטויות שלכם. אם תמשיכו כך נישאר בסוף רק עם השדים והרוחות.

  578. יובל, הגסטפו עיכב תגובה שלי לחקירה.
    אהבתי את ה”מה אתה עשית חוץ מניסיונות חשיבה לא רלוונטיים ושימוש מעוות ברעיונות לא שלך לצורך התעללות במי שלא נראה מכופתר מספיק לטעמך הסנובי?”

  579. יובל, יובל, יובל.

    בזמנו כתבת לי: “יותר משאני מתעקש לבדוק מה הקהל שלי (לא) מבין, אני מנסה להבין מדוע אינני מצליח להסביר.”

    אני חושב שיש לי תשובה.

    אתה יוצא מתוך הנחה, שאנשים יודעים מה עובר לך בראש, וזאת בלי שתתקשר איתם.

    רוצה דוגמה? כמה פעמים בקשת ממני את דעתי על המודל שלך, ושכחת דבר אחד: שאין לי מושג מהו המודל שלך. בקשת לדעת מה אני אומר על מודל החומר האפל שלך, אך לא שלחת לי (עד היום) את רעיונותיך בקשר לאותו חומר אפל. גם באתר לא כתבת.

    אני יודע שאתה מתכתב עם רבים מהמגיבים, ואתה יודע פרטים אישיים עליהם ומחליף עמם רעיונות. לי אין כל מושג מיהו ר.ח., נקודה, רפאים, או אבי ג. אינני יודע בני כמה הם, ואיפה הם גרים ( נראה לי ש ר.ח. בבוסטון, בגלל הצ’ארלס).

    כשהתחלתי לכתוב באתר, חשבתי שיהודה הוא בטח בן 25. רק אחרי שראיתי את הוידאו, הבנתי שמדובר באדם מבוגר.

    לכן כשמאיר התחיל לכתוב, חשבתי שמדובר אולי בגימנזיסט חובב מדע, וחששתי שהוא עלול לקבל את הרושם שמה שאנחנו מדסקסים הוא מיינסטרים, וזה מה שיתקבע לו בראש. מכאן האזהרות. ברור שאחרי שהתברר לי שהוא מבוגר, שולט בחומר, ובעל רעיונות ובלוג משלו, ניתן להתיר את הרצועה ולהתחרע.

    אני טורח לציין כל הזמן שהרעיונות שלי הם ספקולציות, שעד אשר יוכחו, באמצעות נוסחאות או עדיף, ניסוי, מקומם הראוי בללה לנד. אני חושב שזה נכון לגבי כולנו.

    “שימוש מעוות ברעיונות לא שלך לצורך התעללות במי שלא נראה מכופתר מספיק לטעמך הסנובי?”

    עוד הפעם, אין לי מושג למה ולמי אתה מתכוון. האם לרפאים? אינני יודע בן כמה הוא, אני רק יודע שהוא התחיל פתאום להיטפל אלי ללא שום פרובוקציה מצידי. אחר כך הגענו להפסקת נשק, אבל הוא קצת חזר לעצמו.

    אומר לך את האמת – אם הכוונה אליו, לא נראה לי כלל שהוא נפגע ממני. אולי משועשע. הראש שלו זה גוטה ואלי ממציצים, והקטע הוא שגם שלי. רפאים, אתה מוזמן לצאת מפרישתך הגאיונה, להגיב, ולהעלם.

    “את זה יכול כל פיסיקאי לבנות מאינרציה.” אבל זו בדיוק השאלה: כיצד פועלת האינרציה? התאוריה של איינשטיין אינה אינטואיטיבית, או כדבריך, פרימיטיבית. של מאך כן. ואל הנוסחה של מאיר עמירם עדיין לא הגענו. האם גם אותה עלי לנחש?

    וחוץ מזה, אינני מבין את הבעיה האישית שלך איתי. אני שוטף מוחות? את הכלים בבית אני בקושי יודע לשטוף. אני באור הזרקורים? מי מונע ממך להתכתב עם מי שרק תרצה, אם הם יהיו מעוניינים להגיב. בניגוד לדברי נקודה, אין פה עניין של זמן אויר. אם מישהו לא מעוניין במשהו, הוא יכול לדלג קדימה. דב הניס מגיב מפעם לפעם, אך לא רבים מגיבים לתגובותיו. אני ניסיתי פעם, ונוכחתי שאיני מבין מה הוא אומר, אז הפסקתי. אני מוצא עניין גדול בתאורייה של מאיר, ואשמח מאוד אם נוכל לפתח אותה ולדון בה. למי זה מפריע? גם במודל שלך הייתי שמח מאוד לדון, אילו רק הייתי יודע או מבין מהו.

    אז בקיצור יובל, מספיק עם הצ’ילבה. אם אתה רוצה שאגיב בעתיד לדבריך, ציין זאת בפירוש, אינני קורא מחשבות, אינני רוצה לפגוע בך או במישהוא, ואין לי כל עניין בקרבות אגו.

  580. ר.ח.
    ראשית, בוא נגמור את נושא ל.ס.

    מסה לפי ל.ס. היא רשת עם חורים. חלק עוברים, חלק לא.
    חור שחור הוא קיר בטון. כלום לא עובר.
    התאוריה נבדקה עיי שמנה וסלתה, כולל ניוטון עצמו. אף אחד לא טען שלא תיווצר גרביטציה (עד כמה שידוע לי). אף אחד גם לא טען שהצורה הגאומטרית חשובה. הטענות היו אחרות, והחשובה בהן בעיית החיכוך. אני מאמין שהרעיון שלי יכול לפתור זאת, אך כפי שציינתי פעמים רבות, זוהי הצגת משנה. הנושא החשוב לי הוא היחסות. לזאת ניגש עכשיו.

    ” מבחינה טכנית תהיינה בעיות רבות שהרי החיישן הנע יהיה במרחק אחר ממקור האור מאשר החיישן הנח”.

    אז זהו, שלא. הרעיון הוא למקם את שניהם קרוב מאוד זה לזה.

    הרעיון הראשון שלי היה למקם שני גלאים קרובים, אחד רגיל והשני ליד שפופרת קתודית אשר בה האלקטרונים יכולים להגיע למהירויות גבוהות מאוד. לירות פוטון באלקטרון הנע, ואז, עקב אפקט קומפטון והסטת האלקטרון, למדוד את מקום וזמן פגיעת האלקטרון המוסט ולהשוות אותו לזמן הפגיעה של הגלאי לידו, שבו פוגע פוטון רגיל.

    הרעיון השני היה לבדוק אם אפשר לעשות משהו עם מאיצי חלקיקים. בשביל זה נפגשתי עם גיל טרייביש ממעבדת מאיצי חלקיקים בUCLA. לא אלאה אותך בפרטים, אך בשורה התחתונה, לא. לא כרגע.

    הרעיון השלישי הוא טכני, כפי שאני אוהב. נסה לקרוא ולראות אם יש בו איזה שהוא כשל לוגי או טכני.

    מה שאנחנו רוצים הוא, שני גלאים באותה נקודה, אחד נח והשני נע במהירות. זה כדי לא להכעיס את איינשטיין.
    ניתן לעשות זאת בצורה הבאה: משדר רדיו, שולח סיגנל ממוקד לנקודה המרוחקת ממנו 300 ק”מ. בנקודה יש שני מקלטים: הראשון בעל אנטנה רגילה, והשני בעל אנטנה בצורת דיסק המקביל לקרקע ומסתובב במהירות עצומה. מה שיצא הוא, שמבחינת הסיגנל הפוגע במשדר ב’, המהירות המשיקית של הדיסק זהה לגביו לאנטנה שטסה באותה מהירות.

    הזמן החולף לסיגנל לעבור מהמשדר למקלטים הוא כאלפית השניה. הטיימרים שבררתי עליהם מדוייקים ברמה של כמעט פיקו שניות. הייתי רוצה טיימרים של פמטו שניות, אך אלו הרבה יותר מסובכים.

    זה הרעיון. בדוק, כסח, והגב.

  581. אמרתי: “תנועת כל המאסות ביקום היא ביחס לחומר האפל. התנועה ההדדית בין מאסות היא רק תוצר משני”.

    הוא ענה: “ראשית, רצוי שלא נבלבל קוראים בין הדעות המקובלות והדעות הפרטיות שלנו. עיקרון מאך, אליבא ד’מאך, הוא כפי שהצגתי אותו. התוספת: “תנועת כל המאסות ביקום היא ביחס לחומר האפל.” היא רעיון שלך, וצריך לציין זאת, כדי לא לבלבל קוראים העלולים לחשוב שזו דעת המיינסטרים. עד כמה שידוע לי, לא. אינני אומר שזה לא רעיון מעניין, אך עד שיתבסס, חייבים להתיחס אליו כספקולציה”, וסתם עליו את הגולל.

    לזרוק “ספקולציה” אחת כדי לפנות מקום לאחרות; להשליך דעה פרטית שלי כדי לקדם דעות פרטיות שלו; להזהיר את הקוראים שדעתי אינה מקובעת במיינסטרים, כאילו שדעותיו כן. דמגוג! סרסור של מילים ושוטף מוחות.

    להבדיל מן הספקולציות שלו, שממלאות את החלל בהמון הנחות ישיות, על קיומו של החומר האפל אנחנו כבר יודעים.

    ישראל! עשיתי לך הנחה, אך רק כאן באתר. לא יהיו יותר מיילים פרטיים. שאלת איך ציר הגלגל המסתובב שומר על כיוונו במרחב. את זה יכול כל פיסיקאי לבנות מאינרציה. אם אתה שואל מה יוצר את האינרציה, אז אלברט איינשטיין כבר קישר בין אינרציה לגרביטציה (ר’ ת”ה הכללית) ומאיר עמירם גם הראה נוסחה מקורית שלו לעניין הזה. אני מציג את הגרביטציה/אינרציה כאינטראקציה בין החומר האפל לבין עצמו וכנראה אשתמש גם בנוסחאות של מאיר בשלב החישובים. ומה אתה עשית חוץ מניסיונות חשיבה לא רלוונטיים ושימוש מעוות ברעיונות לא שלך לצורך התעללות במי שלא נראה מכופתר מספיק לטעמך הסנובי?

  582. ישראל,
    קודם כל שיהיה בהצלחה.
    דבר שני נראה לי שתצטרך אותה כי מבחינה טכנית תהיינה בעיות רבות שהרי החיישן הנע יהיה במרחק אחר ממקור האור מאשר החיישן הנח ותצטרך גם מדידת מרחק מאד מאד מדוייקת על מנת לקזז את אפקט המרחק (אלא אם כן תערוך את הניסוי בין כדור הארץ לאנדרומדה ואז יהיה זניח אבל מן הסתם יתעוררו בעיות טכניות אחרות).

    נקודה אחרונה, לאחר שתסיים, בהנחה שאכן תראה הבדל בזמנים, תערוך מסיבה, תפתח את השמפניות ואז תישמע דפיקה בדלת. ומי יעמוד שם אם לא איינשטיין. ומה שהוא יגיד לך זה: “הר שפירא, הרי זה ברור שהמערכת הנעה תראה פוטונים לפני זו הנחה מאחר וכמו שטענתי בתורת היחסות הפרטית שלי במערכת הנעה מהר המרחקים יתקצרו והזמן יעבור יותר מהר. יתירה מכך,” הוא יוסיף ויאמר “האפקט הזה הוכח כבר נסיונית ע”י שני שעונים אטומיים שאחד טס והשני במנוחה כמו שלמיטב זכרוני, שלצערי כבר אינו כמו שהיה באותה שנה מופלאה 1905, הראה לך ר.ח פעם באחד התגובות שלו”.
    אל תשכח להכין בשבילו שנאפס, הוא שונא שמפניה.

  583. ר.ח.
    הפרש לחצים איננו רוח, למרות שהוא יכול לגרום לרוח. בכדור הרגל שילדיך בועטים יש הפרש לחצים עצום בין החוץ לפנים, אך הוא מתבטא בכוח, לא רוח ולא אנרגיה. כך גם הגרביטציה, לסאז’ית או ניוטונית, מתבטאת ב כ ו ח משיכה. גם הכסא עליו אתה יושב מפעיל כוח, אך רק אם תהיה תנועה בכיוון הכוח, תוכל לדבר על אנרגיה, כמו בסירה המאיצה באמצעות מפרש.

    ישר לניסוי.
    תוכל ללכוד את פגזנו, אם תניע את הארץ בכיוון תנועת הפגז. אם מהירות הפגז היא 12 קמ/שנ יחסית לארץ, הוא ימלט ולא ישוב. אולם אם תניע את הארץ במהירות של 1 קמ/שנ לכיוון תנועת הפגז, מהירות הפגז היחסית לארץ תרד ל 11 קמ/שנ, הוא ילכד.

    לכן, אם יש משהו בשטויות שלי, ותשלח קרן אור, המורכבת מפוטונים רבים, ובציוד הקצה תהיינה שתי מערכות גילוי המצויידות בטיימרים מדוייקים, אחת נחה יחסית למקור הקרינה והשניה נעה במהירות גבוהה יחסית לראשונה, תלכוד המערכת השניה פוטונים שהיו שקופים מבחינתה אילו לא נעה, וזאת לפני אחותה הנייחת.

    ומכיוון שהמערכת הראשונה קולטת פוטונים הנעים במהירות האור, הרי אם קלטה המערכת השניה פוטונים עוד לפני הראשונה (ואת זה יראו הטיימרים), הרי שהיא קלטה פוטונים הנעים במהירות העולה על זו של האור.
    מ.ש.ל.

    לפי הפרוש הזה, יתכן שהבעיה באופרה בסרן היא בכלל בציוד הקצה, המורכב מרכיבי ציוד (חלקיקי יסוד לדוגמה) הנעים במהירות גבוהה בכיוון תנועת הנייטרינו.

    אפשר להמשיך לסתירות שקיימות לדעתי אצל איינשטיין (זוכר את שעוני הטמפרטורה?), אך נראה לי שהתמצינו קצת לעכשיו. מה גם שכנראה מצאתי איזה ציוד יכול להוות טיימר בדיוק של פיקו שניות, כך שנראה לאן זה יתגלגל.

    מאיר.
    לא ברור לי איך אפשר להתכחש לעקרון מאך. הרי מספיק שתסתובב בסחרחרת בלונה פארק, ותצלם את הכוכבים למעלה בוידאו. אם תסתכל על הוידאו, האם לא תוכל לדעת רק לפי הוידאו עד כמה הסתובב לך הראש בזמן הסיבוב?

  584. ישראל,

    התשובה לגבי מהירותם היחסית של חלקיקי ל.ס. לא מניחה את דעתי. קצת קשה לי כרגע להעביר את חוסר הנוחות הזה להסבר משכנע, אז לכשיבשיל אכתבהו.

    לגבי מאך, המבחן איננו מדוע צלחת מעופפת מסתובבת יחסית לכוכבים הרחוקים, אלא האם לכשנסובב את הכוכבים הרחוקים סביב הצלחת היא תרגיש בכך, והתשובה שלי היא “לא”.
    כמובן שלשיטתי עקרון מאך איננו נכון.

    אתה כותב “אולם לדעתי, כל פתרון אינטואיטיבי, וזה מה שאני מחפש, למנגנון הגרביטציה, חייב לכלול מנגנון דמוי ל.ס. אחרת איך יודע יופיטר שאני מזיז את היד בארץ?”

    לא ברור לי למה אתה מחפש דווקא פתרון אינטואיטיבי. אינטואיציה זה דבר נזיל.
    גם לא ברור לי על סמך מה אתה בטוח שיופיטר יודע שאתה מזיז את היד בארץ.

    לשיטתי יופיטר יודע שאתה מזיז את היד בארץ אבל ברזולוציה הרבה יותר נמוכה ממה שאפשר לצפות מגרביטציה לה סאז’ית. הוא יודע זאת באמצעות במנגנון לחלוטין בלתי לה-סאז’י (ולכן גם יפהפה) שיתואר על ידי ככל הנראה במאמר העשירי. בשלב הזה אני מרשה לעצמי רק להזכיר לצורך העניין את מה שכבר כתבתי במאמרים קודמים, שגרביטציה היא פלואידית, ומתפשטת סביב מסה לא בצורה קוית אלא בצורה נפחית. היא ממלאת את המרחב מבלי להשאיר “רווחים”.

    יופיטר יקבל מסר על הזזת היד, אבל המסר הזה יטשטש ויתערבל במסעו עד יופיטר תוך התמזגות עם שדה הכבידה הכללי של ארץ עד לחוסר יכולת להבחין בו (כאלגוריה בלבד, שכשוך יד באגם יורגש על ידי אדם שעומד קרוב, אבל ילך ויטשט עם המרחק). לעומת זאת אם מסה כלשהי תיעלם מכדור הארץ, נניח בצורת קרינה (או בצורת ניסוי מחשבתי), יופיטר יחוש בשינוי מסת הארץ באופן מיידי.

    הנה לך (ובינתיים אי אפשר להשמיץ את המאמרים שלי בהזכרת אקדח במערכה השניה מבלי שהוא יורה במערכה החמישית) מנגנון שאיננו דמוי ל.ס. שמודיע על נוכחות מסה במהירות הרבה יותר גבוהה ממהירות האור.

  585. ישראל,

    כנראה שאני לא מבין את ל.ס כי גם אחרי כמה פעמים שעיינתי בתאוריה שלו (טוב, אני מודה בויקפדיה) אני לא מצליח להבין איך אם הוא מדבר על הפרש לחצים משני צידי גוף, לא תיווצר רוח. רוח = תנועת אוויר ממקום לחץ גבוה לנמוך, לא אני אמרתי את זה, דני רופ אמר. ברור מהאיורים בתאוריה שלו שנוצרת זרימת חלקיקים למקום הלחץ הנמוך. מה זה אם לא רוח??
    וממתי צורתו הגאומטרית של גוף אינה משפיעה על רוח? כבר אמרנו ימאות.

    לדעתי יש גם הבדל מאד גדול בין ניוטון המתייחס למרכז המסה כנקודת מוקד הגרויטציה לבין ל.ס. לפי ניוטון אין קשר למימדים של המסה (חור שחור יכול להכיל מסה יותר מכוכב צדק למשל) והגרויטציה נובעת מנקודה גיאומטרית חסרת מימדים במרכז המסה.
    לעומת זאת לפי ל.ס גודל המסה ומימדיה משנים גם משנים. ולכן ההפרכה ל ל.ס היא כוכב עם מנהרה. לפי נוטון תיווצר תנועה הרמונית במנהרה כי הפגז יימשך למרכז הכדור. לפי ל.ס (וגם לפי האתר שלך) אין שום סיבה לתנועה הרמונית.

    דבר נוסף שאדון ל.ס אם היה חי או אתה ויהודה נציגיו עלי אדמות צריכים להסביר הוא איך ייתכן לאור “הפרשי הלחצים שלכם” שלחור שחור בקוטר מטר תיתכן גרויטציה יותר חזקה מזו של השמש?

    לגבי החידה שלך, אם אתה אומר ש “אנו, כנסיינים חיצוניים למערכת, בעלי כוחות על להזזת מסות” אזי כל שעלינו לעשות הוא להגדיל את מסת הארץ והפגז יילכד. האם זה מה שהתכוונת?

    יאללה תן את הניסוי ונגמור עם זה.

  586. מאיר
    את מהירות חלקיקי ל.ס., כמו כל מהירות אחרת, מודדים אך ורק יחסית זה אל זה, או יחסית לצופה, או מודד, חיצוני. במודל ל. ס. האורגינלי, מהירותם לפחות 100,000 פעם מהירות האור. אצל ראובן ניר, מהירותם כמהירות האור. ( ואני שואל: אז מה מהירות המערכת הכוללת את כל החלקיקים ביחידת נפח? יחסית למה? אלא אם כן מניחים איזו מערכת מנוחה מוחלטת כלשהיא, עיין ערך ניסוי מ”מ.). אצלי, לכל מודד בכל מהירות של המודד , לחלקיק בודד תהיה מהירות שונה, כמו מהירות מולקולות אויר יחסית למודדים שונים. לכן החלקיקים נעים במהירויות מ 0 עד אינסוף, לכל הכיוונים.

    בקשר למודל שלך. עדיין לא הבנתי איך זה מסתדר עם עקרון מאך. אם מקור האינרציה הוא החלקיק עצמו, מדוע צלחת מעופפת מסתובבת שמודדת כוח צנטריפוגלי, מסתובבת גם יחסית לכוכבים הרחוקים? איך הם קשורים?

    אבל אל יאוש. אותה שאלה אפשר לשאול גם לגבי המרחב זמן של איינשטיין (שאלתי, שאלתי), כך שכנראה חוסר ההבנה הוא אצלי. אבל אם תוכל להסביר, אז ממש ממש יופי.

  587. ר.ח.
    נכון, מהירות הגל היא ביחס לתווך כמובן. אבל לכל התווכים המוכרים, אויר, מים, חוט, מהירות 0 יחסית לגורם חיצוני כלשהו. לכן דיברתי על “מערכת סגורה”. אם אתה מאמין שלא, תן דוגמה סותרת. ב”אתר האקטיבי” המערכת היא פתוחה (זוכר את מקס?). מהירות התקדמות הגלים היא אך ורק יחסית לצופה, או מודד, ולכן זהה בכל מערכת יחוס.

    דווקא עניתי לך בקשר לכוכב עם חור: “הגרביטציה אצל לסאז’ זהה לגמרי לזו של ניוטון, רק לסאז’ נותן לה סיבה, ניוטון לא. לכן תהיה תנועה הרמונית לפגז שהצעת.”

    אני חושב שיתכן שלא לגמרי נהירה לך ( או לי ) תאוריית ל.ס. כי אתה ממשיך להזכיר “רוח” וצורה גאומטרית לגוף. אילו אינן מופיעות בתאוריית ל.ס. ע”ע. הגרביטציה אצל ל.ס. כאמור, זהה לגמרי לזו של ניוטון, ומכיוון שאצל ניוטון אין משמעות לצורה גאומטרית, גם אצל ל.ס. לא.

    “עוד לא ענית לי מה גישתך לגרביטציה. האם אתה חושב שחלקיקי האתר האקטיבי הם הגורמים במהירות נמוכה ע”י דחיפה לגרביטציה? בדומה ללאסג’ הזכור לטוב?”

    אינני יודע. אותי מעניינת כרגע יותר בעיית מהירות האור. רק ציינתי שאותו פתרון לגבי מהירות האור, יכול לפתור את בעיית החיכוך אצל ל.ס. מכיוון שאינני מצוי ברזי המסה האפלה, ומייקל שכנע אותי בעבר שהיא נבדקה תוך התחשבות בפושינג גרביטי, אז הנושא עדיין פתוח, לפחות לגבי, למרות שמעניין מאוד לראות איך האינטגרציה הייתה עובדת על מסה אפלה אם היינו מחליפים גרביטציה ניוטונית בפושינג גרביטי, ומוסיפים את עקרון מאך. אולם לדעתי, כל פתרון אינטואיטיבי, וזה מה שאני מחפש, למנגנון הגרביטציה, חייב לכלול מנגנון דמוי ל.ס. אחרת איך יודע יופיטר שאני מזיז את היד בארץ?

    בחדר אטום תהיה גרביטציה לפי ל.ס. הקירות אינם מהווים מחסום מוחלט מפני חלקיקי ל.ס., שום דבר לא מהווה בידוד מפני חלקיקי ל.ס. כמו שאי אפשר לבודד גרביטציה. אבל כן, הקירות נמשכים קצת זה אל זה אצל ל.ס. (גם אצל ניוטון).

    “לסיכום, מה שאני מבין מהמודל שלך הוא שאתה חושב שיש אתר עם חלקיקים בעלי מהירות משתנה מ-0 ועד אינסוף. מעל מהירות האור הם שקופים לגבי המסות ויכולים לעבור דרכן. פחות מזה הם דוחפים את המסות ויוצרים את הגרויטציה.”

    כן. רק המהירות היא יחסית זו אל זו, אין דבר כזה מהירות שאינה יחסית. לחלקיקים יש גם את התכונות שמקסוול ציין, של דיאפולים מגנטים.

    נמתין כרגע עם הניסוי, יש לי גרוי בעין, קשה לכתוב. אבל הנה חידה שתמחיש את רעיון הניסוי: בדוגמת הפגז שמגיע למנהרה בארץ מעל למהירות המילוט, ובהנחה שזהו פגז חשוב מאין כמוהו, איך אנו, כנסיינים חיצוניים למערכת, בעלי כוחות על להזזת מסות, אך בתחום הפיזיקה המוכרת, יכולים ללכוד את הפגז עיי הארץ?

  588. ישראל,

    בעוד אשר פתרת את ייחוס מהירותו של גוף למהירות הממוצעת של חלקיקי ל.ס. (שהיא תמיד אפס ככל שדנים באותם חלקיקי ל.ס. להם הגוף המסיבי איננו שקוף), לא פתרת ביחס למה מודדים את מהירות חלקיקי ל.ס. עצמם.

    באמת ביחס למה? ביחס לריק המוחלט של ניוטון?

  589. ישראל,
    כנראה שאני לא מצליח להבהיר את עצמי. מה הקשר לכך שהצופים יכולים את הכיוון??
    הנקודה היא שמהירות גל היא יחסית לתווך או לחלקיקים שמניעים אותו ולא למשהו חיצוני.
    בנוסף, להבנתי, גל הוא ההתקדמות של עירור או מצב חלקיקים (ולא החלקיקים עצמם). המתנד עליו אתה מדבר הוא זה שיוצר את הגל והוא לא מן העניין.

    לפני הניסוי. עוד לא ענית לי מה גישתך לגרביטציה. האם אתה חושב שחלקיקי האתר האקטיבי הם הגורמים במהירות נמוכה ע”י דחיפה לגרביטציה? בדומה ללאסג’ הזכור לטוב? אם כן, אני לא עדיין לא מבין מדוע הגיאומטריה של המסה לא משפיעה של צורתה. גם לא התייחסת לניסוי שהצעתי לגבי כוכב עם חור.
    המודל שלך, אם אני מבין נכון לא מתייחס לגרויטציה כאל נקודה במרכז המסה כמו בניוטון.
    למעשה, במחשבה נוספת בחדר אטום לפי המודלים של גרויטציה לפי דחיפה לא הייתה צריכה להיות גרויטציה כלל וכלל כמו שאין רוח.

    לסיכום, מה שאני מבין מהמודל שלך הוא שאתה חושב שיש אתר עם חלקיקים בעלי מהירות משתנה מ-0 ועד אינסוף. מעל מהירות האור הם שקופים לגבי המסות ויכולים לעבור דרכן. פחות מזה הם דוחפים את המסות ויוצרים את הגרויטציה.

    אם נראה לך שאני לא מבין את המודל אתה יכול לתמצת אותו ב 3 – 4 נקודות.
    אם הבנתי אתה יכול להמשיך לניסוי.

  590. אגב, שמתי לב שבגזרות ת”ח של פונקציות הגלים, הפכתי את הסדר זמן/מרחק. אך הכוונה היתה טובה.
    ובענין הקזינואים – אני נוסע רק לתערוכות. בעבר היה לי איתם דיל. הם היו נותנים לי סוויטות, ארוחות, הצגות, צ’ופרים, הכל בחינם – ובתמורה הייתי לוקח את הכסף שלהם. אבל עכשיו הקמצנים התהפכו עלי כמו יובל, כך שאני כמעט לא מגיע לשם יותר.

  591. יובל.
    כרצונך. למרות שלא ברור לי מה אמרתי עוד הפעם. עייפתי מלנחש את כוונותיך. אם יש לך משהו להגיד – הגד.

    ר.ח.

    קודם כל, הגל באצטדיון איננו גל טבעי. כל צופה יכול להחליט לקטוע אותו או להפוך כיוון. זה שונה אצל גלי קול.

    1. במקרה של גל קול, יש כאן שני דברים – מהירות התקדמות הגל, הנקבעת לפי מהירותן הממוצעת של מולקולות האויר יחסית למרכז המהירות הממוצע שלהן ליחידת נפח המסוגלת לשאת גלי קול.

    במקרה של אויר המהירות הממוצעת היא כ 40% מעל מהירות הקול. יחסית למה? אם תשים את האצבע בנקודה מסויימת בחדר ללא רוח, שבו מהירות הקול כ 330 מ/שנ, ותעשה שקלול ממוצע של כל מהירויות המולקולות יחסית לאצבע, תמצא שהמהירות הממוצעת היא כ480מ/שנ (אם אני זוכר נכון).

    2. אם תנשוב רוח, מהירות הקול תהיה עכשיו 330 מ/שנ יחסית לעלה נידף ברוח, שמהירותו 0 יחסית לרוח, אך מהירות העלה והרוח יכולה להיות 50 מ/שנ יחסית לאדמה.

    אין כל קשר למהירות העירור. אחרת היינו מקבלים מהירויות קול שונות למתנדים שונים בעלי מהירויות מתנד שונות. הדבר אינו כך. הקשר היחיד למתנד – או העירור – הוא בגובה הצליל, אך לא במהירות ההתקדמות של הגל.

    3. אין לי מושג למה הוא עוד הפעם הצ’ילבה שלי. אני מתעניין בטכנולוגיה, לא פסיכולוגיה.

    4. כשתהיה מוכן, נוכל להתקדם לקראת מבחן ההפרכה והניסוי.

  592. ישראל,

    אני לא מסכים עם הקביעה שלך “מרכז המהירויות של האנשים באצטדיון, הוא האצטדיון עצמו. לכן הגל גם מתקדם יחסית לאצטדיון.”. שים אלף אסטרונאוטים בחלל ותגיד לראשון להרים ולהוריד את ידיו. לשני תאמר לעשות אותו דבר ברגע שראה את הראשון מוריד את ידיו ואז השלישי והרביעי ותראה גל שמתקדם. לאיצטדיון אין שום קשר לגל או למהירותו. זו נקבעת כפונקציה של הזמן שעבר בין החלקיקים שעוברים עירור (אסטרונאוטים שמרימים את ידיהם בדוגמא הנ”ל) למרחק שבינהם.

    וחוץ מזה אם אתה כזה פרסונה נון גרטה בקזינו מה אתה נוסע לוגאס כל כך הרבה?
    וחוץ מזה מה אתה מרגיז את יובל שלנו?

  593. ר.ח.
    מרכז המהירויות של האנשים באצטדיון, הוא האצטדיון עצמו. לכן הגל גם מתקדם יחסית לאצטדיון. גם גל קול ברוח, יתקדם במהירות הקול יחסית לרוח, ולא לאדמה.

    תוכל לראות זאת יפה אם תיקח את משוואות הגלים הקלסיות. הפונקציה היא של שני משתנים, מרחק וזמן. אם תגזור חלקית את הפונקציה יחסית למרחק, תקבל משוואה של תנועה הרמונית פשוטה. אך אם תגזור אותה חלקית יחסית לזמן, תקבל גל ק ו ס י נ ו ס קפוא, יחסית למערכת הצירים, שבדוגמת האצטדיון, יהיה זהה למערכת האצטדיון.

    אולם מה תקבל בדוגמת האתר האקטיבי?

    ובקשר למקסוול ולורנץ: לב הטיעון שלי היה שאני לא כמותם. שצריך דה וינצ’י, או דקארט, או גלילאו, כדי לקרוא תגר על אריסטו. אני לא.

    נכון, הקזינואים רעי הלב חושבים אחרת, ותקעו את פרצופי המכוער במערכת ה FACIAL RECOGNITION, אך אני יודע את האמת המרה. BY DEFAULT – יש לי טעות בחישובים.

  594. יובל
    נכון שהתרגיל לא הצליח. כל מי שהסתכל על הגדרת המודל שנתתי, היה אמור להגיע למסקנה שמדובר במודל הזוי לחלוטין, קשקוש בלבוש בקש מבולבש. בתסריט המקורי, רפאים הפתי היה אמור לצעוד למלכודת, לכסח את המודל בדרכו הציורית הרגילה, ולשלוח אותי פעם נוספת למכון לרפי שכל.

    אני הייתי אמור להסכים איתו עקרונית, להנהן הנהוני הסכמה, להתעניין אם הוא יוכל לסדר לי מיטה נוחה במוסד, רצוי ליד החלון, ואז לציין דרך אגב, שזהו מודל האתר של מקסוול, יצירת הפאר המדעית של המאה ה19, שבאמצעותה הוא חישב את מהירות האור מתוך קבועי החשמל והמגנטיות ואת המשוואות האלמותיות שלו.

    כאמור – הטריק לא עבד. רפאים הערום הריח מלכודת, ונמנע מלצעוד אליה. מה שמביא אותי להצעה לרפאים: בוא, הצטרף אלינו בללה לנד. קח כוס בירה, הבא את אחד מהרעיונות שיש לך (שמתי לב שאתה מתעניין במתמטיקה), הפסק לחשוש ממה יחשבו ה”מבינים” ושטח רעיונותיך לבקורת עמיתים.

    אם תחליט להשאר בצד המומחים המכסחים – הנה הצעה לשיפור הטכניקה. במקום לרדת עלינו אישית – כסח את רעיונותינו. אתה יותר מדי ירוק, ואנחנו יותר מדי זקנים ועייפים, שנתרגש מהשטויות שלך. לעומת זאת, אם תכניס לנו בעקב אכילס שלנו, התאוריות, תוכל להוציא אותנו משווי משקל, כמו שעשית לי עם המטען החשמלי המואץ.

    התחל עם התאורייה של מאיר. מה שהוא אומר זה דבר פשוט: כל חלקיק יוצר גרביטציה, ולכן אם ננסה להזיז את החלקיק ממרכז הגרביטציה של עצמו, יווצר כוח המתנגד לתנועה, או בקיצור, אינרציה.

    1. האם יש פה איזו בעייה מובנית?
    2. כיצד זה מסתדר עם עיקרון מאך? הרי לפי מאך, התאוצה היא ביחס לכוכבים הרחוקים, ראה דוגמת הצלחות המעופפות המסתובבות, זו שאינה מודדת כוח צנטריפוגלי גם אינה מסתובבת יחסית לכוכבים.

    בצורה כזו, גם תעורר כבוד, וגם, אם זה בכלל חשוב לך, תוכל לעזור לנו לראות דברים שאולי פספסנו.

  595. ישראל,
    אני חייב לציין שאני לא מבין.
    גל קול באוויר הוא תנודה חזקה של מולקולות במקום מסויים ביחס למה שהם היו קודם ולא ביחס לאדמה או כל דבר אחר. רואים את זה יפה בגל במשחק כדורגל. אנשים יושבים, קמים, יושבים ואז אלה שלידם עושים אותו דבר ונוצר גל. הגל לא נע ביחס לאיצטדיון אלא ביחס למצב האנשים קודם.
    יש “עירור” זמני של האנשים או המולקולות ביחס למצבם קודם וכשהעירור הזה מתקדם זהו גל.
    לסיכום, מה שמתקדם בגל הוא מצב החלקיקים ולא החלקיקים עצמם.

    בנוסף לא ברור מה הטיעון שלך לגבי מקסוול ולורנץ. זה שהם לא חשבו על זה קודם אומר שזה לא נכון? בגלל הלך מחשבה כזה העולם נתקע בימי הבניים מאות שנים כי “אם אריסטו לא אמר את זה קודם אז זה כנראה לא נכון”. תתבייש לך דקה ותמשיך מייד עם קו המחשבה שלך. רוב התגליות וההמצאות הן כאלה שאח”כ אומרים “וואלה, איך לא חשבו על זה קודם??” עובדה. לא חשבו על זה קודם!

  596. ישראל,

    אני מסכים איתך שבהתעלם מהפתרון הפשוט שהצגתי, אינרציה זה עניין מסובך.

    אחר שהבננו מי האבא והאמא של אינרציה 🙂 נוכל לתאר לעצמנו יותר מדוגמה אחת של יקום ללא אינרציה:

    לדוגמא, יקום בו מסות מקובעות למרחב, אך פוצחות בתנועה במהירות האור אחר שהופעל עליהן כח בעוצמה שמעבר לרמת סף מסויימת (אסוציאציה פרוידיאנית לאפקט הפוטואלקטרי).
    או לדוגמא יקום בו מסות מגיבות לכח המופעל עליהן ברציפות, במהירות העומדת ביחס ישר לכח הפועל, ועוצרות במקומן ברגע שהוסר הכח.

    חלק מהיקומים ללא אינרציה הם אפלים. אם נעקוב בעתיד אחר הכתיבה של זה שאיחד את חוקי ניוטון, נגלה שלא בהכרח צריך לנדוד ליקומים מקבילים כדי ליהנות מחוסר אינרציה. יובאו שם דוגמאות לחוסר אינרציה בחלקים האפלים שביקומנו אנו.

  597. ישראל!
    אני לא מדבר על ההתעללות שלך בחסרי ישע, אלא על הסלט שאתה מקדיח

  598. ישראל,
    טוב אז בנתיים שאנחנו מחכים לחברה הטובים שיגמרו למיין את התגובות המצונזרות אין לנו אלא להתענג על תנ”כנו. להלן פסוקו של יום:

    וּבְאַרְבַּע עֶשְׂרֵה שָׁנָה בָּא כְדָרְלָעֹמֶר וְהַמְּלָכִים אֲשֶׁר אִתּוֹ וַיַּכּוּ אֶת רְפָאִים

    מֵתִים בַּל יִחְיוּ רְפָאִים בַּל יָקֻמוּ לָכֵן פָּקַדְתָּ וַתַּשְׁמִידֵם וַתְּאַבֵּד כָּל זֵכֶר לָמוֹ

    ִחְיוּ מֵתֶיךָ נְבֵלָתִי יְקוּמוּן הָקִיצוּ וְרַנְּנוּ שֹׁכְנֵי עָפָר כִּי טַל אוֹרֹת טַלֶּךָ וָאָרֶץ רְפָאִים תַּפִּיל

    אָדָם תּוֹעֶה מִדֶּרֶךְ הַשְׂכֵּל בִּקְהַל רְפָאִים יָנוּחַ.

    תגיד, זה לא קצת מוקדם לכם ב LA?

  599. אנא ערף, הסתכלתי בפסוק של רפאים, ומה מצאתי?

    “מִי פֶתִי? יָסֻר הֵנָּה!”

    וְלֹא יָדַע כִּי רְפָאִים שָׁם, בְּעִמְקֵי שְׁאוֹל קְרֻאֶיהָ.

  600. ר.ח.

    המוחבראת לקחו את התגובה שלי לחקירה. נקווה שישחררו אותה בקרוב. הנחתי אותה לא להסגיר שום אינפורמציה מלבד שם, דרגה, מספר אישי, וכמה מלמולים הזויים כיאה לאביה מולידה.

  601. ר.ח.
    הנה הגיע הרגע: קביעות מהירות האור בכל מערכת ייחוס.
    ראשית, וידוי מביש: אני לא ממש מתעניין בלסאז’. משאיר את זה ליהודה. ועוד סוד נורא: דוגמת הפגז לא הייתה שלמה. היא הובאה רק כדי להכשיר את הלבבות ללב הטיעון: שקיפות החומר לחלקיקים היפוטתים מעל מהירויות מסויימות.

    בוא נחזור למקסוול.
    ראינו שהוא הצליח באמצעות המודל שלו לקבוע את מהירות האור מתוך קבועי החשמל והמגנטיות. השאלה היא: מהירות האור יחסית למה?

    מקסוול, לורנץ, ורבים מבני דורם, הניחו שיחסית לאתר. ומכאן נסיון מייכלסון ומורלי: הנסיון לקבוע מהי מערכת המנוחה של האתר.

    עכשיו: לאורך כל הדרך, אני אומר שכנראה שיש טעות במודל שלי. זה הזמן לאמר מדוע, לוגית, חייבת להיות לי טעות.

    היחס שלי לשמות כמו “מקסוול” או “לורנץ” מקביל ליחס של יהודי חרדי לשמות כמו “הרמב”ם” או “הרבי מלובביץ” בהבדל אחד: החרדי מאמין שיש מישהו מעל גיבוריו. אני מאמין שהם הדבר עצמו.

    כשמייקלסון מתכנן ניסוי שמטרתו לקבוע מהי מערכת המנוחה של האתר, השאלה הראשונה שצצה לי היא: איזו מערכת מנוחה?

    הרי תמונת היקום ב1887 היא של יקום אינסופי, הומוגני ואיזוטרופי. קיימת אמנם הסתייגות SEELIGER, על התפלגות הכוכבים, אך היא אינה קשורה לעניינינו.

    לכן, אם נאמר שהיה מייקלסון מצליח למצוא את מערכת מנוחת האתר, ונאמר שהיא נעה יחסית אלינו במהירות של 1887 קמ/שנ לכיוון קונסטלציית האריה, היינו צריכים לשאול: למה דווקא זו? מה מיוחד בה? איפה ההומוגניות?

    זה כמו למצוא את נקודת מרכז היקום האינסופי. למה דווקא זו?

    וכאן אני יודע שחייבת להיות לי טעות. אין, אין מצב שאני חשבתי על זה ולורנץ לא. נכון, התקבלתי להרווארד, אבל רק לתואר ראשון. ז”א התקבלתי למשרת מנקה השרותים בהרווארד, אבל רק של הסטודנטים לתואר ראשון. לשרותי הסגל לקחו מישהו יותר מתאים.

    אז זה שלא ידוע לי שמקסוול ולורנץ העלו הסתייגות זו, מראה לי לוגית, שיש לי טעות.

    אבל עד שאדע איפה הטעות, אני חייב לנסות לפתור את התעלומה, כדי שאוכל למות מאושר.

    אז הנה הרעיון.

    נחשוב על גל קול באויר. הוא חסר משקל, אך יש לו תנע ומהירות. ממש כמו לפוטון. השאלה היא: מהירות יחסית למה?
    כאן יש לנו תשובה יפה ופשוטה: יחסית לממוצע מהירויות מולקולות האויר. אם אין רוח, יחסית לאדמה. אם יש רוח, יחסית לרוח. אם באוירון, יחסית לאוירון.

    אך מה היה אם היקום עצמו, האינסופי, ההומוגני, היה מלא במולקולות אויר? יחסית למה היה נע גל הקול?

    הרי אם היקום אינו מתפשט (לא לשכוח, 1887, יקום אינסופי ואיזוטרופי), והוא מלא במולקולות אויר, הרי שאנו אמורים לקבל גל קול, ואפילו בעל מהירות מסויימת ומוחלטת, אך יחסית למה?

    כאן בא לעזרתנו מודל האוטוסטרדה: במקום מימד אחד, נפתח אותה ל 3 מימדים. במקום 100 מסלולים למימד, אינסוף מסלולים למימד. זהו ההגיון היחיד שאני יכול למצוא.

    וכמו בדוגמת האוטוסטרדה, המולקולות היחידות שקיימות עבורינו בכל מהירות, הן אלו שמתחת למהירות מסויימת. כל המהירות יותר שקופות עבורינו. אם נעבור למהירות שונה, מולקולות אשר מקודם השפיעו עלינו, יהפכו פתאום לשקופות, ומולקולות שהיו שקופות, פתאום תהפכנה לממשיות.

    וגל הקול, (גל קוסינוס! לא סינוס) יתקדם תמיד במהירות אחת וקבועה יחסית לנו, לא משנה באיזה מערכת יחוס, או מהירות, נהייה. כי כפי שהראינו בדוגמת האוטוסטרדה, בכל מהירות שהיא אנחנו חוזרים לאותו המצב יחסית למכוניות, או יחסית ל”אתר האקטיבי” במודל שלנו.

    זהו עד כאן. הפסקת עיכול רעיונות וכסאח. מה גם ש 5 בבוקר ב ל.א.

  602. קספר,
    לא אמרת שאתה עוזב? האמת שבחרת פסוק יפה. מודה ועוזב ירוחם.

    ישראל,
    לדעתי אתה טועה ומטעה. הגלים במודל המקורי שאתה מתייחס אליו הם גלי קוסינוס ולא סינוס וזה מה שעושה את כל ההבדל ויוצר את האפקט האלקטרו מגנטי. זה גם מסתדר עם משוואת פנרוז והשערת גולדבאך למספרים אי רציונאליים.

  603. יובלי.
    אנא, אל תהיה ספוילר, טוב? תן למהלך הדברים הטבעי לעשות את שלו.

  604. ישראל!

    “היקום מלא במעין חלקיקים קטנים דמויי כדור, כשבכל כדור זרמים זורמים מקוטב אחד של הכדור לשני. בין הכדורים יש גלגלי תנופה וציריות, גלגלי סרק, הזורמים יוצרים מערבולות, המתקדמות בצורת גלי סינוס מאונכים זה לזה ביקום, וכך נוצרים גלים אלקטרומגנטים”.

    בשביל מה אתה עושה את זה!? ככה לימדו אותך להתנהג!?

  605. ישראל

    חמוד.
    אתה רוצה ביקורת בונה? טוב נו הנה:

    אתה הראשון שזיהה את גודל הבעיה, והראשון שהבין איך להתמודד איתה.
    אתה יותר חכם ממורלי, ויותר נחוש ממיקלסון.
    אתה גם נע יותר טוב מהוקינג.
    אין עוד אנשים מבוגרים ובעלי ניסיון כמוך, מי כמוך יודע..
    ובלעדיי התאוריה שלך היקום פשוט לא היה כמו שהוא עכשיו.
    ניוטון מתהפך בקבר אחרי שקרא את המודל שלך. (איך הגאון הזה ידע, הוא בטח שואל את עצמו)
    הפתרונות שלך הם ברמה עולמית, מה אומרים על זה המדענים?

    משלי ט’ פס ח’

  606. מאיר
    תודה על הקומפלימנט. בדרך כלל אם אני מקבל איזו מחמאה באתר, זה משהו בסגנון: אתה כותב יפה, למרות שאתה טמבל. גם משמח לראות שיש כבר שניים שלמים שקוראים באמת מה שאני כותב.

    כפי שכתבתי, בעיית האינרציה הרבה יותר סבוכה לדעתי מאשר ניתן לתאר בכמה משפטים. זה לא עניין של מאך, איינשטיין או חלקיק היגס. זו שאלה יקומית וקיומית, מסדר הגודל של השאלות שיובל נדרש להן. השאלה לדעתי צריכה להיות: איך, מבחינה לוגית, אפשר בכלל לתאר יקום ללא אינרציה?

    כדי לראות זאת, נסה לדמיין יקום ללא גרביטציה. במקרה כזה, תוכל לשים את כדה”א והירח זה ליד זה, במרחק מטר, ולא תהייה שום משיכה בינהם. מוזר, אבל הגיוני.

    אך איך יראה יקום ללא אינרציה? מה תהיינה התוצאות הסופיות של התנגשות בין כדור רובה לירח? איך נוכל לחשב כמותית את התוצאות ללא חוק שימור התנע? האם הן תהיינה אקראיות לגמרי? כי אם יש שיטה והגיון ונוסחאות, הרי שהן חייבות לכלול בתוכן את המסקנה שגופים מתנגדים לשינוי במצב תנועתם ביחס ישיר לכמות המסה שהם מכילים, ומכאן קיבלנו: אינרציה. וזאת באופן לוגי, ללא תאור מנגנון.

    ובקשר למודל ל.ס. – למסה אצל ל.ס. יש רק תפקיד אחד: היא מפרש המתנגד לתנועת החלקיקים. נקודה. מכל הידוע לי, זהו המנגנון היחיד המסביר אינטואיטיבית מדוע מה שקורה פ ה משפיע על מה שקורה ש ם. אלא אם כן אתה מעדיף את: “המסות מעוותות את המרחב זמן” (איך בדיוק?).

    אני צריך ללכת להכין תגובה שתסביר את עניין מהירות האור ומבחן ההפרכה. המאור האחרון של הלוגיקה הצרופה עזב אותנו להצטרף לשאר החכמים הפורשים. נשארנו מיותמים, כמו עץ עזוב, בלי הורה או יד מכוונת. חבורה פטטית של “מדענים” המתעסקים בקקה, ומעלים את הדחלילים שלהם כדי שינופצו ויושלחו למגרש הגרוטאות של התאורייות ההזויות..

    אח, איזה כף, איזה שקט, סופסוף אפשר לנשום!

    חכמים..

    כשאני שומע את כל הפסלנים, שאין להם ולו רסיס רעיון מקורי, ורק יודעים למצוא פגמים אצל אחרים, אני חושב על חברי צביקה, ממציא, מהנדס בגוגול ואיש חכם באמת, ועל סופי השבוע המרתקים שאנחנו מעבירים בדיונים בענייני מדע, למגינת ליבן של הנשים הנזנחות. צביקה אוהב להשתעשע ברעיונות, אנחנו מחפשים בצוותא פתרונות לבעיות , ללא התנשאות, ללא פלצנות, מה גם שצביקה, ב ו ג ר טכניון, טוען, באישורה של וויקיפדיה, שעם כל הכבוד לרמתם של מוסדות החינוך הגבוה בארץ, הם אינם מגיעים לרמת האוניברסטאות בחו”ל, על מאות אלפי האוריינטלים הממלאים אותם, סיניים מוכשרים וחרוצים עד אין קץ, הלומדים 16 יח”ל לפני שיוצאים לעבודה בערב לעזור למשפחות, משלימים דוקטורט בגיל 25, וכל זאת עם חיוך נבוך על השפתיים ונכונות לעזור, ובלי להתרברב בידיעותיהם והשכלתם.

    טוב, נסחפתי קצת, נלך להכין תגובה לר.ח.

  607. בשלב זה אני יורד מהשאלות לגבי אור ופוטונים. אלו שאלות חישוביות שאולי יש להן פתרון בלתי מזיק.

  608. ישראל,
    אני עוקב. הבנתי איך התגברת על טיעון פיינמן. גאוני.
    אבל אני לא אוהב את ל.ס.

    חלק מהשאלות:
    האם לא שפכת את המים עם התינוק? במודל המקורי היה הסבר לאחד ההיבטים של אינרציה: האצת המסה מפגישה אותה עם חלקיקים רבים יותר נגד כיוון התנועה, ומכאן נובעת התנגדות המסה להאצתה. במודל שלך נדרש מנגנון נפרד עבור היבט זה (לא התעמקתי התאוריית ל.ס., אבל על פניו נראה שבכל מודל להסאז’י אין התמודדות עם השאלה מהי בכלל מסה, שהרי חלקיקי ל.ס. עצמם מופיעים עם מסה מובנית. כמו כן אין התמודדות עם השאלה כיצד היא “מתמצאת” במרחב לשמור על תנועה במהירות קצובה בקו ישר).

    איך קרן אור “מתעדשת” על ידי חלקיקי ל.ס. מבלי להתבדר? איך פגיעה אקראית של חלקיקי ל.ס. בפוטון העושה את דרכו מליארדי שנות אור בחלל להסאז’י לא הופכת את המסלול שלו לכאוטי?

  609. רפאי.ם, חכה רגע, אל תעזוב עדיין. אני זקוק לאינטואיציה החייתית שלך. יש איזה מודל שאני חייב לקבל עליו ביקורת בריאה.

    מה דעתך על המודל הבא: היקום מלא במעין חלקיקים קטנים דמויי כדור, כשבכל כדור זרמים זורמים מקוטב אחד של הכדור לשני. בין הכדורים יש גלגלי תנופה וציריות, גלגלי סרק, הזורמים יוצרים מערבולות, המתקדמות בצורת גלי סינוס מאונכים זה לזה ביקום, וכך נוצרים גלים אלקטרומגנטים.

    ביקורת כנה. יש לך רשות לכסח כאוות נפשך ולהשתמש בכל שפה שתבחר.

  610. בינתיים, עד שישחררו התגובה, מה עם איוב ט’ פס כד’?
    או משלי י”ז פס כ”ח?

  611. יובל

    אני רואה שאתה נהנה מהאוננות האינטלקטואלית כביכול בינך לבין ישראל (עם עזרה מידי פעם מהר.ח),
    כל מה שאני יכול לאחל לך הוא בהצלחה.
    אעשה כחכמים אחרים ואפרוש מהדיון. (אבל אני מבטיח שאמשיך לקרוא, אתם בהחלט משעשעים 🙂 ).

  612. אחזור בשמחה אחרי שהמודל שלך יתבהר ויוסכם על הכל. בינתים נדרשתי לסוגיית האצת האור במהלך מסעו בן מיליארדי השנים מן הגלקסיות הרחוקות אלינו. קיבלתי איתות על כך שיתכן כי הוא דווקא מאט, ואם זה נכון אז עלי לשנות משהו מהותי בסיפור. זה מעניק לך שהות להמשיך לחגוג סולו לאור הזרקורים. ר.ח טוחן אותך היטב. כל הכבוד והיקר לו. וגם רפאי.ם זכור לטוב. הוא אולי אינו מתקדם כמוך בחומר, אבל יש לו אינטואיציה בריאה (ר’ תהלים פרק ח פסוק ג)

  613. עוד פעם תגובה ממתינה לאישור. מנגנון הבקרה פה התחרפן לגמרי.
    איפה יובל ומאיר? יובלי התהפך עלינו לגמרי, אפילו תמונה הוא החליף. אגב, הוא שלח לי חלק מהמודל שלו. לאלו מביננו שלא ראו אותו מקודם, וחשבו שהוא מבוסס רק על “בראשית היה אין שאיין עצמו ליש” אני יכול להרגיע שהמודל מרהיב עין ופיזיקלי וגאומטרי לגמרי. יובלי מבולבלי פשוט יצא כנראה מנקודת ההנחה שכולנו מכירים את המודל, כנראה דרך טלפתיה.

    אז עזבו תאוריות שמאוריות. אין ספק שהאתר בלי יובל משמים בהרבה. מי ימלוץ מליצות? מי ישביענו שפת עבר? מי ילמדנו הסטורית עמנו?
    אז לעזוב הכל עכשיו ולקרוא לו מיד לסדר:
    יובלי חזור!

  614. גם בפצצות שהכנו בקיבוץ השתמשנו בנייטרונים איטיים. במיוחד בפצצות לגבות.
    וכשהיינו יורים באבטיחים, הכדורים האיטיים היו עושים את כל הנזק. המהירים היו בקושי פוגעים בגרעין לבן.
    ולמה ללכת רחוק? זקני הדינוזאורים בפינת החי במשק מספרים תמיד על המטאורים שהיו פוגעים בארץ. המהירים, אלו שהגיע במהירויות של חצי מהירות האור, היו עושים חור קטן וממשיכים בלי שאף אחד הרגיש בהם. האיטיים היו אלו שעשו תמיד בלגנים והכחדות.

    ובכלל: כולם, ללא יוצא מהכלל, לא מספרים תמיד על מיליארדי הנייטרינוס שחולפים דרכם יום יום. הם לא מספרים, כי הם בכלל לא מודעים לקיומם, כי למרות כל מכשירי המדידה כמעט בלתי אפשרי למדוד אותם, וזאת למרות שאין ספק בקיומם. הם גם מהירים מאוד. מה שעשוי לענות על השאלה: על השאלה: “גוף א’ יהיה שקוף לגוף ב’ במידה וגוף א’ נע במהירות מסוימת דרך גוף ב”.

    ר.ח.

    אני מעוניין בפיסיקה וביקורת בונה. אין ספק שהוא שיפר את הטון והעיר הערה נכונה. אם נמשיך ככה, אין לי שום בעיות איתו.

    אבל כל זאת הוא צדדי לגמרי לעניינינו. אפילו דוגמת הפגז איננה שלמה, וניתנה רק כדי לכמת, בצורה לא מושלמת, את מה שידוע לנו מהאינטואיציה: שמעל מהירות פגיעה מסויימת, הולכת ופוחתת השפעת ההתנגשויות בין גופים.

    לא מאמינים? תלו גוש פלסטלינה על חוט וירו עליו קליעים במהירויות שונות. תראו למי מגיב הגוש הכי הרבה.

    קשה לי לקבל את הפירוש שלך להשפעת הצורה הגיאומטרית על הגרביטציה במודל לסאז’. לא הרחבנו בנושא, אך אל תשכח שהתאורייה נבדקה כמעט עיי כל המי ומי, והסתייגות זו מעולם לא הועלתה. אין פה עניין אם התאוריה תעבוד או לא. היא אמורה לעבוד, אין כמעט ספק, אלא שלפי מודל לסאז’ המקורי, אנחנו אמורים לקבל גרביטציה, ללא קשר לצורה גאומטרית, אבל גם חיכוך.

    מבחינתם של החלקיקים, כל האינטראקציה היא רק בין חלקיקי יסוד בעלי צורה גאומטרית זהה. מושג ה”רוח” אינו מופיע בתאוריית לסאז’ (ע”ע). הפרש הלחצים בהדמיה הוא בגלל שיש לך שני מפרשים באויר עומד, שבין שניהם יש כמות מדוללת של אויר. אפשר לקרוא לזה רוח, אך זו רוח שונה מרוח בים, אלא אם כן אתה חושב על רוח הלוחצת על מפרש עומד, שאז היא אינה שונה מלחץ אויר רגיל.

    הגרביטציה אצל לסאז’ זהה לגמרי לזו של ניוטון, רק לסאז’ נותן לה סיבה, ניוטון לא. לכן תהיה תנועה הרמונית לפגז שהצעת.
    איך יעבוד המפרש שהצעת בקרבת מסה גדולה, אם מבחינת הגוף בנפילה חופשית הוא במנוחה?

    אני חייב לזוז, ולנטיין דיי. אשה, שוקולד, פרחים, או נבוט בראש. נמשיך אחר כך עם הניסוי.

  615. לישראל שפירא
    מניסיוני בפצצות האטום שהכנתי בילדותי בשכונה שלי, דווקא הנטרונים האיטיים היו המוצלחים והראו תוצאות..
    הסיבה- שהמהירים היו חולפים ביעף דרך ה235 וכמעט לא שיחררו נטרונים לפעולת השרשרת. לעומת זאת הנטרונים האיטיים, דווקא ניבלעו ע”י האורניום 235 והוא ניסה לשחרר את עצמו מהאורח הלא קרוא עד ששבשיעול אחד גדול מידי, התחלק לשני אטומים קטנים יותר ועוד נטרונים שהמשיכו בשרשרת ועוד מסה ואולי גם העצבים שלו שהפכו לאנרגיה
    אבל זה היה בשכונה שלי, אני לא יודע איך זה היה בשכונות של הצפונים.
    יום טוב
    סבדרמיש יהודה

  616. ישראל,

    ראיתי שהתחלת להעלות באוב ולעשות סיאנסים עם שדים ורוחות אז לא רציתי להפריע.

    נחזור רגע לפגז וכדור הארץ.
    לפי התאוריה המקובלת ככל שמסה יותר גדולה כך כח המשיכה שלה גדול יותר. לא משנה מה צורתה ואם יש בה חורים.
    לפי התאוריה שלך של לסאג’ ולא נשכח את האדון סברדמיש המכובד הגרביטציה נובעת מלחץ של חלקיקים על המסה (אתה גם הוספת את המהירות הגבוהה שמבטלת גרביטציה אולם זה לא לעניין כרגע).
    לפי מה שאני מבין וכבר דנו בכך בעבר אולם עכשיו פרשת את משנתך ביתר בהירות, במודל שלך לצורה הגאומטרית של המסה צריכה להיות השפעה על הגרביטציה שלה. לא כל שכן לחור באמצע כדור שבו לא תהיה שום גרויטציה כי תהיה תנועה חופשית של החלקיקים שמניעים אותה. כך שלא משנה באיזו מהירות הפגז יגיע, בחור לא תפעל עליו שום גרויטציה ומבחינתו הוא בחלל הריק. לא צריך מהירות בריחה ולא כלום.

    ומזה גם נובע ניסוי הפרכה לתאוריה שלך, של יהודה ולסאג’ כאחד.
    קח מטאור עם כח משיכה X. קדח מנהרה מקצה לקצה. הכנס חפץ כל שהוא למנהרה. אם אתם צודקים לא תפעל עליו שום משיכה ולא תהיה תנועה הרמונית. באם תיווצר כזו אזי טעיתם.

    גם בדוגמת האוטוסטרדה שלך לצורה הגאומטרית של הפלסטלינה או המפרש תהיה השפעה על כיוון תנועתה כמו שמפרש משולש של סירה יידחף אותה בכל זווית חוץ מאשר בין -45 +45 (מה שמכונה ה- iron). כך גם מפרש גרויטציוני (למעשה כל מסה) בקירבה למסה אחרת אמורה להתנהג כמו מפרש.
    אני מסכים שבחלל אין “רוח” כי החלקיקים מאזנים זה את זה אולם בקירבת מסה גדולה כן יש “רוח” ולכן גאומטריית המסה כן צריכה להשפיע.
    אבל מה? גלילאו הראה שלא, הרי לך מבחן הפרכה שכבר בוצע.

    בכל מקרה, תמשיך, מה מיוחד במהירות האור ומה ניסוי ההפרכה שלך?

  617. שאלה ליהודה

    לו אתה איזוטופ אורניום 235 החרד לקיומך, ממי אתה חושש יותר: נייטרונים מהירים או נייטרונים איטיים?

    אם תוכל, הסבר גם למה.

  618. איך אפשר להגיד שהם מקרינים? – בגלל מה שמתקבל בחישובים של אלקטרודינמיקה קוונטית.
    למה אי אפשר למדוד אותם? בגלל עיקרון אי הוודאות.

    בכל אופן, התשובות שלי מוגבלות בגלל הידע המוגבל שלי בנושא הזה.
    מי שכן יכול לענות לך על השאלות ברצינות וללא חירטוטים באתר הזה הם אהוד ומכאל רוטשילד.
    אני גם מאוד אשמח אם לפחות מישהו מהם ישתתף בדיון הזה.

  619. יפה. אני גאה בך. זו נקודה שלא חשבתי עליה. בדיוק בשביל זה אני מעמיד את הרעיונות לביקורת.
    אולם שים לב לשתי נקודות: 1. האלקטרון המאיץ מקרין גם ללא קשר להעברת תנע לטבעת. 2. זה לא ממש משנה לעצם העניין – זו הייתה רק דוגמה, ואני מודה שלא מוצלחת (הרבה שנים מאז התעסקתי בנושא זה).

    ועוד דבר. אינני מנסה למכור תאוריה – להיפך. אני מנסה למצוא מה לא בסדר בה, כדי שאוכל להתפנות לעניינים אחרים.

    לצערי, באמת, זה שהדוגמה אינה מוצלחת, לא סותם את הגולל על הרעיון. נסה לעשות את אותו הדבר בגוף שאינו טעון חשמלית, כמו הפגז, או בכלל, מכיוון שאנו מדברים על חלקים היפוטתים ( חלקיקים דמויי חלקיקי לסאז’), איך אפשר להגיד שהם מקרינים? חוץ מנושא הקרינה, האם הדוגמה שלי לה הייתה עובדת?

    אני הולך לישון עכשיו (1.30 בלוס אנג’לס). אינני חושב שזה סותם את הגולל על הרעיון, אבל אם נראה לך שכן, הסבר למה.
    תודה בכנות על הביקורת הבונה.

  620. יהודה

    עיקר הבעיה היא לשכנע את ישראל בכך שמטען חשמלי שנע בתאוצה פולט קרינה (ואפילו מטענים שנמצאים במנוחה פולטים פוטונים וירטואלים).

  621. לכולם
    התגובות כאן ממשיכות וממשיכות ואני חייב לבקש מאבי לעקוב אחרי התאריך כי תוך זמן קצר השישה מיליארד שנים יהיה מיושן ונצטרך לעדכן אותו!.
    יום טוב
    סבדרמיש יהודה

  622. ומה אם חלקיקי האוויר, או אלקטרונים או פוזיטרונים או גרביטונים או משהו אחר שנעים דרך החור בטבעת ומתנגשים באלקטרונים שחוצים אותם? הרי האלקטרונים שלך כן מעבירים תנע כאשר הם מצליחים לחדור דרך הטבעת וגם אחר כך כאשר הם נעים דרך החור בטבעת הם מעבירים תנע לחלקיקים שנמצאים שם. אתה לא מתחשב בהם למען הדוגמה?

    סתם לדוגמה: אם תכניס את האצבע שלך לטבעת והאצבע שלך תהווה חסם לאלקטרונים שלך, האם האלקטרונים שלך – ‘במהירות מילוט’ – יחדרו דרך האצבע שלך מבלי להשאיר תנע?

    אני לא מצליח להבין את הטענות שלך, ברצינות. אם אתה יכול בבקשה תכתוב אבסטרקט של הרעיון שלך.

  623. בסדר, יצא לי קצת לחשוב, נראה לי שדוגמת הפגז עלולה לבלבל. נחזור לדוגמת המטען החשמלי. היא גם יותר ראלית לעניין השקיפות.

    נחשוב על טבעת מתכת התלויה על חוט וטעונה במטען חשמלי שלילי. אלקטרונים מתקרבים למרכזה מימין במהירויות שונות.

    הטענה שלי היא: מתחת למהירות מסויימת VM כל האלקטרונים יהדפו אחורה מהטבעת ועיי כך יעבירו לה תנע. לעומת זאת, ממהירות VM ומעלה, האלקטרונים יעברו את הטבעת לצד שמאל, יאיצו חזרה ויחזרו למהירותם האוריגינלית בלי להשאיר כל חותם בצורת תנע על הטבעת. בשורה התחתונה, האלקטרונים והטבעת שקופים זה לזה.

    השגות?

  624. מאיר
    ובתנועה הרמונית לא עובר תנע למערכת ארץ/פגז?

    אני חייב לרוץ לעבודה, נדון כשאשוב.

  625. ישראל,
    תגובתי מלפני 24 שעות לא פורסמה. כנראה שהיתה ארוכה מידי אז אכווץ אותה ל-20%:

    לגבי הפגז שנלכד, לא הבנתי מדוע הוא לא יכנס לתנודה (כמו) הרמונית, ועל כן לא יעביר תנע (הרי הסכמנו שלא משנה האם הוא רכש את מהירותו התחילית בסמוך לחללית בעקבות ירי או בעקבות נפילה חופשית ממרחק רב יותר).

    לגבי התאוריה הזהה, אני סקפטי לגבי הזהות שלה לאור מסדר הזיהוי החפוז. אין מצב.

  626. לא, זה לא מה שאני טוען.
    יתכן שלא ירדתי מספיק לפרטים. זה הזמן לעשות זאת עכשיו.
    למעשה, כל גוף שמגיע לכדור הארץ מהחלל ויעבור דרך המנהרה שלנו, ופועלים עליו רק שיקולי גרביטציה לא ישאיר על הארץ חותמת בצורת תנע.
    הגוף יאיץ בכיוון הארץ, יגיע למהירות המילוט ומעלה בפני הארץ, יגיע למהירותו המקסימלית במרכז הארץ ויצא מהצד השני בדיוק באותה המהירות בה הגיע אליה, שמכיוון שהיא גבוהה ממהירות המילוט, הגוף אכן יימלט מכבידת הארץ ויהיה מוכן לעשות שנית את אותו התרגיל עם כל פלנטה אחרת.

  627. בקשר לפגז 12

    זה דומה לכך שתטען שמטוס שעובר את מהירות הקול, נמנע מחיכוך עם האוויר.

  628. ישראל

    אם אתה מחפש תוצאה הכי מדוייקת, אז כאשר אתה מחשב את תנועת כדור הארץ ותנועת השמש, אתה לא יכול להתחשב רק בנתונים של כדור הארץ והשמש. אתה חייב לכלול גם ערכים שאינם קשורים ישירות לכד”א ושמש כמו ההשפעות של פלנטות אחרות. השפעות מהתופעות/תהליכים שקורים מסביב לכד”א, מסביב לשמש ועוד..
    בגלל שאי אפשר לכמת את הנתונים האלו בצורה מושלמת, אז התוצאות במדידות גם לא מושלמות.
    זה מתבטא בעיקר במקרים שבהם מנסים לחשב מסה של ניטרינו למשל, או את הנפח של היקום..

  629. שכחתי להוסיף – כלום אינך רואה את ההבדל הברור והחד משמעי בין פגז 11, שהעביר את כמות התנע הגבוהה ביותר לארץ, ללא כל שיקולים קוואנטיים, לפגז 12, שנזקק לשיקולים אלו?

  630. בסדר. בוא ניקח את אותו ההגיון ונפעיל אותו על כל מערכת אחרת שכוללת גרביטציה. מה עם תנועת כדור הארץ סביב השמש? גם היא אמורה להית מואטת? מה עם תנועת מטוטלת? מה עם תנועת מולקולות אויר בתוך בקבוק אטום?

    והחשוב מכל: מה זה בדיוק משנה לעצם הטיעון? נאמר שאתה צודק ואחרי מאה מילירד תואמי ארץ ניווכח שאכן, מהירות הפגז פחתה במטר שלם לשניה! האם זה משנה הנחת היסוד שלנו שבכל מסעו של הפגז הוא לא השאיר כל חותם על הפלנטות אותן חצה במעופו בצורת תנע אותו נוכל למדוד? כי אם לא נוכל למדוד – כיצד נוכל לדעת שהוא היה שם אי פעם? האין הוא נהפך ל “נייטרינו” – אנחנו יודעים שהוא שם, אך איננו יכולים למדוד את השפעתו?

  631. כן, בערך.
    עד כמה שאני מבין בתורת הקוונטים (וזה מעט) – אם הפגז יעבור דרך המנהרה, אז אפילו במהירות המילוט- הפגז עדיין יהיה נתון להשפעות (שינוי בערך כלשהו כמו מסה תנע.. עד לרמה מסוימת) חיצוניות בגלל העובדה שאפילו בריק עצמו ישנם תנודות קוונטיות. כלומר הפגז שיעבור דרך המנהרה במהירות המילוט – גם אם לא תוכל למדוד שום השפעה עליו- עדיין תהיה עליו השפעה כשלהי מינימלית כפי שעולה מחישובים מתמטיים (ואל תבקש ממני להראות לך את החישובים 🙂 ).

  632. בוא נראה.

    ניקח את דוגמת המנהרה דרך כדה”א. הבנתי שאתה טוען שבגלל אפקטים קוואנטים תואט תנועת הפגז. הבנתי נכון?

  633. הבעיה היא שעם הידע של היום, אתה כבר לא יכול להגיד שיש מערכת כזאת בטבע שתהווה מערך של “אפס כוחות”. כלומר, לא קיימת מערכת שתוכל לתאר שהיא לא תשפיע בשום צורה על חלקיק שיעבור דרכה.

  634. אגב, בקשר ל’סגנונות כתיבה’ – באופן אישי אני מעדיף את התוכן על פני הסגנון. מצדי תכתוב גם עם שגיות כתיב, העיקר שיהיה תוכן שיהיה אפשר להסיק ממנו מסקנות הגיוניות ובעלות משמעות.

  635. הבנתי שיש לך בעיה עם דוגמת הפגז וכדה”א. פרט בבקשה מה לא בסדר בדוגמה.

  636. ישראל

    אוקיי, מקובל עליי הטון שלך.
    (אני חייב להגיד שמה שאמרת על ר.ח היה משעשע 🙂 אבל נעזוב אותו הוא לא לעניין).

    אתה מוזמן לקרוא את התגובות שלי שוב, ולהסביר לי מה היה לא נכון בדבריי שסתרו את דבריך.
    כמובן שגם אני אקרא, ואם אמצא משהו לא הגיוני בדברי- שתשובותיך הצליחו להפריך- אגיב גם עליהם.

  637. ר.ח רפאי.ם

    עוד הפעם התגובה שלי ל ר.ח. ממתינה לאישור.

    אני תמיד פתוח לביקורת. אך על הרעיונות, לא מתקפה אישית.

    ייתכן שאני טועה כמובן. מה אתך? אתה מעל לטעות?

    קח כל תגובה שלי ונסה להראות לי איפה אני טועה. השתדל לדבר פיזיקה בלבד. ברגע שתעבור לפסים אישיים, נפרד לדרכנו לשלום.

    ולדעתי, אתה טועה בקשר לר.ח.
    חוץ מזה שהוא אחד הכותבים הכי מנומקים והגיוניים באתר, הוא גם כותב יפה מעניין ומשעשע, ואף פעם לא אחמיץ תגובה שלו, בין אם אסכים איתו או לא. כנ”ל לגבי יהודה ויובל. תוכל ללמוד מהם הרבה בענייני סגנון, במיוחד מיובל. חבל שגיא עזב אותנו, גם הוא כתב מאוד יפה.

    יש פה כותבים מאוד הגיוניים אך קצת יבשים. מדע אמנם אינו טלנובלה, אך רצוי לדעתי לשמור על חיוניות והומור, אחרת נרדם כולנו. זו הסיבה שאני משתדל להכניס קצת שטותניקיות וסיפורים אישיים.

    אז בבקשה, הראה לי איפה אתה חושב שאני טועה, אני תמיד מזמין כסאח על הרעיונות שלי. אבל אין זאת אומרת שאני חייב להסכים איתך, ואל תעבור לפסים אישיים בכל מקרה.

  638. Si, patron, pero hay otra problema

    ניוטון ניסח את חוק ההתמדה, כמו את חוק הגרביטציה, אך לא נתן להם הסבר.

    לגבי ההתמדה הוא דיבר על “מרחב מוחלט” ערטילאי, ברוח האל (במילותיו של ניוטון עצמו) ונתן את דוגמת הדלי המסתובב, הגלגול הקודם של הBSA של יהודה, כדי להמחיש את הבעיה. אם יש מנגנון אינרציה המוסבר היטב עיי ניוטון, מדוע נדרש לכך מאך שנה 200 שנה מאוחר יותר? או איינשטיין?

    כנ”ל לגבי גרביטציה. ניוטון תאר וכימת את התופעה, אך מעולם לא התיימר להסביר את המנגנון, ורק הפטיר בחמיצות “אשאיר זאת לקורא להחליט כיצד פועלת הגרביטציה”.

    אולם קפצת צעד אחד קדימה. הנחת שאני מנסה להסביר את האינרציה באמצעות המנגנון שתארתי. זה לא מדוייק. אינך יכול להסביר אינרציה באמצעות העברת תנע, שמקורו באינרציה. נגיע לכך מאוחר יותר. לעומת זאת נוכל לדבר על גרביטציה.

    אם בדוגמת האוטוסטרדה במקום מפרש אחד נכניס שניים במרחק מסויים זה מזה, הרי לפי לסאז’ וההגיון תיווצר משיכה בין המפרשים. את בעיית החיכוך התרמודינמית של החלקיקים עם המסה פתרנו לדעתי, בדוגמת אינטראקציית הפגז עם כדה”א. תנע ודחיפה עוברים, אך אין חיכוך. אך מה עם בעיית החיכוך השניה, עליה דיבר פיינמן, בעיית החיכוך של הפלנטות בתנועתן בחלל עם החלקיקים?

    בוא נחדד את הבעייה. פיינמן אומר: נכון, תהיה משיכה במודל לסאז’, אך הכוכבים והפלנטות בתנועתן יתנגשו עם החלקיקים, ואלו יתנגדו לתנועה כמו רוח המתנגדת למפרש.

    אולם שים לב שבעייה זו אינה קיימת במודל האוטוסטרדה. בכל מהירות שהיא למפרש, או לפלסטלינה, הם מבחינתם במצב מנוחה. אם נוסיף מימד נוסף לדוגמה החד מימדית שלנו, אם הפלסטלינה נעה גם מזרחה או מערבה, ואותו מנגנון אוטוסטרדה פועל גם במימד זה, הרי הפלסטלינה נמצאת מבחינתה במהירות 0 יחסית למכוניות ואין חיכוך.

    לכן, אם הארץ מושכת את הירח, והוא נע גם בתנועת ההקפה שלו בווקטור המאונך לכיוון המשיכה, הרי שלא יווצר כל חיכוך עם החלקיקים. הירח מבחינתו במנוחה.

    לגבי האינרציה,השאלה יותר מסובכת, למרות שבאופן אינטואיטיבי זה נראה ברור: המפרשים מבחינתם במנוחה בכל מהירות קבועה, אך בזמן האצה הם נתקלים ביותר מכוניות, המתנגדות להן עד שהמפרש מתייצב במהירות קבועה חדשה, ואז שוב כל הכוחות מתאזנים.

    למרות הפשטות לכאורה, ההסבר כאן יותר מורכב, וצריך גם לתת תשובה לשאלה שהציג מאך: מה הקשר לכוכבים הרחוקים? מדוע בדוגמת הצלחות המעופפות הצלחת היחידה שאינה מסתובבת, ואשר בה לא נמדוד כוח צנטריפוגלי, היא גם זו הנחה יחסית לכוכבים הרחוקים?

    נניח לזה כרגע. אני רוצה שנסגור את נושא הגרביטציה, כדי שנוכל להתקדם לחלק המעניין באמת: קביעות מהירות האור בכל מערכת יחוס. אם החזקת מעמד עד כה, אבטיח לך בונוס נחמד. בסוף הדיון המייגע יש מבחן הפרכה, אותו ניסוי עליו אני מדבר, שיכול לאשר אם יש משהו בכל הרעיון, או שזהו סתם תרגיל אינטלקטואלי לשעות הפנאי.

    השגות?

  639. ישראל

    אני תמיד שואל שאלות כשאני לא מבין.
    אותך, למשל, אני לא יכול לשאול שאלות מפני שהתשובה שתתן לרוב תהיה שגויה. ואחרי שיסבירו לך למה היא שגויה אתה עדיין תתעקש על כך שאתה צודק. אפילו אם ילדה קטנה מוכיחה לך שאתה טועה אתה עדיין לא תקבל את האמת.

    אני מזמין אותך ללמוד עוד קצת פיזיקה (אפילו שאתה כבר בגיל כל כך מופלג שאתה כפי הנראה האדם הכי מבוגר עם הכי הרבה ניסיון שחי בעולם).
    ולהבין בעצמך היכן אתה טועה, במקום לבקש ממני או מאחרים להראות לך איפה אתה טועה.

    לגבי ר.ח, אם תשרוד באתר הזה עוד קצת זמן תוכל להיווכח שמדובר בעצם בחיידק שניטפל לחיידקים מזיקים (אבל במקום לרפא אותם הוא נשאר צמוד להם ללא שום השפעה עליהם 🙂 ).

    בכל אופן, אם יצא לך להתמיד בלימודי פיזיקה, אז אתה מוזמן לקרוא את התגובות שלי שוב ולהתייחס עליהן.
    וכמובן אני מוכן לפתוח איתך דף חדש במידה ואתה מבין איפה היו הטעויות שלך ואתה מוכן להודות בהם מבלי להיעלב.

  640. ישראל,
    עד כאן ברור. לעניות דעתי היה איש אחד שקראו לו ניוטון שניסח את זה ב-1687 בלי פלסטלינות ואפילו בלי אוטוסטרדות. הוא קרא לזה החוק הראשון ע”ש ניוטון:

    Lex I: Corpus omne perseverare in statu suo quiescendi vel movendi uniformiter in directum, nisi quatenus a viribus impressis cogitur statum illum mutare.

    או בשפת עבר:
    “גוף ישאף להתמיד במהירותו, כל עוד סכום כל הכוחות הפועלים עליו הינו אפס”

    אתה יכול להמשיך הלאה.

    לגבי החבר שלך, אתה רואה? לקח את הכדורים נרגע ואפילו מנסה לדבר עניינית. אבל עוד מעט ייצא שוב הילד הרע.

  641. כתבתי תגובה לר.ח הממתינה לאישור. בינתיים רצוי שנבהיר כמה דברים:

    ר.ח רפאי.ם

    אם אינך מתכוון להעליב, השתדל להמנע מכינויים כמו “טיפש”. הרבה אנשים יחשבו משום מה שכוונתך היא דווקא כן להעליב.

    אם אינך מבין משהו, אתה תמיד מוזמן לשאול. ייתכן שההבנה שלי שגויה, אך ייתכן שגם שלך. בשביל זה מקיימים דיון.

    למיטב ידיעתי, לא יווצר כל חיכוך בגלל אפקטים קוואנטים, אחרת אותו הגיון יתפוס לכל המתנדים הטבעיים בטבע. אם למדת פיסיקה, ראה בכל ספר סטנדרטי לתלמידי פיסיקה שנה א’ בפרק העוסק בגרוויטציה את דוגמת המנהרה דרך כדה”א.

    בכל אופן – זה לא משנה כלל. זוהי רק דוגמה לרעיון, שבהחלט יכול להחשב לספקולטיבי עד אשר ייתמך בנוסחאות או ניסוי. (אתה עצמך הזכרת זאת). אם לא שמת לב, אנחנו עוסקים פה במודלים תאורטיים, מחוץ למיינסטרים, המסוגלים להסביר דברים שאין עליהם כידוע לנו הסבר מספק. הדברים הם:

    1. גרביטציה.
    2. אינרציה.
    3. קביעות מהירות האור בכל מערכת ייחוס.
    4. אי לוקליות.
    5. מסה ואנרגיה אפלות.

    ברור לרובנו שכל מה שאומרים אני, יובל, מאיר או יהודה, ספקולטיבי, אלא אם כן נוכל לספק את הנוסחאות, או עדיף אפילו, ניסוי.

    אם תרצה הבהרות, בקש. אתה בהחלט יכול גם לדלג על תגובות אם קשה לך. אתה יכול גם להעלות רעיונות משלך.

    אם תרצה לפתוח דף חדש, אין בעיה. אבל אם תתקוף – תחטוף.

    יעל.
    איזה יופי! הבת שלי סיימה לא מזמן תואר ראשון בביוכימיה. היא מתעתדת להקדיש את חייה למחקר אקדמי. היא קבורה כל הזמן בספרים ולימודים ומבסוטה עד הגג.

  642. כיתה ט׳ ותואר ראשון בפיסיקה?
    צריך להיות באמת עילוי בשביל זה..
    היכן את עושה את התואר? באוניברסיטה הפתוחה?

  643. ר.ח.

    בשביל ההמשך, חשוב שנגיע להסכמה על העקרון היסודי. לכן אעבור על ההסתייגויות:

    ב. זה מה לימדו אותנו בשנה א’. נכון, זה היה לפני המון שנים, כך שאולי התבלבלתי או ששינו את זה מאז.

    בכל אופן, עניין החיכוך הגרביטציוני אמור היה לתפוס גם במקרה של גוף הנופל בתוך המנהרה בנפילה חופשית, ואילו בספרים כתוב שתיווצר תנועה הרמונית בעלת מחזור של 84.2 דקות, כך שכנראה שאין חיכוך.. (חיכוך עם מה? הרי אין אויר. כל המתנדים ההרמונים הטבעיים על אותו עקרון במשך מליוני שנים).

    ג. ראה ב’.

    ד. שמעתי. על כל תהליך במאיץ חלקיקים קיימים מיליארדי תהליכים ללא נזק. ע”ע אויר. לכן אמרתי “רוב”.

    ה. בדיוק על הטכיונים ולכידתם נדון אם נגיע עד רעיון הניסוי אותו אני זומם.

    ו. אולי אפשר לדבר עם אבי שאם אינו יכול לחסום אותו בפינה עם מחסום פה, אז לפחות שיכפיל לו את מינון הכדורים?

    במחשבה שניה, להכפיל לא יספיק.

    לשלש!

    בשורה התחתונה, תכלית כל הדיון הזה היתה כדי להגיע לכך, שייתכנו חלקיקים דמויי חלקיקי לסאז’ בעלי התכונה הבאה: כשהם פוגעים בחומר הם מעבירים לו תנע ללא בזבוז בצורת חום, אך מעל מהירות מסויימת הם עוברים את החומר ללא השפעה עליו או עליהם.

    עולים לאוטוסטרדה.

    האוטוסטרדה הינה מודל לסאז’ במימד אחד של אורך. במקום חלקיקים מכל הכיוונים ובכל המהירויות, אנו מתרכזים רק באותם חלקיקים שנעים לאורך ישר נתון, בשני הכיוונים ובכל המהירויות. אנו נתרכז במהירויות עד 500 מ/ש.

    נוכל לדמות זאת לאוטוסטרדה בעלת 100 מסלולים, 50 בכל כיוון, צפון ודרום. המכונית נעות במהירויות של 10 מ/ש, 20 מ/ש, …. עד 500 מ/ש. לצורך הדיון, אנחנו מרכזים את המכוניות בעלות אותה מהירות במסלול אחד ויחיד. לכן במסלול מס 1 תהיינה כל המכוניות עם מהירות של 10 מ/ש, אחר כך 20 מ/ש עד מסלול 50 שבו מכוניות במהירות 500 מ/ש. אותו הדבר בכוון הנגדי.

    1. אם נמתח קו דמיוני לרוחב האוטוסטרדה, נראה שהתנע הכולל של המכוניות החוצות את הקו שווה ל-0 מכיוון שהמכוניות משני הכיוונים מקזזות זו את זו.

    2. אם במקום קו נשתמש בישות, לדוגמה שדה כוח חשמלי, או לצורך המחשה מפרש או פלסטלינה מיוחדת שהמכוניות יכולות לעבור דרכה בלי לעוות אותה, נראה שקיבלנו גם כן תנע 0 על הפלסטלינה.

    3. נניח לפי מה שהסקנו קודם, שהפלסטלינה רגישה רק למהירויות של עד 100 מ/ש. מכוניות העוברות את הפלסטלינה במהירות יחסית אליה הגבוהה מ100 מ/ש שקופות עבור הפלסטלינה והיא שקופה עבורן. מתחת למהירות יחסית זו, המכוניות תפעלנה כוח על הפלסטלינה.

    4. נראה מה קורה כשהנחנו את הפלסטלינה לרוחב האוטוסטרדה:

    כל המכוניות בעלות המהירות 110 מ/ש ומעלה, בשני הכיוונים, שקופות עבור הפלס’

    5. המכוניות בעלות מהירות 100 מ/ש ומטה מפעילות כוח על הפלס’, אך מקזזות זו את זו. סך כל הכוח המופעל על הפלס’ שווה ל-0 והיא נשארת במקומה. (אותו דבר קורה לכל מפרש באויר עומד. מולקולות האויר המהירות מקזזות זו את זו).

    6. מה יקרה אם נשים פלס’ או מפרש ב’ הנעה במהירות של 10 מ/ש צפונה יחסית לאוטוסטרדה?

    המכוניות במסלול מס 1 לכיוון צפון, שיחסית לפלס’ א’ נעו במהירות 10 מ/ש, יהיו בעלות מהירות 0 יחסית לפלס’ ב’. אלו במסלול 2 שנעו במהירות 20 מ/ש יחסית לא’ ינועו במהירות 10 מ/ש יחסית לב’ וכו. כל המכוניות בכל המסלולים בכיוון צפון תראינה לפלס’ ב’ כנוסעות במהירות הנמוכה ב10 מ/ש ממה שהן נראות לפלס’ א’. מנגד, המכוניות שנוסעות לכיוון דרום תראינה לב’ כנוסעות במהירות הגבוהה ב10 מ/ש ממה שתמדוד פלס’ א’.

    אולם שים לב שמכיוון שכל מכונית בעלת מהירות 100 מ/ש ומעלה לכל כיוון, למעשה שקופה עבור הפלסטלינה, הרי שגם סכום הכוחות על פלס’ ב’ הוא 0. ומכאן שהיא תשאר באותה המהירות של 10 מ/ש לכיוון צפון. אם נחזור למפרש, גם במהירות של 10 מ/ש הוא לא יחוש בכל רוח, ולפי אותו הגיון גם במהירות של 200 מ/ש, או 350 מ/ש, ולא משנה באיזה כיוון. בכל מהירות שנשים את הפלס’ או המפרש יחסית לאוטוסטרדה, בה הם ישארו, כאשר עבורם הם נמצאים במערכת “אויר עומד”.

    נעשה הפסקת עיכול רעיונות ושאלות, נדון בהמשך.

  644. שלום מר ישראל שפירא,

    כן, אני לומדת בכיתה ט’ אבל אני גם עושה תואר ראשון בפיזיקה. ואם להיות לגמרי כנים אז חלק מהדברים שכתבתי כאן זה דברים שאבא שלי אמר לי.

    אני מקווה שלא העלבתי אף אחד ומה שכתב ר.ח. רפאי.ם ממש לא היה בכוונות שלי.

  645. ישראל

    נעלבת.
    (אני מצטער. לא התכוונתי לפגוע בך)
    בקשר למודל שלך הוא שגוי כבר בהתחלה.
    גם אם תקדח חור דרך כדה”א – עדיין לדפנות (שעוטפות החור) של כדה”א תהיה השפעה (גם אם מינימלית ביותר) על כל חלקיק שיעבור דרכן.
    גם אם קליעים 12-20 יעברו דרך כדה”א, עדיין הם יאבדו חלק מזערי מהתנע שלהם.
    וגם אם תנסה לחשוב על מערכת אחרת במקום ‘חור בכדור הארץ’ שתהווה ואקום אתה לא תמצא, כי גם בואקום ישנם תנודות קוונטיות שישפיעו על חלקיק קוונטי.

    – ובבקשה על תעלב מאנשים שמוכיחים אותך בטעותך.

    ר.ח

    מה שמוזר הוא שאתה תמיד בוחר לצדד באנשים שטועים (ולפעמים גם משקרים). טוב נו, זה האופי שלך וכזה הוא אתה.

  646. ישראל,
    א. מסכם איתך
    ב. מסכים שהם יעברו, לא בטוח לגבי ה “לא יעבירו תנע”. אני חושב שהגרויטציה אחרי שיעברו את מרכז תהווה כח מתנגד, מעין חיכוך, שתוריד את מהירותם.
    ג. אם צדקתי ב – ב’ זה לא נכון ובסופו של דבר הם יכלאו באחד הכוכבים.
    ד. לא מסכים, “התנגשויות אלסטיות בין חלקיקי יסוד ברוב המקרים אינן גורמות לנזק או חיכוך” – על מאיץ חלקיקים שמעת?
    ה. למיטב הבנתי נייטרינו ייחודי בתכונה זו. כל שאר החקיקים מתנגשים זה בזה. ראה מאיץ חלקיקים. פוטונים מתנגשים ונבלעים בחומר ומעבירים לו את האנרגיה שלהם ולכן אור לא עובר מסך. גם נייטרינו מתנגש בסופו של דבר, עובדה שנקלט בגלאים. פעם כשסיפורי המדע בדיוני היו מעניינים באמת ולא רק קשקושי פנטסיה, דיברו על חלקיק בשם טאכיון שנע רק במהירות גבוהה ממהירות האור והוא אכן עובר כל דבר ולכן לא מתגלה.
    ו. הוא משהו מוזר. סכיזופרן, לפעמים הוא מנסה לאמר דברי טעם אולם אז פתאום יוצא לו הילד הרע והוא חוזר לסורו. אולי זה בימים שהוא שוכח לקחת את הכדורים הסגולים.

  647. יעל
    אני מקווה שלא נפגעת ששאלתי אם את יעל מכתה ט’. אני זוכר יעל כזו מאחת הכתבות ותהיתי אם זו את. דווקא זה מאוד קול עם נערה מתעניינת בנושאים אלו.

  648. ר.ח.
    לפני שנעלה על האוטוסטרדה, הייתי רוצה שנבהיר במדוייק לאיזו צומת הגענו, ונסנכרן את עמדתינו כדי למנוע אי הבנות.

    נתאר ניסוי מחשבתי:

    1. מנהרה נקדחת בין שני קטבי כדה”א.

    2. לפתח המנהרה מגיעים בהפרשים של יממה זה מזה 20 קליעים בעלי מהירויות מ 1 קמ/שנ עד 20 קמ/שנ.

    3. איננו לוקחים כרגע בחשבון את המערכת הכוללת את מקור והסטוריית הקליע. נניח שזה לא רלוונטי כרגע לטיעון. ( זה לא).

    טענה א’: קליעים מס’ 1-11 יילכדו במשיכת כדה”א ויעבירו למערכת ארץ/קליע את התנע שלהם.

    טענה ב’: קליעים מס’ 12-20 לא יילכדו בכבידת הארץ, לא יעבירו לה כל תנע, וימשיכו בדרכם לחלל, חופשיים מכבידת הארץ.

    טענה ג’: אם בהמשך מסעם של קליעים 12-20, או כל קליע אחר המגיע לפני הארץ במהירות העולה על מהירות המילוט של 11.2 קמ/שנ, יהיה תואם ארץ אחר, או אפילו אלף בשורה, יחזור התהליך על עצמו כשהקליעים עוברים את הפלנטות בלי להשאיר עליהן חותם בצורת תנע. בסוף התהליך ישארו הקליעים באותה המהירות בדיוק לכל שלב נתון בתהליך.

    טענה ד’: בניגוד להתנגשויות בין פגזים לאדמה, התנגשויות אלסטיות בין חלקיקי יסוד ברוב המקרים אינן גורמות לנזק או חיכוך, כמו במקרה של מולקולות גז הכלואות בבלון של אמישרגז, היכולות להתנגש זו בזו לאורך שנים ארוכות בלי לאבד ממהירותן הממוצעת או לשנות את הרכבן.

    טענה ה’: מודל זה אינו מודל בעלמא לצורך הטיעון – כך באמת מתרחשים הדברים במקרים רבים במשפחת החלקיקים האלמנטרים, ע”ע נייטרינו וחדירותו הרבה לחומר.

    טענה ו’: הטרול המכונה ר.ח רפאי.ם הוא קשקשן חסר תקנה ללא הבנה מינימלית בחוקי הפיזיקה. באתרים בעלי פיקוח הדוק יותר הוא היה נחסם מזמן בגין הטרדה סדרתית של מגיבים.

    אם יש לך אי אילו השגות על הטענות, זה הזמן ללבן אותן לפני שעולים על האוטוסטרדה.

  649. ישראל,

    הבנתי את עיקרון השקיפות לגבי כדור הארץ עם חור. זה יחייב אותך בעתיד להסביר מה המקבילה של החור בחלקיקים כי ברור שבאנלוגיה של כדור הארץ, אם אין חור, לא משנה מה תהיה מהירותו של הפגז/מטאור, לא הייתי רוצה בסביבת הפגיעה (וגם רחוק מהסביבה), אבל נעזוב את זה בנתיים .
    אתה יכול להמשיך עם האוטוסטרדה.

  650. רפעין האימבציל.

    שאלה זו עדיף שתפנה אל מייקל. אתה הרי החסיד השוטה שלו, תרתי משמע. למרות שמוזר הדבר, אם אדם כה מוכשר, ישחית זמן יקר על דפ”ר האתר.
    בקיצור, תחפף נודניק.
    יאללה, עצבן, להתחיל עם הקללות. אבל משהו טוב, אה? לא התת רמה הרגילה.

  651. ישראל

    לפי איזו תורה אתה קובע -‘גוף א’ יהיה שקוף לגוף ב’ במידה וגוף א’ נע במהירות מסוימת דרך גוף ב”?
    בעצם, עזוב. אתה לא תבין גם אם ילדה בכיתה ט’ תיתן לך בראש עם פטיש 5 קילו שרשום עליו ‘טיפש’.

  652. מאיר, עברתי קצת על המודל שלך. הוא נחמדד ללא ספק, אך סובל מהבעייה שהוא איננו ראשוני. קראתי תאוריה זהה לפני 6-7 שנים, אך לא הצלחתי למצוא אותה בקישורים במחשב. אם תרצה (מאוד), אנסה לראות אם יש לי את המאמר במחשבים ישנים בבויידם.

  653. לא אכפת לו. זה גם יכול להיות אסטרואיד מהחלל. החללית היתה רק לצורך הדגמה. הנקודה היא זו: מעל מהירות מסויימת של הגעת הפגז לארץ ( במקרה שלנו – מהירות המילוט 11.2קמ/שנ) הגוף יעבור את הארץ ללא השפעה עליה או עליו, ולכן הוא “שקוף” מבחינתה. מה שענה על שאלתו של ר.ח. :

    “אם אתה אומר שגרוויטציה ותאוצה נובעות מכל שחלקיקים דוחפים את החומר אז אתה לא יכול לבוא מצד שני ולטעון שהחומר שקוף להם. האם כוונתך שבמהירות נמוכה הם גרוויטציונים/תאוצה ובגבוהה שקופים?”

  654. אם מה שמתרחש באיזור החללית (או רחוק יותר) איננו משנה, אז “מה איכפת” לכדור הארץ האם הפגז רכש את מהירותו ההתחלתית כתוצאה מירי מחללית או כתוצאה מנפילה חופשית ממקום כלשהו רחוק יותר?

  655. הפלטפורמה היורה אינה קשורה למערכת פגז/ארץ. היא יכולה להיות במרחק 100 שנות אור מן הארץ ואין לה שום אפשרות לדעת אם הפגז נלכד בכבידת הארץ או לא.

  656. ישראל,
    גם פגז שנורה גורם לרתיעה של הפלטפורמה היורה. תוספת הוקטור V כפול מסתו של הפגז שווה לתוספת וקטור בכיוון הנגדי של מהירות החללית כפול מסתה.
    כן אני בין היתר מדיון התארכות הזמנים מתישהו. אני כבר לא זוכר על מה היה הדיון ואם הייתי בעד או נגד, אבל אישית ברור לי (ולא שחלילה נעלמו מעיני התצפיות והניסויים הטוענים אחרת) שלחלקיקים אלמנטריים יש שעון פנימי שדיוקו איננו תלוי במהירות תנועתם מהסיבה הפשוטה שכולם נעים במהירות האור, וש”צופה במנוחה” הוא ספקולציה שנוח לרתום אותה לצורך חישובים מסויימים, אבל היא חסרת בסיס פיזיקלי.

  657. יעל

    לא זכור לי שדיברתי על מסה אפלה אי פעם.

    האם את יעל מכתה ט’?

    מאיר

    זה אינו טיל, ואין גזי פליטה. פשוט פגז 155ממ. חשוב עליו כעל גוף בנפילה חפשית עם תוספת וקטור מהירות V בכיוון הארץ.

    האם אתה מאיר מדיון התארכות הזמנים בשנה שעברה?

  658. ישראל,
    התנע שהחללית רוכשת בכיוון הפוך לתנועת הטיל ו/או התנע של גזי הפליטה המאיצים את הטיל, בהתאם לשיטת השיגור) מקזזים את התנע של הטיל לעניין תרומת תנע השיגור לכדור הארץ.

  659. סליחה ישראל,

    יתכן והטעות היא בי: אני לא מבינה איך ניתן לדבר על תיאוריה כאשר אין בסיס נתונים להתייחס אליו. על כדור הארץ קל לבחון התנאים הפיזיקליים, אולי אפילו רק מהאינטואיציה והניסיון האישי. אני לא מצליחה להבין איך אתם יודעים מה ואיך קורה עם מסה אפלה ואנרגיה אפלה? מרחק של מיליוני ומיליארדי שנות אור מאיתנו. אתם מקושרים לנתוני המחקר בנושא?

    הקישור שהבאתי עושה בדיוק את זה – נלקחו מספר תיאוריות מובילות ונבחנו מול נתונים שנאספו לאחרונה מלווינים וטלסקופים.

  660. ר.ח,
    היות שדילגנו על השלבים הבסיסיים, לא נותר לנו אלא לקבל את תנועתם של חלקיקי החומר האפל כאקסיומה. כדי לסבר את האוזן אזכיר רק ששרשראות ה”אין” הינן דינמיות.
    בפיסיקה אנחנו מקשרים כל תנועה למושג “אנרגיה”. אולם המושג הזה עצמו אינו מוגדר בפיסיקה. ההצגה של אנרגיה כמאסה, ולהיפך, היא מעגלית ואינה מגדירה אף אחת מהן. על פי המודל שלי, התנודות של חלקיקי החומר האפל הן אלה שמגדירות גם את האנרגיה וגם את המאסה

  661. ר.ח רפאי.ם,
    ראשית, אינני לומד בהרווארד.
    שנית, לא רק כאן אלא גם באוניברסיטה בה אני לומד לא מבינים את הרעיונות שלי.
    מי שעושה את הסלט זה לא אני. הרעיונות שאני מביא הם פשוטים, אך לא מסתדרים עם האינטואיציה הפיסיקלית. כפי שכבר אמרתי, הפיסיקה היא תוצר מאוחר של המודל ומשום כך אין לייחס לו שום חוק פיסיקלי.

  662. מאיר.
    במקרה הראשון, שבו נלכד הפגז בכוח המשיכה של הארץ, חייב התנע לעבור למערכת פגז/ארץ. אחרת, לאן נעלם תנע הפגז? איפה שימור התנע?
    במקרה השני הפגז ממשיך בתנועתו באותה מהירות ממנה נורה מהחללית, ולכן מאותם שיקולי שימור תנע, לא הועבר כל תנע לארץ.

    יתרה מזאת – זו אינה רק דוגמה לצורך המחשה. זו הדרך שבה פועלים הדברים ברמת החלקיקים האלמנטריים, כמו אלקטרון או פרוטון המגיעים לאיזור בעל מטען חשמלי, ולא משנה עם המטען הוא בעל אותו הסימן או הפוך.

    ר.ח.

    “אז אם אין אנרגיה ואין כוחות איך החלקיקים האפלים נעים ?”
    חלקיקים או גופים אינם זקוקים לאנרגיה או כוחות כדי לנוע. מבחינתם, כל עוד אין תאוצה, הם במצב מנוחה.

    יעל.

    הראי לי בבקשה תגובה אחת שלי ללא קשר למציאות, למען אשפר דרכי.
    ולא ממש הבנתי את רלוונטיות הקישור ששלחת. תוכלי לבאר?

  663. ישראל,
    לדעתי בכל המקרים של הפגז שתארת לא תהיה העברה נטו של תנע בסיום התהליך.
    במקרה של פגז הנורה מרחוק (נניח שהוא רודף אחר כדור הארץ) הוא יאט את מהירות כדור הארץ עם התקרבותו (תוך רכישת תוספת תנע לפגז) ויאיץ את כדור הארץ עם התרחקותו (תוך החזרת אותה כמות התנע שרכש בהתקרבותו).

  664. בוקר טוב יובל, ישראל ויהודה,

    אני עוקבת אחרי הדיון הזה, למרות שחייבת לציין שלא ראיתי משהו מפתיע מבחינת החדשנות שלא לדבר על הקשר לעובדות והמציאות.

    אם תרצו, יש קישור רלוונטי לנושא הדיון:

    http://arxiv.org/abs/1202.0892

  665. יובל,
    אתה אומר “בשלב הזה עדיין אין מה לדבר על אנרגיה, משום שהמונח הזה לא מוגדר ברמה זו של חלקיקי החומר האפל”.
    אז אם אין אנרגיה ואין כוחות איך החלקיקים האפלים נעים ?

  666. בחיי יובל מה מלמדים אתכם שם בהרוארד? קראתי שוב את מה שכתבת ואני חייב להגיד לך שאם לא היינו מתכתבים ב”הידען” הייתי בטוח שאנחנו במסעדת אל באבור בגליל המערבי- מרוב הסלט שעשית בין המושגים.

  667. יובל

    זה בדיוק מה שאני מנסה להסביר לך, צריך להבין את הדברים הבסיסיים.
    בבסיס המודל שלך אתה טוען שלא היה שום דבר – איך אתה טוען זאת כאשר בפועל מתקיים ‘שום דבר’ – גם אם הוא בלתי גדיר?
    אתה מתקדם משם וטוען שעוד לפני הפיזיקה יש כל מיני חלקיקי חומר אפל ועוד קיימת בינהם מכניקה.
    – איך זה ייתכן? איך תיתכן מכניקה של חלקיק לפני שקיימת פיזיקה? או שאני טיפש או שאתה גאון.

    אתה רואה מה אתה עושה? אתה מחליף את הפיזיקה המודרנית בפיזיקה שלך. ועושה זאת ללא הוכחות בדמות משוואות מתמטיות.

    הדרך הנכונה תהיה, שוב פעם, לגזור את הפיזיקה שעדיין לא גילינו מתוך הפיזיקה הקיימת. ולא להיפך. (הבאתי לך כמה רעיונות כדי לעזור למודל שלך אבל אתה מתעלם).

  668. סטודנט
    “זה נראה שאתה לא יודע מה זה תהליך הפיך או לא מבין מה משמעותו.”
    אני דווקא חושב שכן, אבל מי אני שאשפוט?

    ר.ח.

    “אם אתה אומר שגרוויטציה ותאוצה נובעות מכל שחלקיקים דוחפים את החומר אז אתה לא יכול לבוא מצד שני ולטעון שהחומר שקוף להם. האם כוונתך שבמהירות נמוכה הם גרוויטציונים/תאוצה ובגבוהה שקופים?

    זה בדיוק מה שאני אומר. הרי דוגמה:

    נאמר שאתה קודח מנהרה מקוטב לקוטב בכדה”א. אם אשמוט כדור בקוטב אחד, הוא יפול במהירות גוברת עד למרכז כדה”א ואז יתחל להאיט עד שיגיע לקוטב השני, שם יעצר ויתחיל במסעו חזרה. אם לא נפריע לו, הוא ישאר לעד באותה תנועה הרמונית פשוטה.

    מה יקרה אם נירה פגז תותח מחללית החונה במרחק מסוים מכדה”א לכיוון המנהרה? אם הפגז יגיע לפני הארץ במהירות הנמוכה ממהירות המילוט (11.2 קמ/שנ למיטב זכרוני), הוא ילכד בכבידת הארץ ויתחיל גם כן בתנועה הרמונית, כשהפעם התנועה ההרמונית חורגת מגבולות שטח פני הארץ. במקרה זה יעביר הפגז מומנטום למערכת ארץ/פגז וידחוף קדימה את הארץ, וזאת ללא מגע ישר עימה, חיכוך או עיוות כלשהו.

    אם לעומת זאת מהירות הפגז היא מעל מהירות המילוט, הרי שהפגז, אחרי שיתרחק מרחק רב מהארץ, ישאיר את הארץ בדיוק באותה מהירות כקודם, וגם הארץ לא תשפיע כהוא זה על מהירות הפגז, שיחזור לאותה המהירות בו ירתה אותו החללית באותו המרחק מהארץ. שום מומנטום לא הועבר, שום חיכוך לא נוצר, שום עיוות, וכולם חזרו למצבם הקודם, מבסוטים חלאס. הפגז מוכן לעשות את אותו תרגיל עם תואם כדה”א הבא, או המיליארד שאחריו, בלי בעיה ובלי תוספת אנרגיה.

    “אם כך מי צריך את השקופים? במה הם מוסיפים ידע?”

    על כך – במודל האוטוסטרדה. אולם אני חייב לודא את שקיבלת את עיקרון השקיפות. אי אפשר להתקדם בלעדיו.

    יובל.

    “כאשר אתה לוקח תופעה פיסיקלית ומשליך ממנה על תופעות אחרות, אינך מקדם דבר אלא רק יוצר טענה מעגלית.” (הרבה עושים זאת. ראה תאוריית מקסוול. הצלחה כבירה.) למשל, אתה מסיק מן “המטוטלת הבליסטית” לגבי תוצאות ניסוי מייקלסון מורלי, ובכך טוען למעשה כי ל”חלקיקי” האור יש תנע. (יש להם. נוסחת איינשטיין המפורסמת בנויה על כך.) אתה מתעלם מן הדואליות גל-חלקיק של האור ( אה? איפה? עוד לא הגענו לכך עדיין) או מקבל כמובן מאליו שלקליע הרובה הפוגע בגוש הפלסטלינה יש תנועה גלית . (לא מבין למה אתה מתכוון. למה גלית?) יש עוד הרבה ביקורת, אבל פרה פרה.
    בתמימותי ניסיתי להראות שיש לתהיותיך פתרונות אחרים, (נו, איפה?) אבל אתה עסוק רק בייחצון הדברים שלך. (לא עוסק בשום ייחצון. מעמיד לביקורת. רוב התגובות פה הם על דבריך. האם גם אתה מייחצן?). אם תרצה ללמוד את המודל שלי אחרי שתגמור למצות את הגיגיך, אהלן וסהלן. (מנסה בכל כוחי ללמוד את התאורייה שלך, אבל קשה מאוד כשלא יודעים מהי. רק עכשיו נודע לי שאתה מתכתב עם ר.ח. אולי שלחת לו את המודל. לי לא, כך שאין לי אפשרות לדעת על מה בדיוק אתה מדבר. אתה גם לא עונה לשאלות. שאלתי אותך מה מייצב את גלגל האופנוע. איזה הסבר יש לך לניסוי מ-מ. איך אתה מסביר את אי הלוקליות. את הכבידה. השאלות הן ממוקדות, התשובות לא.)

    חוץ מזה, לטענתך על מהירות שונה לאור במרחקים גדולים מהארץ: האין זה מחייב ערכים שונים לקבועי החשמל והמגנטיות?

    טוב, עכשיו לחייך כולם.
    🙂

  669. יובל,

    זו במקרה הטוב השערה. אתה יכול לאשש את ההשערה עם ניסויים שתעשה, במידה ותצליח להגיע לירח ולמדוד את מהירות האור שם. או לאנדומדה.

  670. יובל

    חשדתי שבזה מדובר.
    הנטייה לטעות היא בדיוק בנקודה הזאת שבה אתה חושב שהתנועה של האור מואצת, במקום להסתמך על המדידה (הנכונה) שקובעת שמהירות האור היא קבוע, ומה שגורם להאצה (לאפקט דופלר) הוא אפקט אחר שעדיין לא מוסבר במלואו אבל מוגדר כ- אנרגיה אפלה. (הבעייתיות בקביעה היא מפני שבין התחזית הקוונטית לבין התחזית הקוסמולוגית קיים פער בתוצאות שמתבטא בכ120 סדרי גודל – לפי ויקיפדיה)

  671. סטודנט, טכניון,
    אדרבא! הבא נא תצפית מאנדרומדה.
    לי, אגב, יש אישושים לכך שמהירות האור אינה קבועה. קח את התופעה של תזוזת פסי הבליעה בספקטרום המגיע מגלקסיות רחוקות. הסתמכות על הטענה שמהירות האור היא קבועה, יחד עם ייחוס התופעה לאפקט דופלר, מביאה להסכמה על כך שהגלקסיות מתרחקות במהירות גדלה והולכת ולאמונה בקיומה של “אנרגיה אפלה”. לעומת זאת, ראיית תנועת האור כמואצת תוך כדי מסעו אלינו מסבירה את התופעה בלי להצטרך לאנרגיה האפלה.

  672. יובל

    מה עם פוטונים שמגיעים מסופרנובות? זה שהחישובים מראים תוצאות שקובעות שהפוטון נע במהירות האור – זה לא מספיק?

  673. יובל
    אתה לא יכול להגיד שבמודל שלך נבנית הפיזיקה מאוחר יותר אם אין לך הוכחות לכך איך זה קורה. ההוכחות האלה יכולות להיות רק משוואות מתמטיות שמסבירות פיזיקה ושממנה ניתן לגזור את המשמעות הפילוסופית של התופעה. ולא להיפך.
    כמו שאמר סטיבן הוקינג: הפילוסופיה מתה.

  674. סטודנט, טכניון,
    עד כמה שידוע לי, כל הניסויים המאששים כי מהירות האור בריק היא קבועה נערכו בסביבתו הקרובה של כוכב הלכת שלנו. כדי להזים את טענתי לא אבקש להרחיק עד צדק ושבתאי, למשל, אלא אסתפק בתצפית כזו שנערכה על פני הירח הפרטי שלנו.

  675. ר.ח רפאי.ם,
    נכון שטרם התעמקתי בדברים שאמרת, אך זה משום ששעתם עדיין לא הגיעה. כרגע אני מציג שלב מאד בסיסי של המודל שלי. תורם של הפיסיקה ומושגיה יגיע אחרי שהשאלות הנוכחיות יתלבנו

  676. יובל,

    ההוכחה לא נמצאת בקישור (או בשום מקום אחר, אין במדע הוכחות). בקישור נכתב שמהירות האור בריק היא קבועה וזה אושש בניסויים.

  677. ר.ח רפאי.ם,
    בפסקה הראשונה אתה מדבר על ריק, אנרגיה, קוואנטים ופיסיקה בכלל. המדע הזה ומונחיו לא קיימים בשלב זה של המודל שאני מביא אלא נבנים יותר מאוחר, וחבל להתעכב עליהם כעת.
    על פי עצתו של ר.ח זנחתי, לעת עתה, את הסיפור על ה”אין” לבדו והסכמתי בצער מסוים לדלג לשלב מתקדם יותר. הייתי שמח לחזור אליו, אך בגלל חוסר היכולת שלי להסביר אותה, חוששני שלא אעשה זאת בקרוב.

  678. יובל
    עם כל הכבוד לר.ח ולסטודנט, טכניון – ויש כבוד – ר.ח מסביר לך את הנקודות שאני מציין לפניך (ואינך התייחסת אליהם). וסטודנט, הזכיר כבר שהוא לא מתמצא כל כך בפיזיקה קוונטית ומעדיף לא לדבר על דברים שאינו מבין. צודק. וזכותו.

  679. סטודנט, טכניון
    אינני אומר שאינך דובר אמת, אבל בקישור שנתת אין הוכחה לכך שמהירות האור בריק הינה קבועה. יש טענה כזאת, אבל לא הוכחה. ציון העובדה שלומדים את זה בכל האוניברסיטאות בעולם (ולא רק בטכניון שנה ב) אינו משנה את העובדה שמהירות האור בריק לא נמדדה במרחק של עשרות קילומטרים מחוץ לכדור הארץ, שלא לדבר על מערכת השמש ומעבר לה. אם לא היה ברור מלכתחילה, הטענה שלי היא לגבי מהירות האור לא רק בסביבתנו הקרובה כי אם בכל מקום ביקום.

  680. מכאל רוטשילד

    אם אתה כבר קורא את התגובה הזאת, אז למה שלא תצטרף לדיון? (אני יודע שאתה עסוק ב”מסעות הצלב” 🙂 ואפילו כבר כתבת כאן כמה שאלות ליובל חייקין בתחילת הדיון, אבל בכל זאת, זרוק איזו פנינת חוכמה לקוראים שבינינו, שלא נשתעמם לפחות 🙂 )

  681. יובל
    מהירות האור בריק קבועה. והריק עצמו מוגדר כאנרגית נקודת האפס.
    אין עוד ריק מלבדו (בעולם שבו אנו חיים. {אולי בעולם הפנטזיות כן..}).
    אנרגיית נקודת האפס, משום ש- 0 מעלות קלווין (273.15- מעלות צלזיוס), הוא מצב שבו מערכת (פיזיקלית) נמצאת במצב היסוד שלה. בטמפרטורה זו (לפי תורת הקוונטים), כל חלקיק קוונטי נמצא במצב אנרגטי המינימלי ביותר. במידה ומצב האנרגיה של חלקיק הוא 0, אזי החלקיק ישהה במצב מנוחה (כלומר ללא תזוזה), ומיקומו יהיה ודאי. זהו מצב שלא יתכן מבחינה קוונטית מכיוון שלפי עיקרון האי-וודאות לא יתכן שיהיה מידע גם על מיקומו וגם על מהירות תנועתו של החלקיק. לכן, לפי מכניקת הקוונטים- לא יתכן חלקיק שימצא במצב של אנרגית נקודת האפס, ומכאן שהריק עצמו לא יתכן להיות במצב של אפס מוחלט. חייבות להתקיים פלוקטואציות קוונטיות כלשהן בתוך הריק עצמו.

    מה שאתה מציע הוא פתרון (אין-אין) ש – לדבריך – קודם לפיזיקה והכל, בעוד ש,
    הפיתרון שלך נמצא בגבולות הפיזיקה עצמה (ואפילו מנסה להחליף את הפיזיקה הקיימת). ואין לך שום דרך לבנות משהו מתוך משהו לא קיים. אתה רק יכול להתבסס על פיזיקה קיימת ולעזור למצוא לה פתרונות, במקום – לבנות מודל שיסביר את הפיזיקה הקיימת.

  682. יובל,

    זה מוסבר בקישור שהבאתי (אתה מוזמן לקרוא אותו במלואו). מכיוון שאני לא אדריך אותך איך לקרוא אותו, אציין כאסמכתא שמהירות האור בריק, c, היא קבוע פיסיקלי, וזה נלמד, למשל, בקורס פיסיקה 2 בטכניון.

  683. סטודנט, טכניון,
    כשאתה אומר שאתה מעלה את הטענה למען שאר הקוראים, אתה רומז שאני טועה ומטעה. כדי שאתה לא תיפול באותה קטגוריה, רצוי שתביא את ההוכחה המפורשת לדבריך.

  684. ר.ח,
    המצב שאתה מציין שהוא בלתי אפשרי הוא אכן בלתי אפשרי בפיסיקה. אך המודל הזה אמור לבנות את הפיסיקה מתוך המטפיסיקה, כך שההגדרות התופסות בפיסיקה לא קיימות בו. בשלב הזה עדיין אין מה לדבר על אנרגיה, משום שהמונח הזה לא מוגדר ברמה זו של חלקיקי החומר האפל.
    מה שלא ראו במאיצי החלקיקים זה מה שלא חיפשו. אבל לא צריך ללכת רחוק כל כך. האם מצבם הרגיל של פרוטונים ביקום הוא יונים חיוביים או שהוא גרעיני אטומים של מימן? איך, לדעת הפיסיקאים, הצליחו כל הפרוטונים בכל נקודה ונקודה ברחבי היקום למצוא לעצמם אלקטרונים?

  685. יובל,
    אתה אומר “כאשר שני חלקיקים מתנגשים, לא קורה שום דבר. לא קיים אצלם שימור תנע או התנגשות אלסטית.”
    מצד שני אתה טוען שהחלקיקים צפידים כלומר בנקודה במרחב בזמן נתון לא יכולים להיות שני חלקיקים. זה מצב לא אפשרי לפי הפיזיקה מאחר ושני חלקיקים נעים זה מול זה. לשניהם אנרגיה קינטית. או ששניהם יעצרו וייצמדו אחד לשני ואז להיכן נעלמה האנרגיה הקינטית? או ששניהם ינועו כמו כדורי ביליארד ואז זו כן התנגשות אלסטית.

    בנוסף אני לא מבין איך מהחורים המפעפעים במרכז הפרוטון נוצר אלקטרון? איך לא ראו שום דבר מכל אלה במאיצי חלקיקים?

  686. זה כן הוכח, והרבה פעמים. אני לא צריך להשתכנע, קישרתי למען שאר הקוראים.

  687. סטודנט, טכניון
    זה מה שהוחלט ואף הוגדר, אך לא הוכח. היכנס בקישור שציינת לקישור “Is the speed of light constant”
    ותראה שם “c is constant by definition”

  688. ר.ח,
    מה שאתה אומר הינו נכון אך אינו סתירה. אכן במרכזם של מבנים בריונים נוצרים “חורים” ואלה מפעפעים החוצה. כך, למשל, נוצר האלקטרון המקיף את הפרוטון באטום המימן.
    כאשר שני חלקיקים מתנגשים, לא קורה שום דבר. לא קיים אצלם שימור תנע או התנגשות אלסטית. התנועה שלהם היא אקראית וספונטאנית. הם יכולים להישאר צמודים זה לזה או להמשיך לטייל. מכיוון שאין להם נטייה לברוח אחד מן השני, יכולים הרבה חלקיקים כאלה להתאסף בצפיפות גדולה (וכאמור לעיל, בצפיפות מאד גדולה יהיו כאלה שיהפכו ל”חורים” של חלל אדיש).
    עדות מחקרית למהירות אור המשתנה בין ריק לריק נמצאה בניסוי אדינגטון. תנועת האור בריק שבקרבת השמש הינה גבוהה יותר מזו שבריק המרוחק מן השמש. הפיסיקאים מתעלמים מן הפירוש הזה, וזאת משום שבעקבות ניסוי אחר, שנערך כמה עשרות שנים קודם (מייקלסון מורלי), הוחלט לקבל את המסקנה שמהירות האור הינה קבועה בכל ריק.

  689. יובל,
    יש כאן סתירה. מצד אחד אתה אומר שחומר אפל מרוכז = חומר באריוני. מצד שני אתה אומר שכמו במשחק החיים חומר אפל מרוכז הופך לכלום. מכאן שבמרכז של כל חומר בריוני צריך להיות “חור” של מה שאתה קורא חלל אדיש. האם משהו מעין זה נצפה? לא נראה לי.

    דבר שני, אתה מדבר על צפידות כלומר, אם אני מבין אותך נכון, בחלל מסויים יכול להיות בזמן נתון רק חלקיק צפיד אחד, נכון? אם כן, מה קורה כאשר שני חלקיקים מתנגשים?

    דבר שלישי, אתה טוען “ככל שהצפיפות גדולה יותר כך גדלה מהירות האור”. נכון שמהירות האור משתנה מתווך לתווך אולם בריק, לפי הפיזיקה המקובלת מהירותו מקסימלית: C. אתה טוען שיש סוגי ריק כתלות בריכוז החומר האפל. האם אי פעם נמדדה מהירות אור בריק שונה מ- C? האם יש איזה שהיא עדות מחקרית לסוגי ריק? גם לא נראה לי.

  690. ר.ח, ברשותך נעשה קצת סדר.
    הסיפור הכאילו מיתולוגי על ה”אין” וה”יש” נועד בעיקר להסביר משהו בהגדרות המכניקה של החומר האפל. חלקיקי החומר האפל אינם תולדה ישירה מיידית. הסיפור על ה”שרשראות” ממחיש בראש ובראשונה את מושג הזמן. אפשר לדלג על השלבים האלה, להתחיל ישר מן החומר האפל ופשוט לקבל את הגדרותיו כאקסיומה. עוד מעט אשלח לך משהו במייל, אך קודם אסגור כאן כמה קצוות.
    משפט החלקיק של החומר האפל, שלושה סעיפים:
    * היקום בנוי ממספר גדל והולך של חלקיקי חומר אפל ומאינסוף חלל אדיש (“ריק”). בהינתן די חלל אדיש מסביב לחלקיק חומר אפל, משרה החלקיק מעצמו על סביבתו ליצירת חלקיקים נוספים; בהינתן מספר גדול מדי של חלקיקים בסביבת חלל אדיש מועט, מתים חלקיקים בגלל צפיפות ומותירים אחריהם חלל אדיש.
    * צפידות: חלקיק אפל בודד תופס נפח מוגדר במרחב אשר אף חלק ממנו אינו תפוס בוזמנית על ידי חלקיק אפל אחר (החלל האדיש אינו צפיד וחלקיקים יכולים לחדור אליו).
    * החלקיק האפל הינו בעל תנועה עצמית עצמונית המשתנה באורח אקראי בכיוונה ובמהירותה ללא הגבלה, להוציא אחת המתבקשת מן הסעיף הקודם: תנועתו של חלקיק אפל אינה מתקיימת אל תוך נפח התפוס באותו רגע בידי חלקיק יסודי אחר.
    בשלושת הסעיפים דלעיל לא מוגדרות מאסה ואנרגיה ולא מוגדרים כוחות. בכלל, שום דבר שאנו מכירים מן הפיסיקה אינו מתקיים ברמה הזאת. “חלל אדיש” איננו ה”אין” המיתולוגי. והחלקיק האפל איננו “אין” שנהפך ל”יש” או להפך. באמצעות הסיפור על ה”אין” הבראשיתי אפשר לבנות את שלושת הסעיפים האלה ממהות פשוטה יותר, אך כאמור דילגנו עליו (בצער, משום שהדרך לכשעצמה יפה).
    חלקיקי החומר האפל הם צפידים. אוספים שלהם עשויים להגיע לצפיפות כלשהי, אך תמיד תהיה בין החלקיקים כמות מסוימת של חלל אדיש. צפיפות החלקיקים מעבר לסף מסוים יוצרת את החומר הבריוני, ואילו החלל האדיש אחראי לתופעות האלקטרומגנטיות. הקשר בין צפיפות החומר האפל לבין התופעות האלקטרומגנטיות מתבטא במהירות האור. ככל שהצפיפות גדולה יותר כך גדלה מהירות האור, אך בצפיפות גבוהה מעבר לסף מסוים (בתוך חלקיקים בריונים) האור אינו יכול לעבור. כמו כן, האור מגלה התנהגות דואלית שאפשר להבין אותה מן התיאור הזה.
    מאסה איננה תכונה של חלקיק בריוני אלא של צפיפות החומר האפל שבסביבתו. היא מתבטאת בגרביטציה (וכמו שמאיר עמירם מראה, גם באינרציה) ובאינטראקציה עם גלים אלקטרומגנטיים (עידוש כבידתי).
    מבחן ההפרכה שהצעת, “איך זה שיש גלקסיות ללא חומר אפל?”, אינו מבחן מושלם משום שלכל גלקסיה צפיפות חומר אפל הייחודית לה ואפשר לענות לשאלה ולהגיד שבגלקסיות המדוברות החומר האפל הינו בצפיפות קטנה יותר מאשר בגלקסיות אחרות.

  691. יובל,
    כבר עכשיו, עוד לפני שהבנתי את המודל שלך על בוריו ההרגשה שלי היא שאתה מנסה לקפוץ גבוה מידי ולהסביר הכל בכל.
    למה שלא תתחיל מקביעה פשוטה שלך : “החומר המוכר הבריוני הוא חומר אפל צפוף”. לטענה זו יש לדעתי תחזיות להפרכה פשוטות. למשל, איך זה שיש גלקסיות ללא חומר אפל? מדוע כל החומר האפל שם מרוכז כחומר בריוני?

    הסיפור של האיין והיש מורכב ומסובך ולדעתי ללא תימוכין כלל ונשמע לי יותר כמו מיתולגיה מאשר מדע.
    אולם כמו שכתבתי אני עדיין לא מבין את הקשר הישיר בין החלק הראשון (איין-יש) לשני (חומר אפל) וייתכן שהשני לא יכול להתקיים ללא הראשון.

  692. יובל,
    עדיין לא ברור לי הקשר הישיר בין ההתחלה להמשך.
    האם שרשראות ה”איין” הן החומר האפל? או שכל זוג חלקיקי איין (מה זה חלקיקי איין?) שהפך ליש הוא חלקיק חומר אפל ומה שבינהם הוא “חלקיק” איין?
    האם אותם חלקיקים של “רווח” בין החומר האפל, הענן הגלי, הם חלקיקי איין?
    לסיום, מה זה “מתים”? שחוזרים להיות איין?

  693. סטודנט, טכניון
    לאחרונה שיבשתי את כינויך בגלל עיכובי התגובות. לאור ההבהרה האחרונה אני מקוה שכעת אוכל לחזור ולפנות אליך באופן מכובד

  694. ר.ח, תודה
    נכון. דילגתי. ההסבר המפורט ארוך ומכיל איורים להמחשה.
    האסוציאציה לפריונים הביאה אותי בשעתו (לפני 40 שנה, כשעדיין לא שמענו על פריונים ופרות משוגעות) לקרוא למודל שלי “הביולוגיה של הפיסיקה” :-).
    הדרך בה אני מראה כיצד האין מאיין את עצמו אך בכל זאת נשאר אקטיבי היא על ידי בניית ה”שרשראות”. שני “אין”ים צמודים הם ביחד “יש”, אך כל אחד מהם לחוד הוא “אין”.
    כל החלקיקים שאנחנו מכירים הם תולדות של שלב מאוחר. עד החומר האפל, ועד בכלל, ישנה אחידות בכול. השונות מתחילה רק בשלב החומר האפל והיא אינה אלא צפיפויות שונות בהן החומר האפל מסתדר. הגורם לצפיפויות השונות הוא גיאומטרי. הנטייה של החלקיקים בשלב שלפני החומר האפל היא להתרבות. אך בצפיפות מעבר לסף מסויים הם “מתים” (מזכיר מעט את משחק החיים). אלא שסידורים גיאומטריים שונים מכתיבים צפיפויות מירביות שונות.
    החומר האפל מצטופף ליצירת מבנים. בצפיפות קבועה, שומרים המבנים על יציבות בדרך של שיווי משקל דינמי. למבנה היציב אנחנו בפיסיקה קוראים “פרוטון”,
    הרווח שבין חלקיקי החומר האפל שאני מכנה “חלל ריק” (ואין לטעות בינו לבין החלל הריק של הפיסיקה אשר לפי המודל הוא אינו ריק בכלל) גם הוא מהווה מעין חלקיקים. אלא שאלה אינם חלקיקים “צפידים” כמו חלקיקי החומר האפל. הם יכולים להתבטא כחלקיקים כאשר הם מוקפים בחלקיקי חומר אפל, אך הם יכולים גם להופיע כחלק מאיזשהו “ענן”. יש להם אופי דואלי, דהינו הם יכולים לנוע כגל, אך כשגל כזה נתקל במחסום הוא חוזר להתנהג כחלקיק. הפרוטון הוא מחסום משום שצפיפות החומר האפל בתוכו מגבילה את חופש התנועה של חלקיקים ולכן גם את תנועת החלל הריק שביניהם.
    הרגע ראיתי את השורה התחתונה. אשלח לך מייל בקרוב. אך מכיוון שכבר כתבתי, הריני מפרסם.

  695. לאור תגובתך זו עברתי על סבך הגדרות החסימה וגיליתי שבעבר היה מישהו שהזדהה באותו אופן ושנטה להגיב תגובות פוגעניות ונטולות אינפורמציה.
    מכיוון שאותו אדם פסק, כנראה, מהטרדותיו, שחררתי את החסימה ואני מקווה שמעכשיו תיתקל בפחות בעיות

  696. ישראל,

    ושכחתי לציין שאכן יש השעיה בתגובות שלי. למה – שאלה מצוינת. בעבר נאמר לי שזה בשל לינקים שאני מצרף, אבל בתגובות האחרונות אין שום לינק.

  697. יובל,
    רגע, רגע גם לך. אתה רץ מהר מידי בשבילי.
    אהבתי את רעיון תגובת השרשרת שבו גרעין “כלום” אחד הופך ליש ומתחיל לשנות את מי שליידו בהתאם (מזכיר לי פריונים ביולוגיים מעולם אחר, זמן אחר ומחקר אחר, אולם זו האסוציאציה שלי).

    כאן יש בעייה. מה קורה עכשיו ליש+אין? אם התשובה אין אז התהליך יישאר ברמה של אין + אין = יש והיש הזה נהרס. אם התשובה שיש אינרטי ולא מגיב עם האין אזי מהר מאד הכל יהיה “יש”.

    כך שאם מתבצעת תגובת שרשרת כזו, תוך זמן כל שהוא (תלוי במהירות תגובת השרשרת הזו, האם היא מוגבלת במהירות האור?) הכל יהפוך ליש, נכון?

    שלב הבא, כאן נתקעתי. מה טיבו של ה”יש” הראשוני הזה? איך הוא הפך לכל החלקיקים שאנו מכירים? מדוע הוא לא אחיד אם מוצאו מ”כלום” אחיד?
    איך מכאן נובעות ההגדרות של חומר אפל, דואליות חלקיקי וכו’?
    אם אתה מרגיש לא נח להתדיין בטוקבק של מאמר אתה יכול לשלוח לי במייל.

  698. ישראל,
    רגע, רגע, לפני האוטוסטרדה.
    אתה לא יכול להחזיק במקל בשני קצותיו או לאכול את העוגה ולהשאיר אותה שלימה.
    אם אתה אומר שגרוויטציה ותאוצה נובעות מכל שחלקיקים דוחפים את החומר אז אתה לא יכול לבוא מצד שני ולטעון שהחומר שקוף להם. האם כוונתך שבמהירות נמוכה הם גרוויטציונים/תאוצה ובגבוהה שקופים?
    אם כך מי צריך את השקופים? במה הם מוסיפים ידע?
    נקודה שניה, להבנתי הפיזיקלית הצנועה, כדור שחודר דרך בלוק במהירות עצומה אינו נותר ללא השפעה, מהירותו חייבת לרדת. התהליך שתארת לגבי האלקטרון חייב להשפיע באיזה שהוא אופן על האלקטרון. כלומר, אם במהירות נמוכה הם משפיעים על החומר ויש להם אינטראקציה איתו לא ייתכן שבמהירות גבוהה האי האינטראקציה תעלם. OK הבלוק לא יזוז אולם הכדור “ירגיש” את המעבר בחום ואובדן מהירות. ונראה לי שאני קצת חוזר על עצמי. אז הסברים בבקשה.

  699. ישראל,

    זה נראה שאתה לא יודע מה זה תהליך הפיך או לא מבין מה משמעותו. אם אחרי כל סיבוב של הצלחת המערכת או סביבתה נמצאות במצב שונה (מכל סיבה שהיא, כולל חלקיקים מומצאים), התהליך הזה לא הפיך מהגדרה. ואז, אני לא יודע למה אתה מחפש הסברים “משלך” – כבר יש כאלה. אם התהליך לא הפיך, האנטרופיה עולה. אם הוא הפיך, לא.

  700. קצת ביקורת על ה”מודל” שלך, ובכן:
    כאשר אתה לוקח תופעה פיסיקלית ומשליך ממנה על תופעות אחרות, אינך מקדם דבר אלא רק יוצר טענה מעגלית. למשל, אתה מסיק מן “המטוטלת הבליסטית” לגבי תוצאות ניסוי מייקלסון מורלי, ובכך טוען למעשה כי ל”חלקיקי” האור יש תנע. אתה מתעלם מן הדואליות גל-חלקיק של האור (או מקבל כמובן מאליו שלקליע הרובה הפוגע בגוש הפלסטלינה יש תנועה גלית). יש עוד הרבה ביקורת, אבל פרה פרה.
    בתמימותי ניסיתי להראות שיש לתהיותיך פתרונות אחרים, אבל אתה עסוק רק בייחצון הדברים שלך. אם תרצה ללמוד את המודל שלי אחרי שתגמור למצות את הגיגיך, אהלן וסהלן.

  701. יובל.

    קודם כל אומר את דעתי: רוב המודלים הפיזיקליים שאינם מיינסטרים הם “הזויים” במידה זו או אחרת, אם אינם מגובים בנוסחאות או בניסויים חד משמעיים.

    באתר:

    http://www.wbabin.net/

    ובאתרים אחרים, תוכל למצוא מאות רבות של תאוריות, מאמרים, ורעיונות, כולל התאוריה שאיינשטיין הטיפש כלל לא הגה את היחסות, אלא העתיק אותה מאשתו הראשונה.

    ה”מודל” שלי אינו יוצא מהכלל, מה שגורם לניר חברי לפחות פעמיים בשבוע להמליץ לשלוח אותי לאשפוז כפוי. מתקשורת קודמת איתך, התעורר בי הרושם שאתה מחבב סוג זה של הומור שחור ( זוכר את “מותר הבהמה מן האדם?), ולכן הרשתי לעצמי להתבטא כפי שהתבטאתי. אך לצערי פגעתי בך, ועל כך אני מצר. אתה עושה עבודה חשובה באתר, גם בהתווית סיגנון כתיבה קולח ונעים, וגם ברשת התת קרקעית שרקמת בין המגיבים באמצעות אימיילים. יישר כוח!

    בקשר למודל שלך:
    כנראה שאני אכן קשה תפיסה, כי לא הצלחתי להבין איך בדוגמה שנתת, של גלגל האופנוע, נקבל אינרציה. ז”א: מדוע בזמן שהגלגל מסתובב, קשה לנענע אותו ואילו כשאינו מסתובב, קל?

    אם תוכל אתה, או מישהו מהמגיבים, להסביר לי נקודה זו, אשמח.

    בקשר לשאר: ייתכן שהמודל שלך, הוא אכן תאור נכון של היווצרות היקום. מה שאני מחפש הוא את מבחן ההפרכה, וכמו תמיד את התשובה לשאלה: למה?

    קשה לי קצת לעקוב אחרי השתלשלות המודל, מכיוון שמעולם לא ראיתי אותו במקשה אחת, אלא רק בחלקים: באימייל, בתגובות, בשאלות. אם תוכל לשלוח לי אותו כולו, או פשוט לפרסם באתר או בלינק, זה יקל מאוד. כשתעשה זאת, אוכל להגיב ברצינות.
    לדוגמה: הרעיון שלך על תנועה יחסית רק לחומר האפל מעניין, אך מכיוון שאין לי מושג למה אתה מתכוון בחומר אפל, אני תקוע ומתוסכל.

    אז אחכה לשאר המודל, אחינו.

    ובינתיים, מה עם קצת ביקורת על הרעיונות שלי? זה המקום להתחרע ולכסח..

  702. סטודנט.

    נראה לי שיש איזו השעיה בתגובות שלך, כי תמיד הן מופיעות מאוחר, ואפשר לפספס אותן.

    נחזור על הדוגמה:

    צלחת מסתובבת בחלל. האם יש שינוי באנטרופיה של מערכת הצלחת בין סיבוב 20 לסיבוב 80?

    התשובה הסטנדרטית – לא. התהליך הפיך. לפי ה”מודל” שלי, שהיקום מורכב מ”חלקיקים” והם “מתפשטים” כמו פרודות גז בבלון מתנפח – כן. בכל סיבוב תחזור הצלחת למצב שונה במערכת, שבו יש פחות חלקיקים ליחידת נפח.

    אם נשליך על מודל המפץ הגדול, נראה לי שיש משהו בדבר, כי תאורטית מהירות הסיבוב הזוויתית של הצלחת יכולה להיות של סיבוב אחד למיליארד שנה, כך שאין ספק שיש עליה באנטרופיית המערכת הכוללת את הצלחת והיקום אחרי כל סיבוב. מכיוון שמערכת הצלחת בלבד כוללת חלק מהיקום, הרי שהאנטרופיה של מערכת זו עולה גם. לא?

    ר.ח.

    אתה חד עין. עלית על הבעייתיות של הרעיון, כדאי שנחזור עליה כדי שנוכל לחדד אותה:

    “מעבר למהירות מסויימת, מולקולות פשוט מפסיקות להשפיע, ולכן נהפכות ל”שקופות” מבחינת החומר בו הן פוגעות (עד פה הבנתי זה מה שאתה מנסה עם המטוטלת הבליסטית שלך, אני לא כל כך מסכים כי כדור שיעבור במהירות עצומה דרך בלוק אמנם לא יזיז אותו אולם יגרום לחור, הרס חומר ושחרור חום גבוה כך שאתה לא יכול להגיד שהבלוק שקוף)

    זוהי בעיית החיכוך השניה במודל לסאז’ – שאותה העלה לורד קלווין אם אני זוכר נכון. הבעיה התרמודינמית. החיכוך של החלקיקים עם החומר.

    ראשית, שים לב לכך שעדיין נקבל גרביטציה ואינרציה, אבל בעייתית. אבל אני מאמין שקיים פתרון לבעיה זו, שלורד קלווין וקודמיו לא היו יכולים לעלות עליו עם הידע שהיה להם בזמנו.

    שאלה 1: האם קיים מין חלקיק כזה, שעובר מסות בלי להשפיע עליהן כלל, שמסוגל לחדור לא רק בול עץ אלא את כדור הארץ כולו, וגם 100 שמשות בנוסף, בלי להיות מושפע או משפיע על המסות, בלי לאבד מהירות בחיכוך, חלקיק שאפילו שמו נגזר מן המושגים “שקוף” “אדיש” או בקיצור “נייטרלי”?
    …..

    בוא נמשיך:

    ולכן, (מה לכן? איך זה נובע ממה שכתבת קודם?? האם כוונתך שמעל מהירות האור הגלאים שלנו הופכים לשקופים מבחינת האור? אם כן למה דוקא במהירות הזו? ולמה שלא נוכל לעשות גלאים יותר “מוצקים” שימדדו מהירויות יותר גבוהות?) כל גל שיתקדם באתר האקטיבי, יתקדם למעשה בהרבה מהירויות, אולי אפילו כל המהירויות, אך לגבי המודד, או הצופה, הוא תמיד יופיע כבעל מהירות אחת ויחידה (במקרה שלנו – מהירות האור).”

    שאלה 2: האם אין חלקיק זה בדיוק (נייטרינו! נייטרינו!) חשוד בנטייתו לעבור את מהירות האור לפעמים?

    …..

    אבל איך זה קורה? איך יכול חלקיק במהירות מסויימת לעבור מסה כאילו הייתה שקופה עבורו?

    חשוב על אלקטרון המגיע לאיזור בעל מטען שלילי. המטען דוחה את האלקטרון והודף אותו אחורה. אך מעבר למהירות מסויימת של האלקטרון, הוא יצליח לחדור את האיזור, והמטען שקודם התנגד לו, עכשיו דוחף אותו מאחורה, כך שהתוצאה נטו היא שהאלקטרון חדר את המטען ללא השפעה נטו על המטען, ללא איבוד אנרגיה, ללא חור, ללא חיכוך, ללא הרס, וללא זברבירים. האלקטרון מעל למהירות מסויימת פשוט שקוף מבחינת המטען. לא?

    ולמה מהירות האור? זו חייבת להיות מהירות מסויימת ומוגדרת, כמו בכל מודל הידרודינמי אחר. זה מה שיצא.

    אולי אפשר לבנות גלאים מוצקים יותר, שיחסמו את החלקיקים או הקרינה, אך זה מסובך מאוד. גלאים בנויים מחומר, שכאמור שקוף לקרינה, כמו שהארץ שקופה לנייטרינו.

    החלקיקים – או הקרינה – במודל, מתנהגים כמו האלקטרון, למרות שהם כמובן לא אלקטרונים. הם קטנים בכמה סדרי גודל מכל הידוע לנו. הם חייבים להיות, כדי לאפשר תנועה גלית לחלקיקים. אך לא זו הנקודה עכשיו. האם אתה רואה שלא רק שיתכן תאורטית חלקיק שבמהירות מסויימת נהפך ל”שקוף”, אלא שחלקיק כזה אפילו קיים?

    אם כן, נוכל לעבור לשלב הבא, שהוא די מרתק, שלב האוטוסטרדה.

  703. ר.ח,

    אולי באמת נעלבתי מישראל, אבל יותר מכך התייאשתי ממנו. אני פשוט לא מצליח להסביר לו.
    הבנת נכון את הדברים, וגם ההערה שלך “(what ever it means)” היא במקום. את זה שהאין (במינוח שלי “שלילה”) הוא אקטיבי אני משליך מן המציאות הידועה לנו ונכון לעכשיו מקבל את זה כאקסיומה.

    המפץ הגדול שהפיסיקאים מדברים עליו הוא תופעה אפשרית הנגזרת מן המודל שלי, ואני באמת מציג את זה בהמשך (אחרי מספר שלבים הגדרות) כך:
    ברגע הנתון, בו אנו מתחילים את ההצגה, היקום הוא אינסופי (או גדל בלי הגבלה לאינסוף), חד-מימדי בלתי הפיך בזמן ורב-מימדי הפיך במרחק, ומאוכלס כולו בעצמי “יש ראשוני”* במצב אדיש*. רק עצם “יש ראשוני” אחד הינו פעיל. הסובבים אותו מושפעים ממנו, נהפכים לפעילים וממשיכים ומעבירים את תכונתם זו הלאה ללא הגבלה, והרי לנו “מפץ גדול”.
    (*”עצמי יש ראשוני” ו”מצב אדיש” מוגדרים היכנשהו בפרק קודם כתולדה של שלילה).

    שאלת אם ה”יש” הנוצר הוא ערב רב של חלקיקים. התשובה היא “לא”, או “לא בדיוק”. היות שהשלילה שוללת את עצמה ללא הפסק, נוצרות “שרשראות” של שלילות אשר ביחד הן “יש” אך כל חוליה היא שלילה. המילה “שרשראות” היא במרכאות, משום שלא מדובר בהכרח במבנה מרחבי. אם תרצה, תוכל להשוות אותן למרכיבים הבסיסיים של תורת המיתרים.

    הלאה, לא הרבה. בשלב מסוים אני מגיע להגדרה של חלקיקי החומר האפל ומזה ל”מושבות” הפרוטונים ולהגדרת הגרביטציה. מכיוון “מפתיע” אחר לגמרי מגיעה גם הגדרה של פוטונים (ואלקטרונים) הנובעת אף היא מן המודל הזה. המודל מסביר יפה את הדואליות גל-חלקיק של האור כמו גם את האינטראקציה בין הקרינה האלקטרומגנטית לבין חלקיקי החומר האפל המתבטאת בין היתר בתופעת העידוש הכבידתי.

    התנועה היחסית בין גופים הינה למעשה לא בין עצמים לבין עצמים אחרים אלא בינם לבין החומר האפל. ברור שישנה תנועה יחסית בין עצמים, אך השפעת התנועה על העצמים נובעת מן האינטראקציה עם החומר האפל ולא מן האינטראקציה בין העצמים. היות שעד היום התעלמנו מן החומר האפל, “נאלצנו” לייחס את התנועה למה שידענו, ואלה היו עצמים אחרים – מה שפתח פתח להרבה פרדוקסים. גם הגרביטציה הנצפית בין גופים לא נובעת מהם אלא מריכוז החומר האפל שסביבם.

    יש הרבה שאלות שהמודל עונה עליהן, אבל מספיק לבינתים.

  704. יובל,

    ההגיון של התחלת המודל שלך ברור ותקן אותי אם אני טועה.
    1) אנו רוצים לדעת מה היה בהתחלה. לצורך כך אנו מניחים שהייתה התחלה והדברים לא היו קיימים מאז ומתמיד.
    2) מאחר וכל יש שמקורו מיש רק דוחה את ההתחלה אחורה חייבים להניח שההתחלה הייתה מכלום או איין או אפסו או איך שלא נקרא לזה, חושך על פני תהום נשמע לי תאור לא רע.
    3) אותו איין היה חייב ליצור את ההתחלה כי פרט לו כולל אותו הכל כלום ואין כלום.
    4) מכאן שהוא איין את עצמו (what ever it means) ליש (נעזוב את השאלות איך ולמה וניקח את זה כעובדה).
    עד כאן נכון?
    השאלות מכאן, לפי המודל שלך :
    1) האם המפץ הגדול לפי התאוריות הקונוונציונליות (ואל תתחיל להעלב לי עכשיו כמו מישראל) הוא תהליך האיון שאתה מדבר עליו? אם לא מה ההבדל? הרי גם תאוריית המפץ הגדול מדברת על מצב ללא מרחב ויצירת מרחב וחומר יש מאיין.
    2) מה טיבו של אותו יש שנוצר? האם הוא מורכב מערב רב של חלקיקים או שהיה חלקיק בראשיתי שממנו נוצרו הדברים, אם כן איך?
    3) מה הלאה מכאן

  705. טוב, נגיד,
    תשכח מהגלגל. חשבתי שתשמח שאני משתמש במשהו שאתה הבאת, אבל זו לא דוגמה טובה.
    לא אמרתי “מושבות חלקיקים שמחליפות חלקיקים זו עם זו”. אתה לא לקחת את מה שאמרתי ברצינות, ולכן שיבשת. אם יש החלפות של חלקיקים בין מושבות, זה לא רלוונטי כרגע. המושבות נהרסות ונבנות ללא הפסק אך הן שומרות על גודל קבוע, כי הן בשיווי משקל דינמי עם סביבתן. כאשר מושבה נהרסת במקום אחד ונבנית במקום סמוך, היא שינתה מקום. אבל החלקיקים המרכיבים אותה לא זזו בדבוקה אחת אלא התחלפו עם הסביבה. לכן התנועה היא גלית.
    לא נראה לי שהבנת, אבל אני לוקח את האשמה על עצמי היות שלא הסברתי טוב.
    אמור לי בבקשה מה הבנת (ולא סתם ב”העתק-הדבק”, בבקשה) ונראה אם אפשר להמשיך.

  706. ישראל,

    תוכל להסביר את המשפטים הבאים שכתבת (השאלות והערות שלי בסוגריים):

    “מעבר למהירות מסויימת, מולקולות פשוט מפסיקות להשפיע, ולכן נהפכות ל”שקופות” מבחינת החומר בו הן פוגעות (עד פה הבנתי זה מה שאתה מנסה עם המטוטלת הבליסטית שלך, אני לא כל כך מסכים כי כדור שיעבור במהירות עצומה דרך בלוק אמנם לא יזיז אותו אולם יגרום לחור, הרס חומר ושחרור חום גבוה כך שאתה לא יכול להגיד שהבלוק שקוף)

    ולכן, (מה לכן? איך זה נובע ממה שכתבת קודם?? האם כוונתך שמעל מהירות האור הגלאים שלנו הופכים לשקופים מבחינת האור? אם כן למה דוקא במהירות הזו? ולמה שלא נוכל לעשות גלאים יותר “מוצקים” שימדדו מהירויות יותר גבוהות?) כל גל שיתקדם באתר האקטיבי, יתקדם למעשה בהרבה מהירויות, אולי אפילו כל המהירויות, אך לגבי המודד, או הצופה, הוא תמיד יופיע כבעל מהירות אחת ויחידה (במקרה שלנו – מהירות האור).”

  707. הנה מה שהבנתי בליווי הערות:

    הנה לך קורטוב של המודל, וכדי לשמח אותך אתחיל דוקא מתיאורים שלך: גלגל של אופנוע המסתובב בתוך מים. ראה את הגלגל עצמו עשוי מים. ( בסדר עד כה). כדי להבדיל בינו לבין המים בהם הוא שרוי, הנח לרגע שהוא קפוא, דהינו עשוי קרח. ( סביר, לצורך המחשה.) .היות שכך, הוא מקפיא מים שנמצאים סביבו. (מה? למה? טוב בוא נראה לאן חותר יובלי) אך ההשפעה היא הדדית ויש גם מים המופשרים ממנו ומשתחררים לסביבה. ( זוכר את זה מפרק א’, מושבות חלקיקים שמחליפות חלקיקים זו עם זו. ) מחד, הוא נע ביחס למים בהם הוא שרוי. ( מקובל) אך מאידך, התנועה היחסית היא כלל לא בין הגלגל והמים, וכל מה שנע הוא מצב הקיפאון. ( מתחיל להתבאר) לכן, באנלוגיה הזאת, אפשר לראות את תנועתו של חלקיק בתוך החומר האפל כבעלת אופי גלי. ( שו? מאיפה צץ האופי הגלי? ) עד כאן ברור? ( לא! ברור שזה מה שיובלי טוען, אך לא ברור למה!)

    אם אפשר, הבא הסברים להערות שהערתי בסוגריים.

    נסה לעשות אותו דבר, ביקורות בסוגריים, לרעיון שלי. הוא מובא כמעט בשלמותו ב

    https://www.hayadan.org.il/astronomers-reach-new-frontiers-of-dark-matter-130112/#comment-324924

  708. יובל, מאיר
    בדיוק בדקתי עכשיו את המייל, ואכן יש שם מכתב מכם! אז אציץ בו בוויקנד. לילה טוב.

  709. יובל.
    למרות דבריך, אני מרגיש שיתכן שפרוז הפרזתי פרז. אני מקווה שאתה יודע כמובן שאין לי שום כוונה לפגוע סתם – רק הבעתי תסכול מכך שאנו מתקדמים לאט מדי, כשכל אחד אומר כמה מילים על מודלו, ומצפה שכולם יבינו למה הוא מתכוון בדיוק מתוך שברי המשפטים.

    אני די רציני בקשר לרעיונות שלי – למרות שאני די משוכנע שיש לי איזו טעות בסיסית, ולכן אני תמיד מבקש ביקורת נוקבת עליהם (לא עלי). אני מאמין שאם ועד שיוכח שיש בהם משהו, הם בהחלט יכולים להחשב להזויים. זו גם הסיבה שהפסקתי עם האימיילים – בגלל שלא קיבלתי שום ביקורת ממך, ואתה לא הזמנת שום ביקורת ממני.

    אז בוא נמשיך הלאה. אם תהיה מעוניין לדסקס את מודלך – גלה מהו מא’ עד ת’, שנבין. אם לא, אשחרר אותך כפי שביקשת, ונמשיך להתלוצץ כרגיל.

    אתה וכולם תמיד מוזמנים לכסח את האמאמא של כל רעיון אותו אני מעלה. אבל בפרטנות ידענות ומקשיבנות.

    מאיר.

    נראה כמו אתר נחמד, ונכון, זו אותה נוסחה. תמיד חפשתי אותה באינטרנט לשווא. למעשה, גזרתי אותה ממאמרים של פיזיקאי בשם וודווארד. אולי יעניין אותך:

    http://physics.fullerton.edu/~jimw/general/inertia/index.htm

    הוא גם הפנה אותי לספרון נהדר של דניס שימה הקושר יחדיו את כל נושא האינרציה והגרביטציה בנוסחאות.

    הייתי רוצה לשמוע יותר על המודל שלך, בעיקר לראות את הנוסחאות המצורפות.

    לילה טוב לכולם.

  710. לא ברוטאלי, ישראל, ממש לא. אבל, איך להגיד, די … עזוב. למה לריב? אתה כותב יפה. תמשיך ככה.
    הנקודה שאתה מתעקש לא לראות היא שאני מנסה להסביר כאן את המודל שלב אחר שלב. אם אינך מבין את השלב הנוכחי, אינך יכול לעבור לשלב הבא. אמרת שקראת פרק אחד, ואני מאמין לך. אבל לא בטוח שהבנת. אתה חורץ משפט על הדברים שלי, מכנה אותם הזויים ומספר לנו על שיחות עם הכלב שלך. עצם העובדה שאתה מוציא הכל החוצה כאן בפומבי בסגנון פופוליסטי, במקום להתעמק ברצינות ולהתכתב באימייל, למשל, מחשידה אותך בעיניי כאחד שמאוהב בכתיבה של עצמו (ובצדק, אתה אכן כותב יפה) אך לא ככזה שאוהב לקרוא. אולי אני טועה, אבל בינתים אינני מרים את הכפפה שהשלכת לי ברוב חן. אינני זקוק נואשות לחיזוקים ממך או מגולשים אחרים, אינני מרבה בכתיבה כמוך, ויש לי סבלנות. כל שנותר לי הוא להתנצל על הטורח שגרמתי לך ועל בזבוז זמנך היקר. אנא סלח ומחל וחדל.

  711. ישראל,

    לגבי GM=RC^2, אני פוקד לעיתים את הבלוג של העלמה הזו
    http://riofriospacetime.blogspot.com/
    ומכיון ש R=tC (כאשר t הוא גיל היקום) הרי שהעלמה ריופריו טוענת לתאוריה אותה פיתחה בהיותה סטודנטית, שבמרכזה בדיוק אותה נוסחה שהבאת, ואשר לדבריה מסבירה את האנרגיה האפלה .

    על פי המודל שלי מסת היקום היא הרבה פחות מרבע המסה המקובלת כיום, ולכן אין לי ברירה אלא לפקפק בחשיבות הקשר הזה.

    אבל אני לגמרי מודה לך על הפירגון, ובהזדמנות חגיגית זו גם ליובל.
    הדבר המעניין הוא שפיזיקאים וחובבי פיזיקה כל כך עסוקים היום בלחפש אנרגיה אפלה, עד שאין מי שישים לב לתגלית מהסוג שניוטון היה יכול לקרוא לה “המחשבה השמחה של חיי”.

    לכן , אם מותר להשתמש במילים שלך, אני גאה ביובל ובך שתוך כדי הדיון הסוער כאן הצלחתם להבחין במשהו שאולי יש לו חשיבות חריגה (את ה”אולי” הוספתי בשבילכם. כמי שטחן את הנושא עד דק, רמת הבטחון שלי בחשיבות של זה היא מוחלטת).

  712. יובל.
    עזוב קורטוב שמורטוב. דנתי במה שכתבת עם כלאבי, הוא היועץ שלי לענייני מודלים הזויים. הגענו למסקנה שרב הנסתר על הגלוי.
    אז למה שלא תכתוב הכל מהתחלה ועד הסוף, כפי שאתה מטיף לנו? זה כדי שנפסיק למלמל כל הזמן: “אבל למה?” או “מה פתאום?” או “ריבונו של עולם, על מה בדיוק הוא מקשקש?”

    ובפעם האלף: נא להסביר את נושא האי לוקליות! ולא באמצעות אקסיומה כמו: אם שני חלקיקים עוברים שזירה, עוברות תכונות חלקיק א’ לב’ באפס זמן בכל מרחק.

    אם אני ברוטלי מדי לאוזן בריטית יקירי וחברי, ציין זאת ואעדן סגנוני.

    מאיר.

    אני די גאה בך. (לא גאה – גאה בך!).

    אתה נותן תאור איכותי וגם כמותי. יפה.

    מה דעתך על הנתונים הבאים ( אפשר למצוא בויקיפדיה).

    אם:
    G = קבוע הגרביטציה.
    C = מהירות האור.
    M = מסת היקום המשוערת. (אפשר למצוא בוויקיפדיה).
    R = רדיוס היקום המשוער. (כנ”ל).

    אז: GM=RC^2 בקירוב.

    מדהים לא? ומה שלא פחות מדהים הוא שאם נשתמש ב dimensional analysis, אחרי כל הצמצומים משני צידי המשואה, נשאר רק עם:

    F = MA

    החוק השני של ניוטון, חוק האינרציה.

  713. ר.ח רפאי.ם
    הרעיונות שלך, יפים ומעניינים ככל שיהיו, הינם מתקדמים מאד ביחס למודל שלי. הם לקוחים מן הפיסיקה והמתמטיקה, בעוד שהרעיון שלי אינו מכיר כלל את התחומים האלה משום שהוא פרימיטיבי לחלוטין

  714. יובל
    הרעיון שלך היה ברור לי (ולא רק לי) עוד בפעמים הראשונות שהצגת אותו.
    במקום ללכת סחור סחור, אציג לך כאן רעיון שאולי ישנה משהו בדיון התקוע הזה:
    בתמונה הגדולה: האין שאתה מציע הוא כעין השדה שמגדיר את המעטפת של היקום (לא היקום עצמו- אלא בתוך מה שנמצא היקום). מה שאני טוען (כ’תוספת למודל שלך’) זה שהשדה של הואקום הוא שדה שנובע מתוך השדה (שברמה המתמטית מהווה קבוצה סגורה)- שנובע מתוך שדה שהוא ה”אין” שלך.
    כלומר השדה שהצעתי הוא מעין ‘שדה כיול’ בין השדה (אין) שלך לבין שדה הואקום (שהוא השדה שבו מתחילה הפיזיקה). ולפי דעתי השדה הזה (מעבר לכך שלדעתי הוא קיים) הוא שדה של אנרגיה אפלה.

  715. נראה לי שהבנתי. ברור כשמש.
    אתה התחרפנת לחלוטין. זה ברור כשמש.
    🙂

  716. ישראל שפירא,
    הנה לך קורטוב של המודל, וכדי לשמח אותך אתחיל דוקא מתיאורים שלך: גלגל של אופנוע המסתובב בתוך מים. ראה את הגלגל עצמו עשוי מים. כדי להבדיל בינו לבין המים בהם הוא שרוי, הנח לרגע שהוא קפוא, דהינו עשוי קרח. היות שכך, הוא מקפיא מים שנמצאים סביבו. אך ההשפעה היא הדדית ויש גם מים המופשרים ממנו ומשתחררים לסביבה. מחד, הוא נע ביחס למים בהם הוא שרוי. אך מאידך, התנועה היחסית היא כלל לא בין הגלגל והמים, וכל מה שנע הוא מצב הקיפאון. לכן, באנלוגיה הזאת, אפשר לראות את תנועתו של חלקיק בתוך החומר האפל כבעלת אופי גלי. עד כאן ברור?

  717. סטודנטכניון,
    שאלת אם אני “פשוט טוען ליצירת יש מאין”. זה הרי ברור, ואני שמח שעלית על זה. אך לא זו טענתי העיקרית. אני מצביע על המנגנון הגורם לזה. את הדברים האלה כבר כתבתי כאן ב”הידען”. תוכל לראותם בקישור הבא:
    https://www.hayadan.org.il/between-god-and-science-2807117/#comment-300326

  718. ישראל,

    השינוי באנטרופית היקום מורכב מסה”כ השינויים באנטרופיה של התהליכים ביקום. מכיוון שרוב התהליכים המתרחשים הם לא הפיכים (נקראים לעיתים התהליכים הטבעיים), האנטרופיה ביקום עולה. התהליך שאתה תארת, הוא הפיך (אם הבנתי נכון) ולכן אינו תורם לעליית האנטרופיה של היקום.
    אני מציע שתגדיר שוב את הדוגמה, כי נראה לי שיש אי הבנה על מה מדובר.

  719. ההודעה האנונימית לעיל היתה פליטת עכבר. אפשר להתעלם ממנה או להשמידה יחד עם הנוכחית

  720. ישראל,

    תאוצה קבועה a מתקבלת כאשר שינוי המהירות של החלקיק ליחידת זמן הוא קבוע.
    זמן המחזור של חלקיק הוא קבוע.

    אם כוח חיצוני כופה על החלקיק בכל מחזור היסט של מרחק קבוע dX ממרכז השדה שנוצר במחזור הקודם (וכפיה פירושה הכרחת החלקיק לזוז מהמקום אליו הוא מכוון את עצמו מאליו ללא השפעת כח חיצון עקב כל מה שכבר קרה במחזורים קודמים) אזי החלקיק ינוע בתאוצה קבועה a, שגודלה f *dx/dt כאשר dt הוא זמן המחזור ו-f הוא מספר המחזורים לשניה

    השינוי בדלתא G הוא פרופורציוני להיסט, ועל כן פרופורציוני לכוח המסיט (הפיתוח של שלב זה מתוך חוק היפוך הריבועים מופיע בעמוד 5 במאמר שאני מקווה שקיבלת למייל)

    מכאן שחלקיק שמסתו m ינוע בתאוצה a הפרופורציונית לכוח F המופעל עליו.

  721. יובל.
    אתה מלא סתירות כרימון עצמו.

    קודם אתה כותב “הפתרון שלי לבעיות שאתה מצביע עליהן, אשר חוק מאך ממחיש בחוזקה, גלום במודל החומר האפל. האם תואיל להתעמק בו?”

    וכשאני מנסה להתעמק, ומבקש לראות את המודל, אתה משתפן: “כל עוד הוא בעיניך רק ספקולציה, אתה מוזמן להמשיך להנעים את זמננו בפניני לשונך.”

    ואנו – אנה אנו באים?

    אני כל הזמן מבקש שתכסחו את רעיונותי, מציג אותם בפירוש, ולא מקבל תגובות. אתה כאילו מזמין ביקורת, אבל נמנע מהצגת הרעיון בפירוש.

    ואני – אנה אני בא?

    מאיר:

    מה שאני מבקש לראות הוא איך אתה מקבל את F = ma

    סטודנט.

    להבנתי, האנטרופיה של היקום הולכת וגדלה עם הזמן. לכן, אם נחלק זמן זה ליחידות קטנות, הרי שגם בזמן מועט כמו משך סיבוב של צלחת בחלל יש עליה מסוימת באנטרופיית היקום, לא?

  722. ישראל,
    ההסבר בקצרה בהתייחס לאיור שבקישור
    http://tinyurl.com/inertia-fig
    לצורך ההדגמה האיור מתייחס לחתך דרך הקוטר של חלקיק בעל גאומטרית טבעת (כידוע ה”חור” של טבעת ריק, ולכן המסה של החלקיק פזורה על ההיקף). ניתן לסובב את האיור בכל זוית במרחב ולכן כל מה שתקף לגבי טבעת תקף גם לגבי חלקיק בעל גאומטריה של מעטפת כדור.
    העקומה בצבע ירוק מתארת את שדה הגרביטציה העצמי של החלקיק. כצפוי לגרביטציה קוונטית, השדה גועש במרכז החלקיק ומתייצב לצורה ניוטונית כשמתרחקים מהמרכז.
    חלקיקים מוצאים את דרכם בשדה גרביטציה באמצעות דחף הנוצר בהיקפם כלפי המרכז. ככל שהשדה דליל יותר הדחף כלפי המרכז חזק יותר.
    הגרביטציה המוכרת לנו היא כאשר חלקיק אחד מגיב לשדה של חלקיק שני. במקרה כזה מרכז השדה של החלקיק השני הוא מחוץ לגבולות החלקיק הראשון, ולכן תתקבל “נפילה חופשית” קונבנציונלית של כל חלקיק כלפי רעהו.
    המקרה הפרטי של התמדה שונה מהמקרה הראשון רק בכך שמרכז השדה הוא בתוך החלקיק, מכיוון שמדובר בשדה המיוצר על ידי החלקיק עצמו.
    כפי שניתן לראות בחלקו הראשון של האיור – החלקיק בשיווי משקל מול השדה, ומה טוב ומה נעים.
    בחלקו השני של האיור כח חיצוני מסיט את החלקיק ממרכז השדה ימינה. החלקיק טרם הספיק ליצור את שדהו במיקום החדש מכיון שהוא יוצר את השדה במחזורים, ומחזור יצירה חדש טרם הגיע.
    מה שכן הגיע הוא חלק המחזור בו החלקיק מגיב לשדה.
    מכיוון שהופרה הסימטריה של החלקיק כלפי השדה, נוצר הפרש פוטנציאלים גרביטציוני הגורם לחלקיק ליפול נפילה חופשית למצב הסימטרי.
    ככל שהכוח המאיץ מנסה להסיט את החלקיק מרחק רב יותר ממרכז השדה הוא נתקל בהפרש פוטנציאלים גדול יותר, כלומר בהתנגדות רבה יותר לשינוי המצב.
    השרטוט מגזים בהסטה. אם נמדוד את ההסטה ב-X, המקרה הרגיל למהירויות קטנות מתבטא בהיסט קטן dX.
    ניתן להוכיח די בקלות (על ידי שימוש בחוק היפוך הריבועים הניוטוני) שהפרש הפוטנציאלים הגרביטציוני דלתא G הינו לינארי ל-dx ככל שמדובר בהיסט קטן (אך הופך להיות מעריכי כשההיסט גדול). מכיון שהכוח הגרביטציוני פרופורציוני ל-g הרי שככל שננסה להאיץ חלקיק בתאוצה גדולה יותר (היינו להתמיד בהסטתו ממרכז שדהו מרחק רב יותר למחזור) ניתקל בהתנגדות רבה יותר.

    הנה הוכחנו ששלושת חוקי התנועה של ניוטון נובעים מחוק היפוך הריבועים עבור חלקיק במנוחה.

    עבור חלקיק בתנועה במהירות קצובה הכל כנ”ל, בהבדל שבכל מחזור של יצירת גרביטציה השדה נוצר על ידי חלקיק שהואץ למהירות זו, כלומר חלקיק שכפינו עליו לחולל את שדה הגרביטציה שלו בתנאי פתיחה אסימטריים, מה שמנציח את אסימטרית השדה ואת המהירות.

  723. ישראל שפירא!!!
    בדיוק רגע טרם יציאתך לעיר ההימורים התייחסת למודל שלי בהסתייגות כדלקמן: “… היא רעיון שלך, וצריך לציין זאת, כדי לא לבלבל קוראים העלולים לחשוב שזו דעת המיינסטרים. עד כמה שידוע לי, לא. אינני אומר שזה לא רעיון מעניין, אך עד שיתבסס, חייבים להתיחס אליו כספקולציה”.
    כל עוד הוא בעיניך רק ספקולציה, אתה מוזמן להמשיך להנעים את זמננו בפניני לשונך.

  724. אני קורא את תגובותיך, אבל איפה המודל עצמו? סטייל פרק א’ מודל ישן.

  725. לילה טוב ישראל,
    שמח בחלקך שעדיין יש לך מכנסיים לעורך.
    אני מדבר כאן כל הזמן רק על המודל של החומר האפל, בזמן שאתה טוחן סתירות בין מודלים פיסיקאליים שעבר עליהם הכלח.

  726. ישראל שפירא, ברוך שובך בחתיכה אחת.
    שאלת על שורש i והראיתי לך את הדרך. אתה סלקטיבי בתגובותיך.
    בחומר ששלחתי לך אין מה להפריך. יש בו רק ההגדרות.
    הפתרון שלי לבעיות שאתה מצביע עליהן, אשר חוק מאך ממחיש בחוזקה, גלום במודל החומר האפל. האם תואיל להתעמק בו?

  727. ר.ח רפאי.ם,
    לא אני דיברתי על “קבוצה ריקה”. זה אתה אמרת. קבוצה ריקה היא מונח מתמטי שמציין ישות (לא “אינות”) ניטראלית. ה”אין” שאני מדבר עליו אינו ניטראלי.
    גם לא דיברתי על אנרגיה או על פיסיקה בכלל. אלה הם “יישים” שנוצרים מתוך יחסי הגומלין שבין ה”אין” לבין עצמו.
    יהיה לי חבל אם תעזוב את הדיון, משום שבשאלותיך ותהיותיך אתה עוזר לי לחדד את הרעיון.

  728. חזרנו מווגאס, עם זוג מכנסיים חדש.

    סטודנט.
    ” אתה תארת תהליך הפיך (ומחזורי) עבורו השינוי באנטרופיה הוא אפס. אנטרופית היקום לא גדלה בתהליך כזה.”

    אנטרופית יקום גדלה כל הזמן, לא? למה היא לא גדלה גם בתהליך שבו חולף זמן במערכת הצלחת?

    מאיר.
    ברור שהכוונה לחלקיקי יסוד.
    קראתי פעם כתבה שעסקה ברעיון דומה. אבל כמו בדוגמת כדה”א, שבמרכזו אין גרביטציה כלל, איך הוא יצור אינרציה?
    אם תוכל לשלוח לינק או לפרט פה את הרעיון שלך, בעיקר פיתוח אלגברי של חוקי ניוטון, נשמח.

    יובלי המשמיצן.

    התאוריות שלך אינו עומדות במבחן ההפרכה.

    “הלוקאליות (והעדרה) מופיעה בפרק 6 של המודל שלי (פרק 4 במודל הישן שאתה לא טרחת לקרוא אפילו את הפרק הראשון שלו). מי שאיכזב כאן הוא אתה.”

    קראתי גם קראתי. ציינת ששלחת כדי שאלמד איך לבנות מודלים. לשאלותי החוזרות אם אתה מעונין בביקורת לא ענית. איך ניתן להפריך?
    אנא פרק 6, אי לוקליות.

    “אתה לא ממש חזק במתמטיקה ויש סיכוי שתפסיד את כל הונך שם בנבדה. היזהר והישמר לך!”

    אני לגמרי איתך בעניין הזה. הבעיה היא הקזינואים הקמצנים, שכבר שנים אוסרים עלי להתקרב לשולחנות הבלאק ג’ק (המשחק היחידי בקזינו שבו ניתן לנצח את הבית). פראיירים.
    גם כאן, לא ניתן להפריך.

    “אם החלוקה באפס אינה בעיה, אנא הראה לי שימושים מוצלחים שלה.”

    האם זה לא מה שאנחנו עושים פחות או יותר כאשר אנו גוזרים נגזרות מפונקציות?

    ולמה אף אחד לא מתייחס לשאלה שלי בקשר למטוטלת הבליסטית?

    https://www.hayadan.org.il/astronomers-reach-new-frontiers-of-dark-matter-130112/#comment-326491

  729. יובל
    אני אכתוב בקצרה:
    אתה טוען שהאיין לא נוצר מכלום מפני שזה כלום. זה דומה לכך שתגדיר את הכלום כקבוצה ריקה.
    הקבוצה הריקה הזאת תהיה נכונה רק בהיבט של מודל מתמטי. מההיבט הפיזיקלי- הקבוצה הריקה הזאת אינה יכולה להתקיים, מפני שהמצב האנרגטי המינימלי שבו יכולה להימצא מערכת פיזיקלית- נניח שמדובר באנרגיית הואקום- היא גדולה מאפס. היא אפילו אינה שווה לאפס כמו שאתה מנסה לטעון.
    אני מקווה שלפחות זה מספיק ברור, כי אם לא אז אנחנו לא יכולים להסכים אפילו על ההגדרות של הדברים ובמצב כזה אין טעם להמשיך בדיון.

  730. במקרא אפס הוא גם “אין” וגם שם של אל. ר’ עמוס פרק ו פסוק י: …ואמר לאשר בירכתי הבית העוד עמך ואמר אפס ואמר הס כי לא להזכיר בשם אלהים.

  731. יובל,
    חיסור במקום חיבור, איין במקום יש. עברת לצד האפל או מה? תיזהר שלא תהפוך לדארת ויידר.
    דרך אגב אני לא בטוח שאפסו אל המים המתוקים ויוצר העולם השומרי הוא מקור המילה אפס.
    אפס היא פשוט שם נרדף לאיין בתנ”ך

  732. ר.ח רפאי.ם
    זו לא הטענה שלי. לא אמרתי “אין ואין”. אין שם מקום לאות ו”ו.
    כל שאמרתי הוא שראשית הכל היא “אין”.
    נניח שראשית הכל לא הייתה “אין” אלא “יש” כלשהו. השאלה הטבעית לשאול היא “מניין הוא בא”. אם תגיד שה”יש” הזה נוצר מאיזשהו “יש” אחר, אז עדיין תישאר השאלה הזאת בלתי פתורה, כי בסך הכל הסטנו את התשובה. זה בדיוק כמו להגיד שהעולם נברא בידי בורא, אבל איננו יודעים מי ברא את הבורא הזה. לעומת זאת, אינך שואל מניין בא ה”אין”, משום שהוא “אין” ולכן לא הגיע משום מקום. ומכאן אני מוצא כי נקודת המוצא ההגיונית ביותר היא “אין”. אין שום דבר חוץ מן ה”אין”, ולכן הוא “אין” מוחלט.
    כעת נשאלת השאלה איך ה”אין” יוצר את ה”יש”. ובכן, היות שהוא “אין” מוחלט, הוא מאיין את הכל, ובראש ובראשונה את עצמו. מתוך מה שאנחנו מכירים בעולמנו, שלילת שלילה היא חיוב ואני משליך מזה על ה”אין” הראשוני. ה”אין” מאיין את עצמו ל”יש”. בבקשה נסה לחשוב על זה רגע. זה לא מסובך.

  733. בנוסף, תנסה ליישם את המודל שלך על העולם האמיתי (הפיזיקלי) ולא רק המתמטי.
    אחרת המודל שלך לא יחזיק מים.

  734. יובל

    הטענה שלך (אין ואין) מתאימה לכך שאתה תטען שבהתחלה הייתה ‘קבוצה ריקה’.
    עדיין, שוב אני (ולא רק אני) חוזר ואומר, כבר בזאת אתה מחיל הגדרה על משהו שהוא לדעתך בלתי ניתן להגדרה.

    תשמע, אם נדבר דוגרי ולא בשפה מדעית, אם תצליח לפתור את הפרדוקס הזה (להגדיר משהו כמשהו ‘בלתי מוגדר’) אז תזכה בלא מעט פרסים (מתנות הכוונה. לא חומייני) 🙂

  735. כיוונתי בעיקר לכך שהסדר של הטבעיים (להבדיל מזה של השלמים, הרציונליים והממשיים) הוא סדר טוב (המושג המתמטי – Well ordering), מה שמאפשר לנו להשתמש באינדוקציה.

  736. גדי אלכסנדרוביץ’,”
    אכן במספרים הטבעיים, ולא רק בהם, מוגדר יחס סדר. גם השמות שלהם “טבעיים”, “שלמים”, “רציונלים” ו”ממשיים” נובעים מן המציאות הפיסיקלית שהם משקפים. אם כל מה שחשוב במתמטיקה זה ההתאמה שלה למציאות היומיומית, אז לא הייתי מתווכח איתך.

    סטודנטכניון,
    בלקסיקון הפרימיטיבי שלי המילה “חיבור” לא קיימת. בשפה יותר מתקדמת למדנו לקצר ולהגיד “חיבור” במקום “חיסור חיסור”. אבל אני עדיין רק בשלב העוברי.

  737. יובל,
    אני מודה שנכנסתי באמצע הדיון ולא שמתי לב שהתייחסת למודל שלך.
    בכל מקרה, לא נראה לי שצריכה להיות לך סיבה מיוחדת לרצות להתאים את דברי מאך המנוח למודל שלך. הדעה הרווחת היא שנכון להיום אין הוכחה או הפרכה לעקרון מאך כך שדבריו אינם מעלים ואנם מורידים.

  738. יובל,

    אתה שוב אומר כי חיסור חיסור הוא חיבור. כבר אמרתי שאם אתה בוחר להגדיר שחיסור חיסור הוא חיבור – אז אני מבין. במקרה כזה חסכת את הגדרת הנגדי אבל הוספת את ההגדרה הזו.

  739. אין קשר לתורת המספרים.

    הגדרת יחס הסדר של המספרים הטבעיים היא ממש לא שרירותית, ויחס סדר זה (והכללותיו לסודרים) הוא ככל הנראה אחד הדברים החשובים במתמטיקה.

  740. סטודנטכניון,
    תורת המספרים היא תחום סבוך בהרבה מזה שאנחנו מציגים כאן. הגדרת יחס הסדר היא שרירותית. מסיבות הנוחות לנו החלטנו כי אפשר לסדר את המספרים על פני איזשהו סרגל. אך אם תבחן את המרוכבים, למשל, תראה כי ההחלטה הזו לא תופסת. כל מספר הוא ייחודי לעצמו, ולכל אחד תכונות מעניינות. לאחד המספרים, זה שאנו מכנים “אפס”, יש תכונה העושה אותו מאד שימושי בחיי היומיום שלנו. כפי שאתה יכול להגיד כי “מספר נגדי” למספר a הוא זה שחיבורו עם a נותן אפס, אתה יכול להגיד שחיסור a מן המספר הניטראלי נותן מספר שחיבורו (כלומר חיסור חיסורו) למספר a נותן בחזרה אפס. אפשר לטעון, ובצדק, שזו רק סמנטיקה. אבל זה לא משנה דבר לטענה שלי כי כל מה שאנחנו עושים עם פעולת החיבור אפשר לעשות עם חיסור בלבד.

  741. מאיר,
    אמרתי “החומר האפל” ולא “המאסה האפלה”. כאשר האופי של החומר האפל אינו ברור לרובנו, זה אולי נשמע כמו דקדוקי עניות. אולם החומר האפל שעליו דיברתי הוא המרכיב הבסיסי של כל החומר ביקום. כל המאסה ביקום היא של החומר האפל אשר בחלקו גם מתגבש לכדי חומר “לא אפל”. תנועתו במרחב של גוף העשוי מחומר “לא אפל” איננה אלא תנועה גלית על פני החומר האפל.
    אין בלבי דבר רע על מאך המנוח. רק תיקנתי את ניסוח דבריו כך שיתאימו למודל החומר האפל שלי. כפי שציין ישראל, המודל הזה אינו נחלת הכלל (לפי שעה).
    ישראל ביקש מאיתנו “שלא נשתולל” בשעה שהוא מפסיד את מכנסיו בעיר ההימורים. משום כך אינני מתייחס לדבריך המכוונים אליו. אולם אשמח מאד לקרוא את המאמר שלך.
    בתודה מראש

  742. יובל,

    “אז מה? מה הקשר בין זה לבין הטענה שלי שכל פעולת חיבור ניתנת לביצוע על ידי חיסור?”
    – אין קשר בין הדיון בטענתך כי חילוק באפס הוא בעיה לבין הדיון בחיסור חיבור.

    “ולא ברור לי למה אתה מביא לכאן מטריצות.”
    – מכיוון שכפל מטריצות שאינן מסדר מתאים אינו מוגדר. רציתי לדעת האם גם למצב בו מנסים בכל מקרה לעשות זאת היית קורה בעיה.

    “ולא ברור לי מה אתה מנסה להוכיח בשאלה כמה שוה איי פחות פחות איי.”
    – שום דבר. אני מנסה לעשות מה שתארת – לבסס חישוב על חיסור במקום על חיבור, ללא הגדרת איבר נגדי (שטענת כי אפשר לחסוך) וללא ההגדרה כי חיסור חיסור הוא חיבור. אפשר להניח לזה, אם אתה חושב שלא הבנתי אותך באיזשהו שלב וזה אמור להיות משהו פשוט ביותר.

  743. ר.ח רפאי.ם,
    למרות (או בגלל) גישתך השלילית, הרי נכון לעכשיו אתה קרוב יותר מכל שאר המגיבים להבין את הטענה שלי. אתה צודק לחלוטין בכך שאני מבסס את התחלת קיום היקום על “שקר”. למעשה, אני מדבר לא רק על שקר אלא על “שלילה” בכלל. להגדרה הזו אפשר להכניס כל מה שיש לו משמעות “שלילית”. למשל “שקר” הוא שלילה של “אמת”, ו”אין” הוא שלילה של “יש”. “ריק” הוא שלילה של קיום, ובמובן הזה גם “אפס” הוא שלילה משום שבין היתר הוא מציין “ריק”.
    רק אוסיף, למען הדיוק, כי לא אמרתי “אין ואין יצרו יש”. וו החיבור בביטוי שהבאת משמש כפעולת “ועוד”, אך בנקודה הבראשיתית שעליה אני מדבר לא קיימת הפעולה הזאת. מה שאני אומר הוא שראשית הכול שלילה, והשלילה שוללת הכול משום שזוהי מהותה. היא שוללת את עצמה, ולתוצר של שלילת השלילה אנחנו קוראים “חיוב”, “יש”, “אמת”, “קיים”, “יציב”, “אפשרי” או כל מה שאפשר למצוא בו אופי “חיובי”.

  744. יובל
    כל זה נכון, אבל, רק במסגרת המתמטיקה. אם אתה תנסה ליישם את A במרחב הפיזיקלי תצטרך להגדיר משהו פיזיקלי בתור A. במתמטיקה אתה יכול להמציא דבר וירטואלי, אבל בפיזיקה – אפילו חלקיקים יסודיים שמעולם לא נצפו, כן הוגדרו בשביל שיהיה אפשר לערוך ניסויים ומחקרים. ההגדרות שלהם נבעו מתוך המשוואות המתמטיות. אבל אותם משוואות מתמטיות התבססו על תופעות פיזיקליות מוגדרות מלפני-כן.
    כשאתה אומר “אין ואין יצרו יש”- אתה חייב להגדיר מה זה ‘אין’ ומה זה ‘יש’.
    ברגע שאתה אומר: במודל שלי בזמן היווצרות היש לא קיים שום דבר אחר- אתה מפיל את המודל שלך מפני שהוא מתבסס על דבר שקר ולא דבר אמת. זהו דבר שקר מפני שכן יש משהו בזמן היווצרות ה’יש’ וזה ה’אין’. אבל אתה טוען שאין הזה לא מוגדר כיון ששום דבר לא קיים באותו רגע (על פי המודל שלך).
    זה כמו שתגיד קיים 0, אבל ה0 הוא לא באמת קיים. זאת טאוטולוגיה שאינה מסבירה דבר.

  745. יובל,
    בתשובה ל –
    https://www.hayadan.org.il/astronomers-reach-new-frontiers-of-dark-matter-130112/#comment-326447
    א. כשמאך התעסק עם הדלי של ניוטון וניסח את הרעיונות שלו עדיין לא היה ידוע על חומר אפל.
    ב. מסה היא מסה בלי קשר להיותה אפלה או לא אפלה, ולמסה אפלה יש שדה כבידה כמו למסה לא אפלה (למעשה כל מה שידוע לנו על מסה אפלה זה שדה הכבידה שלה). לכן נוכחות (או העדר) של מסה אפלה איננו דבר שיכול לשנות את העקרוני שבעקרון מאך.

    ישראל
    בתשובה ל-
    https://www.hayadan.org.il/astronomers-reach-new-frontiers-of-dark-matter-130112/#comment-326486

    הבנת נכון את הטענה שלי, אלא שיש להיזהר בניסוח שלה. אני מדבר על חלקיקים אלמנטריים, ולא על גופים. כלומר, אני לא טוען ש”קשה להזיז את כדור הארץ ממקומו” מכיון שהוא נוטה ליפול לתוך שדה הכבידה של עצמו. טענה כזו איננה נכונה מכיוון שלפי תאורמת הקליפות ( shell theorem ) סך כוחות הכבידה הפועלים על נקודת מסה בתוך כדור העשוי חומר אחיד הולך וקטן ככל שמתקרבים למרכז (כלומר לא ניתן לטעון שבגלל שהמרחק של כדור הארץ ממרכזו העצמי שואף לאפס אזי הכבידה העצמית הפועלת עליו שואפת לערך אינסופי)

    מה שאני כן טוען הוא שמכיוון שמסת כל גוף (ובכלל זה כדור הארץ) היא סכום המסות של החלקיקים האלמנטריים המהווים אותו, ומכיוון שהמרחק של המעטפת המסיבית של כל חלקיק אלמנטרי כזה ממרכזה הוא קרוב מאוד לאפס, הרי שהשפעת כח הכבידה העצמי של החלקיק על החלקיק עצמו חייבת להיות ניכרת, ואכן אנו מכירים אותה היטב, כ”תופעת ההתמדה”. מכיוון שחלקיק אלמנטרי איננו מסה צפידה (אלא מסה בגאומטריה של מעטפת בלבד), לא חלה עליו תאורמת הקליפה. בקיצור, ההתמדה של גוף דוגמת כדור הארץ היא סכום ההתמדות של החלקיקים האלמנטריים המהווים אותו.

    דרך אגב, במאמר שפרסמתי אני מוכיח (אלגברית) ששלושת חוקי התנועה של ניוטון נובעים באופן טבעי מהמודל הזה, כלומר למעשה כל חוקי ניוטון נובעים מחוק היפוך הריבועים של ניוטון. לא רק זאת, מהאלגברה הפשוטה של מודל הזה נובע באופן טבעי שחוקי התנועה בניסוחם הניוטוני נכונים רק עבור מהירויות קטנות, ואילו עבור מהירויות גבוהות (“יחסותיות” בלשון העם) מתקבלת הגדלה אקספוננציאלית של התנע. כלומר יש גם הסבר ניוטוני לכך שלא ניתן לעבור את מהירות האור.

  746. ר.ח רפאי.ם, בהתנצלות על הקיצור,
    יש מספר שאנחנו קוראים לו מינוס איי, אבל באותה מידה יכולנו לקרוא לו מוישה או כל שם שמתחשק לנו. הסיבה לכך שאנחנו קוראים לו דווקא בשם הזה היא הקשר המעניין שיש בינו לבין מספר אחר, שאנחנו קוראים לו איי (ללא המינוס). הפעולה החשבונית שמבצעים כדי להגיע מ-איי אל מינוס איי היא חיסור של איי מן המספר הניטראלי (שקרוי בשפה העברית על שם האל השומרי אפסו). אין צורך לתת לו הגדרה מיוחדת, משום שהוא מספר כמו כל המספרים.

  747. סטודנטכניון,
    ממש לא ברור לי מה אתה מנסה להביע. החלוקה באפס אינה מוגדרת באלגברה, כי היא לא חד-ערכית. אז מה? מה הקשר בין זה לבין הטענה שלי שכל פעולת חיבור ניתנת לביצוע על ידי חיסור? ולא ברור לי למה אתה מביא לכאן מטריצות. האם מערכת הידען משלמת לך כדי להגיב? לי לא. אתה בטח רץ כעת לחפש איך התחיל הדיון בחלוקה באפס ומוצא שאמרתי את זה היכנשהו. אז נכון. אמרתי. אבל לא כחלק מרכזי או מהותי בסיפור המאד פשוט.
    ולא ברור לי מה אתה מנסה להוכיח בשאלה כמה שוה איי פחות פחות איי. הרי את זה לומדים עוד בבית הספר היסודי. האם אתה צריך אותי כדי לדעת שזה שני איי?

  748. לגדי אלכסנדרוביץ
    ברור לי שפעולת החיבור היא איחוד של שתי קבוצות, אבל בעוד שבחיבור אנו עושים זאת עם שתי קבוצות זרות למשל 5+3 שווה 8 אזי באחוד קבוצות אין חובה כזאת לקבוצות זרות.
    אבל זה משהו מזיכרון הלימודים מהעבר שלי ואני מראש מתנצל אם אני טועה.
    יום טוב
    סבדרמיש יהודה

  749. יהודה – קיים מודל למספרים הטבעיים שבו 0 הוא הקבוצה הריקה ו-S (פעולת העוקב) מוגדרת על קבוצה A בתור

    S(A)=A\cup{A}.

    כלומר, איחוד של A עם הקבוצה שהאיבר היחיד שלה הוא A. באופן הזה אכן מקבלים את פעולת החיבור באמצעות איחוד של קבוצות.

  750. סילחו לי על בורותי או תומתי אבל נידמה לי שפעולת החבור מוגדרת אחרת.יש משהו יותר בסיסי מפעולת החבור והוא איחוד קבוצות על פי תורת הקבוצות של קנטור. למשל הקבוצה שמכילה את האברים
    א,ב,ג,ו,ט, איחוד עם הקבוצה המכילה את האברים א,ה,ל,מ, היא הקבוצה המכילה את המספרים א,ב,ג,ה,ו,ט ל,מ.
    המקרה של אחד אחוד אחד הוא מקרה פרטי של חיבור שני אברים זהים
    לילה טוב
    סבדרמיש יהודה

  751. יובל
    כרגע הדיון נוטה לכיוון המתמטי, אבל בהתחלה הדיון נטה לכיוון התורה של הקוונטים ובמיוחד למכניקה שלהם.
    במתמטיקה אתה יכול להמציא דברים וירטואלים כמו למשל משוואות שאינן מתאימות לתופעות פיזיקליות קיימות.
    כאשר אתה מגדיר (a-) אזי ההגדרה של זה צריכה להיות ברורה וקבילה.
    החלת הגדרה על איבר כלשהו חייבת להתבסס על משהו פיזיקלי- אם אתה רוצה שהתוצאות יהיו קבילות בעולם פיזיקלי (ניתן להתבסס על תצפיות על תופעות פיזיקליות ומדידתן כדי לקבוע ערך שלהן בעולם פיזיקלי).
    מה שכתבת ,אולי, נכון (אני מדבר על המשוואה).
    אבל את מה אתה מגדיר בתור (a-)? למה אתה מחסיר משהו ממשהו שאינך הגדרת מלכתחילה?
    לדוגמה: אני דיברתי על שדות במרחב הקוונטי. לקחתי את ‘נקודת האפס’ והנחתי השערה שהנקודה הנמוכה ביותר אינה שווה לאפס (היא יכולה להיות גדולה או קטנה מאפס- לדעתי היא קטנה מאפס).
    כשהנחתי השערה זו התבססתי על מסקנה שנובעת מתוצאות של החלת הגדרת השדה (במרחב הקוונטי) כשדה מסוים (שדה שמוגדר כוואקום- ובו מתנהלים כל התהליכים הפיזיקליים המוכרים. כולל המכניקה של חלקיקים). מקווה שעד כאן הייתי מספיק ברור כדי שנוכל להמשיך דיון ענייני לפחות בינינו.

    מה שכן בטוח- שסטודנט וגדי אלכסנדרוביץ’ מבינים ויודעים על מה הם מדברים. גם אם הם לא יודעים הרבה אני מייעץ לך להבין את מה שהם אומרים (גם אם מעורבת בזה אלגברה פשוטה). לפחות בנתיים, וכל הכבוד להם והערכה רבה שלי כלפיהם על ההשתתפות העניני בדיון, הם מהווים תחליף ראוי למכאל רוטשילד. 🙂

  752. הגדרת החיבור הסטנדרטי מבוססת על כך שאם אתה חייב, נאמר למשפחת פשע, 1000 ש”ח ועוד 1000 ש”ח, ואינך יודע מה הם רוצים ממך, אז אתה תמות.

  753. יובל,

    החלוקה באפס אינה בעיה כי לא מחלקים באפס. לא מבצעים פעולה שלא מוגדרת. האם לדעתך גם הכפלת מטריצות שאינן מסדר מתאים היא בעיה?

    אמרת שאפשר להגדיר את החיבור בעזרת החיסור ולבסס חישובים כך, אבל לא חישבת. אחזור על השאלה ששאלתי: למה שווה הביטוי a-(-a) ?

  754. גדי אלכסנדרוביץ’,
    ביקשת ממני להגדיר משהו, ואני לתומי חשבתי שאתה מקיש על הדברים שלי. אני חושב שכמו סטודנטכניון גם אתה לא מבין על מה אני מדבר. אולי כי חשבתם שמדובר בפילוסופיה מסובכת. כל מה שהתכוונתי לומר זה שבאמצעות פעולות חיסור בלבד אפשר להגיע לכל מה שמגיעים אליו באמצעות פעולות חיבור. לא התכוונתי לשום דבר עמוק מזה. אנא סלח.
    וכן, בהזדמנות אשמח לקרוא את הפוסטים שלך. תודה.

  755. יובל, הסיבה שבגללה מספיק להגדיר חיבור עבור הטבעיים היא שמרגע שהוגדר חיבור עבור הטבעיים, יתר ההגדרות באות מאליהן בהסתמך על פעולה זו (יש לי סדרת פוסטים על בניית מערכות המספרים אם אתה מתעניין).

    את ההגדרה שנתת לחיסור איני מבין. האם תוכל להסביר לי מהו x-1 על פיה?

    בהגדרה לחיבור שהצגתי, x+1 מתקבל כך:
    x+1=x+S(0)=S(x+0)=S(x) – באגף ימין קיבלנו ביטוי שאינו כולל +.

  756. סטודנטכניון,
    אם החלוקה באפס אינה בעיה, אנא הראה לי שימושים מוצלחים שלה.
    אמרתי שאפשר להגדיר את החיבור בעזרת החיסור, ואתה הבנת נכון שאפשר גם לבסס חישובים על חיסור במקום על חיבור.

    גדי אלכסנדרוביץ’,
    אתה הולך רק על הטבעיים. האם זה בכוונה להכעיס? אני אוהב את המרוכבים, אבל כדי ללכת לקראתך אעבוד עם השלמים. הגדרת החיסור (על פיך) מתבססת על פונקציית העוקב, S, ומוגדרת רקורסיבית:
    x–0=x לכל x שלם, וכן
    (x–(0–S(y))=S(x–(0–y) לכל x ,y שלמים.
    ובבקשה אל תשלח אותי ללינקים, משום שלא את כולם אני יכול לפתוח.

  757. יובל, אתה מנסה להציע הגדרה לא סטנדרטית; בדרך כלל בשלב הזה נהוג גם לתת את ההגדרה ולא להשאיר אותה כ”תרגיל”.

    הגדרת החיבור הסטנדרטית מתבססת על פונקציית העוקב, S, ומוגדרת רקורסיבית:
    x+0=x לכל x טבעי, ו-
    x+S(y)=S(x+y) לכל x,y טבעיים.

    ראה למשל כאן:

    http://he.wikipedia.org/wiki/%D7%9E%D7%A2%D7%A8%D7%9B%D7%AA_%D7%A4%D7%90%D7%A0%D7%95

  758. גדי אלכסנדרוביץ’,
    הצעתי לך תרגיל. אתה רוצה שאני אעשה אותו במקומך? תן לי בבקשה הגדרה של חיבור ואני אראה לך שזו בעצם הגדרה של חיסור.

  759. “ואילו אני סבור שאתה מדבר בטאוטולוגיות.”
    – ייתכן, אבל “בעיית החלוקה באפס” היא לא בעיה.

  760. יובל, לא ענית לי. אני שואל איך אתה מגדיר את החיסור, ואתה במקום לענות אומר רק שזה דומה למשהו אחר (אני לא סבור שזה דומה בכלל, עבור ההגדרה שאני מכיר לחיבור). אתה משתמש גם במונחים שאיני מכיר בהקשר הזה כלל (“קשיר”).

    אני מבקש ממך שוב, אם אפשר – הגדר לי “חיסור”. כלומר, תן הגדרה מפורשת. הנח שאני לא יודע כמעט כלום על מתמטיקה.

  761. יובל,

    אם כך, אז למה שווה הביטוי a-(-a) ?

    אמרת שאפשר להגדיר את החיבור בעזרת החיסור, ולמיטב הבנתי גם שאפשר גם לבסס חישובים על חיסור במקום על חיבור.

  762. גדי אלכסנדרוביץ’,
    הגדרת החיסור מאד דומה להגדרת החיבור. אנו דורשים מן החיבור שיהיה קשיר ויענה על תכונות מסוימות, וזה גם מה שאנו דורשים מן החיסור. אתה מוזמן, כתרגיל, להגדיר את כל מה שאתה יודע על חיבור בלי להשתמש בסימן ‘+’ אלא רק בסימן ‘-‘. זה מעט מסורבל, אך אפשרי.

  763. יובל,

    איך אתה מסיק שהביטוי a-(-a) שווה a+a בלי ההגדרה שחיסור חיסור הוא חיבור?

    “אתה אומר “לא מוגדר” ואני אומר “יש להם בעיה”; שמא אנחנו שוב מתווכחים על סמנטיקה?”
    – אני לא חושב שזה סמנטיקה. אם דבר לא מוגדר, אין לו משמעות.

  764. סטודנטכניון,
    ממש לא הוספתי הגדרה. רק ציינתי שאנחנו, מטעמי חיסכון ונוחות, קוראים לחיסור-חיסור בשם חיבור.
    והלוגריתם של אפס אינו מוגדר בתוך האלגברה מאותה סיבה שהחלוקה באפס אינה מוגדרת שם. אתה אומר “לא מוגדר” ואני אומר “יש להם בעיה”; שמא אנחנו שוב מתווכחים על סמנטיקה?

  765. יובל,

    אני לא יודע איך יראו חיי מעכשיו, כשאני יודע שלא עמדתי בציפיות שלך.

    לעניין, אני עדיין לא בטוח שהבנתי.

    a-(0-b) = a-((c-c)-b) = a+b ?

    איפה מתבצע המעבר מחיסור לחיבור? אם הבנתי נכון, “חסכת” את הגדרת האיבר הנגדי בחיבור, אך הוספת הגדרה שחיסור חיסור הוא חיבור.

    “יש למתמטיקאים בעיה עם הלוגריתם של אפס המקבילה לבעיית החלוקה באפס.”
    – אין להם בעיה, זה פשוט לא מוגדר. אתה מוזמן לקרוא את מה שכתב ד”ר אלכסנדרוביץ’.

  766. סטודנטכניון,
    דווקא ממך ציפיתי שתבין את הדברים ללא קושי. מן התואר שאתה מחזיק לעצמך כאן חשבתי שעשית אלגברה שנה א.
    שאלת איך עושים חיבור רק עם חיסור, ובכן: (a-(0-b נותן מה שנוח לנו לכתוב כ a+b. שים לב שאינני מגדיר קיומו של איבר נגדי אלא רק משתמש בתכונתו של האפס כמספר ניטראלי בפעולת החיסור. בכך אמנם הוספתי הרבה סוגריימים ובסך הכל עשיתי שימוש ביותר סימנים, אך חסכתי הגדרה אחת. שמא תגיד כי בעשותי כך הכנסתי את הגדרת המספר הנגדי בדלת האחורית, אוכל להוסיף איבר שרירותי, נגיד c ולכתוב (c-c) במקום 0. לצורך קיצור, נוח לנו לפתוח סוגריים ולהחליף סימנים ממינוס לפלוס, אך הנוחות הפרטית שלנו אינה גורם שיש להביא בחשבון.
    ר.ח רפאי.ם שאל מדוע “מינוס כפול מינוס נותן פלוס”. במילים שלי “שלילת שלילה נותנת חיוב”. כאמור, המילים “פלוס” או “חיוב” מיותרות.
    אנו מגדירים חילוק ככפל וזה שוב לצורך קיצור. באותה דרך יכולנו להגדיר כפל כחילוק. יכולנו, בכלל, להשתמש במערכת הגדרות שונה: במקום הגדרות חיבור וכפל יכולנו להשתמש בחיסור ובלוגריתמים. יש למתמטיקאים בעיה עם הלוגריתם של אפס המקבילה לבעיית החלוקה באפס.

  767. ישראל שפירא!
    אתה לא ממש חזק במתמטיקה ויש סיכוי שתפסיד את כל הונך שם בנבדה. היזהר והישמר לך!
    צא וחשב (1+i) בחזקת 2. האם יצא לך שני i? אם כן, אתה בדרך הנכונה לחשב את שורש i.
    משה קליין עניין אותי במתמטיקאי אנגלי בשם ספנסר בראון שמצא כי כפי שיש מספר “דמיוני” שהוא השורש של מינוס 1, יש גם ערך אמת “דמיוני” שהחלתו על עצמו יוצרת שלילה.
    הלוקאליות (והעדרה) מופיעה בפרק 6 של המודל שלי (פרק 4 במודל הישן שאתה לא טרחת לקרוא אפילו את הפרק הראשון שלו). מי שאיכזב כאן הוא אתה.

  768. ישראל,

    “ברור, אך לפי התאור שלי, יתכן שיש גורם פיזיקלי נוסף בגללו השינוי באנטרופיה איננו 0. התפשטות היקום. לכן, במערכת הצלחת המסתובבת, בזמן סיבובה הולכת צפיפות היקום וקטנה ומכאן שהאנטרופיה גדלה.”
    – אתה תארת תהליך הפיך (ומחזורי) עבורו השינוי באנטרופיה הוא אפס. אנטרופית היקום לא גדלה בתהליך כזה.

  769. ר.ח.

    אני מקווה שסגרנו על כך, באישורו של פיינמן, שבמודל לסאז’ תהיה גרביטציה, אך תיווצר בעיית חיכוך של הפלנטות עם החלקיקים.

    סגרנו גם, בעידודו הנלהב של מקסוול, שמודל האתר הוא מודל נהדר אם הוא מצליח לחלץ את מהירות האור מתוך קבועי החשמל והמגנטיות. הבעיה היא: מהירות האור יחסית למה?

    וראינו גם, לפי עצתו של מאך, שכל מסה ביקום משפיע על כל מסה אחרת, כמו במקרה הכוכבים הרחוקים. ההשפעה היא אינרציה. השאלה היא: איך?

    כאן אנחנו תקועים. אין, או לפחות לי לא ידוע, שום פתרון במיינסטרים שנותן תשובה הולמת לשאלות אלו. אם מישהו מכיר תשובה כזו, שיקום עכשיו.

    אנחנו נמצאים בשלב קריטי בדיון, לפחות מבחינתי. אני עומד להציע רעיון שיכול לתת כיוון. אני רוצה לתאר ניסוי מחשבתי, מסקנתי בקשר אליו, וביקורת ממך ומכולם.

    סטודנט בטח מכיר מלימודי שנה א’ את השאלה הבאה ( דוגמה 3 בRESNICH HALLIDAY, PHYSICS, פרק 10, COLLISIONS

    המטוטלת הבליסטית.

    לוקחים בול עץ המחובר לתקרה באמצעות חוטים ויורים בו כדור בעל מסה ידועה. באמצעות שיקולי תנע, ניתן לחשב את מהירות הפגיעה של הכדור. ככל שהמהירות גבוהה יותר, יוסט בול בעץ יותר.

    השאלה שלי היא זו: ברור שבמהירות מסויימת, VM, יחדור הכדור ויעבור לצידו השני של העץ. מה אם נירה כדורים במהירות גבוהה בהרבה מVM? המסקנה שלי היא שמעל למהירות מסויימת ההיסט של בול העץ ילך ויקטן, ובמהירות מסויימת הם יפסיקו בכלל להשפיע על הבול ויעברו אותו בלי להזיז אותו כלל.

    לכן נקבל מעין עקומת פעמון של מהירויות המשפיעות על בול העץ: הנמוכות והמהירות מאוד כמעט אינן מזיזות אותו, והמהירויות האמצעיות מזיזות אותו יותר ויותר, עד למהירות מסויימת שמסיטה אותו באופן מקסימלי ואילו מהירויות מעבר למהירות זו מסיטות אותו פחות ופחות עד שבמהירות מסויימת הכדורים נהפכים למעשה לנייטרלים מבחינת ההשפעה שלהם על הבול שהיא 0.

    הסתייגויות?

    אני נוסע לווגאס לכמה ימים, כך שלא אוכל להגיב. נסו לראות אם יש לי איזו טעות. במידה ולא, אני מאמין שניתן לבנות מודל, מחשבתי ודאי, אך גם בפועל, במעבדה, שיניב גרביטציה ללא חיכוך, אינרציה, והחשוב מכל: גל שיתפשט במהירות זהה בכל מערכת יחוס.

    נא לא להשתולל בהעדרי, ולא לעשות דברים אותם לא הייתי עושה בעצמי!

  770. סטודנט

    “- אני לא מכיר את העבודה של פואנקרה, אבל גם בולצמן צודק. מה שאתה מתאר הוא תהליך הפיך, עבורו השינוי באנטרופיה שווה לאפס. ”

    ברור, אך לפי התאור שלי, יתכן שיש גורם פיזיקלי נוסף בגללו השינוי באנטרופיה איננו 0. התפשטות היקום. לכן, במערכת הצלחת המסתובבת, בזמן סיבובה הולכת צפיפות היקום וקטנה ומכאן שהאנטרופיה גדלה.

    מאיר.
    מה שאתה אומר מעניין. אני רוצה לוודא שהבנתי אותך נכון. ניקח את המשפט האחרון שלך:

    “התמדה היא נפילה חופשית של חלקיק אלמנטרי בתוך שדה הכבידה העצמי שלו”

    שלהבנתי אומר כך: אם כל גוף יוצר משיכה, אז אם ננסה להסיט את הגוף ממרכז המשיכה של עצמו, הוא יתנגד לכך כמו שיתנגד לכל גוף אחר, ומכאן נקבל התמדה. הבינותי?

    יובלי.
    ראשית, רצוי שלא נבלבל קוראים בין הדעות המקובלות והדעות הפרטיות שלנו. עיקרון מאך, אליבא ד’מאך, הוא כפי שהצגתי אותו. התוספת: “תנועת כל המאסות ביקום היא ביחס לחומר האפל.” היא רעיון שלך, וצריך לציין זאת, כדי לא לבלבל קוראים העלולים לחשוב שזו דעת המיינסטרים. עד כמה שידוע לי, לא. אינני אומר שזה לא רעיון מעניין, אך עד שיתבסס, חייבים להתיחס אליו כספקולציה.

    ובעניין ה1-. מה עם i, שורש של 1-? האם גם לו יש שורש?

    ובכלל, מה ההשלכות הפילוסופיות לגבי האמונה ב L1? האם נוכל אולי גם להאמין באל מינוס אחד? זה מאד הגיוני לדעתי, כי בעבר האמנו באלים רבים, צמצמנו לאל אחד, האתאיסטים מאמינים באפס, האם אין זה השלב הלוגי המתבקש, דוקי?

    חוץ מזאת שכזוב איכזבתני כזב. מה ביקשתי סך הכל? שתסביר לי כיצד פועלת האי לוקליות במודלך? זה כל כך הרבה לבקש? מילא יהודה הנעלם, שהבטיח “החיכוך עלי!” ונעלם. אבל אתה, סר גלהאד האבירי, משאיר כך פצועים מדממים בשדה?

  771. יובל,

    כתבת: “ביסוס חישובים על חיבור במקום על חיסור מצריך את הגדרת קיומו של מספר נגדי.”
    שאלתי: איך אתה מבסס חישוב רק על חיסור? תוכל לומר מה לא מובן בשאלה?

    אמרת: “אך אפשר להראות כי האקסיומה האחרת מסובכת יותר.”
    שאלתי: איזו אקסיומה? שוב, תוכל לומר מה לא מובן בשאלה?

    דבר נוסף:
    “(וחילוק. המתמטיקאים נזהרים מן ההיבט הזה שלו).”
    – חילוק מוגדר בעזרת הכפל (כפל במספר ההופכי). ל-0 אין הופכי, לכן חילוק ב-0 אינו מוגדר (לפחות לא במתמטיקה “רגילה”). עוד אפשר לקרוא בבלוג של גדי אלכסנדרוביץ’: http://www.gadial.net/?p=168

  772. ר.ח רפאי.ם
    האנלוגיה שהבאת אינה מקובלת עלי, ולו רק מן הסיבה שביקום הבראשיתי שאני מנסה לבנות לא קיים שום דבר ובוודאי שלא מטענים חשמליים. אפילו האנלוגיה שאני מביא, מן המתמטיקה (והלוגיקה), הינה מוגזמת מאד, אך היא הקטנה ביותר שהצלחתי למצוא.

  773. יובל, ידידנו
    לדעתי לא האין והאין יצרו יש (לפי האנלוגיה שלך)- לדעתי היש והיש יצרו אין.
    ידוע שכל מטען חיובי/שלילי דוחה זה את זה. הכוח שנמצא בין שני מטענים חיוביים יוצר כוח דחייה.
    תחשוב על זה..

  774. יובל,

    0 הוא כן איבר ניטרלי – לחיבור. לכפל, איבר היחידה (“הניטרלי”) הוא 1.

    אגב, מדוע לא ענית על השאלות ששאלתי? (הגבת ב”אני לא מבין מה אתה לא מבין”)

  775. ר.ח, ידידי ורעי,
    במובן מסוים, ה”אין” הבראשיתי שעליו אני מדבר הוא אפס, אך הוא אינו ניטראלי. שים לב כי גם האפס אינו איבר ניטראלי אלא הרסני בפעולות כפל (וחילוק. המתמטיקאים נזהרים מן ההיבט הזה שלו). אני מסתכל על כל היבט אפשרי של “אין”, “שלילה”, “אפס”, “ריק” וכו’, ומנסה להסיק מאלה על ה”אין” הבראשיתי.
    וכן, אתה צודק בשיפוטך. בין היתר אני מבסס את המודל שלי גם על סמנטיקה ומילים שנובעות מהיגיון אנושי ללא ביסוס אמפירי כל שהוא. היות שאיננו מכירים אותו – איננו פוסלים שום אפשרות והכל מובא בחשבון. מדובר בראשיתו של כל דבר. בשלב הזה אין כלום, גם לא מתמטיקה או לוגיקה.
    אגב, המיתולוגיה השומרית מתחילה, כנראה, עם מודל האפס והאינסוף: בראשית היה הזוג אפסו ותיהמת, שהם האפס והתהום האינסופי. הזיווג ביניהם (אפס כפול אינסוף) יצר את היקום ואפסו נעלם. אחר כך באו הסיפורים.

  776. מאיר,
    הניסוח של עקרון מאך אינו מדויק. תנועת כל המאסות ביקום היא ביחס לחומר האפל. התנועה ההדדית בין מאסות היא רק תוצר משני.

  777. יובל ידידי,
    במחשבה נוספת. המקבילות המתמטיות של “יש” ו” שלילה” הן + ו – בהתאמה. איין הוא 0 (ולא מינוס כמו שאתה מציג זאת). אתה יכול לקבל אפס מחיבור שני ישים שווים: פלוס + מינוס אולם מ- 0 בלבד אתה לא יכול לקבל “יש” כך שזה בדיוק הפוך למה שאתה אומר.

  778. יובל,
    האם אתה מבסס את המודל שלך על סמך סמנטיקה ומילים? מילים שנובעות מהגיון אנושי ללא ביסוס אמפירי כל שהוא?
    ובכל זאת בתור תרגיל מחשבתי אני אלך איתך. התשובה היא :”יש איין”. נשמע כמו אוקסימורון אולם גם האיין שלך הוא לא אמיתי אלא מהווה איזה שהוא “יש” כמו קבוצה ריקה במתימטיקה. ברגע שאתה אומר “האיין האינסופי” או “האיין איין עצמו” כלומר שיש לו תכונות או שהוא ביצע פעולה, גם אם זו התאיינות, הפכת אותו ליש עם תכונות ומאפיינים.
    האם אפס הוא “איין” להגדרתך? אני חושב שלא כי ל-0 יש תכונות מאד מוגדרות.
    דרך אגב אם תתחיל את המיתולגיה/מודל במשפט “בראשית היה 0 ” אתה תראה שאתה לא יכול להמשיך כי אין דרך מתימטית ליצור מספר מ-0 בלבד. האיין שלך הוא למעשה מספר שלילי שמהווה ראי לחיוביים ולא באמת – 0.

  779. אחר שישראל הזכיר את עקרון מאך ואת שאלת המקור של תופעת ההתמדה, אני רק שאלה/ות.

    אם עקרון מאך נכון, ולכל המסות ביקום יש את התכונה המפתיעה להתנגד לשינוי במצב התנועה של כל המסות האחרות ביקום, ואם זה נכון שכל המסות ביקום הן בקירוב טוב סכום המסות של החלקיקים האלמנטריים המהווים אותן, אזי המסקנה שלכל חלקיק אלמנטרי בעל מסה יש את התכונה המפתיעה להתנגד לשינוי במצב התנועה של כל המסות האחרות ביקום, היא בלתי נמנעת.

    אם לחלקיק אלמנטרי יש את ההשפעה המפתיעה הזו על כל המסות האחרות ביקום, האם לא סביר להניח שתהיה לו את ההשפעה המפתיעה הזו גם לגבי עצמו, ואף הרבה יותר מלגבי אחרים אם נביא בחשבון שהמרחק שלו מעצמו שואף לאפס (ונזכור את חוק היפוך הריבועים של ניוטון)?

    ובכן, אולי זה לא יהיה סביר להניח כך, אם נניח ששדה הכבידה של חלקיק אלמנטרי הוא ישות בלתי נפרדת מהחלקיק. אבל מדוע שנניח כך?

    האם אתם יכולים להצביע על תצפית כלשהי הסותרת את מי מההנחות הבאות:

    מסתו של חלקיק אלמנטרי מבוזרת בגיאומטריה דמויית מעטפת כדור (חלקיק אלמנטרי איננו נקודה “טהורה”).
    חלקיק אלמנטרי הוא ישות פיזיקלית נפרדת משדה הכבידה שלו.
    חלקיק אלמנטרי מחולל את שדה הכבידה שלו במחזורים (כבידה קוונטית).
    חלקיק אלמנטרי נוטה ליפול לתוך שדה הכבידה העצמי אותו הוא חולל במחזור אחד, אם הוסט ממקומו בטרם יצר את כבידתו העצמית במחזור הבא.
    תנועת חלקיק אלמנטרי היא אבסולוטית ביחס לסכום שדות הכבידה הפועלים עליו, ובראשם כבידתו העצמית.
    “התמדה היא נפילה חופשית של חלקיק אלמנטרי בתוך שדה הכבידה העצמי שלו”
    ?

  780. ר.ח רפאי.ם,

    תחום אנרגיית הואקום והחלקיקים הוירטואליים קשור לקוונטים ברמה מאוד גבוהה – אני לא מכיר את התחום הזה יותר ממך.

    יובל,

    לא ברור למה לא הבנת מה לא הבנתי – שאלתי שאלות ספציפיות.

  781. ר.ח,
    הדוגמאות שהבאת אינן מוצלחות. בדוגמה מן המיתולוגיה המצרית מופיע האל רע די בהתחלת הסיפור, ובמיתולגיה הסינית מופיע “פּאן גוּ, שהיה חכם יותר מהארץ ואלוהי יותר מהשמיים”.
    אתה שואל על סבירות ועדויות המבוססות על תצפיות; את אלה אנו רואים כל הזמן בתכונותיה של השלילה. נסיונות המסוגלים להפריך אותו: נסה ליצור “אין” רק מתוך “יש” (ללא שימוש במילים שיש להן משמעות של שלילה) או, מאידך, נסה לשלול שלילה באופן שלא יתן לך חיוב. אם יש גם לך רעיונות, אנא שתף.

  782. יובל,
    אתה אומר “ההבדל בין המודל שלי לבין תאור הבריאה במיתולוגיות השונות הוא רק בשאלת קיומו של בורא תבוני. ב”מיתולוגיה” שלי אין כזה.”

    להלן מיתולוגיות נוספות ללא בורא תבוני :
    http://www.planetnana.co.il/myth16/Egypt/egypt_creation.htm
    http://www.planetnana.co.il/myth169/chinese_creation.htm

    האם המודל שלך נבדל מהם? אם כן אז במה? בסבירות? בעדויות המבוססות על תצפיות ? בנסיונות המסוגלים להפריך אותו?

  783. סטודנט
    לא רוצה לבאס אותך או משהו, אבל אני לא כזה מבין גדול בפיזיקה ומתמטיקה. המונחים אומנם מוכרים לי, ואני לומד אותם, אבל עדיין חסר לי לא מעט ידע בנושאים האלו, לכן וודאי כבר שמת לב שהרבה אני לא אוכל לספר לך.
    כשדיברת על “שדה” התכוונת למבנה אלגברי. אני חשבתי בכיוון מכניקת הקוונטים. אבל אז הבאת את הקישור עם הפרופסור, והבנתי שאין לי מה להוסיף על דבריו. הוא בעצם כבר אמר את מה שרציתי לכתוב. בכל אופן, הנה בניסוח שלי את הדברים: אם ניקח את השדה ונגדיר את השדה כוואקום – אז אוטומטית הוואקום מקבל משמעות שונה מהמשמעות המקורית. כלומר השדה המקורי – לפני שהשדה קיבל הגדרה – הוא שדה נייטרלי. אפשר גם להגיד שדה וירטואלי, אין לו גודל פיזיקלי ממשי. בעצם התערבות וההגדרה של השדה- השדה מקבל משמעות שונה. זאת אומרת שהשדה הופך להיות שדה כלשהו שהוא לא השדה המקורי. כבר כאן נוצרים שני מושגים לשדה אחד, והם שונים בתכליתם. מכיון ששדה וירטואלי לא קיים באמת בטבע, אבל שדה שמוגדר כוואקום כן יכול להתקיים בטבע, מכאן שהאנרגיה היוצרת את הוואקום נוצרה מאנרגיה שהגיע משדה אחר.
    בקיצור, אם אנרגיית הואקום נוצרה מאנרגיה אחרת, היא אינה יכולה להיות שווה לאפס. (אם תשאל אותי איך הגעתי למסקנה הזאת אז אני אענה לך מאוחר יותר כי אני יהיה די עסוק בזמן הקרוב).

  784. מעניין שכל אחד מביא פה ראיות מאיך שמתיחסים לניוטון או לאיינשטיין או לבולצמן….

    🙂

  785. יובל,

    “הגדרת חיסור כחיבור “מספר נגדי” זו סמנטיקה מיותרת.”
    – מיותר זה בטוח לא.

    “ביסוס חישובים על חיבור במקום על חיסור מצריך את הגדרת קיומו של מספר נגדי.”
    – איך אתה מבסס חישוב רק על חיסור?

    “ומדוע מינוס מינוס נותן פלוס? זוהי בדיוק השאלה שתישאר בלתי פתורה כאשר כל שאר השאלות ייפתרו. נכון לעכשיו אנו מקבלים זאת כאקסיומה או כמשפט הנובע מאקסיומה אחרת. אך אפשר להראות כי האקסיומה האחרת מסובכת יותר.”
    – זו שאלה בלתי פתורה רק בעולם שבוא אתה לא מקבל את הנחות היסוד באלגברה.
    איזה אקסיומה אחרת מסובכת יותר?

    בשורה התחתונה, אני לא מבין מה אתה מנסה להגיד ואיפה אתה רואה בעיה בהגדרות הקיימות.

  786. סטודנט, טכניון,
    תודה על הערתך.
    אכן, אתה צודק בזה שאפשר להגדיר את החיסור באמצעות החיבור. אך זה מצריך את המצאת המספר הנגדי.

  787. ר.ח,
    ההבדל בין המודל שלי לבין תאור הבריאה במיתולוגיות השונות הוא רק בשאלת קיומו של בורא תבוני. ב”מיתולוגיה” שלי אין כזה.
    השאלות שהעלית על אנרגיה, ועל פיסיקה בכלל, אינן רלוונטיות בשלב הזה של המודל, אך הן אמורות להיות מוגדרות ומוסברות על פיו.

  788. חיבור/חיסור
    הגדרת חיסור כחיבור “מספר נגדי” זו סמנטיקה מיותרת. ביסוס חישובים על חיבור במקום על חיסור מצריך את הגדרת קיומו של מספר נגדי.
    ומדוע מינוס מינוס נותן פלוס? זוהי בדיוק השאלה שתישאר בלתי פתורה כאשר כל שאר השאלות ייפתרו. נכון לעכשיו אנו מקבלים זאת כאקסיומה או כמשפט הנובע מאקסיומה אחרת. אך אפשר להראות כי האקסיומה האחרת מסובכת יותר.

  789. ישראל,

    “אני מדבר גם על אנטרופיה וגם על זמן.”
    – אז חשוב לדעת ולהבין את ההגדרות.

    “כדי לראות זאת, חשוב על צלחת הסובבת על צירה בחלל במשך שנים. מבחינת פואנקרה, כל פעם שהצלחת השלימה סיבוב שלם, היא חזרה לאותו המצב בדיוק כמו קודם. אם נזניח את השינויים האפסיים המתרחשים בצלחת עצמה, פואנקרה צודק. המצב זהה לחלוטין בסיבוב 542 למצב בסיבוב 37, למרות שאין ספק ש542 קרה בזמן מאוחר יותר.”
    – אני לא מכיר את העבודה של פואנקרה, אבל גם בולצמן צודק. מה שאתה מתאר הוא תהליך הפיך, עבורו השינוי באנטרופיה שווה לאפס. אפשר להקביל את הדוגמה שלך למערכת של גז אידיאלי שנמצא בש”מ במיכל סגור ומבודד, ולאמירה שאין לנו דרך לדעת אם כשחלקיק מס’ 1 היה בנקודה מס’ 1 הוא נקודה מאוחרת יותר בזמן מאשר כשחלקיק מס’ 1 היה בנקודה 2. נו, זה ברור, זה צבר מיקרוקנוני, עבורו האנטרופיה קבועה.

    “פואנקרה צודק – לפי הידוע לו בסוף המאה ה19. לכן הוא גם גמר על בולצמן, שטען שהאנטרופיה אינה הפיכה בזמן סופי.”
    – לא נכון, לפחות לא עבור הדוגמה שנתת כאן. תקרא שוב מה טען בולצמן ומה הוא הוכיח (אפילו בערך שהבאת מויקיפדיה).

    בנוגע לחלקיקים דמויי לה סאז’ – אני לא מכיר, אז לא אכנס לזה.

    ר.ח רפאי.ם,

    “או במילים אחרות, אין דבר כזה כלום, או וואקום וכו’..”
    – ואקום כפי שאנחנו מבינים אותו, כלום מוחלט, כנראה לא קיים. פרופ’ דיוויד גרוס מזכיר את זה בהרצאה שלו בטכניון (דקה ~25): http://www.youtube.com/user/Technion#p/c/4/iNmZqH01nX0 (הרצאה מומלצת באופן כללי)

  790. סטודנט
    https://www.hayadan.org.il/astronomers-reach-new-frontiers-of-dark-matter-130112/#comment-326233

    הדלקת אצלי איזה רעיון, אני אפרט עליו כאן יותר מאוחר. בגדול אני יכול להגיד לך עכשיו שהגעתי למסקנה ש ‘אין דבר כזה שאין דבר כזה’- כלומר ה’כלום’ זה גם ‘משהו’ ,רק משהו אחר. או במילים אחרות, אין דבר כזה כלום, או וואקום וכו’.. 🙂

  791. יהודה
    מאחורי כל הגדרה כזאת בפיזיקה יש פילוסופיה שלמה.
    מה זה מספר שלילי?
    למספר אי-שלילי יש ייצוג בעולם פיזיקלי (למשל “1” יכול להיות מוצג כ- תפוח אחד).
    למספר שלילי אין ערך פיזיקלי אמיתי, מה שיש זאת המשמעות של הערך.

  792. ר.ח.
    בקשר לגליליאו – די ברור, לא? מה ההבדל בין מנוחה לתנועה במהירות קבועה? תנועה – יחסית למה?
    המערכת המוחלטת היחידה הקיימת היא מערכת ה CMBR ואכן אפשר להגדיר תנועה יחסית אליה. (המהירות שלנו – כ370 קמ/שנ לכיוון קונסטלצית ליאו, אם אני זוכר נכון). אבל חוץ מזה, במיוחד בזמן גליליאו ואפילו איינשטיין, כל התנועות האינרציאליות שקולות למנוחה, ורק תנועה יחסית שווה משהו.
    וכמובן תאוצה. זו מוחלטת.

  793. סטודנט.
    אני מדבר גם על אנטרופיה וגם על זמן.
    כדי לראות זאת, חשוב על צלחת הסובבת על צירה בחלל במשך שנים. מבחינת פואנקרה, כל פעם שהצלחת השלימה סיבוב שלם, היא חזרה לאותו המצב בדיוק כמו קודם. אם נזניח את השינויים האפסיים המתרחשים בצלחת עצמה, פואנקרה צודק. המצב זהה לחלוטין בסיבוב 542 למצב בסיבוב 37, למרות שאין ספק ש542 קרה בזמן מאוחר יותר.

    פואנקרה צודק – לפי הידוע לו בסוף המאה ה19. לכן הוא גם גמר על בולצמן, שטען שהאנטרופיה אינה הפיכה בזמן סופי.

    אני טוען שיתכן שקיימת אפשרות נוספת: אם נאמר שסיבת הכבידה והאינרציה הם לדוגמא חלקיקים דמויי חלקיקי לה סאז’, והמערכת מתפשטת כמו בתאוריית המפץ הגדול, אז בכל סיבוב הצלחת חוזרת למצב שונה מאשר בסיבוב הקודם, מצב שבו יש פחות חלקיקים ליחידת נפח.

    לכן קיבלנו הגדרה מדוייקת לכל מצב זמן בהיסטורית היקום: בכל שלב ניתן לדעת בכל מקום ביקום מהו הזמן המדויק, המוגדר כמספר חלקיקים ליחידת נפח. טמפרטורת היקום הינה פונקציה ישירה של זמן זה, לפי נוסחת פרידמן, והיא, כמו הזמן, מעולם לא חוזרים על עצמם בנקודה מסויימת ביקום.

    יהיה מעניין לנסות לראות אם הנגזרת השניה של המרחק ביחס לזמן לפי הגדרה זו, התאוצה, אכן נותנת לנו את החוק השני של ניוטון במודל הידרודינמי. כשיהי לי יותר זמן אשתעשע בזאת, נשמע כמו פאן.

    ר.ח.

    נו טוף..
    אז מדדנו כוח צנטריפוגלי בתוך הצלחות המעופפות, כאשר החלונות בהן סגורים. נגיד אפילו 10 מהן המסתובבות יחסית זו לזו, והגענו למסקנה שרק מס 3 באמת נמצאת במנוחה. כל השאר מסתובבות, חלק עם כיוון השעון חלק נגד, ואנחנו יודעים אפילו מה המהירות שבה הן מסתובבות, וכל זאת רק עיי מדידה פשוטה בתוך הצלחות עם מד כוח פשוט!

    אבל השאלה הגדולה היא – מסתובבות יחסית למה?

    זו לזו? הרי אם נוציא 9 מתוכן מהמערכת, העשירית תמדוד אותו כוח בדיוק כמו קודם, וזאת בחלל ריק לחלוטין!

    ההפתעה הגדולה מצפה לנו כשנפתח את החלונות.

    כי מסתבר, שרק בחללית שמדדה כוח 0, הכוכבים הרחוקים בחוץ יראו כאילו הם עומדים. בכל שאר הצלחות, הכוכבים יראו כאילו הם חגים, ובדיוק באותה המהירות אך בכיוון ההפוך למהירות שמדדנו בצלחת.

    זהו עיקרון מאך. האינרציה נגרמת בגלל כל המסות ביקום.

    לאיינשטיין יש תאור שונה. לשיטתו, הגרביטציה והאינרציה שקולות לחלוטין. לכן הכוח שחשים הנוסעים בצלחות המסתובבות יקרא עיי איינשטיין כוח גרביטציוני ולא צנטריפוגלי. קצת מוזר אמנם בכיווניו, אך עדיין כוח.

    בתנועה בקו ישר, הבעיה עם תאוצה היא בדיוק אותה הדבר: תאוצה יחסית למה? אם מכונית מאיצה יחסית לכביש, מדוע לא נוכל לאמר שהכביש הוא זה שמאיץ יחסית למכונית? שיידבק הוא לכיסא, לא אנחנו.

    ועוד דבר: אין מדובר בכוח זניח, אלא כוח עצום: קח מסור חשמלי מסתובב ותראה איזה כוח הוא יפעיל עליך, אם תנסה להטות אותו לצדדים. אם המסור מהיר וכבד מספיק, הוא יפיל אותך על הרצפה אם תנסה לטלטל אותו.

    מה יש שם, שמתנגד לנו כל כך חזק?

    לפי ה”מודל” שלי, יתכן מאוד שאילו חלקיקים דמויי חלקיקי לסאז’. כפי שהראתי, במהירות קבועה המערכת נמצאת בשיווי משקל. בזמן תאוצה, פועל כוח, כמו שאויר מתנגד למפרש הנע דרכו. לא הוכחתי, אך אם אוכל לבסס מתמטית את מושג הזמן כצפיפות חלקיקים, נוכל לתאר תאוצה כנגזרת שניה של מרחק ביחס לזמן, ונראה מה נקבל.
    אך את זה אני משאיר בינתיים לך.

    יהודה צודק. המטען ה”שלילי” של האלקטרון הוא פשוט תוצאה של הטלת מטבע לא מוצלחת. עדיף היה לקרוא לו חיובי, כי עודף אלקטרונים באטום אמור להפוך אותו לחיובי יותר. אך זה רק עניין של סימון.

  794. ר.ח.
    אמרת :- “העובדות מראות לנו שהמטען החשמלי של האלקטרון נקבע כמינוס. מדוע הוא נקבע כמינוס ולא כפלוס?”. סוף ציטוט.
    לדעתי כל מה שיש לנו כאן אלה שני חלקיקים שהחלטת לתת לאחד את הפלוס ולאחד את המינוס. אין תכונה מיוחדת לאלקטרון שתחייב דווקא לו לתת את הסימן מינוס. שים לב שדווקא מסת הפרוטון שמטענו פלוס, נמוכה יותר ממסת הניטרון שמטענו אפס, ואולי היה הגיוני יותר לתת דווקא לפרוטון את הסימן מינוס ..
    סתם חומר למחשבה של תחילת שבוע.
    יום טוב
    סבדרמיש יהודה

  795. ישראל,

    בקשר לשאלה ביחס למה הכח הצנטרפוגלי אנא קרא :

    http://he.wikipedia.org/wiki/מערכת_ייחוס

    במיוחד את הפיסקה על עיקרון היחסות של גלילאו וכח מדומה ותכתוב מה דעתך.
    ותרחיב , תרחיב, תרגיש חופשי לא צריך לשאול כל פעם, קבל OK אוטומטי מראש.

    יובל,
    להגיד בראשית היה אין זה לא תאוריה אלא מיסטיקה. מה טיבו של האין? האם הוא קיים היום ? האם ניתן לעשות ניסוי (אפילו מחשבתי ) שייבדוק את טיבו? מה פירוש מאיין את עצמו? מאיזה אנרגיה? אם הוא איין אמיתי לא תהיה בו גם אנרגיה ואם יש בו אנרגיה אז איזה מין איין זה?
    בקיצור שאלות על גבי שאלות בלי בדל הוכחה חוץ מסיפור יפה. מה הבדל העקרוני בין הסיפור שלך לבראשית ברא ה’ את השמיים… או כל שאר הסיפורים המופיעים פה: http://he.wikipedia.org/wiki/בריאת_העולם?
    האם העובדה שקראת לשלך “מודל” ולא מיתולוגיה הופכת אותו לשונה?

  796. ישראל,

    “ברור שהאנטרופיה אינה הפיכה בזמן במערכות בסדר גודל של מספר אבוגדרו ומעלה.”
    – גם במערכות עם מס’ חלקיקים מסדר גודל הרבה יותר קטן.

    “אבל עדיין קיימת השאלה של הפיכות חוקי ניוטון בזמן, כפי שהראה פואנקרה, תותח על מתמטי.
    ההצעה שלי היא לקשור כל רגע מסוים בזמן עם מצב מוגדר של מספר חלקיקים ליחידת נפח, כפי שניתן לעשות עם בלון מתנפח.”
    – אז אתה לא מדבר על אנטרופיה, אתה מדבר על הפיכות חוקי ניוטון בזמן. האנטרופיה מוגדרת במכניקה הסטטיסטית, היא תכונה סטטיסטית של מערכת.
    את מה שהצעת ולאן אתה מנסה להתקדם עם זה לא הבנתי.

  797. ר.ח רפאי.ם,

    “…צצה השאלה “מאין הגיע ה (a-) ?”
    – מהשדה. האיבר הנגדי בחיבור המסומן בתור (-a) הוא איבר בשדה, שאם תחבר אותו אל a תקבל את האיבר הניטרלי (“אדיש חיבורי”). המינוס לפניו הוא רק סימון, יכולנו לסמן אותו אחרת. האיבר הניטרלי הוא גם איבר בשדה, שאם תחבר אותו אל a תקבל את a. הקיום של איבר נגדי בחיבור מאפשר להגדיר את פעולת החיסור באופן שאתה מכיר.

    “כלומר אתה חייב להניח ‘משהו’ כדי להגדיר את ה”לא כלום”. ואז ה’לא כלום’ הופך להיות ‘משהו’.”
    – אני חושב שכן. אתה חייב להניח הנחות יסוד כלשהן כדי להגדיר.

  798. סטודנט.
    מאמר נחמד, תודה.
    ברור שהאנטרופיה אינה הפיכה בזמן במערכות בסדר גודל של מספר אבוגדרו ומעלה. את זה כבר סגרנו מזמן. אבל עדיין קיימת השאלה של הפיכות חוקי ניוטון בזמן, כפי שהראה פואנקרה, תותח על מתמטי.
    ההצעה שלי היא לקשור כל רגע מסוים בזמן עם מצב מוגדר של מספר חלקיקים ליחידת נפח, כפי שניתן לעשות עם בלון מתנפח.

    ר.ח.

    הבנת נכון.
    אבל הצינור המעשן לא יעזור. כל חללית במערכת שלה נמצאת במנוחה, אז חיכוך עם מה? זהו בדיוק הפוסטולט הראשון של היחסות: אי אפשר לדעת בשום צורה מי נע ומי נח.

    אולם שים לב לכך שבדוגמת הצלחות המעופפות המצב שונה. אפשר לקבוע בוודאות איזו צלחת נחה ואיזו מסתובבת, באיזו מהירות, ולאיזה כיוון. ומכאן השאלה: מסתובבת יחסית למה?
    זהו הדלי המסתובב של ניוטון, ומכאן עיקרון מאך ועקרון האקווילנטיות ביחסית הכללית. אם תרצה, נרחיב.

    אין לי יומרות לתיאוריית על חדשה. להיפך, אני כמעט בטוח שיש לי איזו טעות בסיסית בהבנת הסתירה כביכול בתאוריות הקיימות. אולם עד כה אף אחד לא הצליח לשכנע אותי, מה גם שכל אחד אומר משהו שונה, ובכל גירסה קיימות סתירות בסיסיות מובנות.

    כמו תמיד, אם תרצה, נרחיב.

    רק שים לב כמה יפה פותר הרעיון שלי את בעיית קביעות מהירות האור בכל מערכות היחוס, החיכוך אצל ל.ס., ומאפשר אינרציה ואי לוקליות.

    כמו תמיד, אם תרצה …. כאילו הציל עולם ומלואו!
    אוי התבלבלתי באימרה.

    רפאים.
    חלש.
    אין לי בעיה עם כסאח, רק עם רמת כתיבה נמוכה.
    חבל שנבזבז זמנינו על התנצחות חסרת טעם. הצעתי לך לקרוא את התגובות שלי ולשאול שאלות. אם אינך מעוניין, או אינך מסוגל, אז יאללה, תגיד את המילה האחרונה ותשחרר.

    יובלי
    5.5
    קיוינו לקבל את התאורייה המלאה, או לפחות את עניין האי לוקליות. איפה?

  799. סטודנט
    קודם כל תודה על הקישור. מומלץ לכולם.
    מכאל ר. ענה תשובה דומה לשלך, אלא שאז צצה השאלה “מאין הגיע ה (a-) ?
    כלומר אתה חייב להניח ‘משהו’ כדי להגדיר את ה”לא כלום”. ואז ה’לא כלום’ הופך להיות ‘משהו’.
    סליחה על ההסבר הלקוני מקווה שהבנת למה התכוונתי.

  800. יהודה
    אבל העובדות מראות לנו שהמטען החשמלי של האלקטרון נקבע כמינוס. מדוע הוא נקבע כמינוס ולא כפלוס?

  801. ר.ח
    האם לא יהיה יותר נכון לומר שהאלקטרון הוא ההפך מפרוטון. תן לאחד מינוס והשני יקבל פלוס ולא חשוב מה למי. היינו יכולים להחליט שסימנו של האלקטרון הוא פלוס ואז הפרוטון היה מקבל את המינוס.
    כך שהשאלה שצריכה להשאל:- מה מהות ההפכים שמקבלים האלקטרון והפרוטון. או, מה משמעותו של המושג “מטען חשמלי”?
    ויובל
    לא מקבל את נכונות פעולות החיבור שלך
    שבוע טוב
    סבדרמיש יהודה

  802. יובל,

    “…ולמעשה ניתן להגדיר את החיבור בעזרת החיסור (אך לא להיפך).”
    אני דווקא מכיר את הגדרת החיסור הנובעת מקיום איבר נגדי, כלומר החיסור מוגדר בעזרת החיבור. איפה ראית הגדרה של החיבור בעזרת חיסור ולמה אתה טוען שלא ניתן ההפך?

  803. בנוסף, לדעתי, הדרך הנכונה לענות על זה תהיה דרך ההבנה מדוע האלקטרון הוא מינוס.

  804. יובל
    בוא נתחיל מהתחלה:

    מדוע – ועוד – שווה +?
    (שאלתי את מכאל רוטשילד לא מזמן את אותה שאלה).

  805. ר.ח רפאי.ם,
    הרי דוגמה לרעיון פילוסופי:
    ראשית הכל – ה”אין”, השלילה. אין דבר חוץ מן ה”אין” ולכן אין לשלילה דבר לשלול חוץ מאשר את עצמה. השלילה שוללת את עצמה ושוללת ושוללת ושוללת בלי הגבלה, לאינסוף, שהרי זוהי מהותה ומשמעותה היחידה. וכך מן ה”לא” נוצר ה”לא-לא”, שמכונה בפינו בקיצור “כן”, וה”לא-לא-לא”, שבפשטות מכונה “לא”, וכו’. וכך מן ה”אין” נוצר ה”אין-אין”, שמכונה בפינו “יש”, וה”אין-אין-אין” שאנו, באינטואיציה שלנו, חושבים אותו ל”אין” ומדמים שאין הבדל בין “אין” יחיד ל”אין” משולש או מחומש או משובע וכו’. השימוש במספרים כאן הוא בהשאלה מן האריתמטיקה: לשם קיצור אנו קוראים ל”מינוס-מינוס” בשם “פלוס” וכך אנו מקבלים כאילו בצד פעולת החיסור קיימת גם פעולת החיבור. “חיבור” הוא שם קיצור ל”פעמיים חיסור”, ומשום חוסר גיוון באריתמטיקה הוא גם קיצור ל”ארבע פעמים חיסור”, “שש פעמים חיסור” וכן הלאה, ולמעשה ניתן להגדיר את החיבור בעזרת החיסור (אך לא להיפך). בדרך זו ה”יש” הוא שם קיצור לצרופים כלשהם של “אין”, ולמעשה ניתן להגדיר את כל ה”יש” באמצעות ה”אין” לבדו.

  806. אגב, יובל

    “בראשית לא היה דבר, אין; האין הוא שלילה אקטיבית והוא איין את עצמו ליש”” – איך?
    (אשמח לתשובה מפורטת בשביל שיהיה מובן ויהיה על מה לדון)

  807. ר.ח, אין סכנה לשלימות כובעך 🙂
    סבדרמיש יהודה, אין סיכוי שאקבל פרס נובל. זה מחולק רק לאנשים חיים וניתן כמה עשרות שנים לאחר התגלית שזיכתה אותם. נניח שנפרסם בשנה הקרובה ואף נזכה להוקרה רבה, עד שיגיע תורנו נהיה בני 90 שנה לפחות. מה הסבירות שנחיה עד אז? 🙁

  808. ר.ח רפאי.ם,
    כן. לעולם לא נגיע אל האופק, אך אנו בדרך הנכונה אליו. זה השילוב המנצח של תקוות אשליה עם חשיבה ביקורתית שהביאנו עד הלום וימשיך להוליכנו הלאה.

  809. יובל
    נכון שהאינטלקטואלים מנסים להתקרב אל התאוריה הזאת. אבל לדעתי, תאוריה כזאת היא אשליה.
    והאשליה הזאת היא ה’מדרבן’ את האינטלקטואלים לחתור אליה.
    לולא התקווה (ויש שיגידו אמונה) לא היינו מגיעים לאן שהגענו.

  810. ר. ח. יקירי
    דמעות אושר לחו את עיני. הנה, הנה. יש אחד המאמין בצדקת דרכנו אל הפרס (הפרס בהי הידיעה)
    ולגבי הדברים בתגובתך:- הבדיחה מוכרת לי מהערב ומצאה חן בעיניי מאוד. היא תסופר מידי פעם בהרצאותיי.
    יש לדייק ואכן המתמטיקאי היה הצודק, כי רק דברים מוכחים יכולים להתקבל כנכונים:- יש שדה אחד לפחות בסקוטלנד ובו לפחות כבשה אחת שצידה האחד לפחות שחור.
    הפיזיקאי והביאולוג טעו כי יתכן והכבשה אינה שחורה אלא מצידה האחד.
    ולגבי השתנות מהירות האור קבל את נוסחת ההשתנות שלי כתרומה לבדיחתך המקסימה , ובכן:-
    מהירות האור פרופורציונאלית לשורש טמפרטורת הרקע של היקום.
    סתם, מסקנות מיקום גאזי פשוט.
    ט.ל.ח.
    שבוע טוב
    סבדרמיש יהודה

  811. ר.ח רפאי.ם, בעניין הפקפוק אם תאוריה של הכל תיתכן אי פעם.
    אני נוטה להסכים איתך. ואף על פי כן, השאיפה האיטלקטואלית, לפחות של כמה מאיתנו, היא להתקרב לתיאוריה כזאת עד כמה שניתן. כך זה, למשל, עם סיפור המפץ הגדול ועם תורת המיתרים. התיאוריה שלי מתחילה (בפשטנות רבה) בערך כך: “בראשית לא היה דבר, אין; האין הוא שלילה אקטיבית והוא איין את עצמו ליש”. גם אם מזה נגיע לאיזשהו מודל הכולל את כל הפיסיקה, עדיין תישארנה פתוחות שאלות כגון “מה יש בשלילה שעושה אותה אקטיבית?”. אולם השאלות האלה אינן רבות, ומשום כך תיאוריה כזאת מתקרבת מאד אל היעד הנכסף של “תיאוריה של הכל”.

  812. יהודה,
    קודם כל אין לי מטרות אפלות. אני כאן על תקן של מבקר קולנוע. בניגוד אליך לישראל וליובל קטונטי מלפתור את בעיית הבעיות בפיזיקה הנוכחית ולמצוא תאוריה שתסביר הכל בצורה שונה. למרות שנראה לי ששלושתיכם טועים אני באמת שמאחל לכם הצלחה ואני מבטיח להגיע לשטוקהולם לטקס ולהגיד לחתן הפרס: כל הכבוד!! מודה אני שלא האמנתי שתגיע עד הלום ובכל זאת עשית זאת! ואז אוריד את הכובע, אקוד קידה נאה ואוכל בהנאה את כובעי (דרך אגב הכובע כבר מוכן אז כל מה שנותר לכם הוא רק לכתוב את המשוואות או למדוד מהירויות משתנות של האור או כל דבר אחר שישכנע בצידקת דבריכם).

    לעצם העניין יהודה, כ-ל התצפיות כמו אלה של מייקלסון מורלי וכ-ל ניסיונות מעבדה הראו שכאן באזור מערכת השמש בזמן הזה של 15 מיליארד שנה למפץ הגדול מהירות האור בריק היא 300 אלף ק”מ בשניה ללא תלות יחסותית. כמובן שאפשר להגיד כן, אבל לפני 100 שנה זה היה אחרת או כן, באנדרומדה זה אחרת אבל אז אוקהם יתהפך בקיברו ואת זה אנחנו ממש לא רוצים, לא?

    בנושא זה אצטט בדיחה עם זקן :
    ביולוג, פיזיקאי ומתמטיקאי נוסעים לאירלנד, ובנסיעה ברכבת בתוכה הם רואים כבשה שחורה בשדה.
    “מעניין” , אמר הביולוג, ” כל הכבשים כאן שחורות.”
    “לא בדיוק”, השיב הפיזיקאי, “לפחות חלק מהכבשים כאן שחורות”.
    והמתמטיקאי עונה: “בארץ זו יש לפחות שדה אחד, בו לפחות כבשה אחת, שלפחות אחד מצדדיה שחור”.

  813. שרוליק (שפירא)

    שנינותך פשוט נוטפת ממך. איך אתה לא נמוג ברקע?
    אתה הצבת מספר שאלות. דורש תשובה עליהם. אומר שלך יש רעיון לפתרונות. אבל לא מראה שום פתרון. אתה מבולבל? גם אני ועוד כמה כאן. אם אתה לא יודע להסביר את עצמך בבירור זה סימן שאתה בעצמך לא מבין מה שאתה רוצה להסביר.
    מה קרה? התעוררת בבוקר עם הפרשי לחץ תוך גולגלתיים, הרגשת כבידה חזקה בבטן וכשהסתכלת הצידה ראית את כלאבי עם גרב על האוזן, ובקבוק ריק של בלו לייבל דבוקה לכף שלו ולא הצלחת להחליט מי מבין שניכם מסתובב?

  814. יובל
    אני לא חושב שתאוריה של הכל תיתכן אי פעם. הכוונה שלי היתה שתאוריה ברמה של תאוריה של הכל, או ברמה של תורת היחסות או תורה של ניוטון, למשל, תביא איתה תשובות ושאלות פילוסופיות חדשות. (בכל אופן, דעתי די דומה לדברים שכתבת בתגובתך אלי)

  815. ר.ח
    יש לי הרגשה שאתה “מכופף” את הדברים לכיוון דעתך,
    למה כוונתי:-
    אתה מניח שמהירות האור הטבעית היא להיות קבועה?
    אתה זוכר מה קרה לא מזמן בתקופת היקום האינפלציוני , ממש כמה חלקיקי שנייה לאחר המפץ הגדול?
    החליטו להניח התפשטות יקום אינפלציונית במקום לשנות את מהירות האור לגבוהה יותר. ממש מגוחך!
    אתה גם מנסה לשמור על הקביעות של האור כדי שמה?,,, שלא תהיינה בעיות קוונטיות?, זאת גישה שאם לא הייתי מעריך אותך הייתי קורא לה מגוחכת.
    ולסיום אתה משתף את התער של אוקהם להשגת מטרותך האפלות לגבי קביעות האור
    “תער אוקהם שאומר שכרגע אין לנו שום סיבה להניח שמהירות האור שונה באיזה שהוא מקום אחר.”. סוף ציטוט. מהכרותי הקרובה את וויליאם הנזיר מאוקהם שעל שמו התער, הנ”ל היה קמצן וסגפן בצורה זוועתית. אני בטוח שאם הוא היה חי היום הוא היה משתדל להאט את מהירות האור.
    ערב טוב
    סבדרמיש יהודה

  816. ישראל,
    אני חושב שהבנתי. בדוגמא השנייה אם אחת החלליות מסתובבת והשניה לא, אזי במסתובבת יהיה כח צנטרפוגלי שיזרוק דברים לצדדים ובשניה לא, האם אני צודק?

    לגבי הדוגמא הראשונה, נניח שכל חללית תשים צינור מעשן שיוצא מהחללית, האם לא ניתן יהיה לדעת מי אכן נעה? האם העשן ימשיך לנוע איתה? אני חושב שמספיק שיהיה חיכוך קטן שבקטנים כדי שהעשן יגיד בסופו של דבר מי נעה ומי נחה ביחס למרחב. אם נחזור שוב לימאות,זו הדרך שבה מדדו בעבר מהירות אוניה ביחס לזרם המקיף אותה, ע”י זריקת חבל עם קשרים למים וספירת מהירות יציאת הקשרים מהאוניה (ומכאן יחידת המהירות קשר).

  817. יהודה,

    קיומה של תאוריה “של הכל: אינו מוכח כמובן אולם ישנם שיקולים שתומכים בה. בינהם ניתן למנות שיקולים אסטטיים שאולי נשמעים כלא קשורים אולם הוכיחו את עצמם עד היום, למשל בתורות החלקיקים : טבלה מחזורית, המודל הסטנדרטי, סימטריות בטבע ועוד ועוד.
    דבר שני בהנחה שקבועים פיזיקליים משתנים עם הזמן או במרחב היא בעייתית מאד.
    נניח שבמרחב האור מתנהג בצורה משתנה, היכן הגבול? האם הגבול הוא קוונטי (כלומר מהירות האור קופצת מ – X לY) בנקודה מסויימת ואם כן אז מה מיוחד בנקודה זו? אולי המעבר הוא הדרגתי, אם כך מה קובע את הגרדיאנט? כנ”ל לגבי כל קבוע פיזיקלי אחר. נקודה אחרונה היא תער אוקהם שאומר שכרגע אין לנו שום סיבה להניח שמהירות האור שונה באיזה שהוא מקום אחר.

  818. ישראל,

    כתוב שם (למרות שזה תומך בטענה שלי, אמרתי לך כבר שויקיפדיה זה לא מקור אמין):
    “בולצמן הוכיח מתמטית תוך שימוש בשיטות פיזיקליות כי מצב פיזיקלי של מערכת סגורה יכול לחזור על עצמו רק אם האנטרופיה איננה גדלה. האנטרופיה לפי הוכחתו חייבת לגדול או להשאר קבועה בזמן, אך איננה יכולה לקטון באופן טבעי. האנטרופיה תגדל עבור כל תהליך בלתי-הפיך (ורובם המוחלט של התהליכים הטבעיים כוללים את הגדלת האנטרופיה). ”

    כמו שאמרתי, קיים הסבר לכך שאנטרופיה גדלה עם הזמן ולא ההפך. כבר אמרתי לך בדיון בעבר, בדוגמה פשוטה עם גז אידיאלי, שמתמטית אכן קיימת אפשרות להקטנת האנטרופיה, אך מצב כזה אינו פיזיקלי כי הוא דורש זמן אינסופי (אינסופי שגדול מגיל היקום).

    מאמר מעניין בנושא:
    http://www.scholarpedia.org/article/Time%27s_arrow_and_Boltzmann%27s_entropy

  819. סבדרמיש יהודה היקר גם,
    דיויד יום הינו, על פי מה שאתה מספר, אמפיריציסט. הוא כמובן צודק, במובן הצר שהוא מדבר עליו, ובאמת יש מקומות בהם מקבלים את שיטתו (למשל בבתי משפט). אלא שהאמפיריציזם, נכון ככל שיהיה, לא יקדם אותנו הרבה.
    שבת שלום ומבורך

  820. יובל היקר
    התער של אוקאם בא לבחור בפשוט ביותר מתוך נוחיות. וודאי שאין לעשות זאת מתוך קביעת הנכונות. בוחרים בניוטון לא בגלל שהוא נכון יותר מאיינשטיין- יחסות! כי נוח לעבוד איתו אפילו בגלקסיות.
    נוסחה או תיאוריה נכונה רק היכן שהיא נבדקה. דיוויד יום, הפילוסוף האנגלי, וגם רבים אחרים, אמרו שאם ראית נכונות של אלף מיקרים זה רק אומר מסקנה אחת שהאלף מיקרים שבדקת הם נכונים. המקרה הבא עלול להיות שונה. כל מיני “כללים” כגון התער של אוקאם או עקרונות קוסמולוגיים למיניהם הם סתם נסיונות עלובים של מדענים שמרגישים מוגבלים ביכולתם.
    ודרך אגב, לקחת רק אפשרות אחת מתוך אינסוף ולהשליך מסקנות ממנה על הכל זה מוגזם ביותר
    שבת שלום
    סבדרמיש יהודה

  821. ישראל המתוק,
    לאט לאט אתה מצליח לשכנע אותי כי מדע הפיסיקה הוא בעצם ענף של הליריקה.
    נ.ב. בן כמה תומר היום?

  822. סבדרמיש יהודה,
    ישראל שפירא מביט על מודלים פיסיקליים קיימים ומגלה כי הם סותרים זה את זה. זו בדיוק הסיבה מדוע נחוצה לנו תיאוריה אחת שתסביר הכל.
    כאשר ניוטון קבע כי כל חלקיק של חומר בכל מקום ביקום מתנהג אותו דבר הוא פשוט החיל את תערו של אוקהם. אולי הוא טעה, כהנחתך, אולם אם לא היה עושה זאת הוא היה מניח כי בכל מקום ומקום קיימים חוקי טבע הייחודיים לו. כפי שראינו החל מימי איינשטיין, הפיסיקה של ניוטון אכן אינה מדויקת, אולם היא לא נפסלה אלא “הורחבה”. באותו אופן, גם מהירות האור עשויה להתברר כזקוקה לתוספת נוסחאות.

  823. ר.ח רפאי.ם,
    היות שהפנית את הדברים אלי, אני עונה במקום ר.ח:
    באוניברסיטה בה אני לומד נקרא המקצוע פיסיקה עד לא מזמן בשם “הפילוסופיה של הטבע”. אולי קיבלה המילה פילוסופיה משמעות בלתי מכובדת (יעני, “פאלסף”) או שפשוט קרה שם מה שקורה בכל מקום בו הרעיונות שהתחילו כפילוסופיה מתגבשים לכדי נוסחאות מסודרות.
    כשאני שואל את השאלה שטרם זכתה למענה מסודר, “מה ראשית הדברים?”, הרי זה עדיין בגדר פילוסופיה. אולם כאשר תימצא לה התשובה שתהיה מוסכמת על כלל הקהילה המדעית, היא תקבל את מקומה המכובד והראוי על מדף הפיסיקה.

    וכעת אענה לסבדרמיש

  824. יובלי החמוד.

    כלאבי התחיל את שמו כ”רגאזי” אך מהר מאוד קיצרנו אותו לכלבלב, ובקיצור כלבלבון, ובקיצור נמרץ כלבלבונון, ובקיצור נמרץ ביותר כלבלבונוניניו, ומכאן – כלאבי.

    ככתוב: כלאבי כלאבי בן כלביים, שחום פרוה וחום עיניים.

    ובקשר למיצי, היא אכן מופיעה באחד הבתים בשיר עבור הילד:

    זמן חלף, תינוקינו בגן,
    ושובב הוא גם רב שובב
    וניסינו מילים חדשות ללמדו
    ומעט להרחיב אופקיו.
    העכבר עושה סקוויק סקוויקי,
    הציפור מצייצת צוויץ צוויצי,
    לחתול קוראים לקיקי
    לחתולה אומרים מיצי.

    אז גלגל תומריקו עיניו למרום
    והסביר, אוטו אוטו אוטו
    וסולידה נתנה לו מטרנה לטעום
    ושניהם שרו את המוטו.

    אוטו אוטו אוטו
    אין שני לו ואין כמותו
    והמון תינוקות מריעים
    בוונטורה פינת דה סוטו
    והנאני סולידד, נאנחת בצד,
    קה בוניטו מי פרסיוסוטו.

  825. סטודנט

    http://he.wikipedia.org/wiki/%D7%9C%D7%95%D7%93%D7%95%D7%95%D7%99%D7%92_%D7%91%D7%95%D7%9C%D7%A6%D7%9E%D7%9F
    ר.ח.

    אם ראית את היוטיוב של פיינמן, מודל ל.ס. של גז ללא התנגשויות באמת יכול להוביל לגרביטציה, לדוגמה בין שני גושי פלסטלינה שיספגו חלק מההתנגשויות, ושניתן להראות שהיא פרופורציונית הופכית למרחק בריבוע.

    השאלה היא: האם זה מה שקורה באמת במציאות?

    נאמר ששני גושים כאלו מדמים את השמש והארץ. אם הארץ תנוע סביב השמש, הרי שהיא תתנגש בחלקיקים המשיקים לכיוון הסיבוב שלה, בדיוק כמו שאם תסובב אבן הקשורה לחוט, היא תתנגש באויר סביבה ותיצור חיכוך. זוהי בעית החיכוך אצל ל.ס.

    יהודה, ר.ח. בענין האינרציה וג’ירוסקופ.

    כדי לוודא שאנחנו מדברים על אותו הדבר, אתן לכם דוגמה יומיומית מהחלל.

    נאמר שיש שתי חלליות הנעות במהירות מסויימת וקבועה, ללא תאוצה, זו יחסית לזו. שתיהן מצלמות בוידאו זו את זו, והוידאוס נשלחים לכדור הארץ.

    בכדור הארץ מנתחים את הוידאוס, המראה כצפוי תמונה זהה אך הפוכה: הוידאו של חללית א’ מראה את חללית ב’ מתרחקת במהירות V, והוידאו של ב’ מראה את א’ מתרחקת במהירות V-.

    שאלות: האם יכולה מי מהחלליות לטעון שהיא נחה והשניה נעה? האם ניתן למדוד איזה כוח בתוך החללית שיקבע חד משמעית מי באמת נחה? האם ניתן להסיק כוח זה מצפיה בוידאוס בלבד?

    נחזור כעת על הניסוי, אך במקום חלליות נשתמש בצלחות מעופפות המסתובבות על צירן במהירות זויתית קבועה, אחת מעל השניה במרחק כמה מטרים זו מזו.

    הן שולחות את הוידאוס לכדור הארץ, ושוב נשאלות אותן השאלות: האם אפשר רק מהוידאוס לדעת אם אחת באמת נחה, והאם ניתן למדוד איזה כוח בצלחות שיקבע מי נחה ומי מסתובבת? ואם מסתובבת יחסית למה? ובאיזו מהירות זוויתית בסיבובים לשניה?

    נסו לענות על השאלות, כדי שנוכל להסתנכרן.

    ר.ח רפאי.ם

    אני מציע שנמשיך להתיחס זה אל זה כפי שהתיחסנו עד אתמול כשעלה לך השתן לראש: לא להתייחס כלל. אם תראה קצת יותר דרך ארץ, נוכל לפתוח בדיון. אם תתקוף, תחטוף חזרה.

    אין לי כוח וזמן להתחיל להסביר לך מהתחלה את כל הבעיתיות שקיימת לדעתי במודלים קיימים, ואת הרעיון שלי. הם די מפורטים בתגובות שלי. אם יש לך איזו שאלה קונקרטית, אשמח לענות.

  826. ליובל ר.ח ואחרים
    אתם טועים את הטעות שטעו כל הגדולים. מתלהבים ובקלות משייכים (תיאוריה) על הכל!
    ניוטון למשל גילה את הגרביטציה ומיד החליט שהיא קיימת לכל פרודה שביקום. למה?, הוא הוכיח שהיא קיימת בכל נקודה שביקום? לא! אבל מה איכפת לו להצהיר כך?. ומאז אנו לא מקבלים שזה יכול להיות אחרת.
    עוד דוגמא :- ההצהרה שמהירות האור זהה בכל נקודה ביקום ובכל זמן. מישהו באמת מדד זאת?, הרי בחיים אי אפשר היה להוכיח דבר כזה בנקודת הזמן הבודדת בה נעשה הדבר באיזו שהיא פינה נידחת בשביל החלב, כך וכך שנים לאחר המפץ הגדול.
    למה אני מספר לכם זאת?
    כי הנה אתם מנסים למצוא תיאוריה על הכל! למה?, האם תוכלו להוכיח שתיאוריה שתגלו תהיה נכונה על הכל? היא תמיד תשאר רק במסגרת הצהרה חגיגית
    בואו נצטמצם בתיאוריות הפשוטות שלנו ומידי פעם נכרסם משהו מ….התיאוריה של הכל.
    ודרך אגב, אני לא בטוח שיש הוכחה שתיאוריה של הכל קיימת!. כתבתי פעם מאמר “אבולוציה של תיאוריות” שמסקנה ממנו היא שלהגיד שתיאוריה של הכל קיימת זה כמו להגיד שקיים ייצור הכי מפותח שאינו דורש יותר אבולוציה. ניראה לי שזה קצת לא הגיוני.
    סתם כך הרהורים נוגים בבוקר יום שבת.
    יום טוב
    סבדרמיש יהודה

  827. יובל

    נראה לי שר.ח פשוט לא הבין שהפתרון הפיזיקלי/מתמטי בדמות ‘תאוריה של הכל’ יגיע ביחד עם פיתרון ברמה הפילוסופית – כמקשה אחת.

  828. ר.ח, לתגובתך זו https://www.hayadan.org.il/astronomers-reach-new-frontiers-of-dark-matter-130112/
    המיתולוגיות אכן הציגו ניסיון לפתור את התעלומה, אך לא הצליחו לתת פתרון אלגנטי לשאלה מי ברא את הבורא. גם הפיסיקה היום אינה מתיימרת ליתן פתרון כזה אלא רק משתדלת להציג מציאות בראשיתית פשוטה ככל האפשר. אני לא חייב לצאת דווקא מן השאלה “איך הכל התחיל?”. כוונתי היא להראות כי כל תופעות העולם הגשמי הינן פיתוחים של ישות יסודית אחת ויחידה. לא רק אני אלא רבים מאיתנו כאן ובשדות מדע אחרים מחפשים תיאוריה אחת של הכל. זה כנראה נובע מן האינטואיציה ומניסיון עתיר דורות לפיהם לכל תופעה מורכבת יש אב קדמון פשוט. זה נכון לא רק בפיסיקה (ובמדעי הטבע בכלל), אלא גם במדעי החברה והרוח ובתחומים רבים אחרים. השאלה “איך הכל התחיל?” וקודם לה השאלה “מה זה בכלל הכל?”, כפי שחידדת יפה, היא רלוונטית מאד.

  829. ישראל

    מה שהמדענים יודעים כיום- לה סאז’ לא ילמד כבר לעולם. עזוב אותו במנוחה.
    מה אתה מציע?
    לא כפיתרון, אלא באופן כללי, מה אתה מציע? רעיון? או קיי, אתה רוצה שאני אענה לך בנימוס, בסדר.
    אל תפנה אותי ללינקים, תכתוב בתגובה שלך אלי מה הרעיון שלך. אני אשקול איך להתייחס אליך.

  830. אז חזרנו לעינייני האופנועים האם לכך הכוונה?
    ובכן, אם כך, השאלה לא כל כך פשוטה לחלוטין ולכל המזדרזים לעלות תשובות סתמיות, הציע לערוך את הניסוי הבא בשני שלבים ניסוי שיעזור להתמדה שבדבר.
    להלן הניסוי:-
    שלב ראשון:- נתון אופנוע בי. אס. איי “תרנגול” הנע בכביש מנקודה A לנקודה B. מסקנה האופנוע מגיע למטרתו!
    שלב שני:- נתון אופנוע ארלי דוידזון הנע שוב בכביש מנקודה A לנקודה B כשבדרך שלולית שמן (שהשאיר אולי ה בי. אס. איי).מסקנה- לא יגיע.
    מה קרה כאן?, מה מנע מאופנוע ההרלי דוידזון להתמוטט ולא להגיע למטרתו?
    מדוע חלקיקיכם שצלחו בבי. אס. איי. ניכשלו בהרלי?
    אתם מבינים שחלקיקים שמרחפים מעל לכביש ולא מתלכלכים בשמן לא יגישו כאן תשובה נכונה על מגש של כסף!
    ממתין לתשובתכם המחוכמה
    שבת שלום
    סבדרמיש יהודה

  831. לא בלון יומולודת. בלון גז מתכתי אטום. לא מתרחב, לא מקרין. עדיין מהרגע שיש למולקולות אנרגיה תחילית, הן תשארנה בתנועה כל הזמן ללא מקור חיצוני של אנרגיה.
    שאלה ב’ צריכה להיות מופנית למקסוול.
    לשאלה ג’ יש לי תשובה: הוא הלך לכל הרוחות והשדים.

  832. ישראל,
    מה עם הג’ירוסקופ? מה אתה רוצה מהג’ירוסקופ? ומה זו בעיית החכוך המפורסמת שכל כך מפריעה בל.ס ? שלמסות יהיה חיכוך עם החלקיקים?

  833. ישראל,

    “אם אתה מכיר את המאבק שניטש בזמנו על הפיכות האנטרופיה (ששיאה בהתאבדותו של בולצמן), הרי שהתאוריה שלי מציעה הסבר מתמטי לאי הפיכות האנטרופיה בזמן.”
    – ההסבר למדוע האנטרופיה גודלת עם הזמן קיים במסגרת תרמודינמיקה סטטיסטית. אולי אני לא מבין ואתה מדבר על משהו אחר.

  834. יובל,

    הדרישה שלך “איך הכל התחיל?” היא לא רלוונטית בעליל לפני שמבינים מה זה “הכל”. ועל השאלה הזו – מה זה הכל מנסים לענות האדונים הנכבדים ישראל וסברדמיש ועוד כמה פיגורות מדעיות. אחרי שנבין ב”ה מה זה “הכל” אז אולי נתחיל לנסות להבין גם איך הוא התחיל.
    אם אתה מתעקש אתה יכול לקרוא כמה תשובות לשאלה “איך הכל התחיל ” במיתוגיות הבבליות/יווניות/אינדיאניות/נורדיות או כל אחת אחרת. מה לעשות שמחינה מדעית השאלה ככל הנראה לא רלוונטית כרגע כמו גם מה קדם למפץ הגדול.

    ישראל,
    1) לגבי הבלון, ברור שהבלון נמצא באינטראקציה עם הסביבה. הסביבה קובעת את נפחו על סמך הפרש הלחצים פנים/חוץ. נפח בלון הכנס אות לחדר ואקום הוא יתרחב ויתרחב ויתפוצץ או שאם הוא בלון פלאים בלתי פציץ הוא יתרחב עד גבולות החדר. תוציא אותו מצוללת בקרקעית הים ומייד הוא יתכווץ לנקודה קטנה.
    כך שמהירות החלקיקים בבלון כן נקבעת ע”י הסביבה החיצונית.
    2) נכון שבאוויר עומד יש תנועת חלקיקים מתמדת אולם מאחר והיא רנדומלית היא מתקזזת ובמאקרו נראה ששום דבר לא זז. רוח היא כמו שאתה אומר עוד וקטור שדואג שלכיוון אחד יש יותר תנועה מלכיוון השני. אולם כדי שייווצר הוקטור הנ”ל צריכה להיות איזו שהיא סיבה, קרי השקעת אנרגיה. בכדור הארץ, כאמור זה הסיבוב שלו ושינויי הטמפרטורה, לחות ועוד. מה יגרום לחלקיקי האתר האקטיבי שלך לפתח וקטור (רוח) שכזו בחלל?

    קספר רפ*אים, מה קרה? קיבלת חופשה מהמוסד? הם יודעים ששוב אתה מלכלך באינטרנט?

  835. לישראל
    פתרון עית החיכוך לא מתאים לאנשים חסרי רוח חיים בחלקיקים שלהם!
    לך לדרכך לכל רוחות השמים, וחזור משוכנע ברוח הקודש
    שבת שלום קודש
    סבדרמיש יהודה

  836. אפשר אני אולי שואל אתכם שאלה?
    מה הבעיה שלכם עם ה”התמדה” שכל כך ברורה לי וכל כך מפריעה לכם?
    שבת שלום
    סבדרמיש יהודה

  837. יהודה
    אני צריך לזוז, אך חלקיקי לה סאז’ אינם גז. בגז אין גרביטציה כפי שציין ר.ח., אבל מודל ל.ס. כן. גרביטציה. בעייתית, אך גרביטציה.
    יהיה אשר יהיה – אצל ל.ס. אין רוח.
    ופעם המאה: מה עם החיכוך??

  838. סטודנט.

    אם קראת מה שכתבתי, מוצעת שם אלטרנטיבה לפירוש של היחסות. היא קרוב לודאי שגויה, אך איני מוצא בה סתירה, ורוצה שמישהו יראה לי איפה. היא פותרת את הבעיה שקיימת לדעתי עם נושא התארכות הזמנים אצל איינשטיין. היא מאפשרת מהירות על אורית, ומבחן ההפרכה שלה הוא בניסוי אותו אני מבקש לעשות.
    אם אתה מכיר את המאבק שניטש בזמנו על הפיכות האנטרופיה (ששיאה בהתאבדותו של בולצמן), הרי שהתאוריה שלי מציעה הסבר מתמטי לאי הפיכות האנטרופיה בזמן.

  839. ישראל שפירא
    אז זהו שלהסאז’ טעה
    הוא דיבר רק על חלקיקים נעים שיוצרים גרביטציה. נקודה.
    אבל
    בפעם המאה
    אם יש חלקיקים נעים בכל המרחב אזי הם גז
    מזה נובע הפרשי לחצים ורוחות
    זה שלהסאז’ התעלם מהם זו בעיה שלו ומחדל.
    בנוסף ביקום הפשוט שלי יש לגז גם עוד תכונה שיש לכל גז והיא הדרך החופשית הממוצעת של חלקיקי הגז.
    מסקנה הכרחית מכך- הגרביטציה נמוגה בחלל היקום יותר מהר מרבוע המרחק
    לכן במרחקים הגדולים ישארו פעילות רק הרוחות.
    כמה שזה פשוט
    איך אמר האדמירל הטורקי כשהביט למרחקים:-
    ניוטון יוק!
    שבת שלום
    סבדרמיש יהודה

  840. ר.ח. רפאים החביב.

    “הביאו מודל אחד הכולל את הפיסיקה כולה כמקשה אחת ואל תסתתרו מאחורי אמירה סתומה כגון “אני לא מדבר על מודל פילוסופי”.” בבקשה, הבא. אותי זה לא מעניין.

    “אולי בן אדם אחד מבין על מה אתה שח, והוא כבר הסביר לך איפה אתה טועה.” מה, אתה לא מבין? אז על מה בדיוק אתה מדבר?

    “אתה ממשיך לדבר על ‘מודל’, אבל לפחות לי אישית לא ברור על מה אתה מדבר בכלל.” “מודל” הוא מה שיובלי אומר. אני משתדל לדבר על “רעיונות”. אם אתה לא מבין, למרות שהכל כתוב, שאל שאלות, בנימוס, ואשמח לענות.

    “ובמקום זאת אני מוצא תגובה מטופשת” הוצאת לי את המילים מהפה.

  841. יהודה, ר.ח.

    עיינתי שנית בלהסאז’נו

    http://en.wikipedia.org/wiki/Le_Sage%27s_theory_of_gravitation

    אין מדובר ברוח.

    יש להבדיל בין רוח לבין תנועת חלקיקים, או מולקולות.
    באויר “עומד” מולקולות האויר נעות במהירות גבוהה. כאשר נושבת רוח – לדוגמה 5 קמ/ש – מהירות הרוח הינה וקטור נוסף לתנועה האקראית של המולקולות. המולקולות עצמן ממשיכות לנוע במהירויות גבוהות בהרבה מ 5 קמ/ש.

    מודל לה סאז’ הוא בדיוק כמו אויר עומד, שבו אין רוח, אך ללא ההתנגשויות שבין המולקולות, או התנגשויות במרחקים גדולים מאוד.

    יובלי המקשן (האם אתה מבסוט מהאבולוציה של כינוייך?)

    למה שארצה לדעת מי יצר את חלקיקי האתר האקטיבי? או חלקיקי ל.ס.? האם מקסוול שאל שאלה זו? לה סאז? מאך?

    ישראל לא שואל שאלות קיומיות. ישראל פיון קטן במשחק החיים הגדול. משאיר מלאכת בריאה לכל יכול, הוקינג, ויובלי הפילוסוף.

    הולך לעפוטה לפלפל מוח לבועלים. קרלוס איבד את האוטו, וצריך לשלוח את ראול פנים להביא אוטו של חוזה לך ברח.

  842. ר.ח רפאי.ם, חן חן על הפירגון.
    ישראל וסברדמיש, שתבינו: לי אין בעיה לא עם אתר פעיל ולא עם גזקום. ממש לא משנה לי אם אלה קיימיים או לא. האמירה שיש תופעה מסוימת או תופעה אחרת שמסבירות משהו תוך כדי התבססות על תופעה או אינטואיציה פיסיקלית קיימת אינה פותרת דבר.

  843. ישראל
    יובל העיר הערה נבונה:
    “הביאו מודל אחד הכולל את הפיסיקה כולה כמקשה אחת ואל תסתתרו מאחורי אמירה סתומה כגון “אני לא מדבר על מודל פילוסופי”.”
    אולי בן אדם אחד מבין על מה אתה שח, והוא כבר הסביר לך איפה אתה טועה.
    אתה ממשיך לדבר על ‘מודל’, אבל לפחות לי אישית לא ברור על מה אתה מדבר בכלל. (לכן אני מנסה גם לא להתערב חוץ מהמקרים הבודדים שבהם אני נכנס לכתבה הזאת כדי לקרוא תגובה שאולי ממנה אשכיל ובמקום זאת אני מוצא תגובה מטופשת)

  844. גם על ישראל אין פותחין פה. הוא התחיל.
    בפרוש הזמנתי כסאח על הרעיונות שלי, לא עלי אישית.

  845. ישראל,

    “איך תוכל באמצעות מנהור לשלוח סיגנל מהר מן האור?”
    – לא יודע, זה לא מה שהבנתי שאתה רוצה. אבל בכל אופן, אם בסינגל אתה מתכוון מידע, אתה כנראה לא יכול. אין אפשרות להעביר חומר או מידע במהירות שגדולה ממהירות האור.

    “איזה הסבר לאי הפיכות האנטרופיה שאינו סטטיסטי?”
    – האנטרופיה מוגדרת במכניקה סטטיסטית.

    “דוגמת הבלון היתה בשביל ר.ח., ששאל מאיפה מגיעה האנרגיה למערכת. באופן אידיאלי, הבלון מבודד לגמרי ובשיווי משקל, ואינו נזקק לאנרגיה חיצונית. למרות זאת, אין ירידה במהירות המולקולות בתוך הבלון, כפי שחשש ר.ח.”
    – אני כנראה לא מבין את הדוגמה. אתה מתאר בלון סגור ומבודד לגמרי מסביבתו, כלומר שאין לו אינטראקציה איתה כלל – אז על איזה ש”מ מדובר? ויותר מזה, לא ברור למה אתה אומר “למרות זאת”. זה לא למרות, זו בדיוק הסיבה שמהירות המולקולות בבלון לא יורדת – כי אין לבלון שום אינטראקציה עם הסביבה שלו.

  846. אתר פעיל או גזקום, זה הכל דיבורים ומשחקים ומעשים אָין. נגיד שיש אֶתֶר פעיל, מניין הוא בא? ונניח שחלקיקי הגזקום שרירים וקיימים, מי יצר אותם? המודלים האלה אינם טובים יותר מן המפץ הגדול או מתורת המיתרים, משום שגם אלה וגם אלה אינם פותרים את שאלת היסוד: “איך הכל התחיל?”. בשלב הזה האם לא עדיף כבר להיבנות על פרק א בבראשית?
    שאלה טובה שאל ישראל על ההתמדה. סבדרמיש מקבל את ההתמדה כמובנת מאליה ובונה עליה את חלקיקי הגזקום. אבל אמור לנו, בבקשה ממך, האם חלקיקי הגזקום נותנים לנו את המנגנון היוצר את ההתמדה? והאתר הפעיל, מה נותן לו את תנועתו אנה ואנה?
    אנא התעשתו והביאו מודל אחד הכולל את הפיסיקה כולה כמקשה אחת ואל תסתתרו מאחורי אמירה סתומה כגון “אני לא מדבר על מודל פילוסופי”.

  847. ישראל! פתחת תיבת פדורה ועברת על יהרג ובל יעבור מספר אחד: אין פותחין ג’ורה על ר.ח רפאי.ם! ההמשך באחריותך המלאה.
    אם יהיה לי כלב אקרא לו מיצי. אבל אני זומם דווקא על חתול ששמו בני-ג’רסי יהיה צִ’ילְבָּה. מה זה “כְּלָבִי”? זה לא פסנתרן בגרמנית?

  848. ישראל,
    תודה על התשובות. אני צריך לחשוב קצת על הדברים שכתבת.

    דבר אחד רק אומר שאני מסכים עם יהודה ש “כן אם אמרת לה סאז’ אזי אמרת בנוסף לגרביטציה של חלקיקים גם דחיפה של רוחות שני דברים שונים הקשורים בינהם ונובעים מתופעת החלקיקים.”

    ולא כמו שאתה אומר “. רוח. לא קיימת אצל לה סאז’ למיטב ידיעתי, רק אצל יהודה.”

    שהרי מה זה הפרש לחצים אם לא שבצד אחד סטטיסטית פוגעים יותר חלקיקים מאשר בצד השני ואז יש דחיפה או לחץ לכיוון הצד עם מספר החלקיקים הקטן? בקיצור רוח.

  849. לישראל
    למה אתה אומר:-
    . רוח. לא קיימת אצל לה סאז’ למיטב ידיעתי, רק אצל יהודה. מעניין, אך לא סביר לדעתי. סוף ציטוט.
    הרי לה סאז’ וחלקיקיו הנעים מכל מקום לכל מקום הם גז. זו הגדרה של גז
    לכן אם אמרת לה סאז’ אזי אמרת בנוסף לגרביטציה של חלקיקים גם דחיפה של רוחות
    שני דברים שונים הקשורים בינהם ונובעים מתופעת החלקיקים.
    לא יכול להיות אחרת!! בזה כבר הייתם חייבים להיות משוכנעים כי זה אפילו לא צריך הוכחה.
    שבת שלום
    סבדרמיש יהודה

  850. סטודנט.
    תודה על הלינקים.
    1. איך תוכל באמצעות מנהור לשלוח סיגנל מהר מן האור?
    2. איזה הסבר לאי הפיכות האנטרופיה שאינו סטטיסטי?
    3. דוגמת הבלון היתה בשביל ר.ח., ששאל מאיפה מגיעה האנרגיה למערכת. באופן אידיאלי, הבלון מבודד לגמרי ובשיווי משקל, ואינו נזקק לאנרגיה חיצונית. למרות זאת, אין ירידה במהירות המולקולות בתוך הבלון, כפי שחשש ר.ח.

    אם אינך מאמין בחלקיקים או קרינה, מה מייצב את הג’ירוסקופ בחלל?

  851. ר.ח רפאי.ם

    תודה, באמת. בדיוק חשבתי לעצמי: מעניין מה חושב לעצמו ר.ח רפאי.ם המז’נון? איזה הארות נוכל לקבל מהמאור הגדול של תורת ההגיון, שמעולם לא העלה רעיון אחד מקורי וכל עיסוקו הוא בקטילה?

    ביננו, האם אתה מבין בכלל את נושא הדיון פה? האם קראת משהו? יש לך אילו הערות לעצם הענין?
    לך הביתה, ילד, תפסיק להפריע למבוגרים.

  852. ישראל
    כשאתה מדבר על המודל שלך אתה נשמע כמו יהודה סבדרמיש כאשר הוא מדבר על המודל שלו.
    וכשיהודה מדבר על המודל שלו הוא נשמע כמו חנין זועובי אחרי קריאת דו”ח גולדסטון על סיפון המרמרה ביום טוב.

    בכל אופן, מה דעתך לשלוח את המודל שלך לביקורת עמיתיך השוכנים לא רחוק ממך על אותה יבשת, לעזוב אותנו בשקט לזמן מה, תלך לחייקינג בהרים עם כלאבי והיידי בת ההרים, ובבוא הזמן תרד אלינו מההרים עם תשובה מעמיתיך לגבי המודל שלך?

  853. “אם תיקח בלון אטום ומוכסף מלא באויר לחלל, שבו הטמפרטורה כ300K, תראה שגם אחרי שעות רבות הטמפרטורה תרד רק במקצת, ז”א שמהירות מולקולות האויר בתוך הבלון קרובה ל400 מ/שניה ביחס למערכת המנוחה שלהן, שהוא הבלון. זהו שיווי המשקל עליו אני מדבר. אם עכשיו יפוצץ הבלון, המולקולות תתרחקנה זו מזו במהירות, ואיתן ירד הלחץ.”
    – צריך להבחין בין תיאור של פלקטואציות בטמפ’ במצב ש”מ לבין ירידה ממשית בה. אם הטמפ’ יורדת עם הזמן, אז המערכת לא בש”מ. יש להגדיר את הבעיה – מהן האינטראקציות האפשריות בין מערכת הבלון לסביבה (חלל) ומהן לאחר הניפוץ? כלומר, האם הבלון יכול להחליף אנרגיה ו/או חלקיקים עם החלל? האם הוא יכול להתכווץ/להמתח?

  854. ישראל,

    אני לא בטוח שהבנתי. איך זה שונה ממנהור של חלקיק שנמצא בבור פונטציאל ומתואר ע”י פונקצית גל סטציונרית?

    בנוגע לאנטרופיה – כבר יש לזה הסבר.

  855. ר.ח. הקיפוד.

    איתך אני מקווה, אפשר להתחרע ולכתוב. בוא ניקח את הסעיפים שלך אחד לאחד.

    1. “שיווי משקל. מה השיווי משקל באויר?” אם תיקח בלון אטום ומוכסף מלא באויר לחלל, שבו הטמפרטורה כ300K, תראה שגם אחרי שעות רבות הטמפרטורה תרד רק במקצת, ז”א שמהירות מולקולות האויר בתוך הבלון קרובה ל400 מ/שניה ביחס למערכת המנוחה שלהן, שהוא הבלון. זהו שיווי המשקל עליו אני מדבר. אם עכשיו יפוצץ הבלון, המולקולות תתרחקנה זו מזו במהירות, ואיתן ירד הלחץ.

    מודל המפץ הגדול דומה מאוד לבלונינו. אינני שואל “מאיפה באה האנרגיה של התפשטות היקום”? אני רק אומר שאם זה כך, המודל שלי פותר את בעיית קביעות מהירות האור בכל מערכות היחוס, וגם את בעיית החיכוך במודל לה סאז’.

    הצבע בכוס שלך יהיה אחיד, אך אין זאת אומרת שמולקולות הצבע הפסיקו לטוס במהירות גבוהה לכל הכיוונים. הן פשוט מתנגשות זו בזו, לכן הן מתפשטות לאט בהתחלה. האנרגיה – ממולקולות המים שבכוס.

    אבל למה לבוא אלי בשאלות? זהו המודל של מקסוול. אותן בעיות קיימות אצלו, ולמרות זאת זהו מודל מוצלח למדי, אם הוא הצליח לחלץ ממנו את משוואה 136, מהירות האור, לא? אני רק הוספתי את השינוי ההכרחי לדעתי, שאין שום מערכת מנוחה לאתר בסגול, ומכאן השם אתר אקטיבי.

    2. לה סאז’. הכדורים שלך לא רצו זה אל זה מכיוון שבאויר יש התנגשויות בין החלקיקים. אצל לה סאז’ אין, או אין כמעט, ואצל יהודה המרחק הממוצע שעוברות המולקולות בין ההתנגשויות כה גדול, שהמודל מאפשר משיכה. אין מחלוקת ( אני מאמין) בין המדענים שמשיכה אכן תתקיים במודל לה סאז’ – הבעיות אין אחרות, בראש וראשונה בעיית החיכוך.

    המודל שלי פותר את בעיית החיכוך. אם תשאל, אפרט.

    3. אי לוקליות. המודל שלי לא התיימר מעולם לפתור את הבעיה. הוא רק מאפשר היתכנות הפיתרון. נדון אם תתעקש.

    4. מדידה. עצם הענין, ניסוי. אם יש משהו בשטויות שלי, הרי שהזמן הוא אבסולוטי. הוא ניתן לכימות בכל נקודה כמספר מסויים של חלקיקים ליחידת נפח. לא ניתן לחזור בזמן, כי בכל נקודה אליה נחזור יהיו בעתיד פחות חלקיקים ליחידת נפח מאשר בעבר.
    אך זה מתנגש אם התארכות הזמנים של איינשטיין. לכן – ואם תרצה אפרט – ההצעה שלי היא שהאור למעשה נע בכמה מהירויות, אולם אנו כצופים יכולים למדוד רק אחת: C. פוטונים מהירים יותר אינם ניתנים למדידה על ידנו, והם לגבינו שקופים, או נייטרלים, או בקיצור: ניטרינו. אולם אם באותה נקודה בה אנו מודדים תהיה מערכת נוספת הנעה במהירות גבוהה יחסית לנו, אפילו קטנה בהרבה מC, היא תוכל למדוד אותם פוטונים מהירים יותר, עוד לפני אחיהם האיטיים הנעים במהירות האור בלבד.

    ולכן הצורך בטיימר מדויק, ומכאן הדחיפות: צריך להשיג את הג’מע בסרן.

    5. מבנה אוירודינמי. לא, זה לא קשור. כל מסה תידחף. קרא את הערך של לה סאז’ בוויקי, או אם תרצה, אפרט.

    6. רוח. לא קיימת אצל לה סאז’ למיטב ידיעתי, רק אצל יהודה. מעניין, אך לא סביר לדעתי.

    7. הגורם לרוח. או למשיכה לפי ל.ס. הפרש לחצים כמובן. כפי שצויין, איננו שואלים מאיפה הגיעה האנרגיה הבראשיתית, או כמו קאנט, ממה מורכבים החלקיקים המרכיבים את החלקיקים מהם בנויים החלקיקים. ל.ס אומר רק שאם תכניס לתא חלקיקים רבים ומהירים שכמעט אינם מתנגשים זה בזה, ותוסיף לתא שני גושי פלסטלינה גדולים, שימסכו את החלקיקים זה מזה, תיווצר משיכה בין הפלסטלינות בכוח ההופכי למרחק בריבוע. הוא כנראה צודק. זה לא עובד עם 2 מפרשים בחדר, כי לחץ האויר בתווך בין המפרשים זהה ללחץ החיצוני, בגלל התנגשויות מולקולות האויר. זה כן יעבוד אם תשתמש בסילוני מים.

    והיום אגב לקחתי את כלאבי לאגם, כך שאל לך להקל ראש בכישורי הימאות שלי.

  856. סטודנט.
    אפילו לא הגענו לתאוריה שלי, כי הוא דיבר רוב הזמן. מה שרציתי לדעת הוא אם ניתן לגרום את קריסת פונקצית הגל של חלקיקים מהירים בנקודה אשר בה הם לא אמורים להיות, בגלל מגבלת מהירות האור, וזאת עיי התנגשות בחלקיקים מהירים אחרים. הוא טען שלא נראה לו כי זה אפשרי, אך לא יכל לציין ניסוי ספציפי. הבטיח לשלוח לינקים לכתבות או יוטיוב, אבל לא קיבלתי עד כה.

    אגב, אם ייצא לך פעם להתעמק במודל שלי, תראה שהוא יכול להסביר יפה מדוע האנטרופיה אינה הפיכה.

    יובלי החנפן.

    מזה? אני משקיע ומזיע, ובסוף יהודה קוצר את כל הקומפלימנטים? ראה עצמך כקרוא לסדר.
    יש יותר מדי ספרים בארץ, כולל ספרי ילדים. אני מסתפק בסיפורים לילדים לפני השינה, ושירים לאשה ביומולדת ובאניברסרי. מה גם שלא נראה לי שרמת הכתיבה שלי יכולה להדיר פרנסה ממאיר שלו ואלתרמן.
    אבל למען היושר והאגו, דברים שכתבתי ראו מעט אור, אפילו ב”גלילאו”, כשהוא עדיין היה ירחון. אם תרצה, אשלח לך לינק.

    שזירה. אם נצא מנקודת ההנחה שאין דבר נורא ואיום יותר מאי לוקליות, אני מצווה עליך בזאת, כאביר, להסביר מיד איך הדבר מסתדר במודל שלך. בכלל, יקל על כולנו, עכשיו כששיחזרת את התיקיות, שתפרסם הכל ונגמור עניין.

    גור אריה.
    אני ישן הרבה יותר טוב בלילה עכשיו, אחרי שפתרת את בעיית החיכוך אצל ל.ס. אולי תסכים לשתף אותנו בכל זאת? אל תדאג, אף אחד לא יגנוב לך את הרעיון. כדאי גם, לפני הפרסום בסיינס ונייצ’ר של פיתרון בעיה שהעסיקה כמעט כל מדען מאז ניוטון, שתעמיד אותה קצת לביקורת עמיתים, לא כן אחינו?

    ר.ח.
    אני הולך קצת לשחק שחמט. חכה, חכה, כשאחזור נבוא איתך חשבון.

  857. סבדרמיש, יהודה,
    לא רוצה לריב איתך כדי שלא תרעיל לי את הקפה בהזדמנות שניפגש. את העבודה אשאיר לר.ח, ותאמין לי שיש לו המון מה להגיד ואני מסכים איתו מראש.

  858. ישראל,

    תוספת להודעה הקודמת:
    לאור העובדה שאתה מתעניין במדע ורוצה ללמוד אותו לפני שאתה ממציא אותו מחדש, הרשה לי להמליץ לך על אתר (שאני מניח שאתה לא מכיר) המרכז ידע עצום בעברית עפ”י קורסים אקדמיים: http://www.hapetek.co.il/?page_id=41
    תוכל למצוא שם מידע רב ואמין בדרך כלל בנושאים שאני רואה שאתה מתעניין בהם – רק בחר את הקורס המתאים.

  859. יובל המבולבל
    אם היצירות הסיפרותיות כוונו אלי, ובכן לי היה כבר הכבוד ליצור סרט “המחקר” שעסק בבורא עולם.
    וחוץ מזה יש עוד יצירות למגרה
    לידיעתם של המעוניינים
    לילה טוב פעם שלישית
    סבדרמיש יהודה

  860. ר.ח.
    אמרת :
    “רוח בכדור הארץ היא דבר משתנה וכאוטי. ה”רוח” של חלקיקי לסא’ג/סברדמיש קבועה וכיוונית ומשפיעה באותה צורה על כל מערכת שמש, כל גלקסיה וכל עצם בכדור הארץ. הגיוני?”. סוף ציטוט.
    ובכן, מר ר.ח., למה בכלל שרוח תישוב בכדור הארץ לכל כיוון, בכל עוצמה ואילו בכל היקום אתה מכריח את רוח הגזקום לנוע ישרה כמו סרגל ובלי סטיות?
    לא הגיוני בכלל!
    כל רוח והכבוד שלה!
    לכיוון אנדרומדה 600 ק”מ לשניה
    מסביב לשביל החלב 230 ק”מ לשנייה
    וכו’
    לילה טוב
    ומשבים קלים הלילה
    סבדרמיש יהודה

  861. חזרתי מהאגודה לאסטרונומיה זה עתה
    חברה לא להתבלבל, ואתם מתבלבלים בהגדרת היקום הפשוט!
    כלומר:
    יש את הגרביטציה שאותה יוצרת תנועת החלקיקים של לה סאז. במילים אחרות, באנלוגיה לדוגמא לאויר, תנועת המולקולות אמורה ליצור גרביטציה. את זה צריך להוכיח ויש לי הוכחה (לטוב או לרע).
    מלבד זאת יש את תנועת הגז עצמו, רוח, שמזיזה גופים גדולים כמו גלקסיות וצבירי גלקסיות. לא צריך בשביל זה גרביטציה של ניוטון או של לה סאז’!
    גם את זה צריך להוכיח.
    מתנועת החיכוך תתעלמו.
    החיכוך עלי!
    (או שכן או ש(:))
    לילה טוב
    סבדרמיש יהודה

  862. ר.ח,
    נראה לי שלישראל, כמו גם ליהודה, אין משנה סדורה (למען ההגינות אודה כי גם במשנה שלי חסרה חוליה). אבל אני מתלהב מאד מן הכתיבה שלו. לא ברור לי איך זה שהוא עדיין לא פירסם איזה רומן או לפחות ספר לילדים 😉

  863. עוד נקודות לגבי אותו לסאג’ וגזקום –
    1) כדי שחלקיקים ידחפו צריך מבנה אווירודינמי כמו שרואים בכנף, מפרש או אפילו ככל שנוגע ל”רוח השמש” והתוכניות להשתמש בה להנעה. כך צורה שאינה אווירודינמית והיא לא תנוע בצורה יעילה. בגרויטציה מדברים על כל מסה ומסה ל-ל-א קשר לצורתה או הרכבה. גלילאו?

    2) רוח בכדור הארץ היא דבר משתנה וכאוטי. ה”רוח” של חלקיקי לסא’ג/סברדמיש קבועה וכיוונית ומשפיעה באותה צורה על כל מערכת שמש, כל גלקסיה וכל עצם בכדור הארץ. הגיוני?

    3) ושוב אותה שאלה, מה מניע את הה”רוח” הנ”ל? למה ששתי מסות תידחפנה זו אל זו אם לרוח יש כיוון אחד? תנסה להעמיד שני מפרשים בכיוון כל שהוא לרוח כך שבמקום ששניהם ינוע לאותו כיוון הם ירוצו זה אל זה. משום מה לא נראה לי שלסאג’ יהודה או איש הרים שכמותך חזקים בימאות.

  864. ישראל,

    תתפלא אבל דוקא כן הסתכלתי בלינק במשוואה 136 וסביבותיה. וכן אני מכיר עד לעייפה ( אודה ולא אבוש בעיקר מהידען) את מודל לסאג’ והיקום הפשוט של מיודענו.

    לגבי המודל שלך:
    1) אתה אומר “שיווי משקל. מה השיווי משקל באויר?”. האוויר נע כי כדור הארץ נע, וכי השמש מחממת ובלילה האוויר מתקרר, גזים עולים מן האדמה וכעוד כהנה וכהנה. האם אתה מציע שכל היקום מסתובב ולכן האתר שלך נע? ( דרך אגב אתה צריך למצוא לו שם נורמאלי כי זה נקרא כל פעם כאתר – כמו כל מקום ואתר, כמו איזה אתר באינטרנט). שאחרת אין להבין למה אין שיווי משקל ומה מניע את חלקיקי האתר עד למהירויות אינסופיות? קח כל מערכת סגורה, כוס לצורך העניין טפטף דיו ומהר מאד הצבע יהיה אחיד בשיווי משקל. ואל תגיד לי אלה תכונותיהם, זה נפנוף ידיים, מהיכן האנרגיה???
    2) לגבי הכדורים שירוצו זה לזה (וזה נכון ללסאג’ וגזקום), האם בחדר עם גז בשיווי משקל, אוויר לצורך העניין, שני כדורים ירוצו זה אל זה? הרגע ביצעתי את הניסוי עם שני כדורי גומי על הdesk שלי והפלא ופלא הם לא זזו.
    3) איך המודל שלך מסביר את האי לוקליות? להגיד יש חלקיקים שנעים במהירות אינסופית זה נפנוף ידיים. זה כמו להגיד יש דברים שאנו לא מבינים. עדיין השאלה תקיפה, גם אם יש חלקיקים מהירים אינסופית, איך הם “יודעים” לעבור מאלקטרון שזור בדיוק למשנהו במרחק שנות אור? באותה מידה הייתי יכול לטעון שהמרחב מקופל כך ששני האלקטרונים השזורים הם בעצם אחד בממד אחר. כמו שתצייר שתי נקודות על דף שנראות רחוקות זו מזו ותקפל אותו במימד השלישי כך שתגענה זו בזו.

    4) מסקרן אותי אותי, איך אתה חושב שמדידה מאד מאד מאד מדוייקת תאשש את המודל שלך?

    אם אתה צריך עוד זמן בהרים תרגיש חופשי

  865. ישראל,

    האם שאלת אותו על רעיון איך לעשות את הניסוי שאתה רוצה? על התאוריה שלך? הרעיונות והביקורת שאני יכול לתת לך הם בערך כלום מול מה שיכול פיזיקאי בכיר במעבדת חלקיקים. שגיאות שלי יקח למצוא שבוע (אם בכלל), הוא ימצא מיידית.
    בכל אופן, לא התעמקתי בהודעות האחרונות, כי אין לי זמן לקרוא/להגיב כראוי.

    אגב, קומיקס משעשע: http://abstrusegoose.com/272

  866. תודה. כעת הבנתי. למעשה, המודל שלי מסביר את השזירה הקוואנטית, אך לא שמתי לב לזה. אלה לא המקטבים שמשפיעים על הסביבה אלא הסביבה היא זו שמאורגנת מראש.
    לא קוצר סבלנותי כי אם חוסר הזמן הוא בעוכרי היום. אחזור בהקדם אנשאללה. ושוב תודה.

  867. סטודנט.
    זה לא רעיון רע, אם אגיע לשלב שאוכל לאמר להם בדיוק מה צריך. נפגשתי לפני כמה שבועות עם אחד הבכירים במעבדת חלקיקים בUCLA, אבל לא יצא מזה הרבה לדאבוני, מה גם שלמרות מהירותם הרבה של חלקיקים במאיצים, נראה לי שאסתבך מהר מאוד עם אפקטים קוואנטים. עדיף מבחינתי למדוד גלי רדיו עם טיימר מדויק. גם הרבה יותר כף. עדיין מחפש רעיונות איך לעשות ניסוי.
    אם יצא לך לקרוא מה שכתבתי, האם מצאת איזו שגיאה יסודית בטיעון?

    יובלי קצר הסבלנות.

    אשתדל לקצר בסעיפי, למען לא יעלה לך הסעיף לראש:

    מסה, אנרגיה, אה?
    1. גרביטיציה: “זה נראה כאילו הם נמשכים זה לזה” זה לא רק נראה. הם אפילו מתנגשים בסוף. גם זה רק נראה?
    2. יש דרישה, יש.
    3. אז תסביר. יהיה אשר יהיה, האם אתה טוען שלאתר מערכת מנוחה מסויימת? למה דוקא זו?
    4. בכל חנות ספרים יש ספרים על שזירה. יש גם את ההרצאות המצוינות של סטנפורד:
    http://www.youtube.com/watch?v=0Eeuqh9QfNI
    שים לב להרצאה 5 דקה 27. משפט בל. ומה שחשוב אולי לך: 1:12 שם מצוין במפורש שההוכחה היא מתמטית. ניסוי אספקט בא רק להראות את שהיה ברור גם בלעדיו.

    אם תהיה מעונין, אוכל גם אני לנסות להסביר, אך זה עלול לארוך חצי עמוד שלם, כך ששקול אם תעמוד בכך.

    5. אם תואיל לפרסם את המודל שלך מהתחלה ועד הסוף ולא בחלקים, אולי נוכל לדעת במה מדובר.

    אגב, האם קראת מה שכתבתי? נכון, זה היה כפי שביקש ר.ח., לאט ובזהירות. יש הסתייגויות? מישהו? יהודה? אתם בכלל עוד שם?
    או שאולי המודל פשוט מושלם וזהו?
    יאללה, כסאח! ( לא אותי, את המודל).

  868. מנפלאות שועל האש ומוזילה: לפני יומיים או שלושה, באמצע כתיבת השורות הבאות העלים לי הוירוס את הדפדפן. שיחזרתי דברים במחשב אחר, והשחזור יצא כה מוצלח עד כי אפילו הדברים שנעלמו לי באמצע כתיבתם שוחזרו אף על פי שלא נשלחו. אמנם רוב הדברים כבר לא רלוונטים כי שיחזרתים מהראש ושיגרתים הנה, וההקדמה חנפנית ולא עניינית. אך בשביל הקוריוז ולמען ההלל, השבח ויחסי הציבור למוזילה הרי הם. אגב, זה לא אומר שחזרתי לפעילות מלאה. עוד רבה הדרך לשחזור, עוד רבה המלחמה בוירוס (מי כתב את זה?):

    ישראל, הערבה לך שנתך? אני טובע בים של וירוסים, ואתה כהרגלך כותב מגילות. אתה פשוט אוהב לכתוב, יפה והרבה, ועושה בעיקר לייק לעצמך. האם אתה בטוח שאתה במקצוע/תחביב הנכון?

    אתה לא רוצה לכתוב מודל תאולוגי או פילוסופי, אך יש כמה שאלות שעליך לשאול עוד לפני הצבת קושיותיך. למשל, מה זו מאסה ומה זו אנרגיה? יש לנו משוואות המציגות מאסה כטרנספורמציה של אנרגיה ולהיפך, אך אלה מביאות רק לטענות מעגליות. ההגדרות האלה צריכות לנבוע ממשהו חיצוני להן, ורצוי שיהיה פשוט. המודל שלי עושה זאת.
    ולקושיותיך, בקיצור:

    1. גרביטיציה – מוסברת אצלי. במקום לשלוח אותך לקישור, קבל נא העתק-הדבק:
    הגרביטציה איננה תכונה של חלקיק אלא תופעה הנצפית מאוסף של חלקיקים. חשוב על פרוטון כעל “מושבה” של חלקיקים. חלקיקים עוזבים אותו אקראית ואחרים מצטרפים. מספר החלקיקים המאכלסים את הפרוטון ברגע נתון הינו פונקציה של הצפיפות האזורית הכוללת. חשוב כעת על שני פרוטונים. חלקיקים עוזבים אותם ואחרים מצטרפים. שני הפרוטונים נהרסים ונבנים ללא הפסק, אך במהלך ההרס והבניה מחדש הם מתקרבים זה אל זה, משום שבאזור שביניהם נמצא ריכוז חלקיקים הגדול יותר מן האזור שמחוץ להם. זה נראה כאילו הם נמשכים זה לזה, ולתכונה הזו אנו קוראים בשם “גרביטציה”. י

    2. אינרציה – מוסברת אף מוסברת. תיאור מפורט יינתן על פי דרישה.

    3. קביעות מהירות האור בכל מערכת יחוס – זו שגיאה הנובעת מהבנה לא נכונה של תוצאות ניסוי מייקלסון מורלי. גם זה מוסבר אצלי.

    4. אי לוקליות – עדיין לא השתכנעתי שהניסוי במקטבים לא השפיע על התווך המוליך. אתה אומר “כבר דיברנו על זה”, אך מי שדיבר על זה זה רק אתה. כאמור, נכונות התופעה הזאת מסמנת למודל שלי את הדרך הישירה הַפָּחָה, וגם אם מלחמתי בה אינה צודקת הרי היא מובנת.

    5. מסה ואנרגיה אפלות – המודל שלי מתחילתו ועד סופו הוא של חלקיקים שאני מייחס כיום למאסה האפלה. האמונה באנרגיה האפלה נובעת מהבנה לא נכונה של הגורם לספקטרום המיוחד קווי הבליעה. על פי המודל שלי, מהירות האור גדלה ככל שהוא מתקרב אלינו, וזה יוצר אפקט דמוי דופלר. בגלל ההבנה השגויה כי מהירות האור קבועה בכל מערכת ייחוס (קושיה 3 שלך), מתקבל רק ההסבר כי מדובר באפקט דופלר.

  869. טוב, חזרתי מההליכה עם האשה והכלב, כלאבי.

    בדרך שוחחנו קצת על פיזיקה. האשה התעניינה מאוד בתאוריה של מקסוול, בעיקר במערבולות באתר, שהזכירו לה את המערבולות שהילד עושה באמבטיה עם כלאבי. כלאבי התעניין יותר בקוואנטים, בעיקר באי וודאות ובחתול של שרדינגר, ורצה לדעת אם הוא טעים יותר כשהוא חי או כשהוא מת.

    אני ניתחתי את ההררכיה המשפחתית בבית שפירא והגעתי למסקנות העגומות ש: בעצם, אני מתעניין בעיקר באשה, שבעצם מתעניינת בעיקר בילד, שבעצם מתעניין בעיקר בכלאבי, שבעצם מתעניין בעיקר בעצם.

    ר.ח., אם הצצת בלינק (זה בסדר, אני יודע שלא), אתה רואה שהשאלה הראשונה שלך תופסת גם לגבי מודל האתר של מקסוול. אף על פי כן זהו מודל מצוין. הוא אינו שואל מאיפה האנרגיה של חלקיקי האתר, מה הכוח המניע אותם, מאיפה הגיעו ומה השקפתם הפוליטית. הוא פשוט אומר שבמידה והם קיימים, ולהם התכונות שאותן פירט, הרי נוכל לקבל גלים באתר שמהירותם כמהירות האור. בימיו של מקסוול איש לא ידע על רדיו, ורבים פקפקו ברעיון זה שנשמע כהזוי. רק אחרי ניסוי הרץ הוכחה התאוריה.

    אם מהירות חלקיקי האתר תלך ותקטן איני יודע – אינסוף פחות קבוע הוא עדיין אינסוף – אך יש תאוריות גם במיינסטרים שטוענות בהגיון רב שמהירות האור הייתה גבוהה יותר בעבר.

    שיווי משקל. מה השיווי משקל באויר? גם שם המולקולות טסות בכל הכיוונים ואינן נרגעות. אם אתה מתכוון לשיווי משקל לגבי מערכת מנוחה כלשהי, אז איזו? למה דווקא זו? מה עם עיקרון ההומוגניות לכל כיוון?

    מנוחה. כן, בסוף היא תגיע למנוחה. אנטרופיה מקסימלית. כל תאוריה מדברת על כך.

    2. שזירה. (אני מניח שהתכוונת לאי לוקליות). מעולם לא טענתי שיש לי הסבר טוב לאי לוקליות. טענתי שהמודל הנ”ל מאפשר היתכנות אי לוקליות, והוא כן. יש לי איזה רעיון, אבל זה בשלב מאוחר יותר.

    3. גרביטציה. האם אתה מכיר את מודל לה סאז’? הבעיה נדונה פה רבות באתר. המודל מאפשר גרביטציה לפי התאור שלי, אך משאיר פתוחות בעיות שונות, ובראשן החיכוך של החלקיקים עם הפלנטות בתנועה.
    בעיה זו לא קיימת במודל שלי, כיוון שהפלנטות אף פעם לא נמצאות בתנועה, מבחינתן הן תמיד במנוחה יחסית לאתר האקטיבי. אם תרצה, נרחיב זאת אחר כך בניתוח דוגמת האוטוסטרדה, צמצום הרעיון למימד אחד.

    וכן, לפי לה סאז’ אם תכניס שני כדורים לחדר ובו “גז” כפי שאתה מכנה אותו, או “גזקום” כפי שיהודה מכנה אותו, אך ללא התנגשויות כמו בגז רגיל, הם ירוצו זה אל זה.
    יהודה גם יוכל להסביר מדוע ממוצע המהירויות של חלקיקי האתר גבוה בממוצע בכ40 אחוז ממהירות האור.

    בכל אופן, בעיית הגרביטציה מעניינת אותי פחות, ומודל לה סאז’ לוקה גם בבעיות אחרות. רק ציינתי שפתרון בעיית קביעות מהירות האור בכל מערכות היחוס, פותרת גם את בעיית החיכוך אצל לה סאז’. אינני מנסה לבנות תמ”ג. אני מנסה לעשות משהו אחר, יותר דחוף וקונקרטי: לבנות מכשיר שיצליח לשלוח אות מהר יותר מהאור. זה התפקיד שלי. התפקיד שלך – של כולם בעצם – זה לכסח את האמאמא של המודל שלי, להכניס קצת בינה בראשי הסתום, ולהסביר לי מה לא בסדר ברעיונות ההזויים שלי, לפני שאבזבז זמן וכסף על מכשירים חסרי תועלת.

  870. יובל.
    בעניני וירוסים תפנה לר.ח. מחשבים למייקל.
    מה שכן, בעניני זדון, תפנה אלי.
    תתענג על ההפסקה, אבל אל תעלם כמו יהודה, אה?

    ר.ח.

    גן חובה (בסגול) כמו חן חובה.

    אני גם כן יוצא להפסקה קצרה ( קליפורניה + ינואר = 25 מעלות, ומכאן אשה + כלב = הליכה בהרים).

    בינתיים, הצץ בלינק

    http://en.wikisource.org/wiki/On_Physical_Lines_of_Force

    וסקרול למטה למשוואה 136.

    נרחיב כשנשוב.

  871. ישראל,

    למרות הזלזול שלך באינטילגנציה של ביולוגים (גן ילדים??) אני שמח שסוף סוף הואלת לתת כמה כיוונים למודל שלך. הוא מודל מעניין אבל אפילו הדיוט מגן חובה רואהכמה פערים לא מוסברים. למשל:
    1) אתה טוען שחלקיקי האתר האקטיבי שלך נעים במהירויות מ- 0 ועד לאינסוף. מה הכח שמניע אותם? מדוע הם לא מגיעים שיווי משקל? אם טמפרטורת היקום מייצגת את מהירותן אזי המהירות שלהם הולכת וקטנה. אתה גם טוען שהמערכת תתפשט ללא כיוון מוגדר, נכון, עד שתגיע למנוחה.
    2) בשזירה, זה שאתה אומר שיש חלקיקים עם מהירות אניסופית לא פותר כלום. למה בקריסה של אלקטרון שזור נוצרים פתאום חלקיקים במהירות אינסופית? למה לא פחות? איך זה שהם “יודעים” היכן האלקטרון השני ו”עפים עד אליו בדיוק?
    3) גרויטציה? האם אתה חושב שהפרש לחצים בין חלקיקי האתר האקטיבי שלך אחראיים על הגרויטציה (או ששוב לא הבנתי?). האם אתה חושב שאם תכניס שני כדורים לחדר מלא גז הם מייד “ירוצו” זה אל זה או שיתחילו להסתובב זה סביב זה בגלל ה”לחץ” של חלקיקי הגז?

  872. וירוס זדוני השתלט לי על המחשב. עד שזה יודבר אין לי גישה לקבצים שלי.
    1. את לורנץ הבאתי רק כדוגמה להסבר העשוי להתקבל על הדעת. הוא לא היחיד.
    2. השיחה עם צל חופשי לא החלה בקישור שנתתי לך. בנוסף, יש גם כאן שיחה קצרה עם “בא והולך”.
    3. בסדר. תקרא לזה שיחלוף. האם יש מודל סטנדרטי של שיחלוף גרביטונים? ואם יש, האם הוא מציג את חלקיקי החומר הבריוני כמושבות של גרביטונים? אנא, קישור.
    4. עשית די. בהזדמנות אבדוק לעומק.
    לא שמעתי על הספר שנקרא THE FINAL THEORY. בהזדמנות אחפש.
    וכעת אני יוצא להפסקה ארוכה. תודה לך על ההתמדה. היה שלום

  873. 1. בסדר. לורנץ. הגופים מתכווצים בכיוון התנועה. אני מניח שתנועה יחסית לאתר. האין זה מחייב שלאתר מערכת מנוחה מוחלטת כלשהיא? אחרי הכל, את אותה מערכת מנוחה ניסה ניסוי מ-מ למצוא, לא?

    2. מה שמצאתי בקישור שנתת לצל בעניין גרביטציה הוא “מה שנראה כמו השפעת הגרביטציה על תנועת האור (“עיקום המרחב”), נובע מכך שהאור נע מהר יותר באזור צפוף (אך לא צפוף מדי, שאז הוא נחסם) ושתופעת המשיכה בין “מושבות” של חלקיקים הינה חזקה יותר באזור צפוף. במקום להשתמש במונח “עיקום המרחב” אני אומר “שינוי הצפיפות”.

    האם אני אמור להבין מכך איך עובדת הגרביטציה? האם פספסתי משהו?

    3. “חלקיקים עוזבים את המושבות האלה באופן אקראי וחלקיקים מצטרפים” נשמע כמו שיחלוף.

    4. ההסבר על מה שהולך עם המקטבים נמצא בכל ספר על שזירה. נתתי לך בעבר את שם הספר והעמוד, וגם העתקתי עבורך את הקטע הנדון. אינני יכול לעשות יותר מכך.

    האם שמעת על ספר שנקרא THE FINAL THEORY? יש שם רעיונות קצת דומים, אני מאמין.

  874. משתולל אצלי וירוס, וזה אילץ אותי לעבור למחשב אחר עם אותיות קטנות. לכן “קלסיות”.
    למשל, ההתכווצות שהציע הנדריק לורנץ יכולה להוות הסבר טוב.
    לא הבנת את העניין של המושבות. זבשך.
    מי דיבר על שיחלוף חלקיקים?
    לא אוכל לענות לך על המקטבים לפני שאבין בדיוק מה הלך שם

  875. 1. מה שגוי בהבנת מ-מ?
    2. איך בניית מושבות יוצרת משיכה? למה בכוח הופכי בריבוע?
    3. במה שיחלוף חלקיקים שונה מהמודל הסטנדרטי של שיחלוף גרביטונים?
    4. כיצד יוכלו המקטבים לתקשר אם מצבם נקבע רק אחרי שהפוטונים עזבו את המקור?
    5. מהן הקלסיות הרחוקות? (השגיאה הראשונה שלך בעברית עד כה).

  876. ולעניין האי-לוקאליות בשזירה הקוואנטית: לא שוכנעתי כי הניסוי לא השפיע על התווך. אתה אומר שכבר דיברנו על זה, אך בינתים מי שדיבר על זה בין שנינו זה רק אתה.
    והיה, אם יתברר כי הניסוי אכן לא משפיע, עדיין אפשר יהיה לחפש לזה הסבר בתוך המודל שלי, אך מן הגשר הזה אין טעם שאנסה לקפוץ בטרם אגיע אליו.

  877. א. הבנה שגויה של תוצאות ניסוי מייקלסון מורלי.
    ב. הנחה סבירה כי אותן תוצאות יתקבלו בכל מקום ביקום.

    ולקושיה אחרת שלך:
    היות שהוסכם כי מהירות האור קבועה, למרות שהיא לא, האופן היחיד בו מקובל לפרש את בריחת קווי ספקטרום הבליעה הוא אפקט דופלר, ומכאן האמונה בקיומה של אנרגיה אפלה. במודל שלי, האור המגיע מהקלסיות הרחוקות מאיץ ככל שהוא מתקרב אלינו, וזה יוצר תופעה דמויית דופלר.

    הקשת גם על החומר האפל: המודל שלי מבוסס כולו על חלקיקים שאותם אני מייחס, נכון לעכשיו, לחומר האפל.

    גרביטציה מוסברת אצלי בבהירות. הצץ נא שוב בדברי אל צל חופשי בהם אני מציג את חלקיקי החומר הבריוני כמושבות של חלקיקי חומר אפל. חלקיקים עוזבים את המושבות האלה באופן אקראי וחלקיקים מצטרפים, וכך המושבות נהרסות ונבנות ללא הפסק. ריכוז החומר האפל במרחב בין שתי מושבות כאלה הינו גדול יותר מאשר ריכוזו באזורים אחרים, ולפיכך המושבות נוטות להיבנות מחדש על הציר שביניהן והמרחק שביניהן קטן. מתחת למרחק מסויים מתחזקת נוכחות של כוחות חשמליים הבולמים את ההתקרבות הזו, אך אלה נובעים מתכונה אחרת של ריכוזי החלקיקים. גם אינרציה מוסברת באופן דומה, ועל כך בהזדמנות אחרת.

  878. בסדר, גם לי קשה לכתוב. להבא אשתדל להציג רעיונות בריקוד.

    עכשיו תורך. איך מוסברת קביעות מהירות האור בכל מערכת יחוס?

  879. ישראל, הבסתני. ימי הקריאה שלי עברו חלפו, ואתה לא חדל לכתוב מגילות.
    הריני מבטיחך נאמנה כי המודל שלי פותר את קושיותיך, אבל תצטרך לחכות עד שיפורסם בדרך המקובלת.

  880. יובלי, עזוב מודלים פיזיקליים, תמיד תישאר מודל לכולנו לטאקט ועברית צחה. רק בשביל זה היה שווה המאמץ שעשינו לשלוח אותך לבריטניה. אני נותן לך אישור לקרוא אותי לסדר אם אני מגזים ומתפרע בברוטליות שלי.

    קראתי את קישורך, ועדיין לא הצלחתי לדלות ממנו איך בדיוק העסק פועל. כנראה שרב הנסתר על הגלוי. נגיד אשר נגיד על לה סאז’, לפחות אצלו הכל ברור פשוט והגיוני, ומכאן קסמו. זו הדרך הנכונה לבנות מודלים לדעתי, אפילו אם הם שגויים או לא מושלמים.

    אינני מכיר שיטה מדעית שאינה פשוטה, עקבית והגיונית. איני מגיע למסקנה שאם מודל א’ אינו מתיישב עם מודל ב’, משמע שאחד אינו נכון. יתכן שאפשר להרחיב אותם כך שיתיישבו. לורד קלווין, (תמונתו אני מאמין מתנוססת בבית ספרך), שלל את גילו המופלג של כדה”א משיקולים תרמודינמיים. אחרי איינשטיין, הורחבו שני המודלים ושולבו בהצלחה. כנ”ל לגבי נדידת היבשות.

    אני מאמין גם שמודלים לא צריכים להיבנות אלא אם כן יש לכך סיבה. אחרת מה הטעם? שיחה על כוס קפה על עניני דיומא, על הא ועל דא ובעיקר על הא? על המכונית החדשה של של יוסי? על גילה שקיצרה את החצאית כיוון שהיא אוהבת את גיורא המדריך? על התאוריה החדשה של רמי המשלבת כבידה ואלקטרומגנטיות עם מע”מ וספגטי ופרשנות של הבבאזוזו?

    בקיצור, כדי לבנות מודל צריך שהוא ישפר את הקיים.

    בקשר לרעיון שלי.

    אני לוקח כמה תופעות שלדעתי ההסבר להן אינו קיים או אינו מספק, ותופעה אחת שלדעתי יש לה מודל מצוין אך לא מושלם. התופעות שההסבר להן אינו שלם הן:

    1. גרביטיציה

    2. אינרציה.

    3. קביעות מהירות האור בכל מערכת יחוס.

    4. אי לוקליות.

    5. מסה ואנרגיה אפלות.

    התופעה שההסבר לה הוא מצוין אך לא שלם היא האלקטרומגנטיות.

    אני מנסה לראות אם אפשר להכניס איזה שינוי בהסברים שניתנו לתופעות אלו, כדי לגרום להן לעבוד בצורה חלקה יותר, ואם אפשר להשתלב זו בזו.

    1. גרביטציה. מדוע שני גופים רחוקים מושכים זה את זה?

    2. אינרציה. מה מתנגד לגוף בזמן האצה? מה מייצב את הג’ירוסקופ בזמן סיבוב?

    3. קביעות מהירות האור בכל מערכת יחוס. האם יש הסבר פיזיקלי לתופעה, או שחייבים כמו איינשטיין להשאיר אותה כפוסטולט? ואם יש הסבר, האם הוא פותר את כל המוזרות של תורת היחסות?

    4. אי לוקליות. התופעה המוזרה מכולן. תכונות של חלקיק א’ עוברות לחלקיק ב’, בכל מרחק ובאפס זמן, כאילו שני החלקיקים הם אחד. ולמרות זאת, אין אפשרות לשלוח אינפורמציה באמצעות התופעה.

    5. מסה ואנרגיה אפלות. מדוע אין הגלקסיות מתנהגות כראוי לפי המודלים הקיימים.

    אלקטרומגנטיות. כיצד פועלת פעולה מרחוק שמקורה בשתי תופעות מוכרות: חשמל ומגנטיות.

    נתחיל מהאלקטרומגנטיות.

    מקסוול נתן הסבר מצוין לתופעה. לפי ההסבר שלו, היקום מלא ב”אתר”. האתר הוא חלקיקים קטנים בעלי דיפול מגנטי, שניתן לראות אותם כאילו נוזל זורם מהקוטב ה”צפוני” של הדיפול ל”דרומי”. באמצעות מודל זה, הוא הצליח לחשב באמצעים הידרודינמים ומכניים את התכונות של מערבולות באתר, להראות שהן יוצרות את התופעות הידועות של משיכה ודחיה הידועים לנו מהחשמל והמגנטיות, ולהראות שהן מתקדמות באתר כגלים מאונכים זה לזה במהירות האור.

    אולם כאן הבעיה: מהירות האור יחסית למה? מקסוול, ורוב בני דורו הניחו שיחסית לאתר כמובן, כמו שגל קול מתקדם יחסית לאויר. נותר רק למצוא רק יחסית למה נייח האתר, והכל מסתדר.

    ההמשך ידוע: ניסוי מייכלסון מורלי לא הצליח למצוא כל מערכת מנוחה לאתר. איינשטיין פתר את הבעיה עיי פוסטולט 2 ביחסות: מהירות האור זהה בכל מערכת יחוס. מה שמשתנה הוא הזמן והמרחק. אין אתר.

    נעשה הפסקה קצרה בסיפור ונשאל כמה שאלות:

    1. אם אין אתר, כיצד הצליח מקסוול לחשב כל כך טוב את מהירות האור מתוך קבועי החשמל והמגנטיות? מקרה?

    2. בזמן ניסוי מייכלסון מורלי מודל היקום היה של יקום אינסופי ואיזוטרופי בכל כיוון. נאמר שהניסוי היה מצליח, ומערכת המנוחה של האתר הייתה מתגלה כמערכת המנוחה של בארבור 17. השאלה היא: מדוע דווקא מערכת מנוחה זו? מה כל כך מיוחד בה?

    3. כידוע, הפתרון של איינשטיין כולל ויתור על הזמן והמרחק המוחלטים. זהו פיתרון רדיקלי, נוגד אינטואיציה, וכפי שניסיתי להראות בעבר אינו מתיישב עם תאוריית המפץ הגדול. מאידך, איזו אפשרות אחרת עומדת לפנינו שתתישב עם העובדה שמהירות האור זהה לכל מודד בכל מערכת יחוס?

    כדאי לעמת תופעה זו עם תופעת האי לוקליות. אי לוקליות היא תופעה כה בלתי נתפסת ולא אינטואיטיבית, שכל הסבר אחר עדיף עליה, כמו שניסית אתה, יובל, בהסבר התקשור בין המקטבים. למרות שרבים ניסו להראות שהתופעה אינה נכונה (ראה דבריו של ניק הרברט בקישור לר.ח.) עד היום איש לא הצליח לתת פיתרון אלטרנטיבי, ואנו נאלצים לקבל את העובדה שהמציאות היא קוואנטית, לא לוקלית ולא אינטואיטיבית.

    אבל מה עם פוסטולט 2? האם ניתן למצוא הסבר, משונה ככל שיהיה, לכך שמהירות האור זהה לכל מודד ושעדיין תשאיר על כנה את הזמן המוחלט?

    התשובה שלי היא שכן.

    עכשיו, לפני שיתחילו הצעקות. כל מה שאני הולך להגיד הוא בגדר ספקולציה, שרוב הסיכויים שאינה נכונה במציאות. אינני מתימר כמובן לסתור את איינשטיין, אלא רק להעלות תאוריה, שאני לא מוצא בה פגם, ושהייתי שמח מאוד אילו מישהו מן המגיבים היה מראה לי מה לא בסדר איתה.

    אבל קודם צריך לקרוא למקס.

    מקס הוא מפקד מיתולוגי מהצבא. כל מה שהיינו עושים הוא היה אומר לנו: אז מה, תפסתם ת-ת, אה? וכל מה שהיינו אומרים, הוא היא אומר: אז מה, פתחתם ת-ת, אה?

    מניתוח התאוריה של מקס, הגעתי למסקנה שהשיטה בצבא היא לתפוס ת-ת ולפתוח אותו.

    אותו הדבר לגבי תאוריית מקסוול. פשוט צריך לפתוח את מודל האתר.

    מה הכוונה. אם ניקח גלים באויר, נראה שהם נעים תמיד במערכת סגורה. אם בחדר, אז מהירות הקול היא יחסית לחדר. אם במטוס, אז יחסית למטוס. אם באויר החופשי, אז יחסית לכדה”א. אם נושבת רוח, אז יחסית לרוח.

    אבל מה קורה אם המערכת פתוחה לחלוטין? יחסית למה ינועו הגלים?

    כדי להבין את הרעיון, נחשוב על בלון בחלל. מהירות הגלים בתוך הבלון תהיה יחסית לבלון. נאמר שהבלון פוגש בלון אחר, הנע במהירות שונה, ושניהם מתאחדים לבלון אחד גדול יותר. מהירות הגלים תהיה עכשיו יחסית לבלון החדש. נוסיף עוד ועוד בלונים,, ונקבל כל פעם מרכז מהירויות חדש לגלים.

    עכשיו מה יקרה אם נאחד אינסוף בלונים הנעים בכל המהירויות לכל הכיוונים, ופשוט נפוצץ אותם?
    התוצאה היא שלא יכול להיות מרכז מהירויות משותף לאויר המשתחרר, כי אז יהיה עדיפות לבלון מסוים שיחסית אליו מרכז המהירויות לא השתנה, מה שסותר את עיקרון ההומוגניות. ואם כל המערכת נמצאת בלחץ, נקבל “אתר אקטיבי” שבו בכל נקודה ישנן מולקולות הנעות בכל הכיוונים ובכל המהירויות מאפס עד אינסוף. המערכת גם תתפשט, ללא מרכז מוגדר.

    וכמו שהראתי בעבר:

    https://www.hayadan.org.il/particles-in-the-dark-2111111/#comment-319159

    מעבר למהירות מסויימת, מולקולות פשוט מפסיקות להשפיע, ולכן נהפכות ל”שקופות” מבחינת החומר בו הן פוגעות.

    ולכן, כל גל שיתקדם באתר האקטיבי, יתקדם למעשה בהרבה מהירויות, אולי אפילו כל המהירויות, אך לגבי המודד, או הצופה, הוא תמיד יופיע כבעל מהירות אחת ויחידה (במקרה שלנו – מהירות האור).

    שים לב שהזמן המוחלט נשאר על כנו, לשמחתו הרבה של אוקהם. אם נמשיך לפתח את המודל, נראה שניתן אפילו להגיע לתיאור מתמטי לא רע של “מהו זמן?”.

    עכשיו, אם נראה את האתר האקטיבי כאוסף של מגנטים קטנים הנעים בכל המהירויות, כפי שתאר מקסוול בהצלחה כה רבה, נראה שחוץ מהענין המגנטי, המגנטים המעופפים האלו יכולים לשמש גם כחלקיקי לה סאז’, אלא שהפעם אין לנו את בעית החיכוך. הסיבה: לא משנה באיזו מהירות נע גוף דרך החלקיקים, הוא מבחינתו עומד, וזאת מאותה הסיבה שהסברנו קודם: אין מרכז מהירויות משותף, כמו בתאור לה סאז’ הקלאסי. במימד אחד, ניתן לחשוב על אוטוסטרדה דו כיוונית, בעלת אינסוף מסלולים שבכל מסלול נעות מכוניות במהירות מסויימת. אם תזרוק פלסטלינה באמצע, היא לא תזוז, כי כל המכוניות מקזזות זו את זו מכל הכיוונים, ולכן הפלסטלינה מבחינתה תמיד במנוחה, לא משנה באיזו מהירות היא נמצאת. היא תתקל בהתנגדות רק כשתנסה להאיץ, וכאשר תגיע למהירות קבועה שוב יתאפס הכוח הפועל עליה. דבר זה מסביר את האינרציה.

    שים לב גם להיתכנות האי לוקליות, כי הרי יש לנו באתר האקטיבי חלקיקים הנעים בכל המהירויות כולל אינסוף. שים לב גם לדמיון לגלי קול: הללו נעים במהירות מסויימת – מהירות הקול – אך נעים בתווך, האויר, שמורכב ממולקולות שמהירותן יכולה להגיע לפי כמה ממהירות הקול לפי התפלגות מקסוול – בולצמן. מולקולות אלו נושאות עימן אינפורמציה, אך לנו אין אפשרות לשלוח אינפורמציה מעל למהירות הקול.

    זהו עד כאן לעכשיו. לילה טוב.

  881. ישראל,
    לא אמשוך אותך במילים. ממה שראיתי אצלך, אכן התרשמתי שאתה מן הקטלנים. שיטתך פשוטה, עקבית והגיונית. אתה נוטל שני מודלים שאינם מתיישבים זה עם זה ומגיע למסקנה שלפחות אחד מהם אינו נכון. אתה אפילו מציע פתרונות, וזה מבורך.
    המודל שלי אמנם מתחיל מן האפס, אך אינו מתעכב שם יותר מחלקיק השניה. בין היתר הוא גם מציג “מפץ גדול”, ומי שרוצה יכול למצוא בו נקודות קונפורמיזם, אך זה פחות חשוב מדברים אחרים שהוא מציג. אחד הדברים שמאד רלוונטיים בו לתקופתנו זה הקשר שהוא מוצא בין הגרביטציה לבין משפחת הכוחות האלקטרומגנטים. תוכל לראות משהו מזה בהתכתבות האחרונה שלי עם “צל חופשי” בקישור הזה: https://www.hayadan.org.il/opera-confirms-and-submits-results-but-unease-remains-191111/#comment-324749
    אינני מתעקש איתך שתתחיל מן המקבילה ל”ויהי אור”, אך אם יש בידיך להראות, למשל, כיצד הגרביטציה והאלקטרומגנטיות קשורים זו בזו, אנא שתף.

  882. יובלי התחמן.

    אתה מנסה לגרום לי למשוך את האש, אה? לא, לא. התפקיד שלי הוא לקטול מודלים. שלך – להעלות אותם ולהיקטל. אנחנו קוראים על כל מיני הדלפות לתקשורת בקשר למודל שלך, שנסחטו בעינויים. אתה הבכיר ועליך להתחיל. שאלתי אותך בעבר אם אתה רוצה פירגון או כסאח ולא קיבלתי תשובה. אבל דבר אחד אני יכול להבטיחך נאמנה: אם המודל שלך מתחיל מ”בראשית לא היה שום דבר, והשום דבר איין את עצמו ליש”?, כך סתם בלי הסבר למה, אז נראה לי שעדיף שתפרסם אותו במדור המדעי של יתד נאמן.

    בקשר למודל שלי, שהוא הרבה פחות מאורגן ומסודר משלך, ולמעשה עד שהועלה עניין המודלים אפילו לא ידעתי שיש לי מודל, הרי שמתוך נסיון העבר הקצר והמר שלי, אני מעדיף לא לדסקס אותו עם מי שאינו ממש מעונין בנושא, או שלא מכיר את הנושאים הבאים:

    1. עיקרון מאך.
    2. גרביטציה.
    3. אינרציה.
    4. יחסות, ובראש ובראשונה התארכות הזמנים.
    5. אי לוקליות בשזירה קואנטית.

    זו הסיבה שאני מעביד בפרך את מי שגילה התענינות (עד עתה, רק אתה ו ר.ח.), ושולח ללינקים. ראשית, כדי לוודא שמכירים את החומר. שנית, כדי להבין שהחומר לא ממש קל. שלישית, כדי להיות בטוחים שהנושא באמת מעניין את השואל, אחרת הוא ישבר מהר מאוד כשנפתח את הנושא.

    כמובן שאני מוכן לעמוד בקריטריונים של עצמי. אם יש לך אילו לינקים שעלי לקרוא בקשר למודל שלך, שלח אותי אליהם.

    אגב, אני לא מעונין בשום חיזוקים חיוביים. רק ביקורת כסאח, אבל מקצועית ועניינית.

  883. “בעבר האשמת אותי (לא בצדק לדעתי) בהתפלספות, וציינת גם שצריך ללמוד את הפורמליזם המתמטי. אני מאמין שזה בדיוק מה שעשיתי עם נושא השזירה הקוואנטית, כולל הפורמליזם המתמטי במספרים מרוכבים. ”
    – אני לא יכול לדעת מה באמת אתה או אחר יודעים, רק לקבל רושם על פי תגובות. מן הסתם אני יכול לקבל רושם מוטעה.

    “נראה לי שזה עדיף על הפניות לכתבה של גלית הכוללת חצי עמוד.”
    – אין ספק. עוד יותר עדיף לעשות דוקטורט בקוונטים.
    ואגב, הכתבה היא לא של גלית – גלית היא השואלת. מי הכותב אתה יכול למצוא בסוף הכתבה.

    “אגב, ציטוט מהכתבה: “במכניקה הקוונטית, ברגע שאנו מבצעים מדידה על חלקיק מסויים, פוטון לדוגמא, אנו אומרים כי פונקציית הגל קרסה ברגע שאנו מבצעים את המדידה” שעונה אולי לשאלתך “(אגב, איפה מוזכרת שם קריסת פונקצית הגל?)”.”
    – אתה טועה. קישרתי לשתי כתבות: בראשונה לא מצאתי איפה מוזכרת קריסת פונקצית הגל – ועליה שאלתי את השאלה. בכתבה השניה היא אכן מוזכרת, אבל לא עליה שאלתי.

    לא סתם קישרתי לכתבות האלה. הכותבים שם (למיטב ידיעתי) הם חוקרים במכון – אפשר ללמוד מהם דבר או שניים.

    “דרך הקיצור עליה אתה כותב כוללת במקרה שלי שני ספרים מלאים בנושא השזירה, ואת ההרצאות הנהדרות של סוסקינד:”
    – אתה נשמע מתגונן, לא ברור לי למה. אמרת מקודם שאתה אוהב תכלס, מה יכולתי להסיק מזה?
    בכל אופן, חזק ואמץ. צריך ללמוד מהפיזיקאים בעולם ולא לשלול את עבודתם בלי להבין מה הם בכלל עושים.

    “אין פה שום עניין של דיון. או שאתה מכיר מכשיר העושה זאת או שלא.”
    – שאלת אם יש לי רעיון, לא אם אני מכיר מכשיר שעושה זאת.

    “מדוע כה רבים לא מסמפטים את מכניקת הקוואנטים? מדוע לחפש כמו איינשטיין אמת עמוקה יותר, דטרמיניסטית, מדוע לא פשוט לקבל את האופי ההסתברותי של המציאות ולסגור עניין?”
    – זו שאלה פילוסופית, אבל אני חושב שהתשובה עשויה להיות קצרה: תורת הקוונטים לא מתיישבת עם האינטואיציה.

    “עד כאן. בהצלחה עם מודל הפלורסנציה .”
    – תודה. בהצלחה גם לך בלימודי הקוונטים.

  884. ישראל שפירא,
    ברור שהמודל שלך אינו עוסק בשום אספקט פילוסופי או אקסיומטי והוא פיזיקלי ומתמטי לחלוטין. כך באמת צריך. אבל הוא חייב להתחיל ממשהו. התואיל לספר לנו ממה?

  885. בא והולך,
    צר לי על לכתך. במעט שדיברנו עזרת לי מאד. מכל המתכתבים אתי, רק אתה וצל חופשי היחידים שגם מקשיבים. כל השאר (ואני בתוכם) רק מטיפים את תורתם ולא מקשיבים לאחרים.

  886. קבלו תיקון – מה שיש בא’ ואין בב’, + מה שיש בב’ ואין בג’, – גדול או שווה ממה שיש בא’ ואין בג’.

  887. סטודנט.

    ראשית אני רוצה שתדע שאני מעריך אותך מכמה סיבות. הראשונה, שבחרת ללכת לטכניון ולא לעריכת דין, שאין לי ספק שהיית יכול להצליח מאוד גם שם. שנית, שאתה משקיע ומחפש את האמת.

    טוב, אחרי שהתחנפנו, נוכל לחזור לעניין.

    1. בעבר האשמת אותי (לא בצדק לדעתי) בהתפלספות, וציינת גם שצריך ללמוד את הפורמליזם המתמטי. אני מאמין שזה בדיוק מה שעשיתי עם נושא השזירה הקוואנטית, כולל הפורמליזם המתמטי במספרים מרוכבים. נראה לי שזה עדיף על הפניות לכתבה של גלית הכוללת חצי עמוד. אגב, ציטוט מהכתבה: “במכניקה הקוונטית, ברגע שאנו מבצעים מדידה על חלקיק מסויים, פוטון לדוגמא, אנו אומרים כי פונקציית הגל קרסה ברגע שאנו מבצעים את המדידה” שעונה אולי לשאלתך “(אגב, איפה מוזכרת שם קריסת פונקצית הגל?)”.

    2. אחרי שקראתי את הכתבה של מריוס, מתחזקת אצלי ההרגשה שרווחת איזו אי הבנה בסיסית בנושא השזירה הקוואנטית ואי הלוקליות. יתכן שאי ההבנה היא בעצם אצלי, ועל זה אפשר לדסקס.

    לפי הבנתי, בשזירה קוונטית הרי ש:

    א. אינפורמצית הספין או הקיטוב עוברת מיידית מחלקיק א’ לחלקיק ב’.

    ב. לנו אין אפשרות לשלוח שום מידע באמצעות השזירה.

    פרדוקסלי? כלל וכלל לא. הפירוש הוא שאינפורמציה ללא ספק עוברת, אך אנו, אם ננסה לשלוח אינפורמציה, ניכשל.

    3. בפרדוקס EPR איינשטיין טעה פעמיים. בראשונה, כשטען שאי לוקליות אינה אפשרית. בשניה, כשטען שמעבר איפורמציית הספין במהירות העולה על האור סותרת את היחסות. היא לא.

    4. “אינני רואה איך זה ממש קשור לדיון שלנו פה.” – זה לא. לא ברור לי למה העלת את זה.” העלתי את הנושא בנסיון לראות אם פתרון בעיית החיכוך אצל ל.ס. של יהודה כוללת גם את התכנות אי לוקליות. לפי המודל שלי כן, ורציתי לראות אם אני ויהודה באותו הכיוון.

    5 “אני אוהב ללמוד ולהבין מהיסוד עד לתוצאה. זו הדרך הנכונה לפי דעתי, ודרכי קיצור הן פספוס גדול בלמידה.” דרך הקיצור עליה אתה כותב כוללת במקרה שלי שני ספרים מלאים בנושא השזירה, ואת ההרצאות הנהדרות של סוסקינד:

    http://www.youtube.com/watch?v=0Eeuqh9QfNI

    6. “יש לך איזה רעיון כיצד למדוד זמן הגעה של אות רדיו בדיוק של ננושניות ויותר?”

    אין פה שום עניין של דיון. או שאתה מכיר מכשיר העושה זאת או שלא.

    7. “מאמין גם שרבים חשים כמוני, ואיינשטיין בראש, אחרת לא היה נלחם בה כל חייו.” – ונכשל, אל תשכח”. הוא נכשל – למיטב ידיעתנו. אני תמיד אומר שאילו חי, אולי היה יוצא מהתסבוכת, שעדיין לא היתה קיימת לפני משפט בל. אבל עדיין עומדת השאלה: מדוע כה רבים לא מסמפטים את מכניקת הקוואנטים? מדוע לחפש כמו איינשטיין אמת עמוקה יותר, דטרמיניסטית, מדוע לא פשוט לקבל את האופי ההסתברותי של המציאות ולסגור עניין?

    עד כאן. בהצלחה עם מודל הפלורסנציה .

    יובל.

    המודל שלי אינו עוסק בשום אספקט פילוסופי או אקסיומטי. הוא פיזיקלי ומתמטי לחלוטין.

    ר.ח.

    “אני לתומי חשבתי שאתה מעוניין לפרסם ולדון” – רק אם יש ביקוש, ועם מי שמכיר את המאטריה. (לא מטריה!). אם אתה מעוניין, עליך לקרוא את הקישורים שהבאתי. גם אתה לא היית דן על החיסון עם מי שהיה אומר לך שבכלל אינו מבין מה הבעיה עם הגנים של הנגיפים, ובכלל צריך לסגור את כל הגנים הציבוריים ולשלוח את כל הנגיפים לגן חובה.

    ולסיום, מכיוון שמייקל פרש בינתיים, חידה מתמטית:

    משפט אי השוויון בל :

    1 + \operatorname{C}(b, c) \geq |\operatorname{C}(a, b) – \operatorname{C}(a, c)|,

    אומר בעצם ש:

    מה שיש בא’ ואין בב’, + מה שיש בב’ ואין בג’, – גדול או שווה ממה שיש בב’ ואין בג’.

    נא להראות. (לא חייבים להוכיח, אבל רצוי).

    רמז: הפתרון הפשוט ביותר הוא גיאומטרי.

  888. סטודנט
    “איך קריסת פונקצית גל של חלקיק אחד (נטרינו כפי שקראת לו) גורמת לקריסתה של פונקצית גל של חלקיק אחר (פוטון) (באיזו מסגרת אתה מתאר את האינטראקציה?)” – כיוונתי למסגרת של “גז יחודי קר מאוד” כפוטנציאל למשוואת גלים יחודית שמתבססת על חלקיקי נטרינו ואינטרקציה עם חלקיקי חומר אפל (על בסיס אותו גז מאוד קר). אבל עזוב, נראה לי שסתם הסתבכתי.

  889. יהודה,

    “אם הבסיס לא נכון אני לא אבזבז זמן ללמוד על “דברים אפלים”” – ברור לי שאנחנו כנראה רואים בצורה אחרת את הדרך שבה מדע עובד. אני לא פוסל תאוריות מורכבות ברמה הזו (ושנתמכות ע”י הקהילה המדעית) בלי להבין אותן – וכשאני אומר להבין אני מתכוון להתמחות. ברוח דברי פרופ’ שכטמן – למד נושא והפוך למומחה בו.
    חבל שאתה רואה את זה כירידה לפסים אישיים או זלזול (שזה אגב משהו שאני רואה אצלך כלפי הפיזיקאים בעולם).

  890. סטודנט יקר

    אתה לא יכול לשכנע אותי במילים:-
    “יהודה, את ה”חוק” שלימדו אותך בשנה הראשונה בתיכון אתה לא שוכח, אבל את מה שחוקרים אלפי מדענים אתה פוסל בלי להבין. בהחלט היגיון בריא.” סוף ציטוט.
    המדע הוא לא דמוקרטי, ולא מקבלים בו דברים לפי הצבעה, ואלפי מדענים, ואפילו אלפי סטודנטים בטכניון לא ישנו זאת.
    ולא איכפת לי להיות הבודד שמקבל את החוק הבסיסי ביותר במדע, שמלמדים אותו בצדק בשנה הראשונה בתיכון , שאומר:-

    אם התוצאות בשטח לא מתאימות לנוסחה אפילו לזרוק את הנוסחה או לפחות לתקן אותה.
    אין לשנות את הנתונים נקודה!.

    וזה שאתה רומז שאין לי הגיון בריא בגלל שאני דוגל ב”חוק” זה לא מוסיף לך
    ואתה עוד רושם אותו בזילזול במרכאות כפולות,
    אני לא אקדם שום רעיונות שמתפקידם לאחוז בנוסחאות שלא מתאימות לנתונים בשטח
    אם הבסיס לא נכון אני לא אבזבז זמן ללמוד על “דברים אפלים”
    בנוסף, חבל שאתה עובר לפסים אישיים ומזלזל במגיבים
    ודרך אגב אני סיימתי בהצטיינות את לימודי במכללת שנקר במגמה לניהול תעשייתי. כך שלמדתי “קצת” מעבר לשנה הראשונה בתיכון.
    יום טוב
    סבדרמיש יהודה

    נ. ב. אתה מכיר את מיקל?, אתה הולך ונהיה יותר ויותר דומה לו….נידמה לי שגם הוא למד בטכניון

  891. ישראל,
    מה החידות האלה? סעיף 2 אצל הסטודנט? דוגמא? איזו דוגמא?
    אני לתומי חשבתי שאתה מעוניין לפרסם ולדון אבל אם זה סודי כמו החיסון לשפעת שלי אז למה אתה בכלל מעלה את הנושא? ועוד בחידות?

    If you want to shoot, shoot

  892. ליובל
    תודה על התשובה.
    אכן לא הבנתי את המודל שלך. עדיין לא ברור לי כיצד המודל שלך עובד אבל אני בטוח שישראל ושות’ יציקו לך בשאלות נוקבות ואני אנסה להבין.
    כעת הגיע הזמן לחלק השני בניק שלי.

  893. ישראל,

    אם אתה מוטרד מתכונת הספין – אז למה לה אתה לא מחפש פירושים?

    “…ושהיא הוציאה את כל הכיף מהפיזיקה.” – לא מסכים. מההיכרות המעטה שיש לי עם תורת הקוונטים, היא מעניינת יותר משיעורים “קלאסים”.

    “מאמין גם שרבים חשים כמוני, ואיינשטיין בראש, אחרת לא היה נלחם בה כל חייו.” – ונכשל, אל תשכח.

    “אינני רואה איך זה ממש קשור לדיון שלנו פה.” – זה לא. לא ברור לי למה העלת את זה.

    “נראה לי שאתה מעדיף תכלס, וגם אני לפעמים.” – האמת שלא, אני לא מעדיף תכלס. אני אוהב ללמוד ולהבין מהיסוד עד לתוצאה. זו הדרך הנכונה לפי דעתי, ודרכי קיצור הן פספוס גדול בלמידה. אני רוצה להאמין שאתה יודע שלא בונים הבנה אמיתית לפי “תכלס”. אני זוכר שהמרצה שלי בקוונטים אמר שלדעתו צריך לעשות את הקורס כמה פעמים, כי כל פעם מבינים עומק נוסף.
    תחשוב שההבנה שלך היא גל שבאופן קלאסי לא יכול לצאת מתחומי האינטואיציה ולחדור לקופסה השחורה של המכניקה הקוונטית, אבל אם תתקרב למחסום הפוטנציאל – תוכל להתמנהר לשם עם זמן חיים מספיק ארוך שיתן לך תמונה של מה לכל הרוחות הולך בתיבה הזו.

    “יש איזה הסבר פיזיקלי לענין חוץ מ”קריסה בו זמנית של פונקציית הגל”?” – שוב אומר שתקרא שוב את הכתבה שקישרתי אליה מקודם. (איפה מוזכרת בה קריסת פונקצית הגל?)

    קישור נוסף בעניין:
    http://davidson.weizmann.ac.il/online/askexpert/physics/%D7%91%D7%9E%D7%9B%D7%A0%D7%99%D7%A7%D7%AA-%D7%94%D7%A7%D7%95%D7%95%D7%90%D7%A0%D7%98%D7%99%D7%9D-%D7%97%D7%9C%D7%A7%D7%99%D7%A7%D7%99%D7%9D-%D7%A9%D7%96%D7%95%D7%A8%D7%99%D7%9D-%D7%9E%D7%95%D7%A9%D7%A4%D7%A2%D7%99%D7%9D-%D7%96%D7%94-%D7%9E%D7%96%D7%94-%D7%9C%D7%9C%D7%90-%D7%AA%D7%9C%D7%95%D7%AA-%D7%91%D7%9E%D7%A8%D7%97%D7%A7-%D7%9C%D7%9E%D7%94-%D7%90%D7%99%D7%9F-%D7%96%D7%94-%D7%A0%D7%97%D7%A9%D7%91-%D7%9E%D7%99%D7%93%D7%A2-%D7%A9%D7%A2%D7%95%D7%91%D7%A8-%D7%9E%D7%94

    “יש לך איזה רעיון כיצד למדוד זמן הגעה של אות רדיו בדיוק של ננושניות ויותר?”
    “https://www.hayadan.org.il/astronomers-reach-new-frontiers-of-dark-matter-130112/#comment-324216” – מצטער, אין לי כל כך זמן להיכנס לעוד דיון, אני צריך להתחיל לפתח מודל בקשר לפלורסנציה למחקר שאני שותף בו.

  894. בא והולך,
    שמחתי למצוא אח רחוק. אף אני הדיוט מוחלט.
    חוששני שלא הבנת את המודל שלי. אין אצלי שני חלקיקים כי אם חלקיק אחד בלבד. אפשר להתייחס אל “החלל הריק” הנמצא בין החלקיקים כאל מעין חלקיק, אלא שמימדיו תלויים בצפיפות בה מסודרים החלקיקים. (המרכאות הן כי מה שנקרא היום חלל ריק איננו ריק בכלל אלא מאוכלס בחלקיקים בצפיפות גבוהה). בפשטנות, המודל שלי אומר כי היקום מאוכלס באינסוף חלקיקים צפידים ובאינסוף חלל ריק. “החלל הריק” אחראי לתופעות אלקטרומגנטיות, בעוד החלקיקים אחראים לגרביטציה ולחומר הבריוני. יש יחסי גומלין בין צבירים של חלקיקים לבין החלל הריק, ואלה מתבטאים בתופעת העידוש הכבידתי ובחסימה מוחלטת של האור באזורים בהם צפיפות החלקיקים גדולה עד כדי יצירת חומר בריוני.
    ולשאלותיך, בקיצור: 1) לא נראה לי שהבנת. 2) הצפיפות הבין-גלקטית משתנה בהתאם לנקודה בה היא נמדדת וגורמת לאפקט דמוי דופלר (שמביא את הפיסיקאים למסקנה, הלא בהכרח נכונה, כי הגלקסיות מאיצות).

  895. ישראל,
    את המודל שלי התחלתי לטפטף החוצה ונתקלתי בחוסר הבנה. זה ארוך ולא מתאים ל”כלל השיבר”* שעליו חונכתי.
    האם המודל שלך מתחיל מ”בראשית לא היה שום דבר, והשום דבר איין את עצמו ליש”?
    *כלל השיבר: אורכה של תגובה לא יעלה על שיבר אחד, על מנת שלא תחרוג מגבולות המסך

  896. ליובל
    כהדיוט מוחלט בעינייני פיסיקה קראתי בעניין את המודל (החלק שהודלף ממנו) שאתה מציע כתחליף לאנרגיה אפלה. ברשותך אני אנסה לסכם את המודל שהצעת בצורה פשטנית (למיטב הבנתי הקלוקלת) ולשתף אותך בבעייה שמטרידה אותי במודל שלך.

    על פי המודל שלף ישנם שני סוגי חלקיקים אחד אשר אחראי ליצירת גרביטציה. הראשון הינו החלקיק היוצר את המסה האפלה (ברשותך נכנה אותו גרביטון אפל) והשני אחראי ליצירת גרביטציה של המסה ה”נורמלית” (אשר נכנה אותו בשם גרביטון מואר). הגרביטון האפל מתנהג כפי שניתן לצפות מגרביטון וכל הגרביטונים האפלים מושכים בחדווה זה את זה. יתרה מכך, הגרביטון האפל אינו מפגין בררנות יתר בנוגע ל”משיכה” והוא מושך אליו גם גרביטונים מוארים. השינוי שאתה מציע הינו בתכונות הגרביטון המואר. גרביטון זה בניגוד לשמו דווקא מקיים יחסי דחייה אם גרביטונים מוארים אחרים. אבל כאשר יש לו אינטראקציה עם גרביטונים אפלים הוא משנה את חברבורותיו וייחסי הדחייה מתחלפים בייחסי משיכה. על פי הטיעון שלך ביקום כזה בתוך הגלקסיות (היכן שהחומר האפל קיים) אנחנו ניתקל בגרביטציה רגילה ואילו במרחב הבין גלקטי ישלטו יחסי דחייה אשר יביאו להתפשטותו של היקום.

    אם (וזה אם גדול) הבנתי נכון את המודל שלך אז זו השאלה שלי. כידוע היקום לא רק מאיץ אלא קצב התאוצה שלו הולך וגדל. על מנת שהמודל שלך יסביר את ההתנהגות הזאת מספר הגרביטונים המוארים ליחידת שטח במרחב הבין גלקטי אמור לגדול כל העת. מהו המנגנון ביקום שלך אשר מגדיל את מספר הגרביטונים המוארים ליחידת שטח? ההסבר היחידי שאני יכול לחשוב עליו הינו שהגרביטונים המוארים נפלטים מהכוכבים אל המרחב הבין גלקטי וקצב הפליטה עולה על קצב התפשטות היקום. המשמעות הינה שהמרחב הבין גלקטי אמור לההפך להיות “מואר” יותר ויותר. אבל, אם אני לא טועה (ושוב זה אם גדול) הצפיפות במרחב הבין גלקטי אמורה ללכת ולקטון עם השנים.
    בקיצור השאלות שלי הם: 1) האם הבנתי את המודל שלך בצורה נכונה 2) האם קיימת תוצאה אימפירית שהצפיפות במרחב הבין גלקטי הולכת וגדלה?

  897. סטודנט

    “אתה לא מוטרד מתכונת הספין?” מוטרד מאוד. בעיקר בלילה, כשהאלקטרונים מזמזמים עם הספין המעצבן שלהם ולא נותנים לישון. אילו אני מח”א, הייתי מפציץ את כל המנהרות שלהם, שיפסיקו להבריח מרכזי מסה באנרגיות בדידות.

    “לא תצליח להבין את המכניקה הקוונטית” מאמין שהבנתי מזמן את הדבר החשוב ביותר עבורי בנוגע למכניקת הקוונטים: שאני לא מת עליה, ושהיא הוציאה את כל הכיף מהפיזיקה. מאמין גם שרבים חשים כמוני, ואיינשטיין בראש, אחרת לא היה נלחם בה כל חייו. אם קראת את הקישור לכתבה שלי על “משפטו של החוק השני של התרמודינמיקה” אולי אתה זוכר שאצלנו בקיבוץ גל היה גל וחלקיק היה חלקיק, ושום גל לא העז להתנהג כמו חלקיק, ואלקטרונים היו חגים בשמחה סביב אמא’לה גרעין, במסילות אליפטיות חדות ומדידות.

    עכשיו, אשמח כמובן לדסקס איתך המילטוניאנים ולנגראז’יאנים, למרות שחלפו כמה שנים טובות, ובזמן שאני התעסקתי בהמילטוניאן, קרוב לודאי שאתה התעסקת בלהיוולד. אם תעלה נושא, נדון. אינני רואה איך זה ממש קשור לדיון שלנו פה. נראה לי שאתה מעדיף תכלס, וגם אני לפעמים. אבל עד כה לא זכיתי ממך למענה על שאלות שהצגתי, אפילו בכתבה זו, למרות שכ ב ו ג ר טכניון אין לי ספק שתוכל להתמודד עימן. אחזור עליהן ואוסיף:

    1. “סטודנט – אבל גם לפי הלינק ששלחת אינפורמצית קיטוב הפוטון יכולה להגיע לפוטון השני באפס זמן למרחק אלפי שנות אור.
    יש איזה הסבר פיזיקלי לענין חוץ מ”קריסה בו זמנית של פונקציית הגל”?

    2.

    https://www.hayadan.org.il/astronomers-reach-new-frontiers-of-dark-matter-130112/#comment-324216

    3. עד כמה אתה מכיר את נושא התנגשויות החלקיקים? אפקט קומפטון?

    4. יש לך איזה רעיון כיצד למדוד זמן הגעה של אות רדיו בדיוק של ננושניות ויותר?

    אם תוכל להקדיש זמן ומחשבה לשאלות אלו, אודה לך.

    ר.ח.

    אין ארוחות חינם. גם אתה לא תמכור לי את החיסון לנזלת אותו פיתחת ללא תשלום. יובל זחל וענה על שאלות לפני שהתחלנו להתכתב. התשלום הוא סעיף 2 אצל סטודנט. מה גם שיהיה לך קשה להבין במה מדובר אם לא תקרא את הדוגמה המצורפת, שרק ממנה אפשר להמשיך לניסוי.

    יובל.

    אין יותר בראשיתי ממודלי, אבל כמובן שהוא אינו יכול לסתור עובדות ידועות. מכיוון שמודלך ערוך ומוכן לקריאה, למה לא תבקש בשנית אישור מביטחון שדה להסיר את החיסיון עקב הביקוש הציבורי?

  898. ר.ח, תודה על התזכורת. אשתדל לא לצעוק. בגן קראו לי “יובל מנובל”, וזה נדבק.
    עד שישראל יגמור להבריע לבועלים (מה הוא? קבלן?) אגיד משהו.
    אני מניח שישראל, כמוני, מחפש מודל אחד של כל הפיסיקה. לכן גם לדעתו כל תופעות העולם הגשמי אמורות להיות קשורות זו בזו – לדוגמה, אי-לוקאליות בשזירה קוונטית קשורה לטמפרטורת קרינת הרקע הקוסמית כמדד לגילו של היקום.
    אך יש בינינו הבדל מסוים. ישראל נוטל מודלים קיימים ומחפש את הקשר ביניהם, בעוד שאני בונה מודל אחד המתחיל מאפס ובונה באמצעותו את הפיסיקה. בדרך, מן הסתם, אני בונה גם חלק מן המודלים המקובלים, אך אינני מקבל אותם כתורה מסיני.
    ישראל מצביע על הסתירות הקיימות בין המודלים השונים ומבקש ליישב ביניהם. כשאני רואה סתירה, אני מיד פוסל את כל הסתורים והסותרים ומביא גירסה חדשה משלי. כך אני עושה לא רק בפיסיקה, ואפשר למצוא את “טביעות האצבעות” האפייניות שלי גם בדרך בא אני מתייחס, בין היתר, לכתבי הקודש של הדתות השונות.

  899. יהודה,

    את ה”חוק” שלימדו אותך בשנה הראשונה בתיכון אתה לא שוכח, אבל את מה שחוקרים אלפי מדענים אתה פוסל בלי להבין. בהחלט היגיון בריא.

  900. ישראל,
    תסביר, אני אשמור על יובל שלא יצעק.
    מעל מהירות האור ללא שזירה? זה קשור לרעיון הזמן שלך עם שעוני טמפרטורת היקום?

  901. יובל
    משקלך בזהב. אני חדש יחסית באתר, לא קראתי את הכתבה של מריוס והתגובות, רק זוכר שאוהב אותו אני עוד מתקופת גליליאו המודפס. הולך להבריע לבועלים, נקרא כשנחזור.

  902. ישראל,

    אתה לא מוטרד מתכונת הספין? מאפקט המנהרה? מהעובדה שמולקולה יכולה להסתובב סביב מרכז המסה שלה רק באנרגיות בדידות?
    לא תצליח להבין את המכניקה הקוונטית בדרך חשיבה קלאסית. אם אתה מוכן להתנתק מהחשיבה הזו, קרא שוב את הכתבה, אם לא, בהצלחה. (אגב, איפה מוזכרת שם קריסת פונקצית הגל?)

  903. לסטודנט
    בכל זאת
    יש חוק שלמדנו בשנה הראשונה בתיכון והוא:- אם תיאוריה לא מתאימה למדידות אז זרוק אותה אל תתקן את המדידות כדי שיתאימו לתיאוריה. כניראה שיש רבים ששכחו את השנה הראשונה בתיכון ומעדיפים לשנות את הנתונים. מוסיפים עוד מסה כדי שהס”ה יתאים לתוצאות בשטח של מהירות גלקסיות גבוהה מידי. אז נכון שאסור לזרוק תיאוריה מיד כשמוצאים בעייה, אבל סטודנט יקר , כבר שמונים שנה מחפשים את המסה ה”אפלה ” הזאת תסכים איתי שבלי הצלחה יתרה. מוכן להתערב גם איתך על גלידה שעד סוף השנה לא יגלו אותה. אני אוהב גלידות (עם קצפת!)
    לילה טוב
    סבדרמיש יהודה

  904. ר.ח.

    לא קשור לשזירה. אפשר להסביר, אך זה עלול להיות ארוך. יובל יצעק.

    יובל.

    לי יש סבלנות וזמן. אתה יכול להאריך עד כמה שנדרש. ידע אותי מראש אם אתה מעונין בכסאח או בהנהוני הסכמה. שוט.

    ואם צריך להשליך, אז משליכים!

  905. אה, כמובן.

    איך בדיוק???

    תזכור ש:
    1) once מדדת השזירה מתבטלת
    2) אתה לא יכול להשפיע על תוצאות המדידה, הן נכפות עליך ועל זה שמחזיק את החלקיק השזור השני. כך שאם מדדת 1 השני אכן יקבל 0 מהר ממהירות האור אולם אינך יכול לגרום לחלקיק שלך להיות 1 אלא רק למדוד אותו.
    אז איך תעביר מידע?
    .

  906. משחרר את הכלב, האישה עם רצועה ומחסום פה עוד מהחתונה.
    סטודנט – אבל גם לפי הלינק ששלחת אינפורמצית קיטוב הפוטון יכולה להגיע לפוטון השני באפס זמן למרחק אלפי שנות אור.
    יש איזה הסבר פיזיקלי לענין חוץ מ”קריסה בו זמנית של פונקציית הגל”?

    ר.ח.
    לא קראת על הניסוי כי לא כתבתי עליו.
    לא כתבתי כי אף אחד לא ביקש.

    ומכיוון שביקשת –

    לשלוח סיגנל עם אינפורמציה במהירות העולה על מהירות האור כמובן.

  907. ישראל,

    “לא הבנתי מה כוונתך ב”לא עובר מידע “.”
    “נראה לי שהבנתי למה התכוונת. לכך שאי אפשר לשלוח מידע – זה נכון – אבל אין פירוש הדבר שמידע לא עובר.” – מידע “ממשי” לא עובר, אין דרך “לחלץ” מידע מהמדידות.

    אגב, קישור בעניין:
    http://davidson.weizmann.ac.il/online/askexpert/physics/%D7%94%D7%A1%D7%91%D7%A8-%D7%A2%D7%9C-%D7%A0%D7%99%D7%A1%D7%95%D7%99-%D7%94-epr

    יובל,

    “…מודלים אשר אמנם נותנים הסבר לתצפיות אך לא הוכחו באופן מוחלט.” – אין דבר כזה מוכח באופן מוחלט. מודלים יכולים רק לקבל אישושים, שכן הם בסה”כ תיאור (מקורב) שלנו של המציאות. אין דבר כזה פונקצית גל, אלקטרון או פוטון, יש רק מידע שאנחנו אוספים על הסביבה ומבינים כך או אחרת.

    “…משום שיש בדבריו רמז לכך שעלינו לקחת את המודלים האלה כנקודת מוצא…” – זה לא ברור שאנחנו עומדים על כתפי ענקים? אתה באמת חושב שהמדע היה מתקדם לו כל פעם שהייתה בעיה במודל היו זורקים הכל ומתחילים מאפס?

    “אבל, בלי קשר, כדאי לדעת מה אומרים הפיזיקאים ה”מקצועיים”, כי אפשר לקבל מהם רעיונות.” – זה עשוי להיות מתאים לאנשים שמחזיקים בתכונות שהיו לאינשטיין, שרדינגר ודיראק. מכיוון שאף אחד מהמגיבים לא בעל תכונות כאלו, וסביר מאוד להניח שגם לא הקוראים, הטענה לא מתאימה. הדרך למדע ברמה גבוהה (כמו התאוריות באסטרופיזיקה שיהודה שולל כי הן לא מקובלות עליו לפי מה שלמד בשנה הראשונה בתיכון), עבור הרוב המוחלט של האנשים, עוברת דרך התמחות בצורה מסודרת – כלומר תואר ראשון, שני, שלישי, פוסט-דוק וכו’. אין קיצורי דרך. לא ממציאים מודל לתיאור היקום לפני שלומדים את הבסיס לפיזיקה המודרנית. וגם, אין פיזיקאים “מקצועיים” ופיזיקאים “לא מקצועיים” – יש פיזיקאים ויש לא פיזיקאים (או מדענים ולא מדענים), כאשר הראשונים מקדמים את הפיזיקה (המדע) והאחרונים לא.
    לי המשפט הזה נשמע כניסיון להמעיט בערכה של המתודה המדעית, כאילו שהעבודה שהפיזיקאים ה”לא מקצועיים” (מי הם?) עושים היא שוות ערך לעבודה שעושים “פיזיקאים מקצועיים” (תקן אותי אם לא הבנתי נכון). זה חבל להציג כך את המדע. בני נוער שקוראים כאן עוד עלולים להאמין בזה ולחשוב שלגיטימי לדלג על לימודים גבוהים ולעסוק בהמצאת תאוריות משלהם.

  908. ישראל
    “אני מוציא את הכלב והאשה להליכה בהרים,” – תגיד לי, אתה משחרר אותם שם או רק כשאתה חוזר הביתה?

  909. מה, זה לא ברור?
    אני מוציא את הכלב והאשה להליכה בהרים, נדון כשנשוב.

  910. ישראל? אפילו אשתי לא מרננת אחרי ככה..

    אז עכשיו אחרי שכולנו קראנו והבנו את בל אספקט וניק הרברט (טוב, ברמה כלשהיא, לפחות הבנו שזה מאד מעניין, אה נכון , בעצם את זה ידענו גם קודם) וקראנו על שבע הפלאות הכי לא מוסברות במכניקת הקוונטים:
    http://www.newscientist.com/special/seven-wonders-of-the-quantum-world

    אז אולי במטותא ממך תסביר כמו קיפוד (כלומר לאט ובזהירות) מה הניסוי שאתה רוצה לעשות ומה הוא יראה?
    אל תתרגז, אני יודע שכתבת את זה 100 פעמים לפחות אבל זה טבוע אי שם בהררי המלל של אתר הידען אז מה אכפת לך לנסות שוב?

  911. יהודה
    נכון, אבל מקסוול פיתח את תיאוריית פרדי על שדות כוח ולא להיפך. אז יש גם עניין של כמות הגאונות.
    חוץ מזה שמקסוול מת מאוד צעיר, בעוד שפרדי מת מאוד שעיר, כך שהכול מתאזן.

    יובל.
    כבר דנו בעניין. מצב המקטבים נקבע רק אחרי שהפוטונים יצאו לדרכם, כדי למנוע קומבינות בין המקטבים.

    אבל כפי שאמרתי, ומודה שהשארתי לך את ההתעמקות בבל, העניין הוכח מתמטית. הניסוי בא רק לאשר את התאוריה.

  912. כאמור, אני הדיוט שבהדיוטות והחלטתי לקבל ללא עוררין את דבריך כי הדבר הוכח מתמטית. אולם הבחינה הנסיונית לוקה בחסר. הריני מעתיק-מדביק מן הגרסה העברית:
    רוב הניסויים נערכו על אור מקוטב ולא על הספין של אלקטרונים כפי שמוצע במסמכים המקוריים, מכיוו שניסויים באור מקוטב קלים יותר ליישום. תכונת הקיטוב של האור דומה במובנים רבים לתכונת הספין של אלקטרונים, כאשר הזויות של המקטבים הנן מחצית הזויות המקבילות של גלאי הספין.
    ישנם ניסויים בעלי ערוץ אחד שבו יש גלאי אחד בכל צד. בניסוי כזה מניחים שלכל פוטון או אלקטרון שלא נקלט בגלאי ישנו הערך הקוונטי ההפוך מזה של אלה שנקלטים בגלאי. הנחה כזאת יכולה להכניס אי דיוקים לניסוי.
    אפשרות אחרת, קצת מורכבת יותר, היא ניסוי עם שני ערוצים שבו ישנם שני גלאים בכל צד שמודדים את שני הערכים הקוונטיים האפשריים.
    הבעיה העיקרית בניסויים האלה היא היעילות הנמוכה של הגלאים, שכאשר מכניסים אותה לחישובים הסטטיסטיים מורידה באופן משמעותי את אמינות התוצאות.

    ואני שואל: מניין לנו שמקטבים אינם משפיעים על המרחב שסביבם?

  913. למה תגובתי ממתינה לאישור???
    לא ראיתי שכתבתי מילה אסורה
    יום טוב
    סבדרמיש יהודה

  914. ישראל
    הבאת דוגמא של פראדי ומקסוול
    מקסוול שהוא מושלם- גם גאון וגם ידע ופראדי שהוא רק גאון בלי ידע
    האחוד שלהם הוא גאון עם ידע שזה מקסוול.
    כשאתה אומר:- “התכוונתי לכך שצריכים להיות גם מיכאל פרדי, הגאון האינטואיטיבי חסר ההשכלה המתמטית, וגם מקסוול, שהיה לו את שניהם.”, אתה מתכוון למעשה שצריך להיות רק מקסוול.
    זה לא מקובל עלי!
    ליעל
    31.12.2012 אנו אוכלים גלידה, אני מקווה שעל חשבונך
    אבל אם זה יהיה על חשבוני, אשמח עם כל עולם המדע !
    ודרך אגב אני אוהב עם קצפת!
    יום טוב
    סבדרמיש יהודה

  915. ישראל שפירא,
    רובנו כאן הדיוטות. “לפי הבנתי העניין הוכח מתמטית” לא אומר לי הרבה אם אינך מביא את ההוכחה גופה או לפחות קישור אליה.

  916. נראה לי שהבנתי למה התכוונת. לכך שאי אפשר לשלוח מידע – זה נכון – אבל אין פירוש הדבר שמידע לא עובר.

  917. סטודנט
    לא הבנתי מה כוונתך ב”לא עובר מידע “.

    יש לנו אלקטרון א’ בנקודה א’ ואלקטרון ב’ בנקודה ב’. כשמודדים את הספין של אלקטרון א’, מוצאים אותו במצב למעלה או למטה. אם לא עובר מידע לאלקטרון ב’, כיצד הוא יודע לבחור בספין ההפוך?

    ולמה חשבו איינשטיין ונתן רוזן וכל מי שכותב על הנושא שמידע כן עובר?

    כפי שציינתי, אני שואל את יהודה רק על לה סאז’. שזירה הייתה רק דוגמה. השאלה על אינרצית הגלגל הופנתה אליך.

    כמה פרופסורים ענו על שאלותי, באריכות ופרטנות. שכטמן מעודד לשאול אותם.

    לילה טוב.

  918. אכן, תקול התקלתני תקל. נכון, אין לי לינק למקטב באנדרומדה, וגם אני תהיתי לא פעם על העניין. ואף על פי כן, ויקיפדיה תקיפה בדעתה: “the entangled pair may have been separated by arbitrarily large distances.” וגם בספר שלי “ENTANGLEMENT” כתוב מאחורה: CAN TWO PARTICLES BECOME INEXTRICABLY LINKED…EVEN IF A UNIVERSE SEPARATES THEM?

    מודה, לא עשיתי ניסויים מעבר לשביל החלב.

    אבל לפי הבנתי, העניין הוכח מתמטית.

  919. ישראל,

    אני מדבר על פרדוקס EPR, שהוא דוגמה למה שנקרא שזירה קוונטית. נתן רוזן עלה על התופעה כשהיה צעיר (22), כשחישב את רמות האנרגיה של מולקולת מימן. הוא הסיק מפתרון משוואת שרדינגר, שתחת אילוץ סימטריה מסוים (עקב חוסר יכולת ההבחנה בין שני האלקטרונים), מקבלים שגם במרחק אינסופי כאשר האטומים מופרדים והפונציאל ההדדי מתאפס, פונקצית הגל צריכה לקיים את אותה סימטריה. מעקרון הספרביליות ידוע שפונקצית הגל של המילטוניאן פריק היא מכפלה של פונקציות גל, אך באילוץ הנ”ל מתקבל שהיא סכום.

    בכל אופן, לא עובר מידע בתיאור הפרדוקס הזה ודומיו (זה לא באמת פרדוקס, השם נשמר למיטב ידיעתי מסיבות היסטוריות). חפש על כך בספרות.

    “האם ברור לשנינו כי ניתן לשזור אלקטרונים ( או פוטונים) רבים, לפזר אותם ביקום בנקודות שונות ומרוחקות מליוני שנות אור זה מזה, ושקריסה באחד מהם תביא לקריסה מיידית בכל השאר?” – אני לא יודע, אני לא מתמצא בפיזיקה ניסיונית.

    “אני רוצה לראות אם אותו פתרון כולל את בעיית האי לוקליות. זה הכל.” – אתה משתמש במושגים שזרים לו, הוא אמר.

    “אינטואיציה. כן אני חושב שזה הכרחי. תוכל להסביר לי מה קורה בדוגמת האופנוע עם הגלגל המניע באויר? מדוע כל קשה לנענע אותו לצדדים כשהגלגל מסתובב, למרות שהאופנוע במהירות 0 יחסית לאדמה?” – אני לא יודע כרגע על איזו דוגמה אתה מדבר, אבל זה לא רלוונטי. אתה מצפה שלהדיוטות תהיה אינטואיציה בתורת הקוונטים – וזה מגוחך.

    ר.ח רפאי.ם,

    “זה ככה בגדול (עם כמה נתונים חסרים) נשמע לך הגיוני? משהו בכיוון?” – אני לא מבין בפיזיקת חלקיקים, ואני נוהג שלא להיכנס לנושאים שאני לא מבין בהם. אוכל רק לציין שלא ברורה הכוונה באורך גל 0, ואיך קריסת פונקצית גל של חלקיק אחד (נטרינו כפי שקראת לו) גורמת לקריסתה של פונקצית גל של חלקיק אחר (פוטון) (באיזו מסגרת אתה מתאר את האינטראקציה?)

    ישראל (תגובה שניה),

    אין לי כרגע זמן להציץ שם, אבל אמליץ לך לא לפתח ציפיות. פרופסורים וחברי סגל הם אנשים עסוקים. מאוד.

    “ספין וקיטוב נקבעים רק עם המדידה, ועוברים בדרך פלא לאלקטרון או הפוטון השזור” – תכונות לא עוברות. לתכונה כמו ספין אין משמעות עד שמדדת אותה. אין מציאות אובייקטיבית בה תכונת הספין קיימת בלי קשר למדידה.
    חשוב גם לציין שמדידות שמתארות את ה”פרדוקס” צריכות להיות על גדלים שמיוצגים ע”י אופרטור שלא מתחלף עם האנרגיה, אחרת התופעה יכולה להיות מתוארת באופן קלאסי.

  920. ישראל, שמע נא (עוקף הגדרת מחבת או סיר לחץ),
    הריני עוקב אחר התפתחותך, ומתמוגג מנחת. כעת אתה מתמקד בנושא אחד, וטוב שכך. מה שמפליא אותי נכון לעכשיו זה כיצד לא שמעתי בחדשות שמישהו הציב גלאי פוטונים באנדרומדה. אנא קישור, אם תואיל.
    וברצינות, המרחקים בהם בוצעו הניסויים אינם גדולים. אמנם במהלך הניסויים נעשו סידורים שמטרתם למנוע את האפשרות להשפיע על התווך בין שני הגלאים, אולם היות שאין יודעים עדיין כיצד התווך עצמו פועל (אתה מדבר, למשל, על “אֶתֶּר פעיל”), אי אפשר לדעת אם אמצעי האבטחה אכן מספקים את הסחורה.

  921. ר.ח. – הנה הקישור.

    אבל בתור עונש, אם תצליח להגיע עד לסוף: מדוע אין ההוכחה שלמה?
    רמז: יובל כבר העלה את זה בתגובה קודמת.

    אריה – ברור שהכוונה הייתה לקרינה אלקטרומגנטית שמתפשטת כמעטפת. לא ניתן למקד שזירה קוואנטית כמו קרן לייזר. עובדה שהיא פועלת בכל נקודה ביקום, שלא כמו קרן שלנו, אותה ניתן למקד ואיתה לרקד.

  922. ישראל,

    תודה על ההסבר. אני אחפש הסבר לאי שוויון בל וניסוי אספקט (?) בשפה שאפילו ביולוג יכול להבין. אם יש לך איזה לינק נחמד תשלח.

    תפילות? באיזה סגנון?

  923. אולי זה לא כל כך רלוונטי לדיונים המעמיקים כאן, אבל אין זה נכון לומר שהגלים האלקטרומגנטיים נחלשים. אנו עדים להיחלשותם כפונקציה של המרחק, בגלל שהקרינה מתבדרת ואז עוצמתה ליחידת שטח יורדת. לו למשל הייתה קרן לייזר מרוכזת לגמרי ולא מתבדרת – אז עוצמתה לא הייתה משתנה עם המרחק (או הזמן).

  924. ר.ח.

    עצם זה שחזרת אלינו מראה שתפילות לפעמים עוזרות.

    שזירה. הדילמה שתיארת הועלתה פעמים רבות, כדילמת הפרופסור המפוזר. אם אתה רואה אותו עם גרב צהובה, אתה יודע מיד שהשניה כתומה, ולהפך.

    אז קודם בוא נראה איך זה מסתדר עם פוטונים שזורים. שם הקיטוב זהה – אם אצל פוטון א’ קיטוב מסוים, אז לאחיו אותו קיטוב.

    איך, כיצד? איינשטיין העלה את הרעיון של “משתנים חבויים” – כמו אצל הפרופסור או הקופסאות שלך, הנתונים היו קיימים כבר מקודם, הצפיה רק גילתה אותם. אותו דבר אצל הפוטונים.

    אולם בקוונטים אין משתנים חבויים, רק סופרפוזיציה. הצפיה עצמה גורמת לפונקצית הגל לקרוס, כמו מטבע באויר שנמצא בסופרפוזיציה של עץ ופלי, ורק כשתופסים אותו הוא בוחר מצב אחד ויחד. אותו הדבר עם סביבון מסתובב, שנופל רק על אות אחת.

    בשזירה, עם שני סביבונים שזורים, הרי שאם אחד נופל על נ’, כך גם אחיו.

    עד בל, שני ההסברים של איינשטיין ושל בור, נראו קבילים וללא אפשרות להכרעה. אי שוויון בל, ובעקבותיו ניסוי אספקט, הוכיחו שבור והקוונטים צדקו ולא איינשטיין. ספין וקיטוב נקבעים רק עם המדידה, ועוברים בדרך פלא לאלקטרון או הפוטון השזור, באפס זמן ובכל מרחק, ואפשר גם לשזור חלקיקי יסוד רבים. אם תרצה, אחפש לך את הקישור שמסביר את הניסוי, או אעתיק לך אותו. (ארוך, ר.ח., ארוך).

    יהודה – אם פירשת את דברי על אינטואיציה כהקלה בצורך ללמוד את הפיסיקה הידועה, לא זו היתה הכוונה. התכוונתי לכך שצריכים להיות גם מיכאל פרדי, הגאון האינטואיטיבי חסר ההשכלה המתמטית, וגם מקסוול, שהיה לו את שניהם.

  925. ישראל,

    יש משהו שאף פעם לא הבנתי בקשר לשזירה הזו. להבנתי בתהליך היצירה של האלקטרונים השזורים תמיד ייוצר אחד “למעלה” והשני “למטה”, נכון עד כאן? אז אם כן מה הקסם הגדול?
    נניח שאני שם שני כדורים, אחד לבן ואחד שחור בשתי קופסאות אטומות. אחת מהן אני שולח אלייך לגלקסיית אנדרומדה. אתה פותח את הקופסא ומגלה שהכדור לבן מייד אתה יודע לתדהמתך שזה שנשאר בכדור הארץ הוא שחור! ואכן כשנפתח את הקופסא שנותרה פה, הפלא ופלא , יש בה כדור שחור. האם האינפורמציה עברה במהירות אינסופית? מה ההבדל בין זה לניסוי השזירה?

  926. סטודנט
    מעניין שהזכרת את קריסת פונקצית הגל כי חשבתי על זה בהקשר לניסוי שבוצע לא מזמן על הנטרינו.
    בגדול, הגעתי למסקנה (אולי היא לא נכונה, לי זה נראה די מסתדר למרות שחסרים עדיין כמה פרמטרים)
    שהנטרינו מהווה סוג של תווך שעליו נע הפוטון, כמו ‘חבורת גלים’ יחודית שעליה נע פוטון. כלומר פונקצית גל של ניטרינו קודמת לפונקצית גל של פוטון, בגלל שסופרפוזיציה של נטרינו קורסת (בזמן מדידה של מהירות) -והפוטון נמצא על אותו מסלול של נטרינו- אז גם הס”פ של פוטון קורסת, כמעט באותו זמן אבל על אותו מסלול במרחב כמו של נטרינו, ופונקצית הגל של פוטון משתנה (אורך הגל שווה 0 ). בקיצור, קריסת פונקצית גל של פוטון קוראת בזמן מדידה, אבל רק אחרי שבאותו איזור במרחב עבר נטרינו. זה ככה בגדול (עם כמה נתונים חסרים) נשמע לך הגיוני? משהו בכיוון?

  927. ליעל
    לא ימצאו ב2012 ולא ב 3012 את החלקיק האחראי על המסה האפלה??:,
    השקיעו כבר מיליארדים בחיפושים אחריו, מחפשים אותו משנות השלושים של המאה שעברה, מאז שהמושג מסה אפלה עלה למודעות הציבור.
    לא ניראה לי שהוא קיים.
    מוכן להתערב איתך על כך. (גלידה!)
    חוץ מזה, ביננו, את באמת מאמינה למושג אנרגיית הוואקום??
    לכלום יכול להיות משהוא??
    על בריחת ההגיון זו מהתודעה המדעית מדברים יובל וישראל.
    אני בטוח שאי שם מסתתרת התורה ההגיונית למציאות הפיזיקלית קוסמולוגית, והמדע עוד ימצא אותה.
    המסה והאנרגיה האפלות ישכנו במקום כבוד ליד הפלוגיסטון, הקלוריק וקרינת N !
    ערב טוב
    סבדרמיש יהודה

  928. סטודנט

    אינני יכול להמנע מן ההרגשה שאיננו מדברים על אותו הנושא.

    רק כדי לוודא: האם ברור לשנינו שמדובר לדוגמה על שני אלקטרונים שזורים, אחד בחדר מבודד בת”א, השני בחדר דומה בגלקסיית אנדרומדה. כל עוד לא נמדד הספין, שניהם במצב של סופרפוזיציה של ספין “למעלה” ו”למטה”. ברגע שמודדים אחד מהם, מגלים שהספין של שניהם הפוך תמיד. ההסבר היחיד (לפי בור, איינשטיין, בל ואספקט) הוא שאינפורמצית הספין עברה מיידית מאלקטרון א’, את כל המרחק, חדרה דרך הכוכבים, והגיעה לאלקטרון ב’?

    האם ברור לשנינו כי ניתן לשזור אלקטרונים ( או פוטונים) רבים, לפזר אותם ביקום בנקודות שונות ומרוחקות מליוני שנות אור זה מזה, ושקריסה באחד מהם תביא לקריסה מיידית בכל השאר?

    יהודה. אינני מבקש ממנו הסבר על קוונטים – רק על הבייבי שלו, תאוריית לה סאז’. בפירוש ציינתי: “איני מבקש את פתרון התעלומה כמובן. רק את התכנות הפתרון,”. יהודה טוען שיש לו פתרון סודי לבעיית החיכוך אצל ל.ס. אני רוצה לראות אם אותו פתרון כולל את בעיית האי לוקליות. זה הכל.

    אינטואיציה. כן אני חושב שזה הכרחי. תוכל להסביר לי מה קורה בדוגמת האופנוע עם הגלגל המניע באויר? מדוע כל קשה לנענע אותו לצדדים כשהגלגל מסתובב, למרות שהאופנוע במהירות 0 יחסית לאדמה?

    אם ההסבר שלך כולל מילים כמו “אינרציה” ו”תנע זוויתי”, אז זה לא מספיק לי. הסבר תכלס: מה תופס את הגלגל? אם אתה מאמין שיש לך הסבר פשוט, אתה במחלוקת עם ניוטון ( הדלי המסתובב), מאך ואיינשטיין.

    יובל – לא נעלבת שקראי לך מחב”ת נכון? זה בגלל ה: “שמע , ישראל – אתה אדוני אלוהינו אלוהים אחד!”

  929. למר סברדרמיש,

    התחזית היא שהחלקיק האחראי על ה”מסה האפלה” ימצא השנה ב-2012. כפי שמנסה “סטודנט” להסביר: הבעיה מעניינת בהרבה מנוסחאת ניוטון לכבידה. אולי כדאי לך גם להסתכל מעט על “אנרגיית הואקום” ואפקט קאזימיר שהוכח בנסוי רק בזמן האחרון.

    וליובל,

    אני בזמן האחרון מנסה ללמוד את החידושים שנוגעים לחקר המרחב… יש כמה מאמצים ותובנות מעניינים בתחום הזה.

  930. סבדרמיש יהודה,
    אני, במקומך, לא הייתי ממהר לענות לסטודנט, טכניון. השאלה שלו הייתה פרובוקטיבית משהו, ואצלך נשרפו לא מעט פיוזים. תשובתך, בהתאם, הכילה כמה וכמה פרובוקציות עסיסיות.
    הוא דיבר על הצורך לדעת את יסודות הפיזיקה המודרנית, וכלל בזה מודלים אשר אמנם נותנים הסבר לתצפיות אך לא הוכחו באופן מוחלט. בנקודה הזו אני חושש שהוא שוגה, משום שיש בדבריו רמז לכך שעלינו לקחת את המודלים האלה כנקודת מוצא, ובאופן הזה אנו עלולים להחטיא את האפשרות למצוא מודלים אחרים אשר אולי יהיו טובים מהמקובלים.
    אבל, בלי קשר, כדאי לדעת מה אומרים הפיזיקאים ה”מקצועיים”, כי אפשר לקבל מהם רעיונות.
    אני רואה שמזה זמן רב אתה נלחם בחומר האפל אבל מאמין בקיומם של חלקיקים כלשהם. לא מן הנמנע שהחלקיקים שלך והחומר האפל הם אותו דבר בשינוי שם.

  931. יהודה,

    “אני לא חושב שאני צריך ללמוד את כל הדברים שלא מקובלים עלי” – אז איך אתה “מחליט” שזה לא מקובל עליך? אתה חושב שאתה מבין את התאוריות הקיימות בקוסמולוגיה?

    יש לך בעיה מאוד בסיסית בהבנה של מה זה מדע בכלל ואיך הוא מתקדם. אציג את השגיאות שלך באנלוגיות שנתת:
    “אתה למשל היית לומד עשר שנים לימודי ישיבה לפני שאתה תחליט אם יש אלוהים או אין?” – ההבדל הוא שהמדע לא נתון לאמונה. אם אתה מאמין או לא בתורת היחסות או בתורת הקוונטים לא משנה את נכונותן.

    “אני מבין שאצלך יצביעו לכנסת בבחירות חופשיות אומנם, אבל רק כאלה שיהיה להם תואר דוקטור באזרחות, כי לאחרים אין ידע מספיק בנושא!” – ההצבעה ניתנת לכולם בשל עקרונות דמוקרטיים. במדע אתה לא מצביע מה נכון. יש מודלים, יש ניסויים ויש אישושים.
    אגב, יש אנשים שאכן חושבים כך, ששיטת הבחירות הדמוקרטיות היא לא טובה מהסיבה שלא כולם “ראויים” להצביע. אין לי דעה בנושא, אבל אציין נוסח של האמרה הידועה מאת צ’רצ’יל: הדמוקרטיה היא לא צורת השלטון הטובה ביותר, אלא ההכי פחות גרועה.

    “אני לא מתיימר להביע דיעה בדברים לא מוכרים לי ואיני עונה למשל על שזירה קוונטית או תורת המיתרים.” – ובאסטרופיזיקה אתה מבין? הרי אתה לא מביע דעה, אתה פשוט שולל את אחת התאוריות המורכבות ביותר בפיזיקה היום ומנסה “לפתח” תאוריה משלך, בלי לדעת מה אתה שולל בכלל.

    “לימדו אותי בשנה הראשונה בתיכון שאם המדידות לא מתאימות לנוסחה יש צורך לחפש נוסחה אחרת.” – עצוב לשמוע שככה אתה חושב שעובד המדע.

    “בקיצור, השנה הראשונה בתיכון הספיקה לי,אין לי צורך לשם כך ללמוד על מסה אפלה עשר שנים.” – אז אתה ממש יהיר או ממש תמים. אתה באמת חושב שהשנה הראשונה בתיכון תוביל אותך לפיתוח תאוריה לתיאור היקום?

    “תקבל 13 דקות הסבר על דעתי בנושא, לא עשר שנים!” – אני כבר שמעתי חלק מההרצאה שפרסמת כאן פעם (זו שבאורך ~שעה וחצי). אני מצטער לומר לך, אבל בין מה שאתה מרצה שם לבין מדע אין שום קשר.

  932. לסטודנט
    תגובתי משעה 2.16 אחר חצות אושרה. אשמח לשמוע דעתך על הדברים
    יום טוב
    סבדרמיש יהודה

  933. “כיצד משפיעה קריסת פונקצית הגל בנקודה א’ על קריסתה בנקודה ב’, במרחק עצום, באפס זמן.” – פונקצית הגל מתארת את המערכת ובעת מדידה היא קורסת לערך עצמי מסוים של האופרטור המתאים למשתנה אותו מדדת. אם אתה מקבל את רעיון קריסת פונקצית הגל, אני לא מבין את השאלה. לפני המדידה יש אי ודאות במצב הזהות בשל התיאור הגלי ואחריה יש ודאות מוחלטת בשל המעבר לתיאור קלאסי.
    אגב, יש עבודות ומאמרים בתחום, ואפשר לקבל התאמה בין מכניקה קלאסית לקוונטית תחת שינויים מסוימים, למשל בהמילטוניאן התלוי בזמן.

    “גלים אלקטרומגנטים נחלשים עם המרחק, לא כן פונקצית הגל.” – אני לא מבין כל כך את כוונתך ל”החלשות” וההקבלה לגלים אלקטרומגנטים. קח למשל את פונקצית הגל המקורבת לאלקטרונים באטום הליום – להחלשות של מה אתה מתכוון ומה אתה מקביל למה?

    “קריסה באלקטרון בודד יכולה תיאורטית להביא לקריסה באלקטרונים ביקום כולו.” – לא מבין את הטענה.

    “אם נניח שהקריסה צורכת מעט אנרגיה, איפה חוק שימור האנרגיה?” – באינטראקציה מהמדידה? אני לא מכיר את הנושא ברמה מספיק גבוהה כדי לענות על השאלה, אבל סביר להניח שיש ניתוחים כאלו, למשל במסגרת תורת ההפרעות.

    “לכן, כדי שלא נסטה מהאינטואיציה ל”שדות מרוכבים” ו”מרחבי על מרובי ממדים”, כדי להסביר מה קורה לאלקטרונים פשוטים חביבים ויומיומיים, אני לפחות תמיד מקשיב לבעלי אינטואיציה פיזיקלית פשוטה” – לטעמי לא תגיע בדרך כזו לשום מקום. אלקטרונים פשוטים ויומיומיים? אין שום אינטואיציה באלקטרונים והתנהגותם. אתה למשל לא יכול לדמיין תכונה כמו ספין, להבין את העובדה שאין חלקיק ואין גל – יש גל-חלקיק, או לתפוס אינטואיטיבית איך אלקטרון יכול להיות בשני קצוות בלי לעבור באמצע. איזו אינטואיציה אתה רוצה לשאוב מיהודה בדיוק? אפילו אם באיזשהו שלב בחיים האקדמיים פרופסורים מגיעים לרמת הבנה אינטואיטיבית בתורת הקוונטים – זה לאחר היכרות עמוקה של עשרות שנים עם התחום – ממש לא המצב של יהודה.
    אגב, היה זה פיינמן שטען כי אף אחד לא מבין קוונטים. אני יודע שזה נוח לחשוב שיש תיאור פשוט, הגיוני ואינטואיטיבי לכל דבר – אבל זה פשוט לא כך. יהודה יכול להמשיך להמציא תאוריות עד אחרית הימים, זה לעולם לא יהיה שווה לתאוריה מדעית אמיתית שמבוססת על מחקר וידע אמיתי.

  934. על אינטו ועל איציה ועל מתמטיקה בכלל
    היות שפעולות חשבוניות זהות ניתנות להחלה בהצלחה עקבית על עצמים השונים זה מזה בתכלית (כגון זמן, מרחק, משקל, כסף במכולת, כסף בבנק ועוד), יכולנו להסיק כי כל אלה הם בעצם דבר אחד. אולם היות שלא הצלחנו לראות את הזהות הזו במו עינינו, טבענו את השם “מתמטיקה” ואנחנו מניחים, ללא הצדקה – לבד מן ההצדקה הנסיונית, כי אותם החוקים של המתמטיקה חלים על כל העצמים בעולם.
    עם חלוף הדורות גילינו לדעת כי הדברים מורכבים יותר וכי ישנם עצמים שהתנהגותם המספרית-כמותית איננה בדיוק כמו של עצמים אחרים. כדי לא להמציא חוקים מתמטים חדשים, החלטנו להגיד כי התופעות החדשות אינן אלא התופעות הישנות ברמת מורכבות גדולה יותר. כך, למשל, תורת היחסות איננה פיסיקה חדשה אלא הרחבה של הפיסיקה הניוטונית (או שהפיסיקה הניוטונית היא הפיסיקה האיינשטיינית עם איברים מנוונים).
    כעת, בבואנו לדון בשזירה הקוואנטית, אנו עומדים בצומת דרכים. האם זו תופעה חדשה לחלוטין שאינה נענית לכללי המתמטיקה או שהיא בעצם הרחבה של תופעה אחרת, ידועה, וכל מה שעלינו לעשות לצורך הבנתה זה רק להוסיף משוואות שבמצבים אחרים הן מנוונות.

  935. בשתיים בלילה שלחתי תגובה לסטודנט . למה היא עדין ממתינה לאישור?
    סבדרמיש יהודה

  936. סטודנט
    אין שום בעיה עם השזירה – הבעיה היא אי הלוקליות. כיצד משפיעה קריסת פונקצית הגל בנקודה א’ על קריסתה בנקודה ב’, במרחק עצום, באפס זמן. גלים אלקטרומגנטים נחלשים עם המרחק, לא כן פונקצית הגל. קריסה באלקטרון בודד יכולה תיאורטית להביא לקריסה באלקטרונים ביקום כולו. אם נניח שהקריסה צורכת מעט אנרגיה, איפה חוק שימור האנרגיה?

    אנא, לא להקל ראש ביהודה ותגובותיו. מלבד הסחבקיות (ראה הצהלה הכללית על שובו בכתבה זו), הוא מהווה (לפחות עבורי), ניר לקמוס לסטיה מהאינטואיציה, שהיא הבסיס לתובנה פיזיקלית לפי פיינמן. אם תנסה להכפיל 9993 ב10007 עומדות לפניך כמה אפשרויות. אם אתה בעל אינטואיציה מתמטית כמו מייקל, תשים לב לכך שמדובר ב (7 +10,000) (7- 10,000), ומכיוון ש (A+B)(A-B) שווה A^2 – B^2 אז נקבל 100,000,000-49 שהם99,999,951. ישועת השם כהרף עין. בני דורך ישתמשו במחשבון, ובני דורי בלוגריתמים. יובל ופיינמן יגידו לך שהם לא מבינים מה הבעיה, קח 10,007 חביות, מלא כל אחת ב9,993 ג’ולות, ערבב הכל וספור. פשוט מאוד,מה הסיפור בכלל? והם כמובן צודקים. בשורה התחתונה, מעבר לתעלולים המתמטים, כפל הוא פעולה של סכימה.

    לכן, כדי שלא נסטה מהאינטואיציה ל”שדות מרוכבים” ו”מרחבי על מרובי ממדים”, כדי להסביר מה קורה לאלקטרונים פשוטים חביבים ויומיומיים, אני לפחות תמיד מקשיב לבעלי אינטואיציה פיזיקלית פשוטה, כמו שבמשפט אמריקאי קובעים מושבעים ולא שופט מומחה כמו בישראל.

  937. סטודנט טכניון
    אני לא חושב שאני צריך ללמוד את כל הדברים שלא מקובלים עלי
    אתה למשל היית לומד עשר שנים לימודי ישיבה לפני שאתה תחליט אם יש אלוהים או אין?
    אני מבין שאצלך יצביעו לכנסת בבחירות חופשיות אומנם, אבל רק כאלה שיהיה להם תואר דוקטור באזרחות, כי לאחרים אין ידע מספיק בנושא!
    אנחנו חיים בעולם שאנו יכולים לדעת בו בצורה מושלמת לכל היותר רק חלק זעיר, לכן, את זה עלינו לנצל כדי לקבוע עמדה בבדיקת בעיות שמעניינות אותנו.
    אני לא מתיימר להביע דיעה בדברים לא מוכרים לי ואיני עונה למשל על שזירה קוונטית או תורת המיתרים.
    לימדו אותי בשנה הראשונה בתיכון שאם המדידות לא מתאימות לנוסחה יש צורך לחפש נוסחה אחרת.
    למה לא עושים זאת לגבי גרביטציה?, למה אנחנו מנסים לכופף את המדידות עם כל מיני תוספות אפלות?, מפחדים מניוטון?
    בקיצור, השנה הראשונה בתיכון הספיקה לי,אין לי צורך לשם כך ללמוד על מסה אפלה עשר שנים.
    אם תרצה כנס ל http://www.youtube.com/watch?v=kAo5BQQpBqQ
    תקבל 13 דקות הסבר על דעתי בנושא, לא עשר שנים!
    לילה טוב
    סבדרמיש יהודה
    לילה טוב
    סבדרמיש יהודה

  938. לא הייתי ממהר לומר ש”זו התעלומה הגדולה ביותר שניצבת בחזית הפיזיקה”. זו אכן “תופעה פרדוקסלית” בה עצם המדידה קובעת את המצב הקוונטי. אחד ההסברים לפרדוקס הזה נשען על קריסת פונקצית הגל, והמעבר ממצב קוונטי בו המערכת מתוארת כצירוף לינארי של מס’ פונקציות גל למצב קלאסי דטרמיניסטי. ניסויים שבוצעו הוכיחו את נכונות התפיסה הקוונטית (וסתרו את אי שיוויון בל), בה תכונות כמו ספין לא קיימות באיזשהי מציאות אובייקטיבית, והמדידה היא שמעניקה את הזהות.
    קריסת פונקצית הגל הוא אמנם נושא פתוח בתורת הקוונטים, אבל אתה לא יכול לצפות מיהודה, שבכלל לא מבין על מה מדובר, לפתור בעיה שכזו או לחשוב על פתרון שיהיה לגיטימי.

    אגב, לא מומלץ להסתמך על מה שכתוב בויקיפדיה (בטח שלא העברית) בנושאים ברמה של שזירה קוונטית. ספרות מדעית ומאמרים הם הכתובת.

    יהודה,
    שאלה: אתה לא מזוהה עם מושגים שלומדים בקוונטית 1, אבל מנסה למצוא חלופה לאחת התאוריות המורכבות ביותר בפיזיקה היום. חורים שחורים, סינגולריות, מפץ גדול, מסה אפלה ועוד ועוד. אתה לא חושב, למשל, שרצוי ללמוד את הבסיס לפיזיקה המודרנית לפני שניגשים לפתור את אחת הבעיות המורכבות ביותר בהן היא עוסקת?

  939. אני לא מתמצא בכל דקויות תורת הקוונטים כך שמושגים שאתם משתמשים בהם, זרים לי.
    אבל, אם הגדרת את הבעיה כפי שהגדרת, אזי אינני חושב שיכולה להיות אפשרות להשפעה בו זמנית במרחק של מיליוני שנות אור. אם יש ניסוי שמראה זאת אשמח לראותו ולמצוא פתרון.
    יכולים להיות רעיונות רק אם חלק מן אברי היקום ינוע מהר יותר ממהירות האור. למשל הזעירים ביותר אפילו יותר מנטרינים. הם יבנו מערכת חוקים שתפעל מהר יותר ממהירות האור. תאוריה שנבנתה על סמך חלקיקים גדולים לא חייבת להיות תקיפה על חלקיקים זעירים. למשל מהירות האור הקבועה.
    אני למשל לא מקבל שמהירות האור היא גודל קבוע ביקום וחושב שהיא משתנה בכמה מ”מ לשנייה בשנה אולי אפילו ס”מ לשנייה. ניסוי מיכלזון מורלי לא יכול היה לחלוטין להוכיח שמהירות האור קבועה בפונקציה של זמן כי הוא מדד את המהירות רק בנקודת זמן אחת. יש למדוד בשתי נקודות זמן ולהשוות. לדעתי יתגלה שהמהירות משתנה.
    אני לא חושב שאפשר להגיע למסקנות קיצוניות ע”י בדיקת תאוריה בנקודות שאינן קיצוניות. לדוגמא מהתנהגות נוסחת ניוטון בכוכבים להגיע למסקנות בחורים שחורים. תיאוריה מוגדרת רק היכן שהיא ניבדקה וגם שם היא מוגדרת עם אי וודאות מסויימת הקיימת תמיד במדידות.. בנקודות שהיא לא נימדדה אנו רק יכולים לשער שהיא תקיפה.
    מספיק לבינתיים
    ערב טוב
    סבדרמיש יהודה

  940. בסדר. אני מבין את הצורך בבטחוניות. רק שאלה קטנטנה, לראות אם אנחנו באותו כיוון: האם פתרונך שופך אור על סעיף 6, אי לוקליות בשזירה קוונטית? הרי זו התעלומה הגדולה ביותר שניצבת בחזית הפיזיקה: כיצד יכול ספין של אלקטרון א’ להשפיע על ספין של אלקטרון ב’ במרחק מיליון שנות אור, וזאת באפס זמן.

    איני מבקש את פתרון התעלומה כמובן. רק את התכנות הפתרון, בלי מילים כמו “27 מימדים” או “מודעות” או זבירבירולוגיה יקומית טראנסנדנטלית. תכלס, מה קורה שם.

    אם קשה לך, תשאל את דגני, הוא גר בפינת יהודה הלוי.

  941. ישראל
    האמת, לא בא לי לפרסם את פתרון בעיית החיכוך (אי”ה) שלא במאמר.
    כאן בידען כידוע לא מאפשרים לי.
    אני אנסה במקום אחר אבל זה ייקח קצת זמן.
    אולי אפרסם בסופו של דבר את זה ביו טיוב אבל זה דורש הכנה.
    בינתיים אני עסוק אז בינתיים לה סאג’ וריצ’רד פיינמן יכולים לנוח במשכבם ולא להתרגש יותר מידי.
    כמו כן נילמד את העקרונות 1-15 ונראה מה התיאוריות השונות מסוגלות להציע.
    יהיה לי סוף שבוע עמוס
    יום טוב
    סבדרמיש יהודה
    .

  942. יהודה.
    1. אז איפה פיתרון החיכוך?
    2. מדבריך: “אני זוכר שיש הסבר של מומנט סיבוב ועיקרון הג’ירוסקופ , אבל תמיד ראיתי בזה פלא.”
    אז איך פתאום “לא מבין מה הבעיה”?
    6-3 כדבריך אין באמתחתך הסבר.
    7. אה? יש לך הסבר? לשתף מיד את ישראל! וגם את איינשטיין. לו אין הסבר, אחרת לא היה קובע זאת כפוסטולט, אלא מסביר בעצמו.

    14-8 לא כל כך מצוי ברזי המסה האפלה, זה יותר תחום של יובל המחב”ת. אבל נראה לי שאם תצליח להוכיח פושינג גרביטי, כל החוקים משתנים, ויקומי פשוטי יתפוס את מקומו הראוי ביקום הפשוט. מה שמחזיר 13 שלבים ל-1: איפה פיתרון החיכוך?
    15. הנה נקודת הסכמה.

    השכבות זה משחק. נראה אותך רוכב בשדות על סער, בלי אוכף, בעמידה.

  943. 1הערותיי לשבעת הסעיפים של ישראל:
    1. גרביטציה.- פושינג גרביטי מסביר בעיקר אם יש לי פתרון לחיכוך
    2. אינרציה. אם הכוונה לחוק הראשון של ניוטון אני מקבל אותו בעינים סגורות, לא מבין מה הבעיה
    3. מגנטיות.לא טיפלתי
    4. משיכה/דחיה חשמלית.- כנ”ל
    5. גלים אלקטרומגנטים.- כנ”ל
    6. אי לוקליות בשזירה קוונטית.- לא יודע למה הכוונה
    7. קביעות מהירות האור בכל מערכת יחוס, ללא פוסטולט.- הסבר החיכוך בפושינג גרביטי עשוי להסביר את המהירות הזהה בכל כיוון.

    בנוסף יש להסביר את התופעות הבאות ולבדוק כיצד מתייחס התער של אוקהם לכל אחד מן ההסברים המוצעים:
    8 תנועת הגלקסיות המוזרה
    9 תנועת צבירי גלקסיות המוזרה
    10 עידוש
    11 התפשטות היקום המואצת
    12.המנעות מנקודות סינגולריות של המפץ הגדול ואחרות או לחילופין קבלתן
    13 אופן פיזור ההסבר בגלקסיה וביקום , אקראי או הגיוני.
    14- תכונות חלקיקי המסה האפלה מול חלקיקי הפושינג גרביטי
    15 אפשרויות הפרכה על פי פופר

    הערה: אוקהם הוא שם מקום בסקוטלנד ועל שם המקום קראו לתער. ליוצר התער קראו אם אינני טועה וויליאם- נזיר סגפן בצורה בלתי רגילה שאפילו האפיפיור בזמנו הטיל עליו חרמות

    לילה טוב
    סבדרמיש יהודה

  944. ואכן, כזהו הוא מודלי. אך למען לא ישאר בתחום האפס או הלא כלום, הוא זקוק לאחד מהשניים: משוואות מוחצות וחד משמעיות, משהו סטייל מקסוול, או ניסוי מוצלח, משהו סטייל הרץ. נכון, המודל שלך מנוסח כהלכה ומא’, ואילו אני אומר: קח את מקסוול, הכנס את השינוי הקטן וההכרחי שציינתי, והפלא ופלא, הנה קביעות מהירות האור בכל מערכת יחוס. אין צורך להעתיק את כל התאוריה, על 165 משוואותיה.

    http://en.wikisource.org/wiki/On_Physical_Lines_of_Force

    (שים לב אגב למשוואות 130 -140. בהן עסקינן).

    אותו כנ”ל לגבי מאך ולה סאז’. שמת לב כיצד נפתרת מאליה בעיית החיכוך אצל לה סאז’ באופן אלגנטי? יהודה הבטיח לנו פיתרון ביקומי פשוטי. איפה הוא?

    ניסוי. אתה לא רואה שאני מנסה בכל דרך, כולל כתיבה באתר זה, לתכנן את הניסוי? אבל מה לעשות שזה לא פשוט? אולם כיוון שהבעיה היא טכנית/תקציבית בעיקרה, לא תאורטית, נראה לי שלבסוף הניסוי יבוצע, למרות שכריאליסט, אני נותן לו שברירי האחוז סיכוי להצליח.

    שים לב שתמיד אני מבקש כסאח, לא עדינות. אני זקוק לביקורת עמיתים, אך ריאלית ופרטנית, לא פשטנית.

    שיהיה יום מלטוב.

  945. אפילו הסבלנות שלי פוקעת, לפעמים.

    החומר האפל ניכר, נכון לימינו, בשתי תופעות. האחת, ה”דבק” התוך-גלקטי שאליו התייחסת בדבריך.
    השניה היא העידוש הכבידתי, שעליו לא דיברת.
    אם תטען כי גם העידוש הכבידתי אינו מאפשר מדידה ישירה, לא אגיד שאתה טועה. אבל המדידה באמצעותו מיידית יותר מזו של חישובי מסלולים ומהירויות.
    אתה אומר “מן החישובים שנועדו להתאים את הגרביטציה מתבררת התפלגות חומר אפל שהיא קרובה לאחד חלקי ריבוע המרחק” וזה בדיוק מה שהביא אותי לשאול (כבר לא זוכר באיזו כתבה ומתי) אם זה רק צירוף מקרים או ששדה הגרביטציה ושדה החומר האפל הם אותו דבר.
    לאמירתך “נראה לי שעדיין לא התייחסת לדברי כי לו עשית זאת היית חייב להגיע למסקנה שהשערתך נסתרת על ידי העובדות”: לתומי סברתי שהתייחסתי לדבריך יותר מפעם אחת. אם התעלמתי, הרי זה לא מתוך ניסיון התחמקות. אנא הזכר לי.

    התגובה ממתינה לאישור.

  946. שמע ישראל!
    את מלאכת הכיסוח אני משאיר, בדרך כלל, לאחרים שתחביבם בזה.
    בגדול, אני מתעקש על מודל שמתחיל ממש מאפס, מן הלא-כלום. בדרך הוא יכול לקפוץ לביקור אצל האתר הפעיל או לשתות קפה טורקי עם גזקום בהרצליה או אפילו להסתדר יפה עם שניהם גם יחד. במקרה הגעתי איתו די רחוק בלי להתעכב אצל חברים, אבל הנני סבור כי לא מזיק להיפגש מדי פעם ולהשוות.

  947. יהודה,
    תודה על ההבהרה. בתנאים כתיקונם הייתי מעדיף מלטוב, אבל באלה הימים הטרופים אקבל גם את המלרע במלאהבה.
    האם מקובלת עליך תשובתו של ישראל? הוא מבקש מודל שמסביר שבעה דברים. האם יש לך כזה? לי יש מודל שמסביר, בינתיים, רק ששה מהם, כי לא התעמקתי בשזירה הקוואנטית, אבל מן הסתם אתגבר גם על המכשלה הזאת.
    נעלה ונצליח

  948. יובל
    אשמח לאחד כוחות אם תסביר לי כיצד בחומר האפל או הגזקום מתקיימים התנאים הבאים:

    1. גרביטציה.
    2. אינרציה.
    3. מגנטיות.
    4. משיכה/דחיה חשמלית.
    5. גלים אלקטרומגנטים.
    6. אי לוקליות בשזירה קוונטית.
    7. קביעות מהירות האור בכל מערכת יחוס, ללא פוסטולט.

    כי נראה לי שדי הסברתי לך איך כל אלו מתקיימים באתר האקטיבי, ובאופן אינטואיטיבי. חיכיתי לכסאח לשווא.

    חוץ מהתנאים האלו, אתה צודק ב100 אחוז. כל 3 המודלים זהים.

  949. השכבות כמו רכבות. נידמה לי שקוראים לזה מילרע.
    יום טוב
    סבדרמיש יהודה

  950. ישראל ויהודה,
    למה זה נלכה סחור סחור? הרי החומר האפל שלי, האתר הפעיל והגזקום שלכם הם בדיוק אותם החלקיקים. אנו לא מתווכחים על המהות כי אם על האופן. הבה נאחד כוחות, נטה כולנו שכם אחד ונשיבה ימינו כקדם עד יצא לאור צדקנו וחוכמתנו ברבים, ולא נוסיף דעת עוד מלחמה.
    ובסיסמא Follow the sun to Herzliyah and the horse to Qiryat Anavim נכבוש את עולם הפיסיקה.
    אגב, איך מבטאים “הָשְכָּבוֹת”? במלעיל או במלרע?

  951. יובל
    מה פתאום נקיפה?, פרצסיה???, אנחנו בשכונה קראנו לזה “השכבות ” והיינו מתחרים על ההשכבות המוצלחות ביותר.
    לישראל
    הדוגמא של הסוס היא לא כל כך טובה , אבל אם תנסה לסובב את הסוס אני בטוח שהוא יתנגד.
    זה דוקא מוכיח שלסוסים יש התנגדות למסה אפלה!. אבל זה לא כך עם תרנגולות שמסובבים אותן בערב כיפור בתרגיל “זאת כפרתי” ובכך תורמות את עצמן לחלק האפל של היקום.
    עם חמורים זה לא כך כי הללו לא זזים גם בעמידה והם די בסתירה לחוק השני של ניוטון.
    סתם מידע אישי.
    ט.ל.ח.
    אך איזו שמש יפה זורחת מעל הרצליה!
    יום טוב
    סבדרמיש יהודה

  952. לא רק נקיפה. אני מאמין שיהודה מחפש משהו יותר אינטואיטיבי. אם תיקח את הBSA ותעמיד אותו עם הגלגל המניע באויר החופשי ללא סיבוב, תראה שקל מאוד להטות אותו מצד לצד. שים גז והגע למהירות ספידומטר של 200 קמ”ש, ותראה שיהיה לך קשה מאוד להניע אותו לצדדים, למרות שהאופנוע במהירות 0 יחסית לאדמה. זה יקרה גם בלי אויר. מהו המחזיק את הגלגל המסתובב בכוח כה חזק ומונע ממנו לנטות? אינרציה שמינרציה, תכלס, מה יש שם? למה זה יקרה ביתר שאת עם מים? לא מעורר חשד של חלקיקים קטנים מכל הכיוונים כמו מולקולות אויר או מים?

    ולנו במשק לא היה אופנוע, אבל היה לנו סוס עם לפחות 3.5 כוחות BSA.

  953. ישראל,
    לא גילית לנו את כל הסיפור. כשמנסים להטות את צירו של גלגל בשעת סיבובו, ההטיה “בורחת” בזוית ישרה. זה מה שמאפשר, למשל, לכוון את האופנוע באמצעות הטיית הגוף. יש לזה שם. אתה זוכר מהו?
    (התשובה במהופך)

    היסצרפ זעלבו הפיקנ
    (http://en.wikipedia.org/wiki/Precession)

  954. קח גלגל עם חישורים (שפיזים) והכנס אותם למים. כל עוד הגלגל אינו מסתובב, תוכל להטות אותו בקלות. ברגע שהוא מתחיל להסתובב, תזדקק לכוח גדול כדי להטות אותו. מה נשתנה? מה מפעיל את הכוח המייצב על הגלגל? מה הנמשל?

  955. ישראל ידידי
    אבל האופנוע יציב רק בנסיעה וכידוע גם במנוחה פוגעים החלקיקים באופנוע היקר אבל הוא לא ישכיל לעמוד יציב וכל שאיפתו לסוב ולנוע. כך שהסבר החלקיקים לא התשובה כאן וחלקיקי לה סאז’ לא עוזרים.
    אם כן חייבים לנסות ולהסביר בתאוריה המתחרה- האפלה.
    נניח שבזמן תנועה הופכת חלק מהתנועה הסיבובית למסה אפלה שעוטפת את האופנוע בחיבה ולא נותנת לו ליפול. זה ניראה לי יכולה להיות התשובה. ויש לזה אפילו הוכחה:-
    כשהאופנוע לא נע נעלמת המסה האפלה והוא נופל. הוכחה או לא?
    לילה טוב
    סבדרמיש יהודה

  956. ישראל:
    אני אישית חזרתי להגיב – פשוט מפני שהתפנה לי קצת זמן.
    אני מאד עסוק לאחרונה ובזמן שיש לי אני עונה על התגובות שהופנו כלפי במאמרים קודמים ואחר כך עובר על המאמרים החדשים אחד אחד, קורא את התגובות להם ובודק אם יש לי מה לומר.
    לפחות מבחינתי – זה לא קשור במסה האפלה.

  957. אח, איזו נחת, איזו נוסטלגיה!

    כל החברה הישנים והטובים משכבר הימים בכתבה על קוסמולוגיה. יודה צודק, אין כמו מסה ואנרגיה אפלות למשוך תגובות ולקרב לבבות . מי חסר? ר.ח. כמובן! כרגיל בצ’ארלס. בואו נקרא לו, אל נבוש! אני מוכן לקפוץ בראש! מיד לסדר אקראנו, ואחרי כולכם רננו:

    ר.ח. חזור!

    יהודה.

    אין בכך כל פלא, אם תחשוב על חלקיקים הנעים מכל הכיוונים ופוגעים באופנוע. אם יש לך ספק, דמיין חלקיקים כבדים ומהירים יותר, ותראה שתוכל להתייצב במהירות נמוכה יותר ממה שאפשרי בפועל.

    אכן שקילות במיטבה!

  958. מתי
    היו זמנים!
    ועדין יורד לי ריר כשאני ניתקל בהרלי דוידזון.
    ובגבעתיים לפני כחודשיים ראיתי אחד יפהפה כזה ברחוב רמב”ם.
    וישראל לגבי שאלתך
    הסיבה היחידה שאני לא נופל מאופנוע היא אישתי שלא מאפשרת לי לעלות על אחד כזה.
    אני זוכר שיש הסבר של מומנט סיבוב ועיקרון הג’ירוסקופ , אבל תמיד ראיתי בזה פלא.
    לפחות את הפלא הזה רואים, בניגוד לפלאי המסה והאנרגיה האפלים.
    (:))
    ערב טוב
    סבדרמיש יהודה

  959. יובל:
    זו אינה התייחסות אלא התעלמות.
    לא מודדים חומר אפל ישירות כי לא יודעים לעשות זאת.
    מודדים גרביטציה.
    בגרביטציה של כדור הארץ מבחינים והיא הולכת לפי אחד חלקי ריבוע המרחק.
    בגרביטציה של הגלקסיה מבחינים גם כן ומכיוון שהיא אינה תואמת את החוק של אחד חלקי ריבוע המרחק מניחים שיש חומר אפל.
    מן החישובים שנועדו להתאים את הגרביטציה מתבררת התפלגות חומר אפל שהיא קרובה לאחד חלקי ריבוע המרחק.
    נראה לי שעדיין לא התייחסת לדברי כי לו עשית זאת היית חייב להגיע למסקנה שהשערתך נסתרת על ידי העובדות

  960. מיכאל,
    רוב “הויכוח” ביננו הוא על סמנטיקה. במהות עצמה אין לי ויכוח אתך וחבל להשחית זמן לריק.
    ולעניין הטענה שלא עניתי עליה, “שכל עצם יוצר שדה גרביטציה בשעה שהתפלגות הנדונה של המסה האפלה קיימת רק סביב מרכז הגלקסיה”:
    אמנם לא התייחסתי אליה ישירות, אבל בעקיפין כן. אני חוזר וטוען שהמכשירים שבידינו כיום אינם רגישים דיים למדוד נוכחות של מאסה אפלה בריכוזים קטנים.

  961. יהודה! ברוך שובך התגעגתי!!!
    B.S.A אה? קלאסי משהו
    אני עכשיו גר בגרמניה וקניתי נורטון ישן לשם המשחק כמובן
    והיה לי רויאל אנפילד 62 ששיפצתי

  962. יובל:
    כפי שאמרתי – ריכוז החומר האפל לא מוסק מן ההמחה שבגלקסיה שלנו אין חומר אפל.
    להפך – אם תקרא בקישור שנתתי בעניין שביל החלב תראה שחושבים בגלקסיה שלנו יש המון חומר אפל.
    זה גם עונה לטענתך בעניין הסקת תורת היחסות מן המתרחש בגלקסיה שלנו. זו טענה לגמרי שגויה ואילו הייתה נכונה – רק אז היה בסיס לניסיונך להשוות בין הגלקסיה שלנו לבין אחרות.
    גם המתרחש בגלקסיה שלנו סותר את תורת היחסות אם אין מניחים את קיום המסה האפלה בתוכה.
    בנוסף לכך – אין הבדל משמעותי בין “אין מסה אפלה” לבין “יש כה מעט שאי אפשר לגלות”.
    אם המסה האפלה הייתה הסיבה לקיום הגרביטציה – הגלקסיה הייתה מתפרקת גם אילו הייתה “מעט מאד” מסה אפלה.
    אני מניח שברור לך שלא ענית לטענתי בדבר העובדה שכל עצם יוצר שדה גרביטציה בשעה שהתפלגות הנדונה של המסה האפלה קיימת רק סביב מרכז הגלקסיה – זה לבד מהווה סתירה מוחצת להשערתך ולמעשה לא היה צורך בשום טיעון אחר אבל, כפי שראית – יש עוד הרבה טיעונים אחרים.

    אם אתה רוצה עוד טיעון אז הנה עוד טיעון:
    כוח המשיכה נמצא ביחס הפוך לריבוע המרחק רק…….
    …….
    ……
    בהעדרה של מסה אפלה.

    למעשה הוסיפו את המסה האפלה בגלל שעל פי התצפיות אין היקום מתנהג לפי חוק זה!

    את דברי על הטענה נגד “שיטת ניפוץ החלקיקים” לא הפניתי אליך. הפניתי אותה אל הטענה עצמה.
    זה לא משהו אישי. ידעתי שיעל השמיעה אותה לראשונה אבל לדעתי מדובר בטענה מזלזלת שאין לה מקום והסברתי מדוע.

    יעל:
    קראתי הרבה מדבריו של עילם גרוס.
    יש לנו גם חברים משותפים.
    שום דבר מכל אלה לא מביא אותי למסקנה שהשיטה היחידה שאנחנו יודעים כדי לגלות את החלקיקים האלמנטאריים שיש היא מפגרת.
    אם תמציאי שיטה אחרת לפתות חלקיקים לצאת מתוך מבנים מורכבים יותר יהיה אולי בסיס כלשהו לטענתך אבל גם אז יהיה מדובר בבסיס רעוע.
    כפי שהניסויים מראים עד כה – יש חלקיקים מהותיים בעלי משך חיים קצר ביותר – כאלה שבכלל אינם מרכיב בשום חלקיק שאת יכולה לנסות לפרק.
    בכלל – מהי בעיה ומהו דבר רע?
    לא מדובר כאן בדבר אבסולוטי אלא בתוצאה של השוואה בין דברים.
    למשל – על אדם בריא לא אומרים שהוא חולה וזאת למרות העובדה שאינו יכול לעוף.

  963. יהודה.
    מה לדעתך מייצב אותך בזמן רכיבה על האופנוע שלא תיפול?

  964. מר מיכאל רוטשילד,

    ה”ניפוץ” היא שיטה גרועה ביותר, אכן היא השיטה הכמעט יחידה שיש היום. אתה יודע למה קשה למצוא את ה”היגס”? אתה מבין שבעצם מנסים ליצור אותו ולא לחלץ אותו? אתה מבין למה הסבירות לגלות אותו כל כך נמוכה? הרי ניסוי צריך להניב כל הזמן את אותן תוצאות על מנת שיהיה אמין… למה שזה לא קורה בחקר חלקיקים?

    ממליצה לך לקרוא קצת את כתביו של פרופ’ עילם גרוס (למרות שבטח כבר קראת), ישנה בעיה טכנולוגית גדולה במחקר הבסיסי הזה.

  965. ישראל
    אומנם את הקטע הנ”ל של קופולה אינני מכיר (חור בהשכלה), אבל, זה מזכיר לי את האופנוע האנגלי שהיה לי בתחילת שנות השיבעים, B.S.A (“תרנגול”) , 3.5 כוחות סוס , שדהר בשדות אפלים, עם נערתי (אישתי לעתיד) היפה.
    בלי מסה אפלה, ואת האנרגיות האפלות השארתי לאחרים.
    אך, היו זמנים!,
    יום טוב
    סבדרמיש יהודה

  966. יודה.

    !Hey, man, I thought you left for good

    (ראסטי גיימס לנער האופנוע, קופולה 1983).

  967. תודה מיכאל.

    דיברתי על ריכוז החומר האפל המשתנה מגלקסיה לגלקסיה. זה היה בתשובה לאמירתך שיש גלקסיות ללא חומר אפל, וכוונתי הייתה לומר שיש גלקסיות בהן ריכוז החומר האפל קטן עד כדי כך שאיננו מצליחים להבחין בו. מכשירי התצפית שבידינו אינם רגישים דיים לספק לנו נתונים מדויקים לקביעת כמות החומר האפל וריכוזו, ומשום כך אני מפקפק באמירה כי יש גלקסיות שאין בהן חומר אפל. בהמשך הבאתי הצגה קיצונית, לפיה אצלנו אין חומר אפל, כדי להראות סיבה היפותטית לכך שישנן גלקסיות הנראות לנו כריקות מחומר אפל משום שהתנהגותן (כלומר התנהגות החומר האפל בהן) היא כמו זו שלנו.
    תורת היחסות נולדה בגלקסיה שלנו ונבדקה, הלכה למעשה, רק במערכת של כוכב אחד מתוכה. הנוסחאות שלה נכונות רק כאן (וגם זה בערבון מוגבל). לכן, תורת היחסות של כל גלקסיה כוללת, מן הסתם, נוסחאות וקבועים הייחודיים לה.

    לפני מספר שעות העליתי כאן “שאלת תם” הנוגעת לתכונותיהן האופטיות של עדשות כבידה ולשימושים האפשריים בהן. עד עתה אף אחד לא הרים את הכפפה.

    המשפט “המזלזל” הוא ציטוט מדברי יעל. אהבתי אותו בגלל הפואטיות שבו (וגם כי הוא נתן לי פתחון פה להתרברבות קלה). את הביקורת שלך בעניינו אנא הפנה אליה.

  968. יובל (https://www.hayadan.org.il/astronomers-reach-new-frontiers-of-dark-matter-130112/#comment-323446):
    1. בתגובתי הקודמת כתבתי שכמות החומר האפל שונה מגלקסיה לגלקסיה כך שברור שאחן צורך שתספר לי על כך.
    2. למה שנסתכל על הגלקסיה שלנו כנטולת חומר אפל?! במיוחד – למה שנעשה זאת כאשר הממצאים מראים שיש בה חומר אפל?! http://en.wikipedia.org/wiki/Milky_Way
    3. מה זה “גלקסיה אחרת המתנהגת כמוה”?!
    4. את קיומו של חומר אפל בגלקסיות (כמו גם את כמותו והתפלגותו) מסיקים ממדידות על הגלקסיות עצמן ולא מתוך השוואה לגלקסיה שלנו. ההשוואה היא רק בין התוצאה הנמדדת לבין תחזיות תורת היחסות.

    אישית אני בכלל לא מתלהב מן המשפט המזלזל ” המחקר כיום של חלקיקים אלמנטריים הוא מאוד גס וגולמי”
    כשמקטלגים משהו כ”גס וגולמי” זה צריך להיות ביחס למשהו טוב יותר – שאיננו כזה, לדעת המשווה.
    בכלל לא בטוח שאי פעם תהיה לנו דרך טובה יותר ובדרך הזאת כבר גילינו המון (אבל ממש המון!)
    אגב – זה לא שונה מהותית מן הדרכים שנחשבות למתקדמות ביותר לבדיקת מבנה החומר (למשל באמצעות מיקרוסקופ אלקטרונים). מעניין מי רוצה לקטלג מיקרוסקופ כזה כ”גס וגולמי”.
    ככלל – כל מבנה שאי פעם גילינו נתגלה על ידי פירוקו (=ניפוצו) לחלקיו.
    דווקא בנושא החלקיקים האלמנטאריים זה לא בדיוק המצב כי פעולת ה”ניפוץ” בכלל לא מנפצת. היא לוקחת את סך האנרגיה של ההתנגשות ויוצרת מגוון חלקיקים שבמקרים רבים אינם מהווים חלק מן החלקיקים המתנגשים.

  969. שמתם לב שלמסה והאנרגיה האפלים יש כישרון למשוך אליהם……מגיבים רבים?
    חוץ מזה הייתי עסוק בעינייני ורואה שאתם מסתדרים לא רע בלעדי.
    אז…
    שלום רב
    ולהתראות
    סבדרמיש יהודה

  970. שאלת תם על עידוש כבידתי:
    היות שגלקסיה מהווה, למעשה, עדשה אופטית, הרי יש לה מאפיינים של כזאת, למשל מקדם שבירה ואורך מוקד. אפשר להניח בסבירות גבוהה כי גלקסיות דומות מהוות עדשות דומות. האם אפשר לבסס על ההנחה הזו שיטות לחישוב מרחקיהן של גרמי שמים – הן של העדשות הגלקטיות עצמן והן של האובייקטים שתמונתם מתעוותת על ידן?

  971. שלום מר יובל חייקין,

    אני חושבת שאתה צודק לגמרי. ה”בליעות” שאני ראיתי היו בשחור-לבן, והיו של מימן. ההנחה היא שסופרנובות מטיפוס לה פולטות עוצמה קבועה של אור, לפי העוצמה המגיעה אלינו לכן ניתן לדעת את מרחקן מאיתנו. מעקב אחר סופרנובות אלה לכן מעיד על מהירות תנועתן וגם תאוצתן.

  972. מר יעל (עם ד”ש למר [ח]זיר, אם ישנו כזה בנמצא 🙂 )
    אהבתי את דבריך אלה: “המחקר כיום של חלקיקים אלמנטריים הוא מאוד גס וגולמי, ניפוץ חלקיקים בכדי ללמוד עליהם היא שיטה מאוד מפוקפקת. מאידך ‘טלסקופים’ מבוססי ניטרינים או גלי כבידה נמצאים על הפרק בשלבי פיתוח”, ולמען הקוריוז אוסיף כי כמעט כל יום אני עובר על יד המעבדה בה מותקנים מכשירים לגילוי גלי כבידה. זה בבניין ע”ש קלוין (שעל שמו סקאלת הטמפרטורה) באוניברסיטה של גלזגו.
    מאידך, אמירתך “ב-300,000 שנה הראשונות מהמפץ הגדול היקום היה אטום לפליטת קרינה אלקטרו-מגנטית, לא כך לגבי ניטרינים וכבידה” מציגה דבקות מופתית, שמרנית משהו, במודל מקובל. לא שאני שולל אותו, אך גם אינני מקבל את קביעתו במסמרות.
    הנמשיכה להתכתב פרטי?
    [email protected]

  973. מיכאל,
    ראשית תודה על התייחסותך. להלן שתי השגות על דבריך:
    1) מן התצפיות רואים כי ריכוז החומר האפל משתנה מגלקסיה לגלקסיה. אם אנחנו מסתכלים על הגלקסיה שלנו כמשוללת חומר אפל, הרי כל גלקסיה אחרת המתנהגת כמוה תיחשב בעינינו ריקה גם היא.
    2) גם שיעור החומר הבריוני שאנו יכולים לראות משתנה מגלקסיה לגלקסיה.
    משום כך קשה לי לקבל את מסקנותיך כמוחלטות.

  974. כן, יעל. נראה לי כי כעת דייקת. “חתימות” הסופר-נובה מראות הסחה לאדום הגדלה והולכת עם גידול המרחק בין מקורן לבינינו. כדי לא להישמע מיסתוריים, הרי הסבר מפורט יותר: בתחזית (ספקטרום) מתקבלים פסי בליעה בדפוסים אופייניים הממוקמים באזורים בצבעים מסוימים. באור המגיע מגלקסיות רחוקות, אותם דפוסים של פסי בליעה מופיעים באזורים בצבעים אחרים. ככל שגדל המרחק ממנו מגיע האור, הולכים גם פסי הבליעה ומתרחקים מן המיקום המקורי שלהם. האם צדקתי? נא הערותיך ותיקוניך.

  975. יובל (https://www.hayadan.org.il/astronomers-reach-new-frontiers-of-dark-matter-130112/#comment-323353):
    כבר עניתי לך על זה!
    זה לא יכול להיות כי יש גלקסיות בלי חומר אפל והן גלקסיות כי יש בהן גרביטציה (אחרת היו מתפזרות).
    גם כמות החומר האפל שונה מגלקסיה לגלקסיה ללא קשר לכמות החומר הבריוני.
    בקיצור – זה בכלל לא מסתדר! זו לא שאלה אלא אבחנה עובדתית הנמצאת בסתירה להשערתך.

    עוד אבחנה עובדתית הסותרת את השערתך היא שהתפלגות המסה האפלה היא לפי אחד חלקי ריבוע המרחק רק ביחס למרכז הגלקסיה בשעה שהגרביטציה פועלת לפי יחס זה סביב כל אחד ואחד מן העצמים השונים בגלקסיה.

  976. מר יובל חייקין,

    אכן טעיתי המון פעמים, אבל לא הפעם. תצפיות על “חתימות” סופרנובות הראו שהתפשטות החומר ביקום מואצת. מאחר ולהאיץ מסה נדרש כוח וכוח דורש אנרגיה… אין היום שום הסבר למה היא האנרגיה הזאת שמאיצה את מהירות התרחקות של הגלקסיות אחת מרעותה – זאת ה”אנרגיה האפלה” שאתה מתכוון אליה.

  977. יעל,
    אני חושש שיש בדבריך בלבול מושגים. מה שכתבת מתאים למאסה אפלה, בעוד שאני דיברתי על אנרגיה אפלה. אלה שני דברים שונים.

  978. ישראל שפירא,
    “אתר אקטיבי” זה המינוח שלך. אמנם יש משהו “אקטיבי” במודל המתבשל אצלי, אך אני נזהר לא להשתמש בשם “אתר” היות שזה משמש לציון מודל פשטני מדי.

  979. מר יובל חייקין,

    מה שרשמת על האנרגיה האפלה איננו מדויק – הסחה לאדום נובעת ממהירות יחסית שונה של עצמים שונים, אין לה שום קשר לאנרגיה האפילה!

    האמונה בקיום האנרגיה האפלה נבעה מחקר סופרנובות מטיפוס 1A (או “לה”). המסה האפלה נבעה מ”מחסור” בחומר נראה להסבר תנועה כבידתית בסדרי גודל גלקטיים וכן מעידוש כבידתי.

    יש לזכור שהמחקר כיום של חלקיקים אלמנטריים הוא מאוד גס וגולמי, ניפוץ חלקיקים בכדי ללמוד עליהם היא שיטה מאוד מפוקפקת. מאידך “טלסקופים” מבוססי ניטרינים או גלי כבידה נמצאים על הפרק בשלבי פיתוח. יש לזכור שב-300,000 שנה הראשונות מהמפץ הגדול היקום היה אטום לפליטת אור וקרינה אלקטרו-מגנטי, לא כך לגבי ניטרינים וכבידה.

  980. ר.ח רפאי.ם,
    מדידה ישירה של האנרגיה האפלה תהווה הוכחה ניצחת לקיומה. לכן אני בדעתך שצריך לבנות מכשיר מדידה הולם.
    “משהו בסגנון האתר” זה התווך המוליך את הקרינה האלקטרומגנטית. התווך “שלי” מוליך את כל התופעות הפיסיקליות. היות שבין היתר הוא מוליך גם את הקרינה האלקטרומגנטית וגם את הגרביטציה, תאורו צריך להיות מורכב יותר מזה שניתן לאתר.

    נקודה,
    תצחק, תצחק, אבל יום אחד עוד יתברר שצדקת …

  981. לפי המודלים שלי בעולם של המסה האפלה כבר יש מאות כדורי ארץ אפלים שהתרחשו בהם המון אבולוציות ונוצרו המון יצורים אפלים תבוניים שאוהבים בעיקר לספר בדיחות שחורות אחד על השני.
    הם בטח גם תוהים על ה5% המסה החסרה שלא מסתדרת להם בחישובים שהם מבצעים על גבול האפלה.
    לפי אותם מודלים שלי אצלם השמש היא אפילה וגורמת ללילה ורק חורים שחורים מאירים אצלם את היום האפלולי שלהם.
    אצלם גם הגזע העליון הוא בצבע שחור והם משתמשים בחומרים משחירי כביסה כדי שהכביסה תצא נקיה.
    אחרי מדיטציה ארוכה שם אתה עלול לצאת מושחר ואז כולם באים לשמוע מה יש לך לומר כדי שתחשיך את עיניהם.
    יתכן שהיקום הנראה אצלם הוא גדול בהרבה כי אולי מהירות החושך גבוהה בהרבה ממהירות האור ואז הם מסוגלים לא לראות למרחק גבוה בהרבה ממה שאנו מסוגלים לראות.
    בימי הולדת הם ודאי מכבים נרות ואז מדליקים אותם כדי לבקש משאלה.

  982. * ב”למדוד” הכוונה- פיזית למדוד, כמובן. להמציא טכנולוגיה, במילים אחרות.

  983. יובל
    אני מניח שאתה לקחת בחשבון משהו בסגנון ה’אתר’ כתחליף לאנרגיה אפלה 🙂 , צדקתי?
    בכל אופן, לדעתי האנרגיה האפלה היא באמת אנרגיה (קרינה). ואני גם מסכים עם כל המשוואות הרלוונטיות ,כאלו שמהן נובעת המסקנה שקיימת אנרגיה כלשהי, בלתי רגילה, בנוסף לכל הסוגים שקיימים. רק שאני חושב שצריך לחפש את הדרך למדוד את האנרגיה הזאת.

  984. מיכאל,

    בינתיים לא גיבשתי תאוריה אלא רק הצבעתי על שתי תופעות הדומות זו לזו כמותית ומקומית. ברשותך, אכתוב מחדש את הדברים בתיקון קוסמטי קל.
    החומר האפל מקיף את החומר הבריוני, וצפיפותו פרופורציונית הפוכה לריבוע המרחק מן המרכז.
    גם עצמת שדה הגרביטציה הינה פרופורציונית הפוכה לריבע המרחק מן המרכז.
    הצירוף הלא מקרי הזה מצביע על האפשרות שהחומר האפל ושדה הגרביטציה הינם ישות אחת.

    את הדיון בהשערה (לא תאוריה. רק השערה), כי החומר הבריוני אינו אלא חומר אפל הנמצא בריכוז הגבוה מרמת סף מסוימת, אני מקפיא לעת עתה.

  985. ר.ח רפאי.ם
    האנרגיה האפלה מעולם לא נמדדה. משום כך היא נקראת “אפלה”. קיומה משוער משום ההסחה לאדום של האור המגיע מגלקסיות רחוקות, הסחה ההולכת וגדלה ככל שהמרחק גדל. הפרוש המקובל לתופעה הזאת מתבסס על שתי הנחות: האחת, שמדובר בתוצא דופלר; השניה, שמהירות האור קבועה בכל מקום (ריק) ביקום. מאלה נובע כי הגלקסיות לא רק מתרחקות אלא מאיצות. תנועה בתאוצה מצריכה אנרגיה, אך המודלים הפיסיקלים הקיימים אינם מסבירים את קיומה.
    אמנם היא לא נמדדה, אך אפשר לחשב אותה.
    כאמור, קיומה הוא מסקנה הנובעת משתי ההנחות דלעיל. אם יימצא הסבר אחר, יתכן כי גם הנחת האנרגיה האפלה תיפול.

  986. יש עוד שאלה כמו: מה קדם למה? – החומר האפל לחומר הבריוני או להיפך? או ששני הדברים הם תוצר של אותה אנרגיה? חומר אפל לא מבצע אינטרקציה עם חומר רגיל או קרינה שמוכרים לפיזיקה. מכאן שכנראה חומר אפל נמצא על ‘סולם’ אחר מהסולם שעליו נמדדות כל סוגי הקרינה המוכרים.
    הדרך היחידה לבדוק את זה תהיה לבנות מכשיר שיודע לזהות קרינה- מאותו סוג מסוים, שכרגע אינו ידוע לפיזיקה. זאת אומרת צריך קודם למצוא את האנרגיה שכן יש קשר בינה לבין חומר אפל. אולי זאת האנרגיה האפלה? מישהו יודע איך אפשר למדוד אנרגיה אפלה? 🙂

  987. יובל:
    אלו שאלות שמי שמציע הצעה כשלך צריך לענות עליהן.
    כל עוד לא קיבלו שאלות אלו תשובות (ולחלקן לא תיתכנה כלל תשובות!) אין מה להעלות את התאוריה כיוון שמספר הבעיות שהיא יוצרת גדול בהרבה ממספר הבעיות שהיא פותרת

  988. אלה, אכן, שאלות שאלתיות מאד.
    כל עוד איננו יודעים דבר על המסה האפלה (חוץ מן ההודאה בקיומה), איננו יכולים להשיב על השאלות האלה.
    גדול הפיתוי לחפש להן פתרון על ידי בניית מודל (למשל, חלקיקים) של המסה האפלה.
    כשבונים מודלים אפשר להתפרע ולהמציא תכונות כמה שרוצים, ובלבד שהתוצר הסופי יהיה דומה ככל האפשר למה שמשתקף מן המציאות. זה לא בשמיים וגם לא מעבר לים. מותר לכל אחד לנסות. וכל עוד לא קיים מודל כזה, אין מה לשלול.

  989. יובל:
    כבר דיברנו על עניין הריבוע ואפילו סיפרתי לך שזה הדליק בי איזה רעיון על הקשר בין מסה אפלה ואנרגיה אפלה (רעיון שאותו הסברתי כאן בקווים כלליים אבל בהמשך עבדתי עוד על נוסחאות רלוונטיות והגעתי למסקנה שככל הנראה איננו נכון).
    הרעיון שהמסה האפלה יוצרת את שדה הגרביטציה הוא בפירוש שגוי.
    לכל מסה יש שדה גרביטציה ויש גלקסיות שאין בהן חומר אפל כלל.
    גם הרעיון שהמסה הבריונית היא “מסה אפלה מרוכזת” נראה שגוי.
    איך יכולה מסה אפלה להתרכז כאשר האינטראקציה בינה לבין עצמה היא כה חלשה?
    חוץ מזה – איך יכול להיות שבמאיצי חלקיקים לא ניתזים “רסיסי מסה אפלה” מן ההתנגשויות?
    חוץ מזה – איך הופכת אנרגיה למסה אם המסה אינה ריכוז של אנרגיה אלא של מסה אפלה.

    כפי שאמרתי – הרעיון שמסה אפלה היא מרכיב חשוב ביצירת הגלקסיות הוא התאוריה המקובלת אבל לא צריך למשוך את זה מעבר לנדרש

  990. מיכאל,
    הרי ברור שזה לא צירוף מקרים. זו הייתה שאלה רטורית.
    החומר האפל מקיף את החומר הבריוני, וצפיפותו פרופורציונית הפוכה לריבוע המרחק מן המרכז.
    גם עצמת שדה הגרביטציה הינה פרופורציונית הפוכה לריבע המרחק מן המרחק.
    הצירוף הלא מקרי הזה מצביע על האפשרות שהחומר האפל הוא זה שיוצר את שדה הגרביטציה.
    יתירה מזו, היות שהחומר הבריוני נמצא במרכז שדה הכבידה, היכן שריכוז החומר האפל הוא הגבוה ביותר, נראה לי סביר ביותר להניח כי החומר הבריוני אינו אלא חומר אפל הנמצא בריכוז הגבוה מרמת סף מסוימת.

  991. יובל:
    זה לא צירוף מקרים.
    זו תוצאה של הגרביטציה שנוטה ליצור ריכוזי מסה ואינה מפלה בין סוגי המסה.
    החשיבה המקובלת כיום במדע גורסת שלחומר האפל היה תפקיד מרכזי (תרתי משמע) ביצירת הגלקסיות של החומר הבריוני.

  992. האני האחר:
    התצפית הזאת לא מדדה שינויים בהתפלגות החומר האפל אלא את התפלגותו הנוכחית.
    היא גם לא מדדה חומר אפל בכל היקום אלא רק בטווח שבין מקומנו הנוכחי לבין 6 מיליארד שנות אור מאיתנו (כזה שיכול ליצור עידוש כבידתי על אור הכוכבים יצא לפני 6 מיליארד שנה וכיום הוא מגיע אלינו).
    למעשה – זה אפילו יותר מסובך מזה כי התפשטות היקום בוודאי כבר הזיזה את החומר האפל שאת השפעות העידוש שלו אנו רואים כיום.

    לגבי הדינאמיקה של התפלגות החומר האפל יכולות להיות לנו כיום רק תיאוריות כי אין לנו דרך לצפות בחומר האפל עצמו.
    כל המידע שיש לנו על היקום מגיע באמצעות קרינה אלקטרומגנטית וחלק גדול ממידע זה מוסק מגילויים שונים של אפקט דופלר (כי המרחקים גדולים מדי בשביל לגלות תנועה באופן ישיר).
    החומר האפל מקיים עם הקרינה האלקטרומגנטית רק אינטראטקציה כבידתית ולכן, בין השאר, לא ניתן לגלות תנועה שלו על סמך אפקט דופלר.

    רפאים:
    אינני יודע לאיזה מכשור אתה מצפה.
    על פי התחזיות של התיאוריות המדעיות בנות זמננו – לעולם לא נוכל לצפות למרחק שהוא גדול מן הדרך שיכול היה האור לעשות במהלך חיי היקום.

  993. מיכא*ל,

    תודה על ההסבר, עכשיו זה מסתדר.

    מה לגבי השאלה השנייה?
    ועוד שאלה – לפי הכתבה החומר האפל / אנרגיה אפלה (מחק את המיותר, לא הצלחתי להבין) היה מסודר בצורה של רשת ברחבי היקום לפני 6 מיליארד שנים. האם יש שינוי בצורת הסידור שלו עם התקדמות הזמן?

  994. מכאל
    כשמנסים “לקצר” בהסבר על נושא כזה אז, לפחות אצלי, קורות טעויות 🙂
    נכון, בעזרת הטלסקופים ניתן לצפות עד למרחק של 13.7 מיליארד שנות אור.
    כלומר, הטכנולוגיה מאפשרת כיום לצפות רק עד למרחק של 13.7 מיליארד שנות אור. בהתחשב שהיקום מתפשט בקצב מואץ, אז מה שאנחנו רואים כיום במרחק 13.7 מ’ שנות אור, בעצם נמצא במרחק של 46 מ’ שנות אור. כלומר בזמן התצפית המרחק האמיתי (בפועל) עד לאותו איזור הוא 46 מ’ שנות אור.
    זאת אומרת שתאורטית ניתן לצפות בנפח של X מהיקום
    (4.1X10^32 cubic light years – לפי ויקיפדיה) אבל מתוך הכמות הזאת רואים רק אחוז מסוים (13.7 מ’ שנות אור).
    לגבי השאלה מהו הגורם להתפשטות מואצת של היקום? נניח אנרגיה אפלה.
    האנרגיה הזאת אי אפשר למדוד, לפחות עדיין לא. בפועל ניתן לצפות בכמות מסוימת מן היקום כולו, והכמות הזאת היא X. ברגע שיתגלה חלקיק של האנרגיה האפלה ויבנה המכשור המתאים, ניתן יהיה לצפות גם מעבר למה שנחשב היום כ ‘גיל היקום’, ניתן יהיה לצפות, בעצם, בגבולות של היקום עצמו. עד כמה פרדוקסלי שזה נשמע.
    הרי נהוג לחשוב שהאנרגיה האפלה מתפלגת במידה שווה בכל היקום. ואם כך הדבר אזי יווצר מצב שבו הגבולות של היקום יהיו קיימים גם בין מרחקים קוונטיים. זה פרדוקס לא?
    זה גורם לי לחשוב שאי-גילויו של חלקיק האנרגיה האפלה פותר את הפרדוקס. 🙂

  995. אז כבר לפני כ-6 מיליארד שנים אסטרונומים מדדו את ריכוז החומר האפל ועדיין לא הגיעו למסקנה ?
    אולי הם צריכים עוד 6 מילייארד שנה ?
    Deep Thought

  996. חברים:
    הגלקסיות הרחוקות ביותר שאנחנו רואים נמצאו במרחק 13.7 מיליארד שנות אור מאתנו ברגע שיצא מהן האור שאנחנו קולטים כיום.
    זה לא המקום בו גלקסיות אלו (אלו מהן ששרדו) נמצאות כיום.
    היקום נמצא בהתפשטות מואצת וגלקסיות אלו המשיכו להתרחק מאתנו.
    כל זה ממש ברור וקל אבל מה שקשה יותר זה להבין איך – במהלך 13.7 מיליארד שנים הן הצליחו לצלוח מרחק של 32.3 (ההפרש בין 46 לבין 13.7) מיליארד שנות אור? הרי המהירות המקסימאלית שאפשר לנוע בה אינה עולה על מהירות האור ולכן נראה שבמהלך 13.7 מיליארד שנים אי אפשר לעבור מרחק העולה על 13.7 מיליארד שנות אור!
    זה המקום שבו משתלבת התפשטות היקום עצמו (שהיא גם מואצת, אבל זה לא חשוב כרגע לענייננו).
    התפשטות זו (של המרחב עצמו) יכולה לגרום למרחק בינינו לבין גוף כלשהו במרחב לגדול בקצב העולה על מהירות האור.

    רוח רפאים:
    הטלסקופים שלנו לא רואים למרחק 46 מיליארד שנות אור אלא רק למרחק 13.7 מיליארד שנות אור.
    ל46 מיליארד מגיעים בחישובים ממש (אבל ממש!) לא פשוטים וכל החישובים הללו נעשים תחת הנחות המודל הסטנדרטי של הקוסמולוגיה שהמדענים נוטים להאמין שהוא נכון אבל – כמו שאנחנו יודעים – אין “בטוח” במדע.

    כל זה, כמובן, לא שייך לכתבה שמופיעה כאן.
    בנוסף לכך, למרות שזה מתייחס לציטוט שהביאה יעל מויקיפדיה, זה גם לא מתייחס לשאלתו של י.ש. שדווקא ידע שגיל היקום הוא 13.7 מיליארד שנה (וכתב זאת בגוף השאלה) ולכן תשובתה של יעל אינה עונה לשאלתו.

  997. הכתבה מאוד לא ברורה… ומהתגובות לגמרי התבלבלתי, סליחה על שאלות תם:

    מישהו מוכן להסביר מהי התאוריה המובילה של גיל היקום לעומת הקוטר שלו ואיך שני המספרים מסתדרים?

    ועוד שאלה – לפי הכתבה החומר האפל / אנרגיה אפלה (מחק את המיותר, לא הצלחתי להבין) היה מסודר בצורה של רשת ברחבי היקום לפני 6 מיליארד שנים. האם יש שינוי בצורת הסידור שלו עם התקדמות הזמן?

    תודה.

  998. הנקודות הכי רחוקות שבהם אפשר לצפות (בעזרת טלסקופ), נמצאות במרחק של כ-46 מיליארד שנות אור. לכל כיוון שלא נסתכל. כלומר המסקנה שהתקבלה מהתצפיות מצביע על כך שהיקום הוא קרוב לוודאי ‘ספירה’ (צורה של ‘כדור’ כלומר ‘מעגלי’). במידה ומדידת המרחק (תצפית) נעשית מכדור הארץ, אז כדור הארץ יחשב כמרכז המעגל. ומכאן נגיע למסקנה שרדיוס המעגל הוא כ-46 מיליארד שנות אור. ובחישוב פשוט ניתן להגיע למסקנה שאורך הקוטר של היקום הניצפה (כזה שניתן לצפות בו- ומעברו אינו ניתן לצפייה) הוא בגודל של כ- 93 מיליארד שנות אור.

  999. ב 13 מיליארד שנים היקום צריך להגיע לגודל של 26 מיליארד שנות אור מצד לצד – הלא כן ?

  1000. בכתבה כתוב “היקום מורכב מרשת קוסמית מורכבת של חומר אפל וגלקסיות המשתרעת למרחק של למעלה ממיליארד שנות אור”.

    זה בהחלט לא טעות כי 93 מיליארד זה באמת יותר ממילירד, אבל זה גם לא ממש מדוייק.

  1001. AR:
    אתה צודק.
    מדובר בטעות בתרגום ובמקור כתוב שהמידע ינוצל לחקר האנרגיה האפלה ולא החומר האפל.

    י.ש:
    אין כאן מידע סותר על גיל היקום.
    אינני יודע למה בדיוק התכוונת אבל הדבר הכי קרוב לאי דיוק הוא צירוף הטענה שהאור יצא לדרכו לפני 6 מיליארד שנה עם הטענה שהיקום היה אז בן 6 מיליארד שנה.
    זה באמת קצת לא מדויק כי זה מביא למסקנה שהיקום הוא בן 12 מיליארד ולא 13.7 מיליארד אבל ברור שמדובר כאן בהתבטאות “על בערך” וההבדל לא יכול להיחשב לסתירה.

  1002. עד עכשיו חשבתי ש13.7 מיליארד שנות היקום זה מרחק ההשתרעות שלו, מי מוכן להסביר את ההבדל

  1003. חומר אפל הוא הגורם להאצת היקום?
    חומר אפל אמור להיות הגורם ליצירת הגלקסיות וריכוזם (ויצירת החומר כפועל יוצא)
    אנרגיה אפלה היא האחראית והתשפטות היקום…ואלו שני דברים נפרדים…לפחות ככה הבנתי.

כתיבת תגובה

האימייל לא יוצג באתר. שדות החובה מסומנים *

אתר זה עושה שימוש באקיזמט למניעת הודעות זבל. לחצו כאן כדי ללמוד איך נתוני התגובה שלכם מעובדים.